Download as pdf or txt
Download as pdf or txt
You are on page 1of 802

. .

Elc..ctricit·1 & ·
Magnetism A Master Book of Key Concepts,
Brilliant Methods of Analysis & Short-cuts
. -~ ·.- .. . .., ,_ - ._- - :·:; :... .- ·_ ·_· -.·~:. <~:-_ :- '-<-~~(=_~~ -:)~s~;: .~~(~
Ele~trici ~ -,
Magnetism A Master Book of.Key Concepts,
.

Brilliant Methods of Analysis. & Techniques

·:\ /
:! or, -~
. 1DfsHj ~, PUBLICATIONS

/- ' ab mcm:.dJ door nahin ...


© Copyright Author.
All rights reserved. No part of this publication may be reproduced in any form without prior
permission of Author and Publisher. The author and the publisher do not take any legal
responsibility for any errors or misrepresentations that might have crept in. We have tried
and made our best efforts to provide accurate up-to-date information in this book.
;(<·

.
-· ••...,. ,....-:..
-~
.,_

..
__,
--\___

'
.... ___ .,._- ·-:-.~
.....
- . - ··-
____ . ,...,,
..~. -.;.....:---;:_::.::

-=-<PREFACE

D.C.GUPTA

The valu_able suggestions, admiration and criticism of skeptical readers are the main sources of
inspiration for any author. The feedback I have received is an indication that the book is being
seriously studied by students. I find myself fortunate enough in this regard as I have received a lot of
praise, support, advice from students, teachers and academic institutions which have motivated me to
bring third volume of the 4 volume series for students preparing for IIT-JEE and other competitive
exams.

The book has been written in accordance with the latest pattern of IIT-JEE. The present volume
Electricity and Magnetism contains ten chapters including a chapter for revision. In the revision
chapter formula and hints of all the chapters are given together. In e~ch chapter, a large number of
examples along with numerous method of analysis and short-cuts have been given in a systematic
manner to assistthe students to understand the concepts ofthe subject.

While writing the book I have kept in mind, the need of the students regarding the latest as well as the
changing trends ofthe competitive exams.

I am very thankful to Avinash Agarwal, DeepakAgarwal and entire team of Disha publication for their
kind co-operation. I am also grateful to my wife Vandana Gupta for her cooperation in each stage of the
work.

Every effort has been made lo check the mistakes and misprints, yet it is very difficult to claim
perfection. Any error, omission and suggestions for the. improvement of the book will be gratefully
acknowledged and incorporated in the next edition.

I hope that the book will fulfill the needs ofthe students, for which it has been designed.
'-

D.C.GUPTA
Avantika-11, Ramghat Road.
Aligarh (U.P.) 202001
dcgupta.gyan@gmail.com

' '-'·--
,- - . ·, > - ->
;{:.Ma.thematical Tools & Tips
'. d(xy) ~ d(xy)
- y, X
ax dy (i-viii)
PARTIAL DIFFERENTIATION jl)
INTEGRATION BY SUBSTITUTION (ii)
USES OF DIFFERENTIATION AND INTEGRATION (iv)
SOLID ANGLE (VI)
COMPLEX NUMBER AND PHASOR (viii)

.
Electrostatics
- -

(1-140)
1.1 BASICS OF ELECTROSTATICS 1
Charles Coulomb
1.2 EARTHING OR GROUNDING 4 Charles Coulomb
1.3 GOLD LEAF ELECTROSCOPE 5 (1736-1806) was an
1A , .. LiGtfTNING AND LIGHTNING CONDUCTOR. 6 army engineer who
1.5 COULOMB'S LAW ~10 ' turned to the physical
1.6 ELECTRIC FIELD sciences in 1791. He
1.7 ELECTRIC FIELD LINES (EFL) studied the attraction
1.8 ELECTRIC POTENTIAL ENERGY (EPE) alld repulsion forces
1.9 POTENTIAL DIFFERENCE produced by· magnets
1.10 EQUIPOTENTIAL SURFACE and. by objects with
1.11 ELECTRIC FLUX electrostatic charge. He
1.12 GAUSS'S LAW ~R GAUSS'S THEOREl\;'.I> invented a torsion
I 1.13 APPLYING GAUSS'S LAW : SPHERICAL SYMMETRY balance, based on
' ,•,., • •' I • \
1.14 APPLYING GAUSS'S LAW: CYLINDRl~·SYMME,:RY !,Wisting a Piece of still
1.15 LINE CHARGE OF FINITE LENGTH 1 , -
wire, that could measure
1.16 APPLYING GAuss·s LAW: ,PLANAR SvMMET~v\( ._ 1 tiny Forces accurately. He
1.17 CONDUCTOROFANYSHAPE . '...._J ,: .al~,.
used this to develop his
1.18 MECHANICAL FORCE oN·iHE cHA~GED,p~NDYCroR.
law, which showed that
1.19 ENERGY DENSITY . 1 ':;) -~~ L oJi •
magnetic Forces fade
1.20 ELECTRIC FIELD DU~ TO C~ARGED R.1N~, ~-~--)- .l . (tL rap.Idly, by.the square of
~~~:~
1.21
:~~~NTIA~ ~NE~GY OF s ; ~ i i l f ·cJ~R~~;- '
the distance between the
1.22
1.23 Ei.ECTRICPIPOLE -- magnetic objects.
1.24 A DIPOLE IN AN ELECTRIC FIELD
1.2S FORCE BETWEEN TWO SHORT DIPOLES
1.26 QUANTISATIQN OF CHARGE: MILLIKAN Oil DROP EXPERIMENT
\

> >> >> >


Capacitance
and Capacitor
(141-24~) lllanJarriin lfuan!ffin
2.1 CAPACITOR OR CONDENSER: AN INTRODUCTION 141· Benjamin Franklin
2.2 CAPACITANCE OR CAPACITY 142 ( 1706-J 790) was one
143 of the first people to
2.3 CHARGING A CAPACITOR
study electricity in
2.4 CALCULATING CAPACITANCE 144
detail. In 1752, he flew a
2.5 CAPACITANCE OF PARALLEL PLATE CAPACITOR 146
kite fitted with a metal
2.6 ENERGY STORED IN A CAPACITOR 149 key into a thundercloud.
2.7 FORCE BETWEEN THE PLATES OF A CAPACITOR 150 Sparks flew off the key,
2.8 CAPACITORS IN SERIES AND IN PARALLEL 155 showing that lightning
2.9 SPHERICAL CAPACITOR 166 was a form of electricity.
CYLINDRICAL CAPACITOR 169 Franklin s·aid that
2.10
electricity consisted of
2.11 DIELECTRICS 176
two states of a
2.12 INDUCED OR BOUND CHARGE 177
mysterious fluid, an idea
2.13· TOTAL ENERGY OF THE SYSTEM 184 which is no longer
2:14 VAN DE GRAFF !3ENERATOR 187 believed.
2.15 KIRCHHOFF'S LAWS 187
- -- - - --- -------·

Direct Current and


Electric Circuits
(243-338) <liioor::g$iID{ln<Olhm
Georg Simon Ohm
3.1 INTRODUCTION 243 ( 1789-1854) showed
that all conductors, even
3.2 OHM'S LAW 247 the best metals, resists
the flow of electricity to
3.3 ELECTRICITY FROM CHEMICALS: CELL 253 an extent. The unit of
electrical resistance is

called the ohm in his
3.4 ELECTROMOTIVE FORCE {EMF) 255
honour. Ohm's law says
that the current flow
3.5 CIRCUIT ANALYSIS 258
through a conductor, in
amps, is proportional to
3.6 ELECTRICAL INSTRUMENTS 300 the potential difference
across it, in volts:
volts = amps x ohms
~>»>~~'JI.. > >>~·?>>
Thermal and Chemical J
Effects of Current ~~J
fl
..";.·,,.\

(339-368) -l
Alle™ndro Volta
In 1800, Italian count
4.1 THERMAL EFFECT OF CURRENT: JOULE'S LAW 339 Allessandro Volta (1745-
1827) discovered th,u two
different metals, separated
4.Z ELECTRICAL APPLIANCES 342 by moist chemicals, could
produce a flow of elearic
cfwEe. This was the first
4.3 SEEBECK EFFECT 350 electric cell. Volta piled the
cells together on top of each
other co make the first true
4.4 PELTIER EFFECT 353 b.anery, called the Volca!c
pile. When he touched a wire
from the top of the pile to a
4.5 THOMSON EFFECT 354 wire at the bottom, he got
spaOO: of electricity.
This was the fim time that a
4.6 CHEMICAL EFFECT OF CURRENT 355 reli.tble supply of constantly
flowing electric.al current
Jud become available. It
4.7 FARADAY'S LAW OF ELECTROLYSIS 356 began a whole nm area of
science, and sparked off a
v.ast range of new
inventions.

~>>>>~:]>>:»>»>~
Magnetic Force on Moving
Charges and Conductor
(369-428)
Hans Christian Oersted
5.1 MAGNETISM : AN INTRODUCTION 369 In I 820, Danish sdentist
Hans Christian Oersted
5.2 THE MAGNETIC FIELD 369 (1777- I 85 I) noticed that a
wire wfth an eleoric current
5.3 FORCE ON A MOVING CHARGE 370 pJsslng through it worked
like a magnet, making a
5.4 MOTION OF CHARGED PARTICLE IN UNIFORM MAGNETIC FIELD 374 needle on a n~rby magnetic
com~s move. A compass
5.5 HALL EFFECT: 1879 384 needle is a tiny magnet and
m.agnru an puJI udl other
5.6 MASS SPECTROGRAPH 384 together or push apan.
Oersted re..11ized that the
5.7 CYCLOTRON 385 electric current was
I'
, 1r,-, "•1 , f r • producing magnetism, and
5,8 MAGNETIC FORCE ON CURRENT CARRYING CONDUCTOR 391 he was the first to discover

5.9
! , . :·',

TORQUE ON A CURRENT LOOP IN -~~Gt'<!ETIC FIELD


.,, 397
the elearomagnetic: cffea.
Almost at once, many other
scientists began to
5.10 POTENTIAL ENERGY OF MAGNETIC DIPOLE 398 n:perfmentwith this effea.
~~

Magnetic Effects
of Current.
(429-480)
Andre Marie Ampere
6.1 MAGNETIC FIELD OF MOVING CHARGE 429
Andr~ Marie~-sAmpere
(1775-1836] 'worked· in
6.2 THE BIOT SAVARTS LAW 431 many areas of science,
including physics, chemistry
6.3 SOLENOID, 438 and scientific philosophy.
'Following the work of
6A TOROID.OR ANCHOR RING 439 Oersted, · Ampere made
:many discoveries about
jelectrlcity and the
6.5 AMPERE'S LAW 446
. felectromagnetic effect. He
; noted that the strength of
;the magnetic field around a
wire was related 'to the
amount of current flowing
through_ the wire and the
distance from the wire
{Ampere's law). He devised
the idea of winding wire into
a cOu, the solenoid, to
increase its magnetic
strength. The unit of electric

! current. the amp (ampere],


_ _ is Ilamed in his honour. -

~~ ~;,_ >>> > >>>>>>


Permanent Magnet and
Magnetic Properties of
Substances
(481-522)
7.1 PERMANENT MAGNET: AN INTRODUCTION 481
Nikola Tesla
7.2 COULOMB'S LAW IN MAGNETISM · 482
Engineer Nikola Tesla ,
7.3 EARTH'S M~GNETISM 489 (1856-1943) supported;
the now-accepted use of j
r 7.4
7.5
MOVING COIL GALVAN~MmR

TANGENT LAW
495
496
alternating current for most :
praafcal ilpplicatlons, as :
7.6 497 .shown on later pages: In '
/" , ..JAN~ENT GALVANOMETER
I 888 he built his first
7.7 VIBRATION MAGNETOMETER 498 induction motor, which is
7.8 DEFLECTION MAGNETOMETER 499 the type of motor used in
i many domestic machines
7.9 GAUSS'S LAW IN MAGNETISM· I 500
and appliances. He also i
7.10 MAGNETISM AND ELECTRO~
I' 503 invented a type of'
'transformer, the Tesla coll, ;
7.11 MAGNETIF PROPERTIES OF THE MATERIALS 505
which produces enormous
I voltages and is used in radio ,

l • 'I
· technology.
~»>:»>2?>»»>»~ >> >>>>>
Electromagnetic~,;,: -~'::~···::
lndud1on
.
·" Michael Faraday ( I 791-

(523-630) Michael Faraday


1867) was trained as a book
seller and bookbinder, when
he developed many practical
8.1 ELECTROMAGNETIC INDUCTION: AN INTRODUCTION 523
8.2 MAGNETIC FLUX skills for his later scientific
523
8.3 FARADAY'S LAW OF EMI 524 experiments. He made many
8.4 LENZ'S LAW 525 advances in electricity and
8.5 MECHANISM OF EMI 532 magnetism, including the
8.6 ROTATING CONDUCTOR 529 principles ·of electro-
8.7 INDUCED ELECTRIC FIELD 542 magnetic induction, the
8.8 SELF INDUCTANCE 563
8.9 SELF INDUCTION transformer and the electric
564
8.10 RL- DC CIRCUIT motor. Faraday also devised
568
8.11 THE ~C-OSCILLATIONS 572 the Idea of picturing an
8,12 DAMPED OSCILLATIONS IN AN RLC CIRCUIT 573 invisible magnetic fields as
8.13 MUTUAL INDUCTION . 575 linesofforce .
8.14 COMBINATIONS OF INDUCTORS I 579
8.15 EDDYS CURRENT 591
8.16 AC GENERATOR OR DYNAMO 592
8.17 DC GENERATOR 593
8.18 DC MOTOR 593
8.19 TRANSFORMER 594

>>>> >
AC-and
EM-waves
American scientist Joseph

(631-680) Aenry ( 1797-1 878}


observed the effects of
9.1 ALTERNATING CURRENT (AC) Joseph MffrY electromagnetic induction
'I about one year before Michael
9.2
9.3
AVERAGE AND ROOT MEAN ,SQUARE {RMS) VALUES
THREE SIMPLE CIRCUITS
r \ 633
638
Faraday, but he did not publish
his experiments and results so
9.4· RC-CIRCUIT 641 quickly. Henry made many
9.5 LR-CIRCUIT
II advances i~ the electrical
641
9.6 LC-CIRCUIT ',,I 642
sciences. He devised and
constructed an early electric
9.7 SERIES RLC-CIRCUIT 642 motor, helped Samuel Morse
,i ~1
;JL::
9.8
9.9
9.10
RLC, RESONANCE CIRCUIT
POWER IN AC CIRCUITS
ELECTROMAGNETIC WAVES
I :f"'"'
~ .'t ,. ,.,~ ' · ;\ '
643
645
to develop the telegraph, and
discovered the laws on which
the transformer is based, The
~,,
,-
661
9.11
9.12
POYNTINGVECTOR
ELECTROMAGNETICSPECTRUM
I ¼:·,,_ f-~~t~;~: -~--
j .:,'fff-·"$... :1:i,·:~
665
scientific electrical unit of
Inductive resistance, the
henry, is named after him.
- -- s - ' • ~ - -·-·-
677

~}})}})}})}})}l
Revision Solutions
(Formulae & (Subjective
Important · Exercise)
(681-706) Points) (707-784)
\
0
0 CHAPTER
0
0---
0

Mathematical
I
11)
; !Ii ,,
I
I
·I
j,,

Jtlls & Ti111


J(.,r)
i)y • X

PARTIAL DIFFERENTIATION
In physics, we often come across quantities which depend on two or more variables. For example electric
potential V depends on x, y coordinates as : V = xy. For given pair of value of x and y, Vhas a definite value_ If
we differentiate quantity V w,r.t. x keeping y constant, then it is known as partial differentiation and represented

av av
by -a - Similarly differentiation of Vw.r.t.y keepingx is constant is represented by -a
X y
·

Thus
av a(xy)
--=y
ax ax
av a(xy)
--=x
and
ay ay
In general if/is a function ofn variablesx 1, x2 , •...• .xn, then partial differential coefficient of/with respect tox 1,

keeping all the variables except x 1 as constant can be written as aJ .


ax1
Ex. 1 Given~= a( x 2 + y2) + bz 2 , where a and bare constants. Find partial differei!tiation ofqnv.r.t.x,
yandz. _

,,/
S01.· ·,

= ax2x=2ax,
cl"' a 2 2 2 ·
-"' ,=-[a(x +y )+bz ]'
iJ.Y:,' cly
'.
"i= ax2y=2'()', . '

= b x2z=ibz . ' ,. ~':'

INTEGRATION BY SUBSTITUTION

t ,., -
Ex. 2 J
Evaluate Asin2? t dt, whereA and mare constants.
0

Sol. Substituting 201 = z. .


On differentiating, we have

d(2rot) dz
dt· di

, dz
or 2ro = -·.
di .
.:,
dz
dt = -2ro

. ., '.:.-·A I .
t dt- ·--:.: -Jsm z dz
Thus - - 2ro
0

- Aj.-cosz·j'0
2m

= ~ I- eds 2rotj01 - - .,,


2ro
' ,

A
= -(t-cos2mt) Ans.
2m

' J
\

T 2p
Ex.3 J
Evaluate sin 2 ? t dt, where ? = T
0

1-cos20
Sol. We know that sin 2 0
2 •

T
Jsin 2
001 dt
0

= _!_,,_sin 2001
2 200
r
o

I
-[T-0]
2

T
Ans.
2

Ex. 4 Evaluate J(xdy + ydx)


Sol. We know that d(xy) = xdy+ydx

J(xdy+ ydr) Id(xy)


xy+C Ans.

Ex. 5 Evaluate J(x dy + 2xydx)


2

Sol. We know that

or

Thus we can write


"" \ ,, Ans.

.
//
,_1/

Ex.6 J
· Evaluate sin 30 dO

3 2
Sol. Jsin B dB J sin B sinB dB

= (
cosB--
3
3
cos-+c
B- J--
USES OF DIFFERENTIATION AND INTEGRATION

Finding maxima and minima :


Suppose a quantity is defined by a function ofx as : y =j{x).

' For maximum or minimum value ofy, put dy


, dx
= 0. On doing this the values ofxcan be obtained; some of them
2
may corresponds to maxima and some of them corresponds to minima (ifx has many values). If d ; < 0, then
. dx

2
y is maximum fora certain value ofx and if d ; is> 0, then y will be minimum forotber value ofx.
dx

Ex. 7 Divide 10 into two such parts, so thattheir product is maximum.

Sol. Letx is one of the parts of 10, then second will be 10-x. Ify is their product: then
y x(IO-x)
10x-x2.

For maximum vali.J.e ofy, 0,

or 0

or I0-2x 0
or X 5
Thus 5 and 5 are the two required parts. Their product is 25.

y
Area bounded by the curve
Suppose a curve which is defined by a function of x as; y =j{x). The plot of
the curve is shown in figure. Take a small element ofcurve of width dx. The
corresponding average vall'e on y-axis is y. The area of the shaded strip -+~~t.:---;:--x
(assuming rectangle) x, Xe
dA = ydx Fig. 0.1.
The area bounded between x 1 and x2 is given by

Ex. 8 Figure shows the parabolic curve y = 2x2. Fiud the area bounded by the curve between 0
and 5.
Sol. The area bounded by the curve is given by y

x,
A f ydx
x,
5
f2x2dx Fig. 0.2
0

35

= .2~
~3
0

~(5
3
3 3
-0. )= 250
3 unit Ans.

Ex. 9 Findtheareaboundedbythecurve y=e-x, thex-axisandthey-axis.


Sol. The given function decreases with increase in value of x. For y to be zero,
e-X 0
which gives x = =
At x=O,y=e--0 = l
y
The area bounded by the curve is given by
'

=
f e-x dx Fig. 0.3
0

-1
0

. -e( - -OJ =l
-e Ans.
y,..l?'
' - .~
Finding the average value : ' '~ ,.J' ' -~ \ '

Suppose a quantity y is the function ofx. We have to find its average value between x values from x 1 to x2. The
average value ofy can be obtained as: , -i

f?'/•%•9\c· _,c.,r:·"'-_ i""'1 ·;, !/ ~,P '~,.", ,, I ' .(<:


«;/'a'ver.u,e,=,y =
, :~ l1 : .' '
• f, •' .,,,t ',,,- 1/>

Ex. 10 Find average value of sin8 in one complet~ cycle from Oto 21t radian.
Sol. The.average.value sin0 is given by

-2Jt .
. '.' ~ t -f sin0d0
[sin 8]0average
to 21t 0
(21t-O)

·=' 1- cos 01~"


,. 21t

= [-cos21t-(-coso)]
=- ·.:.1+1=0· . ·-. ·-- Ans.· ~-.
: •t I , , I' J ' l, ' l!i h -:\••

SOLID ANGLE r· 1r'


''' ' ' ~·• I, .
,, .f, -;. .

Solid angle is the analogue in three dimensions ofthe usual angle in two dimensions. The solid angle subtended
by any surface dA at a poin_t O at a distance r away is given by. , ··' ' · '

dro
,
~ -. - · '. -,-
(1) ~---······;:·· •••••. ,-,,..~-
.

.• '

The total solid angle is given by Fig.0.4

·l_!'
.• •1·•
The SI unit of solid angle issteradian. The solid angle will be 41t. It does not depend on the shape of the surface
enclosed. In the following cases .ihe total solid angle subtended is 41t.

• I

Fig. 0.5 Fig. ~.6


Ex. 11 Find solid angle subtended at the centre Oby the shaded surface shown in figure.

Fig. 0.7
Sol. Suppose R is the radius of the spherical surface. The surface area of h height of the spherical cap
is given by
A 21tRh,
where h R( 1--<:os0)
A 27tR2(1--<:os0)
The solid angle subtended is given by

Acos0°
co =
R2
Substituting value of A, we have

co =

21t(l-cos8) Ans.
Sagitta theorem
In a circle ofradius R and a cord oflength d

d d
(2R-h)xh = -x-
2 2

or I· 2Rh-h
2
d:7 Fig. 0.8

Ex. 12 The diameter of aperture of a piano-convex lens is d and its maximum thickness is I. Calculate
radius of curvature of the lens.
Sol. If R is the radius of curvature of the lens, then

d d
(2R-t)t -x-
2 2
d2
2Rt-i2 - Fig. 0.9
4
tn a case, t < < R, i2 can be neglected and

2Rt =

and R = Ans.
,, :1·,.
Complex number"and•phasor ~ . ~ .., . ' ! ' •.

A number (quantity), which is made of two parts; ry,al and imaginary parts is called complex number. If j = ../=i.
is an number (operator), then a phasor A in cartesian form can be written as: ·-,
A = a+jb,
where a is the x-component and b is .they-component of phasor A.
;; ' , .' ,·
The magnitude of A

If 8 is the angle made by phasor A with the 'positiv~ x0 axis, then A


\b
b
tan8 - ,/L'..l.2-~a~'-j'---- X
a

Also/=-!; l=-j; f=+I. Fi[f- 0.10


- In polar form the phasor A can be written as;
A = Acos8+jAsin8.

,. ·,,' ! .. • j. . ......

' ·- ... ...

' ,·, ':, l. ·• • ~ ,


-,

\ \\ _\ ....·, ~
. '\ \
0 CHAPTER
.· ~-·".
.,#
,,.-
\

~' :
\

-~
\~ ~ ,.
~·!!JI
~
0
ii'i'
I ,
-~
0 _ r;,l~ - --
" .···
__ J
I
I

0
0
~--·
,--:, ~~~"'
·\~
r .
.,<ii '
'ii:~
.,
~
(j~
,:, \~

1! l_-,._
:
~~--
'ij.ii I
. I ,!

Eliictrostalics
- d"' - .

1.1 BASICS OF ELECTROSTATICS

Electric charge
The word electric charge was derived from Greek word elektron, meaning amber. It was known to the ancient
Greeks as long ago as 600 RC that amber rubbed with wool, acquires property of attracting light objects. It has
now concluded that there are two kinds of electric charges; positive charge and negative charge_ The experiments
lead to the fundamental results that (l) like charges repel, (2) unlike charges attract.
Where does charge come from?
The process of acquiring charge consists of transferring something from one body to another, so that one body
has an excess and the other a deficiency of that something_ It was not until the end of nineteenth century that
this "something" was found to consist of very small, negatively charged particles, known today as electron
discovered by Sir J.J_ Thomson.
To give a body an excess negative charge, we may add a number of electrons. And to give excess of positive
charge, we may remove the electrons from the body.

The "Charge" of a body refers to its excess charge only. The excess charge is always a very small
fraction of the total positive or negative charge in the body.

Conductors and insulators


Conductors permit the passage of charge through them, while insulators do not. Metals in general are good
conductors, while most of non metals are insulators. Within a metallic conductor, a few outer electrons become

...
./ ,
;_~-

detached from each atom and can move freely throughout the metal in the same way that the molecule of a gas
can move through the spaces between grains of sand in sand-filled container. In fact, these-free electrons are
often referred' to an electron gas. The positive nucleus and the remainder of the electrons remain fixed ,in
position. On the other hand, there are no or at most very few free electrons within an insulator,.

1Free electron

Fig. I.I
Charge given to conducting body spreads over its surface, because of repulsion between the like charges. It
happens because of high mobility of charge carriers in metals. When some charge is put any where on or in the
conducting body, it ultimately wili'speed over its surface.

0-0 Conductor on charging

Fig. 1.2
When charge is given to insulajor, it remains at its position. Because there are no or very few charge carriers in
an insulator. ,7

0-0 Insulator on chnrging

Fig. 1.3
Impact on body after charging
I. When body is charged positively, it means removal of electrons from the body, resulting decrease of
mass of the body.
2. When body is charged negatively, it means addition of electrons on the body, resulting increase of mass
of the body.
3. Body either positively charged or negatively charged, it will increase in size because of repulsion
between the like charges.

0 RO
q= .
~ R ' M'>M,R'>R.-
11: !J

Fig.1.4
EtECTROSTATICS M
Test of charging of body
Let us consider a charged body brings near an uncharged body. It is clear from the figure, that there will be
attraction between them.

:::;, Attraction
g :::;, Aattraction
( :t :t)
Fig. 1.5
The repulsion can be possible when both the bodies are charged. Therefore we can conclude that repulsion is
the sure test of charging.
Methods of charging
I. By rubbing: Static electricity builds when two different non-metal materials rub tog!:!llE:.Rubbing or
friction makes electrons move. This gives one material a positive charge and the other a negative charge.
The charges stay, or remain static, on the surfaces of the materials, until they have a pathway along
which they can flow suddenly, or discharge.
The energy ofrubbing or friction gives extra energy to electrons. This allows some of them to break free
from their nuclei-and wander off on their own. It is known as "separating charge". Some electrons pass
or transfer from one material to the other.
+

@J @J @J e @J
@@e ~'@ => ®@ @)'®
e
@--9 @
ftj Positive
i t
Negative
charge charge
Fig. 1.6

• -"=' n ....,,-~~· ~ ·.·,~~n

.,..

~Glass
~ -,._.,..
(a)
Fig. (a) Two charged rods oftlIC same signs repel each other.
(b) Two charged rods of opposite signs attract each other.

Fig. 1.7
When glass_ rod is rubbed with silk, it aquires positive charge and by convention silk will aquires some
amount of negative charge. Similarly when a plastic rod rubbed with.fur.it acquires negative charge, and
the fur itself is found to acquire a positive charge. Moreover, if the rod and fur are brought into contact
with one another, the charge is found to disappear from both. Similar observations are true for the gla
rod and silk cloth.

..
2 By conduction (by touch without rubbing):
Because of having excess free electrons in metals they can be charged by conduction. When two
conductors, one charged and other uncharged bring in contact, the same type of charge will appear on
both the conductors. Charge on any conductor is proportional to its size.

0
i©J
0
Charge
00
,.

0
Uncharged
=>DI
0
Positive
charge
u Positive
charge
Fig. 1.8
3. By induction (without touch): !fa charged body is brought near an uncharged body, then neutral body
becomes oppositely charged. By induction method, we can charge any type of material body. From the
figure one can understand the steps follow in the charging of the body by induction.

••
f
+' • +
.
+
+ +

_( .l ..L
+'

(a)
(d) (e)
Fig. 1.9. Chargi11g by i11d11ction
The magnitude of induced charge depends on the material of the body being charged. If q is the
inducing charge and q' is the induced charge, then

lq'l=q(,-¾)
For metals k=oo,q'==q ,and for non-metals k<oo, :. q'<q

1.2 EARTHING OR GROUNDING


We know that sea is the very large reservoir of water; from which iffew buckets of water is taken out, the level
of water in sea will remain almost same, practically same. On the same way earth can be regarded as a large
reservoir of oppositely charged particles; electrons and protons, on a whole, is electrically neutral. By any
,, means, ifwe take out or add some electrons to the earth, its charge will remain almost zero, practically zero. Thus
the earth is regarded as a large capacitor with zero potential under all practical situations. Any body connected
to earth becomes its part and hence of zero potential.
(i) When a positively charged body is connected to the earth, the electrons start flowing from the earth till.
the positive charge of the body becomes zero, and hence potential of the body V becomes zero.
Eleclronic
....c._ current
!
Insulating
stand P=,O

Earthing
Fig. I.JO
L---------·--~~-----~----'E_I$_T~i'i;~:j-)l~o_.sr._·:4_n_cs__ g
(ii) When a negatively charged body is connected to earth, the electrons from the body start flowing to the
earth, till the body becomes.neutral and hence its potential becomes zero.
_ . Electronic 1

=-:..-_ ....,t;:::; current


lnsulatin t
stand
t
Earthing
Fig. 1.11

Pi@@ The'charge of the connect~d body to earth·be~;~ifthere ~~o other inducing c-harge
nearby the body.

1.3 GoLD LEAF ELECTROSCOPE

It is used to detect the charge on the body. It consists of a gold leaf attached to the brass strip which is
suspended with the help of a brass rod in a glass jar (see figure).
~-,~-Brass cap

1---.-Brnss rod

Brass --<---+1 \•---+--Gold leaf


strip

Wooden base
Fig. 1.12
Uses of gold leaf electroscope
I. To detect the charge : Bring the body near the brass cap of the electroscope. If the gold leaf diverges
II
outward, the body is electrically charged. However ifno divergence takes place, then body has no charge.
2 Identification of charge:
(a) By conduction : First of all, charge the electroscope positively. Then touch the body under
consideration to the brass cap of the electroscope, If the divergence of the gold leaf increases then
the body is positively charged. However if divergence of gold leaf decreases then body will be
negatively charged.

J
...
.. + .. +

(a) (b) Divergence (c) Divergence


mereases. decreases.
Fig. 1.13
When positively charged body touches the brass ·cap of the electroscope, the positive charge on the
leaf will increase and hence divergence ofit increases. But when negatively charged body touches
the brass cap of the electroscope, the positive charge on the leaf will decrease and hence divergence
of it decreases.
(b) By induction : First charge the electroscope positively. Then bring the b"ody under consideration
near the brass cap of the electroscope.
lfthe divergence ofthotgold leafincreases, then the object is positively charged. However if divergence
of the gold leaf decreases then the·object is negatively charged.

······rr--
l. t + t ± t ± .
'
,------,r++
.------.

-
~ - f""
., .. ~ ., -
- •
- •
• "
+ •

(a) (b) Divergence (c) Divergence


increase. decreases.
Fig. 1.14
When positively charged body brings near the brass cap of the electroscope, the neg~tive charge
induces on the brass'cap and positive charge on the gold leaf and hence divergence of the leaf will
increase. However when negatively charged body brings near the brass cap, the positive charge
induces on the brass cap and negative charge on gold leaf and hence divernence of e leaf will
decrease. .
+
++
Cloud
+++
-f
+++
1.4 LIGHTNING AND LIGHTNING CONDUCTOR
The occurance oflightning depends on the atmospheric conditions. \ \ l I ) .
It needs a weather in which moisture-laden warm air near the earth --.-\·
Electnc c,
~~ ~ ~0·
!
rises rapidly. As the moist air rises, it collides with the other drops of l_ ) ) ,'

discharge \ \ ( \ ,( /
moisture and air molecules and so lose or gain electric charges by
+ +
friction. Usually, the moisture drops become positive and the air +t
molecules negative. When moist air becomes cold enough, it

condenses to form a cloud. The lower part of the cloud usually Li
ghtening
becomes negative and upper most part of the cloud becomes conductor
positive. The sudden movements in the cloud cause this charge t9
discharge in a flash of lightening to another cloud or the earth.
When such a charged cloud moves over a tall building, the positive
charge will induce over it. Since this induce charge is of very high
order, so the building experiences a bursting force and hence cracks
will develope in it. The building can be saved from destruction by
using lightning conductor. Building
.. . --~--- -
A lightning conductor is made of copper and has sharp pointed ,:,-
edges. It is connected to the earth by the metal conductor running
down the building and terminating in a buried metal plate. In the
presence of lightning conductor, most of the charge will induce ~n
it, which soonly move to the earth without harming the building.
Fig. 1.15
Ex. 1 . Fig.'"J.16 slio,vs an uiichat'ged, instiiaied comi"u'ctor '' . ,,~,, 'A B C
•• 1- , , ,, ,_ ,. · ' It ·'•!' -: ,, , ,. , ,; .... I , 1 t, J, • ~ :..,Ai'~[l L -----
... AB,.whichis'incontactwitligoldleafelectroscopeC.A . , , 1 ~+' . /

positively charged rod is brought near the end A. ~ '·


Ansiv.er the following questions.
-- :., ._)

(i) What charge is developed at A and why? ,,. D p


(ii) What charge is present at B, C and D, and why? Fig.1.16
(iii) How is gold leaf affected?
(iv) If positively charged rod is taken away, give your observations.
Sol. (i) The electrons ofth~ atoms of the rod are aUracted and _so end A of the rod becomes negative
and end B thus have deficiency of electrons and so becomes positive if.the cap C was not
CO?nected. As cap' C is conn_ec_ted to the end B of the rod, so defic_iyncy of _elecrrons is
I created at C which will carry on to n-:so
endD becomes positively charged.
(ii) . The end B and C will not have any charge,' wher_e ~s end D has positive charge.
(iii) The gold leaf will diverge, because positive charge on gold leafrepels.positive charge on the
plateP.
(iv) When positively ·charged rod is taken away, the leaves· collapse because the electron from
end A flow .back to the end D and hence no deficiency of electrons remain at the end.
\
Ex.2 A glass rod rubbed with silk is brought close to two
uncharged metallic spheres in contact with each other, .
~ -
inducing charges on them as shown jig'. 1.17. in if:
~
·,I,
Describe what happens when
· (i) the spheres are slightly separated, ~nd "hvtw. · ' ' · ',
(1·1·) th.e gIass ro d.1s su bsequen ti y removed, amlfinauy
ti,,__ 'Sf.,, 4Jt>!.: 'i:, l•?kfj,
S-ej,,.-r-d.1:'e.i',
(iii) _thesphereareseparatedfarapart. [tiu.... ~("lh:) m /J ·i:\lyeq_ofX"- . Fig. 1.17
Sol. (i)
·Little change in the distribution of charges. O , ·., ,·=~1J '
(ii) The charges on the sphere are redistributed; positive and negative charges will face each
other. ,r
. ,,.,
(iii) Charge on each spherewill be uniformly distributed.

l'.n IC1M -w d ,½ ~ . ( i )
SJ,1-unl/.\ C/J2,,---. ;Jl,JArlf!
y/,aJz..cJ;-i2d '

,..
u . Ei.£CTRICl1Y & MA."aNETIS_~M
0

_ _ _.· ~-~~-----· =·

Ex. 3 A metal sphere, a cylindrical conductor and a conical conductor are mounted on insulated stands -
such that they are in electrical contact.A highly charged ebonite rod is brought near sphere X.
Answer the following questions : ·

Ebonitc rod

Fig. 1.19 , cl/ <a


(i) What will be the charges on X, Yand ~ e_.cin..iltll/Jl a..>td +Asi.n --¼.,. Wl 'r.,___ ~ ~
'*(ii) If conductor Z is earthed for a moment and the ebonite rod is removed, state the charges on
X,Y,Z.
l<(ili) In the presence of ebonite rod, conductors Y and Z are removed. What is the charge on each
of them? [Ft'.Jz.,\t l- 1 O.b'IW-thtd-1 Y 11<l )llmuV-'f{J .
(IV) What will you observe if a lighted candle is placed near the pointed end ofZ?
(v) The ebonite rod is made to touch X. What charges are on X, Y and Z.
Sol. (i) The negatively charged ebonite rod repels the electrons and so electrons from X move to the
Z. Therefore X becomes positively charged and Z becomes negatively charged, while Y will
have no charge. ·
(ii) When conductor Z is earthed, its negative charge flows to the e_arth. After removal of the
ebonite rod, all the three become positively charged (the charge of X now spread to all the
three).
When Z is removed in the presence of ebonite rod, it will have negative charge. As soon as Z
is removed more~c~Xwill flow into Y. Thus now Ybecomes negatively charged.
So when Y is now removed, it will have negative charge.
Because of the excessive charge density at the pointed end of conductor Z, it ionise the
surrounding air and so charges leak from it and set up an electric wind. Thus the candle.flame
will bend away from Z.
(v) When ebonite rod is touched with X, the negative charge will conduct in all the three.

Ex. 4 The diagram shows a gold coated pith-ball suspended between


two plates and closer to the plate A.
(i) Plates A and Bare connected to the positive and negative
poles of a high tension battery. What happens to the
pith-ball ? Give reason. +
(ii) What would happen if Bis earthed ? Give reasons. .1 B

Sol. (i) The opposite charges are induced on both sides of the Fig. 1.20
ball. On being close to plate A, the ball experiences a net
force towards this plate, and so string becomes slent
from the vertical.
·\

(ii) When plate B is earthed, it becomes neutral. The opposite charges are.induc_ed on the both
sides <if the ball. Now.plate A exerts net attractive force on the ball. The ball will touch the
. plate A andJ,eyomeucfs[tively ch~rged dlle \g_conduction. N!JW plat~ ,1 _r_epels _tile ball. The
ball moves to mean position, but due to inertia, it crosses the mean position and then touches
the plate B. There it becomes neutral. The ball swing back and repeats.its motion till entire
charge-of plate A_ flows into the earth.

Properties of charge
l. Charge always be associated with mass.
2. It can be transferred from one body to another.
3. Charge is conserved. It can neither be created nor be destroyed.
4. Charge is quantised. That is, any body can have charge in integral multiple ofsmallest·charge called
electronic charge
or q = ±ne ;where n = I, 2, 3, ....... ,e= 1.6 x t0- 19 C.
5. Charge is relativistically invariant. That is, qrest =·qm~tion· -
6. Moving charge produces magnetic field in addition to electric field.
7. Acce!erated charge radiate.s energy.

Types of charge densities .


l. Point charge : When linear size of charged body is much smaller than the distance under consideration,
the size may be ignored and the charge body is called point charge.
2 Linear ~barge density: Charge.per unit length of the obje_ct is called linear charge density. It is given by:

A,
+ + + + +
SI unitof1'. is C/m. Fig. 1.21
3. Surface charge density: The charge per unit surface of the body is called surface charge density. It is
defined by : cr .

O' - dq
- dA
+
"_= ! (simply)
.+
Slu~itofcrisC/m2• Fig. 1.22
4. Volume charge density: The charge per unit volume of the body is called volume charge density. It is
defined by : · +++
"'+ + + +
dq + + + ++
p
dV -I'++++
+ + +.
!L .Fig. 1.23
V (simply)

SI unitofp is C/m3 •
ffl~i!EC1RICl1Y;8if~G~ETISM :: if -·

Charge versus mass


, Charge Mass
I. Electric charge can be positive or negative. . 1. Mass of body is always positive.
2. Charge is conserved. 2. Mass is not conserved as it can be
converted into energy and vice versa by E
=mc1-.
3. Charge is quantized. 3. Quantization of mass is yet to be
established.
4. Charge is relativistically invariant, i.e., qrest = qmotion· 4. Mass is relativistically variant. Mass in
.
motion m= V mo; 2 ,where m .
0 1s rest
I-v2 !c
mass object.
S~between charges may be attractive or repulsive. 5. Force between masses always be attractive.

/1.5 COULOMB'S LAW


The force between two point charges at rest is directly proportional to the product of the charges and inversely
proportional to the square of the distance between them. Thus, if two point charges q 1, q2 are separated by a
distance r in free space, the magnitude of the force (F) between them is given by

where kis·a constant. For historical reason and so simplifies many other fonnulas in eleCtricity and magnetism,
I
the constant k is usually written - - - . Thus Coulomb's law can be written as
. 4 ltEo

:--F- ------·T--[qj"ilq2-I:
= ;
I ... (I)
i 4itE0 r2 !
:._ - - - - ----· ~ - - - -•.• ,,.J

+q, . +q,
~·······························~
r----<
+q . -q,
~ .................. ~
I• F
Fig. J.24
If the particles repel each other, the force on each particle is directed away from the other particle. If the particles
attract each other, the force on each particle is directed toward the other particle.
Coulomb's law has.proved its validity by experimentalteSts; no exceptions to it have ever been found. It holds
even within the atom, correctly describing the force between the positively charged nucleus and each of the
negatively charged electrons. This law also correctly accounts for the forces that bind atoms together to forrn
molecules, and for _the forces that bind atoms and molecules together to forrn solids and liquids.

!'@ § I. In using Coulomb's law in scalar forrn, one should not put charges with signs; only place their
numerical values.
2. · Signs of charges are put in the vector forrn of Coulomb's law.
. :L;/L '. EiiCTROSTATlCS ~
~--~~~=;?_:_· ----------~<-------------------·~
More about Coulomb's law
I. The force between any two charges is independent of the presence of other charges.
2 Coulomb's law holds for point charges (whose dimensions) are small compared to their separation.
3. Coulomb's law holds for charges at rest or nearly at rest.
4. Coulomb's law holds over wide range from atomic distances (l 0- 15 m) to many kilometers.
5. In the CGS system the constant k = I, without unit. The unit of charge is stat coulomb or the esu ( electro
static unit). The conversion factor IC= 3 x I 09 esu. Thus

qlq2 . .
F -2-. (m CGS system ofumts)
r

6. In SI system of unit: In SI system k =_I_, where e 0 is called permittivity of free space. The
41t Eo

introduction of ( - ' - ) will simplify some formulas to be encountered later. Thus coulomb's law is
41t Eo
usually written as

with 9xI0 9 N.m 2 /C 2 ,

and 8.854xl0- 12 ~ -
N-m

Ji@§, In electrostatics problems, charges as large as one coulomb are unusual; the microcoulomb is
often used.
7. When a dielectric medium is completely filled in between the charges, the force between the same
charges decreases by a factor k (dielectric constant). Thus

Frreespace
,_____ r-----.
k
As Fig. 1.25

or I Fmed

Above· equation can also be written as


_ _ qlq2
F
41t E r 2
where, E= k Eo=ErEo, is called permittivity of the medium, and Er is the relative permittivity of the
medium.

...
-.,;:\;*,: " 0

~---'---·'"~.....-~-•:I~:!.!
Dielectric cons(,ant k:
Medium k
Vacuum (free space)
Air (one atmosphere) 1.00546 = I
Water 80
Metals
8. If dielectric is inserted in some part of the space between the charges, then force between them can be
calculated as follows:
When dielectric is filled completely between the charges, the force between them for a separation r is
given by

I
This force is decreased by a factor k in comparison to the force between them in free space. This effect

of decrease of force can be achieved, if we remove the dielectric and increases the separation between
the charges. Let required separation is r. Thus we can write

_1_·_q1qz
47t Eo r'2

which gives
Now move to the situation in which dielectric k is occupied a length I. In this case we can remove the
.dielectric and effectively increase the distant by .fk r . Thus

'Ii

~ 0.
q, Free space

·I
• -
l---1 (r-;-1}->I ----[.fkt+(r-t)]
Fig. 1.26

The force betwee·n the charges now ' F


I qlq2 1

!_ ~ ~n--':o _[(r-t ! + jE1J2::


9. I coulomb of charge
I.6x I0- 1•
6:25 x 10 18 electrons

Vector form of Coulomb's law


(a) In terms of unit vector: If f is the unitvectorofrfrom I towards 2, and charges are placed ~t separation
r, then force on charge q2 due to charge q 1 '

-F21= ( -I-) -
q1q2
- r- .,__.,_.....................q,,
l/1
~
41C Eo r2 ? r.,, ... (2)

Fig: J.27
(b) In terms ofp~sition vector: Consider a system of two charges q 1 and q2 placed at position i'1 and
f with ~espect to the origin or'the co~rdi~ate system,.'. The f~rce on one charge by the other can be
2
obtained as
I
. ,,
t.' Fi+'i1 r2
or 121 Y2 -½

F21 =
(-·_I_) 41t Eo
q1q2 ;21
r2/
Fig. 1.28
X

I q1qz (r2 -i'1) . · ...(3)


=
I
A~-~o.. : 1'2 ~r, 13
If more than two charges are present, then the force exerted on any one say 9i, by all others 92 ,..93 : ....
'etc., can ·be obtained by adding vectorially all the forces. i.e., ·
Fl F12 + F13 + ..... + ...

-- I . [qiqz , +-2-'i3
-2-'i2 q1q3 - + ...]
41t Eo r12 'i2 .
Let us consider situations where the charge is spread over a region instead of being concentrated at
particular point, then

.. 'F·~ I =, _._l_f· q1dq r ...(4)


47t E o ... r.2
. .. .

Two shell theorems


Theorem I : A shell of uniform charge attracts or repels a charged particle that is outside the shell as if all·the
shell's charge were concentrated at its centre. Thus

q' q'
" ·=:; 0
., ;' '"++.GJ·
.:"
. cl-+
' ./ i +
j'·
:.-- r r _,
Fig. 1.28
Theore,;. 2: A s_hell of uniform charge exerts no electrostatic force on a charged particle that is located inside it.

·.,..
·o+
+
' ' oq,fl
+
F=O
. + +
+
Fig. 1.29

.
f/if§f ,LWhen two identical spheres with charges q 1 and q2 touch togethe_r and separated, the charge
qi +qz · (q1 -qz)
on each sphere will be--- . If q2 is negative, then each one has a charge .
2 2
2. If one sphere has charge q and other identical sphere has no charge; then each sphere will have a
charge q/2,
3. If sphere I with a charge q touches to the uncharged sphere 2, of half the radius of I, then charge on
them are: q 1 + q2 = q ...(i)
q I = 2q2 . .. .(ii)

2
On solving both the equations, we get q1 = q , q2 =!!.. .
3 3

Ex. 5 A charge Q is to be divided into two such parts, so that force between them when placed at some
separation is maximum. Find their values.
Sol. Let one of the part is q, the other will be (Q-q). The force between them for separation r is

F
(-1)41t Eo
q(Q-q)
,2 41t!EO (Qq-q2)

dF
For,Fto be maximum, 0
dq

d(Qq-q2)
or 0
dq
or Q-2q 0
Q
or q =
2

Q Q
Therefore the charges are
2 and
2. Ans.

rt Ex. 6 Consider a fixed charge Q and another charge q is placed at a distancex0 from Q on a smooth plane
surface. Find the velocity of charge q as a function ofx.
Sol. Let at any instant the separation between them is x. The force between them is

_ _ Qq,
q
F = 41t Eo x2

_ _ _ Qq
x----i1
and acceleration a = Fig. 1.30
41t E9 m x2

vdv ··
or ="' I Qqx-2
dx 41tEo m
~ - - - - - - - - - - - ·---------~-----E_LE_c__r.o.R_o_sr._'4TICS . ffl
V

or f vdv
0

or
v2 1 11 IX
Qq-
2 41t Eo m -x x0

· or V = [ Qq
2n e0 m x0
{l l}]½ x
Ans.

Ex. 7 Two equally charged identical metal spheres A and B repel each other with a force 2.0 x 10-5N.
Another identical uncharged sphere C is touched to A and then placed at the mid point between A
and B. What is the net electric force on C?
Sol. Suppose charge of each sphere is q\ the force between spheres A and B for separation r,

1 q2
F 2.0 x I o,-5 = ? ... (i)
41t Eo ,--

q <J q/2 q/2 q/2


• • •
A

C B

A ,. B ,.__ r/2 _,___ r/2 --,
(a) (b)

Fig. 1.31
When uncharged sphere C is touched to the sphere A, the charge on each one becomes q/2. Thus
force between A and C

(q/2)(q/2)
F,c F,
41teo (r/2)2
and force between B and C,

F,c

Thus the net force on the sphere C


2F-F=F
2.0 x IO·' N from CtoA. A11s.

Ex. 8 The force between two point charges placed at a separation ris F. Find the force between the same
charges; if half the space between them is filled with a dielectric of dielectric constant 4.
l!l-~~;:ELECTRICI1r ~~~NETIS~:~---
Sol. Given force between two point charges

F =
__ i_
4it Eo r2
When half the space is filled with a dielectric k, the force between them becomes

p
4it Eo [ r r ,...]2
-+-vk
2 2

4it "o [r-+-v4


r r.]2
2 2

4
-F, Ans.
9

Ex. 9 Let us consider two, small balls of masses m 1 and m 2 and having charges 9 1 and 92 respectively
are suspended with the help of two strings with the same point. Discuss aboutthe equilibrium of
the balls.
Sol. Consider the equilibrium offirst ball, we have
7i sin 8 1 Fe .... (i)
and 7i cos8 1 m1g ...(ii)
r,
Fe F F<
From (i) and (ii), tan8 1 ... (iii)
m,g '
m1g _m,,g
Similarly tan8 2 ~ ... (iv) Fig. 1.32
mzg
(i) lfm 1=m2 ,then 8 1 =8 2 .
(ii) lfm1<m2, 0, >82,
•.
(iii) lfm, >m2 then 0, < 82.

Ex. 10 Two small equally charged spheres, each of mass m, are suspended from the same point by silk

threads oflength £. The distance between the spheres x << e. Find the rate d9 with which the
dt

c~argeleaks ~ff each s~here ifiheir approach velocity as v = ':-::, where a is a constant.
. -.ix
r/-,-,,~,-...-..•.--.- - - - - ~ - - - - - - . · ..·-··---E-LE--'c-}tR·-;,;_O_ST._1l'\_Tl_cs_:_;_-.-;--MEi4-····-_·.
[ ________,:- --------~.:..;.,.::...-~---~:.....;,. ________
. ~;li.,••"i" ,. ~

Sol. Considering equilibrium of either ball, we have


Fe
tan0 =
mg
2
For small x, tan0 = sine= x/
e

X
mg
Fig. 1.33
2C

or x3 = ...(i)
2rrE0 mg
Differentiating above equation w.r.t. time, we.have
dx3 C dq2
dt 2rrE0 mg dt

3x2 dx x2q dq
or 2rr Eo mg dt
dt

dq 3x 2 v x 2rr Eo mg
2
3x ( Fx )x2rrE 0 mg
or
dt q q

( 2rr Eo mga) (
33/2)
-¾- ... (ii)

Now from equations (i) and (ii), we have


dq
dt
3Rg
-a 2rrE 0 -
2 £ .
A11s.

Ex. 11 A thin wire ring of radius r has an electric charge q. What will be the increment of the force
strengthening the wire if a point charge q 0 is placed at the ring's centre?
F
Sol. T cos 0/2 •····"(j··,::· .,:3...0..... T cos 0/2
·! q 2,/ ····-2
·-
+---+ .•

+ + T
+ =>
+ +
+....___ __.. +
+
Fig. 1.34
Choose a small element of ring ofcharge dq. The force stretching the each element of wire is
(dq)go
F - -----
e - 41t Eo r2
Since entire loop is in equilibrium, so its small element must be in equilibrium. Let us consider small
element of the loop as shown in fig. 1.34, for the equilibrium, we have · ·

2Tsin~ F =-l_(dq)qo
2 e 41t Eo r 2
:· 0 0
Forsman 0, sm- =
, 2;, 2
·. 0 _ _ (dq)%
2TX-
2 41t Eo r2

or T0

or T

Ans.

Ex. 12 Two identical charged spheres are suspended by strings of equal length. The strings make an
angle 30° with each other. When suspended in a liquid of density 800 kg]m3, the angle remains
the same. What is the dielectric constant of the liquid? The. density of material of sphere is
1600 kg}m3• ·
Sol. Considedng the equilibrium of either sphere we have,
· Tsin0 FeandTcos0=mg

tan0 = F, ... (i)


mg
When the system is placed in liquid
(Felk) Felk
tan 0 =
mg-Vptg
,\ ( mg- mg x800g)
·. 1600

,, ' Felk
... (ii)
-,' (mg 12)

(F)k)

mg mg
"Fig. 1.35
From equations (i) & (ii), w~ get
Fe ~F--'e~l_k~ ==> k =2
mg_ (mg/2)
- .--·-· . -···--------- ~

-----··-·- ·--·------~E_CE_C[H_.o_sr._··_l\_TI_cs_ • , ~
Test charge%: It is proposed charge which is used to define''£ and V etc. It is too small that its effect at the
point under consideration is zero.

- !1'
1.6 ELECTRIC FIELD
The charge and its influence can only be seen when we bring.another charge (test charge) in the vicinity of the
first charge. It means there is something but not visible arouncl every charge, in which other charges experience
force. This something is known as electric field. Electric field is a vector field. The electric field strength, is
usually called electric field at any point is the force experie~ces by unit test charge at that point. If F is the
force acts on test charge q0, then the electric field

F q r qo
E •··························-~
p E
%
Fig. 1.36
The direction of field £ is that of the force F acts on the positive test charge.The SI unit of electric field is
newton per coulomb (N/C). Also I N/C = I Vim
-q r
The field due to negative charge is towards it. •············ ..... ··-4------4 p
E
Fig. 1.37
Electric field due to a point charge : The 'force ft between charge 9 and test charge 9 0 is

F = _I_ qqo , Thus electric field at a distance r from 9 ·.


4JtfSo ,-2

or E = ___l_Jil.'
2
-··-- .. 4ite0 r _
Electric field due to many charges can be obtained by the principle of superposition. Thus if £1, £2 , ..... ,.. ,En
are the electric fields produced by the charges q 1, q2, ........... , q 0 at any point, then resultant field at that point

Concept plus
The force between charged particles was originally thought of as direct and instantaneous interaction between
the charges. That is;
Charge q1 _ _ . Charge q2

I q1q2
or F = 41teo?
Today, we think of the electric field in the interaction between charged objects. Thus have two tasks : (I)
Calculating the electric field produced by a given charge, and (2) calculating the force that a given field exerts
on a charge placed in it.
Charge 9 1 __.fieldE__.Charge9 2

That is E1 92
'
and


(tit-~··~,_;-_·.·_,£=··re=r_t_T._R_1c_·1_JY_._&._·MJ\~·~·•.·~G_N_m_s_M_ _~ - - - - - ~ - _____·.lliJ
1. 7 ELECTRIC FIELD LINES (EFL)
A very nice way to visualise electric field, is the lines of force, now usually called electric field lines. The idea
oflines of force was introduced by Micheal Faraday. These are the imaginary lines, but can be visualised by the
motion of the test charge. 1. • ·

Properties of electric lines of field


1. Every line of field is a continuous and sinooih curve originating from a positive charge and ending on a
negative charge.
2 The tangent to a line of field to any point gives the direction of E at that point.
3. They do not path but leave or end on a charged conductor normally when the charges on the conductor
are in equilibrium.
4. Lines of fields are never intersect. If it happens then we say that at the point of intersection the electric
intensity is zero, otherwise it would have to be taJJgential to two different curves at the same instant
which is not possible.
5. The number per unit area crossing a surface at right angle to the field direction at every point is
proportional to the electric intensity. Hence lines of field are closely spaced where the intensity is large
and are widely separated where the intensity is small.
E

lqi!- 2q,
(a)
(b) (c)
E

""'i---....._

'i
71------

(e)

+
+
~

+
+
+ ~

(I) (g)
Fig. 1.38
I
· We define unit field (1 N/C) arbitrarily as correspori'ding to unit density oflines of force (one line/ I
} m2) To dete,:mine the number of li~es per unit charg~ let lis'draw a spherical surJ'.~ce of radius r aroundJhe
' . .
! charge q, a![ts centre. The elec;icfield E atthe surfac~ is given by E = _l_j____As the density Qf!it1es
I . . ~~?
I of force is f;, \he total numbers. of lines of force origina_ttng ;J q and crossing the .spherical surface wil\ be
I " . . .
,!41tr2E =!L: Hence the numberofli11es originating froll_la•unitcharge is (1/e0).
: Eo-, . 1 , ":

Neutral point : It means the net electric field, at the.point due to all the charges is zero.
(a) Let us consider two charges of same sign are placed af some separation r. Let the neutral point is at a
distance x from q 1• For neutral point,
• I' ,1j,'
qi E2 E1 qe

or
Q •Ii

0
.,,'Ji

F x~cr'-x)3
or . - I Fig. 1.39
-Ez '
That is intensity of fields at the point are equal in magnitude and opposite in directions.

qi q2
41t Eo x2 41t Eo (r-xJ2

x2 !fl..
or
(r-xJ2 qz
After solving for x, we will get the position ofneutral point.
--,
When charges are of same sign, the neutral point must be in betweert_,the charges.

(b) Consider now two charges ofopposite signs, let q 1 = Sq


and q 2 = -2q are placed at some separation.
In this case neutral point will not be in between the·charges. It will be away from charges and close to
charge of smaller magnitude. For neutral point
q, =-2q
·E2
,.____ r - - - - - ~ x ~
Sq
or- Fig. 1.40
41t Eo (r+xf

I
or
4-
. r.
X
f. /

or '
r+x 2· /
or x = r
Thus the.neutral point is at a.distance r from½·. _<'\
, ,-

£ in terms of i' :
Consider a charge q is placed at positio/i , 0· 'with respect to
. ' 'jJ" I
the origin of the coordinate system. To" get the electric field
at a point P, we have

or
'I J,/'H
_ _ qr'
Electric field E Fig. 1.41
41tEo ,.,3 z
I.- ---- ---
or \ E ~
I
'-----""-- ---- ----·
r,

Ex. 13 A positive point charge 50µC is loc~ted in the planexy atthe point vector r0 = 2/ + 3}, where i
and J are the unit vectors of the~ ~nd'.y axis. Find the vector of the electric field strength £ and
,,\

its magnitude at the point with radius vector r= Bl - 5J. Here rO and r are expressed in meter.

Sol. Given q 50µC=50x J0-6C.

We know that E
q(r-ro)
41tEo [r-rol3
~ ( )[(s1-s])-(21+3])]
9xl0 x 50xl0-6 • • _ • 3
j(s; -SJ)-(2; +3J)i

9xJo9 x50xI0-6 (6[-sJ)


J(61-sJ)i3
If

4.5xl0 5 (6', - 81')


103
2
4.5xl0 {61-8])
2.71-3.6} kV Im
4'.5kV/m. Ans.
Equilibrium of system of charg~~ ·,_.
The equilibrium of any object of the system means, net force and net torque on it must be zero.
0

- ,.,l:,'.J
On being point charges, it requires only F net =O for translational equilibrium. For system of charges to be in
equilibrium, the net force on each charge ofthe-~y~t~m must be zero.
Stable / unstable equilibrium
After displacing charged particle from its equilibrium position, if
'
it,,return back to its mean position then it is said
to be in stable equilibrium otherwise unstable equilibrium, The potential energy U of stable equilibrium is
'.I..:!,
minimum while that of unstable it is maximum.

Ex. 14 Discuss the equilibrium ofchargeq 0 as shown in jig. 1.42.

Sol. (a) Consider two charges of same sign, for simplicity take of equal magnitude.

CJ CJo q
!
0, ............... 0( .................

q
f
q
. . . . . . . . . . . . . . . . . . . . . _ . .••••••••••• {,
~:>}:-:~ -...
.. -· ,... .. ..
<1-"' --------- -------~- ~"t>
(a) CJo q (b) q

Fig, 1.42 . ..
Let a test charge q 0 is placed at mid point 'of charges. Displace charge q 0 slightly along line
joining q and q and left free. The charge q 0 ~ill return to its initial position, so it indicates
stable equilibrium along the axis. If the charge q0 is displaced slightly perpendicular to axis of
charges q and q, it will not return to initial po·sition. It indicates unstable equilibrium along
equator.
Or q -qo q

Fig. 1.43
Urystable equilibrium along axis, and stable equilibrium along equator.

Ex. 15 A charge q is placed at the centre of the line joining two charges Q. The system of three charges
to be in equilibrium, what should be the value of q.

Sol. The charge q is in equilibrium, because it is at the centre of two identical charges.

For equilibrium of charge Q, Q q Q


e12 e12

f'net 0 Fig. 1.44

_i/'l___ + Q Q
or (£12)2 ez 0,

-Q
which gives q = Ans.
4

Ex. 16 Two positive charges q1 and q 2 are located at the points with radius vectors r1 and r2 • Find a
negative chargeq and a radius vector ;:3 of the point at which it has to be placed for the force
3
acting on each the three charges to be, equal to zero.
Sol. The position of the charges are shown in the figure
y
For the equilibrium of q 1,

Fi2 + Fi3 0

kq1q2Ci'2 -ii) +k q1q3(ii -"3)


· or
1- -1 3
r2 -lj. 1- -13
fj -r3
0 ... (i)

(i'2 -ii) (ii-"J)


Since
li'2-iil lii-"il
Equilibrium (i) reduces to

q2 q3
Fig.1.45
1-r2 -ri- 12 1-~ -r3- 12
or l,i;(ii-"J)+.{ii;(i'2 -ii)= 0 ... (ii)
Now for the equilibrium of q 3,

F.i1 + F32 0

kq1 q3 (ii - '3) kq2q3 (i'2 -"3)


or . __ 3+ - - 3 0 ... (iii)
l1j-r3J Jr2-r3J

(ii -i)) (i'2 -"3)


Since
lii-'31 Ji'2-"JI
Equation (iii) can be written as

fi((i'2 -"3)+l,i;(ii -"3) 0

fti:r2 + f,i;ii
or r3 ... (iv)
fi( +f,i;
From equations (ii) and (iv), we get

,. Ans.
<fi( +f,i;J2
1.8 ELECTRIC POTENTIAL ENERGY (EPE)

r f~
We have defined the difference in gravitational potential energy
t.U = U1-U;

= -[Wif ]grav;tionalforce •
Similarly we can define change in electric potential energy as Gravitational Electiic field
field
Fig. 1.46

..... (!)
If Wqn
• 1 is the work done by agent to move the
-
charge slowly without change in K.E. ' then we can write
Welectrfo force+ Wagent !),,K.E ..=O

Wagent - Wc1ectric rorce [Felectric = Fagent]

Thus from equation (I), ·!J.U U1-U; ~ [W1r] agent ... (2)

If our reference position is of zero potential energy, i.e.,


U; 0, r;= oo, then
-~---·:-··----- -- -
!u1
;
- U=[w1]
· ~ age_nt
... (3)
- . -----·- --~-,--_ - -
Now let us consider two charged system, in which one charge q is fixed and other charge q 0 brings from infinity
to a separation r from q, then
r

[w!J.gent f F. dr q qo
....................................

--------r·-----~.,dr4
F,""'I i
,.
f Fagen/ dr cos I 80° Fig. 1.47

_ _ q%
41t Eo r

u [wl] agent -__


00
l_qqo
41t Eo r ·

We ·have derived above equatioll' by keeping q fixed and moving q0, The same result wiU be .i
obtained ifboth the charges move.and placed at a sep,.ra\i:n_r. ~or charges qi ,a;d q2, we have , i
' Ou =! , ,, J qJq2 , qi
41'EQ _r
The potential energy of system of charge; sliow~ inj,s/I ABis ·
u ~[q1q2 + q1q3 +·q2'
. 41tEo . r12
r13' r23.
f3]·. . q,.e.·----,,,.23-.-->.,qd
1 · I
- - - - - - - _ _ _ _ _ _ _ _ _ _ _ _ _ _ _ _ _ _ _ _ __ , , __ _ _ _ _ .J Fig. 1.48 ,. , I
1. 9 POTENTIAL DIFFERENCE

In a pipe line carrying "(ater under gravity, the direction of flow of water depends on the difference oflevels of
two ends of the pipes. <Dn the same way the potential difference gives the idea of the direction of the electric
field. The electric field always runs from higher potential to lower potentiaL
The potential difference t,. V between any two points f'and i in an electric field is equal to the difference in
potential energy per unit charge between the two points. Thus potential difference

t,.V r,; E

Since t,.U -[w/] electric force =[w/]agent


1 I
Fig. J.49

...(I)

or ... (2)
Ifwe define potential energy to be zero at infinity, then potential must also be zero at infinity. We can then define
the potential at any point in an electric fieid as

[W! lagent
qo

,or

Potential due to a point charge


The potential at any point in the electric field of a point charge is the work done in bringing a unit test charge
from infinity to that point. The work done on charge q 0 in bringing from infinity to a point Pat a distance r from
the charge q ·

_ _ qqo
[w!lgent 41t Eo r ·
Thus potential at P

'' ·
[w!J.g,nt
V
qo

or I
i
V J' :,q /
-.-<--::;'
41t "Eo',,,r
,'.. . '

It is scalar quantity. Its SI unit is J/C or V.


a

Potential due to many·charges.- · r, , , , · '- ) )r, !.!' '

We can define the net potential at a point due to a system of many charges with the help of the superposition
principle, We calculate the potential d~e to each charge at the given point separately including the sign of
charges. Then we sum_ the potentials. For n charges, the net potential

. • n - - 1 " q-
V = 2, V; =; ----
4
2, _!._ •
i=t ·, 1t ~o i=l 't ·
! -~----·-- "· - ·-..-~-~-- ---·--. - ~
Here qi is.the value of ith charge, and r; is the radial distance of the given point from the charge.
,_ -·---~------------ -- -- - ---------- ----------· ....,. ____,_ _____ __ ,.

In using the formula of potential and potential energy, the sign of the charges should be includ~ ,
' " '
~--. --·· - ~-" - - - - ---------~ :, .
Potential from field

E
"'

Fig. 1.50
Let us consider a charge q is left to move in an electric field, The electric field exerts force on the charge
0

Fe "'. Eqo
For small displacement, work done by the electric field
[dW]electricforce = F,.d; ... (1)
The change in ~otential dV is gi_ven
-[dW lelectric force
dV
%

From equation (1 ), dV -Fe,dr


- - -- --Ed-r
qo

and, V c' ... (2)

Differentiating both sides of above equation, we get

..... (3)

Conclusion : ,1. In the book potential.qifference betweenA and B means VA -VB.


I
.
2. '

Va -VA = -r. B
"
£,di:
VA VB· E
or Fig/1.51
I

I
For 3D field : If Ex,£_,. and Ez are the components of field alongx,y and z-axes respectively, then we can write

£, = _av E = - ~ £ =-av
ax' y ay' z az'
and resultant field
For uniform field, we can write

V V,
E

r §
~

Fig. 1.52
r
. -- ---··-,
or Vi -Vz Ed.!
'----··-----

Ex. 1 7 There is an infinite straight chain of alternating charges q and -q. The distance between
neighbouring charges is equal to a. Find the interaction energy of each charge with all the others.
-q q -q q -q q -q
Sol.
-3a -2a -a O a 2a 3a
The interaction energy of charge at O due to charges on right ofO:

u
1
- -[q(-q)
41t Eo a
+il+ q(-q) +il+ ....
2a 3d 4a
=]
q2
41t Eo a
[-1+_1__ _ _1__+_1__ __
2 3 4
.,.]= g2
~~-fn2
41t Eo a

The same interaction energy due to charges on left of 0, so

I/Total = 21/

2
- 2q fn2
,, 41tEo a

Ex. 18 Determine the electric field strength vector if the potential of this field depends on x, y
co-ordinates as V=a (x2-y2).

av
Sol. We know that, £, --=-2ax
ax . '

av
and Ey --=2ay
ay

E = E,i+Ey}

= 2a(-xi + yj). Ans.


ELEC;iROSTATICS ' ffl
Ex. 19 An electric field E =iAx exi~t in the space, where A= 10 Vll11 2. Take the potentiai at (tom, 20m)
to be zero. Find the potential atthe origin.
Sol. Line BC is the equipotential line, so )'

Ve,= V8 =0
r·oo. 20)
Now, J8A Edx= ro0 Axdx A .,
X
0 Ii
-
12110 =IOx-
A~ 102
or VA-0 Fig. 1.53
2 0 2
or 500 V. An.,.

1.10 EQUIPOTENTIAL SURFACE


A locus of points in space that all have the same potential is called
equipotential surface. On the equipotential:
I. No net work done on a charge by an electric field as the
charge moves between two points on the same equipotential
surface. As VJ = V; and so L\.V = 0
w/ = Ll.Vq0 = o. Equipotential
surface
'
2 Equipotential surface is perpendicular to electric lines of field.
3. It may be of any shape. Fig. 1.54
-!I. The equipotential surfaces are a family of concentric spheres for a sphere of charge or a point charge and
are a family of concentric cylinders for a line charge or cylinder of charge.
V2 /C
"

•1 . ., E 1

~ Equipotential surface
(b)

Equipotential
not spherical ·
(c)

Fig. 1.55
Ex. 20 Some equipotential surfaces are shown iujig. 1.56. What can you say aboutthe magnitude and the
direction of the electric field?

y(cm)

x(cm)

(a) E
(b)
Fig. 1.56
Sol. (a) Draw lines perpendicular to equipotential. These are the. field lines ..

- vhigher .:.. Viower 20-10 10 V


E = d I 0sin30° 5cm
200V/m, E

h
and making an angle 120° with the x-axis.
. .
(b) For spherical surface, we have V=~-
r .
Choose any of the equipotential, let of60V. Fig. 1.57
k
-=> k = 600V-cm
10

dV k
Electric field E --=-
dr ,2

600V-cm
,2

6
--=600V/m Ans.
<o.Il2
Ex.21 A test charge% is moved from A fo C along the path ABC as shown in jig. 1.58. Find potential

Sol. dilferencebehveenpointsDandA.
Since AB is the equipotential line
· . ·
4
: · ~- h ~ -
~~
C: :-£
_
VA VB
Also VA-VD V8 -VD=ExBD
Exbcos8 Fig. 1.58
Ebcose. Ans.
1.11 · ELECTRIC FLUX
The word "flux" comes from the Latin word ~eani°ng "to flm_;". Flow may be ofair, wateror electric field through
any surface. · · ·
Electric flux through any surface may be defined as the total field lines crossing that surface.

The electric flux d<!> through small element of surface -(d,i) is defined as

d<i> = . E1.dA
=_··(Ecos0)dA
,aJp
Small element of
= EdAcos0 surface

The total fl~x through the surface

<I> JEdAcos0 Fig. 1.59

or I <I> Je.aA
, __ - -·
If E is·uniform throughout the surface, then
<I> EAcos0

'
More about cl>

I. We may take dA positive pointing out of surface.

Fig. 1.60
2 Electric flux is a SC!'lar quantity. It may be negative. Its SI unit is V-m.
· 3. In finding flux to charge; place the value of charge with the sign in the formula offlux.

E±v
4.

@ Q A
n
(a) (b) (c)
0 = 0°, <i>=EAcos 0°. <i>=EAcos 90° <i>=EAcos 180°
=EA =Q. =-EA
This is called emerging flux This is called entering flux
Fig. 1.61
lr:dl------------, f'.<Z'·.:·:
FC::tl ELECTRICI1Y & M)\~NETISM
7
Solid angle (co)
Solid angle is the analogue in three dimensions of the usual angle in
two dimensions.
The solid angle subtended by any surface dA at a point O and
distance r away is given by

dAcose
dro
,2

Total solid angle (jJ


L Mcose ... (!)
. ,2

From the figure (b ), 0 = 0, (jJ


LM = 4w2
r2 ,2

or (jJ 47t steradian ... (2) (b)


Fig. 1.62

J.dJ@ I.The total angle subtended by closed surface always be 47t, regardless of the size and shape of
the closed surface.
2 Let us cc;msider the field of point charge placed at 0.

1 I
E q and £'= q
47t Eo ,2 47t Eo ,,2

A<j> £Mcos0°

and A$' = E'M'cos0°; Fig. 1.63

Thus A<j)
A
Flux due to charge inside closed surface n
Consider a small element of closed surface, which subtended solid angle E
dro at the position of charge q. The flux through the element surface,

Total flux
_q_J dro Fig. 1.64
47t Eo
L_·--=~--'-·-~~~-------~---E~i.E_cra_o_SE_ll_n_cs_~Jil-~
,: .:. :• r;t • q •,'I',
As dw=w, f ,•:.
.:..· --·-·w
- 41t Eo

_q_X41t=_I!__
Also w=41t,
41t Eo Eo

The total flux through a closed surface always be qi= .!L, regardless of shape and size of surface
I - Eo
I d .. f h .
1 an pos1t1on·o c arge.

Flux due to charge out of closed surface


Consider a point charge q placed outside the closed surface.
If <l>entering and <l>emerging are the entering flux and emerging
flux through the close surface, then

lf>Total =<l>enteri~g- + 4>emerging


= -qi+qi = 0.

; Fig. 1.65
.
I
Thus electric flux.associated with the closed syrface of any shape due to charge out of the surface will be zero.
' ,_

J
Ex; 22 Consider the"system of three charges as shown injig. 1.66. Calculate the total flux associated
·
1
~,. - · ··
1
• With:thectOSJ~ silrr3ce.' 1
' ·-· · : ·· t• • '· ·· · • · '
:,•;'1, !\,. , '. ·.:·, ,, , , ., , I•• r C/3
Sol. The flux associated wiih charges inside closed surface is

Flux associated due to charge out of closed surface,qi0 = 0.


Ans. Fig. 1.66

Ex. 23 Find total flux associated with the surfaces shoivn injig.. 1.67.
di1 E
0=90°

Sol. (ii)

0 = 90°
,LJ
Fig. 1.67.

_<l>Total = J[ E dAcosl 80°+ EdAcosO°]


_+EdA90°+EdAcos90°
=O
Thus the flux through closed surface is zero.
"'

Ex.24 Find the flux due to electric field through the surfaces as shown infig. 1.68.
J
E <L7i l:

(a)
Fig. 1.68
Sol. (a) Let us consider small element of surface dA. The flux through this element surface
dq, = EdAcose = EdAcosO=E dA
2
The total flux associated ¢, = J EdA = EJ dA = Ex21tR = ~1tER 2
(b) The flux through element dA, E ~
II
dq,= EdAcose
. J.,
The total flux
J
¢,=£ dAcos0
'' ..'",t4
= E J( horiz~~tal ~rojection of d A)
''
''
''
•I'¥,
..'
' ' cos 0
.,.i<L,1
= Ex1tR 2
= 1tER 2 · Ans. Fig. J.69

Ex. 25 A point charge q is placed atthecenire of the cubical box. Find, (a) total flux associated with the box
(b) flux emerging through each face of the box (c) flux through shaded area of surface
q
Sol. (a) Total flux <l>o =- .
Eo
(b) This total flux is emerging equally from each face
of the box (6 faces). Thus flux through each face,
.. -·······
¢,=-q- Fig. J.70
6Eo
(c) The flux through shaded portion

cp•-cp_ q
4 24Eo.

Ex.26 Find flux due to a point charge q P,l~~ed


'.
(a) atcentreoftheface ·· (b) corner of the cubical box.
,.,------";<lq

(a) Fig. 1.71 (b)


Sol. (a) If charge is kept at the.centre of a,face,,,~_irst,we should enclosed the charge by assumi~g an

identical imaginary cube. The total flux through the cubes <!>total = .!L.
. . ~

Thus the flux through given cubi <I>= _q_


2~ /, 5/
------7r~-----;, ,/ I ,,
,··. '. ,r--+------;;f---r------7'(
'''' ,,' : ,,/' 6 /'
/·~-~ : ,<--- ______ .,/
----;----+--+---· ' ''
''
'JJi( l!J,~
'' .
,, I I I I
oq ••;, :_., : I I

~----,-~-,;' '
, r--- ---r------ --- ---t------ 8
I ,
,,.
·, ., ,,
- I
:
I
,,.
,/ ',
,..,/
, • , ,·· : ,/ 1 7, ,.,.
c'•-'-----~-----~··· ,.,-" ____________ 1______________ ;,,.
Fig. 1.72\,
(b) If the charge is placed at the comer ofa cube, seven more identical cubes are required to
. . (l~

enclose the charge completely. The total flux through all the faces of 8 cubes is <!>total = .!L .
. 1~! 11 ;1,• Eo

The contribution of this flux through each cu,be <I>= _q_. This flux emerging equally through
. 8~
three front faces of the cube. Other three faces; junction of which charge is placed have no
---q/8 E q
.flux. Therefore the flux through the fact,,,=,~= e ·
•. '}1 • ')f,
24 0
J,,.' ,,,.• i II l :,
1

-p;g.-1.73
I -. •
Ex. 27 . Find flux due to the charge q placed as shown'fo'j,g. J. 74.

q
• CJ
............ ...q
(a) (b) ' (c)
Fig. 1.74
•/

Sol. (a) <1>=.!L (b) <i>=sO (c) <i>=_q_


Eo · 2e0

.
~
J

l!IVf;~.E.t1i;:GT.ifcJn.:~&. ~GNETISM
1.12 'GAuss's LAW oR·oAuss's THEOREM ,.
Gauss's law is the very powerful tool of electrostatics. It can be used to find total flux associated with the closed
surface due to charges appearing in the space. By doing this we can calculate electric field at any point of the

1', -
a
surface.According to it, the total flux of the net electric field through closed surface (Gaussian surface) equals
' '.'
the net charge enclosed by the surface divided by Eo. Thus if q," is the total charge inside the closed surface,
then
7,;
pi.dA ·

ProofofGauss's law
Consider charges q 1, q 2, ••.•...•. , q" are inside a closed surface and
charges q' 1~ q'2, ••••••• , q'nare outside the surfa<:~- tJ

/_" ,- .• 1
Fig. 1.75
-- - -·-. -~ •1' ' '
If £ 1, £ 2 , ••..•••• ,E.; £'1, E' 2 , •.•••••• ,E'n are the fields produced by the respective charges; ·inside and outside
the closed surface, then resultant electric field at any point P on it
.
"
.. .
The flux of the resultant electric field through the closed surface
, I ,'

<1>= pi.dA = [pi 1.dA+p i 2 .dA+ ....... +pi•.dA]+[p E'i .dA+pi'2 .dA+ ..... +pi•,. .dA] ... (I)

Here pi1.dA is the flux due io charge q 1_which is q 1I Eo and pE' 1.dA is the flux to charge q' 1• which is zero,·

because it is out off the closed surface. Thus_equaiion (I) can be written as;

1L+i£+ + q.]
[ Eo Eo .... Eo + [o+o+ ...... +ol

. '1 . .. -
. . -'
or .. :(21

1. The net charge is the algebraic sum of all the enclosed positive and.negative charges, and it
Can be positive, negative, or zero.
2. The electric field allpearing in the Gauss's law is the net electric field due to all the charges present
inside as well as outside the closed surface. On the other hand, the charge q," appearing in the la':" js
only the c~arge .insfde the closed surface. _. · ·-
J ••
ELECTROSTATICS . ffl
The following observations are useful in calculatin~ total flux
l·,· ''11.

t If E is perpendicular to a surface area A(0 = O) at all points,


and has the same magnitude at all points of the surface, then

pE.dA = pEAcosO= EA.

For spherical surface EA= Ex 41tr 2 = 41tEr 2 .

2 If E is parallel to a surface at all points, EJ. = 0 or 0 ;s 90° and therefore (a)

f
pE.dA = EAcos90° = O.

3. if E is zero at all points of a surface, the integral is zero.


4. The surface ~o which Gauss's law is applied need iiot be a real j)
0=90°
physical surface, such as the surface of a solid body. Indeed, in
most applications of this law, one considers an imagi~ary or
geometrical surface that may be in empty space, embedded in a
solid body, or partly in space and partly within a body.
(b)

Conductor with excess charge


If an excess charge is placed on an isolated conductor, the_ charge ········· ···-----..\
will move entirely over the surface of the conductor. It is because of
the fact that like charges repel each other. An internal electric field
./
.: ../ .
does appear as the conductor is being charged. However, the excess
charge quickly distributes itself such a way that the internal el~ctric . . . ,. _ __ _ . . -:·<
field is zero. The movement of charge then ceases, and the net force
····- ......... -·
• Gaussian surface
on each charge is zero; the charges are then in electrostatic (c)
equilibrium.
Fig. 1.76

The fact can be verify by Gauss's law. It has explained that the electric field E is zero at all points within a
conductor when.the charges in the conductor are at rest. We may take an imaginary surface in the interior of the
conductor·as· shown in figure. Because E is zero everywhere on the surface,

By Gauss's law pE.dA

qin
or 0 'Eo·· . ,
( ..

which gives 0.
--..
. _- . -- --- . ---------~. - ,
. - ~ .. ~/ -
ELECTRICIIY
~-------------·- & MAGNETISM' . .
,..,_" --· -·-----
... --
- .
- .--·-
-
-,:___,
.
__
·... __
-.
-~·:, .·, -----
Cavity in_ the conductor
Suppose there is a cavity in the charged conductor, and there are no
charges within the cavity.

For the surface A, 0


.
Since £ is zero in the conductor, :. qin

Further more, consideration of a surface such as B shows that the


net charge on the surface of the cavity must be z~ro. Thus the entire
charge on the conductor lies on its outer surfac¢~not on the cavity
wall. /
0.


-~

Gaussian
surface

., Fig. 1.77

Charge is placed inside the cavity


I. Suppose that there is a conductor inside 'the cavity but
insulated from it and that the inner conductor has charge q.
' . Cavity
By Gauss's law
!'

ft:.dA

or 0 =
Gaussian Surface
Fig. 1.78
Since E is zero inside the conductor, which gives qin = 0. As the charge inside the cavity is q. so there
must be a charge on the cavity wall, equal and opposite in sign to the charge q. If outer conductor is
initially uncharged before the charge q is inserted, there must be a charge on its outer surface, equal and
opposite to the charge on the cavity wall, that is q.

2
q'""
+

8 .+
+ +

(a) (b)
!1 J)J .J'

Fig. 1.79
Consider a charged conductor having charge q' initially. A charge q is placed in a cavity in the conductor.
The charge on the outer surface of the conductor = q' + q.
3. _Lines of field inside cavity due to charge at its centre
Case I : Spherical cavity ,,Case 2 : Non-spherical cavity

+-~_;i'
zst3i +

+

B
_iillSt_
+q
+

•."~~A
.0 -;s:- -
+ +
E=O • 1 .~1 E=O
+ +

E,, =
Ell. =
r:_, vs E~ </~, VA= V8
(a) '/J (b)
Fig. ~.80 bi
Conductor in external electric field • i1..,t

Consider an uncharged conductor is placed in an external electric field. The free electrons in the conductor
distribute themselves on the surface as shown, Reducing the net electric field inside the conductor to zero and
making the net field at the surface perpendicular to the,surface,

90"
=
·\_...----
+ E-o
E=O

(a) (b) +
Fig. 1.81',
Potential of earthed conductor is zero "''' -'' +-
~ ,_ +
Let us consider two concentric conducting shells. Inner shell is
given a charge q. The charges are induced at outer shell, as shown
+ - + +
in fig. 1.82. The potential of any point on the surface of outer shell
+ +
+
V--1-['f__'f_+'L]--l_'f__ (a)
41t e 0 b b b 4n e 0 b

Now let outer shell is earthed. The charge on outer surface ofo~ter
shell neutralise by electrons coming from earth. But negative charge
on inner surface of outer shell remain in tact because of attraction of
charge of inner shell. Potential at any point on outer shell

V =_I-{' !__'!_}= 0.
4lt Eo b b
., ,i-ir~r
·1boo~ (b)

Fig. 1.82
_1.__E_·
'1/1_· _LE_C_J:_'R_I_C_flY_&_· ·_•_G~NE_·
_MA~· _1:_1S_M_......;,,_··_·:~~:::_-::~-.------,-... ;:•~J
Ex. 28 A point charge q _is placed on the apex ofa cone of semi-vertex angle e . Show thatthe electric flux

'q
through the base of the cone is --(1-cose).
__ 2 Eo

Sol. Consider a Gaussian surface with its centre at the apex of the cone. The flux through the whole

sphere is -'L, so the flux through the base of the cone


Eo
where A = area of sphere below the base of the cone
and A 0 = area of whole sphere which is 4itR 2 •

/~\
\ q ;
//?r.
:
i
ds \ ......
~
:
\
....
'
\... ,.:
/
·••··•... ...-·· •.
······•··· ···.................-····
(a) (b)
Fig. 1.83
To find A, choose a surface element confined in angle da at an angle a. The area of the element
strip
dA (21tr)ds

21tRsina(Rda) [r = Rsin a]
21tR 2 sina da

Surface area A

or A 2
2itR (I -cos0)

A q . 21tR 2 (l-cos0)
Thus desired flux
Ao~= 41tR 2

= _q_(l-cos0)-
2Eo

Ex. 29 Two charges +q1 and-q2 are placed at A andB respectively. A line of force emanates from q at an
1
angle a with the line AB. At what angle will itterminate at--<J2 ?
Sol. The number of lines of force emerges is proportional to the amount of charge. The line of force
emanating from q I spread out equally in all directions. Hence lines of force per unit solid angle are

ql and the numberoflines of force through cone of half angle a is iL.21t(i-cosa). Similarly
47t 47t

the number oflines of force terminating on --</z at angle I} is


L 0 Ei:£cmosrAriCS 0 ffl
i£21t(l-cosP)
41t
. ;~B q, q,
Fig. 1.84
As the total lines of force emanating from q 1 is equal to the total lines of force terminating to q2, so

1L21t(l-cosa) i£21t(I-cosP)
47t 41t

or
q, z - 2 -Cl
- . Sin qz _2sin 2 f
2 2 2 2

or ·sin)?_ fsin~
2 2

1
p = 2~in- [ f sin:]. Ans.

Application of Coulomb'.s law and Gauss's law


Coulomb's law is the fundamental law of electrostatics, but for symmetric charge distribution Gauss's law can be
used. For electrostatics problems, Gauss's law is equivalent to Coulomb's law. In Gauss's law, the closed surface
is ofany shape. The Gaussian surface will often be a sphere, a cylinder, or some other symmetrical form_ But it
must always be a cl~sed surface, so that a clea~ distinction can be made between points that are inside the
surface, on the surface, and outside the surface. ·
1.13 APPLYING GAUSS'S LAW : SPHERICAL SYMMETRY

(i) Charged conducting shell: Fig. 1.85 shows a charged sphe~ical shell of total charge q and
radius R. The charge of the conducting shell will spread uniformly over its outer surface, so charge
inside the shell will be zero.
For the Gaussian surface, r <! R, ..-----·· q+ ·--------
= q.
....... +-----r----+ +;··. . _
qin
:. \ II

By Gauss's law fe.t1"1 - qin _,\___ + T ~E


Eo

- ._______ ~--f _______ ..---~aussian surface


or ~E.dAcosO = .!L
Eo

.!L Fig_ 1.85


or ... : ~ E~dA Eo

or

· Ex41tr-
? .!L. "
Eo
For r < R, the charge inside the closed surface, q;" = 0, and so£= 0.

+if
+ +
+ +
+ +
+. • +

+ +
+
+ --+---+

- - - -. - - - -
Eoc
~. l,,
j ,-
. I
><---+----'=-+r
R
Fig. 1.86. Variation of E wiU1 r.

J@# Above"obtained results are also applicable to the conducting-sphere.

Electric potential :

For the potential at r <= R, the charge of the shell can be assumed at the centre of the shell, and so
~lectric potential at a distance r from the centre of the shell, ·

V = ( 4it1Eo )~. r>R.


+ +if +
+ +
At surface of the shell, r= R, and so + R +
+ +
V = (4lt1Eo)!, r=R +
+
+ +
For r < R, the electric field is zero, and so electric
+
potential remain constant from centre to the surface
of the shell. V
( I Jq
4it Eo // ············-~

r
R

Fig. J.87
Self energy :
The work done in charging the body will store in term of its self energy. In the charging process, the

potential of the surface of the sheH increases from zero to a value (


4 0
1
lt e ) !, so the average potential

during charging
f! .
n' .·1 "

v.v
[o+~{]= __ !l...
2 81tEo R

Self energy. u work done in charging


vavx q

1
( 8it Eo !}
or u = __ t_
Sit e 0 R
As there is no electric field inside the shell, so.this energy resides from surface to infinity.

I .A body is given a charge q in air and isolated from the source, its potential

q
V ---
4it e 0 R •

Fig. 1.88
where R is the radius of the body. When this body is taken into a dielectric medium ofconstantk (say
in water), its potential becomes

V'
___..,_ q V
4iteo k R k

Fig. 1.89
2 A body is charged to a source of potential V and the source remains connected with the body. When
this body is taken into a medium its potential remain same but charge increases to kq. The new
potential of the body
q'
V' ---
47t Eo k R
Since V' V,
-1 q' ___ q
f • 4ite0 kR 47t e0 k R

=> · = kq. q' • Fig. 1.90


(ii) Uniformly charged sphere (non conductor) ;/'
Consider a charged sphere of total charge q and radius R. C~oo~e a Gaussian surface with r > R, the
entire charge lies within it. The charge produces an electric field on the Gaussian surface as if the charge
were a point charge located at the centre, and so
+ :!"-- l.<:i-
E '( - -I -) 2
q ' r>R . ••• ,.t. '•.+
+:' . }' \+
41t Eo r
+i.+
··-.. + + _..··'+
For the Gaussian surface at,r < R, charge enclosed in the sphere of radius r, + -...... +
+ +
-----"--Gaussian
3
4 3 ='}__
,_q_x-rrr r Fig. 1_91 surface
_irrR3 3 R3
,3 l/

+ + +
q'in
By Gauss's law f E.dA Eo + •
:+ + +
+ + +
q;.3 I R3 + +
or 2
E~_4rrr . ,Ea '· . 'HI
'
1
E =·
1
( 4it E )
0
!~ (4~ Eo ;, }
.Jf----'-~--_c:=-__.r
At r = 0,E=0. R
Electric potential : Fig. 1.92
. .
For the potential at r;,, R, the charge of the sphere can be assumed at its centre, and so electric potential
at a dis!an_ce r from the centre of the sphere, .
.
,,;-

V = ( 4rrl~J~ ·'r>R

At the surface of the shell, r = R, and so

V = ( 47t]EJ; r =R

Electric potential at r < R: lfE 1 and E2 are the electric fields outside and inside the sphere, then
,,
. ·E1 ( 4rrleJ r~ and
If
Electric potential at any point is defined as,

v = -[Ed;
+ +
Fig. 1.93

- [J=R(
-I- -
4rr Eo r 2.
q)
dr+ J'(
-I- - dr
R 4rr Eo R 3
qr) ]
'After simplifying, we get
,.· .... ' .. ,.
' . ~ .~ .. - . .•_ ·''
At the. ,,centre of the sphere, r = 0, (. 3 \'L
\8rrE0 /R
3 .. I ) I/
t.e., ,Vcentre z~urface ( ~rr e / R
0
L-----''--=:..+r
Self energy of the charged sphere R
Fig. 1.94
Fig. 1.95 shows a sphere of total charge q and radius R. Take an element of thickness drat a radial
distance r all over the sphere. If q I and q 2 are the charges on the element and the sphere inside, then the
energy of this system,

_ _ qlq2
dU = 4rrEo r

where

and Fig. 1.95

The total energy of the charged sphere

qr J)
(3q 2d
)(
U=JRdU=(-'-)JR F1r r //3
o 4rr e0 o r

3(5 4rrIe )q2R


=----
0

3 q2
or u 201t Eo R .

Ex. 30 Can a metal sphere of radius I cm can hold 'a charge of I coulomb?
Sol. No, The potential of the surface of the sphere is given by

_ _ !]_'
V = 41t e r
0

9
9x,10 X .
1x10-2
9x 10 11 V.
This potential is very much greater than that required to ionise the air (3 x 104 V) and so charge will
leak to the surrounding air.
) ·

1.14 APPLYING GAUSS'S LAW: Cn.JNDRICAL SYMMETRY


Fig. /.96 shows a small part of an infinitely long cylindrical nonconducting rod with a uniform linear charge
density A. Let us find an expression for the electric field Eat a distance r from the axis of the rod .

. _____ li 11: --~Gaussian surface

++± + .~!:.+ + + + + + :ti+:++++


. '·
v------- f ---------· _:

Fig. 1,96
I
Choose a Gaussian surface in the form of a circular cylinder of radius rand length f, coaxial with the rod. The
Gaussian surfacemust be closed, so we include two end caps as part of the surface. There is no flux through the
end caps because £ , being radially directed, and parallel to the end caps at every point.
Using Gauss's law, We have
pi:.dA = q;"
Eo

or [f -E.dA-]endcaps .+ [f-E.dA-]cylindricalsurface =eAe-0


At
or f
O+ £Acos0°=-
Eo

or

At
or Ex2rcrC=-
Eo
.A
E= ..
2rce0 r

This is the electric field due to an infinitely.long, straight line of charge at a point that is at a radial distance r from
the line. The direction of E is radially outward from the line of charge if the charge is positive. The result also
approximates the field of a finite line of charge, at point that is not too near the ends as compared to the distance
from the line. ,,
Electric potential
Let us consider an infinite line of charge of uniform linear charge density A. The electric field at any point Pat
a distance r from the line of charge is given by

E'=--A-
2rc e0 r
The electric potential at the point ' i.1 ..... , ...
I'

,.
+ +
r

+ + + +
= - J,. - -"-- d r cosO° Fig. 1.97
oo 27t Eo
I"

=- 21tAEo I/nr I,.~ ==


Thus the electric potential due to infinitely long line charge is infinite everywhere. In practical problems, we are
interested in potential difference between two points at finite separation. In this case choice of zero potential is
arbitrary. lfwe take v0 = zero at r= r 0 , then

Vp-V. =-
Q J,. ---dr=--lnr
111
A
2n-e
-A I I,.
21t e
0 ,.· 11 'h

or

Vp "-
--[lnr-lnr0]
21t Eo .
1.15 LINE CHARGE OF FINITE LENGTH
Consider a line charge ofuniform linear charge density A. Let us find the electric field at a point P. distance r from
the line charge.

+++++++l++t
X
+ ++
drl+----

Fig. 1.98
Take an element of length dx at a distance x from the reference point as shown in Fig. /.98. The element
subtends an angle 0 from the line of r. The charge on the element, dq = Adx.
The electric field due to the charge of the element

dE(l)dq
= 41tEo (x2+,.2)
,. .
···...~
From the figure x = r tan 0 ,
dx = rse~ 2 e(de)
' I t
2
,,. : . ( 1 ) A.(rsec 0d0)
dE= - - -
·,, ·' 4iteo r 2 tan 2 0+r 2

2
" (
I ) 1,.rsec 0d0
, · ·o:.= 4iteo r 2,(tan 2 0+1)
' I 'f;, , . ., • ,,1 ,'('I _,;_,•/
,, , .
,, , , -~·(···1
--
·.)· ,,_de'.· --· ,, I ,';
. '.
. 4it e 0 r
For the simplicity, we can resol_ve this fi~ld into two perpendicular components;
dE_,.. = d£sin0 and dEy = dEcos0
The net electric field alongx- direction . ,)

~x =epd£sin0

"- J'"·sin0d0
4~~o.;[. ~e" .--~
::rJ .~t
~ 7
,·,
C

' .
' . , _ . I-cos0
. la-• I'· ,
=
4it Eo r e
/

or

Similarly

,. 4it e 0 r
"- fa-P cos0d0

'or. ,,
\ - \ '

The resultant field at point P r'.f, · .'1' '. • ; 'lJ..,.., 11



!.
Special cases .
I. Infinitely long line charge

l!.~.
p

Ex =0 and Ev= "- [sin~+sin~J r


· 4n e 0 ,- 2 2

+ + + + + + +
=--- Fig. 1.99
2n e 0 r

2 Along the end of the line charge

It
a=O and~=-
2

Ex= "' [cos~-coso]


4n e 0 r 2

+ + + + + + +

and Ey '=. "'[·![·]


4n e0 ,-
sm-+smO
2 Fig. I.JOO

=---
4ne0 r

E
tany=_!.= I
Ey

It
Y=-
4
or 45° .

1.16 APPLYING GAUSS'S LAW: PLANAR SYMMETRY

(i) Non conducting sheet : Fig. 1. 101 shows a portion of a thin, infinite, non conducting sheet with a
uniform surface charge density CJ. Let us find the electric field E ata distance r in front of the sheet. By
symmetry, the field must have the same magnitude and the opposite directions at two points equidistance
from the sheet on opposite sides of it. The field lines are directed away from the sheet ~nd perpendicular
to it.

+- r-+
(a) (b)
Fig. (a) Perspective view, (b) side view.

Fig. I.IOI
A Gaussian surface is a closed ,cylinder with end caps of area A, arranged to penetrate the sheet
perpendicular as shown. Because the field lines do not pierce the curved surface. So there is no flux
through this portion of the Gaussian surface. Thus the total flux through the Gaussian surface reduces
to the flux through end caps, which is simply EA + EA = 2EA. Thus by Gauss's law
...
,,,.,"

l' ! j fE.dA= q;"


Eo

2EA = aA
or
Eo
~ • .. ·a" .. . :
iI ·E=-.-·2 ., • • lI
,' __,. __ ,Eo j
Thus the magnitude of the field is independent of the distanbe from the sheet. In practice an infinite
sheet of charge does not exist. The above formula can be used for sheet of finite size if points under.
consideration are not near the edges of the sheet and the distances from the sheet are small compared to
the size of the sheet.
(ii) Conducting sheet : When a charge is given to a conducting sheet, it distributes itself over the entire
outer surface of the sheet. For the thin sheet, the charge distributes on both of its faces. Thus a
conducting sheet is equivalent to the combination of two non-conducting sheets, with the same charge
density. Suppose a is the surface charge density.

a cr a cr
+ + + +
+ + + +
+ +
+ + + +
+ + + +
+ +
+ + + + ...
+ + ·--- + +
r. • ,'; - ..J,",.

+ + ' ! .J ·., ....


,(a)A condu.cting·shcet (b) Two non-conducting
i • ,...,

of charge. sheets of charge.

.. •l

Fig. 1.102
The. electric field thus arises frcifu the superposition of the fields of two non-conducting charged sheets.

\

.. . CJ CJCJ
Thus field on the right of the plate ER=--+--'= -
- ,, .2e0 2e 0 e0
and field at the left of the plate·
CJ
"• E=O
+
+ •
l"ii~- + . . . . I!~(,

+
CJ CJ
Field inside the plate E=---=0 • +
2 e0 2 e0 +

Fig. 1.03

;ww For surface charge density CJ, the total charge on non-conducting sheet of area A, q CJA. The

total charge on conducting sheet, q' = 2CJA = 2q.

(iii) Electric field due to two parallel non-conducting sheets of charge :


Consider two infinite sheets of ch_arge with surface charge densities CJ 1 and CJ2 on them.
cr, "2
+ +

---- 3
+
+
+
+
2
+
+
+
+
--
I

+ +
Fig. 1.104

. E _ CJl CJ2
Field at the point I, 1---+--
2Eo 2eo

3 E3 = ~ i - ~ = CJ1 +CJ2
' 2e 0 2e 0 2e0
-c,
Special case: For CJ 1 = CJ and CJ2·= -CJ +"
+ +
CJ-CJ

--
+ +
£ 1 =--=0
2e0 + .+
3 + 2 +
CJ-(-CJ) CJ
= + +
+ +
CJ-CJ
£3=--=0 Fig. 1.105
2 Eo
Thus in case of. two parallel sheets with equal and opposite charge densities, the field exists only
between the sheets.
!
-
Ex. 31 (a) Two large thin conducting plates are placed parallel to each othe~ at some separation. One
of the plates is given a charge q, find the charges on the faces of the other plate.
(b) q;
If one of the plates is earthed and other plate is given charge find charges on the faces of
the plates.
Sol. (a) Fig. 1.106 shows two large conducting plates, left plate has charge q. Because of induction
of the charge of left plate, the negative charge -q will induce on the front face of the right
plate. As net charge of the right plate is zero, so +q will appear on.th~ other face of the plate.

+q12 -q/2 -q/2 +q/2

.;( B C D

Fig. 1.106
If A, 8, C and D represent the faces of the plates, then
qA Q, qB= +q
qc -q, qo=+q

+q -q

,'

A B C D

. Fig. 1.107

(b) When the plate is earthed the charge of the face D gets neutralise by the elcctrons·coming
from the earth, and so ·
qA=O, qo=+q
qc=-q,qo=O.

Ex. 32 Two large thin conducting plates with charges, q 1 and q 2 are placed parallel to each other, find
charges on the faces of the plates.
Sol. Charges of the plates will dj_stribute equally on their faces, as shown. In addition to these charges,
the charges induce due to mutual interaction. It can be easily. understand by the fig. 1.108.
If A, B, C and D represent the faces of the plates, then
rc~-----'"'-"'' ,;,•;
-
EutmoSTATICS-
q, +qz
2
;, ;·1
q, -qz
2 • ! '~. ,---- -
:+~
: 2
~--
-( q, ;qz) A B C D

and ( q, :qz) Fig.1.108

Special cases :
I lfq 1 = +q andq 2 =+q,then
qA =q
qB =0
qc =O
qD =q
Thus when equal and same sign of charges are given to the
A B C D
plates, the charges will appear on the far faces of the plates.
Fig. 109
2. lfq 1 =+q andq2 =-q,then +q -q
qA =0
qB =+q
qc =-q
and q0 =O'
Thus when plates are given equal and opposite charges, the
charges will appear on the near faces of the plates. A B C D
Fig. 1.110

Ex. 33 Three identical metal plates with _large surface ar_e kept parallel to each other as shown in
jig. 1.111. The left most plate is given a charge Q, the right most a charge-2Q and the middle one
remain neutral. Find the charge appearing on the outer surface of the right most plate.

Q -2Q f_'Q _____Q_: : l


Sol. ,-0
: 1' J"!
·---- -- --~
' -'
1- - -
-Q,
'
___ ..

(a) (b)
Fig. I.Ill
Induce charge will appear on the faces on the plates,iif addition to the charges on the plates. The
charge on the outer surface of the right most plate, ···'·; •;,.;

Ans.
.- -
1.17 CONDUCTOR OF ANY SHAPE
Consider a charged conductor of irregular shape. Surface charge density on it varies from point to point. Take
a small element of area A, with surface charge density cr. Let us construct a Gaussian surface in the shape of a
cylinder. Electric field is directed outward and perpendicular to the surface at that point. So curved surface does
not contribute to the flux. Thus the contribution to the flux is through the end cap only, which is EA.
From Gauss's law ,4

qin
pE.dA Eo Gaussian
smfacc
crA
or EA
Eo

O"
or E = Fig.1.112
Eo
This agrees with the results already ,ol)tained for spherical and plane surfaces and is known as Coulomb's
theorem. '· - ' - 1
•• ( ' , ' f

1.18 MECHANICAL FORCE ON THE CHARGED CONDUCTOR


It is known to us that similar charges repel each other, hence the charge on any part of surface of the conductor
is repelled by the charge on its remaining part. The surface of the conductor thus experiences a force as the
charge is bound to the surface.

. l L , ;
. .-
:'- • .• ·

Fig. 1.113

Let us consider a small element-of conductorof area AA. If £1 and £2 are the fields produced by the element
and rest of the conductor, then resultant field at point P outside the conductor

... (i)

and field at point Q inside the conductor


,-
~ ', ~£1 +£2 0 ... (ii)
On solving above equations, we get
-
.ELECTROSTATICS
'H L:.
Force exerted by rest part of the conductor on the element,
M E2q1'
,,.
~x(cr6A) "
2Eo ,

2
M cr Eo 1
or - - = -(cr/E 0 ) 2 = -E 0 E 2
M 2Eo 2 . 2

For medium ofdielectricconstantk, IM ___ ___ t


r .. ,,_ -- -- - - - .... ,
jM E~ ~~--

Thus the mechanical force per unit area on the charged conductor is cr2 / 2 Eo .
'

1.19 ENERGY DENSITY


,..
The electrostatic energy due to a charge on a conductor remains in tlie i\i'edium surrounding the conductor. The
charge on the conductor simply defines the surface condition existing in the electric field. So the energy is due
to electric field which surrounds the medium. The force on the M area of the charged surface

!).F = (~ E}M.·'.:
2

If the surface is displaced along the direction of force, then AF


the work done= force x displacement

or (E°zE2)Mx_
This work done is stored as energy in the medium. Here (M)x is the
volume swept, and so energy per unit volume or energy density
i 1 ' , ;
IL___
u = -E 0
2
E2
__j
- Fig. 1.114

. I. The direction of E is immaterial in considering its energy, because it is either positive or


negative, £2 is always positive and , so there is always a pull on the charged surface.
2 For a sphere ofradius R, with charge q,

E =

u -1 Eo k [
2
1
4it Eo k R 2
q] 2
in a medium

1
( 32it 2 Eo k
)q2
= R_4'
The electron volt
When any charged particle is accelerated through a potential V,. it acquires a kinetic energy.
K q VJ
For an electron or proton, q = e = 1.6 x 10- 19 C .
K (1.60 x 10-19) VJ
This amount is very small and is multiple of 1.60 x 10- 19, hence we introduce a new unit of energy, known as
electron volt (eV). Thus
1
= ·1· ,._'6 .,,X 10-19 1' . 1t
' _ __ 1· e V.. _______ .i

Force on an uncharged conductor placed in an electric field


Let a small sphere A of volume Vis placed at a distance r from a point charge q at 0. The dotted lines show the
direction of the lines of the force before the introduction of the sphere, while the full lines shov,, the <jirection of
lines of force when sphere is placed in the field. Assuming no distortion of the field occurs due to the presence
of the sphere, the intensity of electric field at the centre of the sphere

E= ( 41t :o k) r~

where k is the dielectric constant of the medium between O and A.

-- ... -..
q
0

·,
• ..
Fig. 1.116

. The potential energy which is confined in volume V,

Let F be the force that the sphere experiences due to the field, then
du
. F =
dr

or
...

1.20 ELECTRIC FIELD DUE TO CHARGED RING.


Let us consider a ring of radius R with a uniformly distributed charge of density A per unit length. We may
imagine the ring to be made of insulating material, so that the charges can be regarded fixed in place.
d

Fig. 1.117
Take a small element of ring of length ds. Since ;\, is the charge per unit length, so the charge on the element
dq = Ms-
Assumingdq to be a point charge, the electric field due to this charge at a distancex from the centre of the ring,

I )dq ( I )Ms
dE =• ( 47tEo --;z= 47tEo ?
From the geometry, ,2 = R2 + x2

dE = (-47t_le_J-(R-2~~-'x~2~)

Fig. 1.117 shows that dE is at an angle 9 to the central axis and has components perpendicular to and parallel
to that axis. All the perpendicular components get cancel out because they are equal in magnitude but point in
different directions. Thus resultant field is due to the parallel components; they all have the same direction, so
the net field

E = Jd£cos0

(;\.x21tR) cos0
41teo (R2+x2)
If q be the total charge on the ring, then

AX27tR = q, also

E ~ (47t!EJ :2 ·
This is the field as the ring "looks like" a point charge.

It is interesting to note that ifwe place an electron of charge e close to the centr, of the ring, it would experienqe
a force - · ' ·-' · · · · · - J . ·

Frest -Ee.
1
-[( 4rr E0) (R2 /:2i312 }e)
Forx<<R,

F rest -. I- -
[( 4rr Eo R3
)qe] '
(-x)
_

Acceleration a ~ '. Fr,;t =(-'-,)__!l:___(-x)


m , 4rr Eo· mR3 . - ,. ,
On comparing with standard equation ofSHM,.a -oix, we get

(0
I ) qe
, ( 4rrEo mR3

and
2rr 4rr Eo mR 3
-.=2rr ,-~~-
.'
T
co qe.
Maximum value of E
dE
For E to be maximum;
dx
o·'. I,.' -

,: lf

or
1
![(4rr EJ(R2+:2)3/2
After simplification, we get
l 0

..

Fz rJ/
Emax, .-
2
.. 4rr Eo [ R2 +(

At the centre. ofring,x = 0, :.E=O


31 [ 1
4rr E0 ; 2 ]

E'
V

~ -Rlli. 0 Rlli.
Fig. 1.118
~x
C'".
~-,;,-.,;:\

Electric potential: Potential at any point on the axis of the ring can be obtained by putting entire charge at any
point on the ring. So the distance of the point from the charge q becomes

r = ~R2 +x2

, I ·q· I q ·~
Potential
~- V - =__ 41tEo 1: - 41tEo ~R2-+x•-:

Ex. 34 Fig. 1.119 shows a plastic rod having a uniformly distributed charge Q. The rod has been bent in
a 28 circular arc ofradius R. Whatis the electric field E and potential due to rod at point P.
Sol. Consider a small element of angular width da at any angle a above the symmetry axis. The charge
on the element

dq R.da
28
The electric field al P due to this charge·

dE =

The resultant field due to two such elements =


2dE cos a, because parallel components of dE,
lie. in the same direction and perpendicular
components get cancelled. Thus. effective field
due to charge at P is dEcos a. Th,e field due to
entire rod
0
E 2fdEcosa
0

9 Qda)
I ( 20
2J ~-~cos a
o 41t Eo R2
Fig. 1.119

( :I',·)· Q
r -···· -
I- . (sin8) A,is.
or I E 41f·Eo, R 2 -0-
I
------
1
Electric potential: V Ans.
( 41t eJ ~-

Ex. 35 Two charged rings, each ofradius R with the charges, Q1 and Q2 are placed parallel with separation
x, between their centres. F,ind electric field E. and potential Vat the centres of the rings.
..
Sol. Elect~k, field.f1t the centre:due to own charge is zero, while due to charge of other ring will be·

.£ ( 41ie0 ) (R2 +Q:2i312

Thus field at the centre of the ring I,

., ~/ )21
'

R R
l
X

Fig. 1.120

and field at the centre of the ring 2

Ez o + (I- ·) Q1x .. -(-1). Q1x


41t e 0 (R2 +x2)3/2 -:; 41t e 0 (R2 +x2i312
·Electric potential Ans.

( 1 · )[QI + Qz ]
v1 41te0 R JR2+x2

and ( Qz
I )[ + Q1 ] ,
Ans.
v2 41t e 0 R JR2 +x2 ·

1.21 CHARGED DISC


•• Electric field: Consider a circula. , 1astic (non conducting) disc of radius R that has a positive surface charge
of uniform density cr. To find the electric field at any point on the axis of the disc, we can divide the disc into
concentric rings and then adding up the contributions of all the rings.

' .

Fig. 1,121. A disc ofradius Rand° unif~rm charge deusity a.'


I
Fig. 1.121 shows a ring element, with radius rand radial width dr. The charge on the ring is
. dq = cr dA = cr (27trdr)
The electric field at P due to this element at a distance.on !he axis o(the disc

I ) (dq)x
dE ( 41tEo (r2+x2)3/2

I ) cr(21trdr)x
= ( 41tEo (r2+x2i312

The field due to whole disc can be find by integrating the above obtained result from r = 0 to r = R. Thus

crx JR 2rdr
E 4Eo o(r2+x2)3/2

Substituting, r2+x2 z
On differentiating, we get
2rdr dz (x is constant)

crx dzJR
E = 4 Eo o zJ/2

R
crx z- 112
I
4Eo (-1/2) IO

or

Special cases :
cr
(i) For infinitely large non conducting disc, R ~ = , :. _£ = 2 Eo .

(ii) For conducting disc, the.charge spreads on both sides of it, and so
cr cr cr
E --+--=-
2Eo 2Eo Eo ·

The electric field due to a conductor of any shape with surface charge cr, is always ~ on its
Eo
surface or close to-the surface.
-
Electric potential
Let us now find the electric potential at any point on the axis of the non-conducting. charged disc, having
surface charged density cr. To find the potential, we can divide the disc into large number of concentric rings
and then adding up the contributions of all the rings.

Fig. 1.122
flg. 1.122 shows a ring element, with radius r with radial width dr, the charge on the ring is
dq =, a(21trdi-)
The electric potential at P due to this element at a distance x on the axis of the disc

dV =

Thus the potential due to the charge on the whole disc

·- cr Rf rdr
V = 2 Eo (r2 +x2)112
' 0 -

or ~ -:-~ ;_~T~~~~2-:; ~;11


aR
At the centre of the disc, x = 0, :. V
2Eo .·

,, For x> >R,wecan write by Binomial


112
V = ~[x(1+~)
2 Eo x2
-x]

2 2
crR ,:, cr(1tR )
4 Eo x 41t Eo x

_l )!L
_( 47t Eo X.

Here q =a(1tR 2) is the total charge of the disc.


-
Potential at the edge of the disc :
To calculate the potential at pomtP, we can divide the disc in large number of rings with Pas the centre. Figure
shows an element of radial width drat radial position r. The arc length AB= 2r8 AJ( ·c_
The charge on the element, / ~
'dq =cr(2,f!dr)

The potential due to this element

dv-(-1Eo-)dq
- 41t r

_I-·) cr(2r9dr)
( 41t Eo r

1
( 2it E )aadr ... (i) Fig. 1.123
0
Potential due to the whole disc
cr 2R .
V --J
21t 0dr
E
0 0
.

From the figure V, · 2Rcos9and


dr = '-2R sin9d9
Substituting the value of dr in equation (i), we have

82=0
V __cr_ J 2R9sin 0d0
21t Eo e,=~,2

crR
or V
lt Eo
Ex. 36 Two tiny conducting balls ofidentical massm and identical charge q hang from non conducting
threads oflength L. Assume that 9 is so small that tan 9 can be replaced.by its approximate equal,
sin 9.
(a) Showthatforequilibrium,

x = [q2 L/(21t Eo mg)] 113


wherex is the separation between the.balls.
(b) What happens to the balls if one of them is discharged, and find the new equilibrium separation
x.

..,:
-'0,0
; L
r···~
.....i ... 1'-: : .q
iF , i
: c '. X----+
1....... mg !·
Fig. 1.124

Sol. (a) Consider the equilibrium of the either ball. Electric foxce. between the balls

Fe =

From the free body diagram, we have

---
q2
Tsin0 ... (i)
41t Eo x2

and Tcos0 = mg. ... (ii)


Dividing equation (i) by (ii), we get

2
q
tan'0 · . 2 ... (iii)
41te0 x mg

, It is given that tan0 sin0


•I i? • (
..:1 • ·;,,- .. (x/2)/L
Substitutir,g this value in (iii) and rearranging·fo( x, we get
,, . '

,
x = q L
2 ]1/3 '
[(21t e0 mg)
(b) _When one ball is discharged, it will touch the other ball which has charge q on it. Due to
conduction the charge divided equally on both the balls (because both the balls are identical).
Therefore each ball will have the charge q/2. They repel each other, and let they are now have
' .
separation x0 between them.

Electric force·between them is

q2
'tan0- = . 2
.. I 6_1t e 0 x0 mg
:! :; . ,:

or (x0 /-2)/ L 2
l61t e 0 x0 mg

Solving above equation for x0, we get


r-.1------------ ----------------- ------------
-
ELECTROSTATI':5__ ~
L____ - ....... -- -------------· ...... ··-- - -- - - - - - - - -

Ex. 37 Fig. J./25 shows a long nonconducting, massless rod of length L, pivoted at its centre and
balanced with a weight Wat a distancexfrom the leftend.Atthe left end and right end of the rod
are attached small conducting spheres with positive charges q and 2q respectively.A distance/,
directly beneath each of these spheres is a fixed sphere with positive charge Q.
(a) Find the distancex when the rod is horizontal and balanced.
(b) What value should h have so that the rod exerts no vertical force on the bearing when the rod
is horizontal and balance?
L---+ F, = (114ite 0 ) Qq F = (l/4rrE /Oq
' 0 2
112 Rod h
7 +2q

+Q +Q
·---··
(a) (b)
Fig. l./25
Sol. (a) Forces acting on the rod are shown in Fig. I. I 25(b). For the rod to be horizontal ,
L, =O.
Taking moment of the forces acting on the rod about the axis passing through bearing (here
this point is chosen due to the fact that the moment of the reaction at the bearing becomes
zero. We can take any other point about which moments are calculated, provided first the
reaction at the bearing must be calculated). Thus

(_1_) Q;
41t e0
.!:+w.(x-l/2) =
h" 2 _ 41t e 0
(-1-)
(2~q) _!:
1, 2

Solving above equation, we get

X
Qql
= [ 81t Eo Wh 2 +
L]
2
Ans.

(b) The rod exerts no vertical force on the bearing, if LF1, = 0,

I
or ( 47tEo
)Qq
,1+
( 1
47tEo
)(2Qq)
T =W
I I
or h = ~(3qQ/47tEo W). Ans.

Ex. 38 Find the electric field strength and the potential atthe centre ofa hemisphere ofradiusR charged
uniformly with the surface density cr.
Sol. Field : Choose an elemental ring of thickness d8 at an angular position 8 from its bisector. The
charge on this element,
dq = (21tr)ds. <J = (21tr)(Rd8) <J = 21t(Rsin 8)Rd8cr = 2crrrR 2 sin 8d8.
~ . , , EtECTRICI1Y~-J!~~.N
-
__
ET._l_SM
_ _~ - - - - -
The electric field due .to this element ring,
fi1,l , I

The total field


d~ ·~= [ 41tJEO ;;' J
,£ 1,=,'
n/2
f I (RcosfJ)dq
o 41t Eo RJ
'
Substituting the value of dq in above equation, we have

E _ "J'
2
(_1_)
41t Eo
2crnR2 sin0d0(Rcos0)
RJ
0

Alter solving, we get 'E - Fig. 1.126

Potential : Potential due to elemental ring dV


r, ;,

Potential due to hemisphere, V '" nf/2(_1_)2crnR2 sin fJ


41tE 0 R
0 '

aR
After solving, we get V
,, 2Eo

Ex. 39 Two identical particles of mass m carrying a charge q each. Initially one is rest on a smooth
horizontal plane and the other is projected along the plane directly towards the first particle from
,, a large distance, with speed v. Find the distance of closest approach.
Sol. When one particle approaches towards the other, due to the repulsion, the other will also start
moving. In this process the velocity of the first decreases while that of second increases until their
velocities become equal. When thfs happens the distance between the particles become let say r.
Let v be the velocity of each particle ~t this instant, then from conservation of momentum, we have
mu+O mv+mv ... (i)
" 11
or V
2
and from conservation of energy, we have

I 2 I 2 I 2 (l/41tEo)q2
-mu +o -mv +-mv +~--'~- .. ,(ii)
' 2, 2 2 r
SolVing above equations, we get .

r =
$ " --«,._,,,._,_'(. - - · - · - - -
-
' . "/ ,.

Ex. 40 A drop of water of mass 18 x 10-6 kg falls away from the bottom ofa charged conducting sphere
of radius 20 cm, carrying with it charge of 10-9 coulomb and leaving on the sphere a uniformly
distributed charge of2.5 x J0-6 coulomb. What is the speed of the drop after it has fallen 30 cm?

I Eo=l/(4itx9x109 )F/m and g=9.8m/s 2 i

Sol. Taking reference level at B I and using principle of conservation of energy, we have
Mechanical energy at A= Mechanical energy at B

or mg x 0.30 + (1 / 41t E0 )q 1q 2 / 0.20 = 0 + -I mv-' +(I/ 41t Eo )q 1qz /(0.20 + 0.30)


2

where 111 = 18 x I 0-6kg, q 1 = 2.5 x I 0-6 C. , q2 = 10-9 C.


Substituting these values in above equation and solving for v, we get
v=3.65 mis Ans.

~ "

y_,0.20m

( OJOm
IJ~ .... ..I
If,

Fig. 1.127

Ex. 41 A hall ofradiusR carries a positive charge whose vol,ume density depends only on a separation r

from the ball's centre as p = Po ( 1- ~), where Po is a constant. Assuming the permitivities of

the ball and the ·environment to be equal to unity. Find


(a} The magnitude of the electric field strength ai a function of the distance, both inside and
outside the ball;
(b) The maximum intensity £max and the corresponding distance r.

Fig. 1.128
-
Sol. (a) Given that
... ,, ., (!--r)
.·.. , ...
,.. p=po
, R '.
Choose an element drat a distance rfrom the centre of the ball. The charge on this element will
be

l 2
The total ch~rge enclosed in t~e sphere ~f radius r, q = Po ( I - ; ) x (4itr dr)

4itp0 l,: -;:)


Electric field at a point (r < R) is given by
_ _ .!f....
\·£1
4it Eo ,-2

~(1-~)
3Eo 4R

R
2
For r~ R, we have q {Po(1-;)x(41tr dr)

itpoR3
--,-
3
,,
I - q
'E. ---
4it Eo r 2

PoR3
or
12 Eo r 2
·I,

(b) We have E p ,. (1-3r/ 4R)


0 . 3 Eo
For electric field to be maximum,
'
dE
'
0
dr'

or
' ·
0
= p0 (1-6r/4R)
3Eo

ZR
or r = T· Ans.
-
Ex.42 A metal sphere ofradius r I with a charge q is placed atthe centre ofan uncharged thin conducting
shell ofradius 'z- Find electric field and potential at the positions r> r 2 , r 1 <r<r2 and r<r1 in the
following cases.

'' Conducting
wire
(a) (b) (c)
Fig. 1.128 ri' ''' I

Sol. (a) When charged metal sphere is placed at the centre of the
shell, an equal amount of negative charge will appear at the
inner surface and positive charge on the outer surface of the
shell. If A, Band Care the respective positions, then A
Forr>r2 : . o.
EA = [EA]< +[EAlz
I ['q q-q] ( 1 ) q Fig. 1.129
41t Eo ? +? = 41t E0 ?
VA [VA]I +[VAlz

1
4n e
[ q q-q] = (
-;+-,-
I
41t E
) q
-;
0 0
For r 1 <r<r2 :
EB [Eal, +[Eah

-- ?q +o-- (l)q
(I)
41t Eo
-- -
41t Eo ,2

VB fVnl, +fVah
1
( 41t EJ~
For r<r 1 :
Ee [EcJ, + fVd2
0 + 0 =O
Ve [VcJ,+fVd2

( -I- )q
+O- -I- - ( )q
41t e 01 lir, , - 41t e 0 'i
(b) When outer surface of the shell is earthed, its charge becomes zero.
1',
For r> r2 :
EA = [EA]I + [EA]z
I [
47t Eo
q q] = 0
-;:z--;:z
VA [VA]I +[VA]z

_I_[!L_!L]=O Fig. 1.130


41t Eo r r

~-
-
For r 1 <r<r2 :
E
:,. B,,,
= [EB]l +[EBJi

- -I - +q O - ( -I-) - q
41t Eo ,.2 - 41t ea ,.2

VB [VB]l +[Vsh

I [ q q]
41t Eo r r2

For r<r 1 :
Ee O+ 0 =O
Ve = [Ve], +[VcJ2
(

~ - ,: ]
1
41t eJ

(c) When sphere and shell are connected by a conducting wire, the entire charge of the sphere
will flow to the outer surface of the shell, so that potential of both becomes equal.
For r>r2 : 'I
EA = [EA]l + [EA]2

0 +-
41t e
'--'L-(-1-)-'L
,.2 - 41t
0
,.2 e
0

VA [VA], +[VAJi Fig. 1.131

,. e7
I q ( I )q
O+ 41t 0 = 41t 0 e7
EB [Es], +[EBJi
0 + O=O \
VB [VB]l +[VB]2

I q ( I ) q
0 + 41t Eo 'i = 41t Eo r2

For r<r 1 :
Ee [EcJ, +[EcJ2

Ve
0 + O=O
\\
[VcJ,+[VcJ2

O+ ( -I -) - q
41t Eo l'z

1
( 41t eJ! ·
l---~--~----~------·-·-_··:_._____ _c~m~_o_s_:r._t1._TI_cs
E_LE_· ___ J4~·~--
Ex. 43 An uncharged sphere ofradius r 1 is placed at the centre of a conducting shell ofradius r2' having
. charge_q. Find charge on the SP.here i_n the follciwing situations. .•

,. '

(a) (b)
Fig. 1.132
Sol. (a) In this case the potential on the surface of the sphere and shell is equal and so no charge will
• I
appear on the sphere. ·
(b) Let q' charge appears on the sphere. The charges on the two surfaces of the shell are shown
in the figure.
As sphere is earthed, so its potential will be zero. i.e.,
q'
O= 41tlE;.['!' !J
q' Ans. Fig. 1.133

Ex. 44 Fig. 1.134 shows three concentric thin spherical shells A, Band C ofradii a, b, c respectively.
The shells A and Care given charges qand--q respectively and the shellB is earthed. Find the
charges appearing on the surfaces of Band C.

(-q + q')

-q'

(a) (b)
Fig. 1.134
Sol. The inner surface of shell B will have charge ~q. Let outer surface of it has chargeq'. The charges
on the surfaces of shell Care shown in figure (b). As shell B is earthed, so its potential will be zero.
Thus

0 -'
41t
-[!L+ q'-q +(-q+q'-q')]
Eo. b b C

.b
After solving, we get q' .q- Ans.
c

Thus charge on outer surface of shell B is qb . Charge on the inner surface of shell C is - qb and
C C

. . . ( qb)
on outer surface -q + ~- .
-
Ex. 45 A dielectric cylinder ofradius a is infinitely long. It is non-uniformly charged such that volume
charge density p varies directly as the jistance from the cylinder. Calculate the electric field
intensities due to it, if p is zero at the axis and is p, on the surface. Also calculate the potential
difference between the axis and the surface.
Sol. Consider a co-axial shell ofradiusxand thickness "
dx and oflength e. The charge on this shell I. "\ 1 dx '\
JX lf
dq = p(2ru:C)dr . - ".I. -·······-············ ...
~
As p varies directly as the distance.,·, hence P
\: /
can be written as p = ex, where c is a constant (
given by the relation '
P, ca Fig. 1.135

C
a
The intensity of electric field at any point Pat a distance r from the axis of the cylinder can be
obtained as : By Gauss law

Ex 2rrre
.!!....
Eo
/
I , •
_r-,,dq
Eo J.'

1
J
. - p(2ru:f)dr
Eo

1 a

. = -J (cx)2ru:fdx
Eo O ,

After putting the value of P, and simplifying, we get


.2
P sl
E = - -
3 Eo a

At, r =a, E = Psa . The field in all the cases is radially outward.
3 Eo
We know that
A
-f E.di'
B

op,,2 pa
vaxis - vsurface -J-s-dr = __
s r 2dr
3E0 a 3aE 0
_J
a 0

Ans.
-
- X
7i. Find the charge contained inside a cubical
A ....

Ex. 46 The electric field in a region is given by E = E 0

volume bounded by the surfacesx=O,x= ey=O,y= e, z=O and z=f.


y
Sol. Atx= O,E=O and x= e, E= £ 0i.
The direction of the field is along the x-axis, so it will
cross theyz-face of the cube. The flux of this field

<I> <i>Ieft face + <!>right face

By Gauss's law,

q
Fig. 1.136
Eo

q Ans.

Ex. 4 7 Fig. 1.137 shows two parallel nonconducting rings arranged with their central axes along a
common line. Ring I has uniform charge q 1 and radius R; ring 2 has uniform charge q 2 and the
same radius R, rings are separated by a distance 3R. The net electric field at point P on the
q,
common line, at distance R from ring I is zero. What is the ratio - ?
q2
qi q~

Sol.
fr~r:=0 Fig. 1.137
The electric field due to the ring at a distance x from the centre is given by

E = ( 41tlEJ(R2+q:2i312
Thus electric field at P due to ring 1

1 1
£1 ( 4it Eo) (R2 ::2)3/2 - ( 4it Eo) z3I~ R2

Electric field at P due to the ring 2

I ) 2q2
( 41t Eo R2 312 ·
5
For the net field at P to be zero

or I ) 2q,
( 4!t1Eo) R2:IJ/2 ( 41tEo R253l2·
which on simplification gives
!ll_ 2x2J/2 25/2
C/2 5312 5312

0.506. Ans.

Ex. 48 A very thin disc is uniformly charged with surface density cr > 0. Find the electric field intensity
Eon the axis of this disc at the point from which the disc is such at an angle a.
Sol. The electric field due to a small element of disc of charge cr dA is

dE = _l_crdA
41t Eo ,-2
The effective field at this point Pis dEcos a. The total field

Ez = fdEcosa
J
crdA
- I- --cosa.
4!t Eo ,-2

dAcosa 0 (J

Here f ,.
2 is the total solid angle which a. Thus

- I-) crQ Ans. Fig. J.138


( 41tEo '

Ex. 49 Find the potential ofanuncharged conducting sphere provided that a point charge q is located at
a distance r from its centre.
~
Sol. ~tia1J:'. i§J'1:_ same for all e_oints of the s~e.
Thus-w0-can.calculate~iis3,_alu.e..at.tbe centre-9 q ,.
of the spJ!er:.e~because on]y_(orJhis poinLiLcl!!L
b~ulated in the most simJJ~iay: ~ l,U
--1 q ' /,A ;J-
L V = -4 - - + V
___ __2'..§o 1:_ e~ ·'/~ ... (i)
"
Fio. J./39

where first term is the potential due to point charge q and V' is the potential due to induced charge
on the surface of the sphere. But entire induced charges are at equal distance a from the point O and
the total induced charge is equal to zero, and so V' = 0. Thus required potential

V = _ _ J_ Ans.
41tEo r

.,
-
'L-----,.1.,;;._ _ _ _ ___,".,s.;. _ __
, -·-, ,-------
', ' :
---~---
·-----·----
- --------- ,,u
. ELECffiOSTAncs
- ---
Ex. 50 A point charge q is within an electrically neutral shell whose outer surface has spherical shape
(see fig. 1.140). Find the potential at the point Plying outside the shell at a distance r from the
centre O of the outer surface.

Fig. 1.140
Sol. The field at the point P is determined only due to
+
induced charges on the outer spherical surface, The
field of the point charge q and of the induced charges + +
q
on the inner surface of the sphere is equal to zero
everywhere outside the cavity. Also. the charge on the + r I'
outer surface of the shell is distributed uniformly and
hence potential at P

q + +
V --- +
41t Eo I' .

Ans. Fig. 1.141

Ex. 51 A point charge q is at a distance r from the centre O of an uncharged spherical conducting layer,
whose inner and outer radii are equal to a and b respectively. Find the potential at the point O if
r<a.
Sol. Equal amount of charges will induce on the inner and outer surface of the conductor. The each
negative charge is at an equal distance a from 0, and so potential due to this charge at 0,

V = _ I _(-q)
- 41t Eo a
Similarly due to positive charge, potential at 0

__ !!..
41tEo b
+ +
Potential due to charge q at 0 +
__ !!.. Fig. 1.142
41tEo r
Thus total potential
V V0 + V_ + V+

47tqEo [~-~+¾]. Ans.

.
-
~---- . ----·-··cc·--· ---------------------
--- --
. EI:!ECTRIC/1¥ & MAGNETISM -- -.
----'-----'-'--·---~--------~·.···--·----~ ··----- -···-·-. -' J
Ex. 52 A system consists of two concentric spheres,the inner sphere ofradius R 1 having a charge q;.
What charge q 2 must be placed onto the outer sphere ofra_dius R 2 to make tlie potential of the
inner sphere equal to zero. What will be the dependence of potential V on the distance r from the
centre of the system? Plot schematically the graph of this dependence, assuming that q < 0.
1
Sol. The potential at the surface of outer sphere

V
I (q' + q2 J
47tEo R1 R2)
It is given that V 0

which on solving gives

q2 = -(;~)q, Ans.

The potential at a distance r from the centre of the system can be written as
I q, +q2
VI = -- - - - ·for r>R
47t Eo r ' 2

4rr Eo
q,-( )q, *
r

_q,
47tEo
['_-;2]
__I
,.
and Vz
_I [.'l!.+g2]
47tEo r R2 ; forr<R 1

VI

,!----+-~>-------•r
R R 1 1
Fig. 1.143 Variation of V with r.

.
-
Ex. 53 A positive charge q is placed in front ofa conducting solid cube at a distance dfrorn its centre. Find
the electric field at the centre of the cube due to the charges appearing on its surface.
Sol. Charge will induce on the surface of the cube </ 'I
due to the charge q. The net electric field at the
+·~------~-
centre of the cube due to all the charges must be
zero. Let £ 1 be the electric field due to the +
charges appearing on the surface of the cube
and if £ 2 is the electric field due to charge q,
+
then
0
+L-----!------'-
d
or
m £, ~ Fig. 1.144
The electric field due to charge q at the centre of the cube.

£2 = ( 47t!EJ :2

£, = ( 47tlEJ :2 · A11s.

Ex. 54 Calculate the flux of electric field E through (i) each of the bases, (ii) the curved surface of a right
~ylindrical closed surface of radius a and length£, due to the charge q situated at its geometrical
centre.

-----(
Fig. 1.145
Sol. Take an element ring of radius rand thickness dr in the right face. of the cylinder (see Fig. I. 145).
The electric field at any point Pon an annular ring is

E - ( I ) I along OP
41t Eo (,. 2 + e:)
The electric flux of this field through the element
d<p £cos ex x 21tr dr

,.
_q_x
41tEo M .
(f/ 2) X21trdr

qCrdr
2)3/2
(
4Eo ,.2 +~
-
. Total
,-. '' flux
-
through the right face of the cylinder
. '.

a qfrdr
~right J 2 )312
0 t
4Eo ( r2+4
2
112
~
q [I f/ ]
= (a2+ ~)
By symmetry the flux through left face of the cylinder will also be same.
Total flux through the cylinder
q
<I>
Eo

!L
or <!>curved + <1>,ighl + <l>1eft Eo

or <!>curved + 2<!>,ight !L
Eo

!L - !L [ I f/2 ]
2
<l>~urved Eo Eo (a 2 + £ I 4)" 2

qf
,, 112 • Ans.
2
2 Eo ( a + ~)

•I
Ex. 55 Determine the law of variation of potential wifh distance in an electric fleld produced between two
parallel infinite plates, bearing equal and opposite charges. · + --+ -
Sol. The electric field between such plates is
+--+ -
constant. If E is the electric field and d is the F
separation between the plates, then potential + --+ -
difference between them
+--+ -
A lJ
... (i) Fig. 1./46
If the potential of the negatively charged plate Bis assumed zero, then
VA-0 = Ed
or ~ = Ed
The potential difference between plate A and an arbitrary point at a distance x from the plate A is
given by
VA,. ... (ii)
From equations (i) and (ii), we get

VX Ans.
Ex. 56 If the electric potential in a region is represented as V= 2x+ 3y~4z, obtain expression for the
electric field strength.
Sol. Given V 2x+3y.-4z

av a
- - =--(2x+3y-4z)=-2
ox OX
av a
- - = --(2x+3y-4z)=-3
~\' ay ay

and £_
av a
- - = --(2x+3y-4z) = 4
oz ilz

Thus E £)+£.,,)+£/.

-2i-3)+4k. Ans.

Ex. 57 Calculate the potential due to a thin charged rod oflength L atthe point along and perpendicular
to its length.
Sol. Suppose a thin rod has a uniform linear charge density A. To find potential at point P, let us divide
the rod into a series of small elements of thickness dx each. Consider such an element at a distance
x from 0. The charge on the element
dq = ').. dr
The potential due to the element at point P

I )Mr
dV = ( 47tEo -,.-

The potential due to the entire rod.

_I-)(L+a) Mr
V ( 47t Eo f
a
r
b r

dq
0:···· t___ _ _ _ _ _ _ _ _ _ __J
.:-----:-
'
X ---l
:-a_,_;______
dx !. _ _ _ _ __,

Fig. 1.147

·_,.
l 47t Eo
2
)en[(l+a)+~b +(L+a) ]
a+ Jb2 +a 2
2
Ans.
- ,--..-. -··1
',·;;,'

. ,_____ :~t!£.:J
Ex. 58 A non-conducting disc ofradius a and uniform surface charge density <J is placed on the ground,

.
"
with its axis vertical. A particle of mass m and positive charge q is dropped, along the axis of the

disc, from height H with zero initial velocity. The particle has !L =
4
1t Eo g
m a
(a) Find the value of H if the particle just reaches the disc
(b) Sketch the potential energy of the particle as a function ofits height and find its equilibrium
position.
Sol. The potential of point Pat a distance H from the centre of the disc is given by:

V .J2 +H 2-H]
= -<J[ a
P 2 Eo
At the centre of the disc, H = 0

(a) · For the partide released at P:


Vo
aa
2Eo c Fig. 1.148
o•~-~a-.:)_...,
As initial and final KE of the particle are zero, so by conservation of mechanical energy, we
have
Decrcase'in gravitational P.E. = ltici"ease in electric P.E.
or mgH [V0 - VP] q
aq I , 2
or mgH -[a-(,ia-+H -H)]
2 Eo

or gH !L(___5!___)[(a+ H)-~a 2 +H 2 ]
m 2Eo

q
Given
m

gH

H
or
2
H
or a+-
2

or

2
2 H
or a +-+aH
4

4
H -a Ans.
3
-
(b) Potential energy of the particle at a height His given by
U Gravitational P.E. + Electrostatic P.E.
MgH+qV

mgH+ crq [Ja 2 +H 2 -H] ... (i) [V= V)


2 Eo

For equilibrium, F = - dU =0
dH

or

or

4E 0 g q crq
As cr m or 2Eo =2mg

2H
or I+~=== 2 0
Ja2+H2

2H
or

or

or
or

a
From equation (i) at H
.J3
a crq. [
mg-+--
.J3 2 Eo

Also atH=O, U=2mga


-
ELECTRICITY-l;;&·.·MAGNEIISM
The potential energy verses height graph is shown in figure
u
2mga
I
vJmga

,-/----',,-----fl
alvJ
Fig. 1.149
Ex. 59 There are two large metallic plates S 1 and S 2 carrying surface charge densities cr 1 and cr2
respectively (cr 1 >cr2) placed at a.distance dapart in vacuum. Find the work done by the electric
field in moving a point charge q distant a (a< d) from S 1 towardsS2 along a line making an angle
7t/4 with the normal to the plates.
Sol. If £ 1 and £ 2 are the respective fields produced by the plates, then net field between the plates
E £ 1 -£2 cr 1 cr0

Force on the point charge


'
'
Work done by the electric field
.
--£,- E I

Fig. 1.150
s2
I w FscosB
I.
11

Ans.
.9~1 •. i'· '

Ex. 60 Two point charges q and-q are separated by distance U. Find the flux of the electric field strength
' • 1h
vector across a circle Of radius R.
Sol. Consider a ring element of thickness drat a distance r from centre.
The flux across this elelllent
\ '
d<j> = E x 21trdr
Total flux l
,-~, R 'q$•---'' ~q
<I> fEx21trdr ;
/ 0

p' _,,( __::;i;c....._ e ---\


i+---
where£= and P=qxU.
4it Eo (r2 + £2.)3/2
Substituting these value in above expression, We have Fig. I.ISi
Substituting, r2 + c2 =z
also 2rdr= dz

p JR -3/2d.
q, -- z z
4Eo 0

-1/2 R
p z

4Eo
2 o

Ans.

Ex. 61 A point<:harge q is isolated at the centre O of a sphere uncharged conducting layer provided with
a small orifice. The inside and outside radii of the layer are equal to a and b respectively. What
amount of work has to be performed to slowly transfer the charge q from the point O through the
orifice and into infinity.
Sol. Because of charge q, the charge will induce over the ~onductor as shown in figure.
The potential at the centre +

V = q (1 aI)
41t Eo b +
Potential difference +
!J.V v_-v
0--q-( _!_ _ _!_)
41t Eo b a +

q (1 I)
41t Eo a b
Fig. 1.152
-
Work done in transferring
,, ' \ the charge dq, from centre to infinity
.
dW (i!.V)dq

Total work W J(i!.V)dq


I I )q
-4itI ( ---
a b
Eo
fqdq
0

=
q2 (
Sit Eo
I I)
-;;-b . Ans.
Charged soap bubble
We know that the excess pressure in a soap bubble of radius r due to surface tension Tis given by
4T
p = -
o r
If now bubble is given a charge q,, then the outward mechanical pressure

,,
~ . 2cr:o = 2 ~o ( 4:r 2 r
', 4T
(P; + 2cr:o )-Po
For the equilibrium of the ~ubble
r
r·- -.... -··- .4T crz- 4T -- - q2--.. ·-··1
Ic__ ___________ r. 32it2 E r4___ JI
P;-Po ,=P=-;-, 2E0 =---""'-·--o_, ... (I)

Thus decrease in pressure due to charge on the bubble Fig. 1.153

-dP = 2 4 ... (2)


" 32it Eo r
_We see that effectiye value of pressure decreases therefore bubble,will expand. By Boyle's Taw
' • • • • :~ •• • P V = ·constant· , ' ,
1

or · Constant
,,
or constant ... (3)
Differentiating equation (3), we get
dP 3dr
p r

or dr i(-:) ... (4)

r
3x
( '
q2
)z__,
32it 2 Eo ,_-4
I
/
~

or
q2 ' ... (5)
96it2 eo ,-'Jp ~,
'
~L
__~___________________ :fg
E_,LE_.·.·~c_m~··_os_r._,i_n_cs
__
Special case : when net excess pressure is to be zero, from equation ( 1) we have

4T cr 2
o· ~ ·---- .
:·, '. , , r 2E ' ·
0
- ----- -- -~·--- •----;

which gives : cr JSTrEo . ·

Ex. 62 A conducting spherical bubble of radius a and thicknesst(t<<a) is charged to a potential V. Now
it collapses to form a spherical droplet. Find the potential of the droplet.
Sol. As mass and charge of the bubble remain conserved, and ifr be the radius of the resulting drop, so

(1rrr
3
)p
or
The potential of the bubble

V
__ '}_
4rr Eo a'
. which gives q 4rrEo aV
Now potential of the resulting drop

V
__ '}_
4rr Eo r
4rr Eo aV
4rr Eo (3a21)l/3

(:if 3 V, Ans.

Ex. 63 The minimum strength of a uniform electric field which can tear a conducting uncharged thin-
walled sphere into two parts is known to beEO' Determine the minimum electric field strength £ 1
required to tear the sphere of twice as large radius if the thickness ofits walls is the same as in
the former case.
Sol. If cr is the surface charge density on the sphere and R its radius, then for tearing of the sphere

cr = JSTREo

or 0 2 R = constant.
Here Tis the rupture strength of the sphere.
Since E oc cr,
E2 R = constant
For the sphere of twice the radius, we have
EJR = Ei(2R),
which on simplification g~ves

Ans.
Ex.64 Two small identic;il balls lying on a horizontal plane are connected by a weightless spring: One
ball is fixed at poinfO and the other is free. The balls are charged identically, as a result of which
the spring length increases two fold. Determine the change in the frequency of harmonic vibrations
of the system. ze ~
Sol. If f be the length of the spring, then extension of the ··,vvs')(';li"itii<Y?:,,;W:;vl:"'?:
spring becomes 2£. '..r.,v-,_....... •..,..,'l,,M.~... ~1•Jo.,.,..rJ
p
Let at any instant free ball is displaced from the mean
position by a distance x which is small comparison to f. • . q
The total mechanical energy of the system Ttlr,:IG,'W,';(.,J~'
1 2 I q2 I 2 · _}-- .r --.j
E -k(f-x) + ~'---+-mv Fig. 1.154
2 41t e 0 (2£-x) 2

I · 2 1 2
-k(f-x) +--q (2f-x) -I +-mv
I 2
2 .41t Eo 2

• 1 I
-k(t-x/+--q2
2
}-I]
[ (2£)- 1{ I-.!_
41t Eo 2£
I
+-mv2
2

1 2+i[I{
2
-- -
-k(f-x)
Eo
x}]I 2
. 41t 2£
I+-

+-mv
2

. I 2q lxl2
-k(t~x). + -2- [ - + -
2
]
+-mv ... (i)
2 , . ., 41t Eo 2£ 4£ 2
At the mean position .

"
kl = ... (ii)

On solving equations (i) and (ii) and simplifying, we get

E = ~kt 2 + kx 2 +.!..mv2 ... (iii)


2 2
dE
For free oscillations, 0
dt

or 5 2 +kx 2 +-mv
-d [ -kf I 2] =
0
dt 2 2

dx m dv
or 2kx-+-x2vx- 0
dt 2 dt
dv 2k
or - --x
dt m
{2k
On comparing with standard equation ofSHM, a=- oi-x, w~ get Ol =\j-;; .The frequency without

charge Olo = ff . ~hus oo = ..fi.00o .


1.22 ELECTRIC POTENTIAL ENERGY OF-SYSTEM OF CHARGES
: i .- • ·t, ,Ir;
For .the calculation of potential energy of the system of charged partic)e~, ,tile method is to compute the
potential energy for each pair of charges separately and then add these algeqrnically. Hepce for n-charge
particle system,

u
_l_ L q;qj
4 1t Eo all pairs ry

or u 1( 1 )f f q;qj
- - - £..,£.., _ _ . , (·~
I r }")'
'
2 41t Eo ~J=l j=l ry ' [

In the expression 2I is introduced as each pair is counted twice. For exa_mple, the energy contribution due to
charge q 1 and q 2 is required once only when i = l and}= 2. But in this expression this term comes twice; once
when i = l ,j = 2 and other when i = 2,j = I. For n - particle system, there are N-pairs, where

I
N -n(n-1)
2

Ex. 65 A positively charged sphere ofmassm =5 kg is attached by a spring of force constant k= 104 Ni
m. The sphere is tied with a thread so that the spring is in its natural length. Another identical,
negatively charged sphere is fixed to the floor, vertically below the positively charged sphere as
shown in jig. 1.155. Initial separation between sphere isx0 = O.SO·m. Now thread is burnt and
maximum elongation of the spring is O.IO m, calculate the charge on each sphere (g= 10 m/s 2).

Sol.

T ··--z
Xo
+
_J______ +

~
\'
.,

Fig. 1.155
In conservative fields the total mechanical energy of the system remains constant. Thus
Mechanical energy at initial position = Mechanical energy at final position
If x be the maximum elongation of the spring and q be the charge on each sphere, then
_1_q(-q) +mgxo +O I q(-q) l 2
----'-'-----'-'----+mg(x0 -x)+-/a
41t Eo x0 41t ~o (x0 -x) 2

or mgx+--q
41t Eo
I 2[ I I] ----
x 0 -x x0
Given,x0 =0.50 m,x=O.l mandk= 104 N/m

4 9 1
½x10 x(0.1)2 5x!Ox0.1+9><10 xq2[(o. _ _) \]
50 0 1 0
'After simplifying, we get
q Ans.

Ex. 66 Find the energy of the system of charges in the form ofan infinity plane grid shown in jig. 1.156.
Sol. The easiest of finding the potential energy is to first select any charge of the system and calculate
,, its potential energy with each of the other charges. Consider a system ofcharges each of magnitude
q shown in figure. If a be the centre.

Fig. 1./56
to centre distance between the charges, then the energy of any charge with its neighbouring one is
2
1
[ lt E - : ]. Let us first sum up the terms from the charges along a line. This sum is due to the
4 0

charges on the left and the right of any charge. For a positive charge (say P) it is given by

2
Two sides of P
L[_
41t Eo a
·
I +-1 - .......~]
_l_+-' __
2a 3a 4a

- 2 q 2 ln2 = -1.386 ( q 2 )
41t Eo a 41t Eo a
Now consider the next adjacent line ofcharges above (or below). The nearest is negative charge Q,
at a distance a. There are two positive charges at R a~d R' at a distance fi a each. The next pair
is at a distance ;Is a , the next is at a distance Fifi a , -Ju a and so on. The potentiai energy of all
the charges on the lines ; one below the charge P and other above the charge P,

2 2
U2= Two lines X Two sides
one above and of P; one right
X
q2[ 1 1 1
471 Eo a -I+ Ji- ,J5 + ,JIO -
]
......=
one below P and one left of P

=-2.54( q2 )
471 e 0 a
Thus total energy of system of charges
U=U 1 +U2

=-3.92( 471q2e 0 a ) Ans.

Ex. 67 Eight point charges are placed atthe corners of a cube of edge a as shown injig. J.157. Find the
work done in dissembling this system of charges.
'I

-q q
a

- \\a
_?-•.... ·······\·· ............. .
.....- na'·... '-
q
..······ ·-.....:\
Fig. 1.157
Sol. The number of charge-pairs
n(n -1) 8(8-1) =28
2 2
Out of these 28 pairs ;
(i) 12 pairs are of opposite charges at separation a each, with total energy

U. = 12[-l_ q(-q)]
1 471 e 0 a

(ii) 12 pairs of similar charges at separation fi a each, with total energy

U. - 12 - I- -qq -]
2 - [ 471e0 fia
(iii) 4 pairs of oppo~ite c_harges at separation ..fj a , with total energy

I q(-q)]
U3 = 4 [ 4it Eo ,ffa ·.
Thus total potential energy of the system of charges
U = U 1 +U2 +U3

_i_i_[-1z+_l2_ __!_]
4it Eo a ..fi. ..fi

-5.83 [ -
1
_i_]
4itE0 a

' [
Thus work done in dissembiling the charge=+ 5.83 41t Eo-;;
I q2] Ans.
,.
Ex. 68 Four point charges+ 8µC, -lµC, -IµC and the+ 8µC are fixed atthe points -.J27/2 m , -Mm ,
+Mm and ,)2112 m respectively on they-axis.A particle of mass 6 x to-4 kg and a charge+
0.1 µC moves along the-xdirection. Its speed at x =+= is v0• Find the least value of v0 for which
the particle will cross the origin. Find also the kinetic energy of the particle atthe origin. Assume
that space is gravity free. Given l/4p Eo= 9xto9 Nm 2 /C 2
Sol. Suppose q 1 = 8 µC, q2 = I µCand q0 =O.lµC. Because of the symmetric positions of the charges, the
force on the charge q0 will be alongx-direction.
If P be a point at a distance x where net force is zero, then
·,
y

or
-q2

or

3 )3/2 -q, B
( -+x
2
or 8 27 2 )3/2 q, A
( -+x
2

After solving, we get X = as m Fig. J.158


If charge q 0 crosses the point P, then it will move by attractive force of the particle towards the
origin, so its speed should be infinitely small at point P.
Thus by conservation of mechanical energy, we have
[K.E. + P.E.]at,nfinity ;;,, [K.E. + P.E.Ja,P

or

After solving, we get

vJ ; , 9
or ( v0)min 3 mis Ans.
Let K is the K.E. of the charge q0 at the origin, then

K +2x-1-[(8X !0-6)X (0.1 xl0- )


6 6
(lx!0- )x(O.Ix10-6)] = ['-mv 2 +
0
o]
41tEo -./27i2 -./312 2
Substituting v0 = 3 mis and simplifying, we get
K = !7.2xJQ-4J Ans.

Ex. 69 Two plane parallel conducting plates 1.5 x H,2 m apart are held horizontal one above the other in
air. The upper plate is maintained as positive potential of 1.5 k volt while the other plate is earthed.
Calculate the number of electrons which must be attached to a small oil drop of mass
4.9 x 10-15 kg between the plates to maintain it at rest, assuming that the density of air is
negligible in comparison with that of oil. If the potential of above plate is suddenly changed to
-1.5 k volt. What is the initial acceleration of the charged drop? Also obtain the terminal velocity
ufthe drop if its radius is 5.0 x 10--<i m and the coefficient of viscosity of air is 1.8 x 10-5 N-sim2
Sol. For the equilibrium, the drop must have negative charge q, and so ~ - - - - - - - -
qE = mg ... (i) I+ + ½,+Eq+ +

mg ·· d
or
q ~ ~j___·____
"_'g_____
V l.5x10 3
Here E -= 105 Vim Fig. 1.159
d i.5 X10-2
m 4.9 X 10-15 kg

(4.9x10- 15 )x(9.8)
q
105
4.8 X JQ-i 9 C
If n is the number of electrons on the oil drop, the

!!_= 4.8x10- 19
n =
e l.6x10- 19
3. Ans.
When polarity of plates is reversed, the electric force also acts in downward direction, and so net
force ,
F mg+qE ... (iQ
From equations (i) and (ii), we get
F 2mg
Initial acceleration of the drop
F 2mg 2
a = - = - - = 2g mis downward Ans.
m m
As drop acceJerates in downward direction, its velocity increases and hence viscous force increases
in upward direction. At certain moment the net force on the drop becomes zero and thereafter drop
will move with constant velocity. Ifv is the terminal velocity, then
2mg = 67TTJrv

V =

,, (4.9xI0- 15 )x9.8
· 3itx(I.8xl0-5 )x(5.0xl0-6)
5.7 x 10-5 m1s Ans.

Ex. 70 Small identical balls with equal charges are fixed at the vertices ofa right 1977 -gon with side a.
"' At a certain instant, one ofihe balls is released, and a sufficiently long time interval later, the ball
adjacent to the first released ball is freed. The kinetic energies of the released balls are found to
differs by Kat a sufficiently long distance from the polygon, Determine the charge q of each ball.
Sol. After a very long time, the potential energy stored in the system will give the kinetic energy to the
first released ball, and so ,,
,,
... (i)

where a 1, a 2, •...••• aN- I are the distances from the first ball (before it was released) to the remaining
balls in the circle, a 1 and aN- I being the distances to the nearest neighbours, i.e, a 1 = aN- l = a.
HereN= 1977.
Similar treatment can be made for second ball. Neglecting the influence of the first released ball,
then we have

I 2
-mv2 ... (ii)
2
From equations (i) and (ii), wehave

1 2 I 2 q2
-mv1 --mv2 =
2 2 4ne0 a,

q2
or K
4it Eo a

or q J41tEo aK Ans.
·. EiEqil9STAT1CS •till
cc.__ _ _ _;_;_;.,~---------"------==="----;~
_1.23 ELECTRIC DIPOLE
A system of two equal and opposite charges fixed at a small distance constitutes a diP,ole. If f is the distance
between the charges+ q and-q, then dipole moment is defined as: q, ----------l q
I
p
- _-
=- - - -q,___ -'
,,-----, ~ ,·P ___T+
It is a vector quantity and its direction is from negative to positive charge.
Potential due toan electric dipole: Consider a dipole AB of dipole moment, P = qf.. Fig. 1.160
We want to calculate the electric potential at a point P, at a distance r from the centre of the dipole. Let liner
makes an angle 8 with the line of dipole.

Fig. 1.161
The electric potential due to the charges of the dipole at P
1
VP = ( 4n EJ[;B - ;A]
f f
From the figure, PA= r+-cos8 and PB= r--cos8, assuming thatf <<r.
2 2

I ) qfcos8
(
4n Eo [,2 - ~ cos2 8]
-
C 2 '
As f << r, hence .£._cos 2 0 can be neglected and putting qf = P, we get
4

_ I)Pease
( 4it Eo . r2 ... (I)

In vector notation it can be written as :

Electric field due to an electric dipole : The electric field at point P varies with rand 0 both,
so we can not get E from the differentiation of V at once. The components of E in two perpendicular
directions are ; the radial component Er, and transverse component £ 8. Thus

E, cs - ~~ = -:r [ 1
4it e
0
P:~se] ·

_ I )2Pcos0
( 4it Eo r3 ... (i)

and
_, av __ 1 a [
r ae r ae 4~ Eo
~Pcos8]
r2

_ I)Psin8
( 4it Eo r3
... (ii)

,, 2
- '-)P~3cos 8+1·
or E ( 4it Eo ... (2)
_r3 ·

Dividing equation (ii) by (i), we get


tan e
tan ex
2
-.: . - . '.¼T
or
t
a tan-I (tan
2·· :
8)'; ... (3)
·- --- -------
Here a is the angle made by the resultant field E with the line of r . The direction of E from the direction of
dipolemoment is e + a.
·1 cases. :
Spec1a " G " p
I. End-on position: At the axis of the dipole, e= 0, and so -f----0;,~-----10 E
r
E
'I
( 4it Eo
)2P Fig. J.i62
r3

and V
---------·-----·__ -~----E~LE_-~c__rB~o_s_t_"-_n
___ cs::~_-ffl
2. Broad side-on position: At the equatorial line of the dipole, 0 = 90° and so

(4~1eJ;
p
7!
£ r
and V 0 q q
+
p
Fig. 1.163

Ex.71 Find the position, where the electric field due to an electric dipole is perpendicul~r to the dipole
moment
Sol. From the figure
E
e+a 90°
a 90°,--0
tan e
We know that tan a
2
tan 0 q e q
or tan(90.:.. 0) · =
2 )----'-..:..I.---{+

tan 0
or cote Fig. 1.164
2
or tane .J2
e tan- 1(-h) Ans.

Electric field and potential due to an electric dipole: Second method


1. End-on position: - - - - - - ,- + C/2------~
:,-r--e/2~
0-----,---0
c/2 ,12
- - - - r·-----..;
Fig. 1.165
The electric field at any point P, at a distance r from the centre of the dipole is the vector sum of the field
produced by both the charges. Thus

2qCr

2Pr
As f << r,

E 1 )2P
( 4rre0 7·
Electric potential: V
_l
4rr E_o r-f/2
[-q __
q] r+f/2

As f<<r,

l ) p
V = ( 4rre 0 ?·
2. Broadside-on position :
The electric field at point Pis the vector sum of the fields produced by the two charges. As the distance
of the point Pis equal for both the charges, and so magnitude of field produced by each charge

E'
/
/; r
/
/
q C/2 q
Fig. 1,166
l q
E
4rr Eo (r +£2 14)
2

The resultant field at the point P


E 2E'sin8

l qf
4rr e 0 ( 2 312
r +f 2 /4 )
p
2
2 f 2
For f << r, r +
4 =r , and so

E = ( 4n!EJ; ·
3. At any position:
Resolve the given dipole moment along OP and perpendicular :
to it. These are Pcos0 and Psin0 and ifE 1 and E2 are the r
respective electric fields produced by them, then
Pcos0
2Pcos0 q
E1 ... (i) ·········-... q
4nE 0 ,-3
,----7'<:-'---._.h-p,,---\+
Psin0 . ...-··
and E2 - - -,.3- . . . (ii) Psin0
4nE 0
Fig. 1.167

At end-on position the angle between P and E is 0°, while at broadside -on position the angle
between P and £ is 180°.
2
/£2 £2 _ I P~3cos 0 + I
Squaring (i) and (ii) and adding, we get£ \/ I + 2 -
4nE 0 r3
1.24 A DIPOLE IN AN ELECTRIC FIELD

Consider an electric dipole placed in an uniform_ field of intensity i:. The ends of the dipole experience equal
and opposite forces, each of magnitude F= Eq.

(It)-·--+J!.°q
(

------+-~-~----+E
i
~(sinO
liq <t---<:--:,·- . ............. ,i,

Fig. 1.168
Thus, because ofuniform field, the net force on the dipole becomes zero and so the centre of mass of the dipole
does not move. However, the forces on the charged ends do produce a net torque ~ on the dipole about its .
centre of mass. The magnitude of this torque
~ = [magnitude of either force] x [ distance between lines of
action of the forces]
Fxtsin0
£qxfsin0 = £(qf)sin0
. or ~ = P£sin0
In vector notation, it can be written as :
... (I)

t
'. ' -

Fig. 1.169
Work done by the agent to increase the angle from 8 1 to 82 :
The torque exerted by the agent to increase the angle

'tag~nt = PE sin 9

Work done, Wagent

.e,'
JPEsin9d9
e,

-- . - - - ... ,.
or '· W.sent = PE(cosfl1 ...:cos82) ... (2)
t-·~-··· '. -____ ,_ .-p..: ' ·~- ..... . ••• :..'

Potential energy of an electric 'dipole : We know that the change in P.E. of the dipole in an
electric field is equal to the work done by the agent, and so
"' u_ := w,genl'
In any situation involving P.E., we are free to 'choose the zero potential energy configuration.
Thus ifwe choose the potential energy to be zero,when the angle 9 = 90°. Therefore
ur~ uj w,gen,
= PE(cos91 -cos92 )

For .9 = 90°·., U; = 0 ; 0 2 = 0 and UI = U ':


1 -,·-:-; ...., 1, ·:; "' f ,. " l

, =
U
~- --- '_..
;
·-PBcos'a·'
:. ~---~ ---~·-· - '
... (3)
. u·
, J
.p~ ... ZI\
p, !..... . . ,. . . . . . s:::
Fig. 1.170 v~riation of U with Q
In vector notation it can be written as

u = -P.E
1.25 FORCE BETWEEN TWO SHORT DIPOLES
I. When dipoles are placed parallel to each other:

Suppose dipoles are placed at a separation r. The P.E. of dipole P2 in the field of dipole ij

,,tr--,~,.,
is

U = -PzE1 cosl80°

E2 l:"1
The force between them is given by . , -1 Fig. 1.171
,ulf:>':1

F
-d&,°"'-...,j_[_l_flP2]
3
= (-l-)3flPz
4
dr dr 4n Eo ,. 41t Eo ,.

As the force is positive, so it is of repulsive nature.


2. When dipoles are placed on the same axis :

Suppose the dipoles are placed at a separation r. The P.E. of the dipole P2 in the field of ij is

U = -P2E 1cos0°
pl 1'2
--=t--·····-----------~-
-(-1-) 2flP2 E, E1
1 4 - ,.____.,
47C Eo /'3

The force between them is given by Fig. 1.172


''
F
'
dU __ .!!_[__1_2flP2]
- dr --:.,.,dr
,. 41tEo ,.3

-(_1_\6flP2
47t Eo') /'4

Here minus sign of F indicates attractive force.

Equilibrium of a dipole in electric field:


When a dipole is placed along the direction of electric field and disturb slightly from this position, it will return
towards initial position. Such a configuration of the dipole is corresponding to stable equilibrium.

0=0° l:
(a) Stable equilibrium
.
-
tl = 180°.
'· (b) Unstable equilibrium ;
p

I;

Fig. I. 1"1,3.
When a dipole is placed anti parallel to the electric field and disturb slightly from this position, it will align in the
direction of field. Such a configuration of the dipole is correspo~ding to unstable equilibrium.
Oscillations of dipole in electric .field :
E ai Sl}J~il angle e with
., " I '

Consider a dipole p placed in uniform field


the direction of E . The electric field exerts a restoring torque~ on
the dipole. Due to whic~ dipole comes to its<meaniposition, and
crosses it due to inertia and repeat this motion again and again.
Thus
Eq
~ -PE sine
For small angle, sin e = e ' Fig. 1.174
~ PE(-0)

~ PE
and angular acceleration a = -=-(-0)
I I
- , I , -
Compare this equation with standard equation of SHM, a= -oi2e , we get

and T = Z1t =21t /I


,. I I
. (1) VP£·
· Here I is the moment of inertia of the dipole aliout an axis passing through centre of mass of the dipole.
Case 1 : When masses of the charges are equal, the centre of mass will lie at the centre of the dipole, and so
c;,
I = m(e12)2 +m(fl2/ o...,...._ ___.______.0
Ill . f/2 t/2 Ill
me 2
Fig. 1./75
~- 2
Case 2: When masses of the charges are unequal. Let .one of the charges is of mass m and other is of2m, if
X is the c.m. of system from m, then
.. - c;,
x "
( mx0+2mx£}
0
m+2m
Ill
--x--+·
. Cf,,,
u
Fig. 1./76_
3

I m (Ur
3
(er +2m
3
.d:} 4 2 2 2
-me +-me
·9 9

~me 2 .

A dipole in non-uniform electric field :
Let us consider a small dipole placed in a non-uniform field,-such as the field due to a point charge. Here, the
forces acting on the two charges of the dipole are not equal in magnitude and also their direction are not exactly
0

opposite to each other. For a short dipole torque can be 6litairied by the formula, '
~ = PE,sine,
The dipole experiences a net translatory force and a torque (in some cases it may be zero).

dipole

~=O.
,\> F.,. ,o 0, ~ ,o 0.
Fig. 1.177
~ ... ' . f
Electric multipoles :
According to the number of charges in the system, it is called the monopole (one-pole), the dipole (two-poles),
the quadrupole (four-poles), the octupole (eight poles) etc. In general the number of poles in a multipole is
always2", wheren=O, 1,2, ...... ..

Electric quadrupole :
The quadrupole is an arrangement of two parallel dipoles ofopposite polarity, but equal otherwise, separated
by small distance. We can regard quadrupole as an arrang,m~nt,oftwo dipoles. Figure shows few quadrupoles.

q - +q

q 2q q
+, :t q + -----------··· ~. q
. ,II
q
(a) (b) (c)
Fig. 1.178

Potential and field due to a quadrupole :


Consider a quadrupole as shown in figure.
r

Fig. 1.179
----· "",,,........,._---:---«---------- ~ -> ' -~- "•.,

1 · 'ELEeTR1c1TY&hAGNETISM- ... ·-------------


.
________
.
r~-.J.
\
The potential at point P due to the charges oft~~ quadrupole
1
I [ q 2q q ]
• V =. 41t Eo PB - PO+ PA

As OP=randAO= BO= e, and LPOA =0·, hence from the figure,


PA (r 2 + e2 -2recos0) 112 and

2 I ]
Thus V r + (r 2 + C2 -2rtcos0) 112

=- q
- u. e
l+-cos0+-)
2 -1/2 ( 2e
-2+ l--cos0+-
e2 )-1/2]
41t Eo [( ,- , , ,-2 r ,-2

Expanding by binomial, we get


1
2 2
V
q
= -- -
e
[( l--cos0+ e (3cos 0-1) + ...... )
41tEo r 2r 2

e· · ,0 _+
_ 2 + (,.+-cos e2 (3cos2 0-1) + ...... )]
r 2r 2

q e2
= ---(3cos 2 0'-1)
41t Eo r3
I I : ( '

£2 '
V q (3co~2 0,-1)
41tEo r 3 .i, •
. -- ---- - - - -- - -- -- -
Similarly electric field can be obtained, whiclfis-·,-
,:o it,.- '
3q£ 2 ' . "" . 1/2
E -~-4-[5cos 4-0''--2cos 2 0+ 1]
41t Eo r

Ex. 72 Fig. 1.180 shows the defletting plates ofan ink-jet printer, with superposed coordinate axes. An
ink drop with a mass m ofl.3 x I0- 10 kg and a negative charge of magnitude Q= J;S x I0-13C
enters the region between the plates, initially moving along the x-axis with speed v, =18 mis. The
length L of the plates is 1.6 cm. Jhe plates are charged and thus produce an electric field at all
points between them.Assume thatfield E is downward directed, uniform, and has a magnitude of
1.4 x 106 N/C. What is the vertical-deflection of the drop at the far edge of the plate? (The
gravitational force on the drop is sinall relative to the electrostatic force acting on the drop and
can be neglected.)

l
\
Sol. The drop is negatively charged and so force'oii'it ~els upward and
F = EQ.
y

"'' I ,•ti
•• •••• ..., .1'
....
f--='-'·c.:.·.:..·-----------'----+X

, __ _ _ _ X =L ~----1

Fig. 1./80
By Newton's second law

!__ = EQ
~.- = m m
Let I is the time taken by the drop to travel the distance L, so
L
L = "/ or t=-.
v_,
The vertical deflection

EQL2
y . 2
2mv:r.,
.;
(I .4x I 06 )x(J .5x I 0- 13 )x ( 1.6 x I0-2 )2
10
,, 2x (1.3x 10- )(18)2
6.4 x Io-4 m. Ans.

I,'
Ex.73 A uniform electric field Eis created betweeu two parallel, charged plates as shown in fig, I. 181.
An electron enters the field symmetrically between the plates with a speed v0• The length of each
plate is f. Find the angle of deviation of the path of the electron as it comes out of the field .

-- .--·t't
-
-·· T
1---=="-------~---i--+X

J.-------x=L ____.....,
. I
Fig. 1.181
Sol. As ihe field directed in downward direction, so forc.e and hence acceleration of the electron,)".i!~be
in upward direction. Acceleration
Ee
a,
111
The time taken in crossing the plate
e
I
Vo
The velocity acquired by the electron in upward direction
Eel
v~, = av1=--
., · ··mv0
Its horizontal component of velocity remain,constant.
lf0 is the angle of deviation of the electron, then
Vy
tan 0
~x
(Ee£/ mv0) '
I
Vo

or 0 tan-I ( - -
Eel) Ans.
mvJ

tQJ# ·The path ofthe charged


. angle with the field. . .
particle in an electric field will be parabolic, if particle is projected ~t some
.

Image method : Method of image consists of replacing a complicated charge distribution by a point
charge or group of charges, called electrical image, which is equivalent to given system of cha(ges.

Ex. 7 4 A point charge q is at a distance rfrom an infinite extend earthed conductor. Find force between
them.
Sol.

I.

q -q q
................................ . => •..... ,........................;......... .'...................
,,___ r---+---~ r-----1
;
!
r ----.j

Fig, 1.182
The electrical image ofcharge q is -q at a distance equal to the distance of q from conductor.
The force between the charge and earthed conductor will be same as the force between q and its
image(-q).

F =
_1_--'l!L. _ (_1_)
41t e 0 (2r) 2 -Eo
q2
41t 4r 2
Work done: Let charge q is to be displaced·from r 1 to r2 from earthed coryductor, then work done
. . . . . ,,·' ' ' . - '. . . '· .
'·! !•, . ~ ,. -,
_.,,,-_.·_ -W =,-·.J
Fdx ',. V=O
I\

'iiH--·"-'--+q>--4-F

q2 I( 1 ),:,
4!t Eo 4 X I\

L(_1__2.)
·J61te 0 ri r2
Fig. 1.183

If r2 = oo,and 'i =r; w =

Ex. 75 Three unlike point charges are arranged as.shown in jig. 1.184, whereAOB is the right angle
formed by two conducting half-planes. The magnitude of each of the charges is q and the distances
between them are shown in the figure. Find
(i) the total charge induced on the conducting half planes and
(ii) the forces acting on the charge--,,.
Sol. The half planes forming the angle A OB go to infinity, and hence their potential V = 0. The equivalent
system with zero potential has the form shown in figure (b). Hence the action of the charges
induced on the conducting half planes is equivalent to the action of the frictious charge -q placed
in the lower left comer of the dashed square.

a -q
. UaF
qr-·····'···o··--.t-'
q:-· -- . -. --- ---- .,
.:a/2 . ' ' .
.,71:0
' ..la
' A ·1·----·v: i
ii"a/2
......:q ,l. ...........~ ............ lq
-q (b)
(a)
Fig. 1.184
By reducing the system to four point charges, we can easily find the required force. Thus
F"" = .fi.F-F'
2
.fi. I q
. 41t Eo a2

Ans.
Ex. 76 A point charge q i's placed between two large parallel conducting plates I and 2 separated by a
distance f. Find the total charges q I and q2 induced on each plate, if the plates are connected by a
wire and the chargeq is located at a'distance CI from the left plate I (see.fig. J./85).
p

l ,., I CT a 2
(a) (b)
Fig. 1./85
Sol. Suppose CJ is the charge density on the surface P. lfcr 1 and cr2 are the surface density on the plates
respectively. then net charge density on plane to be zero
cr+(cr1 +cr2 ) = 0
..o[ .cr. = -,(cr1 +cr 2 ) ... (i)
The plates are connected by the wire,.and hence,the potential difference between them is equal to
,~ zero,.so . 1 1

0 ... (ii)

where

<J1 CJ 2
.-Ci --(f-£1) = 0 ... (iii)
Eo Eo
Solving equations (i) and (iii), we get

Ans. .

1,26 QuANTISATION OF CHARGE : MILLIKAN OIL DROP EXPERIMENT

Millikan's apparatus consists of two parallel metal plates A and B (parallel plate capacitor), are insulated from
each other with a small gap between them. Oil is ·sprayed in small drops from an atomiser above the upper plate
and a few of the drops are allowed tofall through a small hole in this plate. A light beam is incident between the
plates, and a telescope is used to see this motion. Oil drops soooly get terminal velocity. It is found that some
of the oil drops are electrically charged because offriction of mouth ofatomiser. The drops can also be charged
by falling X-rays on them. The drops are usually negative'Jr ~barged.
A

I1? f f"~"
'"£ "
qE
l '-' I T

@
J;;B

B mg mg
Telescope
Fig. 1.186
Suppose a drop has a negative charge q and plates are maintained at a potential difference
• ·,
such
'!
that a downward
fl j •

electric field E = ;s
V
is sep-up between them. The forces on the drop are its weight mg and the upward force

qE. Electric field is adjusted in such a way that the drop remains at rest. Thus
qE mg

mg
or q
E

lfrbe the radius of the drop, then m=pxim.3, and


3

q ... (1)

Radius of oil drop

Now switch off the field and allows the drop to fall freely. If vr is the terminal velocity of the oil drop, then by
Stoke's law, viscous force
F, = 6ltl]rvr ... (2)
As oil drop falls freely , so neglecting buoyant force exerted by air
mg F,

or

Substituting the value ofr in equation (1), we get

:' q = _l81t- )n 3
·_ d - .v?,- I
• ...VAB ___ 2pg .
~ ,. .. (3)

Millikan and his students measured the charges on the different sizes of drops of different oils and found that
every drop had a charge equal to some small integral multiple of charge e. That is, drops were observed with
charges of e, 2e, 3e, ......... etc.
,!J)J!

k~ity. ... ,, ,:,lVICQ Type 1


\.__:_;,I~--~--·-----,····· .. .. - &U!tdde 1.1
-
I. Consider a neutral conducting sphere. A positive .
2q
point charge is placed outside the sphere. The
(c) (d) zero.
net charge on the sphere is then, Eo
(a) negative and distributed uniformly over the 4. Figure shows three electric field lines. IfF A> F8
surface of the sphere and Fc are the forces on a test charge q at the
(b) negative and appears only at the point on positions A, B and C respectively, then
the sphere closest to the point charge
(c) negative and distributed non·uniformly
over the entire surface of the sphere
(d) zero.

q q
3 ,3
2. Consider a system of three charges and

(a) FA > Fn > Fe (b) FA < Fn < Fe


2
- q placed at points A, B and C, respectively,
3 (c) FA>(Fn=Fe) (d) FA<(FB=Fe)
as shown in figure. Take O to be the circle of 5. In the figure the electric lines on the right have
radius Rand angle CAB= 60°. twice the separation of those on the left. If a
yt charge particle takes time t to move a distance x
in left region, then it will take time to travel the
same distance in the right side region is :

I
(a) (b)
2
(a) The electric field at point O is _ _,.q__
2
81t Eo R (c) .fi_ I (d) 21
directed along the negative X·axis. 6. Figure shows an electric quadrupole, with
(b) The potential energy of the system is zero. quadrupole moment (Q = 2q£2). The electric field
(c) The magnitude of the force between the at a distance from its centre at the axis of the
quadrupole is given by

charges at C and B is
q2
o,.__,
q
. . . . . . .m . . . . . . . .0-----
-<J -q
C _$
q r
(d) The potential at point O is 7t Eo R
12

~
1
3. There is a point charge +q inside a hollow sphere (a) (b) ( 1 ) 2Q
( 4it Eo) 47t Eo r4
and a point charge -<J just outside its surface.
The total flux passing through the sphere
(c) ( I ) 3Q
-q q (d) none
(b) - 41t Eo r4
(a) Eo Eo
;•:. ·-lines and-the magnitudes (in N-m2/C) ofihe flux;
7. .A surface h;s th~~rea v~ct~r 1~;(2;'.+: 3J)m 2
:
7
.,. through tne six sides of.'each cube. The dotted

The flux of an'electric field through ii if the field ,:-.. · arrows are ofthe hidden faces. In which situation
,,c ·the,cube enclose net positive charge:
·_· •V , ..,,
is E =4i ....:. :·
m
(a) SV-m (b) 12V-m 1@"
5 ;"J
. ·... . 4
_,.B}·
l(';-3
. . ·. ;_ 5
2-u-··
. :
G_:_.s 5
(c) 20V-m (d) zero · 2 '. , 4 ,r
' "' ", 7
8. In figure a close surface encloses two of the four
positively charged· particles. Which of the \7 \p '.5
particles contribute to the electric field at point (I) (II) cnii
P on the' surface ·:
(a) (b) II
(c). Ill (d) none
I I. · The figure shows three non conducting rods, ·
.. : one circularand two straight. Each has a uniform
. charge of magnitude Q along its top half and
@··.. another along its bottom half. Which of them
q4 .
correctly represen(s the direction of field at point
(a) q1, qz (b) q1, q3 P:

·t3
(c) q1, qi: q3 (d) q1, qz; q3, q4
9. Figure .shows five charged lumps of plasti_c and. +Q
an electrically neutral metal coin. The close •Q~ ··--+
surface Sis indicated in the figure. The net flux
through the surface is :
·----····r,E
~Q -Q
. E p
.
p E
-Q
(I) (11) (Ill)

(a) (b) II
(c) III (d) I and lI
12. A long, hollow conducting cylinder is kept
8
-q4 coaxially inside another long,'hollow conducting
cylinder of larger radius .. Both the cylinders are
~2
\nitially electrically neutral
(qi -qz-q3-q4 +q5) (a) A potential difference-appears between the
(a)
Eo . two cylinders when a charge density is
given t~:) the inner cylinder. · ·
(b) A potential difference appears between two
cylinders when a.charge density is given.to
the outer cylinder. '
(c) No potential difference appears between
(c) (d) zero :the two cylinders when a uniform line charge
Eo
is kept along the axis of the cylinders.
10. The figure shows ttiree situations in which a (d) !'lo potential difference appears between
Gaussian ci.ibe sits iil an electric field. The arrows the twocylinders when same charge density
and the values lndi~ate the directions oft~e field
, is given to both the cylinders.

.
13. A_ small metal ball is brought into contact 16. The distance between the plates of a charged
alternatively with· points A, B and C of the plate capacitor disconnected from °the battery is
charged body shown in figure. After each 5 cm and the intensity of the field in it is£= 300
contact the charge of the ball is determined V/cm. An uncharged metal bar I cm thick is·
approximately by touching it against an introduced into the capacitor parallel to its plates.
electroscope. If qA, q8 and qc are the charges, The potential difference between the plates now
then is:
(a) 1500V (b) 1200V
(c)· 900V (d) zero.
17. The·equipotential surfaces ofa certain field are
shown in figure. It is kno"'. that V1 > V2, The
direction of electric field is correctly represents
(a) qA>qs>qc (b) qA<qs<qc in
(c) qA=qs,qc=0 (d) qA=0,qs=qc
14. An uncharged metal ball is placed in the uniform
electric field ofa plane capacitor. Which of them

(a)~=
correctly represents the field.lines on the ball?

~-
(a)

( b) +
:B@+=
+
.
- +
- +
.
-
-

(C) :~-f=
+
- +
_: + -
(b)

+ . -
(d) · none
15. Two identical metal balls of radius r are at a
distance a from each othe_r and are charged, one (c) ~ J ° \ d ) none
with a potential V1and the other with a potential
V2• The charges on the balls are : ~•
-I
(a) q 1 = V1a, q2 = V2a 18. The intensity of an electric field inside a
capacitor is£. The work done to move a charge
(b) q1 = Vjr, q 2 = V2 r q in a closed rectangular loop is

(a)
+
+
+
.+
E(2a+2b)
ob -

(b) £(2a)
(c) Ea (d) zero
19. Figure shows three -paths along which we can q, ,, -, I

move positively charged sphere ,4 closer to


positively charged sphere B which is fixed in a
plane. lfW1, W2 and W3 aretheworkdonealong
IL=_
the three respective paths, then ~
L/2
(a) W1> W2 > W3 3
(b) W1 > W3 > W2 IJ
(a) I (b) II
(c) W = W = W G, -fA
1 2 3 (c) Ill (d) IV
(d) none of these
24. Charges Q and-2Q are placed at some distance.
20. Three infinite long charged sheets of charge
The locus of points in the plane of the charges
densities-cr, -2cr and cr are placed parallel to xy- where the potential is zero will be
plane at== 0, z =a,== 3a. Electric field at point (a) a straight line (b) a circle
Pis (c) a parabola (d) an ellipse
-2cr • 25. A conducting sphere S 1 intersects a closed
(a) -k surface S2 as shown in the figure. A positive
Eo z
charge q is placed at a point P. What is the value.
of electric tlux through the surface S2 ?
2cr • z=3a~cr
(b) -k •p
Eo z=a -2cr
z=o -cr
-4cr •
(c) -k
Eo
4cr •
(d) -k
Eo
(a) 0
!L
21. Two equal positive charges are kept at points A (b) Eu
and B. The electric potential at the points
between A and B (excluding these points) is q
studied while moving from A to B. The potential (c) <~
0
(a) continuously increases 26. A conducting spherical shell having inner radius
(b) continuously decreases a and outer radius b carries a net charge Q. If a
(c) increases then decreases point charge q is placed at the centre of this
(d) decreases then increases shell, then the surface charge density on the
22. Electric charges are distributed in a small volume. outer surface of the shell is
1 he tlux ofthe electric field through a spherical
Q-q _.fL
,,,rface of radius· I m surrounding the total charge (a) (b)
2
,s I 00 V-m. The flux over the concentric sphere 4nb 4nb 2
of radius 2 m will be Q+q
(a) 25V-m (b) SOV-m (c) (d) zero.
(c) IOOV-m (d) 200V-m 4nb 2
27. A loop of diameter d is rotated in a uniform
L electric field until the position of maximum electric
23. Figure shows an imaginary cube of edge
2 . An flux is found: The tlux in this position is measured
uniformly charged rod oflength L moves towards to be lj>. What is the electric field strength ?
left-at a small but constant speed v. At t = O. the
left end just touches the centre of the face of the
cube opposite it. Which of the graphs shown in
(a)
~
nd2 (b) ::2
the figure represents the flux of the electric field nljld2
through the cube as the rod goes through it (c) (d) 4
28. A sp.here of radius 2 R has a uniform charge and earthed. The charge density on the l~ft face
density p._The difference in the electric potential of this plate is:
at r = R and r = 0 from the centre is :
(a) +cr
-pR2 ~2pR 2
(a) (b) (b) -cr
(c) +2cr
(d) zero
pR2 -pR2
(c) (d) 6 Eo
3 Eo
33. Four similar point charges q are located at the
29. A sphere of radius R carries charge density p vertices of a tetrahedron with ·an edge a. The
proportional. to the square of-the distance from energy of the interaction of charges is : ·
·the centre such that p = CR 2 , where C is a 'I
positive constant. At a distance R/2 froin the
centre, the !l'agnitude of the electric field is

(a)

q
(c) (d) none of these
6q2 4q2
(a)
30. 41t Eo a (b) 41t Eo a.
The work done in moving an electron of charge
e and mass m from A to B along the circular path
2
.(shown in,figure) in the vertical plane _in.the field 3q2 '(d) ,~q-
of charge Q is (c)
. 41t Eo a .. 41t,:e0 a
B
(a) 2mgr
34. Out of gravitation.al, electromagnetic, Vander
-2Qe Waals, electrostatic and nuclear forces; which
(b)
r m·
.-
two are able to provide an attractive force
between two neutrons
(c)
-2Qe
2mgr+--
:,. (a) Electrostatic an'd gravitational
r ,, (b) Electrostatic arid nuclear· :r
A
(d) zero (c) Gravitational and nuclear ,
31. 'ti
Two particles A and B having equal'charges are (d), Some other forces like Vander Waals i ·:
placed at a distance d apart. A third charged 35. · Two small spheres each having the charge +Q, ''
particle placed on the perpendicular bisection are suspended by insulating threads of length L
ofAB at distancex. The third particle experiences from a hook:This arrangement is taken in space ,
maximum force when where there is no gravitational effect, then the
d angle between the two suspensions and the
(a) X = .rz 1 tension in each will be

...· ., I Q2 , , . , • I Q2
d d (a) · 1so·,·-·---(b) 90 , - - . ""T
x=--
(c) (d) x= ../2
3 ,,
47tEo (2L ) 2 ,, . 4ireo L
2../2,
'L ' I, 1,
32. Two plane metal plates are placed p~rallel to each ' 2 2
other, one carries a surface density +a and the (c) 1so· _i_g_ (d) i'so· -·_i_·SL_
other-2cr. A third plate is placed between them . . '·47tEo 2L2 ,· ' . ' 47tEo L2
r . 0c . ·,. . ----;;- ·-- _ _ _ _E_LE_C_•TR_O._ST._J\_Tl_CS_
'·---.. -·- ..~.-' "-~--~-------.,---,~-~·----....::C·-·--·-"-·-·-----·--, -~- ·-·- - -
36. Two charges are at a di~tance dapart. lfa copper
' . d
plate (conducting medium) of thickness is . Es
2 (c) EA=- (d)
r
placed between them. the effective force will be
41. In the electric field of a point charge q, a certain
(a) 2F (b) F/2
charge is carried from point A to B, C, D and £.
(c) 0 (d) ,/2F Then the work done
37. Two spherical conductors B and C having equal
11
radii and carrying equal charges in them repel
. .
each other with a force F when kept apart at :' :

~
some distance. A third spherical conductor
having same radius as that of B but uncharged A
is brought in contact with B, then brought in c············D
contact with C and finally removed away from (a) is least along the path AB
both. The new force of repulsion between Band (b) is least along the path AD
Cis (c) is zero along all the paths AB, AC, AD and
(a) F 14 (b) JF/4 AE
(c)' FI 8 (d) 3F/8 (d) is least along AE
38. A charge q is placed at the centre of the line 42. The magnitude of electric field intensity £ is
joining two equal charges Q. The system of the such that, an electron placed in it would
three charges will be in equilibrium, ifq is equal experience an electrical force equal to its weight
to is given by
mg
_Q _Q (a) mge (b)
(a) (b) e
2 4
e e2
-g
(c) (d) (c) (d)
mg m2

39. The electric potential Vat any point O (x,y, z all 43. A charged particle is free to move in an electric
in metres) in space is given by V=4x 2 volt. The field. It will travel
electric field at the point ( Im, 0, 2m) in volt/metre (a) always along a line of force
i~ (b) along a line of force, if its initial velocity is
(a) 8 along negative x-axis zero
(b) 8 along.positive x-axis (c) along a line of force, if it has some initial
velocity in the direction of an acute angle
(c) 16 along negative x-axis
with the line of force
(d) 16 along positivez-axis
(d) none of the above
40. Figure shows the electric lines of force emerging
44. Two point charges Q and-3Q are placed at some
from a charged body. If the electric field at A and
distance apart. If the electric field at the location
Bare EA and £ 8 respectively and if the
of Q is Ethen at.the locality of--JQ, it is
displacement between A and B is r then
(a) -E (b) E/3
(c) -3£ (d) -EI 3
45. The number of electrons to be put on a spherical
,.
A•···············• B
conctu·ctor of radius ·o.1 m to produce an electric
field of0.036N/Cjust above its surface is
(a) 2.7xJ05 (b) 2.6xl05
(c) 2.5xJ0 5 (d) 2.4x 105
46. Four equal charges Q are placed at the four 51. !fa charged spherical conductorofradius IO cm
comers of a square of each side is a. W~rk do~~ has potential V at a point distant 5 cm from its
in removing a charge--Q from its centre to infinity
.
IS
. ,, centre, then the potential al a point distant 15 cm
from the centre will be

{a) 0 (b) {a) _!_ V (b) ~v


3 3

..fi.Q2 (c) iv
2
(d) 3V
{c) (d)
neoa 52. Ten electrons are equally spac_ed and fixed
47. The magnitude of electric field E in the annular around a circle of radius R. Relative to V=O at
region ofa charged cylindrical capacitor infinity, the electrostatic potential V and the
(a) is same throughout ' electric field Eat the centre Care
{b) is higher near the outer cylinder than ~~ar ' -
the inner cylinder {a) V;, 0 and E¢ 0 (b) V ¢0and E=O
{c) varies as I / r, where r is the distance from
the axis
(c) V=Oand E=O (d) V=Oand E¢0
(d) varies as I/ , 2 , where r is the distance from 53. The displacement of a charge Q in the electric
the axis
field E=e,i+e2}+ei:isi'=ai+bj. The
48. A metallic solid sphere is placed in a uniform
electric field. The lines of force follow the path(s) work done is
shown in figure as.
Q( ae, + be2)

~g~--t--1-i
(a)

2 2
(b) Q~(ae,) +(be2 )
4 · 4
{a) I (b) 2 (c) Q(e1+e2 )Ja 2 +b 2
(c) 3 (d) 4
49. A particle of mass m and charge q is placelat
rest in a uniform electric field E and then rel.eased.
The kinetic energy attained by the particle after 54. Three charges Q, +q and +q are placed at the
moving a distance y is vertices of a right-angled isosceles triangle as
{a) qEy2 (b) qE2y shown. The net electrostatic energy of the
(c) qEy (d) q2Ey configuration is zero if Q is equal· to
50. An electron of mass me initially at rest inoves
tllrough a certain distance in a uniform electric Q, ...
field in time 11.A proton of mass mp also initially
at rest takes time tz
to move through am equal i ······ . .
:
distance in this uniform electric fie,d. Neglecting •,_

the effect of gravity, the ratio of tz /


t 1 is nearly +q: ..............-::-:: +q
equal to ..__a---,

(a) (b) -q -2q


(a) l+..fi. (b)
2+ ..fi.
(C) .( me / mp )''2 (d) 1836 (c) -2q (d) +q

E::~:~:i;r::~~:_I~(:~11~~~~:~'.~~2[~~ ~·
' ' · . . .., ':· • , I ~:' • ·l.
~,t~?~f'STAnCS ~l!a
55. Ffgure's beiow show regular.. hex~gons, with . :6(!: 'i,. thin spherical conducting shell of radius R
" .. , ' . , •I,.,,,;>• ,r,
charges at the v~rtices .. h1which oft~efollowing . ,,;;_,ii }gs. a ~harge q. Another charge Q is placed at
, · cases the electr.,c ,field at the centre•IS not.zero. . : ,: the centre of the shell. The electrostatic potential
.· q. ..'. .. .'...lf
q_ ··'r': at a pointP a distance
R
from the centre of the
q-' ··•q 2
;;<·:. ./_}"· \ ...... -~ shell is
. ( I) . (2)
(q+Q)2
2q.. ....... :2q 2q_. ....... /I (a) (b)
4ire 0 R
q ::_·_· ;-_,_>, q/ \ 2, .: ,,,(!' ''.
. ..;·\ ' _,../'.. 1= 2Q q
2q• .. .:... ·'2q . -'I ........ q . (c) --+---
· (3) • . (4) 47tEoR 47tE 0R
• . '
(a) I (b) 2 : , . . 61. ')\' hollow conducting sphere is placed in an
(c) 3, · (d) 4 . '. eiectric field produced by a point charge placed
56 . . ,Electric , .. potenti_a\ at a.nl' poin.t is ;,,, ·atP as shown in figure. Let VA, Vs, Ve be the
V=-5x+3y+,/i5z, then the magnitude of potentials at point A, Band C respectively. Then
. I •; ,-~fl.
the electric.field is
(a) ·_3./i 4./i .

G
(b)
. . .'"'·.';f, IL,!

(c) . s,fi (d) 7 A c: .... ······i


57. The work done in b;inging a 20 co~lomb charge
from point A to point B for distance 0.2m is 2J.
B
The poten_tial difference between the two points
. ·"·. ·,wiHbe_(iii~olt)·· :•-Ja)_ Ve>Vs (b) Vs> Ve
,. , \, .... (a)'. 01 ,··: (b)'· '8"':·· ,__ .....
(c) 0.1' (dl · o:4 · · ·' (c) VA> Vs (d) ·vA = Ve
58. If 3 charges are placed at the vertices of 62. A pellet carrying charge of 0.5 coulomb is
equilateral triangle of charge q each. What is the "· ! accelerated through a potential of 2,000 volt. It
net potential energy;if the side of equilateral A
attains a kinetic energy equal to
isfcm .. .;1 '•
·(a) IOOOergs (b) IOOOJ
' 2
(a) _ _ g_
_ _ 2qi (c) IOOOkWh (d) 500ergs
(b)
47tE 0 f 4ne 0 e· 63. Electric potential. of earth is taken to be zero
~.1··i,il?~~ause earth is a good
·I . 3q2 _ _ 4q2 ,,,-,o(a). Insulator (b) Conductor
(c)·. 47tE • ~ (d) . 41tEo f •t.b(c)', Semiconductor (d) Dielectric
I .'. 0 • .'
59. -Ail. electrqn having charge 'e and mass 'm' is 1
·
64,. ,., Equipotential surfaces associated with an electric
moving in. a unifo_rm electric. field, E. Its ,;, 'field which is increasing in magnitude along the
acceleration will be ~ ·;t,X!direction are
(a) Planes parallel to yz-plane
e2 E 2e
(a) (b) (bl'. Planes parallel toxy-plarie
m, m
(c) ' Planes parallel toxz-plane
eE mE, (d) Coaxi~l cylinders ofincreasing radii around
(c) (d)
m .e !hex-axis
-- - -- -·,-··· ------. -- ·---------
--------- ,._______ -----· - - -·----- __ - - ·.~·-· -------
.. ELECTRICl1Y & MAGNETISM '
_,_ -~---·-'-----·- _.
65. Figure shows three points A, Band C in a region 69. Two infinitely long parallel conducting plates
. having surface charge densities +cr and -a
of uniform electric field £. The line AB is
respectively, are separated by a small distance.
perpendicular and BC is parallel to the field lines.
The medium between the plates is vacuum. lfe0
Then which of the following holds good. Where
is the dielectric permittivity of vacuum, then the
VA, V8 and Vc represent the electric potential at
electric field in the region between the plates is
points A, B and C respectively
a
(a) OV/m (b) -Y/m
2eo

a 2
cr Vim
B•·············· .. C (c) -V/m (d)
Eo Eo
70. Four point +ve charges of same magnitude (Q)
(a) VA=Va=Vc (b) VA=Va>Vc are placed at four comers ofa rigid square frame
~ ~=~<~ 00 ~>~=~ as shown in figure. The plane of the frame is
66. A charged ball B hangs from a silk thread S, which perpendicular to z-axis. If a -ve point charge is
makes an angle e with a large charged conducting placed at a distance z away from the above frame
sheet P, as shown in the figure. The surface (z<<l)then
charge density a of the sheet is proportional to
(a) sine + Q
p +.
(b) tan0 T
(c) case
(d) cote
+
/.!
s

z-axis
y ······-·-····
L

1
67. Two point charges + Sq· · d -2q are located at Q Q
x = 0 and x = L respectively. The location of a (a) -ve charge oscillates along the z-axis.
point on the x-axis at which the net electric field
" due to these two point charges is zerd '
(b) it moves away from the frame
(c) it moves slowly towards the frame and stays
(a) SL (b) 4L
in the plane of the frame
,.
L (d) it passes through the frame.only once.
"(c) 2L (d) For a g;iven surface the Gaus§•s law is stated as_
4 71.
68. Two thin wire rings each having a radius Rare
placed at a distance d apart with their axes
pE dA = 0 . From this we can conclude that
coinciding. The charges on the two rings are +q (a) E is necessarily zero on the surface
I
and -q. The potential difference between the (b) E is perpendicular to the surface at every
centres of the two rings is 1 point
(a) zero (c) The total flux through the surface is zero
(d) The flux is only going out of the surface
72. A charge q is placed at the centre of the open
(b) Q ['R
4m, end of cylindrical vessel. The flux of the electric
0
field through the surface of the vessel is

(c) QR I 41te0d 2
(a) zero (b)

~ ~R2 +d2]
1
(d) 2;o [ ....'L (d)
(c) 2£o
. --··---- . - ··- --·--·~· .. ---~ - _...,.. ... ~-----
·"·
..

73. Electric charge is uniformly distributed along a 76. Consider the charge configuration and spherical
long straight wire of radius 1mm.The charge per Gaussian surface as shown in the figure. When
cm length of the wire is Q coulomb. Another calculating the flux of the electric field over the
spherical surface the electric field will be due to
cylindrical surface of radius 50 cm and length Im
symmetrically encloses the wire as shown in the /''.. ··· .... 1/2
.... 'It \
figure. The total electric flux passing through :
the cylindrical surface is

g .. -··
(a)
Eo ti
·.J.\i (a)q2
(b) only the positive charges
l00Q (c) all the charges
{b)
Eo Im (d) +q 1 and-q 1
77. An electric Iine of force in the xy plane is given
I0Q by equation x2 + y2 = I. A particle with unit
(c) positive charge, initially at rest at the pointx = I,
(ltEo) ;]isoc;; y = 0 in the xy plane
+''• (a) not move at all
I00Q (b) will move along straight line
(d) (c) will move along the circular line of force
(ltEo)
(d) information is insufficient to draw any
74. q 1, q2 • q3 and q4 are point charges located at conclusion
points as shown in the figure and S is a spherical 78. lfon the concentric hollow spheres of radii rand
Gaussian surface of radius R. Which of the R(> r) the charge Q is distributed such that their
following is true according to the Gauss's law? surface densities are same then the potential at
their common centre is
s
Q(R2 +r2) QR
/lf1 R/ (a)
41tE0 (R+n)
(b)
R+r
/ /1.i I.>;

Q(R+r)
(c) zero (d)
41tEo(R2+,-2)
79. Two identical thin rings each of radius R meters
are coaxially placed at a distance R meters apart.
If Q 1 coulomb and Q2 coulomb are respectively
the charges uniformly spread on the two rings,
, .the· work done in moving a charge q from the
centre of one ring to that of other is
(a) zero

q(Q1 -Qz)(vtz-1)
(b)
v'2.4ite 0 R
(d) None 01 the above
75. If the electric flux entering and leaving an qv'2(Q1 +Q2)
(c)
enclosed surface respectively is q, 1 and q, 2 the 41tEoR
electric charge inside the surface will be
q(Q1+Q2)("'2+1)
(a) (l\l1 + <l>z)Eo (b) <<1>2 - <l>1)Eo (d)
(c) (<!>1 + <l>2l / Eo (d) (l\lz + <l>1l i Eo v'2.41tEoR
80. A charge +q is fixed at each of. th~.,

~~
points x = x0 , x = 3x0 , x = 5x0 ....... infinite, on
the x-axis and a charge-q is fixed at each of the
~~
points x=2x0 ,x=4x0 .x=6x0 ...... infinite.
Here x 0 is a positive constant. Take the electric
(c) ~/Jl~d)
potential at a point due to a charge Q at a
distance r from it to be Q I (41t£0r). Then, the•
84. Two equal point charges are fixed at x = -'a and
potential at the origin due to the above system
x =+a on the x-axis. Another point charge Q is
of charges is
placed at the origin. The change in the electric
(a) 0 (b)
q potential energy of Q, when it is displaced by a
small distancex along thex-axis, is approximately
proportional to
qln2
(c) = (d) (a) X (b) x2
41tEoxo (c) x3 (d) I /X
81. A non-conducting ring of radius 0.5 m carries a 85. A metallic shell has a point charge q kept inside
total charge of I.I Jx10- 10 c distributed non, its cavity. Which one of the following diagrams
uniformly on its circumference produiii,Ilg an
.. ,. . correctly represents the electric lines of forces
electric field E everywhere in,space. The value
f=O
ofthe.Jineintegral J -E.di (f=Obeingcentre
, (a)
/1.=oo

of the ring) in volts is


(a) +2 (b) -1
(c) -2 (d) zero
82. Charge q is uniformly distributed over a thin half
ring ofradius R. The electric field at the centre of
the ring is

q q (c) (d)
(a) (b)
2 2
21t E0 R 47t2EoR2

q q
(c) (d) 86. Two point charges +q and -q are held fixed at
2
41te 0 R 21tEoR2
· (-d, 0) and (d, 0) respectively ofa (x,y) coordinate
83. Three positive charges of equal value q are placed. system. Then
•\
at the vertices of an equilateral triangle: The
resulting lines offorce should IJe sketched as in . .
· (a) ·Eat all points on tliey-axis is along
. . i
(b) The electric field E at all points·on the
x-axis has the same direction

(c) Dipole mon\ent is 2qd directed ob1g i


(d) Work has to be done' i~ bringi~est
. _ch~rge from infinity to th~ origin "
87. Two point charges (+Q) and (-2Q) are fixed on _9.{ ,The variation o(potenti,al with distanceR from a
the x~axis: at positions a ~~d 2a· from" origin - , •o .., fixed point is as shown below. The electric field
! ~ '
respectively. At what positions on the axis, the .;,;; i~t}?=,5~ i~ , .. ; , , . ,
resultant electric field is zero ~~Ufl11'o · ·'
(a) Only·x = Ji.a (b) Only x= :...fi_a ·1r,i;., 11: -J

3a
(c),, , Both·x = ±Ji.a (d)x=--_-only
.2
0

88. A ch~rged parti~l.e' q is shot towards another


charged particle Q which is fixed, with a speed v.
It approaches Q upto a closest distance r and
then returns. If q were given a speed 2v, the
0 2 3 4 5 6
closest distances of approach would be
V
qo-+· ...• ':>Q Distance R in metre
"""r'
(a) r (b) 2r ,.. !.,
(a) 2.5volt/m
•,J,.
(b) -2.Svolt/m
(c) r/2 (d) r(4 . _(c) 2/5volt/m (d) -2/Svolt/m
It(, , •,J
89. Four charges'equal to-Qare placed at the four 92. The electric field due to a uniformly charged
comers ofasquare and a charge q is at its centre. · \ptiere of radius R as a function of the-distance
If the syste~ is in equilibrium the value of q is . ·• L.from its centre is represented graphically by

E
"(a) (b) ~(1+2fi.) ·.,( .'!.f
'
(c) -;(1+2iz) (d) ';'(1+2fi.) (a)
' 'I ' • - •
90._ Pqint_charge q moves from_point P to point S R r
' ... aloQg.t~e eath ,'.'QRS(fi,gu~e:s~~w!l) i~ a uniform
._,. <,irl1 E
·. electric field E pointing coparallel to the positive·
,:."·dri....-J
dire~ti~n -~f the ;-axis. The coordinates' of the
'poinisP,Q,R'imdS~re(a, b; 0), (2a; O," O), (a,-b,
0) and (0, 0, 0) respectively. The work done by (b)
th~. field .in t~e above process is given by the
·, expression R r\

·E
-+-~-~~--• E
:
•b·1!;ln ,r, i
,:
.. ,n\c),rt i
_,,,·.c.·~ - -__ Q-'-"+ .;.c·
.•
-X
t~' U'..!d
:
R

..·,.. _- R
(b) '-tjEa .,

I ; • • \,
. ;•,, •
01
\
"•.
, l/1 .r
93. Two concentric conducting thin spherical shells ,
A, and B having radii rA and r 8 ((r8 > rA) are q- I q
----
charged to QA and-Q8 (IQ8 1> IQAI). The electric
(a)
4lt£o d (b) · 4lt£o d
field along a line (passing through the centre) is

E
q
'(d)
__ -l
(c)
47tEo 2d 4itE0 2d

96. A thin glass rod is bent into a semicircular shape


(a) of radius R. A charge+ Q is uniformly distributed
along the upper hal fand a charge- Q is uniformly
distributed along the lower half as sho_wn in the
figure. The electric field at the centre P, is :

+Q
+
+ !·

)ip
(b)

0 X +~

' l
-
E Q -Q
(c)
(c)

0 ,:., ,. X

97. Five balls numbered I to 5 are suspended using


E 0

separaie threads. Pairs (I, 2), (2, 4), (4, 1) show


electrostatic aitractfon, while pairs (2, 3) and (4,
(d)
~) show repulsion. Therefore ball I must be:
(a) positively charged (b) negatively charged
0 X
(c) ne~tral (d) made ofme~I
98. A and Bare two concentric spheres. IfA is given
94. A particle of charge -q and mass m moves in a a charge Q while B is earthed as shown i,n figure
circular orbit of radius r about a fixed charge+ Q.
The relation between the radius of the orbit r
and the time period T is : B

(a) r= Qq T 2 (b) r 3 Qq r2 -+o+++~


2 I61t3 e0 m
+ +
l61t Eo m + +
.++ + +
Qq Tl
(c) 3
4it e 0 m

95. A point charge q is at a distance d from a (a) the charge density of A and B are same
conducting plane and is to be moved infinitely (b) the field inside and outside A is zero
away from the plane. The energz required to (c) the field between A and B is not zero
achieve this is : (d) the field inside and outside Bis zero
99. A bullet of mass 111 and charge q is fired towards 102. A disk of radius a I 4 having a uniformly
a solid unifonnly charged sphere of radius Rand distributed charge 6 C is placed in the x - y plane
total charge Q. !fit strikes the surface of sphere with its centre at (-a/ 2, 0, 0). A rod of length a
with speed v, the minimum speed v so that it can carrying a uniformly distributed charge
penetrate through the sphere : 8 C is placed on thex- axis fromx=a /4 tox=Sa
I 4. Two point charges~ 7 C and 3 Care placed at
Q Q
(a) (a I 4, -a /4, 0) and (-3a /4, 3a / 4, 0), respectively.
J21tEo mR (b) ,j67tE 0 mR
Consider a cubical surface fonned by six surfaces
x= ± a /2,y= ± a /2, z= ± a /2. The electric flux
Q 3Q
through this cubical surface is
(c) (d) J41t Eo mR
J41tEo mR
100. A plastic disk is charged on one side with a uni- · • \'

form surface charge density cr and then three


quadrant of the disk are removed. The remaining
quadrant is shown in figure, with V = 0 at infin-
( ~
X
ity, the potential due to the remaining quadrant
at point P is :
\~ __) •
p

,.
-2C 2C
(a) (b)
eo eo
. -. --. - - --. - !0C 12C
(c) (d)
eo eo

103. Let P(r)=Lr be the charge density


1tR4
cr
(b) -[R-r] distribution for a solid sphere of radius R and
2 Eo ·total charge Q. For a point 'p' inside the sphere
at distance r 1 from the centre of the sphere the
(c) ~[(r2 + R2)1/2 -r] magnitude of electric field is :
8Eo
(d) none· of these Q Q,/
(a) (b) 4
101. A neutral conducting spherical shell is kept near 41t Eo 1/ 41t Eo R
a charge q as shown. The potential at point P
due to the induced charges is : Q,}
(c) 4 (d) 0
31t Eo R

kq kq
(a) (b)
r r'
. kq kq kq
(c) --- (d) CP
r r'
--.· . --
ELECTRICITY & MAGNETISM_
- --
..-·----··-----------.-·------,,- . ··--····--·,··, ··-·
, - . ,. '. ,
~,
l
- - ---·------ ·-·-·--~ _.,. ______ --- .. _,_ ·----- -------...:>.·---=· ---------···-·----f
/ < , ' --... 0:----------- .... -
, I
lectricity . , '
\
\

;
1; ,.,

L..,....'..- .•r.-
'

- .,,
MCQType2 ·&xetrd4e 1,2
·-. . ...-·

Directions (Qs. 1-17): Read thefollowing questions 7. An electric dipole is placed at the centre of a
and select 1i1e riiht choices. More than one options sphere.
may be corre_ct. (a) The flux of the electric field through the
1. When the separation between two charges is sphere is zero
increased, the electric potential energy of the (b) The electric field is zero at every point on
charges the sphere.
(c) The electric field is not zero anywhere on
(a) may increase (b) may decrease
the sphere
(c) increases (d) decreases
(d) The electric field is zero on a circle on the
2. Which of the following quantities do not depend
sphere.
on the choice of zero potential or zero potential 8. For spherical symmetrical charge, variation of
energy? electric potential with distance from centre is
(a) potential at a point given in diagram. Given that,
(b) potential difference between two points
(c) potential energy of two-charge system V= q forr$R 0 ~ -
,
(d) change in potential energy ofa two-charge 41t e 0 R

3.
system
An electric dipole is placed in an electric field
generated by a point charge.
andV= q
41t Eo r
forr2'.R0 _ i
Ru
,-
(a) The net force on the dipole never be zero. Then which option(s) are correct?
(b) The net force on the dipole may be zero. (a) Total charge_within 2R 0 is q.
-(c) The torque on the dipole due to the field (b) Total electrostatic energy for,-,,; Ro iszero.
11lust be zero. (c) . At r= R 0 electric field is discontinuous.
(d) The torque on the dipole due to the field (d) There will be no charge anywhere except
maybe zero. r=R0 .
4. The electric field·and the electric potential at a 9. Consider the points lying on a straight line joining
point are E and V respectively. ' two fixed opposite charges. Between the charges
(a) lfE=0, Vmaybezero there is
(b) IfV=0,Emaybezero (a) No point where electric field is zero
(b) Only one point where electric field is zero
(c) . If E ;< 0, V may be zero
(c) No point where potential is zero
(d) IfV;<0, Emaybezero. (d) Only one point where potential is zero
5. The electric potential decreases uniformly from 10. An ellipsoidal cavity is carved within a perfect
I 00 V to 50 Vas one moves on the y-axis from conductor. A positive charge q is placed at the
·· y=-Im toy=+I m. The electric fieldattheorigin. centre of the cavity, The points A and B are on_
the cavity surface as shown iri the figure. Then
(a) must be equal to 25 VIm
(b) may be equal to 25 Vim
(c) may be less than 25 VIm
(d) may be greater than 25 VIm
6. Jfthe flux of the electric field is zero.
(a) The electric field must be zero everywhere
on the surface. (a) Electric field near A in the cavity= Electric
(b) The el.~ctric field may be zero-everywhere field near B in the cavity
on the surface. (b) Charge density at A= Charge density at B
(c) The charge inside the surface must be zero. (c) Potential at A= Potential atB
(d) The charge in the vicinity of the surface (d) Total electric field flux through the surface
must be zero. of the cavity is q I e 0
I I. A positively charged thin metal ring of radius R (b) the potential is zero at a point on the axis
is fixed in thexy-plane with its centre at the O.A which is x/5 on the left of the charge -Q/4
negatively charged particle P is· released from (c) electric field is zero at a point on the axis at
rest at the point (0, 0, z0 ), wherez0 > 0. Then the a distance x on the right of the charge
motion of P is -Q/4
(a) Periodic for all values of i 0 satisfying (d) there exist two points on the axis where elec-
tric field is zero.
0 < zo < 00
16. . A non conducting ring of radius R is charged as
(b) Simple harmonic for all values of satisfying
shown in figure :
0< z0 < R
(c) Approximately simple hannonic provided .;-tt-t.~+(J
~'~: ··+
z 0 << R -{ ... l-·_:;+
(d) Such that P crosses O and continues to
move along the negative z-axis
-=-~:::..._L. .i..:q
towards z = -oo
12. A non-conducting solid sphere of radius R is
(a) The electric field is zero at the centre of the
uniformly charged. The magnitude of the electric
nng
field due to the sphere at a distance r from its (b) the electric potential is zero at centre of the
centre ring
(a) Increases as r increases for r < R
(c) the electric potential at the centre is,
(b) Decreases as r increases for 0 < r < =
(c) Decrease as r increases for R <,. < = 2q
V
(d) ls discontinuous at r = R · 4n Eo R
13. A dielectric slab of thickness d is inserted in a (d) the electric field at the centre is
parallel plate capacitor whose negative plate is
atx= 0 and positive plate is atx= 3d. The slab is q
E
equidistant from the plates. The capacitor is "2 Eo R2
given some charge. As one goes from 0 to 3d 17. Figure shows a cross-section of a spherical metal ,,
(a) The magnitude of the electric field remains shell of inner radius Rand out radius 2R. A point
the same charge q is located at a distance R/2 from the
(b) The direction oftheelectric field remains centre of the shell. If the shell is electrically neu-
the same I
tral, then:
(c) The electrfc · potential increases
continuously
(d) The electric potential increases at first, then
decreases and again increases
14. Two point charges +q and -q are held fixed at
(-d, 0) and (d, 0) respectively ofa,:y-plane, Then:
(a) The electric field Eat all points on the x-
axis has the same direction. (a) the electrical field at some point inside shell
(b) Electric field at all points on y-axis is along is zero.
x-axis. (b) the electrical field at point inside shell is
(c) . Work has to be done in bringing a test non-zero
charge from =to the origin (c) the electric:il field at the outer surface of
(d) The dipole moment is 2qd along x-axis.
I . q
15. Two point charges Q and -Q/4 are separated by
a distance X then :
the shell is E = 4m:0 ( 3R /2)

Q X -Q/4 (d) the electrical field at the outer surface is


(a) the potential is zero at a point on the axis E=_l _ _!L
which is x/3 on the right of the charge 4m: 0 2R
-Q/4
Passage for Q. 18 to Q.20 21. The electric field at r= R is
A long cylinder of radius a is charged with a volume (a) independ~nt of a
charge density (b) directly proportional to a
P = Por3 for r $ a (c) directly proportional to a 2
= 0 forr>a (d) inversely proportional to a.
where p0 .is constant and r is the. d~stance 1_11easured 22. For a= 0, the value d (maximum value of p as
from the axis of the cylinder. shown in the figure) is

18. The charge contained in elength of the cylinder (a)


3Ze 3Ze
for r> a is: ·4nR 3 (b) nR3

(a)
2n p0 a 3 e (b)
2n p0 a 5 e 4Ze Ze
3 5 (c) 3 (d) 3nR3
3nR
3
(c)
4n p0 a e (d) zero 23. The electric field within the nucleus is generally
3 observed to be linearly dependent.on r. This
19. The electric field at r < a : implies

p~r4 3 R
(a)
Po' (a) a=O (b) a=-
(b)
5 Eo 3e0 2

5, 2R
Por (c) a=R (d) a = -
(c) (d) zero 3
5 Eo
20. The electric field at r > a
,, p a5
Passage for Q. 24 to Q. 26
Poa3
(a) . (b) ._o_ Two small metallic balls ofradii R I and R2 are in vacuum
3e0 r 3 Eo r e,
at a distance which is considerably exceedi_ng their
. (c)
Poas
(d) zero
dimensions and have a certain total charge Q. It is
known to you that electrical energy of this system is U
5 Eo r = U1 +U2 + U12, where U1 and U2 are their selfenergies
and {!12.i.s their external et)eJgy.
Passage for Q. 21fo Q. 23 24. The electrical energy of the system i,s given by:
The nuclear charge (Ze) is non-uniforinly distributed
within a nucleus"ofradius R. The cnarge density p(r)
(charge ,per unit. volume) is dependent· only on the
(a) U- I .·r ~/2 + R1qtqz
4!C Eo. 2R1 +R2
)
radial distance r from the centre ofthe ~u~leus as shown
in figure, '.fh(~lectric- field is only along the. radiai
direction:-;: ' ·, • ·
<' ,:-~(r

-'di--~~
U =_,_ 2 2
!r!__+2L+ •
qlq2 )
(c) 4!C Eo ( 2R1 2R2 f

a R .. r
(d) zero
25. For the electrical energy U to be minimum. the 26. The potential difference l\etween the balls when
charges q I and q2 on them are : energy,ofthe system is minim':1m : ,
. ;Q, : Q
(a) q1=2,q2=2
. R1 ·R,
(a)
1
4rr Eo ( !: ~ ;:·)
(b) q1 =QR. qz =Q--;- _ I- ( 'II - '12 )
2· I (b)
4rr Eo € _
. QR1 QR2
(c) q1 =R+R ,q2 =R+R ( qi- -
-I 2 I 2 (c) -I- - l/2 )
4rr Eo R1 e
- QR2 QR1
(d) '11= ,qz= (d) zero
. R1 +R2 R1 +R2
. Multiple Matching
27. Column II describe graph for charge distribution_ given in column I match the descrip1ion.
Column I Column II
E \ Elt:ctric
lie d im.:nsitics)

(A) Uniformly charged ring (p) ---¥-------'- r

i· (Ekctric
polcnti,11)

·' ' (8) Infinitely large charge conducting sheet (q) ..=:..--+--~ r
E

· (C) ··in-finite non conducting thin sheet..

"C., V

· . (D) Hollow non conducting sphere: (s)~


' '.
r

27. ,{J\) ,-,-,p,,q, (8) _::.-H, (C)->. r, (D)-> s


&Ufld4e l,3

Read the two statements caref~lly to mark the correct option out of the options.given below. Select the'tight
choice. · ·
(a) If both the statements a.re true and the Stateme11t- 2 is the correct explarnition ~f Stateme11t- l.
(b) . If both the statements are true but Statement - 2 is.not the correct explanation of the Statement- 1.
(c) If Statement- I true butStatement-2 is false. .
(d) If Statement-I is false butStateme,,t-2 is true.

l. Statement - I 7. Statement - I
Some charge is put at the centre of a conducting !fa proton and an electron are placed in the same
sphere. It will move to the surface of the sphere. uniform field. They'experience same force but
Statemeni - 2 different acceler~tion, ·
Conducting sphere has no free electrons at the Statement - 2
centre. By Newton's second law F =mii.
2 Statement - I 8. Statement - I
Charge given to, a conductor comes to its _outer !fa point charge q is placed in front ofan infinite
. surface. · grounded conducting plane surface, the point
Statement - 2 charge will experience a force.
All the electrons of the conductor will move on Statement - 2
to the surface. The force is due to the induced charge on the
3. Statement - I conducting surface which is at zero potential.
Gravitational force can be added vectorially to 9. Statement - I
the electric force. A point charge is brought in an eiectric field, the
Siatement - 2 field ala nearby point will increase or decrease;

• Gravitational field can be added vectorially to


the electric field.
depending on the nature of charge.
Statement - 2
The electric field is independent of the nature of
4. Statemelll - I
A spherical shell made of plastic, contains a charge.
charge q distributed uniformly over its surface IO. Statement - I
giyes zero electric field inside the shell. The lightening conductor at the top of high
Statement - 2 building has sharp pointed ends. It is used to
The electric field at the centre of the shell is zero. prevent the cracks in the building.
Statement - 2
5. Statement - I
The charge density at sharp ends becomes very
A charge q is put inside a cavity in a conductor.
much higher than the building.
The electric field outside the surface of the
I I. Statement - I
conductor is zero.
Electrons move away from a low potential to high
Statement - 2
potential region.
The force on a charge Q placed outside the
Statement - 2
sphere will be non zero.
Electrons have negative charge.
6. Stateme11t - I 12. Statement - I ,.
Electric lines of field cross each other. The tyres of aircraft are slightly conducting.
Statement - 2 Statement - 2
Electric field at a point superimpose to give one If a conductor is connected to ground, the extra
resultant electric field. charge induced on conductor will flow to ground.
13. Statement - I 20. Statement - I
Fourchargesq, q, -q,-q are placed at the comers
ofa square. The electric field at the centre of the The electric flux of the electric field f E•dA_ 1s
square is zero. zero. The electric field is zero everywhere on the
Statement - 2 surface.
The electric potential at the centre of the square Statement - 2
is zero.
The charge inside the surface is zero.
14. Statement - I
When two bodies with charges Q and -Q are 21. Statement - I
brought in contact, the charge on each body A point charge Q is placed in a cavity in the
becomes zero, and hence charge is lost. · metal cube, the electric field at any point in the
Statement 2O cube is zero.
Some energy disappears in the for_m of heat and · Statement - 2
light. The electric field at any point on the cube is not
15. Statement - I -·· .zero.
Dielectric breakdown occurs due to the intense 22. Statement - I
electric field. ·· The point charge q I and q 2 produce fields £ 1
Statement - 2 and £ 2 at any point and potential V1 and V2 at
Cosmic rays make the ionosphere. the same point. The electric field due to both the
16. Statement - I charges at that.point must be £ 1 + £ 2•
A metallic shield in formofa hollow shell may be
Stateme11t - 2
built to block an electric field.
The electric potential at that point due to both
Statement - 2
In a hollow spherical shield, the electric field the charges must be V1 + V2•
inside it is zero at every point. 23. Statement - I
17. Statement - I · Gauss's law cannot.be used for calculating the
For practical purposes, the earth is used as a electric field- due to three equal charges located
reference at_zero potential in electrical circuits. · at the corners of an equilateral ,triangle.
Statement - 2 Statement - 2
The electrical potential of a sphere of radius R, Gauss's law is not valid for this case.
with charge Q uniformly distributed on the 24. Statement - I

w ~"
Q
surface is given by 4lt Eo R .

18. Statement - I
Two metal spheres of radii Rand 2R with charges I - - - cl -------1
Q and 2Q are put in contact. The charge flows
Figure shows a point charge q is placed in front
from sphere of radius 2R towards sphere ofradius R.
of a metal cube at a distance d from its centre.
Statement - 2
The electric field due to this charge at the centre
There is no loss ofelectrical energy of the system.
19. Statement- I I q
A charged.sphere with charge Q is placed inside of cube is 4 lt Eo -;Ji.
a conducting shell and they are connected by a
metal wire. The entire charge of the sphere will Statement - 2
move to the shell. The electric field at the centre of the cube is zero.
Statement - 2
The electric potential of the shell becomes equal
to the sphere.
·
-·:· --.,·, .. ····· . ·····.··:·~-.i·-.. ··-·-,·-.·--:.-···.,.·.. --··: .. ·,··_ ......... ·-._ ...
.~-:~.·-·._.•·.._:.r~--·
•.ELECTRICl1Y&~ MAGNETISM , · ,. , · .. · ... , · . · . , :, . ··
"··7·
... ,., __ ._, ... - . . __,·;---.-~· ·---··----·-···'•- ___ . __ or..-,.,.·~,:•· ._:~ ..1

; 1-- 1·· (d}- 1 21 r·(d) T-41 -1-(c). T j;f. : (d) I. -iir- [laj·-r-101·1 (c) If

i -2-,-- (c)-· [ 22 I (c) ! 42. - :·(tiT i 62 l(b) ! 82 l (a) ! 102 i (a)


r-3-niJY 23 I
(dl :. 43 I I (bl 63 I (bl I 83 I (cl f 103 j (bl
!
j4--: (c) i 24 ·44--I :(af·-, (b) / 64 i (a) ! 84 (b) _ _ ,--- I
! 5 ·-:···ccl--i 25 ·1··,bl-: 45··1 <cl r·s5·-·1 (bl 85·-r <cl · 1 r·-··
:·s·--1·· (cj-·,-·2s ·ic)· r·46 I ·r (cl I 66 i (bl f-86 --; <al . ,-
1 -·,-(a) ·:·27-J-(a)--[--47 j (c)- i 67-f"(c)-:87--l(b)-!-·--~:---
8 I -(dj" --28 ·1 (dfT-4a--r·(d)--[ 68---1- (d) r--sa·1 (d)_I_ - ...
9 . ; {bl I 29 f (dl ; 49 · 1 (cl i 69 r <cl 89 I (bl f r ·
·· 10 - -- iiil- ;Jo I <al i 50 I (bl I 10 , (al 1 90 I (bl r--··1-----
. f1-;-ia> ! 31 I <cl 1 51 I (bl I 11 i (cl i 91 rw 1
. - .-
: 12 ;··caJ-132 [lbl ·152-·1 {ii) -)'12··r<c> ·r-92 ! (bl ... l ___ _
, - 13- i-(a)' f -33nar ,- 53. ;·1ar -r· 73-I ¥': 93 ,-(aj' i
;-14 i (c) ;34-i (c) · 54 ! (b) I 74 I (b) i 94 · j (b) ·,- -
,--15·-,-(d)-i 35 [(a)-[55. , (b} ; 75 i (b) ,-95--j-(d)-: - . ,---··
·16 ·:·(b)--[-36· 1·(c)-j56--i (d) 1··1s·-1(c) -95··1-(d) T·-·:·
-17--~-(aT :'ii. r-fdj-·:-57-·[--(c)": ·1} --i-<cl-i ···91-r··cc)-r -- 1

I ·18·- :--(d) j. 38--;-(b) ~· sa"j-(c)'['--78-'(cij -[ 98 !--(c) -:,-- - f


.-----, · -- --1 ------;---r-7-,--r- -~-....------- r-----·r--
1 19 I (c) , 39 I (a) i 59 i (c) ! 79 1 (b} ! 99 : (c) ,
i ! (a)
20 · 40 ! (a) , 60 ! (d} ! 80 j (d) ; 100 j (c) - : - .
--==~==~= . ..,,_~--~=~~~=~---~-~...JI

1 - 1 I (a) ~J(di'--: 9 1·11


i· 2 --j·(c)--j · 6 JTci}i 10
I (c) I 13
I (d) 1 (a)
I 21
I (b)
11.r· I' I
' ' ' I (a)
I
(d}-j18
!
(d)
I 22 I
i (d)
t3
I i ·(c) i 7
15 f<af--: I· 1_1
19 ! (a)
I
II f7bf-l I (a)
I 23 ! (c)
1-4-j(b-) 18-1 (a) ...,112...i(b)Jfs'--l-(a)--f-20·1 '
- 1
~
• · I ·- 1 I
(d)
I 24
-
I (a)
I
' EucmoSTATICS -~-
! ~---·~·--·- '-----~- .____, ----------- - - - - - --~ - ------ -"~---- --

&¾eltdJe 1. 4
I, The distance between two fixed positive charges 4e and e is /_ How should a third charge q be arranged
for it to be in equilibrium? Under what conditions will the equilibrium of the charge q be stable and when
will it be unstable?
I
Ans. At a distance
3 from the smaller charge. The equilibrium will be stable during longitudinal dis-
placements. if the charge q is positive and unstable if q is negative_
2. Two free positive charges 4e and e are a distance a apart. What charge is needed to achieve equilibrium
for the entire system and where should it be placed ?
' ~ a
Ans. The negative charge q = should be placed at a distance from the charge e_
9 3
3. A negative point charge 2e and a positive charge e are fixed at a distance/ from each other_ Where should
a positive test charge q be placed on the line connecting the charges for it to be in equilibrium? What is
the nature of equilibrium of the test charge with respect to longitudinal motions. Plot the dependence of
'the force acting on this charge on the distance between it and the charge +e.
Ans. The test charge should be placed at a distance a = I ( I + .../ 2) behind the positive charge_ The
equilibriumwill be stab1e_

F
Forces of repulsion
:,.._____a_; !,
-G~-Q~-(Jr,;--,-------- /'
-----@j)-----+------4i------·--------~------ -2e e q
-2e : 0 . +e +q
. :+---1 ---.:
Forces of attraction

4. Two small equally charged conducting balls are suspended from long threads secured at one point. The
charges and masses of the balls are such that they are in equillibrium when the distance between ,hi(TI
is a= 10 cm (the length of the threads L >> a). One of the balls is then discharged_ How will the bal'is-
behave after this? What will be the distance b between the balls when equilibrium is restored?
a
Ans. The balls will first go down, touch each other and then move apart a distance b = ~ .

S. A ball of mass m is suspended from a thread in a capacitor. In the absence of any charges the period of
oscillation of the ball is T1• After the capacitor and the ball are charged the period ofoscillation becomes
T,. What force has the electric field of the capacitor exerted on the ball? What is the length of the thread
fr';,m which the ball is suspended_ Determine the period ofoscillation, if the sign of the ball is reversed.

7/g
Ans_F=3mg;l=- -
-; T-21t
41t 2
-
2g
. Hi
6. Two like point charges of5 cgs electrostatic units each are at some distance from each other. Determine
the potential at a point IO cm from each of the charges_ How will this potential change if the space around

...
the charges is filled with a dielectric of permittivity_ E = 2 ?
Ans. V=300Vand l50V_
7. A charge is unifonnly distributed over the.surface of two concentric conductive spheres (with the same
density cr). What is the magnitude of this charge.if. a quantity of work of Vis required to .transfer one
positive unit of electricity from infinity to the cominon centre of the spheres? The radii of the spheres
arerandR. ·

,2 +R2
Ans. Q ---V.
f_: h, l -- . r+R
8. Suppose all the electrons of I 00 g water are lumped together to fonn a negatively charged particle and all
the nuclei are lumped together to fonn a positively charged particle. If these two particles are placed
I 0.0 cm away from each other, find the'fdrce of attraction between them. Compare it with your weight.
•, ,. " Ans.2.56x!!l2 5 N.
9. o·
Two identical balls, each having a charge of 2.00 x I 7 C and a mass of I00 g, are suspended from a
common point by two insulating strings each 50 cm long. The balls are held at a separation 5.0 cm apart
and then released. Find
(a) the electric force on one of the charged'bali's' '
(b) the components of the resultant force'on,it along and perpendicular to the string
(c) the tension in the string
(d) the acceleration of one of the balls. Answers are to be obtained only for the instant just after the
release.
Ans. (a) 0.144 N, (b) zero, 0.095 N away from the other charge, (c) 0.986 N anl (d) 0.95 m/s2 perpendicular
to the string and going away from the ot.~.~r charge.
10. Two identical pith balls are charged by rubbing against each other. They are suspended from a horizon-
tal rod through two strings of length 20 cm each, the separaiion between the suspension points being 5
cm. In equilibrium, the separation between the balls.in 3 cm. Find the mass of each ball and the tension
in the strings. The charge on each ball has a magnitude 2.0 x I o·8 C.
Ans. 8.2 g, 8.2 x J0·2 N.
11. A wire is bent in the fonn ofa regular hexagon and' a'total charge q is distributed unifonnly on it. What
is the electric field at the centre? You may answer this part without making any numerical calculations.
Ans. zero.
12. A circular wire -loop of radius a carries,a'total charge Q distributed unifonnly over its length. A small
length dl of the wire is cut off. Find the electric field at the centre due to the remaining wire.

13. An electric field of 20 N/C exists along the x-axis in space. Calculate the potential differe~ce V8 - VA
where the points A and B are given by,
(a) A=(O,O);B=(4m,2m)
(b) A=(4m,2m);B=(6m,5m)
(c) A=(O,O);B=(6m,5m)
Do you find any relation between the.~swe.:S of parts (a), (b) and (c)?
Ans. (a)-80V (b) -40V (c)-120V.
•(U~ ,,

14. The electric field in a region is given by •E=·~'E/i+i E0j withE0 = 2.0 x I03 N/C. Find the flux of this
· ,. 5 .5
field through a rectangular surface of ~r~~ ,0.2 ·m2 parallel to the yz- plane.
Ans. 240 N-m2/C.
15. ·, A nonconducting sheet of large surface arei1 and·thickness d contains uniform charge distribution of
density p. Find the electric field at a pointP inside the plate, ata distancex from the central plane. Draw
aqualitativegraphofEagainstxfor0<x<d.- ,· - .o:,,J ·
px
Ans.-.
Eo
16. · (a) Two_ similar point charges q I and q2 are placed .at a distance r apart in air. If a dielectric slab of
thickness I(<< r) and dielectric constant k is put bet\V~~D the charges, calculate the Coulomb force of
repulsion. ,, . 1,- ;
11 1
(b) lfthe thickness of slab covers half the distance b,~!)yeen the charges, the coulomb repulsive force
is reduced in the ratio 9: 4, calculate the dielectric constant of the slab.
I
,-Ans. (a) F q1q2 (b) k=4.
· • 41tEo (r-t+t.,Jk) 2

17. A thin non conducting ring of radius R has a linear, charge density A= Ao cos qi , where Ao is a constant -
' ' , ~ ,L

qi is the azimuthal angle. Find the magnitude of the electric field strength
(a) at the centre of the ring;
(b) on the axis of the ring as a function of the-distance r from its centre. Investigate the obtained
function at r >> R
f'i!'
. A
0 - (b) E
'Ans.(a) E=--
--n-:. 4EoR
18. A charge+ lo-9C is located at the origin in free space and another charge Q at (2, 0, 0). Ifthexcomponent
of the electric field at (3, I, 1) is zero, calculate the value-of Q. Is they component is zero at (3, l, 1)?
,.. , ' Ans.Q=~3 512!(Il 312 x w-9)C;notzero
19. Avery long uniform charged thread oriented along the axis ofa circle ofradiusR rests on its centre with
one oft~e ends. The charge perunit length of the threa~ js equal to A. Find the flux of the vector E across
the circle area.
Ans. '/JI / 2Eo
20. A thin glass md is bent into a semicircle of radius R. A,charge + Q is uniformly distributed along the
upper half and a charge - Q is uniformly distributed along the lower half, as shown in figure. Find the
electric field E atP, the centre of the semicircle.

1
Ans. E = - (~)
1t Eo 1tR

21. One end of a IO cm long silk thread is fixed to a large vertical non conducting plate and the other end is
fastened to a small ball having a mass of 10 g and a charge of 4.0 x I o--6c. In equilibrium, the thread makes
an angle pf60° with the vertical. (a) Find the tensioii in the thread in equilibrium (b) Suppose the ball is
. slightly pushed aside and released. Find the time period'ofthe small oscillations.
· Ans. (a) 0.20 N (b) 0.45 s.
I'

22. In figure, a uni_forrn upward-pointing electric field E of magnitude 2.00 ~. I O~NIC has been set up. between
two µorizontal plates by charging t~e;lower pla\e positively and. t~_e upper, pLaie,)legatively"Tcl!e plates
have length L = 10.0 cm and separation d= 2.00 cm. An elec!ron is the~ ~h~t.!>et)Yt;en.the plaJ~.~ from the.
left edge of the lower plate. The initial ve!ocity v0 of the electron makes an angle 8 = 45° with the lower
plate and has a magnitude Of6.00 X 106 m/S.
(a) Will the electron strikes one of the plate?
(b). lfso, which plate and how far ho~izont~IJy fr~m the Ieti edge?.·. '"·'· "~ ., . ,,·. ' ,i I

,.
I '•. J• ., ' ,· 1· . ,, " " '
' ·· 1r••,

. ,. ,
,'

~-·
. e
__,
,
, .. _;Ins. (aJ'.Electro~ hits !he UP,perP,l_at~ (b).,x,=;2.72 9,;i
(/';, b J ,j
,, ,_
u!

23. In figure, the charge on a neutral isolated conductor is separated by a nearly positively charged rod.
What is the flux through the five Gaussian surfaces . shown'in cross sediciii ?'Assume that the charges
enclosed by S1, S7 , and S, are equal in·magnitude. ·,,, , ,· "
I<,'' .-IJ 'f1.

·. "~

'. -· ·. · · · · · ····-~. '<;


(··:.·.·_·:.·,:.-::_·- ..... ,,-_~_:_._.~~~---:::::· (_~;. \

l / ,~(c++~*!'.~~:condtictof -i--:,~.r::._,~\ : .::~/ L '1);


Rod

~,1! r. , t, ~-..
1

.. \. ::~/;}i::::~:.:::~::::-:/:~·-:-:_:.::,~:-:~>S.~, . ·.· . J'.:-. ·-',,_·'.: • •• -~-':· _··1,


:s -.-........
I
J- ••• sX·--.-. . . . ,11, /',
• 4.,
1,, ·.1 .. ... .

It . Ji"J ) ,, :j
=..i. <1>2 =-..i.
E • . E ' E ' 't'4 '
Ans" <1>1
<i>J =..i. "' =0 <l>s =..i.
E
• ., .-,-,,.
19
, • :.,,.,,o ·, .. ;\f·t~·-··: .. ,,o,.,,._:•;;,,,
1 , • ! ' I · 1 0 1_,1_ ,9.
',·

24. A positron, of charge 1.60 x;I 0- C, revolves in a circular path of"radius r between and concentric with
the cylinders as shown in figu;e. what must be it~ k-i~etic ene~gy"K'is ~le~fr~~ v~It?' A;sii;;,~'ih;i a= 2.0
cm,b=30cm,andA..=30nC/m. -)~ i·•-. ·.,r H·.-·.,,, ' ' · ·,: .., '• 1 •

_.-,,'

,!J. . I; •I ,ll • >,c.Ans.'270e'V.


... ·...,
·,J r .

'[l
'1' 1 ' l ,, ,
25. · In figure;_a sphere of radius a ~rid a.charge +q-uniformly d\sfributed throughout its volume, is concentric
,with.a spherical conducting shell·of inneM:ad_iu,s band R.uter rad[us c. This shell has a net charge of-q.
Find an expression for the electric field as a'timction of the radius r (a) within the sphere (r < a); (b)
between the sphere and the shell (a <r < b); (~) ,inside.thfshell (b < r< c); and (d) outside the shell (r>
c), (e) what are charges on the iriner and outer surfaces of the shell?
,- 11' '
p
. jr '
;"'n
z
-6Q

R
_,
' -: ' ' ' j ,~~ J ,1 /.J. "'
.Q
3 3
·A~":.(a) E=_l_qr (b)£~'-·_l"_!L.(c)E' i· q(c -'r i (d)E=O(e)-qandzero.
''1 • ,., .
·' ·
. '
·· '· · 4it E0
.
,,3 ·
.
4it'eo ,- 2
' .
· ,,
.
' .4it Eo r 2 ,<c3 -b 3 ) . '
2'6.. Acircularplastic roa of radius R has a positive charge +'Q uniformly distributed along one-quarterofits
circumference and a ne1#tive charge of'-{;Qimiformly distributed along the rest of the circumference,
With V =•Oat the infinity, what is the electric potential (a) at the centre C of the circle and (b) at pointP,
,; , _,yhich, is ,on the axi_s of the circle at a distance z from the c_entre? . ,
·· · (I/ 4mo )(-SQ) (I/ 4m0 )(-SQ)
'.,; .!!," ;-· 4.J ~- .. -:,'rt.,,,. ,- •J' , ,Ans, (a) R . , (b) (z2 + R2J112
27..
' '
,Two positive charge;
., - ...
qi and ,j are located at the p~ints with radius vector ij and i'2 . Find a negative
2

,. .... ~.h.arge q3 a~? a ,:adius vector ,Yi of the point_~! whic~ it has. to be placed for the force acting on each_ o_f
,, > 1 o
the three charges to be equal to zero.
'j • I , , ' , • ' , • '

Ans, q3-=-q1q2 !(/q;+.fj;)2, r3 =(lj..{,i; +r2..{q:)!(..{q: +..{,i;)


·,,' ·
28. Point charges q and -q are located at the vertices of a square with diagonals U as shown in figure. Find
the magnitude of the electric field at a point located symmetrically with respect to the vertices of the
qf
square at a distance x from its centre. ~Jr i Jr Ans. E

·- . +q

. ~di! ·. ; . ; '
1 ,' ' ..1~ :: fl ~' I; ., ' 1.' •
+q 1• ',·' . ' ,,
,"', J ':.•ii• !',JC i J " ,,, ,, ( , a '

-q
ir,• t,,
29.. ,.A sy~te!" consists ofa thin ring ofradiusR and a very long uniformly charged thread orienied along the
axis of.th~ ring, wi_th _one ofits end coinciding with the centre of the ring. The total charge on the ring is
equal to q. The charge of the thread (perunit length) is equal to A. Find the interaction force between the
( \ . ; A.
-_ ·ring and the'th,-;;~d. Ans. _q__ .
•· · · 4m 0R
'1'

30. A thread carrying a unifonn charge i...per unit le~gth has the,configuration sho\Vq in figur~.'.Asstjl)ling . a _
curvature radius R to be considerably.less. than the length of the thread, find the ~lectric fi~ld \trength.
at the point 0.

------ 0 ____Q __ _ E=·


Ans.(a) . .
i..,/2 .·
R. (b).£=.Q ..
. , ..
R . 47t,Eo.. . ..

·.__::-:·

32. Three charges ofO. I C each are placed on the comers ofan equilateral triangle.ofsidd .0 m. lfthe energy >
is supplied to this system at the rate or'i.o kW, ho~ ;,,uch time would be required to move ci~e of the.·· ·
charges onto the mid point of the line joining the othenwo? _ . · ,:Ans; 1.8 x t0 5 s .· '. ·
33. Three point charges q, 2q and Sq are to,pe_pla~ed _on~ 9 Cl)l long straight line. Find_ i~{po~ition ~tier:.th~_:
In
charges should be places such that the potential eriergy of the system is minimum. thiS:situation, what ·
is the electric field at the position of the charge' q due to the other two charges? . ' - .
Ans. 2q and Sq at the ends and q is at a dis):anceJ cm· fn,>m_ 2q,,E_"f' o.'
34. A ball of mass of mass 10-2 kg and having a•charge +·3 xl~C is tied at one end oh I·;,, fongthr~iid,.Th~ .- -
other end of the thread' is fixed and a charge -3 x ]()~ C is placed at this end. The ball can ;,,o\ie
i~a '. · -
circular orbit of radius I m in the vertical plane. lniiially, the ball is at the botto~. Find minimunfiniiial
horizontal velocity of the ball so that it will be able to complete the full circle. ·
" .
Ans.. v=7.62m/s. '

35. An inclined plane is making an angle a= 30° with the horizontal electric field of I00 Vim as showri'in
figure. A particle of mass I kg and charge 0.0 I C is allowed to slide down from a height o{ J· m. If the· ·
coefficient of friction is 0.2, find the time it will take the particle to reach the bottom.

Im
µ =0.2

. Ans. t= 1.345s
36. An electron is constrained to the central axis ofthe.ririg of charge of radius R. Show that the.electrostatic
force exerted on the electron can cause.it to oscillate through the centre of the ring with an ·angular

frequency of oo = ~(eq ! 41tc.omR 3) , where q is the charge on the ring, and m is the mass of the electron.'
37. In figure two plastic rods, one ofcharge+q and th.i other~fcharge-q, fonn a circle ofradiusR in an xy ·
plane. An x axis passes through their connecting points, and charge is distributed·uniformly ;,ir 'both ·
rods. Whal are the magnitude and direction of the.electric field E produced ·at the centre of the circle?:·
' ! : ' •' '" - I. . ' . '_ ' • ."

1
Ans. E = - .(-"-)
7t Eo 7tR2 . . .
38. In figure, a pendulum is. hung from the high~r.oftwo large horizontal plates. The pendulum consists of
a small insulating sphere of mass m and charge +q .arid an insulating thread of length £. What is the
period of the pendulum if a uniform electric field Eis set up between the plates by

X
,.r ' .·,
(a) charging the top plate negatively and the _1,9vyer pl_ate positively and
(b) vice versa? In both cases, the field point~ away, 1rom one plate and directed towards the other plate.

Ans.(a) 7'=21t~R/(g-qE/m) (b) T=21t~Rl(g+qElm).


,, - (Jr'

39. In figure, a point charge+q is ata distance d/2 directly


'1,
above

the centre ofa square of sided. What is the
magnitude of the electric flux through the square?
' ,·,::

. .
\ Hint : Think of the square as one face ofa cube "!ith edge d. Ans. q/6e0
I' ..
,.•
. . . .
·~
. .

+ + +
,,_:' + +

..!

40. Figure shows a section through a long, thin walled metallic tnbe of radius R, carrying a charge per unit
length A on its surface. Derive expressions for E• in terms of distance r from the tnbe axis, considering
both(a)r>Rand (b)r<R. Ans. (a) t../21t£0 r, (b) 0
41. Charge is distributed uniformly throughout the voluv,e of an infinitely long cylinder of radius R. (a)
-,::
ShoVI'. that£ ata distance rfrom the cylinder axis(r<R) is given by E = pr /2e0 , where pis the volume
charge density. (b) Write an expression for E when r > R.
lmJc--·E~~~~c~;;;~~~M7~'"'.!';'i< T~~q~?,:~~~r:2~;- ',·~~::1:~::1 1'; ,-~

42, A ,very Jong conducting cylindrical rod oflength Lwith a total.charge+ q is surrounded by;~ conducting
cylindric?I shell (also of length,l)_with ,total charge,,2q, '!_S show11,in,figur~, Find, . , : ,,. ,
/' J.., '" ,1;_1,_ • , • ' • ·~ '.'l/

• p· -

'-f +:+_+ +.+ +-+


+ + ;.!Z+-+ ·+ + +
.+ \~,\,+_;,.
++++++++
',
+· + ·+ + + + +'' +: ·_ii

,,
., ' - ., . ., •/ .-, .:1, . ),·,. 1• I • '~·i
(a) the electric field at points outside the condu~ting shell. .... -, ., l I, .-,_· "
(b) the distribution of the charge on the conducting sheU,'and
(c) -the electric field between shell and rode · •:' ,, .. ,, '•j, ,, 1!H I . ... ,.,,
Ans. (a) E = q l21!£ 0 Lr,radially inward (b)--<J oil both innerand outer surfaces (c) £ = q l21!£0 Lr
43. A large flat nonconducting surface has a uniform charge density cr.Asmall circular hole of radius R has
been cut in the middle of the sheet, as shown in figure. Ignore fringing of the field lines around all edges.
Calculate the electric field at point P, a distancez from the centre of the hole along its axis.
Ans. E=crl2e0 (z 2 +R 2 ) I12
44. A solid -nonconducting sphere of radius R has a non uniform charge distribution ofvoiume charged·
density p = p_,r/ R, where p, is a_constant and r is the distance from.the centre of the sphere. Show that

(a) the total c,harge on the sphere is Q = 1tp.,R3 and '· •

(b) the electric field inside the sphere has a magnitude given by£= Qr 2 / 411£oR 4 •.
45. A nonconducting spherical shell, of inner radius a and outer radius b, has a volume charge density
p = A Ir , where A is ~- constant and r is the distance from the centre of the shell. In addition a point
charge q is located at the centre. What value should A have if the electric field in the shell (as; rs; b) is
to be uniform? ,,
.;, ,• ,•J•·

;/;1 · ,1 1 !i r, .,:J "., ,• ', ( 1'' • I 1·41·•,'

[1, 't ..
-, -; -~ 'J.

" ·' .Ans:'~)21ta2


•·".· j.'_,,' ,· I , , , u·}, ::,,, ·, h-·
46; .What is.the potential at.point P due-to·the_ charge distribution as' given· in the :figure. Rank the situaticiii1
', ' . according to·the 'magnitude of tlie:electric field th'af is set'up at p from-greatest to least.;-, ' l c:'.

•, .'
:,:Q Q

Q~·-... ,.. 0 (FUII angle)


R _::p
y , 120°·.
1 __c;_;;+---'.: I'
I--'--'-'-'

.-··:····· .

I .
.
. '
f'l.

'\ '; 11 \! ,,~ !, I )"..• • (b) • ,1 ." ..• •:,' . ,(c)- , , ·- .


, ,, ft. ,, :.,; , . .,. _. -, • ._ ·1 • • , I

r, .: ,, .. ,· ,_ ..,-.· .- .. , (l/4m;0 )Q., ,.,; . , _ ., ,-,. ___ . .


' Ans. R _ · m each case, Maxnmlrn m (a)Mm,mum m (c)
., .. ~
47: ._ "Aradioai:tive source in the•foim ofa metal sphere ofradiu"s'l o-2 m: e'inits beta particles (electfcins) at th"e·
·' ·. · rate 6'r- 5 x !0 10 particles per second.'The source is electrically insulated:Ho\v' long will h take for its
potential to be raised by 2 volts assuming .that 40 % of the emitted beta particles escape'the'source.
Ans. 700µs.
48._ Three particles, each of mass 1 g and carrying a charge q are suspended from a common point by
· insulated mas~less strings, each I 00 cm long. If the particles are in equilibrium and are located at the
, _C91]1 ~rs 9fa!l e~_ ~j)_ati;_ral tfia~gle of sid~ie_ngth 3 c!", c_a_l~~la_te Jh~,~harge q on each_ pa_rticle. (1:ak,e g = I0
m/_s2). : . . - ·- . -
Ans. 3.162x I0·9 c.
SO. Two identical balls each-of mass 0.10 g and carrying· identical charges are suspended by two insulating
, threads_ o~ equal length from a common point. At equilibrium they arrange themselves at the corners qf
j • , • I
•' ,' ~
', • II• ' , •I ,t • 1 ,
•, ,, • : ,
, • _,, ••
. an eqmlateral tnangle with the apex at the centre of suspension, The distance of the balls from each otlier
·:· '. \ '"is40 cm:
Assuming the -.,,joining
1·_.t,.._ ' 1•,H!!~al,;-_, ,' I','· i. ,_
line
ihe_balls,•.as Iiori~o'ntal,'find't11e
J ,1 , " ; ! _ , • ; I , " " . ; , I• .~
amount•• !
of charge
, -
CJ~ each• b~II.
,
. .·,_,,.,,_ 1
-· , :i _,
. Ans.q=l,91_x.I0· C.
7

SI. A point particle of mass Mis attached to one end of a massless rigid nonconducting rod of length L.
Another point particle of the same mass is attached to the other end of the rod. The two particles carry
charges +q and-q respectively. This arrangement is'hei'd in a region ofa uniform electric field E such
that the rod makes a small angle 0 (say ofabout5 degrees) with the field direction. Find an expression for
the minim~m iime needed for.'the ;od to. become parallel ' to the field after it is setfree. ' '
'· '

'·- · :-·x'. Ml -''

l q ~
~ :.;;"I•
1
Ans.. - L
2 2qE
-'-.
.

52. A' drop of water of mass 18 x 1o-3 g falls away from the bottom of charged conducting sphere of radius
20 cm, carrying it with a·charge ofI0·9 coulomb and-leaving 'on thesphere a imifornily distributed charge-
of2.5 ·x ,JO:f!'coulomb:•What is the speed of the drop after it lias fallen 30 cm ?(4,iE0J- 1,;. 9 x 109 J-m IC 2•
n , , i •h '/ ,,n,i~ Ans:'.3'.658 mis.
·t,

53. Two circular wire loops of radii 0.05 m and 0.09 m, respectively are put such that their axes coincide and
their centres are 0,12 m apart. Charge of 10--6 C is spread uniformly on each loop. Find the potential
difference between the centres of the loops.

A B

R ~!--"'·····
.. ··
... --···
c, ...... · ..·c,
__ .. :-··········'.;:~·-·
,..__d=0.12m__.

Ans. 7.08~ Icr'_v.


54. o:
A charged du.st particle of radius 5 x I 7 mis locatea in a horizontal electric field having an intensity of
6.28 x 105 Vim. The surrounding medium is air with coefficient ofviscosity1] = l.6°x J0·5 N-s/m2 . If this
particle
. .moves with a uniform' horizontal
. ' -
speed . 0.02 mis,
.
find the number of electrons. on it.
Ans. 30.
55. .T.wo equal charges q are.kept fixed at-a and a along thex-axis. A particle of mass m and charge q/2 is
·brought to the origin and given a small displacement along the (i) x-axis and (ii) y-axis. Describe the
motion. in the two cases.

4 3 (-a, 0). q0 (a, 0)


Ans. (i) T = 21t 1t£o,ma · . (ii) This motion is not oscillatory. +--q-~~+------~..,q-x
2q'
•t X

. 56. A circular ring of radius R with'uniform positive chirrge density A per unit length' is located in the yz-
plane with its centre at the origin 0.A particle ofmass m and positive charge q is projected fromi/ie point
P(R..J 3, 0, 0) mllhe positive x-axis directly towards 0, with an initial speed v. Firid the smallest (non-zero)
valµe of the speed v such that the particle does not return to P.
V
+--
R
..Ans. v=
'~A-- g.
· · . 2£0 m

57. . A long charged cylinder of radius a and linear charge density A is surrounded by an earthed hollow co-
axial conducting cylinder of radius b. What is the electric field in tlie space between the two cylinders?
I 21.
Ans. 4-" . - .
' .
•~o .r
58. Three point charges of IC, 2C and 3C are. placed at the. ~omers of an equilateral triangle of side I m.
Calculate the work required to move these charges to the comers of a smaller equilateral triangle of sides
0.5 m as shown in figure.
.. ,
.:·'.".V '•- i ••
'. I . ic
·:

~··

· .2c 3C

Ans. 99 x 109 J.
59. Three concentric ·spherical m;\al ~hells A, Band C ofradii a, b and·c (a< b.< c) h_ave surface charge
densities +a,- Cl and+ Cl respeciively. . .
(i). Find the potentials of three.shells A, Band C.( . .
(ii) If the sheUs A an_d Care ai- ·the same:potential,.obi:<in the relation between th~ radii a, band c.,

cr'.. Cl (a2·-b2 ) cr ("2_.b2+c2) .


Ans. (i) VA =-(a-b+c), VB=- .. +c ,Ve=- · (ii)c=a+b.
Eo Eo b Eo ,, C- .

60. A prot~n moves with speed of7A5 x !05 mis directly towards a free proton originally at rest. Find the
distance of closest approach for the two protons. ·
. . , Ans. 9.94 x J0: 13 m
61. A charge of 4 x I 0-9 coulomb is uniformly distributed overa surface ofa ring-shaped conductor of radius
0.3 m. Calculate the intensity of electric field at a point on the axis of the ring at a distance 0.4 m from the
plarie of conductor and specify its.direction. What is the intensity at the centre of the ring ?
· · at
Ans. E= 115.2 N/C, along the axis of ring, centre E"'0.
· 62. Find the,potential V (x,y) ofan electrostatic field E = a (yi + xj), where a is a constant, i an_d/ are the unit
vect_ors of the x andy axes.
, . Ans. V=-axy+const.
63. Find the potential V(x,y) ofan electrostatic field E = 2 axy i + a(x2- y2)j, where a is a constant, i andj
are the unit vectors of the x al)d y i.xes.

2
· An"Y=~y(y: ~x )+consl.
--··-. -- ----- ------.-----------
ELECTRICI1Y & MAGNETISM - ' ·,.- i
- ----·- - - -. - .. -- -----~-- - -- - ---~- ,._ ·#· ~ --- •.. :. -·-~~-

64. A point charge q is located at a distance r from the centre O of an uncharged conducting spherical layer
whose inside and outside radii are equal to R I and R 2 respectively. Find the potential at the point O if r
<R,.

Am. V = _q_(_I_ __
I +-1-) .
4rre0 r R1 R2
65. A ball of radius R carries a positive charge whose volume density depends only on a separation r from
the ball's centre asp= p 0 ( I - r/R), where p 0 is a constant. Assuming the permittivities of the ball and the
environment to be equal to unity, find : _
(a) the magnitude of the electric field strength as a function of the distance r both inside and outside
the ball;
(b) the maximum intensity £max and the corresponding distance rm.
•' ' I l
· , Por ( 3r ) PoR lbrr<!R:
A11.~. (a)£=- 1 - - t.orr_
-<RE
. =---,
. dE11 .- 4R 12E r- 11
I!,., ,; ;

Ip R
11 - for,- =-R 2
_(b) Emax = - - 111
1
9 Eo •
1
66. A space is filled up with a charge with volume density p = p 0 e_.,,.. • where p 0 and a are positive
constants, r is the distance from the centre of this system, Find the magnitude of the electric field
strength vector as a function of r. fnvestigate thC!' obt~ined expression for the small and large values of
rf r, i.e. at cx,3 << I and crr1 >> I. '

pO · J pr p0
Ans. E (I - e-ro· ) . Accordingly, E = -'-' and E == , .
3Eoru·2 3E 0 3E0ru--
67. Inside a ball charged µniformly with volume density p there is a spherical cavity. The centre of the cavity
is displaced with respect to the centre of the ball by a distance a. Find the field strength E inside the
cavity, ~ssuming the pemiittivity equal to unity. ' .'

Ans. Jeo.
I ap_

68. A soap bubble of radius rand surface tension Tis given a potential V. Show that the new radiusR of the
' ' ..
bubble is related with the initial radius by the equation
- ' ,·
' •3 2· " · •E v2 R
· 1 J P(R 3 ,-r )+4T(R -r 2 )= .o . .I ,l ,; .
2
.wher~ P is the ainiospheric pre~sure.
,,
..
n'

re·
Capeil1tonce end
1ii9acita,1 :!:;-"'

.-
2.1 CAPACITOR OR CONDENSER : . AN INTRODUCTION
l •
~ capacitor i~_an e_lectrical device which .is used to st!,re ,"lectrical energy in the same way that a bucket is a
cp~.tainer for storing water: or a tan~ is a container for s1oring gas. Each of these devices.has fixed cap~city
,vhich does
,-,- -
ncit depend

on1 the quantity

to be 'stored. , .
~ system. consisting of two conductors of any shape, called plates placed at some separation consti\utes a fl
capacitor. When a capacitor is charged, its plat~s have equal and·qpposite charges of +Q an_d-Q. The charge
of capacitor
. . . means the charge on positive
. plate.
'
(Note that Q is not the net charge on the capacitor, which
'
is ...
zero). The symbol that we use to represent a capacitor is --;jf--.
Fig. 2. I shows a capacitor which consists of two conductors, isotated electrically from each other having
charges +Q and - Q.

. ,

.• i . ~ •, ;

,'
,, •, '" ,: .
,,_ .~; ' •. ,, 1--- ,, '
._11, . •'),,

Fig. 2. I Capacitor . •,;~-,,


-
2.2 CAPACITANCE_OR CAPACITY
..
We know that the pote~tfal of a ·charged conductor is proportional to the charge given to it. Thus for any
· capacitor we can write
Q = CV
r·---~,r.,.,..:-
.or Li._£:. .:'.iJ/tr;:2,1 . . .
The proportionality constant C is called the capacitance of.the capacitor. 11s ·va_lue depends only on the shape
and size of the conductors not on the charge or potential difference. The SI unit of capacitance is coulomb/volt
(C/V). It is called I farad (IF), a unit in honour of Michael Faraday.
Why plates with opposite ·charges?

.A B A B
+ + - + +

+ + - + +

+ + - + +
+ + + +
+ + + +

., (a) (b)
· Fig. 2.2
When a plaie Bis brought near a charged plate A, the n_ear face (!f B will have an induce charge (negative) and
.1 far face a positive charge. This negar:ive charge tends to lower the potential 9f A and the positive charge on B
tendscto-raise it. The effect of these charges on A nearly counteract_each other,"but th·e negative charge has little
greater influence as it is nearer to the plate A, thus lowers the potential of A. If the plate'B is now earthed, the
positive charge on Bdis~ppears and negative charge d~~s not go to the earth as it is attracted by the positive
charge on the plate A. Hence the potential of plate A is much lowered due to the presence of the negative charge
on plate B. Thus the potential ofan insulated charged plate be considerably decreased and hence its capacitance
.can be considerably increased when an earthed plate is brought near it. ··
---, .-- ... - -.-- ··,·-·;·---·.·
!/&!#.
i I. By making such an arrangement of-plates, the entire _'electric field and hence electric potential energy
I , • '
: · resides only between the plates, This is the prime "cquiremerit_?f'.the cap~citor,. . ·.
; 2: The plate B need not 'necessarily .be.eartheil:The only requirement is that B m.ust have an equal and'.
j ··. opposite cbarge. · . . . , · · .. ·· · · .. ·· ·" · ·
! 3. :Each isolated conductor has capa~ity, but it ·is'not a capacitor.
~-- - ,~ ; ____\ • -- ..::..i -- -- -- -- -- - ··- -- _. -· ... ---- - - -
Uses of capacitors
I. As we know that capacitor can be used for storing electrical _energy. Energy stored in a large capacitor
is used to accelerate the electrons in 'a cyclotron.
2 Capacitors are very· useful to reduce 'voltage fluctuations in electronic power supplies, to transmit
signals,· to detect electro-magnetic osclllaticins at_ radio_ frequencies -and to ~ther important electronic
circuits.
-
2.3 CHARGING A CAPACITOR

A B
C

"Moving
electron

._..---~-----,I i - . - - - - '
V
Fig. 2.3
Take a capacitor and connect it in an electric circuit with a battery. The energy provided by the battery pulls the
electrons from the plate A and send them to plate B of the capacitor. Due to which positive charge on the plate•
A and negative charge on plate B start increasing. As the pla_tes become oppositely charged, the potential
difference between them increases until it equals the potential difference Vbetween the tenninals of the battery.
Then plate A and the positive tenninal of the battery are at the same potential, and there is no longer an electric
field in the connecting wires. Similarly, plate Band the negative tenninal reach the same potential and there is
then no electric field in the wire beiween them. Thus, with the field zero, there is no further drive of electrons
in the connecting wires. The capacitor is then said to be fully charged, with a potential difference Vand charge Q.
Displacement current
During the charging process of the capacitor, there is an electric current through the connecting wire due to the
movement of electrons. This current is called conduction current. According to Maxwell, there must be an equal
amount of current between the plates of the capacitor. This current which is due to changing electric field
between the plates of capacitor, is called displacement current (id). If a is the charge d_ensity on the plates of the
capacitor, then the electric field between them is given by

. ' (J
E = Eo
a e0 E
If A be the area of each plate, then charge on the plate,
q aA=e0 EA

The displacement current,

d(e 0 EA)
dt

Eo A(:)
di!>,
= Eo--
dt

Thus
-
~ , .:Ei"iii;R,c,n:,&'MAGNETISM-··,. · ~~-
El~.·'~--~---·''''"~-~~----·-".-"~" ·-----""'···..,...;,.,::--,·,.,_. --- ._, --.'.' --
·.· .f.··.:.~r:_':-···_. ?:~."':ct;.,:;:1r.·(':l --·-· ',-"

2.4 CALCULATING CAPACITANCE ,J .r,, . y<"',, :\ , , _ :, ,_ .:) f~ ;:,


Step I · First propose any charge Q on one of the plates of the capacitor.
Step II Find potential difference between the plates of the capacitor. keeping other plate eaithed.
Step Ill To find capacitance, use

C = Q
V
Capacitance of isolated spherical conductor
Consider a spherical conductor of radius R. It is given charge Q. The potential of its surface

.
, _.,
'n., .. .. ' ., ',l
I ' ,' ' :,• . R
Q ': ·•:Q· ' ,,
",r , :. Capacity ,, C
v= _'_I_Q.
Eo R
47t ''
·' ·C 41t_e0 R:· · ·:
,, (,• ' ' ' ' • '! r
Radius of IF isolated conductor
Putting C = IF in above formula, we have
•!. J' • ; '. ' .. / 'i. I • ;•

41t~~ ii.
' ·'. 'l - :.
.,
,R ' ,.
.· 41tE 0
',.: ,,
= 9x I09 m,
This size is more than 1000 times the radius of the earth. Thus a capacitance of IF in practice is not possible.

Ex. 1 Two conductors carrying equal and opposite charges produce a non-uniform electric field as
shown in jig. 2.1. What is the capacitance of the capacitor if the electric field along y-axis varies
as · · ··.] · · :· '·' · '- ·· !'

.E. = g_(i+by2)
e0 A .
where b is a constant. The separation between conductors along y~axis is d.
y

Q_ B
f '
...........T...
d

Fig. 2.5
r----.,;-.----------------.======-- --~-_E~A'cITANCE ~Df?M'ACITOR

Sol. The potential difference between the conductors is given by


B

V v-v=fEdy
A B
A

d
fl(l+b/)dy
Eo A
0

Q I
--y+by
Eo A -3-o
lid

_g___(d + bd3)
Eo A 3

.,TM capacitance C Ans.

Redistribution of charges
If a conductor of capacitance C 1 at potential V1 be VI V2 "
connected to another conductor of capacitance C2 at
potential V2 , the charge flows from the conductor of
higher potential towards the conductor at lower
potential until ihei~entials become equal. Thus
common potential
i.
Conductmg
wire
Total charge
V total capacity Fig. 2.6

Q,+Qz
C1 +Cz
r--·--·-· .... ··-·.
or / V = C11'i +C1V2 [ ... (I)
l._ ________c,_~ c:_2
__ ____ ,.~
If Q'1 and Q'z- are the final char~es on them, then

/
Q', .s_ ... (2)
Q'z Cz
. ' ··:, ...-·
Also Q',+Q'z Q,+Qz ... (3)
...

In the process of flowing of charge, ,there becomes lcls's of.some 'electrical energy'due to.the resistance of the
1
flowing medium (wire) which will convert into heat and light.Thus: ·

Loss of electrical e~ergy F , U;- u,.

or u, \~
= [ -C11'J
I 2 +-C
1 V2] - -
2 2
I [C1 +C2 ] V
,2
L 2 2 2 .

and

Analogy with mechanical


When two containers having different levels of water are connected by a pipe, water flows from the container
having higher level towards the conta-iner having water at lower level, until levels in both the containers become
·'. 0

fJ c::ih. '.. '


1
equa. : , · '
~- I ,: ) '

HI' i •

ii

(a)

Fig. 2.7
After redistribution, the container of larger capacity will have large amount of water. Here also some part of
mechanical energy will convert into heat by1h~'frictipn (resjstance) of the connecting pipe. . .
+ (', - \, I .

2.5 CAPACITANCE OF PARALLEL PLATE CAPACITOR


-Q

It consists of two metallic plates of any shape but of equal size and placed at some
separation. This c6nstitutes a parallel plate capacitor. Suppose a cha~ge Q is given
to plate A; it will induce a charge - Q to the qther.plate B, which is earthed. If d is .
the separation between the plate, then potential difference between the plates
111:..,v,1'.!~i 'Ed;·· .. --· .' ,_,
'
-
'

O' ' ' ' ,J'd-l·~Q.·


.::,
Here£= - . lf Ais the area of each plate, then cr= -A.
~ -
i - I ,
'
'
' A _B _L
1
l --/,.:
V
(Q/A)d
Eo
.
1 ~ I
Fig.i.8
. '.-
·.I,! .,, ,r.'.. i
I
Capacitance
. r ,1.

or . .. (I)
I,:,

Effect of dielectric
Above _obtained expression for the capacitance holds only when the plates are in vacuum or air. Michael
Faraday, first investigated and found that, when a conductor is ·charged in the presence of dielectric, the charge
on th~ Conductor i - •

· [Q]in.dielcctricmedium =·- k[Q]~i;,<ReepingV constant.


Thus ... (2)

If there are n identical plates at equal distances from ,each other and the alternate plates are
. .. ' ,\l

connected· together, then the c~pacitance of the arrange~~nt


· . WI·11 be C.= (n- I) d
Eo A.

Capacitance of parallel- plate capacitor 'with a compound dielectric


On introduction of dielectric in the space between' the pl;;t~?gj- a capacitor, there is decrease in the potential
difference between the P,lates. The potential difference between.the plates Emed
· V V,;, +Vm,-d Q -Q
,,, -·,
E,;, (cl- t) + E med' t

r1•
[~:d-i)+~t]
Eo ... -,,
~· '' .'
k Eo

Capacity ·c Q: ='' ,. _ crA

V [!!:_(d-t)+_<E_]
Eo kEo
. . .,:,'\

or. Fig.2.9

Other cas·es /

· - , r1t1 ,.t,., "e·1>t;,t


1. If a metal slab is introduced, then k= =· ana_~,;"(/-}\ . _
-.:)1 Ii ,(
2. On introducing the dielectric slab, the potential difference between the plates falls ·and the capacity is
. increased. Hence to have the same capacity as before; we 'must increase the distance between the plates
' • ,r I •
\ by . " - . t
- (_}
.
;1::,dt ' ,·

k-1\
X ( k ) . ''
·1 .
3. If there are two dielectrics k 1 and k2 of thickness 11 and 12 respectively, then

C = Eo A

r
I~.,., [d:.(/1 +l2)+.2_+2-].
k1 k2
·.., [d . . ' .
4. If dielectrics k 1, "2, .... of thickness 11, 11, 0 .. respectively are filled the space between the plates, then
'"'· e0 A

--.
5 = ·[21 +~ . . . . +;:,]
5. If unequal charges are placed on-the plates, the~ charge on the right face of the plate A
, '. ·'
QA. =
\
[Q1~~2] {?I {h
VAB "= Ed Q, QI 0_, Q,
\,
2 2 2
x
2

;'
= ~4
, l
0

(QA I A) d
o, - Q, u
- Q, +-=cl
+=
2 2 2 2
Eo A B
'
or VAB = (Qi-Qi)d
2e A
Fig.2.10
0
Capacitance does not depend on the charges, and so it remains constant. \
Leakage current ,·
Dielectric is not a perfectly insulating material. So when the space between the plates is completely tilled with
a dielectric, there flows some charge through the dielectric ·medium. The flow of charge per unit time through a
dielectric medium is called leakage current.
Consider a dielectric of constant k and resistivity p,· The capacitance
!<'eo A (
C = d

. · 1. pd
and resistance R = -,
' A
fr Vis the potential difference bei'ween the plates,_ of capacitor, then current
. I
.__,.v....--~
I V
/ i = Fig. 2.//
R

V VA
7= pd
p-
A

_v_(ke A) __!:£_
ke 0 p d
0
ke 0 p

or
l .I :,1: . . VC
~'·._,_·_=__k_EQ__ p__J
7
2.6 ENERGY STORED IN A CAPACITOR
When we stretch the spring, the elastic potCntial energy will s~~re in the spring. On the similar way, when we
charge the capacitor, an electric potential energy stores in it. This:energy can be recovered if the capacitor is /
allowed to discharge. If the charging is done by a battery, the electrical energy is stored at the expense of
chemical energy of the battery. In charging the capacitor the work is done by the external agent against the
repulsion of the charge which has already gone to the capacitor. This work done will store in terms of electrical
potential energy.
Suppose at any time I, charge q has gone to the capacitor. If C is the capacitance, then potential
' I
q
V = c·_lt. ·J' ,'

If dq amount of charge is brought against the repulsion; then w?rk done


. q ''
dW = vdq= -dq
c.,
Total work done in charging the capacitor to a charge Q
Q
w = fdw=f~dq
0

Q2
or w
_2C

· Energy silired in tr capacitor


\
U
---
'
1 ~cv 2
~;-=----~-Q_V_=__
2
l [v = ~]....(!)
I

This relation is analogous to the elastic potential energy stored in the spring i.e., U = _I_·i;,2 :· -
. ~I 2
Consider a parallel plate capacitor of area A and plate separation d. The electric field is uniform in the region
between the plates and the fringing effects a're neglected. Energy. stored in the capacitor

u = ,'..cv 2
2
_i_EoAv,:2
2 d '\

I (Vr..
- Eo -
2· d
{Ad)

or u -IEo 2 E · x volume ...(2)


\ 2
u 1 'l
-E 2
or
volume 2
0E

... (3)

... (4)
, '' ~ -
I. Above relations are derived for a parallel plate capacitor, but they are true for others also. If

. ;lectric field,E is not unifonn in the total.volume; the~ we ca~ write

2. If a battery is used to charge the capacitor, and Q is the charge delivered by


' -:;\ f1 " ' . ,

C:J
it, then work done by the battery '
W VQ.
But the energy stored in the capacitor
S---;-----I V
Fig. 2.12

The remaining energy= VQ-..!.Qv =d-'QV 'is lost ash.eat and light. Thus we can write
2 .,,;l • I

wextemalsource:, = 2 u.
\ .

2.7 FORCE BETWEEN THE PLATES.OF A CAPACITOR

Methodl: Consider a parallel plate capacitor with plate area A. Suppose a charge Q is given to one plate and
-Q to toe other plate. The force on_ p~a(e B du_e to plate A ( or vice-versa)
FBA ,,.=c. .§A QB.
The.electric field at the position·of plate B due to the plate A is

E -
I
A
~=_Q_ ___,.
2Eo 2Eo A
En . ~:i

_Q_xQ
2Eo A A B
' '
Fig. 2.13

. Q2 .
or FBA=FAB "' F=-.-.. -·- I ...(!)
l. --·- -·---· ___ 2Eo A . i
if Eis the electric field between the plates, then

:I,,. r r}I ,., ., 1 .' 2


-· ,.F,,, = - Eo £ A ... (2)
..I ·". '. ·2
----,.,, ,lr-,-.,-- - ~ ~ - - -
Methodll: The expression for the force can' also be derived by energy method. Suppose Fis the force of
· '!Hr ! •
attraction between the plates. If x is· the separation between the plates, then
.·T€APAcrrANc1:•ANb::€APAl:!11oil'·,;,]Wr.a
- - '"· <::-,:--,.,-.•• ;:··"·· ,·-"'<':HY
·" ';);,.·_._." ,- -
-
l,_•l..j.l •

I 2
u 2 0- E
-E ,..__ _(Ax)
,. ~

. 1 G 1cil J
,'
.du ,,.!.._Eijl~2,A
and
dx 2 F-41---+,•
''
= '
,.:. dU
''
By the definition 'F
dx X dx.a'
j! ,.. r , l' lOJIZ
= --E0
2
EA
.
Fig. 2.14

I ' 2 .
I. Fo; isolated capacitor with constant cha;ge, F= _g__:_i_, This force does not depend on the separatio~
. 2E 0 A· i
between. the plates, and s~:fhe con~tant amount pf f(?~cJ fs needed. Ii

2. For a capacitor having constant p.d. across the plates, lhe force ··

F = (cv_f":_'(Eo A)2~=.!.Eo v22 A



. 2Eo.A-,
I •.J, e,~t
'-
d 2E0 A 2 d 1
In this··case force depends on the separation (d),;between the plates. Thus if we change the separation
now, the variable force is needed. · - ':.J· 1, 1 •
.\ ·.1~:1, ·'

Ex. 2 A capacitor has square plates, each of side a maldng an angie8 with each other as shown injig.
2.15. Show that for small 8, the capacitance Cifgiven by

C
· =
'
E0
d
a 1_a8)
2

2d.
(

~' .... ........ ,

' I

,...._____ a~;-
Fig. 2.15
Sol. The given capacitor may be supposed to be made up of large number of differential capacitors.
a
·· Consider o~e such capacitor of width dx at distancex from 0. The area of the element capacitor.
dA = adx. If d is the separation between the plates at the position of element, then
' · l,;w:
d = d+xtan8=d+x8
. ~ .1- '
\
The capacitance of the.element

Eo(dA) E0 (adx)
dC d' ,(d+x8)

dC
E0 (adx)
d(1+!)

'[E; (adx)(J+ ~rl]


[E; (~dx)(1-~)]
The total capacitance C= ][~(adx)(1-~)]
0

. Eoa
--x---
0x
20

d d 2
0

~[a-0a
d 2d
2
]

Ex.3 . '
Two isolated metallic solid spheres of radii Rand 2R are charged such that both ofthes_e have
same charge densitycr. The spheres are located far away from each other, and connected by a thin
conducting wire. Find the new charge density on the bigger sphere.
Sol.
Method I: Charge on the smaller sphere Q1 =crx411R 2 = 411R 2cr
2 2
Charge of the bigger sphere Q2 = crx411(2R) = 1671R cr
If Q ' 1 and Q '2 are the charges after connected them, then
'Q'1+Q'2 = Q,+Qz
47tR2cr+ I 67tR2cr
or :Q' 1+Q 12 = 207t/l2cr ... (i)
There potential becomes equal, and so
'
LQ'1 _ _ Q'z
4f01E6:
.:,;·"·.
R 411 Eo 2R
or ·Q',,-. 2Q'1 ...(ii)

"'
Solving equations (i) and (ii), we get
20 R2 Cf
-lt
• 3

and
:. Surface density of bigger sphere
40i tR2 Cf
-
Q'2 3
dz =s 4it(2R) 2 4it(2R)
2

5
-0" Ans.
/. 6
Method II: After connection, the common potential

\
5 Ro-
3 Eo
The new charge on bigger sphere

C2V=41tEo (2R)x -
3 Eo (5Rcr)
'
I
= 40 1t~20" Ans.
3 \ .

Ex.4 Two parallel plate condensersA and B having capacitances oflµF and SµF are charged separately
I to the same potential of 100V. Now the positive plate of A is connected to the negative plate ofBand
negative plate ofA to the positive plate of B. Find the final charge on each condenser and total loss
of electrical energy in the condensers.
Sol. The charge on condenser A, Q1 C1V = I x I 0-6 x I 00
\
=, 1oox10- c 6

The charge on condenser B, C2V=5XI0-6xl00

soox10-6 c_
Qiil-Q'
. + - .
' --1?..j ~ ·_ ./
(a) Before conne,:tiqn ·
Fig. 2.16
I
If Vis the common potential of the plates, then
Q1+Q2
V = C1+C2

IOOxl0--6 -500x!00--6
= Ix I o--6. + 5 x I o_--6

or IVI = 200 V
3
If Q ' 1 and Q '2 are the charges after connection, then

200
- µC
3

and

• Initial energy on the condensers


= 1000 µC
3
Ans.

.!..x1 o--6 x1002 +.!..xsx Jo'-6 x1002


2 2
0.03J

Final energy f(c1 +C2)V 2


,, . -
½(10--6 +5xJo--6)x( ~
2 0
r
0.04 J
3

0 04
Loss in electrical energy 0.03- ·
3

(o-;s)1. Ans.

Ex. 5 A conductor is charged from an electrophorous by repeated contacts with a plate which after each
contact js recharged with a quantity Qof electricity from the electrophorous. If q I is the charge of
the com\uctor after the first operation, calculate the ultimate charge.
Sol.
Method I: Suppose C1 and C2 are the capacitances of conductor and electrophorous respectively. On the first
contact, ihe charge taken by the conductor is q, ·and the charge left on electrophorous plate is
Q-ql. I •
Since in contact, the potentials become equal, so
Q-ql .'Ii.
=
C2 C1

QC1
or qi = C1 +C2
In the second contact, let q2 be the charge taken up by the conductor, then q 1 + q2 becomes the
total charge over the conductor and the charge left over C2 is Q - q2 • Again
Q-qi qi +qz
= q
Cz

q1 c1-'
=-
qr
or q2
(C1 +C2)2 Q

3
Similarly the charge on the conductor on the third contact is ~ and so on. Hence the ultimate

charge

Ans.

Method II: We have got _q_l- =.9_


Q-q1 C2
Let qm,x be the maximum charge which can be given to the conductor. When potential ofconductor
becomes equal to the potential of the electrophorous plate, thereafter no charge flows between
them, so
VPlate VConductor

g_ qmax
Cz C1

=> qmax .9_Q= q,Q . Ans.


C2 Q-q

2, 8 CAPACITORS IN SERIES AND PARALLEL

In some circuits, there are many capacitors connected together, w.e can sometimes replace that combination
with an equivalent capacitor. Here we are discussing tw? ~asic combinations of capacitors.
,
Capacitors in series
.·'.
, , ,+Q f--o___
, _+,,...fg -cQ , +$(1 ._.a.______, +Q
(

. c1 c~ 1 C}
V1 ---+-V2 --+1---1::,

L--------+ I - - - - - - - - - ' --~+c; ...- - - -


v
V
(a) Three capacitors connected in series. (b) The equivalent capacitor.
Fig. 2.17
Fig. 2.17 shows three capacitors connected in series with a battery. In series means that the capacitors are
connected by a wire and there·is no other way of flowing ofcharge. When a potential Vis applied across them,
the same amount Q of charge will appear on each of the capacitor. The sum of the potential differences across
all the capacitors is equal to the applied potential difference V. Thus, if V1, Vi and V3 are the potential differences
across the capacitors C1, Ci and C3 respectively, then
V - V 1 + Vi+ V3•
I ,
Here Vi ; JL, Vi ; R and V3 ; 11_. If C"'I is the eqi(,i\,alent capacitance, then
· C1 C2 C3
\ V ; Q!C. .. .

'' Q lL+R+R
C Ci C2 C3

or ...(I) .

For n-capacitors, we can write

,...(2)

Cap~citors in parallel
I
,I

+Q
c,,
=>

.___+'-! t----...
V
I.
Fig. (a) Three capac· ors are ·(b) The eqmvaie,ii capacitor.
connected
, in arnllel
, ,, , Fig. 2.18
\ /,,
\
'
Fig. 2. /8 shows three capacitors are connected in parallel with a battery. In parallel.conn~ction, one plate ofall
the capacitors are connected to the wire through the positive terminal of the battery and similarly second plates
are connected together through negative terminal of the battery. Thus, each capacitor in parallel has the same
potential difference V, which produce~ c_harge on the capacitor. If Qi, Q2and Q3 are the charges on the capacitors
C1, C2 and C2 respectively, then
Q = Q1+Q2+Q3_
Here Q 1= C1V, Q2= C2Vand Q3= C3V. If C,q is the equivalent capacitance, then total charge Q = Ceq V.
or C,qV C 1V+C2V+C3V
ceq c 1 + c 2 +c3 ...(3)

n
For n~capacitors, we can write ceq ~C- ...(4)
£., '·
i=I

Ex. 6 Calculate the capacitance of a parallel plate condenser, with plate area A and distance between
plates d, when filled with a dielectric whose dielectric constant varies as :
d
e(x) = Eo +6x O<x<-
2
\
e(x) = e 0 +6(d-x} f<x<d. - \ \ ·
For what valueof pwould the capacity ofthe condenser twice that when it is without any dieli,ctric ?
Sol.

\ \ -
=:::::::1--/:--t:::='------' I \
X lX
i-- d/2---dt2__,_,, ,
Fig. 2.19 \
The:given capacitor is equivalent to two cond~ns~rs in series. IfC1 and C2are their capacities, then

~ = J[ d~1 + d~2] ...(i)

Take an element of width dx at a distance x from the left plate. Its capacity
(eo+fh)A
dC 1 x5a d/2
dx

and
I dC2
(eo +P(d-x))A d
-<x5ad
dx 2
~:,~t~~~~~5~iiiji~,Ix~~~~i;2=~~=ir~J
Substitut,i_ng ,th,se' values in_ equation (i), we have , ,

,, ~
I
C ·.:[Ii."~~),1~ . t,'.:1]
'I ' '
~p [{ en(~o +lh)}i-{~:n( Eo ,+P(d-~))}%]

;p [{en( 0+Pf)- en 0}-{In


e e eo -en( eo +Pf)}]
t ' < / • ; ,
, I

·':p[en(Eo +Pf )-en Eo]


'J'"
., >' ,,, ,~' dl'e
= -
::r,~{~P,f)-,.: ,' ,..
n
' -~11,

. ';.tns.
AP Eo
The capacitan9e C of a"condenser wiihoufdiclect'ric is

. C' = Eo A
' ' , d
According to the given condition
'. 1:~ ., " .,. . -,c - t C.
or
C 2C'
2 Eo+-
or -·-en ( Pdl
2 d
AP Eo 2Eo A

or p 4Eo en ( Eo +p~)
__ 2 . Ans.
d .· Eo

Ex. 7 . c,1culate the capaci_tance of a c~pacitor filled ;with t_wo diel~ctrics of same dimensions but of
dielectric constant k 1 and k 2 respectively.

(a) - (b)
' Fig. 2.20 ·
Sol. (a) The arrangement can be assumed as two capacitors in parallel. If~ is ihe separation andA is
A
the area of each plate, then each capacitor will occupy area - . If C 1 and C, are the respective
2 -
capacitances. then
C = C I +C2 ·.

Eo k1 (1) Eok2( 1)
d + d

Eo(k1+k2)A
Ans.
2d
(b) This arrangement can be assumed as two capacitors in series. Each capacitor will be of area
d
A and separation - . If C 1 and C, are their respective capacitances, then
2 -
I I
-I -+-
C C1 C2
d d

= _2_+_2_
Eo k A Eo k A
1 2

2EoA( k1k2)
or C d k1+k2 . Ans.

Ex. 8 A capacitor is formed by two square metal-plates of edge a, separated by a distanced. Dielectrics·
of dielectric constant k1 and k 2 are fdled in the gap shown in jig. 2.21. Find the capacitance.

i-------- a-------.;
Fig. 2.21
Sol. Take a small element of the plate oflength dx at a distancex from the left end of the plate. lft 1 and
12 are the thicknesses of the dielectrics of k I and k2 respectively, then
i •
.<
d(a-x) xd
-'--'-- and l2 = - ·
a . a·
The capaci!aryce of el,elJ]ent capacitor. ,, , ..... ,·~

Eo (dA)
dC =
•. r: , ~'

Eo adx

-Eo a 2 dx

[-a-x
- + x]
,.... d
,, -
k1 k-;.
\ • ' I'

dx
The total capacitance CI

. [-a-x
- + x]
-
k1· k2

\\
\ _\
I
!: ..;·. '
'·,._:..- J! • 'i ,; ,: l.

1'

Ans.

" .
Ex. 9 A parallel plate capacitor is constructed using three different dielectric materials as shown in jig.
2.22. The parallel plates across which' a potential difference is applied are of area A and are
separated hy a distanced. Find the capacitance across A and B.
2k
A k B
4k
-d/2 d/2-
Fig. 2.22
Sol. If C 1, C2 and C3 are the capacitances of the parts with dielectrics 2k, 4k and k respectively, then

(2k) Eo zA 2k Eo A
c, d d
C

A
(4k) Eo -
C2 - d
2 4ke 0 A
d
2C

ke0 A 2k Eo A
and C3 C I
d d
2
Here C 1and C2 are in parallel and C3 in series with them, so
I
---+-
C1 +C2 C3

c1 +c2 +c3
(C1 +C2)C3

C+2C+C 4C
(C+2C)C 3C2

4
3C
3 3 (2k Eo A)
or -C=
4 4 d
~ ke 0 A
Ans.
2 d

·Ex. 10 Find the capacitance of the following combinations between A and B.Area of the each plate is A and
separation between nearest two plates is d.

(a)
>
B
• •
A

< (b)
> 13

Fig. 2.23
~~""-"1
.,,,.7;,•7";<~·~-:- - - - - - - ,

- - ~ - : · --------~-~..±:J
Sol. When A and Bare connected to the positive and negative terminals of the battery respectively, the
charges will appear on them as :

I
- - - 0
~
0
Ai"--<
2- -
__________>::,--<>•B
\!) ., _ _ _ _ _

]-_-
A -~QB
+ + + + +
4-:---- . ) ~

(a) (b) .
Fig. 2.24 J ' • f.1tt·? •

(a) It is clear from the figure that there forms two capacitors which are in_ parallel. Thus
-Eo A
C =2C=2--
AB •d 2

+-
~--·93 1-'4~-~
It "· u · -, (a) (b},

Fig. 2.25
i. , • • • e
0 A
(b) There are three capacitors in parallel. Thus CA 8 =3C= 3 d
n.1:

Ex.11 Four identical metal plates are l~cated in air at equal distanced from one another. The area of each
plate is equal to A. Find the capacitance of the system between points A and B if the plates are
interconnected as shown in fig. 2.26.

2 -- +- .,,oA
+ + + + /
3+ + + + +

4-t:_ ~ ~ ~ ~/B 4 •-• -~-"'_.,.'--on


(a) (b)
Fig. 2.26
Sol. (a) A and B are' connected to the positive and negative terminals of the battery. The charge
appear on the plates are shown in figure. The equivalent arrangement of the plates is shown
in figure (a). There forms three capacitors.
- - --~--

A'~~_,·.,_.;;
~µh.
- 2 3
(a)

Fig.. 2.27_
. '
If C is the capacitance of the each capacitor then

(2c)c 2C
CAB
2C+C
=- 3

~ Eo A
3 d
(b) The equivalent arrangement of the plates is shown in figure (b ). There forms three capacitors.
If C is the capacitance of each capacitor, then

C ' 3C
-+C=-
2 "2 -'

Ans.

Ex. 12 Five identical conducting·ptates 1, 2, 3, 4 and 5 are fixed parallel to and equidistant from each
other as shown in jig. 2.28.flates 2 and 5 are connected by a conductor while I and 3 are joined
by another conductor. The /unction of! and 3 and the plate 4 are connected to a source of constant
emf V0• Find :
(i) The effective capacity of the system between the termin_als of the source.
I
(ii) The charge on plates 3 and 5.
Given d= distance between any two successive plates and A = area of either face of each plate.

5
4 ,+ + (-,)

-...__ _::_2_-=-:
+

>----<.:.)
Fig. 2.28

In solving this type ofproblem,first mark the charges on the plates. It should be remembered that:

(i) The plate/plates connected to the positive terminal of the ballery will have net positive charge and
similarly for the negative terminal.
(ii) The plate/plates which are not connected to any terminal of the ballery will have induced charge.
The net induced charge of the plates t~gether must be zero. ·
Sol. The equivalent arrangement of the plates is shown in Fig. 2.29. From these five plates, there forms
four identical; capacitors. These are C12 , C32 , C34 and C54 . The capacitance of each capacitor

e A
0
C= d.

...
' '-1 l }}

+ -
~~·"'i3 t,,4;,-----'---~
C34
Fig. Z.29
Capacitors C 12 an~~C32 .jire i.n parallel and so their cap?citance becomes 2C. Now 2C is in series with
ir..•..• ·• 2C ·
capacitor C54, ang_~p ~lluivalent capacitance becomes .And this now is in parallel with C34•
3
Thus effective ca~i/c!ii!.nce between the terminals becomes
2C 5C
C
eq
= C+-=-
'3 3

·t,•

a+-
_,
~--,;,1----~~

{ ! Q.{. L.-'-'"-;_ _ ____c+'-j ~ - - - - '_ '_ .-...J


Vo
Fig. Z.30
Charge on the capacitor C34 :
· 'l · : •· I Q34 = CV0 •
Charge on the capaci!9rs C540 C 12 and C32 :

Q54 = 2C V.o
3
This charge equally divided on to C 12 and C32 , so

Q12 = Q32= Q= c;o


CV0
The charge on plate I : +Q12=+-3-

The charge on plate 2 :


·101...
'
The charge on plate 3 :

4 4 Eo AVo
-CV.o
3 . 3 d
The charge on plate 4 :

5CV0 5 Eo A Vo
3 3d
The charge on plate 5 : Q5 = +Q54
2CV0 2Eo AVo
Ans.
3 3d

Ex. 13 The distance between the parallel plates of a charged condenser is',iba 5 cm and the intensity of the
field E = 300 V/cm. A slab of dielectric constant k = 5 and 1 cm wide is inserted parallel to the
plates. Determine the potential difference between the plates, before and after the slab is inserted.
If the slab is replaced by a metal plates so that the final potential di!ference remains unchanged,
what be the thickness of the plate?
Sol. Potential difference across the ,plates of condenser without slab
V = Ed= 300x5=1500V
e A
0 0 e A
and C --=--
d 0.05
After the slab is inserted, the capacitance
e0 A J e0 A e0 A
C' = I (0.05-0.01)+ O.OI 0.042
(d-t)+-
k 5
If I" is the p.d. now, then for isolated condenser
CV = CV'

eo A)~150~- . '
CV ( 0.05

C'= (;.~~)
= 1260 V Ans.
Let x be the thickness of the metal plate. Now the capacitance
e0 A
C" = (0.05-x)
If I"' is the p.d. now, across the condenser, then
C'V" = CV

eo A)x1500
CV ( 0.05
V"
C" Eo A
0.05-x
\

or 1260 o.o5 -x)x15oo


( 0.05
After simplifying, we gel x =0.8 cm Ans.
I
''
2.9 SPHERICAL CAPACITOR
It consists of two concentric conducting shells. Let a and bare the radii of
the shells (b > a). If charge Q is given ,to the inner shell, an induce charge-
Q will appear on the inner surface of the outer shell. Its outer surface is
earthed. We know that the electric field inside the inner shell is zero and
between the shells it is due to the charge of inner shell. Thus electric field
at a distance r from the centre of the s_hells,

I Q
E - =-- - 1t e---2- . - _ . (.a <r < b) __ fig. 2.31-. ... ______ .
. ·.::,. ,4.. _._.-:° r ·.·,-:1 /,,.._.:.···,1::.·.: .. ·:.0 --, ;;1 :;'.:.;
. _ . :.. :.,~, ,. : 1- , ·- .. •; .. i-1: ·:·· :a.: i ,, .' ;._. ~ r:1; ~ . . ,i
The p.d. be~een inner and outer.shells;, : .' ,II 1-= Ya~ vb:;;:~J Edr,r,,; .. ,_ · . ,,· ;· ,,_: ; -. :·'
1
. ·.::, .I · 1iiL.. ~ • ·•'-',C!)f:.,.::'l~f}'l.:t•' •,.' .:· J,•,•·;:L~;'!~.b.··: :.,:1:· ~:;,~' '' ."i;.,:i
l I ,• ~ • !.!. j I~ °"':j. :') J '. " "; I :
b
. . I dr. Q
b ..... ,;'1,
r!J...:J. .:;J

I
a
Edr=-- -
41t e 0
a
r2

Q (
41t e 0 a
I I) b
,,.
.!' '·

Capacitance ., C = Q
V
Q
-~.
~.' ·:~ltQE~ ( ~-f) '

or C = 41tEo ( ~ ) -
b-a

' .
Capacity of an earthed sphere enclosed by a concentric spherical shell
Let a is the radius of the sphere and band care the inner and outer radius ,J_
0-
..,
O
--1
of the shell. Suppose the charge Q is given to the• ciuter'shell B and 'the .-,,
induced charge on the inner sphere is -Q 1• Thus + QI charge will appearon
the inner face of the shell Band remaining charge Q-Q 1 will appear on its
outer surface. In this way the system works as the combination of two
capacitors :

(i) that between sphere A and the inner surface of shell B, with capacitance

· cI =
.
,41t e0 (_!!!!___)
q-a B

(ii) and between outer surface of shell B and infinity, with capacitance Fig. 2.32
C2 41t e 0 c
:. Total capacity'' C C 1 + C2 ,_,

4ite [~+c] 0
b-a
Special cases :. -; ; ,.,
:.,....
I. For thin shell c=b, :. C~_;;ltEo [..!!!!__+b] =47tE
b-a O
(.±-) .,
'l,'-'a
' .... • ~ ' ' ,, - , l ' •• J ,

2 As sphere A is earthed, so VA = 0,

or = 0

Ex. 14 T~ree concentric thin spherical shells are of radii a, b, c {a< b < c). The first and third are
connected by a fine wire through a small hole in the second and the second is connected to earth
through a small hole in the third. Show that the capacity of the condensers so formed is

,, [ ab c2 ]
47tEo - - + - - .
b-a c-b

Sol. If charge Q is given to the outer most shell C, then some of the charge will go to the outer surface
of the shell A because they are connected together by a wire. The charge Q 1 on the inner shell will
induce a charge - Q 1 on the inner surface of the shell Band+ Q 1 on its outer surface, Shell C also
,,, induces the_ s)lell B, s9 -net charge on 'its outer surface becomes different from QI' let it becomes+
, Q2 . Thus charge on \nner surface of shell Cbecomes-Q2 . In this way three condensers are formed,
one by the outer surface of A and inner surface of B, second by the outer surface B and inner
, surface of (:'and the thirp by the outer surface of Cand infinity. IfC1, C2 and C3 are their respective
capacities, then

. 47t Eo (..!!!!__),
b-a

~ltEo ( ~ )
c-b

and
The total capacity

' ,, - Fig. 2.33

Ex. 14 (a) Find the cap~citance of spherical capacitor having co~ce11tric shells of radii a and b\
(b >a). The space between the shells is completely filled by a dielectric of constant k.
(b) If dielectric is filled upto radius c(a <c <b) between the shells.
c = 41te 0 k(_!!!!___)
b-a
(b) The given capacitor is equivalent to two capacitors of capacitances C1 and C2 in series,
where
ac
41te0 k - -
c-a
be
and C_, = 41t Eo - -
b- c
If C is the total capacitance, then
I I 1
-·-+-·-
C c, C2

or C
c,c2 Fig. 2.34
C1 +C2
On substituting and simplifying, we get

c=[ kabc]
ka(b-c)+b(c-a) ·
Ans.

Ex. 16 If half the space between two concentric conducting spheres be filled with dielectric of dielectric
constant k and the rest is filled with air. Show that the capacitance of the capacitor thus formed
I
will be same as if the whole part is filled with the dielectric of dielectric constant_ 2( I+ k) .

Sol. Suppose CJ is the charge density on the inner sphere and air in between the spheres of radii a and
b. The p.d. between them ~--- cr

2
CJX41t0 [.!.__.!._] = cra2 [.!.__.!._]
41t e 0 a b- e0 a b

k
Capacitance C Fig. 2.35

. 1
When the medium is filled with dielectric of constant (I+ k), its capacitance becomes
2
l+k
--x41te [ ab ]
C' 0 --
2 b-a
Now ifhalfthe space is tilled with dielectric of dielectric constant k and half with air then in order
to maintain the same potential difference between the spheres, the surface charge density over one
half of the inner sphere will become kcr and over the other half remains cr. Thus

en = Total charge
p.d.

C'.

2.10 CYLINDRICAL CAPACITOR


It consists of two co-axial long cylindrical conductors. Let a and b be the radii of inner and outer cylinders
respectively.
If charge Q is given to the inner cylinder, an induced charge-Q will
appear on the inner surface of the outer cylinder. The outer cylinder _Q Q+[··_-_-_-_·_· .·.~_-:··,+
is earthed. Suppose length of the cylinder which is obviously be the +i ,,+
length of the capacitor is f.. The electric field between the cylinders
1 ,
is due to the charge of the inner cylinder. If ;\. is the charge per unit
length of the cylinder, then electric field at a radial distance r from C
the axis of the cylinder is

E =
;\.
l +!
+i
_. +i
+l r
j+
l+
-.:+----t--...:...._
i+
E-
_
21t Eo r +i:;:··· ·····;·i + ... ~
Potential difference between the cylinders
a
_J Edr Fig. 236
h

= fEdr
a

;\. fdrr
21t e0
b

--f.n -
21t e 0
;\. (b)a
Q
The capacitance C
V

or
:k- 1@1°-(·';[~i!i(;:Il!~tiwi~NETis~'·;t•-~·.;@;-~~~,--~-'.-'~f]
"*. Capacitance· ·between two long parallel conductors ·
Let;; iie the ~harg~:per ~nit.lengJh,or each co~diictor,;Thys,,'for .
a conductor of length t, charge Q = ),.e. Consider a point Pat a
distance x from the axis of•conductor A._Electric field atP
, 0 ·• ".'..'. • ·

- -- --,- -
0--r--.-------r-------.()-
.

----- ,'
_u

'. . -E = EA + EB i irj : i i
2it~ x
0
+ 21te)d-x) l ~x~ ! :
' ),. [ I ,, I ]
= 2ite 0 ~+ d-x
Potential difference between the conductors
.. ,, • f '•,J",
· (d-r)
V= VA-VB= I
I'
Edx ' ,:
A.
i------d' ,_ ., _·,'
· (d-r) • dx-, ' · .,·., . ,.

,. _,.. .I ·[.!.+-"
2n:e0 x "d-x ·· ·
'
I ]. .
Fig. 2.37.
• 1 ,, _·.<I

A [ lnx-tn (d-x )](d-r)


--, _
27t EQ I

-"-[{ t11(d-r)-tnr}-{ fnr-tn(d--:r)}]


21tEo . , ,

__!::,__ tn[(d-r)]
7tEo \ ,r

Capacitance C

or

Ex. 1 7 A capacitor consists oftwo stationary plates shaped as a semicircle of radius Rand a movable plate
made of dielectric with relative permittivity e and capable ofrotating about an axis O between the
stationary plates (figure). The thickness of the movable plate is equal to dwhich is practically the
separation between the stationary plates.A potential difference Vis applied to the capacitor. Find
the magnitude of the moment of for~e; relative to the axis O acting on the niovable plate in the
position shown in the figure.
Sol. - The area of dielectric,
t ;:_; 1
which is out of-plates is-.1
_, :,J_,--i !..:., (ifl!Jt I .··1.ili..J?.,:' J; f;,~ 1 _;'7
.
'.:i;:
1 1
/1, '_H
• '•
1:i,/1 •.,..;
_,
1_,

Rq>>S.R.r~ _R_7ct,_.,_ ;,1·.~ .:.·1·r .:: ,....,.. -:~~-_:·· ........


--2-=-2- ,;/ C -~·.. ,
2
(Assuming a triangle of small (\>) --, · 1 ' i '
The are!' of dielectric inside the plates is " I/
·,-
;·, _ ;;11R 2 R 2q, (11-q,)R 2
_, -----= Fig. 2.38
·" 2 2 2
'
The capacitance of the system
,,, C ·,= 'C1 + C2
_,t :. - ,...::; l ! ·)

\ ',, !

' \., \ ,1: =


" I l ' - r -' • ,

Because of charges on the plates of capacitor, the charge will induce on the dielectric_ Due to which
it experience a force towards the plates which constitutes a torque aboutO. Let this torque rotates
the dielectric by dq,, the work done by the field - · · •
,_, dW _""_ --aU=tdq,
__
,'dU
l dq,
.,, ' \ 1·' -
'1
r:~iwhere ...:u'.,= - C
2
v2

J 2 2 2
= :_~_-,v__ ,, -[Eo R__ +EEo (-1)~ ]·
, ,. 2 , 2d 2d . " . ,•: I
. -· .

Ans.

I ,

Ex. 18, The gap'between the'plates of a pat'allel-plate capacitor is filled .with isotropic dielectric whose
~ •., H • ' • , '

permittivity e varies line~rly from e I to e 2 (e 2 >E 1) in the direction.perpendicular to the plates.


The area of.each plate is eijuai to S,lihe separation between the plates is equal to d•. Find the
capacitanceoftbecapacitor. -~ !t.i •, - 1 ~.:.
Sol. The value of pennittivity of the medium at a distance x from left plate e_,=e 1+ e 2 ~ ei x. The

potential difference between the plates


d
V = JEdr c:r -a
0
s ..,.
I~
d( )ch:
0 eoe.,
,•,1'
d -x-
er J ch:
0 E1 + E2-E1)
EQ (
d X

ad I ( E2)
e 0 (e2 -ei) n ~ Fig. 2.39

Q
Therefore capacitance C = V
crS

[ ad f ( E2 )]
e 0 (e 2 -ei) n ~

(e2 -ei)eo S
Ans.
den ei '
E1

Ex. 19 Find the capacitance of a system of two identical metal balls of radius a if the distance between
their centres is equal to b, with li>>a. The system Is localed in a uniform dielectric with pennltllvlty
e.
Sol. Suppose the charges q and - q are imparted to the balls. Then
v1 = vb, 11 -V~=V
v2 = Vi.,u-V~= -V
The potential difference between the balls
V1 -V2 = 2V
r
,._--b---.
Fig. 2.40

q (b-2a)
211 eoe a(b-a) ·
H~nce, the required capacitance is
q
C
q (b-2a)]
[ 21t EoE a(b-a) ·

21t EoE a(b-a)


(b-2a)
For b >>a, we can write
C = 21tEoE£1. Ans.

Work done in inserting a dielectric slab between the plates of the capacitor
Consider a capacitor of capacitance C and charged to a potential V. The charge on the capacitor is Q =CV.The

energy stored U = ½cv 2 . Let V', Q' and U' are the corresponding values when dielectric is inserted between
the plates.
Case I: Alter charging the capacitor, battery is disconnected and dielectric is inserted between the plates,
then
g Q
C: kC

V Q" =iL=!:::.
C' kC k

u _l_c·v·2 =_l_(kc)(!:::.)2 =_I_ cvi


2 2 k 2 k
u
k
Thus work done w U1-U; =U'-U

_I_ cv 2 __l_cvi Fig. 2.41


2 k 2

= . _l_cv 2(_1_-1)
2 k

= -½cvz(k;I)
Case 2 : After charging the capacitor, if battery remain connected; then
V' V
C' kC
Q' C' V'=kCV=kQ

U' _I_C'V' 2=_l_(kc)v 2


2 2
kU
The work done w U1-U;=U'-U Fig. 2.42

_1_cv 2(k-1)
2 .
Ex. 20 Three capacitors of capacitances 0.002, 0.004 and o.oo·frµF are conitecteii'in series!
Can a potentialof ll,000 V be applied to this battery? What voltage will be received by each
capacitor in the battery. The puncture voltage of each capacitor is 4000 V.
Sol. In series connection; fhe 9harge across each capacitor is equal, and so if vi' v2 and V3 are their
potentials then
. C1V1 ' C2V2 ...(i)
C2V.2 C3V3 ... (ii)
and V1+ V2 + V3 I 1000 ... (iii)
'.;.•
Substituting and solving above equations, _we get
v1 =' 6000V,
V2 . = 3000V
.and V3 = ·2000V .-
So, it is not pos_sjble t_o giv, 11000,\'. withou,t puncture ofany ofth_,,cap~cito.rs, qeca_use. V1 >.400Q;v.

Ex. 21· ·Fig. 2.43 given below shows two identical parallel plate capacitors connecte_d to a battery with
switch S closed. The switch is now opened and the free space between the plates of capaci~ors is
filled with a dielectric of dielectric constant 3. What will be the rati11 of total electrostatic energy,
stored in both capacitors before and after the introduction of the dielectric.

"
vm
1T J·
'
T I\
v-=--
kC
V

(a) ·when S\,;itch•is close& (b) After opening the switch.


Fig. 2.43
Sol. Both the capacitors are in parallel across the source. Therefore p.d. across each is V

_!_,cv 2 ,:_!_cv2
2 2
;, :~ . •' cv2
When switch is opened and dielectric is inserted between the plates of capacitors, the capacitance
I 'I ; V
of each is increased to kC while thejr potertials become V and k.
2
: . =· :-(kC)V
U .I 2 +-kC
I (V)
-
I 2 2 k
_;., D. ' . - ' .. '. -., '·' ' .. ' . ., !i p, ~ ' >

cvz(!:...'
." :-
+_l)
2., ,2k
' ,.

'. )'
'U-
_l , ::!:: ~ I ,
Now
Ur
'(½+2~} (¾+2:3)
'.
Ans.
.;5
I Ex. 22 Find the steady state charge on the capacitor in the circuit shown in jig. 2.44.,
C R,

r
~--+'-l I----"/.',---'

Fii. 2.44
Sol. Capacitor after charging stops the current. Therefore there is no current in the arm of capacitor, The
entire current will pass through resistor R 1•

. ~
I - --
RI +r
Since there is no current in R2, therefore no p.d. across it. The p.d. across capacitor is equal to p.d.
across R 1• i.e.,

Charge on capacitor q-CV Ans.

Ex.23 Three identical metallic plates are kept parallel to one another at a separation of a and b. Th e,
0

outer plates are connected by a thin conducting wire and a charge Q is placed on the central plate.
Find final charges on all the six plate's surfaces.

- -

1
2 3 4 5 6 ~

. - qi lf1 (Q-q,)
C/2
q3'
---{Q-q,
EI H..
a b
A jj C

(a) (b)
Fig. 2.45
Sol. Suppose charge on left face of B is q 1, then right face has charge Q - q 1• The charges on other
plates are shown in figure. The plates A and c;: are connected each other. and so the net induced
charge on them will be zero.
(q2-q1) + q3-(Q-q1) 0
or q2+q3 Q ... (i)
Since A and Care at same potential
VB-VA Vo-Ve
or E1 a E2 b

(Q-qi) b
A Eo

or
(a~\) ... (ii)
Electric-field du9 to- ~JI the ~harges inside an/plate (say C) is zero.-Therefore
qz _qi+ qi +Q-q1_q1-Q_ q3 _
0 ,.
2e0 A 2e0 A 2e0 A 2e0 A 2e 0 A 2e0 A I
After simplifying, we get
q2 = q3 ...(iii)
Solving equations (i), (ii) and (iii), we get
Qb
(a+b)
Q
2
Q
2
The charges on all the six faces of the plates are as:
0 -Qb Ob {la
zna+o
1U2 · :~:"
_ Fig. 2.46
2.11 DIELECTRICS

(a) Polar dielectrics


Molecule of dielectric material has permanent dipole moment (/>) in the absence of electric field,is
known as polar dielectric molecules. But the dipole moments of molecules of substance are randomly
oriented, and in the absence offield their net dipole moment is zero. Example: H20, NH 3, HCI etc.

(±3
Polar molecule
(;>
E=O,P=O Non -polar molecule

(a) (b)
Fig. Z.47
(b) Non-Polar dielectrics
Molecule of dielectric has zero dipole moment in its normal stateis calied non-polar dielectric. Example:
~N2* . . .
Polarisation of dielectrics .,
When a dielectrics is placed in an electric field, dipole moment induces in each part ofit. This is called polarisation.
The polarisation p is defined as the dipole moment per unit volume. Figure shows a dielectric placed in uniform
field. Because of i_t, an induced charge will appear on the faces of the dielectric, which constitute a dipole
moment.
'-,Q' Q'
-I

-I

-I

C --------,i
(a) (b)
Fig. 2.48
dipole moment
p
volume

Q'e Q'
--=-=a'
Ae A
Dielectric strength and dielectric breakdown
If high electric field is applied across a dielectric, then electrons get separate from molecules and dielectric
behaves like a conductor. This is known as dielectric breakdown. The maximum value of electric field that a
material can withstand without breakdown is called dielectric strength. Its SI unit is Vim.

Dielectric strength of air is £ 0 = 3xl06 V / m at one atmosphere.


The maximum charge the sphere of radius Im can have
_ _ Qmax
41t Eo R2

or 3xl0 6 9 XI09 X Qmax


12

which gives ½x10-3 C= 0.33 me.

2.12 INDUCED OR BOUND CHARGE


Consider a·parallel plate capacitor with charge Q on its positiv~ plate and - Q charge on its other plate. The
. CJ Q
electric field between the plates £ 0 = - =- - .
Eo A ~o
When dielectric is placed between the plates it gets polarised and charge is induced on its faces as shown in.fig.
2.49. Let induced charge is Q'-in magnitude. Due to the induced charge the electric field induces inside the
dielectric, whic_h opposes the external field. The electric field due to induced charge

E = -a'= -Q'-

The net electric field inside dielectric is

Emed E+E'
or £med E0 -E'

Since ~=_Q_ Fig.2.49


kEo AkEo
or I
!
. · (
(!,,.= Q 1::--,; ·.;
I) '
1 •• ···-------- '. -

An alternating form of Gauss's law


Let us consider a parallel-plate capacitor filled with·a dielectric slab of dielectric constantk. The charge on the
capacitor is Q. Let us take a close surface (by dotted lines) as shown in figure. The charge enclosed by surface
is (Q-Q'). From Gauss's law
:I .,_, -~u ,)
pE·dA =
:.irll /•b,
;r::,, .d ',. (Q-Q') '[J-Q\Q'~Q
Eo

...........
,
+8
Fig. 2.50

or

Displacement vector : The field due to polarisation E' =~ =.!_.


Eo Eo
As Pis the polarisation (dipole moment per unit volume) which is opposite to the field E'. Thus
,"
_:.I,\ tl p I+ " + + '+ ''i'
- E· -=
' . T T T

~
The net field inside dielectric Eo+E'
';, '

Ii - p T T
-- - X Eo-- - - - .
-, -
Eo p
D EoE
or Eo E+P Eo Eo Fig. 2.51

The quantity r.·;;,::E _; P =. »·:rj


~,,,:... •.- - 0 1 . : s ~ - -',.·._c_:J
is known as the displaceinent vector.
.b,zi, ... ~. Eo Eo
Dbii i . •

or pn.-d;i,' = pEo Eo•dA


')f!j ·,r-

Since
or Qi·.
n:e,

I"v:dA ,:-,-
'Y Qfire

This is the another form of Gauss's law

!d!!J Invariance of Gauss's law: ,'I ', ,

J•f V
I

'I

From the generalization of experimental results, the Gauss's law


t I, ·
pE·dA =..'LEo
is valid not only for fixed

charges but for moving charges also. In the latter case, the surface integral must be taken for a definite
instant of time in a given reference system. Gauss's law is valid for all inertial systems of reference.-- ,
High voltage breakdown or corona discharge
Let us consider two conducting spheres A and B, connected
by a conducting wire. A· charge Q is given to the system.
This charge will share by them in such a way .that their R
potentials become equal. Let q I and q2 be the charges on
them. Thus
VB A B
__ !11...
or Fig. 2.52
4it Eo r

or

II J ,_
qi
-- 2
4itR 2 r qi
--=-X-
and 2
~ R qz
4itr 2

r2 R r
or -X-=-
R2 r R

or we can say
r
-, t Air
It shows that the field is higher at the surface of small sphere. +
+ +·········e ....-·· <±i
The same can be explained with a conductor having sharp ~pd. + @ 0
It can be shown by simple knowledge of lines of force that ffold _
around this sharp end is much higher than the field in the other'.1
regions. Due to this very high field at the sharp ends of the
+ ~i
'··......e© .......
conductor, the air near these points breakdown, and cond~~tor
starts discharging. This is known as corona discharge.
E1 >Ee
Fig. 2.53
Ex. 24 A parallel-plate capacitor is placed in such a way that its plates are horizontal and the lower plate
is dipped into a liquid of dielectric constant k and density p. Each plate has area A. The plates now
connected to a battery which supplies a positive charge of magnitude Q to the upper plate. Find the
rise in the level of the liquid in the space between the plates.
Sol.
------Q
l, _r- ---------\_-
J. . . -¼"'""'tz-:>-.}-=z-f:
.. J. ......................... ....

(a)
r+ ~
(b)
+~Q-
Q

Fig. 2.54
Because of the charges of the plates of capacitor, the charge will induce on the upper and lower
layer (which in contact with negative pla\e of capacitor) of the liquid. The force on upper layer of
liquid is F = E(! in upward direction. Be~ause of this force liquid rises till force equal to weight of
liquid rises. Thus · · ·
F = JIQ'=·,;,g

where

and . mg I= .Ahpg
1,
Charge on lower plate -Q+(!=-Q+ Q(t-¾)
Q
k
The electric field due to both the plates of capacitor

cr cr'
-·-+--
E
2Eo 2e 0

- - - - - - - ( - Q + Q')
2e:A(t+¾) Fig. 2.55

We have I E(!. ·= mg

:. [-Q
2e (i+_l_)]Q(I-_I__)
k fi.
= Ahpg· i
0 A
...-r: 1 ,,.

(kf-t)Q 2

or .i~.
J ;°,)
=
' 2A 2k 2 Eo' pg .
Ans.
Force on a dielectric slab ,/,'•

When a dielectric is placed partially inside a capacitor,,it experiences a force towards the plates. If dielectric is
left free, it will move into the capacitor. The reason is that, because of charges of plates, the opposite charges
are induced on the opposite faces of the dielectric. Because of attraction between opposite charges, a net force
acts on the dielectric towards the capacitor.
(a) When capacitor is kept at constant potential:

ll I 1,l I~ <.: I /C'2/2


+
r~a
+ +:::-JQ'k
~,
+F 1
D-
r r
d
j
Q
Lf-x+x-.J
(a)
, .. (b)
'. I
Fig. 2,-56
Consider a parallel-plate capacitor with plate area (£ x b) and separation between the plates is d, this is
practically the thickness of dielectric slab. The capacitor is connected to a battery of emf V. Letx length
of the dielectric is inside the plates, then capacitance
C = ·C1 +C2 .

e0 b(C-x) e0 kbx
+
d d
Let the dielectric displaces by dx due to the electrostatic force F exerted by the capacitor (field produced
by capacitor) the work done,
dW -dU=Fdx

or F dU ...(i)
dx

This formula we have already studied in mechanics.

Since, u -'2-cv 2

dU
F
dx
(

_v: (:) [As Vis constant]

2
_V .!!..._[e0 b(f-x) + e0 kb]
or 2 dx d d
_v
2
2
[e b{-I)+ e kb]
0
d
0
d

2
'Eo bV (k-l)
2d
2
Thus, the electric ~eld attract the dielectric with a force Eo bV ( k - I) , which is independent ofx.
2d
(b) When capacitor has constant charge:

For constant charge


2C

From equation (i), we have F = dU


- _j
dx

';,_!!__[,Q2]
-,1x 2C
'' - '

[As Q is constant]

~2
"'·Q 2d I
- - - -J X k-1
2e0 b( ) [ e+x(k-1) ] ( )

·.:.,_:-,.

I@§ _This time force is a function ofx.


I _;,,,.;'

Oscillations of dielectric slab between the plates o_f capacitor


Consider the situation shown in figure. The plates .of a capacitor have plate area A and are clamped in the
laboratory. The dielectric slab is released frorii rest with a length a inside the capacitor. Neglect any effect of
friction or gravity, show that the slab execute periodic motion and find its time period.
/

The force acting on .the dielectric slab F = Eo b/;2


2d·
(k-~._ 0 [As "?lculated ~arlier]
,.____ .c - - ~
F
2
Eo b/; (k-1)' e
and acceleration.ofslab a' =
The time taken to travel distance ( £ - a)

(f-a)
m

I ' I2
-a
2
2md

·, ;; 1: i·11 ·.;i"JJ
f I k
i---a--t+-t-a-+i
Ir
which gives =

k_.r
I

The time period to complete the oscillation

-"'--1,
T

r
41=4~ -
'

4
2(£-a)xZmd .___k_,
2
Eo b/; (k'.:.1)

(£-a)md _
8
Eo b/; 2 (k-1)

As A =be b=~ Fig. 2.57


£

(£-a)mfd •
Now T = 8
Eo A/; 2 (k-1).

Ex. 25 A capacitor of capacitance C is charged by connecting itto a battery of emf I; . The capacitor is
now disconnected and reconnected to the battery with the polarity rev_ersed. Calculate the heat
developed in the connecting wire.
Sol. a
Wh~n capacitor is connected to the battery, charge Q ~ Cl; appears on one plate and-Q on the
other plate. With the polarity reversed, the charge - Q appear on the first plate and Q on the other
plate. Thus charge 2 Q, flows through the battery. -;(he work done by battery
W Vq '
l;x2Q=2Cl; 2
"~ ,--- ,,. ............
. I .
2
The energy stored in the capacitor in both the cases, is - Cl; • Therefore by conservation of
.;~•W)"SJ 2
energy
U;+W
heat generated
Ans.
M.:..~,:Etibthicri~.'&~MAGNE'TISM,~~~"1):,. ,..~,
2.13 TOTAL ENERGY OF THE SYSTEM

Total energy of the system includes self energies (internal energy) of all the charges in the system (U;) and
energy due to all possible mutual interactions (U,).
Therefore, total energy of the system U = U; + U,
There are two methods of finding the total energy of the system.

Method I: By using energy density: We know the energy density 11 = ½Eo £


2
, where Eis the electric field
due to all the charges in the system, then total energy of the system (including U; -& Ue)
v,
u = Ju dV
V,
Method II: By this method, we have to calculate U; and U,
separately and sum of these gives total energy of the
system.
q
u, u,.
Selfenergy JVdq
0
Fig. 2.58
Energy due to mutual interaction, let for two charges q I and q2
u. = q 1 (V 12 )
= q, <V21)
where, V12 is the potential due to q 2 at position of l, V21 is the potential due to q I at position of 2
Now U = U;+U,.

!@# •or an isolated charged body, U, 0, :.U U,-

Ex. 26 A spherical conducting thin shell ofradiusR given charge q. Find its total energy.
Sol.
Method 1: The energy of shell resides in the space from r=R tor= =, because field of charge on shell extend

~ and energy
1
upto infinity. The field at a distance r from the centre of shell ( r > R), E = - -
. 4it Eo r

density 11 =

Fig. 2.59
The energy contained in small volume

•. · ', 2 1 I q , )2
dU = ux41tr dr=-<=o ( - -- - \
2 47t Eo r2

Total energy

~ -Eo
1 (-1- -q2 )2 x41tr2dr
J
R
2 41t E 0 r
2 ~
1 q dr
-Eo ( - -)
2 41t Eo X41tJ-
r2
! • R

= _!_Eo
2
(-q
41t
) x41txl-.!.I~
Eo
2

r R

,---·· • .• 2 -·,
or I u = ,,q :. .· '
. -~it,;..EoJLJ
6_ __
Method II: Let some charge is given to the shell, its corresponding potential becomes v. The energy stored on
it is equal to the work done, Le.,
dU = vdq
q

Total energy of shell u Jvdq


0

I q q
- 47tEo f
0
Rdq Fig. 2.60

2
. q.

Ex. 27 A point charge q is placed at the centre of a spherical uncharged conducting shell, with inner
radius a and outer radius b. Find total energy of the system.
Sol.
Method I: The charge will induce on inner and outer surfaces of
shell: -q and q respectively. The-intensity offield at a q
distance r,

Fig. 2.61

The energy density,


The total energy u
a

h
.I_ Eo ( -
1 )2 x 4rcr.2dr
I
a
2
- .!L
4rc Eo ,.2

,
= L(.I._.I.)
8rcE0 ab
I
Metho,!J:11': To un~erstand easily, let suffix I, 2, 3 are used for them as shown in fig. 2.62.
J§J/ U -U 1+U2+U3+U 12 +U 13 +U23 _
++
=U, U2 U3 +q,(V,2) +q, (V13) q2(V23) +
2
=O+ )2 +., q +q ( -I- ( -q )) +q ( _I_ J_ ) +(-q)--J_
I
Fig. 2.62
t'.9Eo b 4rc Eo a 4rc Eo b 41t Eo b

= L(.!._.!.) ~
8rcE0 a b ·

Ex. 28 Find total energy of the system having two thin concentric shells with charges q 1 and q 2•

Sol.
q,2 + q22 +q, ( _I_ q2 )
8rc Eo a 8rc Eo b 4rc Eo b ·

----•Q2
Fig. 2.63. Two concentric
conducting shell with charges q 1_& q0 Ans.

Ex. 29 A spherical shell ofradius R I w''ith a uniform charge q has a point charge% at its centre. Find the
work performed hythe electric forces during the shell expansion from radius R 1 to radius Ri-
Sol. The total energy of the sysiem
.uinitial U;+U,=U,+q 0V0

q2
+ qoq
8rc Eo R1 _4rc Eo R1
.. ·,
·,

q ( qo +})
4rc Eo R1

and final energy ufim,I = _q_ ( 9o +½) Fig.2.64


4rc Eo R2
. .
·ffiJ ;:t;;

Workdone by electric force in expansion


.. W. -.1!,.U = I/initial ,-I/final

Ans.

2.14 VAN DE DRAFF GENERATOR + +


+
It is an instrument which is used to produce very high voltage;
of the order of million volt. !fa charged conductor is brought in
contact to the hollow conductor, its entire charge will transfer to
the hollow conductor no matter how high the potential of the +
latter may be. It consist of hollow metallic conductor A mounted
on a stand and a belt made of non-conducting material which +
runs by means of a high speed rotor over two pulleys. Because +
of the large electric field near the comb D, air ionises and whose ,.__.__Belt
positive ions are repelled by the comb. These positive ions attach·
+
to the surface of the moving belt and are carried by the belt to +
the terminal where they are removed by the another corona +
discharge to the comb E. This comb is connected with the inner D
Insulating
surface of the hollow conductor mounted on the insulating +~
,.. stand
1 •CH-+- Pulley
supports. The charge is thus transferred to the conductor and
increases its potential.
If the electric field just outside the conductor A is sufficient for
dielectric breakdown of air, no more charge can be transferred to
it. For a conductor of radius R, the electric field just outside it is

E Q·' and V -=--.


Q T hus V=ER.
2
41t "'o R · 41tEo R Fig.2. 65 Van De Graff generator.

3
Dielectric strength of air E = 3 x I 0 6 VI m, and for R = l m; V = x I 06 x I = 3 x 106 V. Thus, the potential of

a conducting sphere of radius Im can be raised to 3xl0~ V by this method.

2.15 KIRCHHOFF'S LAWS


Kirchhotrs laws are very useful in analysing mul_tiloops circuits.He provided two laws. These are :
Junction rule :
The algebraic sum of the charges or currents at any junction is zero,

Forcharge:
' • • '
U~::oJ . I
AtjunctionA as shown in fig. 2.66, . • .
q1+q2+q3 0
or q3 -(qi +q2l- Fig. 2.66
For current :

or l:i=.0 I ;,
At A, i1+i2+i3 - 0 Fig. 2.67.
or i3 -(ii+ i2l-
- ; ·1-'.\~''S ;·-. · < : ~ ~..... -

' ""' ,·
;, ;,. 'i _,_,
1-"'
·' ',. ,:· '!
•" . . .
.•

Loop rule
The algebraic sum of the potential differences across all the circuit elements in a closed loop including with emfs
must equal to zero.
or IV = 0.
(a) Circuit having resistors and cells, then
' - ·--~ - . -- - -.
:' I~+IiR
__ ._ -- -- 0
(b) Circuit having capacitors and cells, then
'
! I~+I~· = :o
,__ _c _: __·...
(c) Circuit having resistors, capacitors and cells; then

l I~+I~+IiR
C ·- _ _
L_'. ______
= o,
1 _____.,__

The juction rule is an application ofthe principle of conservation of charge.The loop rule is a consequence
of ~onservation of energy.
For circuit analysis, the following procedures should be followed carefully.
First of all label all the known and unknown carefully, including an assumed sense of direction for each
unknown. Often one does not know in advance the actual direction or sign of an unknown current, emf
or charge, but this does not matter. The solution is carried out using the assumed directions, and if the
actual direction of a particular quantity is opposite to the assumed direction, the value of the quantity
found with negative sign .
.The following guidelines and sign conventions are useful in analysing circuit
problems.
l. One must remember that, the current in any resistor or charge on any capacitor is the net response of all
the sources present in the circuit. Don't think that only nearest one is sending the current or charge.
2 Choose any closed loop in the network, and designate a direction (clockwise or anticlockwise) to
traverse the loop for loop rule.
3. Close loop may or may not have any cell, but one of the close loop must include the cell. For a circuit
having only one cell, choose a close loop, which include this cell.
4. (a) The p.d. across any resistor can be taken negative when moves along the direction of current and
positive for reverse direction of current.
-iR
~
R I

+iR
Fig. 2.68
(b) The p.d. across capacitor can be taken negative when moves from positive plate of capacitor to
negative plate and positive for reverse sense.
-q/C
• qi l~q •
+q/C
Fig. 2.69
.-- ---- ---- --~-·- ----- ------· -----------·
t·?• #
---,

Jee the sign of charge of first coming plate of capacitor only.


-· - - - - -·---- --- - ---- - - --- - - - - - - - - - - - - - - - - - - - - - - ' _____ J
(c) The emf of any cell/battery is taken as negative from its positive tenninal to negative,tenninal and
positive for reverse sense. l

_f
~

• II •
+~
Fig. 2.70
5. It must be remembered that capacitor in steady state stop the direct current, and so there will be no
current in the branch in which capacitor is connected.

Fig. 2.71
6. If capaciiors are connected in series with the sources as shown, the charge on each capacitor will be
same. It must be remembered that the net charge supply by cell will be zero.

_r1~-.q--i
1
'
''
I '
''

cJG:~"\
~ I ~ I

I(., ______ I
UJ =O
Fig. 2.72
7. Finally, the number of equations obtained must always be equal to the number of unknows.
The R-C series circuit
(a) Charging
Consider the circuit as shown in/lg. 2. 73. If the capacitor is initially has no charge, then the ini\ial p.d.

across the capacitor is zero. and the entire emf appears across resistor, causing an initial current io = !.
As the capacitor chafges, its potential increases, and p.d. across resistor decreases, corresponding to a
decrease in current. After a long time when capacitor becomes fully charged, the entire potential will
dccur across capacitor and the current becomes zero.
Let at any instant the current in the resistor is i, then by loop rule

-iR-.'L+~
C
= 0

dq q
or --R--+~ 0
dt C

dq R ~-.'L Fig. 2.73


dt C
dq dt

(~- ~) R
Integrating above expression, we have
I

J;
0

or
R
C
0

or en(s- ~)-ens -I
-
RC

I
or en[1- Js]
RC
Substituting, C~ = q 0 , the maximum or steady state charge on the capacitor and RC=~. capacitive time
constant

I
~

or

Current:

qo e-1/t
RC
cs
-e
-tit
RC

or -
-_ ,.0e-tit [asio = !]
At t= 0,
. -0
10e = ,.0 . .
and at t·= 00 , i e___,
0
0
It means initially capacitor offers no resistance in the circuit. But after charging fully, it offers infinite
resistance.
Potential of capacitor :

Potential, V

or
-·-··· ·. -··-····-· .--·-- ..---. --- - ~" ,

__________
.. .,._____ ---
:·CAPAt;;IT~CEAND CAPAClJ"OR.
-- - - - .
'

Energy stored :

u ·.!_cv2
·2

or U = u0 (1-e- I1 •)2

q = q0 (1-e- 11 ');i = i0e-lit; V = V0 (1-e- 11 ');U = U 0(1-e-' 1')2


Capacitive time constant ('t):

At 1=1:,

= 0,64q0
or at t = 't, i0e- 1
= 0.37 i0
Therefore time constant is the time in which charge on the capacitor grows to 0.64 times the maximum
charge or current decreases to 0.37 times the maximum current.

q i,

'lu ................ .. i,

0.64q,, ..... !

(a) (b)
Fig. 2.74
(b) Discharging
After charging the capacitor, let now source is removed from the circuit.
2
The energy stored in capacitornow used to flow the current. At t= 0, we have q = q0 and U= 1/0 = 52...
2C
Again by loop rule, R C

-iR- !L = 0
C
where q is the charge on capacitor at any time I
s
_dq R !L
or Fig. 2.75

I; I
dt C'
I
dt
or = -~
RC
\
Integrating above expression, we get

f
q
dq
q
t

-f :~
qo 0

or 1£nqlq
qu RC

fn!L I
or [As RC=~]
qo ~

~ --itt' -
or q qoe -

Current: i =

. -lit
-,oe

Potential: V !]_= qo e-1/t


C C

Energy: u ~cv 2
2

½c(voe-,tt)2
I cv.2oe-21/t
-
2
U -·-21/t
oe .
tJ
'lo

(a) (b)
Fig. 2.76
~7~ , W
~---....:-.:.~---'---'-.''-------'----~----'------'~--- CAPACITANCEAND CAPACITOR . .

Ex. 30 A capacitor ofcapacitance C is given a charge Q.AII = O, ii is connected lo an uncharged capacitor


of equal capacitance through a resistance R; Find the charge on the second capacitor as a function
oftime.
Sol. Let at any instant the charge on the uncharged capacitor is q. The charge on charged capacitor
remains (Q-q ).
Now by loop rule,

-iR- !l.+ (Q-q) = 0 ~--~q ,-q


C C

_ dq R+(Q-2q)
nO
or 0 (Q-q) -(Q-q)
di C

dq di
or (Q-2q) Fig. 2.77
CR
Integrating on both sides of above expression,we get

I
q dq
I
0
Q-2q I_<!!_
0
RC

I
RC

21
or ln(Q-2q)-lnQ
RC

21
or
RC

-21
or
RC

q
Q
2 ( 1-e
-2,-
RC
1
. Ans.

In above case the time constant t' = RC .


2

Ex. 31 There are two uncharged identical metallic spheres ofradius a, separated a distanced.A charged
metallic sphere (charge q) of same radius is brought and touches sphere 1. After some time it is
moved away to a far off distance. After this, the sphere 2 is earthed. Find the charge on sphere 2.
Y1 • /_,. '.),:_: ••. ~~

Sol. When sph~re with cha~~~ q·touches metallic spher~ ·;, ;he charge.on -1, comes o_ut to be ½. The
potential due to sphere I at the surface of 2 is

. I qi
V. ~---
21 - 47tEo d

() ... ,
When sphere 2 is earthed, its potential becomes zero.
Let induced charge on it is q', then
•.) J v/ Fig. 2.78

I q I q'
---+---
,. ,. 47t Eo 2d 41t Eo a

I",
which gives q' - qa. Ans.
2d

,
.
·"
Finding the time constant ''
If there are many resistors and capacitors are connected in the circuit, then time constant can be obtained by
using following steps;
(i) make short circuit at the place of battery.
(ii) Find equivalent capacithn~e, if there ~re many capacitors in the circuit. Assuming equivalent capac/·for
I .
~~ . .

(iii) Find net resistance across the ·capacitor, say it is R"'' .


(iv) FinaUy ~.ct= Rnet cnet·

4R

R 6R-=¢, 6{/ R 6}
2R
C C

.'
Fig. 2.79

Ex. 32 Find the time constant of circuit shown in jig. 2. 79.

Sol. After short circuit, the net resistance across capacitor ·

~-' - l. '6Rx6R .
--- · +R=4R
1,.. , . , 6R,:6R,_ , f·, 1. ••. T-
II

··: ,: :• -, i•' -Ans:


•• i , I ! ':. " . ·~
Ex. 33 Find the time constant of the circuits shown in figure.
1l2F

Sol. (a) ~I
ill
20
'NlA\
6Q

'[2~~ (b) 'lOQ f 0.5~- f,on +0.5F


_Fig. 2.80 Fig. 2.81

6xl2 + 2 + 2 =SQ '


0.5+0.5= lF
Rnel 6+12 cnet .,
!Oxlo= 5 Q
'tnet Rnct Cnet Rnet l0+I0
l
8x- 'tnet Rnet Cnet
2
4s 5xl
5s

Ex. 34 A circuit has a section AB as shown infig. 2.82. The emf of the source~ = 10 V, the capacitor
capacitances are equal to C 1 = 1.0 µF, C 2 = 2.0 µF, and the potential difference
VA - V8 =5.0 V. Find voltage across each capacitor.

Fig. 2.82
Sol. Both the capacitors are in series, therefore they will have same charges. Let charge on each isq. By
loop rule,

+5+ .'I.+10+.'L 0
I 2
or q -I0µC

Potential difference across I µF, V - V - q -(-IO) IOV Ans.


P Q C

_!l_ = -(-10)
and potential difference across 2µF, VR - V8 =
C 2 , ·i ,.,,,

= 5V Ans.

Finding the potential difference between any two points in a circuit


Let we have to find the potential difference between P and R in above circuit
Step 1: Connect an imaginary battery between the points, let it is VPR'
Step 2: Choose close loop including the imaginary battery.
Step3: Use loop rule to find VPR"

E1~
Fig. Z.83
5.0 V '

Choose close loop P - R -y- x - P and applying loop rule, we have


q .
C +IO+.VpR = 0

10
or --+l0+VPR 0
l
or VPR O,'
1q
Ex. 35 Find the potentjal diff~~~nc~ V0 - V6 between the points a and b shown in jig. Z.84.
,, ~,·
I e
-{q, + q,,: (q, + q2) ]
6JJ 4µF 6J !4µF i

a
I I h a
I
ql!~,
i h j
i
12V 2µF 12V i2µF i

I I d
I
q,,i,q,
j C
i
j
24V lµF 24V iJµF i
:.................:

(a) (b)
Fig. Z.84
Sol. Suppose the charges on lµF and 2µF are q 1 and q2 respectively. By junction rule the charge at
junction h must be zero. Therefore the charge on 4 µF capacitor be (q 1 + q 2). The proposed charges
are shown in figure.
We have introduced two unknowns and therefore we require two equations, which can be obtained
by loop rule.
In close loop aefba, we have

+6+ (q, +q,) + q, -12


0
4 2
or q 1 +3q2 24 .... (i)
Now in close loop abcda , .

+12-q2 +iL-·24 = 0
2 I
or 2q, -q, 24 .:.(ii)
.,.~ -

Solving equations (i) and (ii), we get


q 1 = 13.7 µCand q2 =3.4 µC
To find Vu - Vh, connect an imaginary battery between a and b, as shown in jig. 2.85.
Now in close loop abxya, we have
~·······-! f·······?~ r'······: 6
0 ' 121' 2µF :

• r, '

= -12+ie..
:......... :.. 1!··············:
or
2 Y v-vr·
ab- 'o- 'b x

3.4
- 12 +- Fig, 2.85
2
-IOJV. Ans.
Finding flow of charges : This can be understand by following example:
Ex. 36 What charges will flow after the shortening of the switch Sw in the circuit illustrated in jig. 2.86
through sections I and 2 in directions indicated hy the arrows?

(a) (b)
Fig, 2.86
Sol, Before shortening of switch both the. capacitors are in series with the cell. -Thus charge on each

capacitorisq·=( c,ci )1;


Ci+C2
After shortening of switch, let charges on them are as shown infig. 2.86(b).
In close loop ab e fa, we have

...(i)'

b -qi F-
and in close loop b c d e b

-q, -I;+~ = 0
J c,
C1 Cz-
...(ii)° ~c,
Solving equations (i) and (ii), we get
q 1 = ff and q2 = I; C2 Fig. 2.87
Flow of charge through section 1
Initially charge of capacitors joining at b is q - q =0
After shortening of switch the charge on them are
qi +qz =0+C~2 =C~2,
Therefore flow of charge through section I final charge at junction b - initial charge at junction b
(qi +q2)-(q-q)
C~2-0=C~2
Flow of charge through section 2 :

-qlf~ q'lh
c, J_

-••+1-----,-_ ___
2-'
J.s ---~I s
---·+I-+•
2
Before shortening of '· '

switch · ·· After shonening of


switch

(a) , . (b)
Fig. 2.88
It is clear from figure,
the flow of charge through section 2. Final charge on capacitor C1
,· '
- initial charge on capacitor C 1
' ',

..,,.
•'1
Ans.

Heat generated
It can be obtained by using.conservation of total energy of the system, That is
Initial energy stored in capacitors
+ work done by sources Final energy stored in capacitors + heat generated
or U;+W 1
u
+H
or H (U;-U}+W

Work done will be positive if work is done by the sources and negative if work is done on the

sources.
~''
· CAPACITANCE AND CAPACITOR

(a) Positive work done by source :


If a capacitor C of potential V (Vmay be zero) is connected to a source of emf~ ( ~ > V)
The work done by source wi II be = ~ (""7)

~(q1-q;)

~(c~-cv)
c~(~-v).
(b) Negative work done by source :
If a capacitor C of potential Vis connected to a source of emf~ ( ~ < V)
The work done by source will be ~t.q
~ (q£q;)
~ (Cl;-CV)
-~(V-~).
Case I. If U = U;, the heat generated H = W. _
1
Case 2. If capacitors remain the part of same circuit then there always be a positive work.
Case3. lfU1 >U;,then H -(UrU;)+W
or W CUrU;)+H
50% 50%
Work done by source
Heat generated H
2

Ex. 37 A capacitor having a capacitance ofl00µF is charged to a potential difference of24V. The charging
battery is disconnected and the capacitor is connected to another battery of 12 V with the positive
plate of the capacitor joined with the positive terminal of the battery.
(a) Find the charges on the capacitor before and after the reconnection.
(b) Find the charge flown through the 12 V battery.
(c) Is work done by the battery oris it done on the battery? Find its magnitude.
(d) Find decrease in electrostatic field energy.
(e) Find the heat developed during the flow of charge after reconnection.
Sol. (a) Initial charge on capacitor,
q; CV
I00 x 24=2400 µC
Final charge after reconnection, ' ,
q = 100xl2=1200µC '• 1f"
1
(b) Charge flow through the battery
t.q qf-qi
1200-2400=-1200 µC
(c) Work done by battery w Vt.q I:,,

12x(-1200)µJ
-14.4mJ
(d) Electrostatic field energy,

2
U; = _!__CV; 2 = _!__x IOOx 10--6 (24)
2 2 .
28.8mJ

2
and ut _!__cv} =_!__xIOOxI0--6
·2 2 ·
x(12)

= 7.2mJ
The decrease in electrostatic field energy
U;-UJ
= 28.8 -7.2
=
21.6ml
(e) To calculate the heat generated, we can use the conservation· of total energy. That is
U; + workdone by battery = Ur+ Heat generated
28.8-14.4 7.2+H
or H = 7.2niJ. ·Ans.

Ex. 38 What amount of heat will be generated in the circuit shown in jig. 2.89 after the switch Sw _is
· shifted from position I to position 2_?

aI.---<'----: iJ:
s~s2 ·
.,
,·. <,. -

Switch is at position 1 Switch shifted to position 2


.' ' , '. . ,

(a) (b)
'
Fig. 2.89
Sol. Let q I and q2 are the charges on the capacitor in the positions pf switch 1 and 2 respectively...··...
Choose close loop ab c defa in figure (a),

or
Now choose close loop a b e fa in figure (b),

~-~1 = 0
C
or q2 = ~,
It is_ clear from above calculators that, the charge supplied by battery I is as such while charge
supplied by battery 2 is increased by Aq 2 = 0-(-C~z) =~2
- ----·-- -
Therefore work. is do!Je,oQly_by, ba!tery 2 ,.which is

= (ili/2)~2
(C~2g2
c~2 ,
• 2 . ·.·

Heat generated =
. c~~ Ans.
2
Short in series
Short ideally has zero resistance, and so potential difference across short will be zero.When a short is used
acrosse the capacitor the entire charge will pass through the short; no charge comes on the capacitor connected
with a short.
Consider the circuit shown in figure.
The charge on capacitor C 1 = C 1~.
The charge on capacitor C2 = 0
The equivalent capacitance Ceq = C 1

Fig. 2.90
Finding equivalent capacitance :Wheat stone bridge type circuit
(a) Balanced circuit : CPQ = ? A

ff -'l. _<l, •«i=i<,isoru.a oolooeoC TOs h,e~"' · ~~~


}~:r-/~'-'c.
;:;~a~a:~-t:i:·JB will have no charge, and ·..
therefore it can be ignored in finding CPQ· ·
p
Y . Q

cPQ = ( c~~~2 )+( c~~c~ 4


) Fig. 2.91

Ex. 39 Find equivalent capacitance between P and Q.


Ignore

Sol. (a) ~br-D.Jr rl !q-o·


Ayir---J -
' . .
Fig. 2.92
The capacitance in dotted circle can be ignored.
C C
·C
PQ
= -+-=C
2 2
Ex. 40 Find eqtiivaient'tapa~itance between points P and Q as shown in fig. 2.94.

p• I IIT~~H~$~Q_·
Fig. 2.94
Sol. In the circuit VA= V8 = V0.Therefore capacitors· connected between A & B and B & D have no
charge. So they can be ignored. The effective circuit is as follows:

(a) P" J lie! IksY lcj I ic-Q


Fig. 2.95

Fig. 2.96
3C C
(c) P~Hf--Q
Fig. 2.97

Now
_ 3CxC -ic Ans.
CPQ - 3C+C- 4
(b) Unbalanced wheat stone bridges:
This type of problems can be solved by circuit analysis: The method can be understand by the following
example:

V R
(a) (b)
Fig. 2.98
Sol.
Method I: Let us connect a battery betweenP and Q. The charge distribution is shown in fig. 2.98. The charge
coming from positive tenninal i~ (q 1 + q2). The same amount ofcharge comes from negative tenninal
of the battery. Due to cross-symmetry, the charge on C1 in both anns will be same. Similarly charge
on C2 in both anns will be same. · ·
The charge on capacitor C3 : Let q3 be the charge on capacitor C3 . By junction rule
q3 -qi +qz = O~q3 =(q, -qz)
Let total charge drawn from the battery is q, then equivalent capaci!a~§e between Pan Q

C = ...(i)

Now we have to find the value of q 1 & q2 • Choose a close loop PA BP, we have

_!l!__ (q, -qz) +~


= 0
C1 C3 C2

or = 0

or qz(-1 +-1) ... (ii)


Cz C3
Now in close loop PA QR SP

-q, -~+V
0
c, C2

!IL+~
or c, Cz = V ...(iii)

From equations (ii) and (iii), we get

q, ( -I + -
I )
q1 C1 C3
V
Ci+ C2 ( -I + -
I )
Cz C3

or q, [ -I+ (C1+C3) ] V
c, c 1(c2 +c3)

1 2 C (C +C3)
or q, = C +C2 +2C3 V ... (iv)
1

C2 (C1+ C3)
From equation (ii) q2 = C1+C2 +2C3
... (v)

q, +qz
Equivalent capacitance C
V

C1(C2 +C3)+C2(C1 + C3)


= C1 +C2 +2C3

or
r--·=
I C
2C1C2 +CiC3 +C2C3
! C1+0+2C3
; . ~. ,, ,', ·'
Methodll:' Vj,-·Vg = (VP-VA)+(VA.'._Vg) ,-,,

-- . -'lL+iL
C C
1 2

-'lL+ q-'qi'
.. ,(i)
q C2
Also = u\...:vA)+(YA-VB)+(Vo-Vgl

q, 2q,-q. q,
=, c;-+~+ c, --~ii)

Eliminate q1 from above equations (i) and (iii) io get


q
C

The equation (i) can be written as

or ... (iii)

The equation (ii) can be written as

C1C3 (vp -Vg) I,


or ...(iv)
2(C1+C3)
Subtracting ~quation (iv) from(iii), we have

(vp-Vg)[ CtC2 _
C2-C1
qc3 ·)-( q + C1
2(c,+C3) - Cr-Ci 2(c,+C3)
)q
or (vp-Vq )[2c,c2 (c, +C3)-qc3(C2 -ci)]
= c, [2(c; +C3)+(C:i ::.q)]q,· , .
2C1C2 +C1C3 +C2C3
or
.C1+C2 +2C3

h&@ Sli!dents are advised t,~ us,: ,method I.' ,


,,

Ex. 42 Find the equivalent cap~citance of the combina\ion sho)Yninjig. 2.99 between Pand Q.
- .

q, L,, q,I -q-'


l1tf' 3µF
l/1 I-qil/-1 -qi
4
~µF

p
2µ/' ~ 211/
Q
=> I-{'· q, Q

'" ¢Pi
4µF q
~111F
311F
8

Fig. 2.99

Sol. It is clear from charge distribution that, there is no exchange of charge between connected arms at
the junction. Therefore they can be separated as. shown. The above circuit now reduces to

l1F
lµI'; . 3µF

.T,F .
p
rµ1 1
1 tF

Fig. 2.100,

11 F. Ans.
For further treatment, go through previous.problem answer

1.

- Infinite ladder : In the ladder shown in jig. 2. I Of, there is repetation of capacitance c, and C,. ,The
capacitor~ c, and C,_connected in the fig~re/ig. 2./02 constitute a unit cell. ,-_, •, • 1'

~ I· 11
C,I
I. H--i""'":.....
I
·Ic2 TC2 ' ·,.TC2 :""'~'"'
Qo
. Fig. 2.1M

7~
____J_, C'2
Fig. 2.102
Assume the equivalent capacitance between given points is C. Since there are infinite unit cells in.the circuit; ..
so we may add one more unit cell or deduct from the circuit, the equivalent oftesulting circuit will remain same,
i.e., C. Finally recluce the circuit to find.C. ln,above problem, we add one more unit cell as follo~s:
) . .'

,1
\. f
. ·
... ~-· _)equivalent of the circuit

Fig. 2.103
It is clear from above theory that Cpg=Cp'g'= C. The circuit now reduces to

P' II--,
----'--c-11-' _T
.J_. (C+ C2)
Q'ao- -
Fig. 2.104 , ,
Now equivalent capacitance C
I I
-+---
c C1 C+C2

I C1+C2+C
C C1(C+C2)
or C 1C + C 1C2 CC 1 +CC2 +C2
or C2 + CC2 -C 1C2 = 0

C ;, , --C2 ±Jcf+4xC1C2
2
Take positive value of C because it can not be negative

:.·, . C = -C2 +JC2 +4C1C2 .


" Ans.
,2 ' '
Ex. 43 A finite ladder is constructed by connecting several sections of 2µF and 4 µF capacitor combinations
as shown injig. 2.105. lt is.terminated by a capacitor of capacitance C. What value should be
chosen for C, such that the equivalent capacitance of the ladder between the points A and B
becomes independent of the number of sections in between ?

ATL.8r~······. · . · . .··17_
; 2µF : 2µF 2µF C

B,
·
I II
I.....................
'
I . . . . . . . . . . . . _I
'° · Fig. 2.105
Sol.· Unit celi'comp~ises2µF and 4µF as in dotted box. ' ',
' . ' ' ' , ' . . . ' " ' .,
' ' .' ' ' / .
We can add, zero unit cell with C ~r one unit cell with C or two.unit cells etc; the equivalent
capa~itance be~een A am\' B should be equal in e~ch case. · · · · · ··
p•

le ~-ttPti.
IµF IµF T(
9,-._ _ T Q• so on

(I) C with zero unit cell(II) C with one unit cell (Ill) C with two unit cells
Fig. 2./06
To get value of C, we may equate any two from above. But for simplicity in calculation, we equate
I and II.

C 2+ ( -4C·r
-
4+C

8+2C+·4C
or C
4+C
or c2 -2c-s 0
C 4µF. A,rs.

Ex. 44 Twelve capacitors, each having a capacitance C, are connected to form a cube (fig. 2.107). Find the
equivalent capacitance between diagonally opposite corners such as P and Q.

.
p
c-{7
p

~7
~
q

.- 0

0 Q q/6.

4
V
E D

(a) (b)
Fig. 2.107
Sol. Let a battery is connected between P and Q. The c_ircuit has symmetry abou! diagonal PQ. Therefore
charge provided by battery say q at P equally distributed into three parts. Similarly the-q provided
by negative terminals at Q is equally distributed in three parts. Again at each junction this charge
divided into two equal parts. The charge distribution is shown infig. 2.107(b).

The equivalent capacitance


Choose close loop PA B Q DE P, we have

(7) {7) (~q)


--+--+--+V 0
C C C

!L 6C
or , !:
V ·,5' .'
•\ .. ' .
or 6C A11s.
CPQ '' ,
5

Ex. 45 Two capacitors A and B with capacities 3µF and 2µF are charged to a potential difference of 100V
and 180V respectively. The plates of the capacitors are connected as shown infig. 2.108with one
wire from each capacitor free. The upper plate A is positive and that of Bis negative.

.,, ' ' .. , .f" .: ; ,,

' . ~ - .} ..
Fig. 2:1oa ., .
An uncharged 2µF capacitor C with lead wires falls on the free ends to complete the circuit.
Calculat'e:
(i) Thefinal charge on the three capacitors and
(ii) The amount of electrostatic energy stored in the system before and after the completion of
the circuit.
Sol. (i) Let q be the charge flowing during redistribution of charge. By Kirchhoff's loop rule, we have

Fig. 2.109

· !L+!L+!L-1so-100 ~ o .,
2 2 3
or · q ·,,; ·, 210µC
-,-. ' . ! , • 0 r1 "r 'I
·1 ,1. J •. '·.
·• The final charges o~'the capacitors ·are· •·

', •· ._, ,',


,r~ '
=' .cv~·,;· \,,
!,,· , , JI ,, , ,
.. •, ·, ,•'
I , • J_.

1 I
·
' ' ,: , , ' , ,
= 3x100-2J0=90µC
• L ~ , ,; I • · • ' ' ' , ; ,[ ,
·'
q8 2xJ80-2J0= ISOµC
I
and qc o,+210.=ZJOµC · · •: .-
CAPACITANC_E_AN_D_C_AP._~_c_1_r_o_R__ ~
(ii) Initial electric potential energy

½x(3x10-6 )x1002 +½x( 2x!0-6 )xl80 2


47.4 mJ
2
1 (9ox10-6J2 1 (1so x10-6J2 1 (21ox10-6)
~--~+ ~--~-+
2 (3xlo-6) 2 (2x10-6) 2 (2x10-6)
18mJ. Ans.

Ex. 46 Three capacitors C1, C2, CJ are connected as shown injig. 2.110. The potentials of P, Q andR are
V1, V2, VJ respectively. Find the potential V0 atthejunction 0.

c2 I IQ

~ I

~---<J>-----eo R
c,
Fig. 2.110
Sol. Let q 1, q2 and q 3 be the charges on the capacitors C 1, C2 and CJ respectively. Then we have

v,-vo .'lL
c'
I

q2
Vo-V2 z· 2

and V0 -VJ
:!l...
C3
By Kirchhotrs I law atjuction 0
q, q,+ q3
or C 1 (V1 -V0 ) C2 <Vo-V2) + c, <Vo- V3)
After simplifying, we get

[ C1V1 +C2V2 +C3V3] _ Ans.


Vo
q +C2 +C3

Ex. 4 7 In fig. 2.111, find the potential difference between the points A and Band between the pointsB and
C in the steady state.

20n
A._._.,,,,.,,,--, ~ ~ 1--__.C
100 V
Fig. 2.111
Sol.

AP ~r~·
The equivalent circuit ca\be d~:"n as in jig. 2.112.,.J

3µF
V (il , ~[
lµF
lµF
10n
20n
A.........,WHr-~~~ C
JOO V
. ~Zlll . · -
At s/eady state, there is no current i~ the resistors, so the potential difference of the battery will
appear entirely across the capacitors.
The potential of point A is equal to A' and that ofCis equal to C'. Thus if V1 and V2 are the required
potential difference, then
V1+V2 ... (i)
and 6V1 ...(ii)
Solving equations (i) and (ii), we get
v1 25V
and v2 75V. Ans.

Ex, 48 What is the equivalent capacitance of the capacitor network between A and B shown in the.fig.
2.113. Each capacitor in the network is of value C.

Fig. 2.113
Sol. The equivalent circuit can_be drawn as:
From the symmetry considerations, the potentials of the points C and Dare equal, and so capacitor
connected in arm CD remain uncharged, and therefore can be ignored.

C
~
(a) Fig. 2.114 (b)
Thus equivalent capacitance between A and B is
C C
C = -+C+-=2C Ans.
AB 2 2
Ex. 49 Two small balls of mass m, bearing a charge q each, are connected by a nonconducting thread of
length U. At a certain instant, the middle of the thread starts moving at a constant velocity v
perpendicular to the direction of the thread atthe initial instant Determine the minim uni distance
d between the balls.
Sol. Initially the velocity of each ball will be v.
The initial energy of the system

U'. = (
4it-
e0 ze
1
)L+z(_I_mv2) · F:"
2 ,g. 2115
·

If dis the minimum distance between the balls, then final energy is only potential energy of the
balls, and therefore

eJ ~
1
Uf = ( 4it
By conseivation of mechanical energy, we have
U; = u1

eJ ;: +z(½mv2) = ( 1eJ ~
1
or ( 4it 4it

After simplifying, we get


,.
'2cq2 ]
d
[ Ans.

Ex. 50 Two parallel-plate capacitors are arranged perpendicular to the common axis. The separation d
e
between the capacitor is much larger than the separation between their plates and than their
size. The capacitors are charged to q 1 and q2 respectively (see jig. 2.Il6). Find the force F of
interaction between the capacitors.

..... 'liM··················q
' d
Gl-···-·---·
,,
Fig. 2.IJ6
Sol. Let us first take the case when the capacitor are placed so that the plates with like charges face each
other.

-x-l···f ··········
·······l··-tq'···············q
-(t+x)-
2

Fig. 2.IJ7
The field produced by first capacitor on the axis at a distance X from the positive plate is

(x>>f)
The force acting on the second capacitor situated at a distance d from the first is
0

F ,;, : Edq~ ,-E(d+1)q2 = q2 [ Er.: F(d~I)]


'
.) '~

=' 3q1q2l2 A1,s,


21t E(i d 4
In this case capacitors will repel-each other with a force F. A similar analysis can be made for the
case when capacitors are placed so that the plates with opposite charges face each other. Then the
capacitors will attract each other with the same amount of force.

Ex. 51 Condensers with capacities C, 2C, 3C and 4C are charged to the voltage V, 2V, 3 V and 4 V
correspondingly. The circuit is closed. Find the voltage on all condensers in the equilibrium.

-c.v
r-f i-ic 1
) 7=2c
4Cl ·, - Jiv ,.

4V L_-----,-<+11;.J-I ' ~
.' .
''

3C,3V
·, ,, Fig. 2.ll8
Sol. The initial charges on the capacito~ are'shown in.fig.2.119 (a)

.
pi+ q1 =CV
I

~q.=4CV
.
·)<JI+
®-
(qi -q)
I_~

q,.=16CV
lL r· lr:-q) . T(q,-q)

+lrJ
q3=9CV
(q,-q~
+I - C
(b)
(a)
Fig. 2.ll9
Suppose q amount of charge flows in the circuit after closing it. The net charge on each capacitor
is shown in figure (b). ·
[
Applying loop rule in A B CD A , we have
q, -q q2 -q q3 -q q4'-q,
--+--+--+-- = 0
C 2C 3C 4C
CV-q 4CV-q 9CV-q 16CV-q '
Or ---+----'-+ . + -O
C 2C 3C 4C -
24
After simplifying, we get q= -CV
5
Potential difference across the capacitors :

!Vil =lq,~ql=~
.!.2_ V
5
,I·, 24
4CV--CV
q2 -q 5
I I
2C · · 2C

~v-
3
', 24
9CV--CV
q3-q 5
I I
3C
7V ''
3C

. 5
q4 -q1 = 116CV-24CV
and
l 4C 4C
14C
5 . Ans.

Ex. 52 lnjig. 2.120, the battery has a potential difference of20V. Find :
(a) The equivalent capacitance of all the.capacitors and (b) the charge stored on that equivalent
capacitance. Find the potential across and charge on (c) capacitor 1, (d) capacitor 2, and (e)
capacitor 3.
C,=2µF
A B C • D
.----';,'-----'~--I
I4µF
2µF
c,=4µF

'""Lt I H
1------':
C, =3µF
Fig. 2.120
Sol. The simplified connections of the capacitors are shown infig. 2.121.
(a) The equivalent capacitance between C and Fis 2µF.

1 1 1
T
--+-1-E
. 2µF 2µF 211F

L----L-3-µF--I• ~3µF
I JJ f----Jo
C 1 =3µF A
(a)


· Fig. 2.121
Capacitors across, the arms BJ, CF and DE are in parallel, +
20V
and so their equivalent capacitance is 6 µF. Similarly
capacitors across JH, and FG are in parallel and so, their
equivalent is 6 µF. Now 6 µF and 6 µFare in series across
the battery, so their equivalenris capacitance 3 µF.
,I 1, .
(b) T he c harge store d on equ,va 1ent capacitor I H
'l ' '
'Q- = C,q V (b)
~: ' •' .;
=; 3x20 Fig. 2.122
u= 60µC Ans.
(c) The potential of the battery is equally divided between the capacitors connected between B
and H. Thus potential difference
V~= VJH = IOV.
The potential difference across capacitor I
! V1 = IO V
and charge 'Q1 = C1 V1 = 3xl0=30µC Ans.
I
(d) The potential difference across C2 is also 10 V. Thus charge
Q2 = c2 v2
= 2Xi0=20µC Ans.
The potential difference a?r,oss C3 will be 5 V and so charge
Q3 C3V3
4x5=20µC. Ans.

Ex. 53 lnfig. 2.123, all the capacitors have a capacitance of6.0 µF, and all the batteries have an emf
10 V. What is the charge on capacitor C?
Sol. Suppose Q is the charge on the capacitor. Choose a close loop ab c defg h ij k Im n a, and apply
w
loop rule, we have ' •

0,

I I

I-1.
TT
Fig. 2.123
·,
which on solving gives
Q c~ 'l
6Xl0=60µC, Ans.

Ex. 54 Three capacitors C1 = 3 µF, C2 = 6 µF and C3 = 6µF have equal charge q = 30 µC each. C1 and C2
I
are connected in series as shown in jig. 2.124. If C3 is connected across the series combination
by connectingA with Cand BwithD and ifresistance of connecting wire isR= 10 0, calculate ,
initial current in the circuit and also heat generated.
Sol. Let after connection the initial current in the circuit is i0.

. ,-/
R=IOQ

C••----+--<I F,____ __.,,D


c, c D
(a)

Applying loop rule in A CD BA, we have Fig. 2.124

30 30 30 .
or -+----,0 xIO 0
3 6 6 '• --.-
IA Ans.
Heat through the wire is generated till charge.flows through it. After getting steady state, no charge
will flow. Let q' be the amount of charge flows, then net charge on each capacitor is shown in jig.
2.125. By loop rule, we have +-@
q-q' q-q'
c;--+-c;----z;-
q+q'
0
(q-q'
A cl
- (q-q') -
I a B

30-q' 30-q' 30+q' R= IOQ


or --+----- =O
3
After simplifying, we get
6

q'
6

15µC. C
(q+ql ,__
- __ _..
D
C3
The final charges on the capacitors are :
q1 q-q'=30-15= 15 µC Fig. 2.125
q 2 = q-q'=30-15=15µC
and q 3 = q+q'=30+15=45µC
The initial energy stored on the capacitors

2
30 [_l_+_l_+_I_]
2 3 6 6
300µ) .
.,
• 2 2 2
Final energy +
..'!L -'lL + ..'IL
2C 2Ci 2C3.
1

15 2 15 2 45 2
-·-+--+--'
2X3 2x6 2x6
225µ)
Heat generated U;-UJ
300-225=75µJ. Ans.

Ex. 55 The capacitors shown in jig. 2.126 has been charged to a potential difference of Vvoltso that it
carries a charge CV with both the switches S 1 and S 2 remaining open. Switch S 1 is closed at I= 0.
At t = R 1C switch S 1 is opened and S 2 is closed. Find 'the charge on the capacitor at
t= (2R 1C+ R 2 C),

SI
R2
--:('o------'
I; 2
Fig. 2.126
Sol. When switch' Sr is closed and S 2 is open. The·circuit is as shown in figure (a). ,
. "" • . - \ - • • I

D· ·,CJ (a) i;
(b)
Fig; 2.127
_The_-c_h~rge_~tored on the_ ~apacitor q0_= CV. Thus
.• -. ,, •• 1 r. , _ i· '_ _
I ·, "RC , ._.·:·
"··' q = qoe 1

I
or q CVe R1C
At t = R IC, the charge on the capacitor is

q, = eve-,= CV
e
When switch S2 is closed and S 1 is open the circuit is shown in figure (b). Applying loop rule in
close loop a b c d a , we have

-r-i(R1 +Rz)+~ = 0 ,, ·,

c~-q'
or j
C(R1 +Ri)
dq' c~-q'
or -·
dt C(R, +R2)

dq' dt
or ·=-
C~-q' c(R1+R2)
On integrating above equation,. we have
q'

CV
J' C~-q
·dq''

(2R1C+R2C)

or -It n (Cl; -q ')l~v C(R1 +R2) Ric


e

. [ cv)· -[2R1C+R2C-R1C]
. orfn(cl;-q')-fn Cl;- ;= . C(Ri +Rz)
7
or fn[ c~_
C~-q']
cv = -I
e

or C~- CV e( c~ -q')
e

q' [c~(1-~)+ ;]- Ans.

Ex. 56 In the following RC-circuit, the capacitor is in the steady state. The initial separation of the
capacitor plates isx0• lfat t= 0, the separation between the plates starts_changing so that a
._constant current flows through R. Find the velocity of the moving plates as a function oftime. The
plate area isA.

Fig. 2.128
Sol. For any separation x between the plates, the capacitance
Eo A
C = -
X

Let q be the instantaneous charge on the capacitor when current i flows through the circuit, by
Kirchhotrs law, we have
-q .
--iR,+~ 0
C

or ~ .'L+;R
C

~ = _!l!_+iR ...(i)
Eo A
Here q and x are variables while~. i and R, A are constants.
Differentiating equation (i) w.r.t. time we have
tfx
q X dq
0 = ----+----
EoA dt EoAdt

dq dx
Putting - =iand - =v,.weget
. dt dt

q =
I' .·
Substituting the value of q in equation (i), we have

~ =

dx
or
dt v= (- E: A)xi
~-iR

dx
or ... (ii)
x2 ( ~oi A 1dt
~-iR

Integrating equation (ii), we have

I xi
X

dx -JC'Eo_
~-,R
A)d1 ),
. xo 0 I

or (i!Eo A), ...(iii)


X Xo ~-iR
Again differentiating w.r.t. time

I dx
-- - (i/Eo A)
xi dt ~-iR

or
I
--(v)
x2
c/E 0
~-iR
A)

V = -(ii Eo_ A)xi


~-,R
... (iv)

Substituting the value ofx from equation (iii) in (iv), we get

i/EoA) I Ans.
v
(
= . ~-iR [(i/ A),+-']2 .
~-1R
Eo_
xo

Ex. 57 The voltage applied across a capacitor having a capacitance of 10 µFis varies as shown in fig.
2.129. Calculate
(a) _the charge and
(b) the energy stored in the capacitor when the terminal voltage is 600 V.
Sol. The charging current is given by

dq d(cv)
(a) i -=---
di di
CdV
di
r
600

400

200

0 2 3 4 5 6 7 g I (s)
Fig. 2./29
600
From Oto 2 s: - 10xl0...;;x
2
3mA.
dV
From2 to 3 s: 0, :. i=O
di

From 3 to 8 s:

-l.2mA

i(mA)

.

:
I

. I (s)
.
0 l 2 4 5 6 7 •
-1.2

Fig. 2.130
(a) Charge when a steady voltage of 600 Vis applied
CV= !Ox!O...;; x600
..
- , ~·-.
6x10-3 c·
.L .
(b) . Energy stored
~ . ' ' .
u Ans.
& ~ 2.1
I. For each circuit in figure are in:

B
C ~____,C
r R
""C.....J""
(a) (b) ~R
(a) i(R+r) (b) (R+r)

~(R+r)
(c) (d) zero
1R
4. Capacitors C1 an~ C,. (C 1 > C2) ar~ con~ected
to a battery, first md1v1dually;then m series. If
(c) q 1, q2 and q3 are the charges stored on them,
(a) series, series, parallel then
(b) series, parallel, parallel (a) q1=q2=q3 (b) q,=q2<q3
(c) qi> q2 = q3 (d) qi> q2 > qJ
(c) parallel, series, parallel
5. A circuit is connected as shown in the figure
(d) none with the switch S open. When the switch is
2. · Figure shows three circuits, each consisting of a closed, the total amount of charge that flows from
switch and two capacitors, initially charged as ytox
indicated. In which circuit the charge on the left
hand capacitor will change after the closing of
the switch is :
T.
3µF 6µF

3Q 6Q
V
.___ _-i. 9V
(a) 0 (b) 54 µC
(c) 27µC (d) 81 µC
6. A parallel plate capacitor C with plates of unit
area and separation d is filled with a liquid of
(I) (2) d"
dielectric constant 1e= 2. The level ofliquid is
3
6i~q-
CT TJC
initially. Suppose the liquid level decreases at a
constant speed v, the time constant as a function
of time tis
6e0 R
(a) II
5d +3vt

(a)
(c)
I
3
(3)

(b) 2
(d) All
(b)
(15d+9vt) e 0 R
1d2 -3dvt-9v 2t 2
L
6e0 R
3. · A circuit consists of a current source of emf~ (c)
5d-3vt
and internal resistance r, capacitors, each of
capacitance C and resistors, each of value R. (15d-9vt) e 0 R
(d)
The voltage across either capacitor is : 1d 2 + 3dvt -9v2t 2
Two identical capacitors are connected in series 12. The graph given below the variation of electric
with a source of potential V. IfQ is the charge on field E (in MV / m) with time t (in µs).
one of the capacitors, the capacitance of each
capacitor is
(a) Q/2V (b) Q/V 0.8
(c) 2Q IV (d) none of these
8. Given R1 =In, R2 = 2 n, C1 = 2µF and 0.4 ............. ..
,--~
C2 =4µF. The time constani (in µs) for the \
circuits I, II, Ill are respectively
o,__-~4---B~+t
(.' I Consider the following statements :
I. The displacement current through a I m2
C, region perpendicular to the field during the
time interval I =Oto t=4 µsis 0.885 A (given
'
Eo= 8.85X 10- 12 SI unit) \
V 2. The displacement current throu~h I m2
(I) (II) region perpendicular to the field during the
time interval t:4µs to 8µs is zero. Which of
the statements given above is/are correct

R, ~-f!.2-t y C2
(a) I only
(c) Both I and 2
(b) 2 only
(d) Neither-I nor 2.
13. A parallel plate air capacitor is connected to a
battery. After charging .fully, the battery is
.disco~nected and the plates are fulled further
V
(Ill) apar!, Which of the followi~g stateme~ts is
correct?
(a) 18, 8/9,4 (b) 18,4,8/9 (a) The electric field between the plates of
(c) 4,8/9, 18 (d) 8/9, 18,4 capacitor decreases. ··
9. The capacitance of a parallel plate capacitor is (b) The electric field between the plates of
C. If one ofits plates is removed, then capacitance capacitor increases.
(c) The electric field between the plates of
of the isolated plate will be
capacitor remains same.
(a) C (b) 2C (d) Potential difference between the plates
(c) Infinite (d) zero. remains the same.
10. The capacitance of the two identical capacitors 14. Two identical capacitors A and B shown in the
placed in series is 2C. Their capacitance when given circuit are joined in series with a battery. If
they are placed in parallel is : a dielectric slab of dielectric constantK is slipped
(a) C (b) 2C between the plates of capacitor B and battery
(c) 4C (d) BC remain connected, then the energy of capacitor
A will
11. Two plates of equal area A : one square of sides
2l x 2l and other rectangular of sides 4l x L are
used to construct a parallel plate capacitor with
plates separation d. If C is the capacitance, then
e0 A (b) Eo A
(a) 2d d
2e0 A (a) decrease (b) increase
(c) d (dr none of these
(c) remain the same (d) be~omes zero.
CAPACITAN~E AND CAPACllPR

15. The capacity ofa parallel plate condenser is 5µF. 22. A frictionless dielegtrjc plat~ S is kept on a
When a glass plate is placed between the plates frictionless table T. A charged parallel plate
of the conductor, its potential becomes I/8th of capacitance C (of which the plates are
the original value. The value of dielectric frictionless) is kept near it. The plate Sis between
constant will be the plates. When the plate S is left between the
(a) 1.6 (b) 5 plates

16.
(c) 8 (d) 40
The insulated spheres of radii R 1 and R2 having
r-------""c__,,.
charges Q 1 and Q2 respectively are connected
c,;;;=;;;:::=;:;;:~ ,..... '·".,,
to each other. There is
(a) no change in the energy of the system
(b) an increase in the energy of te system (a) it will remain stationary on the tab.le
(c) always a decrease in the energy of the (b) it is.pulled by the capacitor and will pass on
system the other end
(d) a decrease in the energy of the system unless (c) it is pulled between the plates and will remain
Q1R2=Q2R1 there
17. In a charged capacitor, the energy resides (d) all the above statements are false
(a) the positive charges 23. A parallel plate capacitor is charged and the
(b) both the positive and negative charges charging battery is then disconnected. If the
(c) the field between the plates plates of the capacitor are moved further apart
(d) around the edge of the capacitor plates by means of insulating handles, then
18. Eight drops of mercury ofequal radii possessing (a) the charge on the capacitor increases
equal charges combine to form a big drop. Then (b) the voltage across the plates decreases
the capacitance of bigger drop compared to each (c) the capacitance increases
lndividual small drop is (d) the electrostatic energy stored in the
(a) 8 times (b) 4 times capacitor increases.
(c) 2 times (d) 32 times 24. A parallel plate capacitorofplate area A and plate
19. Between the plates of a parallel plate condenser, separation d is charged to potential V and then.
a plate of thickness 11 and dielectric constant k 1 the battery is disconnected. A slab of dielectric
is placed. In the rest of the space, there is another constant k is then inserted between the plates of
plate of thickness 12 and dielectric constant k 2 • the capacitors so as to fill the space between the
The potential difference across the condenser plates. If Q, E and W denote respectively, the
will be magnitude of charge on each plate, the electric
field between the plates (after the slab is inserted)
3nd work done on the system in question in the
process of inserting the slab, then state incorrect
relation from the following
e0 AV
(a) Q= - (b) w
d
20. There is an air filled IpF parallel plate capacitor.
When the plate separation is doubled and the (c) E=-
V
kd (d)
W = EoAV
2d
2
k
(i _.!_)
space is filled with wax, the capacitance increases
to 2pF. The dielectric constant of.wax is 25. Work done by an external agent in separating
(a) 2 (b) 4 the parallel plate capacitor is
(c) 6 (d) 8
21. A 6µF capacitor is charged from IO volts to 20 (a) CV (b)
volts. Increase in energy will be
(a) ]8x]0-4 J (b) 9x]o-4 J
(c) 4.5x]o-4J (d) 9x]C,OJ
(c) .!.cv2 (d) none of these
2
~ti!t1Wi~t~9Ntfi~Gi£~ -}r~:~11-m3~f[-\:f ·?::::_l::'."tf:t!t:,:t~:11¥Jttzif:} :~:r:I',~'.J~~lfilj
26. If there are n capacitors in parallel connected to
V volt source, then the energy stored is equal to

(c)
-
(a) CV (b) .!..ncv2
2

27.
(c) cv2 (d) -
2n
1
cv 2
The work done in placing a charge ofSxto- 18
coulomb on a condenser of capacity I 00 micro- 30.
<• -<>HHHHC-
In an adjoining figure are shown three capacitors
farad is C 1, C2 and C3 joined to a battery. The correct
(a) 32x I o-32 joule (b) I 6x I o-32 joule condition will be (Symbols have their usual
meanings)
(c) 3.tx10-26 joule (d) 4x10- 10 joule
V C~ Q,
28. Two identical capacitors are joined in parallel,
charged to a potential V and then separated and
v,c~- -

C
then connected in series i.e. the positive plate of
one is connected to negative of the other 3 C Q3
3
(a) The charges on the free plates connected - -
together are destroyed V
(b) The charges on the free plates are enhanced (a) Q1 =Qz = Q3 ~nd Vi = V2 = V3 = V -
(c) The energy stored in the system increases (b) Q1 = Q2 +Qi and V = Vi +V2 +V3
(d) The potential difference in the free plates (c) Q1 =Q2 +Q3 andV=Jii+V2
becomes2V
(d) Q2 = Q3 and V2 = V3
29. Seven capacitors each of capacity 2µF are to be 3 I. A capacitor of capacity C I is charged to the
potential of V0.0n disconnecting with ttie
10
so connected to have a total capacity µF. battery, it is connected with a capacitor of
11 capacity C2 as shown in the adjoining figure.
Which will be the necessary figure as shown The ratio of energies before and after the
connection of switch S will• be

(a)
(a) (C1 +C2 )/C1 (b) C1!(C1 +C2 )
(c) c 1C2 (d) c1 I C2
32. Fot_1r condensers are joined as shown in the,
_adjoining figure. The capacity of each is 8µF.
The equivalent capacity between the points A
andB will be

(b)
HHI-
(a) 32µF (b) 2µF
(c) 8µF (d) 16µF
~
'------~~--------~
33. The capacities and connection of five capacitors
CAPACITANCE AND:CAPACITOR

. I
are shown in the adjoining figure. The potential ·' · (a)· (b)_ lµF
difference between the points A and Bis 60 volts. 2µF '

Then the equivalent capacity between A and B (c) 2µF (d) I.33µF
and the charge on 5µF capacitance will be 37. Two condensers C 1 and C2 in a circuit are joined
respectively as shown in figure. The potential of point A is V1
and thatofB is V2. The potential ofpointDwill

d1 1
1 11 1r 11- .,
be

L
1
1
IOµF 1 1
81tF B ·HI
I C I
I
J)
' IH-
c2 , '.!
I
(a) 44µF;300µC (b) 16µF; 150µC I C2f'i +C1V2
(c) 15µF;200µC (d) 4µF;50µC
(a) -(f'i +V2) (b)
2 C1+C2
34. In the following circuit, the resultant capacitance
between A and B is I µF. Then value of C is Cif'i +C2V2 C2f'i -CiV2
(c) (d)
C I 1ir· C1+C2 C1+C2

8
'h,d
A-Jtrll i
4 1tF
38. The charge on a capacitor of capacitance I 0µF
connected as shown in the figure is

""Tiu:siµ~ 2.Q .

e
2 0

B• µ I
11
(a) u
32 F
µ (b) 32µF
2V
23 F 32
(c). (d) -µF (a) 20µC (b) I5µC
32µ 23 .
(c) IOµC (d) zero
35. In the circuit shown here C 1 = 6µF, C2 =3µF and 39. The resultant capacitance of given circuit is

6
battery B = 20 V. The switch S 1 is first closed. It
is then opened and afterwards S2 is closed. What ,------.----!'
2

,h_TcT l
is the charge finally on C2 2
c
~-c-·, I31tF
f---c--'l, ~s, cT Tc Q
J SI (a) JC (b) 2C
~I-B-=_2_0_V_
C
(c) C (d)
(a) 120µC (b) S0µC 3
(c) 40µC (d) 20µC 40. Three plates A, B, C each of area 50 cm2 have
36. The effective capacitance between the points P separation 3mm between A and B and 3mm
and Q of the arrangement shown in the figure is ·between B and C. The energy stored when the
plates are fully charged is

A
B
C 121'

(a) J.6xl()-9 J (b) 2.IxI0--9 J


(c) 5xl()-9 J (d) ?xI0-9 J
-~-,
., ..;'2---l

41. In the circuit shown in figure, each capacitor has 45 . . ln the figure.a potential of+ 1200 Vis given to
a capacity of 3µF. The equivaleni capacity point A and point B is earthed, what is the
between A and B is potential at the ooint P?

A~~''' a

2µF
7
3 (a) IOOV (b) 200V
(a) µF (b) 3µF
4 (c) 400V (d) 600V
(c) 6µF (d) 5µF 46. Five identical plates each ofareaA are joined as
shown in the figure. The distance between the
42. In the circuit here, the steady state voltage across
plates is d. The plates are connected to a
capacitor C is a fraction of the battery e.m.f. The
potential difference of V volts. The charge on
fraction il\ decided by plates I and 4 will be
R, ~-----~
C

2 3 4 5
(a) R 1 only (b) R 1 and R2 only
(c) R 1 and R3 only (d) R" R 2 and R 3
43. Two capacitors C1 and C2 = 2C 1 are connected
in a circuit with a switch between them as shown EoAV _2EoAV
in the figure. Initially the switch is open and C 1 (a)
d d
holds charge Q. The switch is closed. At steady
EoAV _2EoAV
state, the charge on each capacitor will be' (b)
d d
' ·, EoAV -2EoAV
(c)
d d
-EoAV -2EoA V
(d) d d
47. In the figure below, what is the potential
difference between the point A and Band between
B and C resP.ectivelv in steadv state
3µ11· lJtF
13
(a) Q,2Q (b) Q/3,2Q/3
I I
44.
(c) 3Q/2,3Q (d) 2Q/3,4Q/3
Two identical capacitors, have the same
31:.d: l lµF

capacitance C. One of them is charged to


potential)'1 and the other to V2. The negative
lOQ
ends of the capacitors are connected together.
lOQ lOOV
When the positive ends are also connected, the ;1L-_,;;WN,---l ~ --~c
decrease in energy of the c9mbined system is
(a) VAB = VBc = 100V
(a) 1 c (Vj 2 -V22) (b) (b) VAB = 75 V, VBC = 25V
4
(c) VAB = 25V, VBc = 75V
(d) (d) VAB = VBc = 50V
,-
' CAPACITANCE AND CAPACITOR
•J-·
48. A parallel plate capacitor of capacitance C is 51. For the circuit shown, which of the following
connected to a battery and is charged to a 1 ·1 statements is· true?
potential difference V. Another capacitor of .,· ... ,....
= 30V

r
i.· .,., ' V V, = 201-" ·
capacita~ce 2C is connected to ·another battery
. s,· ~I l=-§;i ps,
and is charged to potential difference 2 V. The
charging batteries are now disconnected and the c,=2pFC,=3pFJ_
capacitors are connected in parallel to each other
in such a way that the positive terminal of one is -=-- -=-
connected to the negative terminal of the other. (a) With S 1 closed, V1 = 15 V, V2 = 20 V
The final energy of the configuration is (b) With S3 closed, V1 = V2 = 25 V
(c) With S 1 and S2 closed V1 = V2 = 0
25CV 2
(a) zero . (b) (d) WithS1 andS3 c!osed, V1 =30 V, V2 =20V
6
{ 52. Consider the situation shown in the figure. The
,
(c)
3CV 2
(d) - -
9CV 2 ' · •.-i

I ·, 1
capacitor A has a charge q on it whereas B is
un·charged. The charge appearing on the
2 2
capacitor Ba long time after the switch is closed
49. Condenser A has a capacity of l 5µF when it is is
filled with a medium of dielectric cpnstant 15.
Another condenser B has a capacity of I µF with
air between the plates. Both a;e charged
s6parately by a battery of I 00 V. After charging,
~oth are connected in parallel without the battery
!ind the dielectric medium .being. removed. The
common potential now is /
(a) 400 V (b) ~00 V
+ r,
(c) 1200 V (d) ,1600 V ;
50. In an isolated parallel plate capacitor of " A B
1( , . I.
capacitance C, the four surface have charges · (a) , zero (b) q 12
Q1, Q2, Q3 and Q4 as ~hown. The potential J
(d) 2q
1 (c) q
difference between the plates is ,
I 53. A fully charged capacitor has a capacitance C. It
is discharged through a small coil of resistance
wire embedded in a thermally insulated block of
specific heat capacity s and mass m. If the
Q, temperature of the block is raised by l!.T, the
potential difference V across the capacitance is

msl!.T ~2m~l!.T
(a) -- (b)
C
r '
Q1+l2i+Q3+Qi
(a) ,1 mCl!.T
2C (c) ;_..~2mCl!.T (d)
·s s
,, .....
Q2+Q3 ~

(b) 54:, ... J'\,il µF capacitor, a resistance of2.5 MQ is in


2C
,. . ?series with 12 Vbattery. Finsf the time after which
Qz-Q3 • , ,the potential difference across the capacitor is 3
(c) times the potential difference across the resistor.
2C
' :(Given fn(2) = 0.693] \
·Q1+Q4 (a) 13.86s (b) 6.93s
(d)
2C; (c) 7 s (d) 14s
\' '
ELECTRICiIY & 1\-fu:GNETISM

55. Figure (a) shows two capacitors connected in


series and joined to a battery. The graph in figure
(b) shows the variation in potential as one
moves from left to right on the branch containing
the capacitors, if

C:J, c, C. ~'
(a) - (b)

(a) C1 > C2
X
(b) C 1 = C2
(c) C 1 < c2
(d) The information is not sufficient to decide
the relation between C 1 and C2
56. In an RC circuit while charging, the graph oflog
i versus time is as shown by the dotted line in (d)
I
the figure, where i is the current. When the value
of the resistance is doubled, which of the solid
I
curve best represents the variation oflog i versus
time
y
log i
59. Two parallel plate capacitorofcapacitance C and
k:::=--s 2C are connected in parallel and charged to a
R
potential difference V. The battery is then
disconnected and the region between the plates
of the capacitor C is completely filled with a
material of dielectric constant k. The potential
difference across the capacitors now becomes :
(a) P (b) Q
2V 3V
(c) R (d) S (a) (b)
57. Two oppositely charged metal spheres of the k k
same radius (R) are placed far apart. The 3V 2V
capacitance of these two spheres is : (c)
k+2 (d) k+3
(a) 4 m 0 R (b) 8 m 0 R
60. A leaky parallel plate capacitor is filled completely
(c) 2 7!E0R (d) m 0R
58. with a material having dielectric constant k,
Four ways of making a network of five capacitors
electrical conductivity cr. The charge on the plate
of the same value are shown here. Three out of
initially is q0 and the leakage current at time I ls i.
four are identical. The one which is different is :
The time constant of the circuit is 't:
E k . qocr
(a)~=_o_ (b) 1=--
(a)
cr Eok
X y

(c) .= ,. (:~k)
1 0e
(d)
. qocr
to=---
Eok
_L___ :_-------=-- -----------,--C,...AP._1-t_c_rr._;;lN_C_E_AN_D_C_AP._1-t_c_rf_o_R_ _
11 ,i ·
~~~·
61. Two conducting shells of radius a and b are 64. , ,When a de voltage is applied to a capacitor, the
connected by conducting wire as shown in , voltage across its terminals is found to build up
figure. 'The capacity of system is : 1
ip accordance with Ve= 150 (1 - e-20 '). 1f the
current is 1.14 mA after 0.05 s, the capacitance of
the capacitor is :
(a) 3.01 µF (b) 0.03 µF
(c) l.03µF (d) I0.3µF
65. A circuit element is placed in a black box.At t=O,
a switch is closed and the current flowing
through the circuit element and the voltage
across its terminals are recorded to have the wave
(b) 4 llt0 (a+ b) shapes shown in the figure here. The type of
element and its magnitude are:
(c) zero (d) infinite
62. The given circuit consists of a capacitor and i (1)
resistors. The time constant of the circuit is : amp
·~'1L 1.01----~

I le r r
,----~----·1(s)
,, , 0 4
ril
•' ' V (I)

I 4.0 ............._ __
(a) -s (b) 4 s
4
(c) 2s (d) I s

63. A capacitor of capacitance f F is initially OJL-----',4----.1(s)


charged to a potential difference of IO V, and
then the switch Sis closed. The potential (a) inductance of 4 H
difference across the capacitor at/= I sis: (b) resistance of 4 Q
(c) capacitance of I F
(d) avoltagesourceofemf4 V

6Q 3Q

(a) 2.0V (b) 4.0V

(c) O.SV (d) 0.4 V

\
f._nJJ . f;Ei.ECTRIC11Y 8lili.AGNETISM · . -- •.
-----------------,.-========~-_;.-·_·~~======----~·: ~l

Directions (Qs. 1-5) : Read the Jo/lowing questions 4. The separation between the plates of a charged
and select the right choices. More than one options parallel plate capacitor is increased. Which of
may be correct. the following quantities will change ?
(a) charge on the capacitor
1. ' a
A capacitor with no dielectric is connected to (b) potential difference across the capacitor
battery at t=O. Consider a point A in the (c) energy of the capacitor
connecting wires and a point B in between the (d) energy density between the plates of the
plates. capacitor.
5. Which one statement is correct? A parallel plate
(a) There is no current through A.
air condenser is connected with a battery. Its
(b) There is displacement current through B till charge, potential, electric field and energy are
electric field changes between the plates. Q0 , V0 , E0 and V 0 respectively. In order to fill the
(c) There is a current through A as long as the complete space between the plates a dielectric
charging is not complete. i slab is inserted, the battery is still connected.
(d) The current always flows between the Now the corresponding values Q, V, E and V are
plates of capacitor. in relation with_ the initially stated as
2. A capacitor C 1 of capacitance IµF and a (a) Q>Q0 (b) V>V0
capacitor C2 of capacitance 2µF are s,eparately (c) E >£0 (d) V> V0
charged by a common battery for a long time.
Passage for Q. No. 6 to Q. No. 8
The two capacitors are then separately
A system consists of two concentric thin spherical
discharged through equal resistors. Both- the
shells of radii a and b (b > a).
discharge circuits are connected at t=O.
(a) The current in each of the two discharging
circuits is zero at t=O.
(b) The currents in the two discharging circuits
at t=O are equal but not zero.
(c) The currents in the two discharging circuits
6.
®)
at t=O are unequal. Outer shell is earthed and inner shell is given
(d) C 1 losses 50% of its initial charge sooner charge Q, then charge on the outer shell is :
than C2 loses 50% of its initial charge. Q
3. Each plate ofa parallel plate capacitor has a charge (a) zero (b)
2
q on it. The capacitor is now connected to a
battery. Now Qb
(c) (d) -Q
(a) the facing surfaces of the capacitor have a
equal and opposite charges. 7. !finner shell is earthed and charge Q is given to
(b) the two plates of the capacitor have equal the outer shell, then charge on the inner shell
is:
and opposite charges.
(c) the battery supplies equal and opposite _Qa
(a) zero (b)
charges to the two plates. b
(d) The outer surfaces of the plates have equal
Qb
charges. (c) (d) none of these
a
8. The capacitance of the system in second case : 9. The charge on the each capacitor before closing
the switch is :
·· ( ab )
(a) 41tEo. - - (b) 41te0 (a+b)
b-a .

(c) · 47tEQ b (c) 41te 0 ( L)


b-a
',,

(c)

Passage for Q:9 to Q. 11


1O; The switch Sis now closed, the amount of charge
")) '
flows through the section I:
In the circuit shown in the figure the emfofeach battery .. '
' capacitors of capacitances C and C are
is 1; and "i 1; ci(c1 -C2 )
1 2
connected as. shown. (a)
(C1+C2)
I ~ J' "!'()

,,;1,, (c)
; _, I •p,

11. The amount of charge flows through section 2 :


"~ ) ''
l;C1(C1-C2) l;Cz (C1-C2)
(a) (b)
I' C1+C2 (q +Cz)
'"
. '"', (c) zero (d) none of these
,, lff}

Multiple Matching
12. When a·dielectric slab is inserted between the plates ofbne of the two identical capacitors shown in the
figure then match the following:

j___A
1
=Is T
'.!:) g-
EI
Column-I Column-IT
(A) Charge on A ' J_g (p) increases
(B) Potential difference "·,' (q) decreases
across A -·' J ,.

(C)' Potential difference (r) remains constant


across B
(D) Charge on B (s) will change

:,.:

)',; 1'
·.'!'1,.,t:"'1:;e11{{~'
· <w:111:\"
--r-t-.-J;~MCQ
,c n~•,Y Tvpe 3
,:,\1,~ ·--·· :J. ....: ·. .
~

Read. iii
iw~~;t~te~~~ts ca;,r~11y1i> trt;~f
th~~~~ct opti~n'o'u1~itlie optiorts g,;~~bei~~--s~ie~t th~\;gii~
choice. . · , . . . ' . . · ·, j
(a) If both the statements are true and the Statement- 2 is the correctexplauation ofStatement -1, ,,
f • t" : ' ' '.' • . . • "' - • . - ,, ' '
(b) If both the statements are true but Statement - 2 is not the correct explahat10n of the Statement-I,
(c) lfStateme~t-1 truebutSttitemen(-2isfalse .. • · · · · ·;
(d) IfState~ent- l is false butStatement-2 is'true. _, 0
r- --• ~ - - ·. . - - - ~ ~ ~ - ' " - . - - - · • -~-~'~J·~-

I. Statement 1 7: 'statement - I
Second conductor in· the capacitor increases its T\vometal plates having charges Q, -Q face each
capacitance. other at some separation and are dipped into an
Statement - 2 oil tank. Iftheoil is pumped out, the electric field
Second conductor decreases the potential between the pl_ates increases.
difference between the conductors. Statement - 2
2. Statement c 1
The capacitance of a capacitor with charge Q on _ Eair
E med-~
its one plate is C
Statement- 2 8. Statement - 1
The capacitance of a capacitor with charge 2Q
A charge Q 1 is given to the one plate and ·a
on its one plate will be 2C.
charge-Q2 to the other plate ofa capacitor, The.
3. Statement - 1
charge on ,the capacitor is Q1-Q2•
Capacitance C = Q; it means C is proportional Statement - 2
V
toQ. The charge on the capacitor is
Statement - 2
Charge Q = CV; it means charge on the capacitor 9. Statement - 1
is proportional to V. . The plates ofa parallel plate capacitor are given
4. Statement - 1 equal positive charges. The charge on the facing ·
Electric energy resides out of the spherical surfaces of the plates are zero.
isolated conductor. Statement- 2
Statement - 2 The potential difference between the plates is
The electric field at any point inside ·the
zero.
conductor is zero.
IO. Statement - I
5. Statement - 1
The electric energy resides between the plates When a dielectric slab is gradually ,inserted
of a capacitor. between the plates of an isolated parallel-plate
Statement - ·2 capacitor, the energy of the system.decreases,
The electric field at any point outside the plates Statement - 2
is zero. The force between the p!ates_ decreases, ·
6. Statement - 1 11. Statement - 1 .
A parallel plate capacitor is connected across A dielectric is inserted between the plates of a
battery through a key. A dielectric slab of battery connected capacitor. The energy of the
dielectric constant k is introduced between the capacitor increases.
plates. The energy stored becomes k times. Statement - 2
Statement - 2
The surface density of charge on the plate
remain constant.
CAPACITANCE AND:CilJ\PACITOR . Pi

1 (b, c) 4 (b, c). 7 (b)


I. 10 (a)
2 (b, d) 5 (a, d) 8 (d)
I 11 (b)
3 (a, c) 6 (d) 9 (d)

12. A-p, s; 8-p, s; C-q, s; D-p, s

1 (a) 4 (a) 7 (a) 10 (c)


2 (c) 5 (a) 8 (d) 11 (a)
3 (d) 6 (c) 9 (b)
·~ . , ~--..
,.,...--- __ --· . -

=---~
,,, . - -

\C:tri5i,_tY_.'--·-·~--* U_!3 JECTIVE & ~ 2.4


"-_..i"

I. Find the charged supplied by the battery in the arrangement shown in figure.
5.0µF 6.0µF

2.
:·J
Take the potential of the point B in figure to be zero.
Ans. l lOµC.

(a) Find the potentials at the points C and D.


(b) !fa capacitor is connected between C and D, what charge will appearpn this capacitor?
4µF SµF
I I
A B

501'

+
50
Ans. µV ateachpoint(b) zero.
3
3. The particle Pshown in figure has a mass of IO mg and a charge of - 0.01 µC. Each plate has a surface area
100 cm2 on one side. What potential difference V should be applied to the combination to hold the
particle Pin equilibrium?

~p

l_j .4ns.43mV
4. In figure each capacitance C 1 = 2 µF and C2 = 3 µF.

a--llrct-rc7
b"-4
C2
J:' ~ ~ c, C2
d
Cz
(a) Calculate equivalent capacitance of the network between points a and b.
(b) Calculate the chatge on each of capacitors nearest to a and b when p.d. across a and b.
Va{,= 900 volt.
(c) With 900 volts across a and b, calculate the p.d. across c and d [i.e. Vcefl ·-
, Ans. (a) I µF (b) 900 µC(c) l00 V.
___________c__AP._~_C_IT._l\N_CE-~-D-C_AP._~~-;;~;;-
__ -_~

5. Take C 1 = 4.0 µF and C2 = 6.0 µFin figure. Calculate the equivalent capacitance of the combination
between the points indicated.

:r:
clc, Ans. (a)5µF,(b) I0µF.
c, c, C, C2
(a) - (bi
6. Find the equivalent capacitances of the combinations shown in figure between the indicated points.

lµyµ}F
4µFT
-e,7,,,1 I -
':§-FF
I 11
{
f--1--i 4µ FT Ans. (a)
6 µF,
3µF lµF J J
(a) 3µF (b) lµF
(b}-¥- µF.
7. Find the equivalent capacitances of the combinations shown in figure between the indicated points.
2µF 4µF
2µF 4µF I I I
4µ£ 6µF I
I I I
I I I
SµF
T5µF
--+----1 f-1 ~-~-;J 1--+--- ~SµF
4µF SµF
4gF ' µr::!: '
6
4µF I I I
(a) 2µF 4µF
(bl
Ans. (a) 8 µF, (b) 8 µF.
8. Find the potential difference Va - Vb between the points a and b shown in each part of the figure.

~r I _117a b/1\
12VT I4µF Tb4µF 2µ1 T4µF

<a> --~I__/
24-V +
(b)

2v
I
a J2_v

C
1-

2µ1-F_...,T,____,2µF Ans.(a) 12/11 V,(b)-8V,(c)zero.


i, 1
(c)
9. Find the poteniial difference V0 - Vb between the points a and b shown in each part of the figure.

Ans. -10.3 V.
10. Four capacitors of equal capacitance are joined in series with a battery of IO volts. The mid point B of
these capacitors is earthed. Calculate the potentials of A and C.

C +'':J
A I
IOV

l 1----1 C

Ans. VA=+5Vand Vc=-5 V.


11. An infinite circuit is formed by the repetition of same link consisting of two identical capacitors. each of
capacitance C. Calculate the effective capacitance between points x and y.

Ans. 0.618 C.
12. Infinite number of identical capacitors (each of capacity I µF) are connected as shown in figure. Find the
equivalerit capacitance of system between the terminals shown in figure.

C C
l I
C C C C
1----11----11---1
C C C C C C C C
HHHHHHH
HHH•·········IH
16 Capacitors
H H •············1 H
32 Capacitors

Ans. 2µF.
-------------- ------- ---------------- -------------~
, ____ CAPACITANCE AND CAPACITOR
~-- - ---- --- ----- ··--- -- -- ----------------
13. Find the equivalent capacitance of given network of capacitors between A and B. Each capacitor has
capacitance C= 1 µF. '' '

A11s. 0.38 µF.


14. A finite ladder is constructed by connecting several sections of 2 µF, 4 µF capacitor combinations as
shown in figure. It is terminated by a capacitor of capacitance C. What value should be chosen for C,
such that the equivalent capacitance of the ladder between the points A and B becomes independent of
the number of sections in between?

A11s. 4µF.
15. A metal sphere of radius R is charged to a potential V.
(a) Find the electrostatic energy stored in the electric field within a concentric sphere of radius 2 R.
(b) .Show that the electrostatic field energy stored outside the sphere of radius 2 R equals that stored
within it.
Ans. (a) n:E 0Rv 2.
16. A parallel-plate capacitor having plate area 20 cm2 and separation between the plates 1.00 mm is connected
to a battery of 12.0 V. The plates are pulled apart to increase the separation to 2.0 mm.
(a) Calculate the charge flown through the circuit during the process.
(b} How much energy is absorbed by the battery during the process ?
(c) Calculate the stored energy in the electric field before and after the process. ·
(d) Using the expression for the force between the plates, find the work done by the person pulling the
plates apart.
(e) Show and justify that no heat is produced during this transfer of charge as the separation is
increased. ·
Ans.(a)I.06x 10- 10 c(b)l2.7x 10- 10 J,(c)l2.7x 10- 10 J,6.35x 10-IOJ.(d)6.35x 10- 10 1.
17. Figure shows two parallel plate capacitors with fixed plates and connected to two batteries. The separation
between the plates is the same for the two capacitors. The plates are rectangular in shape with width b
and and lengths / 1 and /2. The left halfofthe dielectric slab has a dielectric constantk 1and the right half
k2. Neglecting any friction, find the ratio of the emfofthe left battery to that of the right battery for which
the dielectric slab may remain in equilibrium.

1---1,---, 1---iz---,

t kI k,
t Ans.,
t2f
18. A 5.0 µF capacitor is charged to 12 V. The positive plate of this capacitor is now connected to the
negative terminal of a 12 Vbattery and vice versa. Calculate the heat developed in the connecting wires.
Ans. 1.44 mJ.
19. A capacitor stores 50 µC charge when connected across a battery. When the gap between the plates is
filled with a dielectric, a charge of I 00 µC flows through the battery. Find the dielectric constant of the
material inserted.
Ans.3.
20. A radioactive source in the form of a metal sphere of diameter I o-3 m emits p particles at a constant rate
of 6.25 x IO Io particles per second. If the source is electrically insulated, how long will it take for its
potential to rise by 1.0 volt, assuming that 80% of emitted p particles escape from the surface.
Ans. 6.95 µs.
21. The capacitance of a variable radio capacitor can be changed from 50 pF to 950 pF by turning the dial
from o0 to I 80°. With the di~I set at I 8o0, the capacitor is connected to a400 V battery.After charging, the
capacitor is disconnected from the battery and the dial is turned at o0.
(a) What is the potential difference across the capacitor when the dials reads o0 ?
(b) How much work is required to turn the dial, if friction is neglected?
Ans. (a)7600 V(b) 1.368 x 10-3 J.
22. A battery of IO Vis connected to a capacitor of capacity 0.1 F. The battery is now removed and this
capacitor is connected to a second uncharged capacitor. If the charge distributes equally on these two
capacitors, find the total energy stored in the two capacitors. Further compare this energy with the initial
energy stored in the capacitors.
Ans. 5J, 1/2.
23. Two square metallic plates of I m side are kept 0.0 I m apart, like a parallel plate capacitor, in air in such a
way that one of their edges is perpendicular to an oil surface in a tank filled with in an insulating oil. The
plates_ are connected to a battery of emf 500 V. The plates are then lowered vertically into the oil at a
speed of0.001 mis. Calculate the current drawn from the battery during the process (dielectric constant
ofoil = 11, Eo = 8.85 x 10-12 c2/N-m2).

-. -. ..
. . -. -. .. .. .. .. -. ..
T

. -.
.:: - ........
- - .. .. Ans. 4.425 x 10·9 A.
:::::::: :::: :::::;::
- - -... --.-
-- --.. .. -- .- -. .- -. .- -- .- -....
- .. .-
: : : : : : : : .. : ii~:::::.::::::::
24. Two parallel plate capacitors A and B have the same separation d= 8.85 x J0-4 m between the plates. The
plate areas of A and B are 0.04 m2 and 0.02 m2 respectively. A slab of dielectric constant (relative
permittivity) k = 9 has dimensions such that it can exactly fill the space between the plates of capacitor
B.
(i) The same dielectric slab is now placed inside A as shown in figure. A is then charged to a potential
difference of 110 V. Calculate the capacitance of A and the energy stored in it.
(ii) The battery is disconnected and then the dielectric slab is removed from A. Find the work done by
the external agancy is removing the slab from A.
CAPACITANCE AND CAPACITOR

, (iii) The same dielectric slab is now placed inside B, filling it completely.The two capacitors A and Bare
then connected as shown in figure. Calculate the energy, stored in the.system.

~
L.i~
I IOV \a) (bl
Ans. (i) 2 x 10-9 F, 1.21 x I o-5 J (ii) 6.05 x 10-9 J, 4.84 x I o-5 J (iii) 100 v, I.I x 10-5 J.
25. (a) Find the capacitance of a spherical capacitor, if the outer sphere is charged and the inner sphere is
earthed. Given radius of inner sphere= a, radius of outer sphere = b.
(b) Two conducting spheres are placed concentrically. The inner sphere is earthed and the_ outer given
a charge 30 nC. The radii of sphere are 8 cm and IO cm. What is the charge on the inner sphere? Will
there be any charge on the.inner surface of the outer sphere? Explain.
+

+
Ans. (a) 41tEo ·( -b2-) , (b)-24 nC.
b-a

'.j-
2,6. The lower plate of a parallel plate capacitor lies on an insulating plane. The upper plate is suspended
from one end of a balance. The two plates are joined together by a thin wire and subsequently
disconnected. The balance is achieved. A voltage V = 5000 V is applied between the plates, what
additional weight should be placed to maintain the balance? The separation between the plates d= 5 mm
andtheareaofeachplate,A= 100cm2 . ·. _ _

l. .
l
+++++++!
.
-------
d
Ans. 4.52 g.

27. The two capacitors C 1 and C2 are made of square plates of edge/. The separation between the plates of
capacitors are d 1 and d 2 as shown in figure. A potential difference Vis applied between the points a and
b. An electron is projected between the plates of the upper capacitor along the central line. With what
minimum speed should the electron be projected so that it does not collide with any plate. Consider the
effect of only dielectric forces.
a

--------------·
c,----~---
.. !",.
Ci----'-------'-!. d, eV/ 2 ]t/2
Ans. [
----,--'~--

b
28. Find the potential difference between points A and B of the system shown in figure. If emf is equal to I;

. = 110 V and the capacitance ratio Cz = 11 = 2.0 .


c,
I; I0V
Ans. (I +311 +,r/)

1:q~

-_r.1~;.!~''I'q'l~,·A
Hint: In close l_oop 1-2-5-6-1
~r -(q,+qf T'
6 5 4
.B

q, (q,+qz)+l;=O
C1 C2
and in close loop 1-2-3-4-5-6

- !!L + .'ll_ + .'ll. = 0


c, c, Cz

Solving above equations for q2 . VAB = _.'ll.


. Cz
29. A circuit has a section AB shown in figure. The emf of the source I;= IO V, the capacitor capacitances are
equal to C 1 = 1.0 µF, C2 = 2.0 µF, and the potential difference VA - V8 = 5.0 V. Find the voltage across each
capacitor.

A -----l l----=.'1' I+ : 11-s


Ci C2
·q q (VA -Vo+ /;)C1C2
Ans. Vi =-=I0V, V2 =-=5V,where q=~~~~~~
c, C2 (C1+ C2 )

Hint: L+1;-JL=5.0; Vi =L & Vz =JL


C1 C2 C1 C2
30. In a circuit shown in figure find the potential difference between the left and right plates of each
capacitor.

Hint: Charge on each capacitor will be same, let it is q,

_L+1;,-JL-l;=O
C1 C2 .

[I;, -l;2JC1C2
q=
C1 +C2

Vi =.5L & Vz = _L c,
c, c,
31. Determine the potential difference VA - VB between·pointsA and B of the circuit shown in figure. Under
what condition is it equal to zero ?

c, A
~---i 1 •

- +

~(C2C3 -C1C4) C1 C3
Ans.VA-VB= (Ci+Cz)(C +c ).lnthecasewhen Cz = c .
3 4 4
32. Find the potential difference VA - VB between points A and B of the circuit shown in figure.

33. Determine the potential at point I of the circuit shown in figure assuming the potential at the point Oto
be equal to zero. Using the symmetry of the formula obtained, write the expressions for the potentials at
point 2 and 3.

~'I+ c,
~2C2 +~3C3-~1(C2 +C3)
Ans. Vi •
2 c, 0
(C1 +C2 +C3)

~'I+ 3 c,

~1C1 +~2Cz-~3(C1 +C2)


C1 +C2 +C3
34. What charges will flow after the shorting of the switch Sin the circuit illustrated as shown in the figure
through sections I and 2 in the directions indicated by the arrows ?

Ans. q1 = ~C2 , q2 =
35. A conducting sphere S 1 of radius r is attached to an insulating handle. Another conducting sph~rf' S2
of radius R is mounted as an insulating stand, S2 is initially uncharged. S 1 is given a charge Q, brought
into contact with S2 and removed. S 1 is recharged such that the charge on it is again Q and is again
brought into contact with S2 and removed. This procedure is repeated n times.
(a) Find the electrostatic energy of S2 after n such contacts with S 1.
(b) What is the limiting value of this energy as n---; =.

2
Ans.(a)Un= qn , q .QR R -)"] (b) Q2 R
=[- l(- -
81t e 0 R r R+r 81te 0 r 2
36. The circular plates A and B ofa parallel plate air capacitor have a diameterof 0.1 m and are 2 x 10-3 m apart.
The plates C and D, of a similar capacitor have a diameter of0.12 m and are 3 x I o-3 m apart. Plate A is
earthed. Plates B and D are connected together. Plate C is connected to the positive pole of a 120 V
battery whose negative is earthed. Calculate.- .
~

(a) the combined capacitance of the arr~ngement and


(b) the energy stored in it. ·
HH12ov
_[7. CI 7 C,- .~: _il_
-=- I•

-=-·
-
Ans.(a) l7µF,(b) l2.24x 10-81.

I
i•I ',

t1 / 1

'
l. - - - ,:)--
0 Positive terminal

Steel top plate


0
0 Positive electrode

0 Separator

0 Negative electrode

l,
'.!i '
''
.
D1r.ect Current and
'
Negative terminal

rrt~=IEla~tric ·Cirtcuits~
3.1 INTRODUCTION
When you tum on a light, you are using the sort of electricity that flows along the wires, like water flows along
pipes. This is called current electricity. It is usually made up of billions of electrons flowing along a wire. These
electrons do not move along a wire by themselves. They have to be pushed by potential difference, produced
by the battery.
Inside a wire
J\n ell!ctric cummt consists of billions of free electrons, flowing along a wire. ·n1e electrons "hop" from one atom to the
next, tra\'clling in short bursts. lndi,·idual electrons mo\'c only fractions of a millimetre each
second. But like pushing a row of railway wagons, they at the speed of light, 300,000
kilometres per second.
Direct current, DC
In DC, all the electrons mo,·c in thi.: same <lircction. in all the
time that the ch.:ctricity llows. This type of current is
pro<lw:L-<l by the battL-ri1..-s in tord1L-s cars an<l
similar <lc,·kcs.
overing
F.lectmns or insularion
movi11g stops electrons
along escaping from
wire
F,ach Alternating current, AC
atom has In AC, the direction of electron
a cenrra/ nucleus movement changes many times each
and on area where second. 1l1c ckctrons movi:. one way,
elec/runs orbit thL.'11 the othc..T~ and so on.
Electric current
In an isolated metallic conductor, the free electrons move randomly like the molecules of a gas and so the net
rate of flow of charge through any hypothetisal,plane is zero. If potential difference is applied across the ends
of the conductor, an electric field is set up at every point within the conductor. This field exerts force on free
electrons in opposite to direction of field and will give them a resultant motion. This flow of electrons constitutes
an electric current. It can be defined as the rate at which charge passes through any ~pecified surface area. Thus
- -=-dq-
+o 10-+o a-+
dt.'
6 +-0 .j, a.,.
Conductor without p.d.
SI unit of electric current is C/s. I C/s = I A.
'
Fig. 3.1
We can get the charge that passes through any cross-section of the
wire in time interval from t I to t2 by integration. i.e.,

_+-e+-e+-e
_ _ _,..E
,, +-e+e+-e
q Jidt.
,, Movement
'----+-9-.,__+,'-! ,_-_ _..,.,__~ of electrons
r r·
V
Conductor with p.d.

n Fig. 3.2
More about electric current
I. The current is the same for all cross-sections of a conductor
of non-unifonn cross-sections. It is similar to the water flow.
Charge flows faster where the conducto~ issmaller in cross-
section ~nd slower where the conductor has larger cross-
section.
Fig. 3.3
2 As electrons move faster than positive ions, so the net current is due to electrons. This type of current
I • . .
is known as conduction current.
3. In liquids the flow of current occur due to;both types of ions. In gases it is due to positive ions and
elec~rons. In semiconductors, it is due to fl~,_; of holes and electrons.

1
IA .!£
s
=( I.6x10- 19 ) electrons/s

4.
\ =.6.25 x 10 18 electron/s
Current constitutes by an electron: Let an electron is moving on a circular path of radius r with a speed
v. The current due to its motion at any ~ect\on of path,

'f_
t r

e ev
21tr = 21tr
V
Fig. 3.4
DIRECT Cu~H!'NT-AND.ELE~l~r~lffCEUffSi·..
•,,
,,~11
5. If charge q is distributed over the ring uniformly or non-uniformly,
current at any section of ring

q "d''q,
1'" ·zit'/ ro
~ (L, ~

qro ,, i:i,
q
21t 'I,
Fig. 3.5
6. Direct current (de) and alternating current (ac):

de

~-----__.,

(a) (b)
Fig. 3.6
Current density (7)
It is a characteristic of a point inside a conductor rather than the conductor as a whole. It is defined as the
current per unit area through an infinitesimal area normal to the direction of current flow.
I. The current density at a point P is given by

di'
J -n
dA

l---'--ir (~~)
(a)
o~~-----·-.r..
. (b)
fi

Fig. 3.7
2 If cross-sectional area is not normal to the current, then
di
J=--- or ; = JJ,i::i.
dAcose ,JI ~ ;

3. For uniform distribution of current J ' '


4. It is a vector quantity. Its SI unit is A/m2 •
Drift velocity
When an electric field is applied in an conductor, a force acts on each electron in
the direction opposite to the field. The electrons get bised in their random motion
in favour of the force .. As a result, the electrons drift slowly in this direction. If the
e
electron drifts a distance in a long timer, we can define drift velocity (average
~.~.
---·-_-_
...
...-..-
C
velocity) as : e E
Fig. 3. 8 Dotted path represe11ts the
motio11 ofelectro11 witl,jield E·
-·---- ------
If 't is the average time between two successive collisions, then the distance drifted in this time

e .!._ 01 2 =.!._(£e)t2
2 2 m

f eEt
:. Drift velocity -=--
t 2m
The order of drift velocity is I o--4 mis. It is very small in comparison to thermal speed ( = I o5 mis ) of electron
at room temperature.
Now consider a cylindrical c~nductor of cross-sectional area A placed in electric field E . Take a small length
vdAI of the conductor. The volume of this portion is Avd!J.t. Let n is the number of free electrons per unit
. l. ~ ' •

volume of the wire, then total free electrons in this·volume is nAvd/i/


The charge crossing this area nAvd!J.te
L· ,,,
or !J.q = neAvd
!J.t
i
and -=nevd Fig. 3.9
A
.:! ,.1·
More about drift velocity
'.
. i l ·- ' • , •

We can wnte vd =--. " '


neA

I
(i) If a constant current flows through1a,.cOnductor of non-uniform cross-section, then vd oc -
A
ViR
(ii) Also, vd = - - = - -
neA neA

V V
pfne
,. '
Clearly if conductors of different cross-sectional area are connected with same p.d., the drift velocity is
independent of cross-sectional area.
L
Microscopic view of Ohm's law E = pJ :
eEt
We have got
2m
and . J ne vd
From above equations, we get

(11?
J (n;:t) E

or E = c:;JJ
or ,._ ,', ·= pJ-- -I
--"E-·--·- "'
Ohm's law
2m
where P = - 2- , is called resistivity of the material.
· ne 't · .,. , " .

,----- -i- !
The conductivity of material is defined as I cr = -p ..,,,
L- . - -

1
The quantity R is called conductance.

3.2 0HM 1S LAW -· ,JOO "'

If the physical conditions of conductor (temperature,· le~gtn etc.) do not change, then the cu,;.ent in the
conductor is directly proportional to the potential differen_ce across the conductor.
Resistivity ( P I
It can be defined as the ratio of electric field appl,ied to current density.Thus
r-----E T
IL. -
p

= -
--·-·J__J
I

Resistivity is the property of material which does not depend on size and shape of the conductor. SI unit of
resistivity is Ohm-meter (Q -m). A perfect conductor would have zero resistivity and a perfect insulator have
0

infinite resistivity. Metals and alloys have the lowest resistivities and hence they are the best conductors. It is,
a familiar fact that good electrical conductors, such as metals, are good conductors of heat, while poor electrical
conductors such as plastics are also poor thermal conductor.
Variation of resistivity with temperature
The resistivity of metallic conductors ~ncreases with increllsin-grtemperature as
P, = Po [l+at+Pt 2 ]
where Po resistivity of conductor at 0° C
P, = resistivity of conductor at I° C
is the temperature difference between t and 0° C.
and a. & ~ are temperature coefficients ?f resistance ;,, 1,, ,, JJ';.,

a> P, and positive for metals and negative for non-metals.


p p

Metals \,
Non metals

~-----,
(a) (b)
Fig._ 3.10
\
-------------'----7
•?&## The resistivity
- . -
of the alloy manganin is practically independent of the temperature .
..__ -- - - - --·-- --- -·--·-- -- - -----
. -
~--,~-iii~~ECTRICI1Y"~~ItiJ1g~ETISM· li~;¥,,;:,i.~ •. 0

Resistance
In the relation E = pJ, it is difficult to measure E and J directly. It is, therefore very convenient to put this
relation in macro parameters like current and potential difference. To do this let us consider a conductor of
length eand cross-sectional area A as shown in fig. 3. 11.
,.____ t -----
We can write E = pJ
V i
As E=-andJ=-
e A

V i
p- Fig. 3.11
f A

or V = -(Pf)
I A =iR
This relation is often referred to as Ohm's law.
r pf
where R is called resistance of the specimen and is equal to ,R =A.
The SI unit of R is ohm ( Q ). Its symbol is--"Wlv-.
The material obeying Ohm's law is called ohmic conducto; or a linear conductor. lfohm's law is not obeyed, then
the conductor is non-Ohmic Or non linear.

V V

(a) Ohmic conductor (b) )'Ion-ohmic conductor


Fig.3.12
Fig 3.12 (a) is a curve for metalic conductor. The straight line shows that the resistance of conductor is the
same no matter what applied voltage is used to measure it.fig.3.12 (b) is not straight line and the resistance
depends on voltage applied, the resistance is called dynamic resistance and the conductor is called non-ohmic
conductor.
Thermistor
Thennistor is the heat sensitive resistor. It is usually made by semiconducting material for which V- i plot is not
0
linear. The temperature coefficient ofthennistor is negative and usually large. It is of the order of -0.04/ C.

Fig. 3.13
Thennistors are used for making resistance thennometers which can measure very low temperature.
D1RECTCURHENT AND ELECTRIC CIRCUITS

;w;w 1. Resistance depends on length of wire not on the shape of the wire.

A
e
B A
A B
Fig. 3.14
All the wires have same resistance between A & B. f is the straight length of wire between its ends.
2 V = i R is the general relation between i and V which is applicable to both type of conductors, ohmic as
well as non-ohmic:

Variation of resistance with temperature


As the resistance of any specimen is proportional to its resistivity, which varies with temperature, and also
resistance varies with temperature. Therefore we have
2
R1 = Ro (l+W+~t )
for small I, R, --
- __ = R0.(l+a.t)
.. -
Temperature coefficient of resistance (a)
If R 1 and R2 are the values ofresistance at temperature 11 and 12 respectively, then
R1 R 0 (1 +a.t1 } and R2 = R 0 (1 +a.t2 )

~ I+a1I
R2 l+a12

R2-R1
or a
Rifi-R2t 1

We have assumed a to be constant for all temperature. But actually it varies with temp.

If R1 is the resistance at any temperature, then R1 =R 0 (I+ at), differentiating above equation w.r.t temp,
we get

a = _I (dR1 )
Ro di .

Resistance of conductor of non-uniform cross-section


Considera conductor oflength t and radius at its ends are r 1 and r2• The resistance ofelement under consideration
pdx
dR
rcr 2
r '2
r
where, r = lj+ c·2;1,. ) X
·------- -----------
1---X .
r
pdx l
dR Fig. 3.15

rc[rl +( '2;ri X)
Resistance of whole conductor
e e
R JdR= J pdx
o o [· (r -ri) ]
2
2
7t 11+ - - X
e

or R

Why conductor offers resistance?


Resistance means the hinderance offered to the flow of charge. Electrons in their motion collide with the
positive ions and themselves, due to which resistance in motion occurs. The.resistance mainly occurs due to
collisions 9f electrons with the positive ions.
Super conductors
Kamerlingh found that mercury offers zero resistance at 4.2 K. This phenomenon is called super conductivity
and the metal is called superconductor. Certain alloys become superconductors at rather high temperature. The
resistance of material in the superconducting state is zero and the currents once established in closed
superconducting circuits persist for weeks, even though there is no battery in the circuit.

Fig. 3.16 Resistance-temperattlre graph


for s11percondUctor
Stretching a wire
Consider a wire of length f, radius of cross-sectiqn rand is of resistance R. It is stretched to length f.'. Let its
resistance becomes R '. Assuming volume of material remains constant after stretching or compression, and so

f..' r2
or
e
pC
We have, R and
7tr2

.e· r2
R' R-x-
R. r'2

f. I .e I

R-x-
f e
or R'

,2 · ,2 '
Also R' R-x-·-
r ,2 ,. ,2

or , R' ~(:.r
% change in resistance, if there is small cha.nge in _lengt~
, AR M
-xlOO = 2-xlOO'
R f

Resistors in series and parallel


Series : The resistors are said to be in series, if they provide a single path between the points. The current is
same in each resistor in series, but p.d. across resistor is proportional to its resistance. Let us consider two
resistances R I and R2 connected in series. For series connection · ·

V
Since i is same in both the resistors, (a)
. R
iR ~
-----v-----
. --~--- _. Equivalcnl resislor
or ;_ R___ =_R 1+R2_J (b)
The above equation is true for any number of resistors in series. Thus for n-resistors. Fig. 3.17
R = R 1+R 2+ ......+R"
Parallel : The resistors are said to be in parallel between the points, if each provides an alternative path
between the points. The potential d_ifference is the same across each resistor, but current divide in inverse rariO
of their resistances. For parallel connection of resistors ;1 R1
i1 + iz

V
R

or
r---,--- _
iR -
I -I--·--.
-R-1 +-R-2 !
~------ -----~V
Fig. 3.1$
., l -~----~·-···-"~---~·-. ..J ~
R
For n-resistors in parallel i-----v-------i
Equivalent resistors
Effective value of a Fig. 3.18

(i) In series : Suppose a 1 and a 2 · are the temperature coefficient of resista.nce of the resistors R I and
R2 respectively. Let a be their effective value.
- _;-. R, R, R
a, '½·
~
ct
At0°C Ro1 Roz Ro= Roi+ Roz
Att°C R01 (1 +a 11) Roz{l + azt) Ro(I +at)
Their equivalent resistance at any temperature in series is
R1 R 11 + Rz 1
or R0 (1+at) R01 (l+a 1t)+R0z(l+azt)
or (Roi + Roz>< 1+at) (Roi+ Roz)+ ( Roi a, + Roz Clz ) t
or (Roi +Roz)+ (Roi + Roz) at (Roi +Roz)+ (Roi a, + Roz az) 1
r---· ----·~ , " · ~ ~ - - -------.

After solving, we get !


• ~~ . __ .( R.0.1 a. i. + Roz Clz )· ;1.
1 . . "- + ,,_ .
, _:, .• ~·-- •• ~, ,: ~-~1 "'~2 ,:,_.
(ii) In parallel :

At any temperature 1°C Fig. 3.20


1 1
-+-
R, R11 Rz,

1 1 1
or +
R0 (1 + at) Ro 1(1+a 11) R0z(l+azt)

1. 1 1
or +
Ro1(l+a1t) 1/oz(l+Clzt)
( Roi Roz }1+a1)
Roi +Roz

Roi +Roz 1 1
or +
R01 Roz(l+at) Ro1{l+a11) Roz{l+Clzt)
1
(Roi +Roz){I+atf - 1 (l+a 1t r1 +1- (l+aztf'
Roi Roz Ror Roz
Solving by Binomial theorem, we have
1 1 1 1
-(1-at)+-(1-at) -{I-a11)+-{l-azt)
Roi Roz Roi Roz

or a{-l +-') (-
a1+az- )t
Roi Roz Roi Roz .

or (l
=lcxiS62'"a2Ro1
. 'tloi .+ Ro2 .
J7 J ' -
DIRECT CURRENT AND ELECTRIC CIRCUf[S

Ex. 1 The region between two concentric spheres ofradii r1 and , 2 is filled with a conducting material
of conductivity er . Find resistance between the spheres.
Sol. Choose an element of thickness drat a distance r from the centre. Its resistance

dR dr (') dr
= P 41tr 2 = Cf 41tr2

The resistance between the spheres

,,, ,,, .
R f dR=-41tcr1-f ~
,. 2
n 11

Fig. 3.21

Ans.

Ex. 2 Find resistance of the annular disc between centre and periphery of the disc. The inner and outer
radius of the disc is a and b respectively. The thickness of the disc is I and conductivity of its
material is s .
. Sol. The resistance of a cross-section perpendicular to the current flow on any radius rand thus of area
21trl is given by

dr I dr
dR p-=--
2m·t er 21trt
Total resistance i ;
b I dr
R Jer 21trl
a

or R --en
I ( -b) Ans. Fig. 3.22
21tcrt a

3.3 ELECTRICITY FROM CHEMICALS: CELL


lfwe want to flow the charge through a resistor, we must set-up a potential difference across it. Let us connect
each end of the resistor to a conducting sphere, with one sphere ~barged positively and other negatively as in
fig. 3.23.
. R
~
Fig. 3.23
The problem with such a device is that charge flows till their potential become equal. When that happens, the
flow of charge stops.Therefore, to maintain a steady flow of charge we need a device which can maintain
potential difference between the terminals of the resistor. Such a device is called emf device. It may be a battery,
electric generator or solar cells etc.
~----=-----,-----------~-------·----,---,
E:I__.
ELECTRi/El1:Y& MAGNETISM\

f (S}]f The simplest unit for ll}aking· electripity ·is called a.


/ cell. It makes electricity for chemical reactions and
works like a pump to push electrons along wires. A
battery has two or more cells and some types, such as car
batteries, are rows or "batteries" of single cells, hence our
common name "battery" for single and multiple cells. In a primary F.lcctric animal
Muscles produce tiny
cell, as electricity is produced, the chemicals arc slowly used up. electrical signals as they
Eventually, the chemicals run out and the battery cannot make work. In the electric eel,
electricity any more. In a secondary cell, the chemicals can be these muscles form large
blocks along the body.
replenished orreformed by recharging the cell with electricity. They produce powerful
:mrgi.:s of electricity,
hundreds of volts, like a
"living battery".

Positfre
/(•rmi11(1/
'
Acid-proofcasing

Car batteries
Also called an accwuulator, a
vehicle batteries can be
recharged. The chemical
reaction which has taken place
to make electricity can be
reversed by putting elccuicity
back in, so the battery can be
used again. In a vehicle the
recharging is carried out by an
alternator, which is driven by the
Plalc.1 sit in buth engine. Most car batteries have six
ofs11v11g .rnlplmric acid linked cells, each with an output of
about two volts. Each cell consists of lead
Lead dioxide pla1e
plates, lead dioxide plates and sulphuric acid,
Lt1ad plate Electricity is produced in the reactions between
. the plates and the_ sulphuric ~cid ..
. . ·-. - ' . .
Substances such as acids dissolve in How a cell works
water to fonn charged particles, Positfre Negutin• Eleamns
flow
ions - positive cations (red) and

u
negative anions (blue). In a cell,
LhL--se fum1 the clcclrolyte. When
other matcrh1ls, such as metal roads,
are put in the electrolyte. they act as
electrodes. They attract opposite-
.charged_ ions and cause an electric The ,·k,·1nll}'lC COll~N~ Tll,i dC<.'.lroJc,, ;ITC !he Orpmite cl,:,;-1rkal chafl,'L'"!.
uf dr.1f¥.: p.mido:!> a!Icl ' pui;ili~.: illlo,,k .uid Lhc
current to; flow. ioM, l'O'ili,·c and i;cg~th·c. ru:gati\\" athodc.
altr.i~I and e!,-,:trurni move,
ln.llin!?- the cmrcn.r.
··--- -- -- DIRECT c_~~-~-~~CTRl~ c~~~~~~~ --~
3.4 ELECTROMOTIVE. FORC_E (EMF)
Emf of a source is defined as the workdone per unit charge that the source does in moving charges from its lower
potential terminal to its high potential terminal. i.e.,

dW
dq .

The tenn electromotive force is somewhat confusing in that concept to which it refers as not a force, but a work
per unit charge.

(-)
Source of emf
Fig. 3.24
For a source in open circuit, the potential difference V, is equal to the electromotive force. Thus
V = ~
Under closed circuit condition, the source offers some resistance to the flow of charge. This resistance is called
internal resistance. Because of this internal resistance the potential across the terminals of source is decreased
by i r.
p.d across ,the terminals of the cell
V ~-ir

,. ____
,.______;.r

Fig. 3.25
Now consider the case in which source is being charged by· an· other source like alternator. When this is the
case, the current within the source is from positive terminal to negative term{nal. The terminal voltage is then
greater than the emf ~ and so p.d. across the terminals of the source
V = ~:t-!r:
+ Charger -

Fig. 3.26"
What happens inside a source of emf ?
I. '
Consider a source of emf which maintains a potential difference across its terminals. Let us consider the
charges inside the source. In an open circuit, the charges are in equilibrium, therefore the net force on
each charge must be zero. i.e., ·

F O=F,+Fn
where Fe is the force exerted by field produced by the terminals of the source. This field is electrostatic

in nature. The other force ( F" ) which is equal in ~agriitude to Fe is the force ei<erted by field,js of
nonelectrostatic in nature, whatever its origin is. Thus we can write
Ee+En 0
@+-<t)---+0
or Ee -En 1~~1 Fe·~
Source of emf
Fig. 3.27
2. The electrostatic potential difference Vis defined as the work done per unit charge done by the
electrostatic
. Ee on a. charge moving from positive terminal to negative terminal of the source. And
field

the work done by E,,, perunit charge, when a charge moves from negative terminal to positive terminal
of the source, is called electromotive_ fo_r_c_:5-~fthe source.
When Ee=-En,wehave V = I;.

3. Now suppose that the terminals of a source are connected by a conductin_g wire forming a complete
circuit. The driving force on the free charges in the wire is due to the electrostatic field Ee field set up by
the charged terminals of the source.

fa· <F
"

Fig. 3.28
This set up a current in the wire from positive terminal to negative terminal of the source. The charges on·
the terminals decreases slightly and hence, the electrostatic fields both within the wire and source
decrease also. As a result Ee becomes smaller than En. Hence positiVe charges within the source are
driven towards the p_ositive tel_'ffiinals, and there is a current within the source from negative to positive
terminal of the source. As Ee is some what less than En in closed circuit condition, correspondingly V
is less than ~- The difference is equal to the work per unit charge done by the resultant field, which is
simply ir.

Short circuit : If the terminals of a source are connected by a conducto_r of zero resistance (practically
negligible resistance), the source is said to ~e short circuited.

R=O V=O

I~. I'

Fig. 3.29
"
The short circuit current can be ootained as
.v ·= 0'=s~·ir

or i
r
op·en circuit : When no current is drawn from the source, it is said to be an open circuit. Thus
i = 0
V = s-ir=s-Oxr
or
At a glance
I. For an ideal source of emf(cell) whose internal resistance (r) is zero, VAB =s
1Ji,~
Fig. 3.30
2 A practical cell which has some internal resistance (r) .

1--+-i
• ."'""'"+I---<i'/ or .--j ~
~.r I; -
Fig. 3.31
VAe=s
3. When cell is discharging :
i r
••...,,_,__,,,,.,."fl'.
A -:
Fig. 3.32
4. When cell is charging :

~B
A
Fig. 3.33
5. ,Short circuit: VAB = 0

Fig. 3.34
', 1
or 0 s-_ir => i=~.
r
6. . Open circuit: 0
I s-ir=~-O=s
I .:-1· B

' '
Difference between a· ci\ll and a capacitor
I. Capacitor can provide vari,able ( decreasing) potential in a circuit for short time. While cell can provide
constant potential for a'long'time (many years).
2. The energy stored between plates of capacitor is electrical while energy stored between terminals of cell
is chemical.
3. The plates of capacitors are usually made of same material. The terminals of cell always be of different
materials.
4. The electric field between the plates of capacitor is non zero. But electric field between the terminals of
cell is zero in open circuit.

Fig. 3.36
3.5 CIRCUIT ANALYSIS
Ohm's law can give the solution of the circuit which has one loop or circuit with many loops which can be
reduced to a single loop. To analyse complicated circuits Kirchhoff developed a method which is based on two
laws.
Kirchhoff's first law or junction rule
According to it, the algebraic sun{'of currents at any junction is equal to zero. i.e., Li= 0.
,! Ir, '
or
The sum of currents entering afany ]Unction is equal to the sum of currents leaving the junction.

Fig. 3.37
Kirchhoffs first law is based on conservation of charge; the magnitudes of the currents in the branches must
add to give the magnitude of the current in the original conductor, so that i 1 = i 2 + i .
3
Kirchhoff's second law or loop rule
According to it the algebraic sum of potential drop across all circuit elements in a closed loop is equal to zero.
i.e., L V=O.
Second law is based on conservation of energy.
(a) For a circuit having cells an~ res~.t~r~_, ~e c~n wirte
,, 'LiR+~ = 0.
(b) For a circuit having cells;capacitors and resistors, we have

Guidelines and sign conventions


The guidelines and sign conventions, we have used in previous chapter can also be used here also.
I. Assume the currents in each branch of the circuit. At each junction, they must satisfy junction rule. The
current in any resistor is due to the net resp~11:se o( all the sources present in the circuit.

'
c=------
2 You may choose any close loop, which may have cell or not. But for a circuit having only one cell,
choose a close loop, which include this cell. •~
I •I'
3. (a) The potential drop across any resistor in the direction of flow of current can be taken as negative
and positive for reverse direction of current. -.

-iR
~
+iR
Fig. 3.38
(b) The potential drop across the capacitor is taken as negative from its positive plate to its negative
plate ;nd positive for reverse sense.

-q!C

41-
+q/C
Fig. 3.39

See the sign of charge of first coming plate of the capacitor. In the above case, the first coming
plate when moves from left to right across the capacitor is positive which has charge (q).
(c) The emf of the cell is taken as negative from positive terminal to negative terminal of the cell and
positive for reverse sence.

Fig. 3.40

In the final calculations, some of th~ currents/all currents may be of negative sign. Do'nt be
bother about their sign. The negative sign indicate that the actual direction of current will be opposite of
assumed direction of current. In further calculations you have to use the value of current with the
negative sign.
Grouping of cells : Battery
I. Series grouping :
(a) Right grouping of cells : When each cell in series delivers the current in same direction.

The equivalent emf


--j~-
~eq ~1-r1 ,;:?. r-:.
The equivalent resistance req r1 + r 2 Fig. 3.41
(b) Wrong grouping of cells : When cells deliver current in opposite directions.

-:l ~ f-
::;, 1, "1 ';i:?• ,.2

Fig. 3.42
The equivalent emf
n' ,I
se 9

The equivalent resistance req rl + r2


In a series of 5 identical cells, if one of the cells is wrongly connected;
The net emf of rightly connected
,. ' cells = 4s Wronglv conncct.:d cell
.. -./ .
The net emfofwrongly connected cell= --{,
-l -f -J i+.H ~ .
Theequivalentemfofallcellsinseries = 4s-s ~.r <;,r ,;,r. ~.r ;.r
or ,1 Seq · = 3S Fig. 3.43
The e9uivalent internal resistance req = 5r ,
Thus when one cell is wrongly connected, the net or equivalent emf is decreased by 2s. But internal
resistance is always be additive.
For a battery ofn-identical cells, ifm cells are wrongly connected then effective emf
Seq (n-2m) s
.
req nr · Jhcell
(c) For a battery ofn-identical cells:
~
i-J •······
~,r r
.. ··1<;,r
<.:i,
Net or equivalent emf Seq ns
and equivalent resistance Req nr+.R
I

I ., R
:. Net current Fig. 3.44

___5_
nr+R

0) If R << n r, then i = 5.. This is the current in the resistor due to only one cell. Therefore series
r
grouping of cells in such a case when external resistance is very small in comparison to internal
resistance is of no use.

(ii)If R>> nr, imax = nl. This time the current in resistor R is n times the current due to one cell.
R . . .
,, Therefore series grouping of cells is useful in this case.
2. Parallel grouping
(a) For a battery of n-identical cells, all are rightly connected
Equivalent emf Seq s
r
and equivalent resistar:ice Req -+R
n

J 1cell

_s_ R
Fig.3.45
!.+R
n
1

(i) If R >> ~, i =; . This is the current in the resistor due to onl~-~?; cell. Therefore such grouping
of cells is of no use.

(ii) If R << !:., imax =


n
n5..r That time the current in resistor R is n times the current due to one cell. In
such a case parallel grouping of cells is useful.
(b) Battery having cells of different emf
(i) Two cells are rightly connected : Both the cells delivering currents in same sence.
Method!: ''
In closed loopACDBHGA
-~1 +i1 r1 +iR 0
D
~I
--l-
.R
or B
'1 '1 ;, I;,, r,
F
Similarly --,-
~2
r2
.R
R
'2 G H

and Fig. 3.46

or i

or
Method II : Remove the resisfor'l? and connect the junctions A and B.
: ') -
~1,r1
D
13 i
A•
~z- r2 =>
F -
R
G If

Fig. 3.47 ·
In closed loop A CDBHGA
. .(\
-~1 +i1r1 0

or

similarly

- . ~I ~2
=. 11 +12 =-+-
'i r2

Now

or ~ =

Method III:
,, (i) In closed loopACDBFEA

-~1 +iri +i r2 +~2 0

or

Now

or
I DIRECT CURRENT AND,'E1£.CTRICCIRCUITS

(ii) If cells are wrongly connected: The equivalent emf of them caq,)?~ easily obtained by any of the
methods discussed above. Or otherwise putting -<;2 is place of I:,.· so we.)iave

'-, a '';:~" \ ~ ~ ·

req
--1:L
lj +r2
~
Fig. 3.49

3. Mixed Grouping:
Let there are n-identical cells in a row and m such rows are in parallel. The total number of cells in the
circuit is mn.
;.r ~.r 11~. nr
11:i!. !!!_

R
R
Fig. 3.50
0
Equivalent emf s. 9
ns
nr
and equivalent resistance -+R
m

:. Net current

~
nr
-+R
m

mns
nr+mR

nms
(,r,;;: - ..r,;;ii)2 + 2,r,;;: -JmR
The current i will.be maximum, when denominator in above expression is minimum. It to be minimum
when

nr
or = R
m
or total internal resistance = total external resistance

then
mn S s r,;;;;
imax 2 ,r,;;: ..r,;;ii = 2. Vrll ·
Ex. 3 Find the equivalent emf of the three cells which are connected as shown in jig. 3.23.

Fig. 3.51
Sol. (a) s s
The equivalent emfofcells 1 and 2 is

s1r2 +s2ri

Fig. 3.52

and

(b) s s
The equivalent emf of_cells 1 and 2 is

s = s1r2 -s2r1
(s2ri -~1r2)
'1 +r2

r 1r.,
/;3r3
...____J
i;, r =r
1-------1 L____:. I
+;.2
Al ~
and Fig. 3.53

Ex. 4 Find equivalent emf of four different cells are connected in parallel. Their emfs and internal
resistances are shown injig. 3.54.

A B

Fig. 3.54
~ - - _ _ _ _ _ _ _ _ _ _ _D_IRE_cr
__C_u_RRENT
_ _AN_v_ELECTRIC CIRCU_ITS
___ ~
Sol. The cells can be connected in the following manner

.4

Fig. 3.55

~1
lxl+Zxl =iv n =.!..n
l+I 2 'I 2

4xl-3xl I I
~2 -V,r2 =-Q
I+ I 2 2

2 +~2 1i
~, 1
Now ~ 11 + 12

3 1 I I
-x-+-x-
2 2 2 2
I I
-+-
2 2
1V A11s.

Ex.5 Two sources of current of equal emf are connected in series and have different internal resistances
r 1 and r2 (r2 > r1). Find the external resistance Rat which the potential difference across the
terminals of one of the sources (which one in particular) become equal to zero.

Sol. The current in the circuit is

Since i is same in both the cells


and
L:~~
Fig. 3.56
Since both the cells are giving the currents therefore V1 and V2 can not be negative.
Since r 2 > r 1, therefore firstly V2 can be zero

0 ~ - ( ri+r2+R
2~ ),
2

or R = (ri-,i)- A11s.

The method of principle _of superposition


Complicated circuit can be solved by using the principle of superposition. According to this principle, when a
number of cells are present in a circuit, the current in any resistor is same as the superposition of the currents
due to all the cells, acting one at a time, the others being absent.
Ex.6 Analyse the following electrical circuit.
2.Q

Fig. 3.57
Sol. Method I :
Assuming first only I 0V is present in the circuit. and then only 20V is present in the circuit. Find
current in each case. Finally superpose the currents due to both the cells. It can be understand from
· the fif!. 3.58.

r~,~2:~
IO
-A ?.Q 2.0
20
!_)_ A -A
3
2,11
IOFL _".. rn 3 3 2i
2.Q

(a)
(bJ

2.Q 20V
,---ww,,.,-~5t-ll-;l 20 _.5_ )A= 5A .
-+·~- A=SA 3 3
3 3
10 2.0 2.Q

(C)
Fig. 3.58
Method II:
By using Kirchhoff's method.

2.Q

(i,+i,)
20 2.Q

i,,-----+E~--~D

Fig. 3.59
Assuming the currents in resistors as shown in.fig. 3.59. In closed loop ABEFA, we have
-2 i1-2(i 1+i2)+10 o
m 2/ 1 +½ 5
In closed loop BCDEB, we have
-20+2/2+2(/1+/2) 0
·or . 2/ 1+ 2i2 IO ... (ii)
Solving equations (i) and (ii), we get
i 1 = 0, and i2 = SA
;
_ _ _ _ '?IHE_Cl' CVHHENT AND ELECTHl~ ClHCCJ_l!~ _ - ~
Finding the potential difference ,,
Ex. 7 Consider the following circuit and find potential difference VA- V8 •

IOV. IQ

IQ

p
4Q 13 2Q

Fig. 3.60
Sol. The currents in different resistors can be obtained as follows: The net current drawn from the
source

. IO
,- 2A.
-[1+1+ 6x6]
6+6
This current will divide equally in branches OAP and OBP because they have equal resistances.

IOI'. IQ
2,1
L4 A
IQ 2n 40
0
-·-r
-:- '..m p
IA 4Q 2Q
B
Fig. 3.61
Method!: VrVB = (Vo-VB)-(Va-VA)
= 4xl-2Xl=2V A11s.
Method II: Connect a hypothetical cell between A and B, taking A as positive terminal. Now apply
loop rule in close loop OABO, to get VAB = VA-V8 . Thus
-2 xl-VAB. +4x I 0
or 2V Ans.

Ex. 8 See the circuit shown in fig. 3.62. Find the current through the switch S when it is closed.

!•~-/'-JM}MJ~.•,-,~~0~,~.~JM1Ma--,JQV
i-
2n .
..1.: s
-=F V=O

Fig. 3.62
Sol. Let i I and i 2 be the currents in the branches A O and B O respectively. By junction rule the current
in switch Sis · ·
i1+ i2
V0 -0 20-V
0 0 I0-V
or --~+-~~
2 2 4
or IOV
Vo-0
Current,
2

10-0 =SA
2 . Ans.

Ex. 9 Find out the value of current through 2 Q resistance for the given circuit. Also find the potential
difference between A and B.

A C
G~--~H

IOV 50. 20V

5V 20 ,
F~,---~E-ll-'\/\1\/\r~D..,___ ___,B

Fig. 3.63
Sol. Let i I and i2 be the currents in 5 Q and IO Q resistors respectively.

A' ,..•, •.•1,..1_ _.......,C G,----,n


i . ii

·l. 1011··. 50. JOQ


20/1

: · sv· 20
i, • i E; 1-'V\J'V\,-D-!,--t,~.,--'i B
: 1 3 - :
.4•·........................................... J,l····················'B'
BA

Fig. 3.64
In close loop ACEFA,
-5i 1+10 0
or i1 2A
In close loop GHBDG
-20+ 10i2 0
or i2 2A
Let i3 is the current in 2 Q resistor. At junction E,
;, = }1.+i3
which gives I >.

( 2 Q resistor is in open circuit, therefore current in it will be zero)


To get the potential difference V8A = V8 - VA, connect a hypothetical cell between A and B keeping
Bas positive terminal.
Now in close loop A A' B' B DEF A, we have
+V8 A+5+5i 1 0
or VBA -5-5i1
-5-5x2
-15 V. Ans.

Ex. 10 Find the reading of the ammeter A as shown in.fig. 3.65.

3.0

12V 30

30

30

Fig. 3.65
Sol. The above circuit can be reduced as follows.

12V 3.Q

i=3A
30
IA :=:,
30
IA
30 10
IA

Fig. 3.66
The reading of ammeter A is i, where

. i = ~=3A
3+1
The current distribution in all the resistors is also shown in the fig. 3.66.
More about short
When two points of a circuit are connected together by a resistanceless wire, they are said to be short-circuited.
Since 'short' has theoretically zero resistance, so it gives rise to two important facts :
(i) no voltage can exist across· it because V = ir = i XO = 0 ,
(ii) current through it is very large, thet>retically infinity.
Short in series circuit
Consider a circuit with three resistors
,
in series' with a source of emf as shown in jig
I
3.67 (a)

short
IQ 2Q 3Q
A B C I)

c:7 121 ·
(a)
IQ

~
2Q

12V
(b)
3Q

Fig. 3.67
. 12 '
The equivalent resistance of the circuitR =I+ 2 + 3 = 6Q and resultant currenti = = 2A which is same in
6
all the resistors.
Now 3Q resistor has been shorted out by a wire CD as shown in fig. 3.67 (b) so that Rco = 0.

12
The equivalent resistance of circuit R '= I + 2 = 3Q and current in the resistors i' = = 4A in IQ and 2Q
3
resistors, but no current in 30: resistor.
Short in parallel circuit
Consider a circuit as shown in fig. 3.68 (a).

...I... IQ 2Q 12vi L 2, 3Q

B
(a)
(b) Sh rt
Fig. 3.68
The equivalent resistance of circuit
1 I I
-+-+-
R I 2 3

it which gives R ~Q
11

currents in resistors are iI = . _il, = 12A


I

i2 _il,= 6A
2

i3 _ll_ = 4A
3
Now suppose a short is placed across 3 Q resistor as shwon in fig. 3.68 (b). The entire current will pass through
the short.

But short draws infinite current.


~ DIRECT CVRRENFANDELECTRIC ¢IBcVITS li,:il
L ·--~----'-,----'----------"---E=.I
Ex. 11 In the circuit shown infig.-3.69 E, F, G and Hare cells of emf 2, 1, 3 and'l'Vrespectiv~ly. The
resistances 2,-1, 3 and IQ· are their respective internal resistances. Calculate (a) the potential
difference between B and D and (b) the potential differences across the terminals of each of the
cells G.and H.

r
21' 2Q B
. E (i - i,)
F
IV 11 ·
LV
fl H
Q
IQ IQ i, IQ
IQ

c· G Ci - i,)
Di,i;:..----1 1---"MM"r-+:...JC
rl<'-----1 1------J,,..
31' 3Q 31· 30
(a) (b)
Fig. 3.69
Sol. The proposed currents are shown infig.. 3.69 (b).
In close loop ABDA, we have
-2+2i+2i 1 + Ii+ I = 0
or 3i+2i 1 = I ... (i)
In close loop BCDB, we have
I+ l(i-i 1)+ 3 (i-i 1)-3 -2 i 1 = 0
or 2i-3i 1 = I ...(ii)
After solving equations (i) and (ii), we get
I
; ~-A,
1 13

and
2_A
·13
(a) The potential difference V0- V8 = 2i 1

= 2(- /3") = - I~ V
or 2v
13

(b) The current in the cell is H 1


Since cell G is discharging and His charging
6'
~-ir=3--X3
. 13

~v Ans.
13
. 6
. and ~+1r=l+-xl
. 13

~v. Ans.
13
Ex. 12 A part of a circuit in steady state along with the currents flowing in the branches, the values of
resistances etc. is shown in fig. 3. 70, Calculate the energy stored in the capacitor.

3Q

41' 2A IQ IA
-~ I 5Q

4µ IQ

IQ 24 2Q 4.Q
·--;
3V
Q s
3Q

IA
Fig. 3.70
Sol. After charging fully there is no current in arm of capacitor. By using-junction rule the currents in the
resistors are shown in.fig. 3.7/.

3Q

4V 2A IQ L4 3A
R
~. I' 5Q

4 l.Q

24 IA 2.Q 2A 4Q
--;
Q s
3V
312

IA
Fig. 3.71
I
The p.d. across the capacitor is
Vp-Vg (Vp- VR) + (VR- Vs)+ (Vs- Vg)
5x3+lx3+2xl
20 V
The energy stored in the capacitor is

u = _I_cv 2
2

_I_x4x!0-6 x202
2
800µ1. Ans.
DIRECT CURRENT AND ELECTRIC CIRCUITS
'-------------------- " ,, ,,,,

Ex. 13 Twelve cells each having tlie same emf are connected in series and are kept in a closed box. Some
of the cells are wrongly connected. This battery is connected in series with an ammeter and two
cells identical with each others. The current is 3A when the cells and battery aid each other and
2A when the cells and battery oppose each other. How many cells are wrongly connected?
Sol. Let m cells are wrongly connected in the battery and~ is the emf of each cell, then
~net = (12-2m)~
• When two cells aid the battery, then current
(12-2m)~+2~
3 = ... (i)
R
where R is the total resistance of the circuit.
When two cells opposes the battery, then
= (12-2mg-2~
2 ... (ii)
R
Solving above equations, we get
m = I
Hence one cell is wrongly connected in the battery.

Ex. 14 Find the charges on the four capacitors of capacitances 1 µF, 2µF, 3µF and 4µF shown in jig.
3.72.

(b)

Fig. 3.72
Sol. The current will flow through the resistors only. Let i I and ;2 be the currents in branches AB and CD
respectively, then
6
iI - -=2A
1+2
6
and i2 -=IA
3+3
(i) p.d. across l µF capacitor VAE = lx2 =2V
charge on the capacitor = 1x VAE=2µC
(ii) p.d. across 2µF capacitor VEB = 2x2 =4V
charge on the capacitor = 2x VE8 =8µC
(iii) p.d. across 3µF capacitor VCF =3x1=3V
charge on the capacitor = 3x VcF=9µC
(iv) p.d. across 4µF capacitor VFD = 4x 1=4V
charge on the capacitor = 4x VFD=16µC. Ans.
ELECTRICJTY,&,M)\GNETJ._S_M-.--i-:--,~--···------·- ·;·· --~~l

Ex. 15 A long round conductor of cross-sectional area Sis made of material whose resistivity depends

a
only on a distance rfrom the axis of the conductor P=,. where ais a constant. Find:
. r-

(a) the resistance per unit length of such a conductor


(b) the electric field strength in the conductor due to which a current i flows through it.
Sol. (a) Consider a cylindrical element of radii between rand (r + dr). Its resistance

= __pi_

a
dR ...........................
21trdr
························o··
__ .
I 2nrdr
or -
dR pe ... (i) .. . ••••••·········•·•······
·•·•···········•········ ..... . '

Since all such resistance are in parallel (

Fig. 3.73
R

(where a is the radius of conductor)

2
J 2nrdr =21tfr3dr= 2n(a )= (1ta J2
4

o(~)c ae 0 ae 4 21tae

s2
21tae

21tae
Thus R s2 . ... (ii)

2na
The resistance per unit length of wire R =~

2
(b) Equation (ii) can be written as R = ( 7ta·)(!..). Compare with R = pf , we get p = Z1ta .
. s s s s
, i 21ta 2nai
By Ohms law E=jp=-X--=--.
s s 52
Wheatstone bridge
It is a special type of resistance network, commonly used for comparing resistances or unknown resistance can
be measured in terms of three known resistances. It consists of four resistances connected in the form of a
bridge.
Analysis
Let four resistors P, Q, Rand Sare connected as in fig. 3. 74.
The proposed currents.are as shown infig. 3.74.
At junction A i = i 1+ i2 •.• (i)
In close loop AB DA;
. -Pi 1-Gig+Ri2 =0 ...(ii)
D
and m close loop B CD B;
-Q (i 1-ig) +S(i2 + ig) +G ig=O ...(iii) t----~li
In close loop ABC E FA; F f'
-Pi 1 -Q(i 1-ig) + V= 0 ... (iv) Fig. 3.74
After solving a~ove equations we can get i 1, ;2 and ig. The equivalent resistance between A and C is

On substituting the values of i 1 and i2, we can get RAc· , '

Balanced Wheatstone bridge·.


For balanced bridge; V8 = V0, and so no current flows through the galvanometer Ug = 0), the equations (ii) and
(iii) reduce to
-P i 1+R ;2 =O
..
.!l_= R
or ... (v)
. -i2 p -
and +Si~=O
'....Qi 1
.i1, s
or ·-=- ... (vi)
i2 Q
Now from equations (v) and (vi), we have

R s
p Q
p
or
Q
The balanced bridge now effectively reduces as·:

R
., ~-
l)

•'' J
F~-----< t - - - - - ~ E
V
Fig. 3.75
The equivalent resistance between A and C
I 1
--~+---
(P+Q) (R+S)
R
The relation can be written as
s
;P Q
; R S
That is if positions of the galvanometer and'i,attery are interchanged, then there will still no deflection in the
galvanometer.

Fig. 3.76

Ex. 16 Find the equivalent resistance between the points A and B of the circuit shown in jig. 3. 77.

i c-: :J
10n
~DC'r--'lill

sn ·
(a)

Fig. 3.77
Sol. Methodl:
The given circuit is not balanced bridge therefore we have to find the equivalent resistance either
by circuit analysis by Kirchoffs law or by other method.
Suppose current i enters through the junction A into the circuit. Let i I goes to the 5 n resistor and
the rest i - i 1 goes to 10 Q resistor. By symmetry, the current i, coming out from the junction B will
be composed of a part i I from 5 n resistor and i-i 1, from 10 n resistor. By junction rule the
current in middle 5 n resistor is (2i 1-i) as shown in jig. 3.77(b).
We have VA-VB (VA-Vc)+(Vc-VB)
5i 1 +10(i-i 1)=10i-5i 1 ...(i)
Also (VA-Vc)+(Vc-VD) +(VD-VB)
5i 1 +5(2i 1-i)+5i 1
20i1-5i ...(ii)
From equations (i) and (ii). we get /,·_ t

Substituting this value in equation (i), we get

3;
IOi- Sx-
5
7;

or 7

or 7.
MethodU:

4
-
sn. c- 10n (i, - ;,J
11 i3
i:? : sn
,, 100 50
.. B
.D (i, + i3)
- I
F
I , E

Fig. 3.78
' Supposing a source ·is connected between the terminals A and B. The current distribution is shown
infig. 3.78. ,, ·' · ·
At junction A, ' i i1 + i2
V V
Resistance between A arid B, RAB

In close loop AC DA,


-5i 1 -5i3 +10i2 0 ... (i)
or - i 1- i 3 + 2 ;2 0
In close loop CB D C, ,I
- IO(i 1 -i3 )+ 5 (i2 +i3 )+5 i3 O
or -2i 1+i2 +4i3 0 ...(ii)
Now in close loop AC B EF A
-5i 1-lO(i 1-i3)+V = 0 "'
·.. V '"' ._
or ... (iii)
5
'
From equations (i) and (iii), we get .
V
...(iv)
5
fjl;J · :ll,iit~craic11Y Bi~~~~f11s~i:- "
From equations (ii) and (iii), we get
2V
4 i1 + i2 ... (v)
5
Solving equations (iv) and (v), we get
9V
iI
105
6V
i2
105
V
Also i3
,,· 35
V V
RAB
i1 +i2 ( 9V
105+ 105
6V)
7 n.

. . 6V . . . 9V .
'11 -13 = - = 12 and 12 +13 =-= '1
105 105
Thus the current in 5 Q resistor connected symmetrically is same and currents in l 0 Q resistors also
remains same.
Delta-star transformation
In solving networks, having many loops by the applications of Kirchhoff's method, one sometimes experiences
great difficulty due to large number of simultaneous equations that have to be solved. However, such complicated
networks can be simplified by successively replacing delta meshes by equivalent star system and vice versa.
Suppose we have three resistances R 1, R2 and R3 connected in the form of delta as shown in fig. 3. 79
(a). These three resistances can be replaced by three resistances R 12, R23 and R13 in the form of delta between
the same terminals as shown in fig. 3. 79 (b).
B

-
(a) Delta com1ection. (b) Star connection.
Fig. 3.79
R1 R2
where R12
R1 +R2 +R3
R1 R3
R13 R1+R2 +R3

R2 R3
and R23
R1+R2 +R3
i-.c-__-,----~-~~--·-··---------~~-----~~- ·:~ ~~~---~-.!!.-~_R_::_NT_AN_D_ELEeTRIC__c._1R_c_u_n:_s__ ~

! 1@@ In any network there may be many deltas. Students are advised to transform the delta into star
which has symmetry. For example in a network of three deltas, transform middle delta into star.

Fig. 3.80

Ex. 1 7 Take the previous problem and find its equivalent resistance between the terminals A and B by
using delta-star transformation method.

sn 10n

It sn B

10n sn
D
Fig. 3.81
Sol. We can simplify the circuit by transforming delta A CD into star as follows.
C
./,Q, // 10n
i, ,J
-''<"
A R, \
11..= sn
·'
,p ,:-,
""-'o-? ~
SQ

D D
Fig. 3.82
45
0
4
~n
2
=> A

1s n
2
Fig. 3.83
R1R2 5x!0 ~n R1R3 5x5
~n
R1+R2+R3 5+10+5 2
R13 = 5+10+5 4
R1 +R2 +R3
R2R3 !0x5 =~Q 5 9
R1 +R2 +R3 5+10+5 2 RAB= -+-=7Q
2 2
Ex. 18 Find the equivalent resistance of the circuit shown in jig. 3.84 between the points A and B. Each
resistor has a resistance 1 Q .

: )

Fig. 3.84

Sol. Method I:

Fig. 3.85
Because of symmetry about dotted line PQ, the currents in left part of circuit are same as the right
part of the circuit. The current distribution in the resistors may be as in jig. 3.86 (b). It is very clear
from the figure that resistors having current ;3 can be removed from the junction Q without affecting
the currents in other resistors_as shown in.fig. 3.86 (b). The resulting circuit can be simplified as:

=> Fig. 3.86


A fl

8
-x2
_3_=!Q
~+2 7
3
DIRECT CURRENT AND ELECTf!IC~CtBCUlTS ffl
Method II: Since network of resistors is symmetrical about dotted line PQ, therefore it has equal resistances
about PQ. We can break it into two equal parts as:

l/2Q
'
+
jH
JQ i IQ
.4"---'V\.M.,-~ ~--'\I\N',-~B
R R
Fig. 3.87
The resistance of each half=R, which is equal to

!o
3
.'.!.n
3
p
'
:::;> : • ==> R=_±Q
7
1n IQ
A 'Q A 'Q

----
Fig. 3.88
R R
The equivalent resistance' between A and B
A Q B
Fig. 3.89

4 8
2x-=-Q
7 7
Method III: By delta-star transformation:

IQ

A A

Fig. 3.89

.'.!.n .'.!.n
3 3

IQ lQ
lQ lQ AIL--..VV\-----..vv1.----->B
. --- . ----- . --···--- ···~
. -ELECTRICITY & MAGNETISM ' , · '
- - - - - · - · - · ...~ - · · , • •• H• C _, ~---------~----· --------------~ ---------

Fig. 3.90 ·

Ex. 19 Find resistance between A and Bin the following networks of resistances. Each resistance is
equal tor.
Sol. Solution in brief are:
I. Neglect resistance in dolled
circle because
p R
-:::::.-
Q s
= Ans:
r
2

Fig. 3.91

A 4r
2. Ans:
5
Fig. 3.91

IO!l

3.

Fig. 3.93
4. RAB=? Each resistance is r
C

2r
Ans.-.
3
D
Fig. 3.94
'------"'------"-'--'---D_1_RE_cr_C_v""'"B_B_l:Nf"AND Eu,crBlc CtBCUITS __ M
2,-

!Or
5. r Aus.-.
r 9
r r
A B
Fig. 3.95
The circuit now can be simplified by delta-star transformation.
6. Balanced Wheatstone bridge :

(i) ·A (ii)
•,·». ~'--_::·~-!~·-·'Mlv-v_f..._,l /3
r

(iii)
-. Ans: r
')

Fig. 3.96
lncircle resistor can be neglected.

Hang from A
I'
,. r
HNV>-+r-...,.,_--+B => ,.
7. Ans: -
r 4

Fig. 3.97

Hang from A
B
,.
8.
,. A 4r
Ans·-
=> • 3
r
/3

Fig. 3.98
~- '.ELECTRICJ'lY"&M<\GNETISM

A .•l

Ve =Vo
VE= VF

3
9. => Ans. R=-r
5

£ig. 3.99
Ex. 20 Find the equivalent resistance of the given network ofresistors between the points A and B.
A C

r r r
r
D
r r
,

B F E
Fig. 3.100
Sol. Junctions D and Fare equipotential and so they can be connected together without affecting the
circuit. The simplified circuit is shown in jig. 3.10 I.
A A A

r r/2 r r/2 r
=> =>
r
r/2 r/3

B FID B
5
Fig. 3.101 Ans. RAB=
11 r
Ex. 21 There is an infinite wire grid with square cells as shown injig. 3.102 The resistance of each side
is r. Find the equivalent resistance of the whole grid between any two neighbouring points such as
AandB.

X
r r r r ..,..--,._ r
I r .\j r ' I X

A
,. B r A r B '
r r r r,
r
'
(a) (b)
Fig. 3.102
I'
'
L--- - - -- -------- - -

Sol. Letx is the resistance between A and 8. The circuit now can be reduced as shown infig. 3.103 (c)
and (d). The resistance between A and B is also r, so

( rx )
r+x
3(~)
r+x

("")' . m::::,
( IX )

r+x : ,. r
.... ..

,I B ;I /j

(c) (d)

Fig. 3.103

I I
., = 3(~)+-;:-
r+x

,.
After solving, we get x = - .
2

Ex. 22 A five pointed regular star has been soldered together form of a uniform wire. The resistance of
the section CL is r;
(a) Find the resistance of the section FL.
(b) What is the equivalent resistance across the terminals A and F?

Fig. 3.104
.~77.t:'!:tE<cm,tm,~i~GNF:IISM(\1tit'f'1.1_~_t,__ ~,I~-'. _-____.~ ,- ,,~;o
Sol. Suppose length of A L C = e
The length of the section FL 2ML
2LCsin 18°
2 esin 18°
0.62f
(a) Since resistance of wire is proportional to its length, therefore resistance of section FL, r' =
0.62r.
(b) The given network of resistors is symmetrical about AF, so it can be break into two identical
parts, each with a resistance R. HK has no current because VfFVK. Therefore its resistance
can be neglected. The resistance of left part R can be obtained as :
A A

r r
.-1

·~--.,,.,,,'--....:,;,!! r/
=· ' ~\,
0,48r \

Fig. 3.105
R r+0.48r+0.48 r= l.96r
fl
R 1.96 r
-=--
2 2
0.98 r Ans.

F
Fig. 3./06
Ex. 23 Twelve equal wires, each ofresistance rohm are connected so as to form a skeleton cube. Find the
equivalent resistance between the diagonally opposite points I and 7.
5
8
5 i/6
8 ii3 i/6
if3
ii6
4 i/3
4
i/3
7
7 6 i3 i/6
6
V3
1/6
ii6
2 3 2 - 3 '
(a) 10 9
V
Fig. 3.107
I<
Sol. Connect a source between points I and 7. ' "
The network is symmetrical about the diagonal 1-7. Therefore current.in resistors are distributed
symmetrically about !he diagonal. The current·distribution is shown in..fig. 3. I 07 (b ).
Choose a close loop l-2~3-7-9-10-1, we have
i i i
-r--r--r-+V=O
3 6 3
V 5
or -r
6
V .5
or -=-r Ans.
i 6

Ex. 24 Twelve equal wires each. ofresistance rare joined to form a skeleton cube. Find the equivalent
resistance between two corners on the same edge of the cube.
5
_,i---------,,, 8

4
,.

h-6---+----,J 7

2 3
Fig. 3.108
Sol. Method!:
Connect a source between points I and 2. Let current i enters through point I into the network. The_
network is symmetrical about dotted line.The currents above and below dotted line are symmetrically
· distributed as _shown in.fig. 3./09.
5 y-;
_,i--+-'---=----,,, 8

)' ; (v-z

I0,--,-----',"-+,---¼----'(4
y
X

z
;,.6-o---1----+----J 7
V (v-z)
y (v-z)

9 2 y 3
Fig. 3;j1/9
By junction rule at I, we have
x+2y

V V
... (i)
x+2y
In close loop 1-2-9-10-1, we have

V
.-rx+ V Oor x=- ...(ii)
r
In close loop 1-4-3-2-1
-ry-rz-ry+rx 0
or x-2y-z 0 ... (iii)
In close loop4-8-7-3-4
-r(y-z)- rx2(y-z)-r(y-z)+rz= 0
or -4(y-z)+z 0
or -4y+5z 0 ..'.(iv)
From equations (iii) and (iv), we get
-4y+ 5 (x-2y) 0
or 5x l4y
V
Since X
r

5 V
y -x-
14 r

V V
Now =----
x+2y v 5V
-+2x-
r 14r

7
-r Ans.
12
Method II:

5
,,----------,, 8

k---------,17

2 3

Fig. 3.110
Our previous knowledge reveals that points 3 and 6 must be at the same potential. So must be4 and
5. If points of equal potential are joint by.a wire:the currents in the circuit do not change. The given
network ofresistors c~n bared~uccessively as shown in.fig. 3111.

r r
....
I 4.5 8
r r
...
\

r r r

r
,· . r
I
-
-
2 3.6 7
r r
...

r/2 4 5 r/2 r/2 4 -


r---'IMll~-·;--~wv.--.--,8. . ...
...--'WV\r--..:.;·,..:''---,8

r r/2 ,. >r r/2 21 >


'


>

r/2 r/2 r/2


z,__J·w--.·.,_-;;-'--:if~,,...,.,..v··w.--"7 21-~vw1r--½-----' 7
3, « 3, 6

r/2 4. 5
1,----,4, 5

2r ~ 1r
\ - => r
5 . : 5

r/2
2 3, 6 . 3,6 '

Fig. 3.111

1r
rx-
- -5= -7r Ails.
7r 12
r+-
5
Ex. 25 Twelve wires, each having resistance,, are joined to form a cube as shown in fig. 3.112. Find the
equivalent resistance between the ends of a face diagonal such as I ~nd s,
Sol.
5 r 8 5

r ;,
r
~ ~
r
;,

r
r
r
=
<, 7 1-2i1

r
r "
r
2 J 2 J
Fig. 3.112
r- - ..
V
. 5 ___=• _ V6
'
. ' !73_ =
- !74
5 8

=> '6~---v,v....-----;7

2 ~---'IM/',~--~J

3r
=> R1s=-
4
Ans.

Infinite ladder: Find the equivalent resistance of the infinite network as shown infig.3.113.
r r r
----------
J' r ,. r
r r r
. -.. -. --.. '
Fig. 3.113
Let equivalent resistance between the points is R. The equivalent between the points remain
R if one more cell is connected with the network. i.e.,
r r

r ~rR
R
~r+R
. ..,... r
Fig. 3.114
rR
R r+--+r '"
r+R
rR
2r+--
r+R
or . Rr+R2 2 ,2 +2Rr+ Rr
2
or R -2Rr-2t2 = 0

2
2r±~(2r) +4x2r2
R
2

r±,[3 r
Retain only positive sign, because R always positive.

R = (v'J+I)r. Ans.

Ex. 26 (a) What is the potential difference between points A and Bin fig. 3.//5 when switch Sis
open?
(b) Which point,A or B, is at the higher potential?
(c) What is the final potential ofpointBwhen switch Sis closed?
(d) How much does the charge on each capacitor change when Sis closed?

18V

60 6µF

.4 ' - - - - ' ----...B


s
30

Fig. 3.l/5
Sol. (a) In open circuit, there is no current in the resistors. Therefore,
VA= 18 Vand V8 =0
VA-V8 = 18-0= 18 V Ans. 0
'(b) It is clear from (a), that A is at higher potential.
(c) When switch is closed, the current starts flowing in the
resistors. The circuit can be realised as in jig. 3.116
18V
___!!__ = 2A
6+3
We have 6x2=12V
and 3x2=6V
Since Vp,,,;O, (N Ans.
Fig. 3.l/6
- : ~ _'.
.:16 lL, _ •• 'lto,!M~ ,8. --~ ,,1'!, -~- 'JIJ,
:\11,:;;EEeTRICITY:,c. 1lGNFFISM.Yi21 -; ::"¥· ,. -
t;J,,:l:'::..
-' ' ,' · • _ ·
~1 ,.1,; :=,,
, _ ,-i ,
\
''"-"" ,0 ~-1 ;;;,.,, - ~·-

(d) With the switch S open each capacitor is charge~ with potential 18 V-.Theref~re-char~e on\ \\
_them_:. ~ ,.
, ' • _ Ch:irgec;n 3µF capacitor = 3 x 18 =54 µC
. : - Charge on 6µF capacitor = 6X I 8 = J,()8 µC I
When switch Sis closed, the p.d. across 3µ¥ capacitor
3x2=6V
and charge stored 3 x 6' = 18 µC
The p.d. across 6µF capacitor
6x2= 12V
and charge stored - = 6 x I2J= 72 µC
Now charge in charge on 3µF capacitor_
· = 18-5
= -36µC
and change in charge on 6µF capacitor ·
72-108
= -36µF
/
/

Ex. 2 7 -A de source with internal ;esistance R0 is loaded with three iden9caf;:esistanc~s R interco_nneded
' as shown in fig. 3.117. At what valu<>-0f R will the thermal jimver-generated in thecirc~it'be lhe
highest? · · ~- ·. : · _ \ \ ·
.I - -
--=R -·
I '.
'
!'
l ~ - - ~ .... jj /A'""i,---'Wlw.--)--,B
(a)
(b)
Fig. 3.117 \ I ·.
Sol. The given arrangement of the resistors is equivalent to the three resistors in parallel (fig.·b) Thus
- I

I equivalent resistance between the terminals


) . . \

-R = !!:._
AB 3
For the maximum thermal power, the net external resistance of the circuit is equal to the internal
resistance. Thus
R
3 Ro __ -
or R =/\ TT1:0 Ans. -
• I

Ex. 28 Find the equivalent resistance across th'e terminalsA and B ofii tetrahedronADCk show~ iJ
fig. 3.118. The resistDn~es ofall the edges ofth~ tetrahedron are equal. - - · i ./
I) / - ' -
I
.~ I

,..- A /

Fig. 3.J.f8
/
S~l. ' Fro!11 symmetry cons_iderations it can -be-easily I)

understand that there is no current in the edge CD, and. , • r 1,1 •


, · so is can be ignored for the equivalent resistance. . i1
ii
The resista\cesAD and DB are in series and so equal to ;-
- - -2R: Similarly_resistancesACand'CB are in series and so i2
C
equal to 2R. Now 2R, 2R and Rare in pafallel across the A ;_,
terminals A and:.B ....Tb11s :-c:-.. . _=-~--~= - ;,
, ,
I I I R
-+--+- B
2R 2R R·

or
R
2
Ans.
- Fig. 3.119

Ex.29 Determine the resistance RAO between points A and O of the frame formed by eight identical wires
of resistance R each (see fig.' 3.120).
c; CID
'

3R
2

',''

D
Fig. 3.120 Fig. 3.121 '
Sol. Fro"m symmetry considerations the potentials of points C and D are eq~al. ThFrefore, this circuit
can be replaced by joining the junctions C and D. ·
The equivalent resistance between C and 0,

R 3R
-x- 3R
2 2
R 3R 8 '
--1,-
2 2
R 3R 7R
Resistance ofarrn A CO : R
AC
+R
CO
=-+-=-
2 8 . 8
Now resistance between A and 0

R(7R)
, s 7R
R+ 7R 15
8 /

Y- V 15V
Thus the current
RAo ~ 7R = 7R
"'\ Ans.
\
15
Ex. 30 Determine the resis(anceRAB between points A and B of the frame formed by nine identical wires
of resistance R ea~h (see jig. 3.122).

A R R

R R R
R
'
R R
B
Fig. 3.122
Sol. This circuit can not be simplified by connecting or disconnectingjunction or can not be reduced to
series or parallel connection. So it require circuit. analysis by Kirchoff's law. Let us apply a p.d. V
across the terminals A and B. The current in different resistors are shown in fig. 3. 123

H G
V

Fig. 3.123
By junction rule,
i2 + i3 ,;, i4 + i5 ... (i)
Now use Kirchhoff's second law in closed loopsACEA,.CDECandA CD BG HA, we have
' -i 1R -i3R + i2R = 0 .,.(ii)
- i5R + i4R + i 3R = 0 ...(iii)
and - i 1R- i5R-i2R + V= 0 ... (iv)
Solving above equations, we get
.-._,, ,,

V
and
3R
V V
i1+i2 = ~
RAB
,,+-5
5 V 5V
11 i1
= V
!Ix-
3R
15R
Ans.
11
Ex.31 The circuit diagram shown in.fig. 3.124 consists ofa very)ijigf(~~firiite) number of elements.
The resistances of the resistors in each subsequent element differ/ii~ factor k from the resistances
of the resistors in the previous elements. Determine the resistance RAB between points A and B
if the resistances of the first element areR 1 and R 2•
R1 C kR 1
A --¥._'l'v----ii--¥._W
_--,·······~·····

kR2 .

13-------~-------------
D
Fig. 3.124
Sol. From symmetry considerations, we can remove the first
element from the circuit; the resistance of the remaining
circuit between points C and D will be Rev = k _RAB-
Therefore, the equivalent circuit of the infinite chain
will have the form shown in.fig_ 3_ J25_ 13:----/D,----....1
- Rz(kRAB)
Thus R1+-~-~- Fig. 3.125
R2 +(kRAB)

R1 [R2 +kRAB]+kR2 RAB

Rz RAB +kR~B R1 Rz +kR1 RAB +kRz RAB

or kR°;,B +(R2 -kR1 - kRz) RAB -R1R2 = 0

2
-(Rz -kR1 -kR2)±~(R2 -kR1 -kR2) +4kR1R2
RAB= 2k
As resistance can not be negative, so

Ans.

Ex. 32 An electric circuit is shown in.fig. 3.126. The cells are of negligible internal resistances. Find
(a) the current in 3 Q resistance and the cell of8 volt
(b) the charge on the capacitor.

1 1 , 14V B
1---.--.----< I

sn 3()
f'I·1---+- ·"'~-----,tC
. _ _,,. _."'-...
iz
>[OQ
, sv 6n
a,1-~,1-_--i, D
.---··-··-··- l V.111
1----------
1r,,'-----i 13V j I:611F E
Fig. 3.126
I
.'
I

Sol. At steady state there is no current in the branch of capacitor. The currents in the different branches.
of the circuit are shown in fig. 3. /26. At junction H
i = ii+ ;2 ... (i)
Applying Kir~hliotrs loop rule in close loop A BC HA
+4+3i2 -Si1 0. _·
or Si 1 -3i2 4 ... (ii)
In close loopH CD G H
-3i2 - I0i-6i+8 0
or -3i2-16(i 1+i2)+8 0
or 16i 1+19i2 8 ...(iii)
Solving equations (ii) and (iii), we get
;2 0.SIA
and I.I I A
The current in 8 V cell
"' i1 + i2
~I.II +0.51 = 1.61 A
To get the p.d. across the capacitor,join pointJ andP by a hypothetical cell VJE• which obviously
be th.7P:d. across the capacitor. In close loop GD J F G, we have ,
/ -8+6i+VJE-3 0
or _ -8+6xl.61+VJE-3 0
/or __ VJE 1.33V
Thus the charge on the capacitor
Q CV
6x 1.33
= 7.98µC Ans.

Ex. 33 -
A wire forms a regular hexagon and the angular points are joined to the centre hy wires each of
'

whicli has a resistance (-;) of the side of the hexagon. Find the current entering at one angular

point and leaving it by opposite point if r is the resistance of any one side of hexagon.

Fig.-J;J-27-
/ . r
Sol. Suppose r' is the resistance of each wire connected to centre, where r' = - .
- - n --........,
Because of symmetry the currents in upper half and lower half of the circuit are equal. Using
Kirchhcifl's laws, we have
...(i)
and -Jii)
In close loop ABOA, we have
r
-.i1 r- i4 !_ + i2 0
n n
or ni 1 +i4 ... (iii)
In close loop BCOB, we have
r . r
-ir+i-+14- 0
3 4n n
m 2~ ... (iv)
From equations (i) and (iv), we get
I

From equation (iii)


.1 ni
m1+--
n+2
n+2 .
'2
n(n+3)
Putting the value of i 1 in equation (ii), we get
2(n+2). . (n+.4)(n+I).
n (n +3) 12+12=
n ( n +3) '2

n(n+3) .
Ans.
(n+4)(n+ 1) '·

Ex.34 Find the resistance between the terminals A and B in fig. 3.128.
Sol. The delta FDE may be replaced by its equivalent star as shown in fig. 3.128 (b). In fig. 3.128 (c),
two series resistances along CES and GD.Shave been replaced by a single resistance. The equivalent
resistance

RAB 2Rll(3:1; 6:
C .A
; r-----A

E
2R E,
I
3 ''
(b)
C A C A .-1

: 2R 2R
5R · SR
- -B -5R6 3R
3 3 F B ·-
• 2
·: 2R

. 2/1
D
·-
: 3
E
2/1
3
s B
(d) (e)
(c)

Fig. 3.128

Ex. 35 Using Kirchhofrs Jaw, find the magnitude and polarity of voltage V shown injig. 3./29. The
directions of the hvo current sources are as shown.

A A

+F
2Q 30~1 n 4n s.-1 2n 30.-1 6Q 4Q 8A

/3 B
(a) (b)

Fig. 3.129
Sol. Suppose the directions of current in the circuit are as shown infig. 3.129 (b).Atjunction A
i1 + i3 +8 i2+30
or i 1 - i2 +i3 22 ... (i)
V -V V
where i 1 =-,i2
2
=6. and i3
4
Substituting these values in equation (i), we get

: -(-:)+: = 22
V V V
or -+-+- 22
2 6 4
or V 24V
V 24 . V 24
=2 =y=I2A 12 = - - = - - =-4A
6 6
V 24
6 =4 =6A Ans.
r
'----'---=------'-·~,_•'_D_,_BE_cr_·_C_v_RR_.E_'NT'---···_AN_v_E_LE_CT_R_)cc,..:_C._1R_c_v_1TS
__ . ~

Ex. 36 lnjig. 3.130, all the resistors have a resistance of 4.0 Q and all the batteries are ideal and have an
emf of 4.0 Veach. What is the current through resistor R?

h e d

j n
k
R

.....

q r
Fig. 3.130
Sol. Taking a close loop a b c def g h j k I m n op q rs a and assuming current i in the direction of
movement ofloop, we have
+4-4-4-4-iX4 0
or 2A. Ans.

Ex. 37 Determine the resistance RAB between points A and B of the frame made of thin homogeneous
wire (fig. 3.131), assuming that the number of successively embedded equilateral triangles (wide
sides decreasing by hall) tends to infinity. Side AB is equal to a, and the resistance of unit length
of the wire is p.

a/2 R/2

A +------"R'-------'tB

i--------a·-----<
(a) (b)
Fig. 3.131
Sol.

l .
The resistance of the network between A and B will not change ifwe add one more triangle across
C andD. As the side CD= AB/2, and so its resistance will be RAof2. The equivalent network can be

., [ R RAB
drawn as injig. 3.131 (b). Thus Rm= ;_ . Again R/2 and R/2 are in series with Rev·
R +...AlL .
2
:. Net becomes [~ + R RiB ] - I

--
R+__A!!_
2 I
.
R. RAB ]
RR+ 2
[ RAB
R +--

l
2
Finally RAB -.L

: [R+R+ Ro/
. . R +,-A!L
. '. 2

.
On solvmg, we get RAB
R(J/-1)
=~--~
'(J?-1)
ap .
' 3' ' 3
____.,...-
3.6 ELECTRICAL INSTRUMENTS
Meter bridge
The most simple practical application of Wheatstone bridge is meter.bridge. It consist ofone meter long straight
and uniform wire of mangaJin or constantan stret_ched along a 'meter scale. We know the balanced bridge
p R ," I . . ..

condition, i.e. Q =S. After !etting balance condition wf


can measure unknown resistance by s = R. Let 7,
. .
~lance point is at a distance of Cfrom A; then p
Q
=-C-
l-C ·---(--)--
; 1-f , .. _
'S= -f- R 'I
I_______ .... :
\

I A· B

.,, lu,111111hrn11Eul1111rrml1•1•uml,uwu1lur::'.1.Um\•1!·1J1·1mt,,I
· Metre scale
'--------; ~------'
Fig. 3.132
Post office box
lt is the compact.form of Wheatstone's bridge originally designed to measure the resistances ofelectric cables,
and telegraphwires in post offices. lt consists of three arms AB, BC and AD of the Wheatstone bridge. The
unknown resistance Sis connected in the fourth arm CD. Two spring keys K 1 and K2 are also provided, which
when pressed make internal connections with terminals A and B respectively. Key K 1 is known as battery key
-· and key K2 as the galvanometer key. To determine the unknown resistance, the connectors are made as shown

injig. 3.133. Unknown resistance can then be calculated by S = 7, R. _,


-
B
A p B' 0 C
. I .,
\
R

/
" /
~--)l----1/ ~ 1-"--~
s
',,\ /
(a) Post office box
·K1
(b) Equivalent circuit
Fig. 3.133 /

Potentiometer . _ /
It is an ideal device which is used to measure emf of a cell or to compare emf of the cells. It is also used to
measure internal
/
resistance of a cell. It consists of a long wire usually
.
IO m in length and made up of manganin
.

,,,., or constantari and a battery o(known emf. . .

. /
.-

Potentiometer wire
Fig. 3.134
Potential gradient across the wire
· a is the potential drop per unit length of the potentiometer wire. In the given figure ..
eR0
Ro+Rh
\
potential gradient VAB = e Ro Vim
~ (Ro+Rh)L
Why lo)!g· wire used in p_ote:ntiometer ?
-We known that sehsitivity ofan instrument is inversely proportional to i,

sensitivity
. t oc -

/ / \
I

and ·j oc -=-
-..... .. R /
:. ; sensitivity = R/ ·
Sin<;e r- - ~ :. . R =·£. -
. -····
:. se_ns111v1ty_ ec_ £. /
./

1. Emf of the cell: Connect the cell whose emf, is to be detennined and find balancing length with the help
of galvanometer. Let it is tfrom A.

C ----<J
,lt--1-------~---~---</3

R,. L

Fig. 3.135

The resistance of£ length of wire Roe


[,

Potendial difference across it i RAJ= (


.
e
flo+Rh
)x flol
L
The emfofthe cell VAJ

i.e., emfof cell

£@§ It should be remembered that~< e.


2 Comparision of emrs of cells : The given cells are connected in tum to the galvanometer by means of
two-way key and the position of balance point on the potentiometer wire is obtained for each cell. Let C1
and C2 be the balancing lengths with the cells ~I and ~2 respectively, then


~ ~I
1-
. _,·1
,- wfi~
, ~2 f2 ;

Al-'-------,------~/3

Fig. 3.136
3. Determine internal resistance of the cell : Initially key K' remains open and balancing length C1 is
obtained. Which detennine the emf of cell i.e.,~= kl 1 • ••••(i)
c----
DIRECT CURRENT AND ELECTRIC .CIRCUITS

J
A B
i •
;,r i' /

i' i'
2
Fig. 3.137

Now key K is closed.Again find the new balancing length. Let itis e2. In close loop 1-2-3-4-1, i' =
.
(-c,-)
R+r

The p.d. across resistor R = iR = (-e-)


R+r
R

This p.d. must equal to the p.d. across f 2 length of potentiometer wire.

~ ...(ii)
R+r
From equations (i) and (ii), we have

(kf 1 ) R
kf2
R+r
or e1 R (R + r) f 2
(f1 -f2)R
r =
f2

Ex. 38 Consider the potentiometer circuit arranged as injig. 3.138. The potentiometer wire is 600 cm
long. (a) At what distance from A should jockey touch the wire to get zero de(lection in the
galvanometer? (b) If the jockey touches the wire at a distance of560 cm from A, what will be the
current in the galvanometer ?

~-----i
~ r

R= 15r
A 13

s~
Fig. 3.138
Sol. (a) Let C is the balar.cing point and f is the distance of jockey at which deflection is zero in the
galvanometer. The p.d., across f length of potential wire
iRAC

VAC = l~r x( ~~~ )e


This p.d. is e q u ~ l l . That is

' 1;12 =--J__x !Sr e


16r 600
which gives e = 320cm Ans.
(b) The resistance of560 cm length of wire ,.
!Sr
= -x560=14r
600
Let i is the current in the galvanometer

i; ,. s r
7
io
i
=>
R= 15r (i0 - i) 14r ,.
A B I
5
r ~/2 r
2 ~
F;/2

Fig. 3.139
In close loop 1-2-34-1
-l;/2-ir+ 14r(i0 -i) 0
or 14i0r-15ir
In close loop I-4-5-6-7- I
1;12
\
...(i)

'
"'
-14r(i0 -i)-i0 r-i0r+~ = 0
-16i0r+l4ir = -{
Solving equations (i) and (ii), we get
n"'
••. JI

i = 3-- Ans.
22r · "
Galvanometer
Galvanometer usually uses to detect the d~on of current. When'used-for this purpose its resistance becomes
immaterial. If it uses to measure current or poteirnat-without any modification, its resistance affe'ctl:he cifc~it
parameters. Therefore to make it a useful instrument, galvanometer is to be modified according to requirement.
_ Let__!, is the full scale deflection current of galvanometer, and G is its resistance then potential difference across
it

··•... _
..
··-:
·-...
·-.•. ,
···-...
ig G

I
Fig. 3.140 Galvanometer
Ammeter
It is used to measure direct current. A galvanometer can be changed into an ammeter by connecting small
resistance S, called shunt in parallel ·with the ga-lvanol11eter. To measure the current in a circuit, an ammeter must
be placed in series in the circuit so that the current to be measured actually passes through the ammeter.

Ammeter
e
Fig. 3.141
Figure shows an ammeter in which shunt Sis connected in parallel with galvanometer
Resistance of ammeter

...(i)

If i is the current to be measured, then


S(i-ig) = Gig

.
:- s
I
~-·r ,-,g·1;· J-.
. . G,
i
... (ii)

-- - --- ------·- --
- --- -·-. - . - .l
' s
or I ; = i-- ...(])
1 g .S+G
Equation ( 1) i; the working equation of an ~mmeter ___ -- - -

Also i ( SG )
ig = G S+G
.,. /

. RA
or I ig = 1 G ...
(2)
/~---·· - --- - --- /
'

Let us consider a circuit having a source of emf~ without the aid of ammeter, actual current i = 'f,!R .
.R R

i'

I; I; I
(a) (b)
Fig. 3.142
When ammeter is connected to measure the current, its value changed due to resistance of ammeter. Let it is I'.

i' = _s_<i
R+RA

i' 1R =i , if RA= 0 (ideal ammeter)


.
Percentage error in measuring current by ammeter

i __s_
i-'.'x100 R R+RA xlOO=~xlOO
i I R+RA
R

Conversion of an ammeter ~nto other ammeter


It is same as the conversion of galvanometer into ammeter. If i 1 is the.initial range of ammeter and i is the final
2
range, then

.
'g =i
·( S+G
s ) ... (i)

Fig. 3.143
If s 1 and s 2 are the sentivities of ammeters, then

1 i2
s2 i1
The equation (I) now can be written as

)""
-:;:;-= (
Sz ..S
S+RA , ... (3)

Voltmeter
It is used to measure potential difference betweentwo points
of a circuit containing direct current source. A galvii.nometer
can be changed into a voltmeter by connecting a very large
resistance (R 0) in series with the galvanometer. The voltmeter
is always connected in parallel across the points.
The
I
potential difference across '
R =• potential
· · · "
difference
-
across voltmeter
· • · · --- - I

or , __ V ::."" Jgtl'__;=[lf(<:;_+Ro)_,

or R0 = !.__G ... (4)


-- , _ _,ig_ --
The resistance of voltmeter = G+R 0
The actual potential difference across resistor R -------v-----.......
V = iR Fig. 3.144
When it is measured with a voltmeterofresistanceRv, it is

V
= ,---
. RRv
R+Rv

iR
--R-.
l+-
Rv
If Rv ""?=, V'=iR= V
Thus, the resistance of ideal voltmeter is infinite.
Percentage error in measuring potential difference with a voltmeter

V-V' xl00
V .
[m ("¾l]x100
tR ..

{'1;-l
1+-
R
(00•-R-xlOO
Rv+R
Rv

1
= (
I+ -
Rv JxlOO.
R
for Rv ~ oo, % error= 0
Conversion of one voltmeter into other voltmeter : It is like to change galvanometer
into voltmeter.

~ V = igRv=ig(G+Ro!
~ Vz = ig (RYj + Ro)
~
Fig. 3.145

Ex. 39 An electrical circuit is shown in jig. 3.146. Cal~ulate the potential difference across the resistor
of 400
ohm as will be measured by the voltmeter V of resistance 400 ohm either by applying
Kirchholl's rules or otherwise. ·
~---1 V ,___~
,,
400Q,

lOOQ
1000 200Q

JOOQ
lOV

Fig. 3.146
"
0
EJ,.E~1.J~ICl1Y &,~:A.QNETISM

Sol. The given circuit with the voltmeter reduces to a balanced Wheatstone's bridge.

R,,=400Q
~----<V1--~
2000
400Q

1000 100n 2000


IO0Q IO0Q 200Q
100n
l00O
!OJI
IOV

100 200Q
IO0Q 200Q
100n ~

100n 200Q
-
200n =>

!OV ~------1'-1--------.J
I iOJ•"

Fig. 3.147
The effective resistance across the source = I S0Q.
10 I
Current drawn from the cell i = -=-A
150 15
I
The current is_ equally divided into both the branches. Therefore current in 200 Q is A
30
J

The p.d. across 2Q0 Q 200X-J = 20 V. Ans.


30 3

Ex. 40 A galvanOIJleter has a resistance of30Q and current of2 mA is needed to give full scale deflection.
What is the· resistance needed and how is it to be connected to convert the galvanometer
(a) into an ammeter of0.3 A range?
(b) into a voltmeter of0.2 V range?
Sol. GivenG=30Q,ig=2mA =2 x 10-3 A.
(a) If Sis the required shunt connected in parallel wit_h the galvanometer, then
s
--i
ig
S+G
2 X 10-J
s
--x0.3
or -- S+30

After solving, we get s 2<1._Q Ans.


149
(b) To convert galvanometer into voltmeter a high resistance R0 is connected in series with
galvanometer, and so
V i (G+R 0)
8
or 02 2 x 10-3 (30 +R0)
or Ro 70'2. Ans.

Ex. 41 A galvanometer having 30 divisions has a current sensitivity of 20 µ.A/ division. It has a resistance
_of25 ohm. How will you convert it into an ammeter measuringupto 1 A? How will you now convert
this ammeter into a voltmeter reading upto 1 n
Sol. The full scale deflection current
ig = 30x(20X 10--{j)
= 6xlQ-4A.
If Sis the required value of the shunt connected in para!lel with the gal~anometer, then

ig =
s .
--,
S+G

6 X lQ-4
s
--xi
or
S+25

After solving, we get s Ans.

The resistance of the ammeter

SG
S+G

0.0150x25
0.0150'2
0.0150-t-25
To convert this ammeter into the voltmeter, we can ~se'
V ig(RA +R0)
Here V lV,ig"'lA.
I (0.0150+R0)
or Ro 0.985'2. Ans.
MCQ Typ_e_ 1 _ & ~ 3.1
1. The value of the resistance as measured across 5. In the figure, the resistors are in series or parallel.
terminals A and Bin figure would be Select the right trend :

oarn(a) series, series, series


(b) series, series, parallel
(a) 5kQ (b) I0kQ (c) series, parallel, parallel
(c) 15 k Q (d) 20 k,Q (d) none
2. The equivalent resistance between A and B is 6. The figure here shows conduction electrons
moving leftward through a wire. Select the
R• D quantity/(ies) which have direction rightward :
A---,,---,,w.---,--f,

R, R, R3 l (a)
s- - - s
i, Vd (b) i,j

B _. (c) j,E, vd (d) i,j and E.


7. Figure shows four situations in which positive
(a) RI (b) Rz and negative charges move horizontally through
a region and gives the rate at which each charge
R2 R.; R R1 R.; moves. The situations according to the effective
(c) + I (d)
R2+R4 R1 +R4

-
current through the region, greatest first :
3. The time constant of the circuit is:
7 C/s 3 C/s 4 C/s 2 C/s 3 C!s
e-+ +-0 +-0 +-0
!t
hv,
e-+
5 C!s
+-e

1 (A) (B) (CJ


l C/s
(D)

./$.
fl\\,\, T (a) A>B>C>D
(c) A<B<C<D
(b) (A=B)>C>D
(d) (A=C)>D>B
(a) RC (b) RC/2 8. The equivalent resistance between A and B is :
(c) 2RC (d) =
4. The time constant of the circuit is :
m

J}
B

(a) 10 Q (b) 20 Q
(a) RC/2 (b) RC
(c) 2RC (d) zero (c) 15 Q (d) none
9. The equivalent resistance between A and B is : these resistances are interchanged, the balance
point shifts by 20 cm. Neglecting any correction,
the unknown resistance is
(a) 3Q (b) 4Q
(c) 5Q (d) 6Q
13. A wire ofresistance 4 Q is bent to form a circle.
The resistance between A and B is
;l fJ

_ 33 Q 66 Q
(a) (b} L.......... B
21 45

~Q l "
(c) (d) none
7
10. The figure shows three cylindrical copper
(a) 4'Q (b) 3Q
conductors along with their face areas and 3
lengths. Rank them according to the current (c) -Q (d) none of these
4
through them, greatest first
14. The potential difference, in volt, across the
L resistance R2 in the circuit shown is :

g
'-'-----~) ()A/2 1.5 L
(l) (II) R1~tjQ

L/2 R3=30 a
Od2D Ri"' I~ 1.1
(III)
(a) 5' (b) 7.5
(a) i 1 =;i2 =i3 (b) i 1 >i2 >i3 (c) IO (d) 15
(c) i 1 < i2 < i3 (d) U1 = i3) > i2 15. Consider the following statements regarding the
11. Figure shows a rectangular solid conductor of network shown in the figure :
edge lengths L, 2L, and 3L. A certain potential
R R
difference V is to be applied between pairs of
opposite faces of the conductor as shown in A B
figure : left- right, top-bottom and front-back. In 2R
which pair current is maximum :

I. The equivalent resistance of the network


4R

(a) left-right (b) top-bottom


between points A and B is
3 .
2. The current in resistor 3 R is zero.
(c) front-back (d) equal in all
3. The potential difference across R is equal
12. A resistance of 2 Q is connected across one to the potential difference across 2R.
- gap of a metre-bridge (the length of the wire is Which of the above statement (s) is/are
I 00 cm) and an.unknown resistance, greater than correct?
2 Q , is connected across ihe other gap. Where (a) I alone (b) 2 alone
(c) 2and3 (d) 1,2and3.
16. What is the effective resistance between the
terminals A and B of the mesh shown in figure?

(c)

(d)
(a) 2R (b) R
Ar~----t
(c) R/2 (d) R /3
22. The current from the battery in circuit diagram
17. The temperature coefficient of resistance for a
shown is
wire is 0.00125/°C. At 300K its resistance is I
ohm. The temperature at which the resistance 2!2 A 1n
becomes 2 ohm is
(a) 1154K (b) !!OOK 15V-:;-
(c) 1400 K (d) 1127 K
18. When a piece of aluminium wire of finite length 6n In
is drawn through a series of dies to reduce its
o.sn
diameter to half its original value, its resistance ..
will become 8Q B lOQ
(a) two times (b) four times
(c) eight times (d) sixteen times (a) I A (b) 2A
19. A steady current flows in a metallic conductor of (c) 1.5 A (d) 3A
non-uniform cross-section. The quantity/ 23. In the circuit shown below, the cell has an e.m.f.
quantities constant along the length of the of IO V and internal resistance of I ohm. The
conductor is/are other resistances are shown in the figure. The
(a) current, electric field and drift speed potential difference VA- V8 is
(b) drift speed only
(c) current and drift speed s
= l\lV
f-----,
(d) current only r=lQ"
20. A strip of copper and another of germanium are
cooled from room temperature to 80 K. The IQ
resistance of 4Q A 2n
(a) each of these increases
(b) each of these decreases - -
(c) copper strip increases. and that of
2Q B 40
germanium decreases
(d) copper strip decreases and that of (a) 6 V (b) 4 V
gennanium increases (c) 2 V (d) -2 V
21. Express which of the following setups can be 24. In the given figure, potential difference between
used to verify Ohm's law · AandBis
rM,NW--,--,--{.A
7

lOkQ
(a)
30V - -- _D
. A

IOkQ lOkQ

B
(a) 0 (b) 5volt
(c) !Ovolt (d) 15 volt
25. If each resistance in the figure is of 9 0 then 29. Six equal resistances are connected between
points P, Q and R as shown in the figure. Then
reading ofammeter is
the net resistance will be maximum between
p

: - . '
9V -::-
-

@
Q R
(a) 5A SA (b)
(c) 2A (d) 9A (a) PandQ (b) QandR
26. Two wires of equal diameters, of resistivities p 1 (c) P and R (d) Any two points
and p2 and lengths eI and e2, respectively, are 30. The total current supplied to the circuit by the
joined in series. The equivalent resistivity of the battery is
combination is

P1f1 +p2£2 P1f2 +pzl1


(a) (b)
f1 +£2 f1 -£2

P1f2 +p2f1 Pili -P2f2


(c) (d)
£1 +£2 £1 -£2
Z.7. In the arrangement of resistances shown below,
the effective resistance between points A and B (a) I A (b) 2A
is (c) 4A (d) 6A
sn 10n 15Q
31. In the adjoining circuit, the battery ~ 1has an
p
e.m.f. of 12 volt and zero internal resistance while
A
- 10n 100 - B
th,e battery ~ has _an' e.m.f. of 2 volt. If the
galvanometer G reads zero, then the value of the
resistance X in ohm is
Q
!OU 20n Jon soon
(a) 20 0 (b) 30 o
(c) 90 0 (d) 1100
xn
28. Calculate the equivalent resistance between A
andB D'-----,-.~--~C
312 312 312
(a) 10 (b) 100
(c) 500_ (d) 200
A 30 30 B 32. A cell of e.m.f. ~ is connected with an external
resistance R, then p.d. across cell is V. The
internal resistance of cell will be
30 30 . 30 (~-V)R
(a) (b)
(a) 2-o (b) 30 V
2

(c) 60 (d) 2-o (c) (d)


3
33. The potential difference in open ci.rcui! fora cell in an eel are arranged in I 00 rows, each row
is 2.2 volt. When a 4 ohm resistor is connected stretching horizontally along the body of the fish
between its two electrodes the potential containing 5000 electroplaques. The arrangement
difference becomes 2 volt. The internal resistance is suggestively shown below. Each
of the cell will be electroplaques has an emf of0.15 V and internal
(a) I ohm (b) 0.2 ohm resistance of 0.25 Q
(c) 2.5 ohm (d) 0.4 ohm
34. Consider four circuits shown in the figure below. 0.15 V 0.250
In which circuit power dissipated is greatest ~ FvwM-±j w.-""""'
(Neglect the internal resistance of the power
supply)
~FvwM-±t w.-IM'r-1
100 rows

(a) 1;- - R R
5000
The water surrounding the eel completes a
circuit between the head and its tail. If the water
surrounding it has a resistance of 500 Q, the
current an eel can produce in water is abollt
R (a) 1.5A {b) 3.0A
(c) 15A (d) 30A
36. Find out the value of current through 2 Q
R resistance for the given circuit

--
··wv 5Q
.
100 20v:;

(c) I; 20

R (a) 5A (b) 2A.


{c) zero (d), 4A
37. Two sources of equal emf are connected to an
external resistance R. The internal resistances of
the two sources are R 1 and R2(R 2·> R 1). If the
R potential difference across the source having
internal resistance R2 is zero, then
(d) i; :~ R
(a) R = R1 R2 I (R1 + R2)
R
(b) R = R1R2 I (R2 -Ri)
(c) R = R2 x(R1+R2 )!(R2 -R1)
35. Eels are able to generate currentwith biological
cells called electroplaques. The electroplaques (d) R=R2 -R1
38. A 2 volt battery, a l 5Q resistor and a 42. In the· given figure, battery ~ is balanced on 55
potentiometerofl 00 cm length, all are connected cm length of potentiometer wire but when a
in series. If the resistance of potentiometer wire
is 5Q, then the potential gradient of the resistance of l OQ is connected in parallel with
potentiometer wire is the battery then it balances on 50 cm length of
(a) 0.005V/cm (b) 0.05V/cm the potentiometer wire then internal resistance r
(c) 0.02 V/cm (d) 0.2 V/cm of the battery is
39. In the diagram shown, the reading of voltmeter
is 20 V and that ofammeter is 4 A. The value of R
should be (Consider given ammeter and 2V
voltmeter are not ideal)
i--------1 m------
~---fV>----~
20V A 1---------"B.-_ _,
I'

R
--'---fA '>--JMMl---'--
4A (a) IQ (b) 3Q
(a) equal to 5Q
(b) greater than 5Q (c) IO Q (d) 5Q
(c) less than 5Q 43. A voltmeter has a range O - V with a series
(d) greater or less than 5Q depends on the
resistance R. With a series resistance 2R, the
material ofR
40. AB is a wire of uniform resistance. The range is O -V'. The correct relation between V
galvanometer G shows no ·current when the and V' is
length AC= 20 cm and CB= 80 cm. The resistance (a) V'=2V (b) V' >2V
R is equal to
(c) V'»2V (d) V' <2V
44. The length ofa wire ofa potentiometer is l 00 cm,
and the emf of its standard cell is ~ volt. It is
employed to measure the e.m.f. of a battery
whose internal resistance is 0.5 Q. If the balance
point is obtained at C= 30 cm from the positive
end, the emfof the battery is
(a) 2Q (b) SQ
(c) 20 Q (d) 40 Q (a) 30~
41. In the circuit shown P,;, R, the reading of the 100
galvanometer is same with switch S open or
closed. Then (b) 30~
100.5

30~
(c)
(10~-0.5)

30(~ -0.5i) . . h
(d) , where i 1s the current m t e
(100) .
potentiometer
45. · For the post office box arrangement to determine 50, In the figure shown, the capacity of the
the value of unknown resistance, the unknown
condenser C is 2µF. The current in 2 Q resistor
resistance should be connected between
is
B C D
~ o o o • o o o • I

.I\ '·i

\ " I
i'A'--'.--+-' _ _, o
I •-00\000;000. 3Q

B .)."
I
"i-... Cl
2µF 4.Q

(a) Band C (b) CandD + -


(c) A andD {d) B 1 and C 1 6V 2.8Q
46. In a metre bridge experiment null point is obtained
at 20 cm from one end of the wire when resistance (a) 9A (b) 0.9A
Xis balanced against another resistance Y. If
X < Y, then where will be the new position of the
(c) _!_ A (d) _I_A
null point from the same end, if one decides to 9 0.9
balance a resistance of 4X against Y
{a) 50cm (b) 80cm 51. A torch bulb rated as 4.5 W, 1.5 Vis connected as
{c) 40cm (d) 70cm shown in the figure. The e.m.f. of the cell needed
47. A moving coil galvanometer ofresistance I OOQ to make the bulb glow at full intensity is
is used as an ammeterusing a resistance O. IQThe
maximum deflection current in the galvanometer
is I OOµA. Find the minimum current in the circuit (ii>4.5 W
H 1.5 V
so that the ammeter shows maximum deflection
(a) l00.1 mA {b) l000.1 mA
(c) IO.OJ mA {d) 1.01 mA IQ
48. Potentiometer wire of!ength I m is connected in
series with 490 Q resistance and 2 V battery. If
0.2 mV/cm is the potential gradient, then ~(r = 2.67.Q)
resistance of the potentiometer wire is ~-----<,',-..----~
(a) 4.9 Q (b) 7.9 Q
{c) 5,9Q (a) 4.5 V (b) 1.5 V
(d) 6.9 Q
49. Twelve wires of equal length and same cross- (c) 2.67 V (d) 13.5 V
section are connected m the form of a cube. If 52. A microammeter has a resistance of 100 .Q and
the resistance of each of the wires is R, then the
·effeciive resistance between the two diagonal full scale range of 50 µA. It can be used as a
ends would be voltmeter or as a higher range ammeter provided
a resistance is added to it. Pick the correct range
and resistance combination
(a) 50 V range with IO kQ resistance in series

(b) I O V range with 200 k.Q resistance in series

(c) !OmA rangewith I .Q resistance in parallel·


(a) 2R (b) 12R
(d) IO mA range with 0.1.Q resistance in
(c) ~R (d) BR parallel
6
_ _ _ _··•_ _ _ _ _ _DIREc:r(:~@!!;NT AJVD ELECTRIC CIRCUITS ·1n1
53. The potential difference across 8 ohm resistance 56. A• wire of length L and 3 identical cells of
is 48 volt as shown in the figure. The value of negligible internal resistances are connected in
potential difference across X and Y points will series. Due to current, the temperature of the
be wire is raised by't.T in a time I. A number N of
similar cells is now connected in series with a
.\ I
3!1 wire of the same material and cross-section but
20!1 30!1 60 n of length 2 l. The temperature of the wire is raised
by the same amount t.T in the same time I. the
24!1 sn 4JV value of N is

in
l (a) 4 (b) 6
I
(c) 8 (d) 9
(a) I60 volt (b) 128volt 57. The effective resistance between points P and
(c) 80 volt (d) 62 volt Q of the electrical circuit shown in the figure is
54. In the given circuit, it is observed that the current
/ is independent of the value of the resistance 2R 2R
R6. Then the resistance values must satisfy
2R
- .. .r.. r
I
.... p Q
2R
R,
- R• ....
..
2R
..2R
. R,
(a) 2Rrl(R+r) (b) 8R(R+r)/(3R+r)
(c) 2r+4R (d) 5R/2+2r
(a) R1R2R5 =R3R4~ 58. In the circuit element given here, if the potential
I I at point B, V8 =0, then the potentials ofA and D
(b) R5 + R6 are given as
LA 1.5 n 2.5 !1 2V
(c) -••--,·1-\',/,N~.--i~,--\~~~v~.-+c--ll} b
(d) R1R3 =R2R4 =RsR6
55. ln the given circuit, with steady current, the
potential drop across the capacitor must be (b) VA= +l.5V, Vo= +2V
,.· R (c) VA =+l.5V, Vo =+0.5V

V C
59. The current in a conductor varies with time t as
I= 21 + 312 where/ is in ampere and tin seconds.
2V Electric charge flowing through a section of the
2R
conductor during / = 2 sec to t = 3 sec is
(a) 10 C (b) 24 C
(a) V (b) V/2
(d) 2V/3 (c) 33 C (d) 44 C
(c) V/3
60. In the shown arrangement of the-experiment of we move in the direction of current and return to
the meter bridge if AC corresponding to null P. The potential Vat every point on the path is
deflection of galvanometer is x, what would be plotted against the distance covered (x). Which
its value if the radius of the wire AB is doubled of the following graphs best represents the
resulting curve?
r-----i 1--j 1 - - - - - ,
V

(a)

A-x-C B
~-------x
(a) X (b) x I4
(c) 4x (d) 2x V

61. A battery of internal resistance 4 n is


connected to the network of resistances as
shown. In order to give the maximum power to (b)
the network, the value of R (in Q ) should be

..... ... ~-------.X


R R
• V
·R : 611 R:
-~
-- .. ,.
R 4R
(c)

(a) 419 (b) 8/9


(c) 2 (d) 18 ~-------.x
62. As the switch Sis closed in the circuit shown in
V
figure, current passed through it is

20 V 2[2 Hl 5V
A
2Q (d)

~s ~-------x
64. A voltmeter connected to the terminals of a
(a) 4.5A (b) 6.0A
battery reads 6 V. When the lamps are lighted
(c) 3.0A (d) zero with the battery, the voltmeter reads 4 V. If the
63. The two ends of a unifonn conductor are joined resistance of the lamps is IO n, the internal
to a cell of e.m.f.~ and some internal resistance. resistance of the battery is :
Starting from the midpoint P of the conductor, (a) o.sn (b) IOn
(c) 3n (d) sn
65. Three batteries of negligible internal resistance 69. What will be the change in the resistance of a
and three resistors of 4, 8 and 12 Qare connected circuit consisting of five identical conductors if
as shown in figure here. The current through the two similar conductors are added as shown in
12 Q resistor is : the figure. If R I is the initial resistance and R2
that after connecting the two identical

Rz
conductors, then Ri is :

(a) 1.4

(a) 0.57 A
(c) 0.04A
(b) 1.09 A
(d) 1.14A 70.
(b) 0.4
(c)
(d) 0.6
0.5 NY
In the circuit given here, the batteries I;= 1; 1 = 1.0
66. In the given circuit/0 , V0 and the power supplied V, 1;2 = 2.5 V and resistances R1 = IO Q, R2 = 20 Q.
by the 15 Vbattery respectively are: The potential difference across the capacitor C
800 Q 3kQ is:

50
+
I,,
+
1200Q
(a) 7 mA,8.4 Vand58.8mW
(b) 0.01 mA,2 Vand0.02mW
(c) 7 mA, 8.4 Vand I05 mW
(d) 0.01 mA, 8.4 Vand 105 mW (a) 0.2 V (b) 0.4 V
67. ,In the given network of four resistances, the (c) 1.2 V (d) 0.5 V
equivalent resistance is : 71. In the part of a circuit shown in the figure, the
potential difference between G and H (VG - VH)
will be:

IA

68.
(a) 20Q
(c) J2Q
(b) 5.4Q
(d) 4.5 Q
In an experiment to measure the internal
resistance of a cell, it is found that no current
passes through the galvanometer when the
4Q

t 3V
+
2Q

IQ
SV
+ -

terminals of a cell are connected across 52 cm of


3A
the potentiometer wire. If the cell is shunted by a Hi
resistance of 5 Q, a balance is found when the '
cell is connected across 40 cm of the wire. The
internal resistance of the cell is: (a) ov (b) 15 V
(a) 0.15 Q (b) 1.5 Q
(c) 7V (d) 3V
(c) 0.5 Q (d) 0.05 Q
72, What is the resistance between .p and Q of the 74; A voltmeter has a resistance G ohm and range V
following circuit? Each resistance is of I Q: volt. The value of resistance used in series to
convert it into voltmeter of range nV volt is:
p Q

v3/
(a) nG (b) (11-l)G

G G
(c) (d) (n-1)
n
J e 75, The temperature coefficient of resistance of a
(a) IQ (b) 2n wire is 0.00125 i°C.At300 K, its resistance is IQ.
The resistance of the wire will be 2 n at:
(c) 3Q Cdl In
2
(a) 1127K (b) 1400K
(c) I 154K (d) 1200K
73. Poi,nt a in figure is maintained at a constant
potential of300 V above ground. The reading of
a voltmeter of the proper range, and of resistance
3 x I 04 n, when connected between point b and
ground is nearly :
a
(a) IO V
(b) 42 V
(c) 50 V I 00 kn
(d) 62 V
b

200 kn
&U!Zd4e 3,2
Directions (Qs. 1-3): Read the following questions (a) the current I through the battery is 7.5 mA
and select the right choices. More than one options
(b) the potential difference across RL is 18 V
may be correct.
(c) ratio of powers dissipated in R 1 and R2 is 3
1. When no current is passed through a conductor:
(a) the free electrons do not move (d) if R 1 and R2 are interchanged, magnitude of
(b) the average speed of a free electron over a· the power dissipated in RL will decrease by
large period oftime is zero a factorof9
(c) the average velocity ofa free electron over Directions (Qs. 2-13) : Read the following passages
a large period of time is zero and answer the questions that follows.
(d) the average of the velocities of all the free
electrons at an instant is zero.
Passage for Q.2 to Q.4
2. A battery of emf £ 0 = 5V and internal resistance
Two large parallel pistes are located in vacuum. One of
5W is connected across a long uniform AB of them serves as a cathode, a source of electrons whose
length I m and resistance per unit length 5 Wm- initial velocity is negligible. An electron flow directed
1. !
Two cells ofE 1 =. V and E, = 2V are connected toward the opposite plate produces a space charge
as shown in the figure. : causing the potential in the gap between the plates to

vary as V = a x 413 , where a is a positive constant,


5V SQ
and x is the distance from the cathode.

(a) The null point is a A


(b) If jockey is touched to point B the current 4. The electric field between the plate at a function
in the galvanometer will be going towards of xis:
B
(c) W~en jockey is connected to point A no E= a x 113 (b) E =-a x 213
current is flowing through I V battery.
(d) The null point is at distance of8/l 5 m from (c) (d) none of these
A
3. For the circuit shown in the figure 5. The volume density of space charge as a
functions of x

2 k.Q R 1 4 1/3
/, (a) · a Eo x 113 (b) -a X
9

24 v-~ 6 k.Q R, Rl 1.5 k.Q


4 2/3 4 4/3
(c) --aE0 x (d) aE0 x
9 9
6. The current density 8. The electric field £ 2 in dielectric layer 2 is :
' '
V d 2V
(a)
-~ Eo 03/2
3
rz;;
v--;;; (a) E2=- ·cbl E2 =
(d1 +d2)
d2

(b)
_ _±_ Eo 03/2
9
rz;;
\j--;;; (c) E2 =£I_!_ (d) /!2
P2v,.· .
P1 d2 P1d1 +p2d2
9. The surface density of extraneous charge at the
(c) -~ Eo o3t2 /e boundary between the dielectric layers to be zero,
3 f;;; the condition is :
(d) none of these

dV 4ax 113
[Hint:4(c) E=---;;;=--- ... (i)
3
5. (c)If cr is the surface charge density, then
[Hilll:1(c);8(d) ',
(J
£=- ... (ii) V = E1d1 + E2d2 ... (i)
Eo By equation of continuity
On comparing (i}"and (ii), we get
· E1 E2
cr= 4a
- - E0 x
1/3 )1 =h or -;;;-=i;;- ... (ii)
3 .
On solving (i) and (ii), we can get £ 1'and £ 2 •
Volume charge density,
9 (a).At the boundary between dielectrics, the
dcr 4 2/B surface density of extraneous charge
p=-=--oE0 x
dx 9 O=cr=cr2 -cr1 = EoE2 E2-EoE1 E1. l
6. (b) Current density,j = pv

[M" Passage for Q. 10 to Q. 12


where v =\j---;;;- ] A capacitor C is uncharged and capacitor C 0 has a
charge Q0• These are connected in series with a source
of emf~ in the circuit shown in figure. The switch S is
Passage for Q. 7 to Q.9 closed at I= 0.
The gap between the plates of a parallel plate capacitor
is filled up with two dielectric layers I and 2 with
thicknesses d 1 and d2 , perrnittivities Et and !=2 , and
resistivities Pt and p 2 • A DC voltage Vis applied tp.the
R
capacitor, with electric field directed from layer I to
layer 2. '·'

El £1 d, •S~ . -<;
····················--·················t 10. If i0 is the current in resistor Rat t=,O, then
E2 E,_ d,

7. The electric field £ 1 in dielectric layer I_is:


(b) io
(~)
R

(c)
.
1o =
(~-~) (d) zero
R
I
I \ ' \J
1-· 0

L_
11. If i is the instantaneous current, then 13. If i is the current in the 2 Q resistor, then

(a) i[-.'C +- 1
Co ]+Rdi
dt
=0 (a)
. 5
1=-A
13
(b) i=_i_A
13
. I
(b) ;tc+co)+iR=O (c) i=_!____A (d) i=-A
C 13 26

(c) { ~+
(d) none of these
;J- :i =0
14. The potential difference across 4 Q resistor is:

(a) _!____v
13
(b) 2-v
13
12. The instantaneous current i in the resistor R is: 2-v 2-v
(c) (d)
I I 13 26
(a) i = ioe RC (b) ,. = 1· e-RCo 15. The potential difference across the cell G is :
0 (a) 1.61 V (b) 1.46 V
-t t(C+Co>]' (c) 1.02V (d) zero.
(c) , , R(C+Co) (d) . . - [ RCCo
1 = 10 e r = ,0e
Passage for Q. 13 to Q. 15
In the circuit shown in figure£, F, G and Hare cells of
emf 2, I, 3 and I volt and their internal resistances are
2, I, 3 and I ohm respectively.

2n

Multiple matching
16. Column I gives certain situations in which a straight metallic wire ofresistance R is used and Column II
gives some resulting effects. Match the statement; 'in Column I with the statements in Column II and
indicate your answer by darkening appropriate bubbles in the 4 x 4 matrix given in the ORS.
Co/11111n I Co/11mn II
(A) A charged capacitor is connected to the ends of (p) A constant current flows through the wire
the wire
(B) The wire is moved perpendicular to its length with (q) Thermal energy is generated in the wire
a constant velocity in a uniform magnetic field
perpendicular to the plane of motion
(C) The wire is placed in a constant electric field that (r) A constant potential difference develops
has a direction along the length of the wire between the ends of the wire
(D) A battery of constant emf is connected to the (s) charges of constant magnitude appear at
the ends of the wire ends of the wire.
~-------..,-..-- .,- ... -
,___,___
M_·C_Q _:Type 3
r-:- - - - - - - - ------ ---- - -- -~--~-- - - - . · - - . . . - - - . . -- :
i .Read the two statements carefully to mark tile correct o~tion·out ofthe options given b~lo~. Selecuhe.~ight
1cho1ce." !f. ' e' ·

J· (a) !fbotli the statements are true and the State111e11t- 2 is the corre~t explanation of State111e11t- 1.:
I (b) If both the statements are true butStateme/11 - 2 is not the correct explanation of the State111e11t-'1.
, (c) IfState111e11t-1 truebutState,,;e11t-2,isJalse. ' /1 • · · ·'.

i (d) If State111e11t - 1 is false but State111e11t, 2 is true.

I. State111e11t - l 7. Stateme11t- I
There is no current in the metals in the absence Two non·ideal batteries are connected in series.
of electric field. The equivalent emf is larger than either of the
two.
State111e11t - 2
Statemelll-2
Motion of electron are randomly. The equivalent emf of the batteries will be
2. State111e11t-l average of the two. · ·
A stream of positively charged particle produces 8. Stateme11t - I
an electric field Eat a centrain distance from it. Two non ideal batteries are connected in parallel.
Stateme11t - 2 The equivalent emf is smaller than either of the
two emfs.
A current currying conductor produces an
Statemelll-2
electric field 2E at the same distance.
The equivalent internal resistance is smaller than
3. Stateme11t- I ,L J
either of the two internal resistances.
The electric bulb glows immediately when switch 9. Stateme/11 - I
is on. The resistivity ofa semiconductor increases with
Stateme11t - 2 temperature.
The drift velocity of electrons in a metallic wire Stateme11t - 2
is very high. The atoms of a semiconductor vibrate with
smaller amplitude at higher temperature.
4. Stateme11t-l
10. Statement - I
The connecting wires are made of copper. In the following circuit emf is 2 V and internal
Stateme11t-2 resistance of the cell is I !l and R = I !l, then
The electrical conductivity of copper is high. reading of the voltmeter is I V.
S. Stateme11t - I Stateme11t - 2
Electric field outside the conducting wire which
carries a constant current is zero.
Statemmt-2
r= IQ
Net charge on conducting wire is zero.
6. State111e11t - I
Kirchoff's junction rule follows from R= I!l
conservation of charge.
Stateme11t - 2
\ ' \ ...
V=~-ir where "=2V • i=~=IA
', 2 and
Kirchoff's loop rule follows from conservation '
of momentum. R=I!l.
--------~----D_1RE_cr_C~U~.R_·RENT
__
· ._,._._M'D_._E_LE_·_cr_R_I_c_c_·l_R_c_UI_T,S_~
11. Statement- I Statement - 2
Drift speed is the average speed between two The rate of work done by the battery is equal to
successive collisions. , the power delivered to the circuit.
Statement - 2 16. Statement- I
If/!,.{ is the average distance moved between
A moving coil ammeter can not measure AC.
two collision and l!.t is the corresponding time,
Statement · 2
• M AC does not show thermal effect.
then vd= I1m -
llt--+0 /!,.( 17. Statement - I
12. Statement - I In the figure the resisiors a~e in parallel because
When a current is established in a wire, the free they are along parallel lines.
electrons drift in the_ direction opposite to the Statement - 2
current and so the number of free electrons in The resistors are in series because current is not
the wire continuously decrease. divided between then.
Statement - 2
Charge is a conserved quantity.
13. Statement- I
In an electrolyte, the positive ions move from
left to right and the negative ions from right to
left, so there is no net current flows in an
electrolyte.
Statement - 2 18. Statement -1
The' current flows from cathode to anode in an In a meter bridge experiment, null point for an
electrolyte. unknown resistance is measured. Now, the
unknown resistance is put inside an enclosure
14. Statement - I
maintained at a higher temperature. The null point
The thermal power generated in a resistor is given can be obtained at the same point as before by
by P = i 2R and so it is proportional to i 2• decreasing the value of the standard resistance.
Statement - 2
Sfatement - 2
The thermal power generated in a resistor is given
Resistance of a metal increases with increase in
by P=V; and so it is proportion to i.
temperature.
15. Statement - I
In each electrical circuit, the thermal power
generated is equal to the rate of work done by
battery. r,
J

--- - ------ - DIRECT CURRENT AND ELECTRIC CIRCUITS

&Wtd4e 3.4
I. The current through a wire depends on time as i = (20 + 4t). Find the charge crossed through a section
of the wire in IO second. f·C
·'!' Ans.400C
2. The current in a conductor and the potential difference across its ends are measured by an ammeter and
a voltmeter. The meters draw negligible currents. The ammeter is accurate but the voltmeter has a zero
error (that is, it does not read zero when no potential difference is applied). Caculate the zeroerror if the
readings for two different conditions are 1.75 A, 14.4 Vand 2.75 A, 22.4 V.
Ans. 0.4 V.
3. Determine the values of currents i 1 and i2 in.the circuit shown in figure.

. , i2 -

3A 5A
;, IA

8A 4A

Ans. i 1 = -6A, i2 = 9A.


4. Figure shows an arrangement to measure the emf~ and internal resistance r ofa battery. The voltmeter
has a very heigh resistance and the ammeter also has some resistance. The voltmeter reads 1.52 V when
the switch Sis open. When the switch is closed the voltmeter reading drops to 1.45 V and the ammeter
reads 1.0 A. Find the emf and the internal resistance of the battery.

~---{V)---~

L....::.S,,.____....JA

Ans. 1.52 V, 0.07 Q.


5. The resistance of the rheostat shown in figure is 30 Q. Neglecting the meter resistance, find the minimum
and maximum currents through the ammeter as the rheostat is varied.

5.5 V
+ -

30Q

20n A11s. 0.15A,0.83A.


6. Find the currents through the three resistors shown in figure.

4n

4n 6n

4V 2V
+ l=-----=l +

Ans. Zero in the upper 4 Q resistor and 0.2 A in the rest two.
7. Find the current measured by the ammeter in the circuit shown in figure.

!On 10n Jon

son son
!On !On 10n

~----,--1 6V
+ tc-=---~-------\ A I------'

Ans. 0.4A.
8. The switch S shown in figure is kept closed for a long time and is then opened at t = 0. Find the current
in the middle IO Q resistor alt= 1.0 ms.
25µF

10n S
I ~
+ -
IOn

Ans. 11 mA.
9. In the circuit shown in figure, find the value of unknown resistor R.

SA
SA R
3A

.2on 40V !On


+

+ foov

Ans.4Q.
10. Find the current in each branch of the circuit shown in figure.

IQ 2n
IQ

IQ
2Q

14//
+ -
Ans. 6A, 4A, 2A, 4A, 6A (top to bottom).
11. What are the values of V1 and V2 in the circuit shown in figure?

6Q

+-----v,
+
.=..24V 4Q

2n •
1----...L----v,
v,
Ans. =8V, V2 =-4V.
12. Compute the value of battery current in the circuit shown in figure. All the resistances are in ohm.
6Q

+
4n -=-12v

Ans.6A.
13. Find the current supplied by the battery in the circuit shown in figure.
8Q

+
-=-24v

12n
Ans. SA.
14. Find potential
.
difference V.I.J.. in the circuit shown in figure.
2Q
-x
IQ 4V
+ -
-2v 3Q X
3Q SQ
+ /
4V

r---~
Ans. 3.7 V.
15. Two capacitors are charged in series by a 12 V battery (see figure).
(a) Wh~t is the time ~onstant of the charging circuit ?
(b) After being closed for the length of the time determined in (a), the switch Sis opened. What is the
voltage across the 6 µF capacitor ?

r II1-12V.IQ'
.
---.r
::F
6µF

Ans. (a) 12µs(b)2.53 V.


16. Find the effective resistance between the points A and Bin figure.
;. m

A 3Q B

Ans.2Q
17. Calculate the equivalent resistance between A and B of the circuit shown in figure. The value ofresistances
is in ohm. ·

3Q ' .
2Q

A B

6Q

12Q 9Q 12Q

: \ Ans. 6.75Q.
c_
18.
DIRECT CURRENT AND ELECTRIC emcqn-s
At what value of the resistance Rx in the circuit shown in figure will the total resistance between points
A andB be independent of the number of cells ? -· '

Ans. Rx= R (V3- I).


19. Figure shows an infinite circuit fom1ed by the repetition of the same link, consisting of resistance R1 =4.0
Q and R2 = 3.0 Q. Find the resistance of this circuit between points A and B.

20. Find the equivalent resistance of the resistors network across points A and B.

2Q 15Q
A ---'V\,N\~,--"""',J\,----,
B
IOQ
8Q

IOQ
20Q
30Q 40Q

Ans. 22.SQ.
21. Consider an infinite laddernetwork shown in figure. A voltage is applied between points A and B. If the
voltage is halved after each section, find the ratio RifR 2. Suggest a method to terminate it after a few
sections without introducing much error with attention.

Ans. 1/2.
22. Detennine the current through the battery in the circuit shown in figure :
(a) Immediately after the key K. is closed and
(b) in a long time interval, assuming that the parameters of the circuit are known.

I;
Ans. (a)-
R1
23. A 5.0 µF capacitor having a charge of20 µC is discharged through a wire of resistance 5.0 Q. Find the
heat dissipated in the wire between 25 to 50 µs after the connections are made.
Ans.4.7 µJ.
24. A capacitor of capacitance 8.0 µFis connected to a battery of emf 6.0 V through a-resistance of24 Q. Find
the current in the circuit.
(a) just after the connections are made and
(b) one time constant after the connections are made.
Ans. (a) 0.25A (b) 0.09 A.
25. How many time constants will elapse before the power delivered by the battery drops to half of its·
maximum value in an RC circuit?
Ans. 0.69.
26. The circuit shown in figure is made of a homogeneous wire of constant cross section. Find the ratio
Q 12 I Q34 of the amounts of heat liberated per unit time in conductors 1-2 and 3-4.
3,,s-------,.4

Jl',.------'12

Ans. Qiz =l1+6.J2.


Qi4 .
27. In figure circuit section AB absorbs energy at a rate of 50 W when a currenti =· 1.0 A passes through it in
the indicated direction.
\\ . I; ' /
~ -
.;A R-2.0!l B\\
(a) What is the potential difference between A and B?
.

(b) Emf device X does not have internal resistance. What is its emf?
(c) What is its polarity (the orientation of its positive.and negative tenninal)? .
Ans. (a) 50 V, (b) 48 V, (c) Bis connected to negative tenninal.
28. In the given circuit ~ 1 = 3 ~2 = 2 ~3 = 6V
R 1=2R4 =6Q,R3 =2R2 =4Qand C=5 µF
Find the current in R3 and energy stored in C.

• +

, R,
+
Ans. l.5A,4.0J.
29. Find the ammeter current in the circuit shown in figure.

10n

1 on
I
Ans. -A.
7
30. Find the currents i 1, i 2 and i 3 in the circuit shown in figure.

10n

10n 10n 10n

+
IOOV
3 10n 3
31. Find the current in the 2 Q resistor shown in figure.

B=--'\/\1'\./\r----l 1----""'c
3n
Ans. 1.85A.
32. When the circuit o£figure is in steady state, what would be the p.d. across.the capacitor? A:lso,find the
discharge current at the instant S is opened. ,· ,, ' ,.! :
' j •,

,_j· .. ,\,'

lµF
, '"

Ans. 8V, 1.8A.

,s . ', ·.+·1· '-'-~--~


24V

33, A varying voltage is applied to the clamps AB such that the voltage across the capacitor plates varies as
shown in figure. ' 1

c---------, ·'.'
A••----tf o--------< V
C
' j'·

< R

B----~------<
D----------'
Plot the time dependence of voltage across the terminals C and D.
~ ' , !I ,v

,. '' ..
..,,\ .;-, . -

10 210 ~10
I
4/0 *"
'
L-.J
34. A thin uniform wire AB oflength / m, an unknown resistancex and a·resistance of 12 n are connected
a
, by !hick conducting strips, as shown in figure. A battery and galvanometer (with a sliding jockey
connect~d tp it) are also available. Connecti9ns are .tp ,be mafle.,to)lJeasure; the ~nkn<;)\yn resistance~
using the principle of Wheatstone bridge. Answering the following questions : ·
'

~-Z:J
A B '
(a) Are there positive and negative terminais on th~-~alvahometer?
'C

(b) Copy the figure in your answer book and ~how the ba!lery and the galvanometer (with jockey)
D

connected at appropriate points. : - --' _ , '


(c) After appropriate connections are
made, it is found that no deflection takes place in the gal-.·anometer
when the sliding jockey touches the wire at a distance of 60 cm from A. Obtain the value of the
. , · ~; resistance·ofx. ·
r-,--- - - - - - - - - - - - - ~
1
------------------~
DiREcr CCJR&,Nf AND, ELECTRIC C1Rcmts
' ------'-'---
Ans. (a)_ . Ther_e ar~ no, positive a11d negative tenninals on ,the_ galvanometer, q_ecause only zero.deflection-is
·,;_ne:eded.init.,-:,•, :1·1. . ·1· ••1 ,- ·,·· ,,. ·
(b) The battery and galva119meter withjockey.conne,tions-are shown in figure.

Battery

X C 12Q
D

'---I G \---
Galvanometer
(cl sn.
35. A long cylinder with unifonnly charged surface and cross-sectional radius a = 1.0 cm moves with a
constant velocity Vs' IO mis a!ong its axis. An electric field strength at the surface of the cylinder is equal
to E = 0.9:kV/cm. Find the resulting convection current, that is, the current caused by mechanical
transf~r,\)fa charge; ,,
.. , , , -Ans.!=211£oaEv=0.5µA.
'
36. The gap between the plates ofa parallel plate capacitor is filled with glass of resistivity p = I00 GQ.m. The
capacitance of the capacitor equals C= 4.0 nF. Find the leakage current of the capacitor when a voltage
·VC '
V= 2.0 kV is applied to it. Ans. I = - - = l.5µA.
PE£o
37. Two·sources of current of equal emf are connected in series and have different internal resistances R 1
and R2 (R2 > R 1). Find the external resistance R at which the potential difference across the tenninals of
one of the sources (which one in particular?) becomes equal to zero.
_ Ans. R = R2 -R 1, ~V= 0 in the source of current with internal resistance R2•
38. In the circuit shown in figure the emf of the source is equal to~= 5.0 V and the resistances are equal to
R 1 = 4.0 Q and R2 = 6.0 Q. The internal resistance of the source equals R = 0.10 Q. Find the currents
flowing through the resistances R 1 and R2• '

I J , '1 , , +
R' R, ''.,.
,, , ~ - R ' •.

Ans._ Ii

39. Find the potential difference V1 -: V2 between points I ~nd 2 of the circuit shown in figure if R 1 = IO Q,
R2 = 20Q, ~1 = 5.0 V and ~2 = 2.0 V. The internal resistances of_the ~urrent sources are negligible.

2
,,

., ' ' ' ' 1 ,, .~, •


'EIJE€TR1€fliY, We fflGNETIS

40. In the circuit shown in figure the sources have emf's ~ 1 = 1.0 V and~;= 2.5 V and the resistances have
the values R 1 = IO Q and R2 = 20 n. The internal resistances of the sources are negligible. Find a
potential difference VA- V8 between the plates A and B of the capacitor C.

/;I R,
- +

I; I
-
I+ I
C
A B
I;, R,
+

V V - (~1 -~2)R1
A ns. A - B - (R1 + R2) -0.SV.

41. N sources of current with different emf's are connected as shown in figure. The emf's of the sources are
proportional to their internal resistances, i.e. ~ = aR, where a is an assigned constant. The lead wire
resistance is negligible. Find
(a) the current in the circuit;.
(b) the potential difference between points A and B dividing the circuit inn and N - n links.

Ans. (a)/=a; (b) VA-V8 =0.


42. Figure illustrates a potentiometric circuit by means of which we can vary a voltage Vapplied to a certain
device possessing a resistance R. The potentiometer has a length I and a resistance R0, and voltage V0
is applied to its terminals find the voltage V fed to the device as a function of distance x. Analyse
separately the case R >> R0 •

v,

I -- ....
- - - - - - ~0- - -
' X

- _R ..

V0Rx V0x
Ans. V= . x;forR»Ro,V=--.
· Rl+R 0(1-x) 1
1
43. A 6 volt battery of negligible internal resistance is connected across a uniform wire AB oflength I 00 cm.
The positive terminal ofanother battery of emf 4 Vand internal resistance I Q is joined to the point A as
shown in figure. Take the potential at B to be zero.
t._~>.:...r"---·.·~--"'~;r.:...\_·._<_j;__\_•·~-'----· .. E__c_T_C_·._vR_R_E_NT AND,ELECTRlC CIRCUITS
·D_·_lR_ --~

(a) What are the potentials at the points A and C? 6V


~---rl+ '=-~---,
(b) At which pointD of the wire AB, the potential
is equal to the potential at C?
(c) If the points C and D are connected by a wire, A----~-~D-n
____c B
what will be the current through it ?
(d) If the 4 Vbattery is replaced by 7.5 Vbattery,
+ -
what would be the answers of parts (a) and (b)? 41'

Ans. (a)6 V,2 V(b)AD=66.7cm (c)zero (d)6V,-l.5 V,nosuchpointDexists


44. Find the magnitude and direction of the current flowing through the resistance R in the circuit shown in
figure, if the emf's of the sources are equal to s 1= 1.5 Vand s 2 = 3.7 Vand the resistances are equal to
R 1= IO n, R2 = 20 n, R = 5.0 n. The internal resistances of the sources are negligible.
R, I;,
+I-'_"--

Ans. I = -~=-=~-
<R,s2 -Ris,> 0.02A, the current is directed from the left to the right.
(RR, +R1R2 +R2R)
45. Find the emf and the internal resistance of a source which is equivalent to two batteries connected in
parallel whose emf's are equal to s 1 and s 2 and internal resistances to R I and R2.

Ans. s= (s1R2 +s2R1) ,R, =


R1R2
(R1+ R2) (R1 + R2)
46. Find the current flowing through the resistanceR in the circuit shown in figure. The internal resistances
of the batteries are negligible.
R,

so - +
+ I; -
R
Ans./= [s(R2+R3)+soR3]
[R(R2 +R3)+R2R3]
A
47. In the circuit shown in figure the sources have emf's s 1= 1.5 V,
s 2 =2.0 V, s 3 =2.5 V, and the resistances are equal toR 1 = I OQ, R2 s,_+ + s,
= 20 n, R3 = 30 n. The internal resistances of the sources are
negligible. Find:
(a) The current flowing through the resistance R 1; R, +
(b) A potential difference VA - V8 between the points A and B. - s,
B
[R3(s, -s2J+R2<s, +s3)J
Ans. (a) / 1 0.06A, (b) VA-V8 =s 1 -11R1 =0.9V.
(R1R2 + R2R3 + R3R1)
48. Find how the voltage across the capacitor Cvaries with time I in figure, after the shorting of the switch
S,. at the moment I= 0.

R C

Ans. V=.!__~(l-~-21/RC).
' 2
49. Find a potential difference VA - V.8 between the plates ofa capacitor C in the circuit shown in figure, if
the sources have emf's~ 1 =4.0 Vand ~ = 1.0 Vand the resistances are equal toR 1 =IO Q, R2 = 20 Q and
R3 = 30 Q. The internal resistances of the sources are negligible.

R, C
A B

+
~, -

R, R,

~,
'!. ,: - +

~2R3(R1 +R2)-~1R1(R2 +'R3)]


-1.0V.
(R1R2 + R2R3 + RjR1)
50. A constant voltage V= 25 Vis maintained between points A and B of the circuit. Find the magnitude and
direction of the current flowing through the segment CD if the resistances are equal to R 1 = 1.0 Q,
R2 =2.0Q,R 3 =3.0Qand R4 =4.0Q.

C
,1
'\ A B
\ '
D

·Alis. ')=LOA. The curren;t flows from point Cto pointD..


0 CHAPTER
0
0
0
0
STORED CHEMICAL ENERGY

Thef·mol and Chemical


rrl~.[Elfhl:cts of Cu.r:~e~n.t~
4.1 THERMAL EFFECT OF CURRENT: JOULE'S LAW

.4I-----_µ
Consider a part of circuit, canying current i. If dq charge flows between two points differing in potential by Vin
time dt, then work done by the electric field
dW = Vdq
= Vidt
The rate at which energy enters in this part of the circuit
----,II
i...- - - - V

i.e., Power, p = dW =Vi Fig. 4.1


dt
The SI unit of power is J/s. I J/s = 1 W.
The power dissipated in the resistor
If the portion of the circuit is a resistor, the potential difference is given by V = iR and
11 crmal cncrg)"
P
' vi
Vi=i-R= -
R f
Fig. 4.2
The power enters in the resistor appears as thermal energy. On a microscopic scale the transfer of power in the
form of thermal energy can be understood as: Collisions between the electrons and lattice of ions increase the
amplitude of the thermal vibrations of the lattice and this corresponds to increase in temperature of resistor.
This effect is often called Joule heating effect.·

(@§ The thermal energy produced in the resistor generally referred as the heat produced in the
resistor. This energy is not heat as it does not correspond to any temperature difference.
ELECTRICI'IY & Mj\GNETISM

Power output of a source


Consider a source of emf Sand intefOal resistance r, connected to External circuit
an external circuit. The current i flows out from the source. The
power input into the external circuit,
p Vi
We have, V ~ - i r, '------H+'l-)-IW\NVK :)I----'
and hence, p (~- i r) i I; -I~ternal circuit
~i-i2r Fig. 4.3
The product ~ i is the rate at which work is done on the source. The term i2 r is the rate at which energy is
dissipated in the internal resistance of the source. The power output of the source is the power input to the
remaining part of the circuit. Therefore,------- ,....._V___,.
' pout = ~i-i2r
·-- --------~ - -·----- -
External circuit
Power input to a source
Let us consider the source of emf~ and internal resistance r. The
current i is flowing into the source. The power input into the source
P Vi. r
Here we have
Therefore
V = ~+ir
P;. = (~ + i r) i -
s....,..-.....,,....,-.,...,
Internal circuit
--· -- -· ---- ·-i- -
or i_.1'.;~
_=__~ ~: ~- J Fig. 4.4
Loss of power in transmission lines and cables
When electrical power is transmitted from the generator to the point where it is used, there is always a loss of
energy due to the resistance of the transmission wires owing to the Joule heating effect.
The power used in home is measured by the product of current
i and voltage Vbetween the power lines, as they enter the home -
output power PO = Vi. If the total resistance of the wires from
generator to the home is R, then power lost = i2 R. Hence the
input power to the home isP; = Vi+ i2R; and the voltage at ih·e ~ - - ~
generator must be V + i R. The electrical efficiency 1-.----wmr-',,4---j ~ V
P-=
Po '
11 Vi+/ R
P; P0 =Vi

Home
( ~nerator
or 11 = Vi: : 2 R
Fig. 4.5

'I
fi,¢)§ 1
To keep losses minimum, the power is always supplied at high potential, and so to keep)he
current in the power lines as low as possible

( Prequired)
V

and
. ~'.
I
From the above equation, it·is clear that, Pioss cc V .
The supply voltage from the generator may be 33000 V or more.
THERMAL AND CHEMICAL EFFECT OF CURRENT li;ri1
'----'"=---=-.....:----------------------------~
Ex. 1 Consider following circuit, find power generated in the resistor R.

i = 2A i

:; = 1211. r= 2Q

Fig. 4.6
Sol. The power input in the source
si
12X 2=24W
The power dissipated in the source i2R=2 2 X 2
8.W
The power output of the source
P0 "' 24-8=16W
The power input to the resistor = Vi= ;2 R
22 X4=16W
The power generated in resistor= 16 W
Maximum power theorem
Consider a battery of emfs and internal resistance r is used in a
-- R_
·vv..-,,,,
circuit with a variable external resistance R. The value of R for which,
the power consumed in R is maximum;

The current in the resistor R is


_s_
R+r I
The power consumed in R is l
~' r
sR 22
Fig. 4.7
(R+r)

· dP
P to be maximum - = O,
dR

dP p
or
dR p ma., .•••...••....•.

which gives R ,.
and pmax

R=r
or -- -.
s2 Fig. 4.8
4r
' ----- . ------ --· - -- '
ELECTRICI1Y & Mttf,l'fF[!SM.

Kilowatt-hour
It is the commercial unit of electrical energy. It is knows as I unit.
Thus lkW-h IOOOx 3600J
3.6 X 106]

total power (W) x time(h)


Number of units consumed
!000
4.2 ELECTRICAL APPLIANCES

I. F'lament of electrical bulb: It is made of tungsten which has low resistivity and high melting point.
2. Filament of heating devices: It is made of nichrome which has high resistivity and high melting point.
3. Standard resistances (Resistance box) : Standard resistances are made of manganin or constantan.
These materials have moderate resistivity and very low temperature coefficient of resistance.
4. Fuse wire: It is made of tin-lead alloy. It has low resistivity and low melting point.
Rated or design values : .
Some of the values like; wattage, voltage etc. are printed on the electrical appliances are called rated or design
values. These values give the informations about resistance and allowable current etc. For a bulb of I 00 Wand
200 V, 100 W, 220V
Its resistance can be obtained by
-- -, ·2 ··":. - ;
R :;= Vdesign ; 1

.:.... ___ __ !d_c~igl].. ·--

or R ~ Fig. 4.9
Pdesign

where Vdesign and ?design are the de~ign voltage and power of the bulb.

220 2 .
R
100
4840
[Assuming this resistance constant for all values of supply voltage V]

Pdesign .
Allowable current
Vdesign

100 =2._A
220 11

All electrical appliances are design for same voltage; i.e., 220 V.

Power consumed
Power consumed depend on the supply voltage which may be different from rated power.
I. ,
v:z ·
supply
i
\
1 /~onsumed = -R-- ,
L,_____ ·--··-···'.:__J
For Vsupply = Vdesign; Pconsumed -fdesign
C
Let us now consider two bulbs of wattage P 1 and P2 :
ForVsupply =Vd~s1gn
. =V

Resistance of first bulb·

and resistance of second bulb R= 2

I. Bulbs are connected in series: In series both the bulbs have same current.

(ff )consumed
( P2 )consumed.
If P 1 >P2 then R 1 <R2 ,

:. (Pi )consumed < ( P2 )consumed · '-----v·-----'


Thus in series low wattage bulb glows brighter than high wattage bulb. Fig. 4.10
Total power consumed

Pconsumed =
v2

,----
! .
·--- - -
I . I ·,
or = -+-
' -Pconsume·d Pi P2 I
2. Bulbs are connected in paraileC - .. - - - - -- - ..
In parall_el the voltage across them is same.

(ff )consumed
{Pz )consumed

L----v·-------'
Fig. 4.11

:. (~)consumed > (P2 )consumed


Thus in parallel high wattage bulb glows brighter than less wattage bulb.
v2 v2
Total power consumed, Pconsumed

(R~~R;2)
vi v2
where R = - andR = -
1 fJ 2 p2 .
IPC:~- ·= P1 + Pz. i
/

ELECTRICl'JY & MAGNEPISM:.

Ex. 2 Two bulbs rated25 W-200 Vand 100 W-200 Vareconnected'in seriestoa400 Vsupply. Show
with necessary calculations, which bulb if any will fuse? What should happen if the two buibs
were connected in parallel to the same supply?

Sol. Resistances of the bulbs:


~= 2002 =1600Q
fl 25

V2 ,. 200 2 ·
and -=--=400Q
P2 100

Allowable currents:

and , 400V

Fig. 4.12
(i) When they are connected in series, both have same current

v,upply 400 _I_ A


j
R1 +R2 1600+400 5
> ii
< i2
It mean 25 Wbulb will fuse in series.
(ii) When bulbs are connected in parallel, both the bulb will have same voltage. Thus
'. l-j <
V,upply
;,' ;, R,
R1

400 = _I_A R,
i2
1600 4
> i,

fsupply
400V
and i2 Fig. 4.13
R2

400 = IA
400
> i2
The currents in both the bulbs are greater than their allowable currents. Therefore both bulbs
will fuse in parallel. , ·

Ex. 3 An electric kettle bas two heating elements. One brings itto boil water in time t1 and the other in
timet2• Find thetimettaken by water to boll in kettle when· ·
(i) heating filaments are connected in.series.
(ii) heating filaments are connected in paralle!.
Sol. Let His the heat required to boil the water, then for first coil
v2 v2,,
H -1, => R,
R1 H
v2,2
and for second coil R2
H
(i) When both the filaments are connected in series
v2 v2
H = --/=---/
R,otal R1 + R2

v2
v 21
v2, )'
--'+--2
( H H
--· ----------- -~ ---
-= ___I 1_.+__I2 _____ );
:' - _ t ---
(ii) When both the filaments are connected in parallel
vi
H = --/
_Rtotal

After substituting values of R I and R2, we get

!-! = _!_+2-. I
/
l---~-----~·---'
l1 12 I

Ex. 4 A cell of emf~ and internal resistance r supplies currents for the same time t through external
resistance R 1 and R2 separately. If the heat developed in the external resistance in the two cases
is the sa~e, then what should be the internal resistance?

Sol. In first case

and heat produced in R 1 H, it R1t =(-~-r


R1+r
R1I

In second case i2
-~-
R2+r

and heat produced in R2 H2 i}R2 t=(-~-r


R2 +r
R2I

For the condition given, H, H2

or (-~r
R1+r
R1t R2 +r
~
( - , rR2t

After solving , we get r = JR1R2 . Ans.


Fuse : It is used in series with.the circuit to prevent the electrical appliances from bumini by melting itself
to open the circuit.

; fuse

Fig. 4.14 Fig. 4.15


Let R is the resistance of the fuse wire, then

The heat produced in fuse wire H =

lfh is the heat loss per unit surface area offuse wire, then heat radiated per second
= h x21ti-e
;zpe
At thermal equilibrium h X 21trf.
,r,2

or h
..l£___
2,r2r3
Ac~ording io Newton's law of cooling
h = C.o.0 [where .<10 is the tempe_rature of fuse wire above
surrounding, and C is a constant]

C.<10

or f~e -_-··::~z;2p : : J
: . ~ ~~1t2r3C . j
The above expression is free from length· of the fuse wire"° Hence the function of fuse is independent ofits length
provi(Jed i remains constant. '
For given material of fuse wire
,-,~2--.· ,3 ~;
L_L__~-h...:,~;.J
An important explanation
In houses all electrical appliances are connected in parallel with the supply line. Hence each of them has same
potential. But because ofresistance of current carrying wires, the potential across each of them is different. It
is common experience that when any high voltage appliance is switched on, the other get affected (bulb glows
less brighter). It is due to current carrying cables. To understand more.clearly consider a simple circuit having
a bulb and a heater connected in paralled with a source.
When only bulb _is switched on
50 ~i
200 l.s2
50+ 100
4A
3 j
v 200 v~'"
C::s1
Heater
The power consumed by bulb IOOQ
Bulb IOO.Q

Fig. 4.16
2
P i2R= ( 4) xl00=-
1600-W=177.8W
3 9

Now heater is also switched on, then net current supply

200
2A
0+ !OOxlOO)
( 5 100+100

The current flows in the bulb= IA


The power consumed by the bulb P' PR
I 2 x IOO=IOOW
It is clear from above calculations that when heater in parallel is switched on, bulb consumes less power, and
therefore becomes dim. If the resistance ofcurrent carrying wire becomes zero, there will no such effect occur.

Ex. 5 What amount of heat will be generated in a coil ofresistance R due to a charge q passing through
it if the current in the coil
(a) decreases down to zero uniformly during a time interval l!.t.
(b) decreases down to zero halving its value every l!.t seconds?

Sol. (a) As the current decreases uniformly, its value at any time/, assuming initial current i0

Thus q J J(i--
0
idt = io
0
1
/!,./
)dt

0 I l!J
io \1 _ l._11>.1 =_io_l!.t_ Fig. 4.17
2/!,.t O 2

2q
or
1!,.t

Hence
The heat generated H

Ans.
(b) According to given condition

= io(½f M
When current decreases according to above equation it .becomes zero at I~ = . Therefore
charge will flow till infinite time.

The charge q = f idt= ~f; 0 ('-)1/M


~
· dt
2
0 0
~

;0 fr 'Md1 ·
1
... (i)
0

Substitutin.g z-t/M = z
-I
or -ln2 = lnz
l!.t
Differentiatinirabove·equation, we h'ave 1

:[~·1~·2] =
dlnz·
dt .
-ln2 I dz
or
Ill z dt

or

-Substituting these values in equation (i), we get

q
.~f zx (-t:.t)dz
10 -·- -
ln2 z
0

io(-!it)jzl;.
ln2

= . io(-t:.t)lr''Mi~
ln2 D

iol!.t
ln2
C
6
Thus we have 'o. _- -
qln2
- an d.'= --
qln2(1)'' '
-
l!.t l!.t 2
The heat generated in the time interval Oto =
=
H = Ji2Rdt
0

2
q x2R [ -Zri/JJ]=
2!J.t 2 0

q2R Ans.
l!.t

Ex. 6 A conductor has a temperature independent resistance and a total heat capacity C.Atthe moment
t =OIs connected to a de voltage V. Find the time dependance of the conductor's temperature T
assuming that thermal power dissipated into surrounding space to vary as q = k(T -T0 ), where
k is a constant, T0 is the environmenttemperature (equal to the conductor's temperature at the
initial moment).
Sol. The rate of heat evolved by the resistance
= rate of absorption of heat+ power dissipated into surrounding
vz CdT
or -+k(T-To)
R dt

vz CdT
or --k(T-To)
R dt
I
dt .
or JC
0

In[~-k(T-To)] T

-k ' C

kt
C

In
v: -k(T-To)l
= kt
[ V
R
2
C
., (n[l kt
( 2k )(T-To)] --
V /R C
k(T-T0 ) e-kt/C
or
(V 2 IR)

y2
or k(T-T0 ) -(t-e-kt!C)
.R

or T . To +!'.'.:..(1-e-kttc). Ans.-
. kR

4.3 SEEBECK EFFECT

The conversion of electrical energy from thermal energy was discovered by Seebeck in 1826. According to him
if the junctions of two different metals are kept at different temperatures, then there is an electric current in the
circuit. This effect is called Seebeck effect. The emf produced across the junctions is called thermo-emf and the
resulting current is called thermo-electric current, the pair of metals is called a thermo-couple.
Thermoelectric series
Seebeck arranged different metals in a specific order which gives the thermo-electric series, and showed that
when any two metals of the series are used with junctions at different temperatures, current flows from the metal
earlier in the series to the metal later in the series through the cold junction. Also, the series gives an idea of the
of
relative magnitud~ emf for different thermocouples. Farther apart two rrietals lie in the series, larger is the emf
produced. The thermo-electric series is as : . _\ · .
Animony, Fe, ~d, Zn, Ag, Au, Rb, Mo, Cr, Sn, Pb, Hg, Mn, Cu, -Pt, Ni, co1stantan, bismuth. _
cu· \.. · .,

i--

-i
e
Fig. 4-18 Tlzermoco11ple made of Cu-Fe_
Take as an example, a thermocouple madeup of copper and iron. The current will be from iron to·copper at the
coldjunction. The largest emf will generate in a thermocouple made of antimony and bismuth.
Variation of emf with temperature
Suppose a thermocouple is made up of two metals A and B, and the thermo-emf produced is ~AB· Figure shows
graphically the variation in thermo-emf as the temperature of the hot junction changes.

Smax ··-·············-_.---~,-.-

C n
,,
Fig. 4.19
If le, In and 1, denote the temperature of the cold junction, the neutral temperature and the inversion temperature
respectively, then
,,, - tc = /.-/
' n

le +I;
or. In = ... (I)
2

If the cold junction is kept in ice (0°C) and the hot junction at 1(0 C), then the thermo-emf depends on the
temperature as

~AB ... (2)

where a AB and bAB are constants for a pair of metals A and B. It should be remembered that the parabolic
variation is obtained in the case when either of a and b _is negative.
Neutral and inversion temperature
The temperature of the hot junction at which the thermo-emfis maximum is called the neutral temperature and
the temperature at which the thermo-emf changes in sign is called the inversion temperature.

Thus for ~AB to be maximum,

. d~
0
dt

or :J at+ b~
2
) = o

or a+ bt = 0

t = I = _!!_:_ ... (3)


n . b
t_ -- -· -- - -- - .•

As both a and b are constants for a thermo-couple and so neutral temperature is constant for any thermo-

d~
couple. The quantity -d is called thermoelectric power or seebeck coefficient.
t .

The maximum value ·or thermo-emf

(-a) +2 (-a)
2
a - -- b
~max
b b

- ----2
. or ~max
= a ... (4)
b
Coefficients a and b for thermocouple
2
M<tal ,>ith Pb a(J1Vf'C) b (J1Vf'C )

Al --0.47 0.003
Bi -43.7 --0.47
Cu 2.76 0.012
Au· 2.9 0.0093
Fe 16.6 --0.030
Ni 19.1 --0.030
Pt -1.79 --0.035
Ag· 2.5 0.012
Steel 10.8 ' --0.016

Ex. 7 The cold junction ofa thermocouple is maintained at 20°CNo therm~mfis developed when the
hot junction is at S40°C. Find the neutral temperature.
Sol. Given , = 20°c ,. = 540°c
C ' I •

le +t;
'· 2
20+540
280°C. Ans.
2

.
The expression for thermo emf in a thermocouple is given by the relation ~ = 408 __ ,where 8
~
E X. 8
· 20
is the temperature difference of two junctions. What is the neutral temperature?
iJ2
Sol. Given ~ - 408--
20
For maximum value of~ ,
d~
0
dB

or ·02)
-d( 408--. 0
dB 20

or 40- 28 0
20
Which on solving gives
8 400°C
Thus neutral temperature is 400°C. Ans.
.. •..... THERMAL AND CHEMICAL EFFECT OF CURREN'!_M
Law of intermediate metal
'
Suppose ~AB, ~AC and ~BC are the thermo emfs across the thermo-couples made of A, B; A, C; and B, C
respectively. If the temperature difference across the junctions of all the thermocouples are the same, then

' ~AC ... (5)

Law of intermediate temperature

Let [~AB];'. denote the thermo-emf of a thermocouple made ofmetals A and B when the tem~eratures of the

junctions are t 1 and ½· Then

This is known as the law of intermediate metals.

Ex. 9 Find the emfofa Cu - Fe thermocouple when temperature of hot junction is 100°C and that of cold
junction is 0°C.
Sol. We know that

2
bt
~ = at+-
2
By law ofintermediate metals, we have

~Cu.Fe = ~Cu.Pb + ~Pb.Fe = ~Cu.Pb - ~Fe.Pb

(2.76-16.65) X 100+ ½(0.012 +0.030) X 100


2

-l.179mV Ans.

4.4 PELTIER EFFECT


In I 834, Peltier discovered that if a current is allowed to pass through the junction of two different metals, heat
is either evolved or absorbed at"the junctions.That means the junction is either heated or cooled. This effect is
known as Peltier effect and the heat evolved or absorbe~_is knwon as Peltier heat. Experiments has shown that
the peltier heat transferred at any junction is proportional to the amount of current crossing the junction and
changes its sign when reversing the direction of current. Thus Peltier effect is reversible. If t,,H is the amount of
heat evolved or absorbed when a charge t,,Q is passed through the junction, then we define Peltier coefficient
(Peltier emf) as:
-----'-'-~"-:.1'lil-'-:::;.;t""'q!""E-'-~-m_1_c_i1Y_·
.. _"r_&:_\;_MA'-'ill!'-.G""'•'N_m_'_s~M_···_._ _.,,_,_Y_D_'_________ --'-:=~]
Peltier heat- . ·: -~H 1
II
Charge transferred . t.Q '
,,
Cu
;-

+---i
Fe
Fig. 4.20 Peltier effect
4.5 THOMSON EFFECT
If a metal has a nonuniform temperature and a current is passed through it, heat is absorbed or evolved in
different sections of the metal. This heat is over and above the Joule's heat. This effect is called Thomson effect.
!fa charge t.Q is passed through a smali'section of the metal wire having temperature difference t.t between
·~- ..... ·- .
• /11{
the ends, the Thomson coefficient'· ·· cr = --·. 't ' ' -
t.Qt.t " l ;

Explanation of Seebeck, Reltier.and .. Tliomson effects . , ....


We know that the density of free elec_trons is different in different metals at same temperature. Thus when two
different metals are joined together, the electrons tend to diffuse from the side with higher concentration to the
side with lower concentration. Also the density offree electrons varies inside the same metal, if temperature of
metal is not uniform everywhere. Thus electrons tend to diffuse from the higher concentration to the lower
concentration region. So if the junctions are kept at different temperatures, an• emf. is developed across the
junctions.
,or, ••.•••••••• ·•.•.•,,,
t0 li •- • • • • • -:
(a)
• iR I 1°CCI-=·-=·-=·:I•:I•:I•::::.:,
(b)
··==· ~uc jf

Fig. 4.21 (a) Two metals at same temperature Fig. 4.21 (b) A metals with different temperature
ii
cr_,(Af)-

''
It
i_
---1p--
Fig. 4.22 , • _. , . . .· -. ; ·.
It can be easily understand that at the cold junction the electron density is more than the electron density at the
' j • • ''

hot junction, It will cau;e th!' p;cJ.c~etwee,n thejun~tions .. Th~s, if [h AB J,, ,, [9AB
' '

t ' •

be the emfs at the cqld_and

hotjun~tions r~spectively, !hep the~o,;e~fwill be [IIAB] = [IIAB ],2 -[IIAB ),, .. , ... ,,
It is clear from the above explanation that Seebeck emfis a combination of Peltier emf and Thomson emf. Thus
•. ; ~
~ (I1'.4n], -[IIAii]; +'t.t(aA-aif). '
!·1tr.1 1
~AB
I ,'•
I - - - -~'-'-- , I ·. ' J
'------------·_:·_T_H_ER_MAL
__AN_._v_.c_;.·_H_EM_'·~1c.:..··1'L_··_,£_F_F_E_c_r_o_F_C_·u_R_RENT
___ ag
Difference between Peltier effect and Joule's effect
Peltier effect Joule's effect
I. It is a reversible effect. I. It is an irreversible effect.
2. It takes place at the junctions. 2. It takes place across the entire metal.

3. It may be heating or a cooling effect. 3. It always be a heating effect.


4. Peltier heat is proportional to the current: 4. Joule heat is propoi:ional to the square of the current:

-2
H Peltier oc i· H Joule oe 1

4.6 CHEMICAL EFFECT OF CURRENT


Electrolyte : The liquid which dissociates into its ions on passing current through it is called electrolyte.
Electrolysis : The process of decomposition of electrolyte solution into ions on passing current through it
is called electrolysis.
Electroplating : The article to be electroplated is made as the cathode and the metal to be deposited is made
as the.anode. A solution of the metal is taken as the electrolyte. If gold is to be coated then auric chloride is used
as electrolyte. In case of ornament of silver, electroplating is to be done by AgNO3. The Ag+ ions move to the
cathode and receives an electron from the cathode to become neutral Ag atom and deposited on the cathode.
Ag++e- = Ag.
~---<1[

Ag
Ornament
N03
Ag

AgN0 3

Fig. 4.23

The NO3- ion moves to the anode and gives its extra electrons to it. The NO3-, ion is converted to NO3,
''. NQ3- = '
N0 3 +e.
The NO3 so formed reacts with a silver atom of the anode to form AgNO3 which gets dissolved in the solution.
This way, silver anode slowly dissolved in the solution. Thus, silver anode slowly dissolves and deposited on
the cathode with the concentration of the electrolyte remaining unchanged.
4. 7 FARADAY'S LAW OF ELECTROLYSIS
First law : The- mass of a substance lib,erate~. at an electrode is proportional to the amount of the charge
passing through the electrolyte. Thus
:_,'! _=_ __zQ
If an electric current.of constant magnitude i is passed through an electrolyte for a time t, then
Q = iI
!-;;; ;,,- ~it!'
·where z is a constant called electrochemical eq-;;ivalent(ECE) of the substance. The SI unit ofECE is kg/C.
Second law : The mass of a substance liberated at an ele~trolyte by a given amount of charge is proportional
to the chemical equivalent of.the substance. The chemical equivalent of a substance is

Atomic mass
W=
vaiency' "

If Wi and W2 are the chemical equivalent of two substances, the9 from F~raday is ll law .

. . . (i)

Also from I law

~ ..:1... ... (ii)


m2 z2

From (i) and (ii)

Wi .:L
W2 Z2

Wi w:
or - 2 = constant (F)
ZJ Z2

or
·w = F 1
___ :_ ---- ~--f
,, Fis the proportionality constant called Faraday's constant. IF = 96500 C/eq.

Ex. 10 A brass plate having surface area 200 cm2 on one side is electroplated with 0.10 mm thick silver
layers on both sides using a 15A current Find the time taken to do the job. The specific gravity of
silver is 10.5 and its atomic weight is 107.9 g/mol.
Sol. The volume of silver deposited
V 2 x vol. of silver deposited on one side
2(200 x I o-4 x 0. I x 10-3) m3
4xJ0-6m3
The mass of silver needed m pV= 10.5 x 103 x 4 x I 0-6
42 X J0·3 kg=42g
L _____ THERMAL AND CHEMICAL EFFECT OF CURRENT

Chemical equivalent of silver


107
w -]- = 107g

W 107
z = Ji= 96500
If I is the required time, then by Faraday I law
zit m

m 42 42x96500
I - - 107x15
zi __!_Q2__ X I 5
96500
42minute A11s.

Ex. 11 A current oflA is passed through a dilute solution of sulphuric acid for some time to liberate lg
ofoxygen, How much hydrogen is liberated during this period? How long was the current passed?
Faraday constant= 96500 C/mole,
Sol. Chemical equivalent of oxygen

Atomic mass 16
w valency
=-=8
2 ·

Chemical equivalent of hydrogen


By Faraday II law

mo Wo 8
--=-
mH

m I
or - 0= - g
8 8

I
We have lg ofoxygen =
8 gram-equivalent of hydrogen
I
We know that for liberation of I gram - equivalent weight, it requires 96500C of charge. Thus for
8
. . . 96500 4
gram-equtvalent, 11 requtres a charge = - - = l.12XIO C.
8
If/ is the required time, then

Q l.12xl04
I =
I
J.2x]0 4 s
3 hour20min A11s.
MCQ Typ_e_l &xet:d4e 4.1 ·
I. Which of the following plots may represent the
thermal energy produced in a resistor in a given
time as a function of the ,Iectric current? ·
IQ
IQ 3V
IQ 3V
IQ
Q

(a) I (b) II
(c) Ill (d) N
2. A constant current i is passed through a resistor.
Taking the temperature coefficient ofresistance 3V
into account, indicate which of the plots shown IQ IQ
in figure best represents the power P generated
]Q
in the resistor

I' IV
III (a) P 1 >_P2 > P3 (b) P 1 > P3 > P2
II
(c) P 2 > P 1 > P3 (d) P3 > P2 > P 1

I .
6. C_onsider the following two statements.
I. Free-electron density is different in different
(a) . I (b) II. metals.
(c) Ill (d) N 2 Free-electron density in a metal depends
3. Two resistors R and 2R are connected in series on temperature.
in the circuit. The thermal energy developed in R Seebeck effect is caused
and 2R are in the ratio
(a) due to both statements I and 2
(a) I :2 (b) 2: I
(b) due to I but not due to 2
•I
(c) I :4 (d) 4: I
(c) due to 2 but not due to I
4. Two resistors R and 2R are connected in parallel
(d) neither due to I nor due to 2.
in the circuit. The thermal energy developed inR
and 2R are in the ratio 7. For a copper-iron thermocouple the values of
(I) I :2 (b) 2: I the various temperatures are given below :
T0 = 0°C, TN = 275°C and Ti= 550°C. If T0 is
(c) I :4 (d) 4: I
changed to 10°C, the new value ofTNand T; will
5. Figure shows three resistor configuration R 1, R2
be respectively
and R3 connected to 3 V battery. If the power
(a) 275°Cand 560°C (b) 275°Cand 540°C
dissipated by the configuration R 1, R2 and R3 is
P 1, P 2 and P3 respectively, then (c) 285°Cand 540°C (d) 285°Cand 560°C
THERMAL AND CHEMICAL EFFECT OF CURRENT

8. Theemfofa thermocouple, one junction of which 14. Two heater wires of equal length are first
is at 0°C is given by connected in series and then in parallel. The ratio
E=at+bt2. of heat produced in the two cases is
The Peltier coefficient is given by (Tis in kelvin). (a) 2: I (b) I: 2
(a) T(2 at+ b) (b) T(2a + bt) (c) 4: I (d) I :4
(c) T(a + bt) (d) T(a+2bt) 15. A battery ofe:m.f. 10 Vand internal resistance
9. The emf of a thermocouple, with one junction at 0.5 ohm is connected across a variable resistance
0°C and the other at a temperature, t°C is given R. The value of R for which the power delivered
by in it is maximum is given by

e = [ 501 -12000( I - e-0.oo 2r) Jµv. (a) 2.0ohm


(c) I.0ohm
(b)
(d)
0.25ohm
0.5 ohm
The Peltier coefficient at 0°C will be 16. An electric bulb is designed to draw power POat
(a) 0.71 mV (b) 7.lmV voltage V0 . If the voltage is Vit draws a power P.
(c) 710mV Then
(d) 71 mV
I 0. Forty electric bulbs are connected in series across
a 220 V supply. After one bulb is fused, the
remaining 39 are connected again in series across
the same supply. The illumination will be
(a) more with 40 bulbs than with 39
(b) more with 39 bulbs than with 40
(c) equal in both the cases
17. Find the power of the circuit
(d) in the ratio of 49 2 : 39 2
II. A heater coil is cut into two parts of equal length
and one of them is used in the heater. The ratio
of the heat produced by this half coil to that by
the original coil is
2Q 2Q
(a) 2: I (b) I: 2
(c) I: 4 (d) 4: I
12. What is immaterial for an electric fuse wire?
21'
(a) Its specific resistance
(b) Its radius
(a) 1.5 W (b) 2W
(c)._ Its length
(c) IW (d) None of these
(d) Current flowing through it
18. To deposit one gm equivalent of an element at
13. An electric bulb is rated 220 volt and I 00 watt.
an electrode, the' quantity of electricity needed
Power consumed by it when operated on I I 0
is
volt is
(a) One ampere (b) 96000 ampere
(a) 50 watt (b) 75 watt '
(C) 96500 farad (d) 96500 coulomb
(c) 90 watt (d) 25 watt
19. In an electrolysis experiment, a current i passes 24. In Seebeck series Sb appears before Bi. In a
through two different cells in series, one Sb-Bi thermocouple current flows from
containing a solution ofCuSO4 and the other a
(a) Sb to Bi at the hot junction
solution of AgNO3 . The rate of increase of the
weight of the cathodes in the two cells will be (b) Sb to Bi at the cold junction
(a) in the ratio of the densities of Cu and Ag (c) Bi to Sb at the cold junction
(b) in the ratio of the atomic weight of Cu and (d) none of the above
Ag
25. Thomson coefficient ofa conductor is l0µV/K.
(c) in the ratio of half the atomic weight of Cu The two ends of it are kept at 50°8 and 60°C
to the atomic weight of Ag
respectively. Amount of heat absorbed by the
(d) in the ratio of half the atomic weight of Cu conductor when a charge of I 0C flows through
to half the atomic weight of Ag
it is
20. Faraday's laws of electrolysis are related to
(a) lOO0J (b) lO0J
(a) the atomic number of positive ion
(b) the equivalent weight of electrolyte (c) IOOmJ (d) I mJ
(c) the atomic number of negative ion 26. In the circuit shown in figure, the heat produced
(d) the velocity of positive ion in 5 ohm resistance is IO calories per second.
21. The negative Zn pole ofa Daniel cell, sending a The heat produced in 4 Q resistance is
constant current through a circuit, decreases in
mass by 0.13 g in 30 minutes. If the
electrochemical equivalent ofZn and Cu are 32.5
and 31.5 respectively, the increase in the mass of
the positive Cu pole in this time is
(a) 0.242g (b) 0.190g (a) I cal / sec (b) 2 cal I sec
(c) 0.141 g (d) 0.126g (c) 3 cal/ sec (d) 4 cal I sec
22. The electrochemical equivalent of a metal is
27. A 100 W bulb B 1, and two 60 W bulbs B2 and
3.3x 10-7 kg/ coulomb. The mass of the metal
liberated at the cathode when a 3 A current is B3 , are connected to a ~50 V source, as shown in
passed for 2 second will be /he figure. Now W1, W2 and W3 are the output
(a) 19.8xI0-7 kg (b) 9.39xl0-7 kg powers of the bulbs B 1, B2 and B3, respectively.
Then
(c) 6.6x 10-7 kg (d) I.lxl0-7 kg
23. Two voltameters, one of copper and another of
silver, are joined in parallel. When a total charge
q flows through the voltameters, equal amount
of metals are deposited. If the electrochemical
_equivalents of copper and silver are z I and z2
respectively the charge which flows through the
silver voltameter is

ZJ
(a) q- (b) qZz-
Zz ZJ 250V

_ q_ _q_
(c) (d) (a) Wj > W2 = W3 (b) Wj > W2 > W3
1+-ZJ !+ Zz
Zz ZJ (b) Wj < W2 = W3 (d) ff\ < W2 < W3
,___ _ _ _ _ _ _ _ _ _ _r,_H_E_RMAL
_ _AN_D_C_·_H_E_M_lC_AL EFFECT OF CURRENT --;Im!
28. Tlie three resistance of equal value are arranged (a) 100°c
in the different combinations shown below. (b) 350°c
Arrange them in increasing order of power (c) 1400°C
dissipation
(d) no neutral temperature is possible for this
(!) ~
thermocouple
32. Water of volume 2 litre in a container is heated
with a coil of I kW at 27 °C. The lid of the
container is open and energy dissipates at rate
(II) of 160 J/s. In how much time temperature will rise
from 27°C to 77°C [Given specific heat of water
is 4.2 kJ/kg]
(a) 8min20s (b) 6min2s
(c) 7min (d) 14min
33. Two different metals are joined end to end. One
(Ill) -+--'MMr--+- end is kept at constant temperature and the other
end is heated to a very high temperature. The
graph depicting the thermo e.m.f. is

(IV)
(a)

(a) Ill<II<IV<I (bl II <III <IV<!


(c) I<IV<Ill<Il (d) I<III<ll<IV
29. Silver and copper voltameter are connected in
parallel with a battery of e.m.f. 12 V. In 30 minutes,
I gm of silver and 1.8 gm of copper are liberated.
The power supplied by the battery is (b)
(a) 24.13 J/s (b) 2.413 J/s
I
(b) 0.2413 J/s (d) 2413 J/s
(Zcu=6.6xio-4gm/CandZAg= 11.2 x Io-4gm/C) i;
30. The thermo e.m.f. ofa thermo-couple is 25µVi°C
at room temperature. Agalvanometerof 40 ohm
resistance, capable of detecting current as low
as I o-5 A, is connected with the thermocouple.
The smallest temperature difference that can be
(c)
)
detected by this system is
(a) 20°c (b) 16°C
. (c) 12°C (d) goc
31. The thermo e.m.f. of a thermocouple varies
with the temperature 0 of the hot junction as
E =.a0 + b02 in volt where the ratio alb is 700°C.
(d) (\
· If the cold junction is kept at 0°C, then the neutral
temperature is
34. A constant current i is passed through a resistor. 36. · The charge flowing through a resistor R varies
Taking the temperature coefficient of resistance as Q (t) =at- pt2. The total heat produced is R
into account, indicate which of the plots shown is :
in figure best represents the rate of production
of thermal energy in the resistor a3 R
(a) (b)
2Jl

a3R
(c) (d)
3Jl

(a) a (b) b
(c) C (d) d
35. A I 000 Wheating unit is designed to operate on
a 120 Vline. The line voltage drops to 110 V. The
percentage of heat output drops by :
(a) 9% (b) 27%
(c) 16% (d) 30%
.___ _ _ _ _ _ _ _ _ _ _T_n_E_RMAL
__ AND __E_FFE
__c_n_EM_ICAL __CT_D_F_C_u_R_RENT
___ M
& ~ 4.2
1. The constants a and b for the pair silver-lead are 5. Total energy supplied by the battery is :
2.50 µV/C and 0.012 µV/C 2 respectively. For a
silver-lead thermocouple with colder ju~ction at (a) .!..cv2 (b) cv2
2
O'C, [more than one option may be correct]
(a) there will be no neutral temperature (c) 2CV2 (d) zero
(b) there will be no inversion temperature 6. The instantaneous power dissipated in the
(c) there will not be any thermo-emfeven if the resistor is PR. The total energy dissipated in the
junctions are kept at different temperatures resistor is
(d) there will be no current in the thermocouple
even if the functions are kept at different (a) · .!..cv 2 (bl cv2
2
temperatures.
(c) 2CV2 (d) None of them
Passage for questions 2 to 4 7. Maximum energy stored in the capacitor
The internal resistance ofa dry cell increases gradually I 2
with age, even though the cell is not used. The emf, (a) 2 CV (b) CV2 (c) 2CV2 (d) Infinite.
however, remains fairly constant at about 1.5 V. Dry
cells may be tested for age at the time of purchase by
8. Match the List I with the List II from the
combination shown. In the left side (List I) there
connecting an ammeter directly across-the terminals
are four different conditions and in the right side
of the cell and reading the current. The resistance of (List II), there are ratios of heat produced in each
the ammeter is so small that the cell is practically short resistance of each" condition :
circuited.
2. The short circuit current of the cell is about 30 A. List I List II
(I) Two wires of same resistance (A) I: 2
The internal resistance of the cell approximately
are connected in series and
is :
same current is passed
(a) 0.01 Q (b) 0.02 Q through them
(II) Two wires ofresistance Rand (8)4: I
(c) 0.05 Q (d) 0.IQ
2R ohm are connected in
3. The power generated by the internal resistance series and same p.d. is applied
is: across them
(a) 45 W (b) 60 W (Ill) Two wires of same resistance (C) I: I
(c) 90 W (d) 100 W are connected in parallel and
4. The power generated in the circuit is : same current is flowing
through them
(a) 45 W
(b) 60 W (IV) Two wires of resistances in (D)2: I
(c) littlegreaterthan45 W the ratio I : 2 are connected in
parallel and same p.d. is
(d) little greater than 60 W
applied across them

Passage for questions 5 to 7 (a) I-B; II-A; III-C; IV-D


The current in a charging capacitor(C) is given (b) I-C; II-D; m-e; Iv.:.n

by i = ioe-11 RC, where io = !. For instantaneou.s


(c) I-B; 11-D;.III-A; IV-C
(d) I-A; Il-B; Ill-D; IV-C
potential V of the capacitor, power is given by P = Vi.
& ~ 4.3
- --- - - - -- - · .. ----- - - · -- -· --- --------- -- ·-- --- ·--- ---- - - - ---- ---- - --- • -- -- --·1
' Read the two stateme~ts carefully' to mark the correct option out of the options given b-elow. Select the right i
1
choice. ' ' ,
(a) If both the stat~ments are true and the Statement- 2 is the correct expl~nation of Statement- I."
(b) If both the statements are true but Statement - 2 is not the correct explanatio~·ofthe Statement- 1.
! (c) If Statement- l true butStatement-2 is false. · -
, (d) If St;,tement - 1 is false but Stateme11t - 2 is true.
- - - - -- _J

I. Statement l 7. Statement l
lfa constant potential difference is applied across Neutral temperature of any thermocouple is a
- a bulb, the current slightly decreases as time
basic characteristic.
passes and then becomes constant.
Statement2 Statement2
The resi'stance of the metal increases with All the thermocouples have a neutral
temperature. temperature.
2 Statement 1 8. Statement 1
The 200 W bulb glows more brightly than I 00 W
bulb. The neutral temperature does not depend on the
Statement2 temperature of the cold junction.
The resistance of200 W bulb is less than the I 00 Statement2
Wbulb. The inver~ion ten_:iperature does not depend on
3. Statement l the temperature of the cold junction.
Fuse wire must, have high resistance and low
melting point. 9. Statement 1
Statement2 The possibility ofan electric bulb fusing is higher
Fuse is used for small current only. at the time of switching ON and OFF
4. Statement 1 Statement2
- A domestic electrical appliance, working on a
three pin will continue working even if the top Inductive effects produce a surge at the,time of
pin is removed. switch ON and OFF
Statement? 10. Statement l
The third pin is used only as a safety device. In the given circuit iflamp B or C fuses then light
5. Statement 1 emitted by lamp A decreases.
A laser beam 0.2 W power can drill holes through
a metal sheet, whereas I 000 W torchlight cannot.
Statement2 Ai·
The frequency of laser light is much higher than
that of torch light
6. Statement l V
Electric appliances with metallic body; e.g.
heaters, presses etc. have three pin connections,
whereas an electric bulb has a two pin connection.
Statement2 Statement2
Three pin connections reduce heating of Voltage on A decreases.
connecting cables.
.. -
Ima
~
, ",'._t;:,'."j"
' THERMAL AND .CHEMICAL EFFECT .oF::C:.uRRENT
L_

E'ee,ity,·
---- \x.· . MCQType1
- -----
- --
-
--
/ / ~ & ~ 4.1

.I 1 I (a) [ 7 I (b} I 13 I (d> I 19 I (c> I 25 I (d) I 31 I (d)


I 2 I (c> I s [ (d} I 14 I (d> 1 20 I (b> I 26 I (b} I 32 !I (a)

r 3 I (a> I 9 I (bl 1 15 l'dJ r 21 I (d} I 21 r- (d) I 33 pct)


I 4 I (b) I 10 I (b> I 16 I (b) I 22 I (a> I 2s I (a> l 34 [(cif
I 5 I (c> ! 11 [ (a) [ 17 [ (c) \ 23 [ (d) I 29 I (a> I 35 ~
I
I
6 I (a) I 12 I (c) 118 I (d) I 24 I (b) i 30 I (b) r-~fTci>

-J~~il' MCQ~!IP~-2__ - - / I ~ & ~


..__,-
4.2

1 [(a,b) 3 (a) 5 (b) 7 (a)


-2-[ (c) 4 (c) 6 (a) 8 (b)

_.,;;;;;-~iiy("
\. _ / '
MCQType 3
--- _ -- -
/ / ~ & ~ 4.3

[ 1 (a) [ 3 (c) f 5 (c) [ 7 (cl I 9 (a)

12 ~14 ~n (c)jB (c)!10 (a)

'
& ~ 4.4
1. An imn:,ersion heater rated I 000 W, 220 Vis used to heat 0.0 I m3 of water. Assuming that the power is
supplied at 220 V and 60 % of the power supplied is used to heat the water, how long will it take to
increase the temperature of the water from J5°Cto 40°C?
Ans.29min.
2. Four resistances crrying a current as shown in the diagram are immersed in a box containing ice at 0°C.
How much ice must be put in the box every IO minute to keep the average quantity of ice in the box
constant? Latent heat of .ice= 80 cal/g.

IOQ 5Q
JOA

5Q IOQ

Ans. I 190 g.
3. In circuit shown in figure the heat produced in 5 Q resistor due to current flowing through it is IO calorie
per second.The heat generated in 4 Q resistor is :

4Q 6Q

5Q

Ans. 2 calls.
4. The 2.0 Q resistor shown in figure is dipped into a calorimeter containing water. The heat capacity of the
calorimeter together with water is 2000 J/K.
(a) If the circuit is active foil 5 minutes, What would be the rise in the temperature of the water?
(b) Suppose the 6.0 Q resistor gets burnt. What would be the rise in the temperature of the water in the
next 15 minutes?

6V1 1 _ _!fl.,.
+1 .l-_---'11'1/\/\r--,

6Q
AAA•
. .

::; ;:;:,;:;::::2U_ .............


::::::::::::::....... 7. ::::::::::::.

Ans. (a) 2.9°C (b)3.6°C.


5. A battery of internal resistance 4Q is connected to the network of resistances as shown in figure. What
must be the value of R so that maximum power is delivered to the network ? What is the maximum
power?

R 6R R

+ E,-

Ans. 20,1;,2/16.
6. An electric kettle has two coils. When one coil is switched on, it takes 15 minute to boil water and when
the second coil is switched on, it takes 30 minute. How long will it take to boil water and when both the
coils are used in (i) series and (ii) parallel?
Ans. (i) 45 min. (ii) IO min.
7. A heater is designed to operate with a power of I 000 Win I 00 V line. It is connected to two resistances
of IO Q and Ras shown in figure. If the heater is now given a power of 62.5 W, calculate the value of
resistance R.

Heater
I000W
I0Q

0 - - - - - - - 1 oov------U

Ans.R-5Q.
8. A charged capacitor of 5 x I o-2 F capacity is discharged through a resistor R of 20 Q and a copper
voltmeter of internal resistance 30 Q connected in series. If 4.62 x I 0--6 kg copper is deposited, calculate
the heat generated in the resistor R. (Electrochemical equivalent of copper is 3.3 x I 0-7 kg/C).

R'~30Q
C
q

Ans. 784J.
9. The temperatures of the junctions of a bismuth- silver thermocouple are maintained at 0°C and 0.001 °C.
Find the thermo-emf(Seebeck emt) developed. For bismuth-silver, a--46 x 1Q--6 V/°C and b--0.48 x
1Q--6v1°c2 •
Ans.-4.6 x 10-"V.
//
, /
/
10. Find the time required to liberate 1.0 litre of hydrogen at STP in an electrolytic cell by a current of5.0A.
Ans.29min.
II. A piece of metal weighing 20 g is to be electroplated with 5% of its weight in gold. If the strength of the
available current is 2 A, how long would it take to deposit the required amount of gold E C E of
H = 0.1044 x 10-4, atomic weight of gold= 197.1, atomic weight of hydrogen= 1.008.
Ans. 4 min.27.9 s.
12. Figure shows an electrolyte of AgCl through which a current is passed. It is observed that 2.68 g of silver
is deposited in IO minutes on the cathode. Find the heat developed in the 20 Q resistor during this
period. Atomic weight of silver is I 07.9 g/mol.

zon
~-~\,----l+ f:-------(

... . ,::: --...


. - -- -..- ___ --
,_

. . - - -. - . . ..
:..: :- :..: ;- :·15 :- :..:
.. .- -- .- ---- -~(.·\
"'
•'•\
-- .. --
=: : : : . :j:~ : : :
·····--------·-·····

Ans. l90kJ.
13. A copper wire having cross-sectional area 0.5 mm2 and a length of 0.1 m is initially at 25°C and is
thermally insulated from the surrounding. If a current of 10 A is set up in this wire.
(i) Find the time in which the wire will start melting. The change ofresistance with the temperature of
the wire may be neglected. ·
. (ii) What will be the time taken-iflength of the wire is doubled?
Given for copper wire, density= 9 x I 0 3 kg/m3, specific heat= 9 x 10-2 kcal/kg-°C, melting point
= ) 075°C, Specific resistance ) .6 X ) 0--8 Q-ro,
Ans. (i) 558 s, (ii) 558 s.
14. The potential difference across the terminals of a battery of emf 12 Vand internal resistance 2 Q drops
to 10 V when itis connected to a silver voltmeter. Find the silver deposited at the cathode in half an hour.
Atomic weight of silver is I 07 .9 g/mole.
Ans. 2g.
15. A plate of area 10 cm2 is to be electroplated with copper (density 9000 kg/m3) to a thickness of 10
micrometre on both sides, using a cell of 12 V. Calculate the energy spent by the cell in the process of
deposition. If this energy is used to heat 100 g of water, calculate the rise in the temperature of the water.
ECE of copper= 3 x 10-7 kg/C and specific heat capacity of water= 4200 J/kg-K.
Ans. 7.2 kJ, 17 K.
16. Find the amount of silver liberated at cathode of 0.500 Aof current is passed through AgN0 electrolyte
3
for I hour.Atomic weight of silver is I 07.9 g/mol.
Ans. 2.01 g.
0
CHAPTER
0
0
0
0
--
current in

:MafJl_etic Force on
Mo~Jlg Charges
~J.--AII ConduJEla1===
5.1 MAGNETISM : AN INTRODUCTION

The first magnetic phenomenon observed were those associated with naturally occurring magnets; pieces of
iron found near Magnesia (hence the term "magnet"). The study of magnetic phenomenon remained confined
for thousand of years to magnets made in this way. In 1820 Danish scientist Hans Christian Oersted observed
that a compass was deflected when it placed near a current carrying wire. This made the connection between
electrical and magnetic phenomenon. On this basis Oersted thus demostrated that magnetic effect could be
produced by moving charge. It is now known that all magnetic phenomena result from forces between electric
charges in motion. That is, charges in motion relative to an observer produces a magnetic field as well as an
electric field.
::;I
"


q E*O
B=O
(a) Frnmc S0 is at rest,
q-

(b) Frnme
"

s, is at rest
-
q
,,
E*O
B=O
(e) Frnme S2 is in motion, and
ehmge in it also at rest. and ehmge q is no rclati\·e motion between
moving w.r.t. it. frame and charge.
Fig. 5.1
5.2 THE MAGNETIC FIELD

In finding the force between the charges, it i~ very useful to introduce the concept of electric field and to
describe the interaction in two stages :
(i) Charge creates an electric field E in the space surrounding it;
(ii) The electric field E exerts a force F = qE on a charge placed in the field.
We shall follow the same pattern in describing the forces on a moving charge.
(i) A moving charge or a current creates a magnetic field in the space surrounding it;
(ii) The magnetic field exerts force on a moving charge or a current in the field.
Like electric field, magnetic field is a ve~tor quantity. We shall use the symbol B for magnetic field:
5.3 FORCE ON A MOVING CHARGE
' ·.
The electric force on a charge does not depend on its velocity; it is same whether the charge is moving or not.
The magnetic force, conversely, is found to .~ave ·a magnitude that increases with speed. Further more, the
direction of the force depends on the direction, of the field B and the velocity v. The force ·F always
perpendicula, to both B and v. The magnitude of force 'F is found proportional to the component of v
perpendicular to the field. The force is zero when v and B are parallel or antiparallel. Therefore the magnetic
force
, F ,=·. qv1_B=qvB1. =qvBsinB
.' .
I •} •
-
z

{fl'B sin0 . .,

y
Fig. 5.2
Direction of force
The direction of magnetic force can be obtained by either of the two rules.
1. Right hand rule: lfwe wrap the fingers of the right hand around the line perpendicular to the plane of
v and jj so that they curl around with the sence of rotation, then -.humb points in the direction of
force F·
F

r-+---~~-.v
qQ--,,,r+B

V,

Fig. 5.3
vxB
-, ..

B B

-
V
V
FB
(a) (b) ,i)','1 (c)
The right-hand rule (in If q posifu•e. then the If q negative, then the
which vis swept into Jj direction of Fl=' <J'' x l3 direction of F8 is
through tl1e smaller angle 0 is in the direction opposite to that of
between them) gives the ofvxB .. VX B.
direction of v x [i as
the direction of the thumb.
Fig. 5.4 ·""'
In the diagram shown the force on _the charge

F =· qvBsihOk.
In vector notation, we can write

lI\ ________
F- = q (-vx B-o
.., _ _. _ '
2. Fleming's left hand rule (FLHR): lfwe hold fore finger, middle finger and thumb of the left _hand
mutually perpendicular to each other; the fore finger shows the direction of B, the middle finger shows
the direction of the.velocity of positive charge, then thumb indicates the direction of force.

-(a) (b)

Fig. 5.5
More about magnetic field
I. Other name used for magnetic field are :
· (i) Magnetic induction (old name), \ ;.
(i~ Magnetic flux density.
2 Magnetic field ~an be represented by lines,_ called lines ~ffield,just as the electric field. B is related to
its line of induction in the following ways:
(i) The tangent drawn at any point to a line of field gives the direction of B at that point.
(ii) The number oflines per unit area gives an idea aboutthe magnitude of the magnetic field vector jj .
This field is large if lines are closer and small if they are apart.
The direction of B is represented as follows :
"! ,.
• • • • X X X X

x-
• • • • ./,
X X X

• • • • X X X X

• • • •
•)'.
X X X
'
(a) Equally spaced dols represent (bJ Equally spaced crosses represcm
the field pointing oul of 1hc 1he field into lhe page and
page and perpendicular to it perpendicular to it:

Fig.' 5.6

- - F -
3. The unit of B: The
. unit. of B can be obtained
.. from - . Thus SI unit of B •is NIA-m.
qv .
I NIA-m = I tesla.
Also IT ·= I o4 gauss,
' ' ~,, I Wblm 2:
Lorentz force
., .
When a charged particle moves through a region of space where both electric and"niagnetic fields are acting, the
net force on the charged particle is _equal to.the vector sum c,fthe forces exert by both the fields. Thus net force

;; i- --;;, ·i;·~,q(vxii)
'- - - -·- --····--- ·----
This net force is called Lorentz force.
More about magnetic force·
I. The magnetic force on a charged particle is zero, if either v =.0 or 8 = 0' orl 80' .
2. The magnetic force will be maximum, for8= 90°, which isF=qvB sin90°=qv8.
3. '
The magnetic force is always perpendicular to the direction of mqtion of charged particle, and therefore
magnetic force does no wolk on it. Hence the kinetic energy of moving charged particle in magnetic field
remains constant.
4. In using the formula F = qvB sin8, put the value of the charge q without sign. ·
'
5. In using the formula F = q(vx B), put the .value-of the charge with the proper sign.

Ex. 1 An electron is moving with a velocity


' ' ,, . ~
21 + 3} mis in an electric field of ;',;tensity
. . ' . -

(3i + 6} + 2k) Vim and a magnetic field of (2]+ 3k) tes.la'. Find the'mag~iiu~e and dir~ction of
' • , t ,, ·~ - '

the Lorentz force acting on the electron.


'"
L D
------------
MAGNE11C.FORCE .Qiv MOVING CHARGES
f ' ,-.
& CONDUCTOR
'
Sol. Lorentz force is given by

I I I '

9
- J.6 X 10-l [(31 + 6} + 2k) +( 2/ + 3}) x( 2} + 3k)]

- J.6x '?_- 1_9 [31 + 6] +2k +91-6] + 4k]


19
- l.6x 10- [12i+6k] = 9.6x 10- 19 (2i+.q

F = 9.6x10- 19 ~2 2 +I 2 =2.15x10- 18 N

It is inxz-plane, making an angle 0 with the x-axis, where

-I 2
0 cos r;-·' Ans.
· v5

Ex. 2 A proton beam moves through a region of space where there is a uniform magnetic field of
magnitude 2.0 T, with direction along the positive z-axis as shown injig.5. 7. The protons have

velocity of magnitude 3 x 105 mis in thexz plane, at an angle 30° to the positive z-axis. Find force

on proton, ( q = l,6x 10· 19 c)


F
Sol. The magnitude of force

F qv1_B
~-----~x
qxvsin30_° xB
z
=
19
(1.6xto-_ )x(3x10 )x( }(2) Fig. 5.7
5
½
= 4.8xt 0- 14 N
'
The di'.ection of force is along negative y-axis, thus

F = 4.8x10- 14 (-])N Ans.

Ex. 3 An experimenter's diary reads as follows: "a charged particle is projected in a magnetic field of

(1.oi-J.oj) x 10·3T. The acceleration ofa particle is found to be (o I+ 7.oj) xJ0-6 m/s2". The
number to be left of i in the last expression was not readable. What can this number be?
Sol. As magnetic force is perpendicular to the magnetic field, so
B.j'.- = o
1I
or B·ii = o.
Let value in box provided is k.
i;, ~ ! ,r
7k-3xT = 0=>k=3 Ans.
J:.i(.,

5.4 MOTION OF CHARGED PART~CLE IN UNIFORM MAGNETIC FIELD


The path of charged particle in magnetic field depends on the angle 9 between ii and B. Depending on
different values of 9, the possible cases are:
Case 1 : When 9 is 0° or 180°:

B
-------n
h
Fig. 5.8
,di

For 9 = 0° or 180°, the force on ,\he""'.oving charge F = qvBsin( 0° or


1
!s_o·} =_ 0, and therefore
particle:goes undeviated along a ·straight path: ,.,
.,
Case 2: When8=90°:
(i) When particle is projected from Inside the field, it experiences a force which always
perpendicular to the velocity arid so its path will be circular. The necessary centripetal force
is provided by the magnetic force. !fr be the radius of the path, then

mv2 1 J • o
-- = qvB sin 90
r , ,rf, ..
, • - 1 mv

or ~ ·':' qB
or we can write

...(I)
y
V
or r = B=B(-kJ

q
Let - = a, is called specific charge,
m X
The equation (I) can be written in the form: Fig. 5.9

. . \
· \ t
r· = aB, .,.. .
< '• ,
i ·7 - _-
.

The K.E. of the particle :K . p2


2m
,. or p =_ ·,.JzmK · •::. '.
If charged particle is accelerated by potential V, then
K = qV
,,·.'.
P .J2mK
' -=---
F,rniv
r = ... (2)
qB qB qB
Time period :
Length of path
T
speed

,•·1~ mv
-'2itx-
21tr . , , . qB
= --=
V V

; -~- -2iim·--7
or
qB __ :J
Also linear frequency of ~otation
qB
f= -=--,
T 21tm
,'{JI I• •

and angular frequency Ol 21t/ = Bq.


. ,,, m!

8'@4 1. Time period T,fand roare independent of. v. ,


2 The velocity at any instant can be written as
V = Vxi+vyj
- (ii) When particle is projected out side the field: lfthe length of the magnetic field is enough,
then the angle with which the charged particle emerges out will equal to the angle with which
it enters into the field. Thus we have two cases:
x x x x x x xB
xxxxxxxB
X X X X X X X
X X X X X X X
X X X X X X X
X X X X

X X X X X X X

0 X X X xr x
90° 0 \I 90° q
mv q
" r=-
qB
r
IIW
- (a) 0 r=-
(b) qB
F,ig. 5.10 ._
(a) Time spend in magnetic field ' (b) The time spend in magnetic field
T Jtm 2Jtm
1=-=- I= 28 [T] = ![T], where T = -
2 qB 2Jt 1t qB
PQ = 2r · PQ=2rsin8
...---,..--_-""t,..u---e-,·-TR-IC.,.~I-_
'lY.--- ----s--,M""'.<s.-:-'--,,..,.';"'-~=.;~'".{-,~-,,-:;'"::-i~:,-/-:--t.,,-;~-r.t...-,
.-?-:&-:,\-,m,..,'.,-,,.,,-,G-,•!i"'rn-:-, _-,~-·-::.,.•-----.
"'-i;"",.i,..Ii""",,-:,,-,,,..}""'-__'"",,..,-,.--,--

Case3: When particle is projected at an angle 8, which is not equal to 0°, 90° or.180°:

. tf{iljfJJi[f[[_
vsin0~
". I q
. . . . .
.· .· : / •· /

.
I O O

'
O 0

•B
PCOS 0 .. ·i:
I'
Fig. 5.11
The velocity ofthe particle can be resolved into two components; one along jj , that is vcos8 and
other perpendicular to jj that is l'Sin9. The velocity l'Sin9 will provide circular path and the velocity
vcos9 will provide a straight line. The resulting path is a helical path.
'
mv1. mvsin0
(i) r_=
qB qB

(ii) Since time period ofrotation does not depepd on ~elocity, therefore
- .-
2run
T .= -.qB
(iii) Pitch (p) : The centre to centre distance between two consecutive circular paths is called
pitch. Thus ", ' ·
p = vcos9xT

2run 2runvcos9
vcos 9 x--=---- ,,
qB qB

Comparison of paths.in £-field and B- lield


-- ,_
I. 9=0° I. 9=0°
.

-
q V

Straight Ii ne path
E
,• , -- ' q
- V

B
Straight line path
2 9=90° 2 9=90°
'
I' V

qt 'E
qt •B
Parabolic path Circular path

3. 0°< 9<90° 3. 0° < 9<90°


'
-' -
_q /4
"

: .- E
,c

',
·q1 '

'n
Parabolic natb Helical rmU1
·1 q
mv X X
(i) The radius of path
-·~( ., ' ... --.·r .. ~··
- -:---B
q 111 X X X X X B= B(-k)
X X X X X
1+---x-----t
.1.: - '··''"sihe'' 1 = _ _:l,. .·r:.:.· ·i·
v----
•1,. ,,.
and
. ·r\ \
X ,x X
· Above relation can be used when x,;;; r
X X X X
(ii) Forx>r r
....:....---...i , X '_X, ,

mv q
X X
X X B= /3(-kJ
r = -
,,_.-:qB 0--"""':+' X X X
.·• t 111 I+-- X > r - - . j
and deviation, 9 = 180° as clear from the diagram. Fig. 5.12
(iii) .. - If particle moves for time t inside the field,-then --·--- --
,'! ....r; .. :.h--,··t . { ~.'.....-.!"J;·,!;;·: . . ·lr..... :·o:e··~. co,· ... ,i· ., .'·..,
·.
' !. r.
_, ;:·: .. __
;;
-. (-~~)·
._ ..
:: .:\'
(' ,,,
•. m
: •: _v
As _,, a = _qlm C ·,.
9 Bat ·· .... ··.,.
(iv) Velocity of particle :
··· ...
........... x ...... •·.
We have, 9 = B0at, - .
B= 130 (-k)

Velocity of particle at any time I, V =. ~vxi_-+:vy)


•: - .. :·
. .
= :·yo cos9i+ v0 sin9j Fig. 5.13
,,...
On.substituting the value of 9, we have ~--~--~
·,_)

or C'f·.es _v ·c~s(B0ar)i+v
.-. '. - -~---t~ ......0: .. ~--
. .0
sin(~0 aijTi-;
. !-~ i
. . 1_ •• _, :· ! •

(v) Position of particle: ;: xi+yj

, r= \sin91+(r-rc~s9)] ,·-,, :
. .: p,_,,, .
''
! ,:· /r[sin91+(1-cos9)]] ".

..,,,::;: ti:'::, --vo


B0 a
[s!n(B0 ar)i+{l-cos(B0
- _
w)}J]
Ex. 4 When a proton is released from rest in a room, it starts with an Initial acceleration a 0 towards
west. When it is projected towards north with a speed of vu, It moves with'an initial acceleration
3a0 towards west. Find the electric field and the minimum possible magnet!~ field in the r«1om. ·
Sol. When proton released, it experiences only electric force, so
Fe ma0 =Eq
ma0 ma0
E -- = - - , towards west
q e
When proton is projected, it experiences force due to both the fields, and so
. Fe+ Fm = m(3a0)
As F;
= ma0,
Fm = 2ma0.
The magnetic force on the proton will be ev0B, therefore
ev0B = 2_ma0,
2ma
B = - -0 downward Ans.
ev0

Ex. 5 Aparticleofmassmendchargeqisprojectedintoaregion.havingaperpendlcularmagneticfield
·B. Find the angle of deviation (fig, 5.14) of the particle as it comes out of the magnetic field if the
width d of the region is very slightly small~r than
,
mv mv 2mv
(a) (b) (c)-
qB 2qB qB

' I.'

q
,,

Ii..i----:"-- d - - - - - i
Fig: 5.14
mv
Sol. The radius of path r =
qB

d
For d S r, we have sine
r

(a) mv) .· ·
d= . ( qB sine
=r, :.

..
or e lt d'tan
2 ra Ans.
-· . d ·l;;. ,. '~--~r '
(b) d= ( :;)<r, . _._;
2
sin8_ -~ r ,•,,:. ,. ..• ' ·, -. i-
.

''
,'mv
2qB I
= --=·-
mv 2
qB'
. lt • .
or 8 = 6 radian Ans.

2
(c) d= ( ; ; ) > r, the deviation of particle is therefore 8 = lt radian. Ans.

Ex. 6 Electrons emitted with negligible speed from an electron gun are accelerated through a potential
difference V along the x-axis. These electrons emerges from a narrow hole into a uniform magnetic
field B directed along this axis. However, some of the electrons emerging from the hole make
slightly divergent angles as shown injig. 5.15.
..
~
. ~. .-
----------------+~-~-~---------· .
'=:: ..• x-axis

11111 . B
Fig. 5.15

. . .
. Show that these paraxial electrons are refocused on the x-axis at a distance x = Jap
2
mv •
eB 2
Sol. The velocity of electrons as they emerge out,
' '

I 2
.-mv eV
2

or V = ~2~
The velocity can be resolved into two perpendicular components, vcos8 along x-axis and vsin8 .L to
it. The time taken to complete a circle

T =
2ltln
eB
In this time the distance travelled by electrons along x-axis
x = vcos8xT

(w7 2ltln)
( v---;;;- X ~ COS 8 n ,x
,. '·' • J '
l+X+I
for small 8, cos8 = I Fig. s.16

~ Proved
X
v---;;T
Motion of chai:ged.. particle in both electric and magnetic field
Case I : When E II ii and particle velocity is perpendicular to both the fields
Consider a particle of charge q and mass m projected from the origin with velocity v= ,,0 i into a

region having electric and magneti~ field i:e .. E=E0 ] and B=n0 ). The electric field exerts force
only along y-direction and therefore p~rticle accelerates in this direction with the acceleration
'
ay =-;;;
F,. qEo Th eve Ioc1ty
= ~- · componept .my-d"1r~ct1on
. goes on .mcreasmg
. wit"h"
,~tme. The magnetic
.

field rotates the particle in a circle inxz-plane. The resultant path of the particle is a helical path with
increasing pitch. . '
".
The velocity o{~ particle at any time I would be z
• _ V = _v.)+v_.,]:t:vzk.
;,. ,; • •t. I: ' ~ • • • ' J :· f_;.,, -~, >:-,

,l•':
., . - ,_
and

where v_,

and

·, 8 - 001·= Bq I ' ;_
m
• .' ., r • V

Similarly position vector of the partfcie


r =· xi+y]+zk

= (rsin8)i.+({a_.,d)J+(r~.rc~s0)k ,.,-,
,, . J J ' • ~.' • ,.

or t_ .~::(,;; )sin(B;I}+(½ ;: 12))


8!')}i
;,
-, .. . :,- . ·, '~(';:){1~cos(

'_,

P.4§, .W~i(e moving on a helical p~th the particle touches the y-axis after every T'l•seconcj, where
~·' .I • ; ' , . ~i . t

· 2ron
T= qB •
\
Case 2: When E _j_ jj and the particle is released ~t rest: Consider a particle of charge and mass m, is q
placed at the origin with zero initial velocity into a region ofuniform electric and magnetic field. Let
field £ is acting along x-axis and field B is alongy-axis, that is E = Eoi and B = B0 ].
z
\I: .. :.''
~ ' X A , A

,.. :,,: IJ'x " "


/4 /4 X A X X X X
(f (1":...,CX_JXL_~,-2XL_2!,X_JXL_2!,X-.\'
I=() E
Fig. 5.18
The electric force accelerates the particle along x-axis and so, the particle starts gaining velocity
along x-axis. As soon as particle starts moving, the magnetic fore~ starts acting and bends the
particle. The resulting motion of particle is inxz-plane.
At any instant its velocity
V vxf+vzk
The resultant force is thus given by

F q(E+vxn)

q[ Eoi+(v, i+v})xBoi]

q£0 i + qv_,B0 k+ qv=Bo (-1)


or fr q(E0 -v,B0 )i+qv_,B0 k ...(i)
and acceleration

fr q
-=-(£ . " q •
~vzBo)1+-v.rBok ...(ii)
ii
m m 0 m
This acceleration has two components

dv, q ( )
a.r = -dt· =-Eo-vzBo
m
...(iii)

dv. q
and a_ = - - =-v,B0 ... (iv)
dt m ·
Difterentiating equation (iii) w.r.t. time, we get
2
d v.r = _!!_Bo ( dvz)
d1 2 m l dt
Now substiMing d:
dv
from equation (iv), we have

or ...(v)
This is the equation similar to equation of SHM Le.,.
, • 1 , _ , • , • ,• ~i . ' • ,.
1'.• . d~X H ' ••?' i~
= - orx
dt 2 ,

where (I) (q!o)


and its solution for v., is
= A sin (W+cj>) ...(vi)
All= 0, v., = 0, hencecj> = 0
. dvx
Agam, di m4 cosox ...(vii)
From equation (iii), at t=0,
_,. -"· ' "
v, .;0
dvx qEo _;,·1
and
dt m
Now we have

coA cos(roxo)
qEo .. '
m
qEo
or A
mro

where (I) .= (q!o)


A

....(viii)

q
.-vx Bo
m

= qBo x Eo sin rot


m B0
dv, = qEo sinrot
or
dt m

or dv, qEo sin.rot dt


m
E I
or v, !L.9..
m
Jsin rot
,
dt
0
£ . • ···, : 1
r r ,; .• , -~
• ".; ·- ~ •. , i
_ q o (1-cosro,r .
.,\ · "·'"'y,,,_,,, __ .,. ,_ ,-,:,ifx(q!Jo)· ,, -· ,_
,,-;,; ~--_,·, ·;-i .li ,, .',.., ·- ', 1·, - · ,, - ~ l•. ,; ,' t, · . · ,n_ I ,. ' .. ~ "r ,,

or Eo (1-~osrot)
_Bo. .
j ...(IX)

From equation (viii),.we have


dx Eo .
--~ = -smrot
dt Bo
E ,_·
or X = _Q_
Boo,
J
sin rot dt

=- Eo J-cosro1J'
Boro o .
! ·. r . . : .,_ _ . . . ,........, ., '
'-""" . - ·.- E . . - 1 '
·,· ... · '· v-i-;,. ~x· ""=_·,,· ~'_'(t.e;·cosrot)- · 1 ,,
, or - . Boro:- .-." ...(I)
, . . L~~ t = . J ~ - ..
Also from equ~tion ((x), we h!lv<; , ,.. , . ,
" . dz '· Eo
· , -(1-cqsrot)
.
·-·'. '' '• I l
'dt· 8 0.. , . _
'··

or

~ .·. ,£,·,:_--. ;i"


or · l z = , ~(ror,:...sinrot)'· ..,(2)
L ' . Doro ' - . . .
Equations (1) and (2), together repiesents a cycloid which is defined as the path genenited by the
point on the circumference of a wheel rolling on the ground. 'I/;£' .
z .

47t

0)

21t
0)

Cycloid
• ·-;-. l,..' ......·'; ,.f .•• ~ 1.·,. . ,.

2E I.'
_o";..2.R ,> I

.... _:,. .;j .r :.•' , ~ .. B0ro . .- •• ~ '\1

. •, I, . , ~l r' ~ ., ,. ,:, ! · i; :· \• ., ..:::, I j_J,' , : ., , _,, • . ·,r: rJ ·, , ! , r•


., -r.. - ~-- -,- :.. ~. ' '',.'.
t·; . . Fig. 5.19 ; ·' • '.<· r r · ·!
·!r.P.1.-:
E:I---~
:'~lJECTRIGITY"i&1MA!1'Niffi~r,({-~~~-;?2~:r-';
' :-- ',.. ---- , .
-; -~,-\~1\(,t-~,
. . . ------"·~·...
J c : . •: •.

5.5 HALL EFFECT: 1879 ,


According to Hall, ifa current carrying conductor is placed in a transverse magnetic field, an emfis setup across
the conductor perpendicular to both current and magnetic field. This was first observed by Hall, and therefore
is called Hall effect.
/J

L.
I' 0
=--,,-+---...-----
#:~ 0 • ~ /,1/.

A
µ~~:i.....la/4~./~·-;,,
.
-•~B# f -+--... ;
_.. --k ·................... "(" --~
;···· . . PJj .,

l'ig. 5.20
Suppose we have a steady current i flowing in a uniform conducting strip along x-axis. If the strip is placed in
a uniform magnetic field B, acting along the y-axis, the field 8, will exert a deflecting force on electrons alongz-
axis and on positive ions along negativez-axis. Because of this force, the electrons displace towards front face,
ABCD and positive ions on back face PQRS. The displacement of charge carriers gives rise to a transverse field,
known as Hall electric field which acts along positive,z-axis, and opposes the side ways drift of the carriers.
When equilibrium is reach~d in which the magnetic deflecting.force on the charge carriers is just balanced by
the electric force caused by Hall electric field. Thus '
evdB, = E,;e
or · "e; -·;, --~iJ ' . .(I)
The Hall potential

VH =• EHxb
vdBb
!'.,; ,.,
t .
i
Also v'd
neA
As A bf,

_i_Bb
VH =
nebf
- ---iB ----
or VH = ...(2)
ne{

5.6 MAss SPECTROGRAPH


It is used to find mass number, to determine the relative abundance of isotopes or to produce separated
.
isotopes. ,
'
An instrument in which we use a photographic plate and obtain a series of lines on it are known as mass
spectrographs. Another instrument in which an electric or magnetic field is ~djusted to make each part of the
spectrum in tum to fall on a fixed detecting slit and i'n~astired electrically are called mass spectrometers.
All mass spectroscopes start with an ion source where the ions are produced by electron bombardment of
gases. The ions are first set in motion with the help of an accelerating potential.
In the spectrograph designed by Bainbridge, a velocity selector is used to obtain ions of particular velocity v.
The velocity selector allows a beam of a positive ions having the same velocity v to pass undeviated through
crossed electric and magnetic fields Eand B. In this case forces due to these fields are equal and opposite. i.e.,

Fe Fm
or qE = qvB

E~J ... (i)

1: (BX
s,I: • • B'
• •
s,

~~- -~-,. ~-~=:~~,:.,


•...+-t---t,.-+

Fig. 5.21 Mass Spectrograph.


All the ions having the same velocity, v = EIB enter the analysing chamber through slit. In this chamber another
magnetic field B is applied perpendicular to the plane of paper and in outward direction. Due to this field, ions
of different masses move in circles of different radii such that

mv 2
qvB'
r

qrB' · 1
or m = ... (ii)
V
~------·---
Assuming equal charges on each ion, the mass of each ion is proportional to the radius of its path. Ions of
different isotopes converse at different points on the photographic plate. The relative abundance of the isotopes
is measured from the densities of the photographic images they produce. For two isotopes of masses m 1 and
m2 , we have
r·-------- ,
,~ = ~-i
i mz rz I
L - ------·- '

5. 7 CYCLOTRON
Cyclotron is a device which is used to accelerate positive particles like a-particle, deuteron etc.·
It is based on the fact that the electric field accelerates a charged particle and the magnetic field keeps it
revolving in circular orbits of increasing radius.
It consists of two hollow D-shaped metallic chambers D 1 and D 2 called dees. The dees are connected to the
source of high frequency electric field. The whole apparatus is placed between the two poles of a strong
electromagnet N-S as shown in Fig. 5.22. The magnetic field acts perpendicular to the plane of the dees.
j 'J 1

High--~'-,...,
C2:J
frequency
A.C. source
6
f- 10 Hz
D,
'-:::~-\l~

Ea s ,:
Target

Fig. 5.22
(0 Cyclotron frequency : Time taken by charged particle to describe a semicircular path,
1tr 1tm
I=-=-
v qB
The period of oscillating electric field

2nm I B
T 21 = B and cyclotron frequency f =- =_q_.
q · T 2nm
(ii) Maximum kinetic energy of the particle:
mv
(a) We have r =
qB
mv0 r0qB
for --=}Vo=--
qB m
(r0, maximum radius ofcircular paih)

K..E. .!.mv/ =.!.,n ('oqB)2 =[q2 B2


2 2 m 2m
)r/
(b) Also, K.E. = work done by electric source
K..E. fx[qVx2]
. ~ 2/qV.

Ex. 7 Two positive ions having the same charge q but different masses, m 1 and m 2 are accelerated
horizontally from rest through a potential difference V. They then enter a region where there is
uniform field jj normal to the plane of the trajectory.
(a) Show that, if the beam entered the magnetic field along x-axis, the value of the
y-coordinate for each ion at any time tis approximately
I
I ; = Bx2 (-q-)z
BmV
providedx remain much smaller thany.
(b) Can this arrangement be used for isotope separation ?
· · MAGtfE11c;FoRi:E ON Mov1NG CHARGES & CoNvucroR
Sol. (a) The path of ion is shown injig. 5.23. The magnetic force provides an acceleration which is
towards the centre of path C. At any instant the acceleration
C
X X X
F = qvB 0
X X
m m
and ay

forsmallx, 8 ~ 0, cos8 ~ I

qvB
m
For X = Vt, Fig. 5.23 ... (i)

y = .!.a
2 y
t2 ...(ii)

From above equations, we get

.!_ qBx 2
... (iii)
2 mv
Since ions are accelerated through potential V,

.!.mv2 qV
2

Which gives V = ~2;;.


y
I
qBx2 2
Now y
2 m~2~B
Bx (-q
8mV

(b) From equation (iii) may1

Ex. 8 A particle of charge +q and mass m moving under the influence of a uniform field E i and a

uniform maguetic field Bk follows a trajectory from P to Q. The velocities at P and Q are vi and
-2v ]. Find therateofdoingworkbythefields. ."
Sol. y
Magnetic field does no work on the moving charged
particle. It bends the path ofihe particle. Electric field p V E
does work on the particle. Due to which its speed
changes. l
a @B
Workdone by electric field= 8K.E
or
> '

AKE. !o-'--------+-,,-x
2a ---.AQ
2v
or (Eq)x2a Fig. 5.24

2
3mv
or (Eq)
4a

Now rate of doing work at P Fe•v =(Eq)xvxcos0°


2
3mv
--xv
. 4a

' 3 mv3
Ans.
4 a

Ex. 9 A circular wire loop ofradius r can with stand a radial force T before breaking. A particle of mass
m and charge q (q > 0) is sliding over the wire.A magnetic field Bis. applied normal to the plane of
the wire. What maximum speed vmax the particle can have before the loop breaks ?
Sol. The centripetal force availa_!:(e ~s
f,, '.• = -(T+qvB)
From Newton's second law · ..
(". • 'I

2
mv
(T+qvB)
r
2
mv
or .,.---qvB-T
r , 0

Solving above equation for v, we get

vmax
- r[
qB + q 20 2 +-- 4Tm] Ans.
2m , r ·

Ex. 6 A particle carries a ch~rge of 4 x 10·9C. When it moves with a velocity v 1 of 3 x 104 mis at 45°
above thex-axis in the ,y-plane, a uniform field exerts a force F 1 along negative z-axis. When the
particle moves with a velocity v2 of 2 x 10~ mis along the z-a,xis, there is a force of 4 x 10·5 N
exerted on it along the x-axis. What are the magnitude and direction of the magnetic field?
Sol. Given,

,,,

In first case;
The force acts along z-axis, so field may be

B = Bxi+By}-
. . "

In the second case, z

Fig. 5.26

or (4xlo-9 )[2x10 4 kx(B)+By])]

or (4xlo-9 ) ( 2x 104 ) [ Bx (-])+By (-7)]

or
On comparing both sides of above expression, we get
Bx '0andBy=-0.5
Thus B o+(-o.s])r
'
--0.5} T Ans.

Ex. 11 A proton accelerated by a potential difference V= 500 kV flies through a uniform transverse
magnetic field with induction B = 0.51 T. The field occupies a region of space d = 10 cm thickness
(see jig. 5.27). Find the angle through which the proton deviates from the initial direction of
motion.

Fig. 5.27
Sol. If v is the speed of the proton, then

.!..mv2 qV
2
{itjv·'
or
~~:
V

Here q and m are the charge and mass of the proton respectively.
The proton traverses circular path of radius R in the magnetic field, where
}i,. 'I' mv 0
R =.
qB
From the figure
,, .
d
sin a.
R
d

qB

BdJ 2mV
q Fig. 5.28

On substituting the values, we get ct= 30° Ans.

Ex. 12 A particle with specific charge qhn moves rectilinearly due to an electric field E = E 0-,,x, where
a is a positive constant, xis the distance from the point where the particle was initially at rest.
Find
(a) the distance covered by the particle till the moment it come to a stand still;
{b) the acceleration of the particle at that moment.
Sol. The force exerted by the el~'ctric field on the particle
F Eq=q(E0 -ax).
Acceleration of the particle
dv
v- .'L(E0 -ax) ...(i)
dx m
Upon integrating, we have

Givenatx=0, v=O; :. C=0.

Thus ...(ii)

(a) The velocity of the particle again will be zero.

o = ~ ( E x-½ax2 )
0

2£0
or X = X
m
=--
a Ans.
(b) The corresponding acceleration, from equation (i)

·,, q (
m E0 -ax--;-
2Eo)
- qEo
Ans.
m
MAGNETIC FORCE ON MOVlf{G:CHARGES & CONDUCTOR

Ex. 13 A slightly divergent beam of non-relativistic charged particles accelerated by a potential difference
V propagates from a point A along the axis of a straight solenoid. The beam is brought into focus
at a distance l from the pointAat two successive values of magnetic induction B I and Bi- Find the
specific charge qlm of the particles.
Sol.

·····~············ -············B
Fig. 5.29
Suppose v0 is the velocity of the particle, then

I 2
-mv0 qV
2

or
w--
v--;;;-, ... (i)

For slightly divergent particle, 0--, O


v0cos0 ~ v0 . ....

If r is the time to travel a distance l, then 1 = _!_. in this dur~tion, let particle completes n circles for
Vo ,,
B I and (n + I) for B 2, then
l 21tm
n--
vo qB1

(n+I) 21tm ... (ii)


qBi

B1
On solving n = B2-B1
Substituting this values of v0 and n in equation (ii) and solving, we get

l B1 ) 27tm
( B2-B1 qB1

or !L Ans.
i 2(B2· -B1 )2
,n

5.8 MAGNETIC FORCE ON CURRENT CARRYING CONDUCTOR

When a current carrying conductor is placed in a magnetic field, magnetic force ts acted upon the moving
charges (electrons) within the conductor. This force is transmitted to the material of the conductor, and the
conductor as a whole experiences a force distributed along its length.
,.,;

Suppose a conducting wire of length£ carrying current i, lies in a magnetic field B (see fig. 5.30).

Im Im Im Im

Fig. 5.30
Consider a small element of length di of the wire. The free electrons drift with a speed vd opposite to the
direction of the current. We know thati =jA =neAvd· Here A is the area of cross section of the wire and n is the
number of free electrons per unit volume. Each electron experiences an average magnetic force

The number of free electrons in the element is nAd£. Thus the magnetic force on the wire is

dF'm = (nAdf)fm

Ifwe take di along the direction of the current, then -vd ( dt) = vd ( dt) and the above expression becomes

or dF'm

idlxB
e
Quantity id is called current element.
The force on the wire oflength £

[X = iix.B.
The above equation can be used in two stages
To find the magnitude of force by F= Bit sin9 and the direction of force can be obtained by Fleming left hand
rule or by right hand palm rule.
Force on curved conductor
Let us consider a conducting wire ofarbitrary shape and is placed in uniform magnitude field B.
The force on di length of the conductor dF'm = idlx B. To get force on the whole wire, integrate above
equation over the length of the wire. Thus
Q
JidtxB
Fm
p
pU.;-_
~Q :B
{!dc]xs Fig. 5.31

iPQxB.
The other simpler way to get the force on current carrying wire is : draw straight line joining the ends of the
conductor (here PQ), and then find its component perpendicular to B , here it is PQ sin8. Therefore
F = Bi(PQ sinB).

Ex. 14 Find the force on the conductors placed in uniform magnetic field as shown in the figure.

y
PIO
B

8
B
Sol. (a) (b)
X

F= Bi(PQsin 8), PQ=O


direction of force along+ z-axis F=BixO=O

y X
B= B(-k)
X X

X
PQ='J,.
X X
y Xu·X
!=~(-k)
0 X X X X
X
(c) (d) X X X X X
p Q
~x~~x __x~~x~~x~-.x
X X X X X X

F=BixPQ=BiA, F=Bi(PQ),
direction of force along + y-axis direction of force along + y -axis
(perpendicular to line PQ)

(e)
y

X
XOB
XX X X X

=.j3(-.j)
X
X

X
(f)
y

X
xxxxx

X
B= B(-k)
X

X X X
X X

F=O F=Bi(PQ)
direction of force pendicular to line PQ.
y y p

B=Bi ,,j )i B=Bi


(g) (h) (i
Q
' - - - - - - - -.. x ~ ~ - - - - - -.. x

F=O

"Magnetic force on any close loop in a uniform field acting perpen_dicular to plane ofloop always
be zero: .

Ex. 15 In the jig. 5.32 shown a semi-circular wire loop of radius R is placed in a uniform magnetic field
B. The plane of the loop is perpendicular to the magnetic field. Find magnettc force on the loop.

Sol. Consider an element of length dC = Rd0 in segment PS. y B= B(-k)


direction of force is normal to di. Similar element is tak,
force.

The resultant force on these elements


2dFsin0
2 Bi(Rd0)sin9 Fig. 5.32
The resultant force on the entire loop

.".
2
F f dF
0

.".
,, J
2BiR sin 9d8
0

2BiRl-cos9I t 2

= 2BiR[-cos%-(-coso)]

Bix(2R)

or F Bi(PQ) Ans.
/

The direction of force is perpendicular to line PQ. i.e., alongy-axis.


,•1, MAGNETIC FORCE ON MoilJNG CHARGES & CONDUCTOR
Ex. 16 Find in brief the force on the conductor shown in jig. 5.33.

__,:?-\ " ( \x
Sol. (a) F= 21JiR (b) x xJJ.::.Fi2BiR

8'~
X

X
'\_J ~\_j_),,x
2i
x x . x F?f 2BiR
X Xj X
I
X X X X

Fig. 5.33

Ex. 1 7 A circular loop ofradius a, carrying a current i, is placed in a two dimensional magnetic field. The
centre of the loop concides with the centre of the field as shown in Fig. 5.34. The strength of the
magnetic field at the periphery of the loop is B. Find the magnetic force on the wire.
Sol. Consider an element of length df, the force on it dF =Bi(df) sin90° = Bi(df) and perpendicular to
dR, and B,
that is along z-axis. For any of the element of the loop, the direction is along z-axis.
Therefore force on entire loop Y

21ta
F J Bi(df)k
0 -
B

21ta
Bi J df k
0 '------'~------x

or F Bi(21ta)k Fig. 5.34 Ans.

Ex. 18 A hypothetical magnetic field existing in a region is given by ii = Boe,, where e, denotes the
unit vector along the radial direction. Acircnlar loop of radius a, carrying a current i, is placed
with its plane parallel to the .l)'- plane and the centre at (0, 0, ti). Find the magnitude of the
magnetic force acting on the loop.
Sol. The situation is shown in the fig. 5.35.

a
sine = Bo
B0 sine
The given field can be resolved into two components: """
B0sin8, which acts all round the periphery ofloop and
lie in the plane of the loop and other component B0cos8
acts all round the periphery and perp~ndicular to the
plane of the loop. y
Fig, 5.JS
b:W 0
COS0

(a) (b)

Fig. 5.36
Force on the loop due to B 0cos8 is zero. The force is only due to B0sin0, which is
F = (B0 sin0)ix2rra

0
= 2Birrax
Ja2+d2

2Birra2
or F = Ans.
Ja2+d2

Ex.19 Fig. 5.37 shows a circular wire-loop ofradius a, carrying a current i, placed in a perpendicular
magneticfieldB. ·,
· (a) Take a small part df of the wire. Find force on this part of the wire exerted by the magnetic
field.
(b) Find "the force of compression in the wire.
Sol. y

X
X
OXBXX(I
.....
X •
I
X

X X X X

X X X X ,: X

Fig. 5.37
(a) The force on the element by magnetic field dF = Bi(d(), towards the centre ofloop (By FLHR).
(b) Let us now consider an element, cutout from the loop. The force exerted by rest part ofloop
or. the element is shown injig. 5.38. Let this force is T. ·
••. "(Jfi_,---~}J:Jii" ...

~T
Bi(df)
. Fig. 5.38
Since net force on the entire loop is zero, so the net force on-its element also be zero.
Therefore the equilibrium of element gives

2Tsin~ Bi(df)
2
. 8 8
for small e,sm- =
. 2 2
9
2Tx- Bi( a9) [d£=a9]
2
which gives T Bia .Ans.

Ex. 20 The magnetic field existing in a region is given by

ii = B0[1+;]k.
A square loop of edge£ and carrying a current i, is placed with its edges parallel tox-y axis. Find
the magnitude of the net magnetic force experienced by the loop.
Sol. Magnetic field atx = 0,
andatx=e,

Bi 2Bo k
The situation is shown in the.fig. 5.39.
The forces on the sides BC and DA are equal and opposite and get cancelled.
z
The force on side AB, Fj Boil j

And the force on side CD, Fi -2B0il]


Therefore net force on the loop

i Fi+ Fi
A
-B0il j
or F Bil. X Ans.

5.9 TORQUE ON A CURRENT LOOP IN MAGNETIC FIELD

Let us consider a rectangular loop ( a X b) is placed in a magnetic field iJ. The normal of!oop makes an angle

9 with the direction of iJ .


B
...... (90°-B) i
B
Bib A Bia cos0

···············~:;·················
, Bia co• :;::::::--- Bib

D~a ;,
Fig. 5.40
The force exerted by the magnetic field on four sides of the loop is shown in.fig. 5.41. On sides AB and CD, the
force Biacos9 is equal and opposite and their line of action is same. While on sides BC and AD, the force on
each is Bib and opposite in directions. Their line of actions are at a separation of a sin 0. Therefore the net force
on entire loop, Fnet = 0. But there is a net torque which is . (90-e)
~ Bibxasin0 1b ~
Bi(axb)sin9 ~ 1;b
BiAsine Fig. 5.41
If coil consists of N turns, the torque on entire coil is
't = (NiA) B sine
or 't _ = MB sine ...(!)
where M= NiA, is known as magnetic moment of the loop of N-turns.
In vector notation, it can be written as
... (2)

5.10 POTENTIAL ENERGY OF MAGNETIC DIPOLE


When the current loop or a magnetic dipole is placed in magnetic field B, work must be done by an external
agent to change the orientation of the dipole. This work done by agent becomes the potential energy of the
magnetic dipole. Magnetk energy is assumed to be zero when M is .l to B. If the dipole is rotated through an
angle 8 from zero energy position, then
8

u Wagent = f 't d0

9?1
M
90°

8
MB J sin8d8
90°

or E u -='.~.I;sfil Highest energy Lowest energy

or u = -if,jj Fig. 5.42

' -- --·, ;' 7 _, ... -;·· - ' " ' ,, .,

#@§ ,Above ·results are· d~rived for rectangular loop. Btit they can be used:for a pla~e loop o~ any!
,_ shape/ -
i: · For circular loop : M IA "i: }tcR2
and - 't = MB sjn8 = (irc//2) Bsin8
. ,Fnet 0 '· __
2 For solenoid: If windings are closely spaced, the solenoid can be approximated by a number of circular ,
loops. · · · ·
' ,J -r = NiAB sine.

Ex.21 Thefig. 5.43 shows four orientations, at an angle 8, ·or a magnetic dipole moment M in a magnetic

fi((b1ae;ld.Find:
the magnitude of the torque' on the dipole and cr,~-
_ M _ ~M. -(j)B
the potential energy of the dipole.
Sol. For the dipole
I, 81 8 @ lvf Fig. 5.43 M ©
2, 82 (180° -8)
3, 83 (180°+8)
4, 84 = (360°-0)
L--'--~--M_A_G_N_ci_.l_G_F_o_R_c_E_O_N_M_·o--'v_1N_G_ __G_Es_&_.:_c;'-o-~_D_u_c.;..~o-'-_R---"'~"-'·~:--·
.. _C_H_Ml
(a) Torque on the dipole is given by t = MB sin0: " ·
t1 MB sine
t2 MB sin (180° -0) = MB sine
t3 MB sin (180° + 0) =-MB sine
t4 MB sin (360° -0) =-MB sine
(b) Potential energy of a dipole in magnetic field is given by V = -MB case.
V1 -MB case
U2 -MB cos (180° -0) = MB case
V3 -MBcos(180°+0)=MBcos0
u4 -MB cos(360° - 0) = -MB case
Analogy between electric dipole and magnetic dipole
Electric dipole Magnetic dipole

I. p= qf. I.
direction from -q to +q direction from S to N

2 F.net = 0 2 F.net =O
3. ~ = PxE 3. ~ = MxB
4. V = -PEcose · 4. U=-MBcos0

More about M
(a) The direction of M is the direction of thumb of the right hand if the fingers of this curl around the loop
are in the direction of the current.

Fig. 5.44(a)
(b) Direction of M can also be determined as in the case of electric dipole, the dipole moment p has a
direction from negative to positive charge. In the similar way direction of M is from south magnetic pole
to north magnetic pole. The south and north poles can be determined by the sence of flow of current.
The side from where the current seems to be clockwise becomes south pole [SJ and the opposite side
from it seems anticlockwise becomes north pole [N].

i
@s-....-.

Fig. 5.44(b)
{c) M ofa rectangular square loop can also the determined by this method

M iA
A-----~B
;(ABxBi;)
= ;(ncxcD)
P-----.. c
=· ;(cDxD1) Fig. 5.45

.= ;(DAxAB)

Ex. 22 A square loop of side£ carries a current!. It is placed as shown in Fig. 5.46. Find the magnetic
moment of the loop.
Sol. Method I:
where z

and n = sin60°i-sin60°} A
B
../3; _j_ ........
2 2 Q y
60°
'£2
M ¼-(ffi-J)- ft
X

Methodll: M = ;(oc~cn). Fig. 5.46

Here oc £ cos 60°/ +£sin_ 60°]

../3 ··]
fi ,/3i] '[;-+-1
-+--=,
2 2 2 .2

and CB £k

M = u [(f+ ~ 1)x1c]
2

u [-J ../3 ,]
2 +
. 2 2
.. .

2i£2 [ ../3i-j.
• •] Ans. .

Ex. 23 Find the magnitude of magnetic moment ofthe current carrying loopABCDEFA. Each side of the -
loop is £ and current in the loop is i.
MAGNETIC FORCE ON MOVING CHARGES & CONDUCTOR
Sol. The given loop can break into two loops to get magnetic mom~nt that is in ABEFA and BCDEB.

B
I ·····•· •····
······-
··················If:." E

Fig. 5.47
The magnetic moment of each loop
M iA
= ;ez
Their magnetic moments are perpendicular to each other. Hence

Mnet = .fi. M = .fi. i e2 • Ans.

Ex. 24 A thin uniform ring ofradius R carrying uniform charge q and mass III rotates about its axis with
angular velocity c:o. Find the ratio of its magnetic moment and angular momentum.

Sol. The equivalent current in the ring

+
+ + + +
T 27t 27t q+ R +
+
(l) + +
+
+ + + + +
Magnetic moment M iA

qro X7tR2 Fig. 5.48


27t

qroR 2
2
Angular momentum L Iw
(mR2Jro

M q
Ans.
L 2m

Ex. 25 A positive charge q is distributed over a circular ring ofradius a. It is placed in a horizontal plane
and is rotated about its axis at a uniform angular speed c:o.A horizontal magnetic field B exists in
the space. Find the torque acting on the ring due to the magnetic force.
·1 ') "
Sol. =
We know that
Here
't

0
MBsin0
90°
+ + +
M
+O)+ +
+
-
B
't MB=iAB +
+ +
. 1\ + + + + .,.

( ~ Jxrra
2
Fig. 5.49
B
r

J..qoxi2B
2 . Ans.

Ex. 26 A thin insulated wire forms a plane spiral of Nlight turns carrying a current i. The radii ofinside
and outside turns are equal to a and b. Find the magnetic moment of the spiral with a given
current. .i- - ;
Sol. Consider dr width of the spiral, at a distance r from the centre. Numberoftums in this
dr
'··' '·. . N
•,·.u dN = -(-)dr
b-a
The magnetic moment of'i!~rrent loop, having dN turns _is,
dM (dN) iA
Total magnetic moment '·, " :

b ;,''
(-,',; .1
JdNiA Fig. 5.50
a

(_!!_)f
b-a
dr
a
iX1[r
2

1tNifb 2d
-- r r
(b-a) a

' ,'
icNi (b3-a3). Ans.
3(b-a)

Ex. 27 Fig. 5.51 shows a circular coil with 250 turns, an area A of 2.52x t0"4 m 2 , and a currentoflOO
µ A. The coil is at rest in a uniform magnetic field of magnitude B = 0.85 T with its magnetic
dipole moment M initially aligned with B.
(a) What is the direction of the current in the coil ?
(b) How much workwould the torque applied by an external agent to do on the coil to rotate it9O°
from its initial orientation, so that M is perpendicular to ii and the coil is again at rest ?
.~,~,,~ MAq~ir'tl~i¥lli:~#J.MJJu,ifc; GHAii{ES i eoNvvcrroR
Sol. (a) According to the right:hand'
rule:
. the direciion .
. - of .the current through the wires from th_e right
... side of the coil is f'rom top to bottom (see figure).
(b) Work done by the agent is given by ''
82 ,. ' , . 82
w J,:dB= JMBsin0d0
01 01

MB[cos0 1-cos02 ] ----.n


Given0 1=0°,02 =90° Fig. 5.51
w MB [ cos0°-<:os90°]
MB
= NiAB

(250) (100;10:?) (2.52xl0-4) (0.85)


•' I •• 1J I
= 5.4xl0-6 J =5.4 µ J Ans.

Ex. 28 The flat insuiating disc ofradius a carries an excess charge on its surface is of a C/m2• Consider
disc to rotate around the axis passing through its centre and perpendicular to its plane with
angular speed m_radls. If a ~agnetic field ii is dire~t~~8~rpendicular to the rotation axis, then
find the torque acts on the disc. . . ·
Sol. Suppose the disc is placed in xy-plane and is rotated about the z-axis. Consider an annular ring of
. radius r and of thickness dr, the charge on this ring u=- & euc/-·
dq = a(2rrrdr) , -
As the ring rotates with angular velocity m, so the current £r
z
dq a(21trdr)
= dt= 21t
m
awdr
The torque on the current loop
B
't = i A.xii. .
Hence the torque on this annular ring

Fig. 5.52

crmr w-(rrr 2 Bsin90°)


1tamr 3 Bdr
·ti·; r F11·, •
3
and 't = 1tam BJ r dr,, "

Ans.
ELEC'{,RICI'IY & MAGNETISM

Ex. 29 Consider a non conducting plate ofradius a and mass m which has a charge q distributed uniformly'
over it. The plate is rotated about its axis with an angular speed co. Show that the magnetic moment
,,, M q
Mand the angular momentumL of the plate are related as - = -
L 2111
Sol. If c, is the surface charge density, then
q = cma2 •
Current i = mordr(calculated in previous example)
The magnetic moment of the element ring
dM i(dA)
crwrdr (ro-2)
1tcrwr3 dr
a

and M
3
itcrro r dr J
0

4
ltO"Ola
4

2
2
(ita cr) ro: =/q:a
2
"µ I
The angular momentum of the disc about its axis
2
ma
L = --Ol
2

qroa 2
The ratio _4_=__9__ Ans.
,roma2 2m
2

Ex. 30 Two metal strips, each oflength f, are clamped parallel to each other on a horizontal floor with a
separation b between them.A wire of mass m lies on them perpendicular as shown in Fig. 5.53.A
vertically upward magnetic field of strength B exists in the space.~ met~ st!!JJs an;~h but
the coefficient of friction between wire and the floor is µ.A current i is established when the
switch Sis closed at the insta~t t= 0. Discuss the motion of the w i ~ theswitchisclosed. How
far away from the strips will, the wire reach ?
• • • • • • • •

:I
'
"T
l ~
• •
I•
!



f,




• b
.l_

Fig. 5.53
1· ' - '.
L ___ MAGNETIC FORCE ON MOVING CHARGES & CONDUCTOR
Sol. When current starts flowing in the wire, it experiences.a force, F = Bib of constant magnitude. Due
to which it accelerates on the metal strips. Thereafter when wire falls on to the floor, it retards due
to friction and finally stops. Thus ·

F Bib
a =. - = - -
m m
The velocity gained by the wire on the strips
v2 = 0+2aC
= 2
Bibf
m
Letx be the distance moved by the wire on the floor, its final velocity become zero, and so
0 = v2 -a'x

or X
v2
2a'

Here a' µmg =µg


m

2Bibf]
[ m Bibf '
X =-- Ans.
2µg µmg

Ex. 31 (a) A flexible free conductor is placed near a strong long bar magnet.
How will the conductor arrange itself if current is passed through it from the top to the
bottom?
(b) Currents are passed through two free rectilinear conductors arranged at right angles as
shown in Fig. 5.54 (b).
How will the interaction oft.he magnetic fields of the currents change the position of the
conductors relative to each other?
(c) A rectilinear current i2 is passed along the axis of a circular current ; 1 figure (c).
With what force are the currents interacting?
(d) A soft spiral spring hangs freely. The lower end of the string is immersed in a cup of
mercury. The spring and the cup are connected to a dc:source as shown in jig. 5.54 (d). What
will happen to the spring after the circuit is closed by a switch S ?
(e) In infinite rectilinear energized conductor AB has near it a movable uniform rectilinear
conductor CD offinite length the whole of which lies on one side ofAB and in a plane passing
through AB jig. 5.54 (e). What will happen to the conductor CD if current is passed through
it in the direction indicated by the arrow?
(I) Two vertical circular conductors with approximately equal diameters are arranged in
mutually perpendicular planes as shown injig. 5.54 (I).
How will the conductors behave if a current is passed through them in the directions indicated
by the arrows ?
(g) An energized wire ring is freely suspended from soft infeed conductors as shown in jig.
5.54 (g). A horizontal magnet is brought close to the ring.
What will happen to the ring in this case?
(h) A copper disc is secured on a horizontal axis and placed between the poles of a strong magnet
so that the north pole ofthe magnet is arranged on the right fig. 5.54 (h). The bottom of the
disc is immersed in &'~ip of mercury. The axis of the disc and the cup are connected to a dc-
source. What will happ'en to the disc when the circuit is closed?
·.,11, I \ /

' fl '

... Y
~--••H•••••7
(a) (b) (c)

C D
A
11 ,II, , ,
(d) J .. , . : ,. -
(e) (f)
/)'

,- ---, 'I
J. ..1,· J'

..__-.... +

(g) (h)

Fig. 5.54
Sol. (a) The conductor will wind itself around the magnet as shown in.fig, 5.54. (a).
In order to determine how the conductor moves, find the direction of the lines of force in the
sections adjoining the points A and B and apply the left-hand rule to establish the direction
of motion of these sections.
(b) The conductors will tend to tum so that they become parallel to each other and then will be
mutually attracted.
In order to solve the problem, consider the action of the magnetic field of current J1 on the ,,
·~
sections of the conductor with current J2 adjoining th~ points A, 0 and B. ,.
(c) F=O.
The lines of force of the magnetic field of curf,ent. 12 are concentric circles. The current 11
passes alo_ng one of these lines of force. For \his reason the magnetic field will not act on the
current 11• All the elements of the conductor wit.!J. current 12 also coincide everywhere in
direction with the axial line of force of the magnetic field set up by.the current/1• Likewise, the
current 12 will not be acted upon by any forces from the side of the magnetic field.

.... ,.
ff . ·:·}:· .;-{:.::.:-.__
....
. ..
.'.... .'. ..... .
I I I I I 0
o o I t I I

0 f o
' ' 'o
I O
0 0 I I O 0
I I O I I I
I I I I O l

ttt
1 I O
tjt I I 0
I

.. ..
I

I
0
I
O
'
I
I
'
I
I

I
I
. ...
O
'
' '
I
O
I
I
I

O
I
I
0
O

...... . ../ .
I I O I I 0
I I I I o 0
'
•.. ....',[J.
......
....... . ..-...
;:
: ..
••
.... ·.-·
ff

Fig. 5.55
(d) The end of the spring will perform periodic oscillatory motions.
When the circuit is closed each coil of the spring will, in the same way as circulating current,
set up its own magnetic field and attract the adjacent coils. The spring will be compressed and
its lower end will leave the mercury. The current circuit will be interrupted, the magnetic field
will vanish and the spring will then straighten itself out. As soon as the end of the spring
touches the mercury the entire process will be repeated again.

;._,·, ':.,'
A
rJ~
C O D

(b)

Fig. 5.56
. J

."J
'• ,11

(e) The conductor CD will first move upwards along the conductor AB, turning at the same time
as shown in Fig., and then will move away from it.
The direction of motion ofeach element in the conductor CD can be found from the left-hand
rule. Since it is given that the conductor is uniform its centre of gravity will lie at the point 0.
The intensity of.the magnetic field at the point C will be greater than at the point D. Larger
forces will act on the elements adjoining the point Cfrom the side of the magnetic field than
on the identical elements in contact with the point D. The point of application of the resultant
ofall the forces acting on the conductor CD will lie to the left of its centre of gravity. Therefore,
as the conductor CD moves up it will at the same time begin to revolve clockwise around the
point 0.

A
Fig. 5.57
(f) They will tum and set themselves parallel so that the directions of the currents flowing in
them are the same.
(g) The ring will be attracted to the magnet, fit itself round it and move along, stopping at the
neutral line.
(h) The disc will begin to rotate clockwise.
:MCQTy:ee 1 &xe,rdJe S, 1

1. The figure shows four directions for the velocity


v ii
vector v of a positively charged particle moving
through a uniform electric field E (directed out
of the page and represented with an encircled
0ii '\ F's
Fn0 ~0v~
I3 Fs
dot) and a uniform magnetic field ii. Of all four (I) (II) (III)
directions, which might result in a net force of
(a) I (b) 11,Ill
zero?
(c) III (d) I, II, III
v of a
- B
4. For four situations, here is the velocity
proton at a certain instant as it moves through a

uniform magnetic field ii :


(;) (i) v=2i-3] and ii:a4k
E
© (ii) v=3t+2] and ii=-4k
(a) 1 (b) 2
(c) 3 (d) 4
(iii) v = 3 ]- 2k and ii = 4/
2_, The figure shows a metallic, rectangular solid
that is to move at a certain speed v through the
(iv) v = 20/ and ii= -4/
If F 1, F2, F 3 and F4 are the magnitude of forces
uniform magnetic field ii. You have three
in the four situations respectively, then
choices for the dire.ction of the velocity of the
solid: vx, vyand vzparallel tox,y andzdirections (a) Fi = F2 = F3 = F4
respectively. For which choice is the front face (b) Fi =F2;F3 =F4
'at lower potential ?
(c) Fi = F2 = F3, F4 =0
y
(d) fi = F2 = F3 = 0, F4 at 0
5. A particle is projected in a plane perpendicular
to a uniform magnetic field. The area bounded
by the path described by the particle is
proportional to
(a) the velocity
(b) the momentum
(a) VX (b) Vy
(c) v, (d) none (c) the kinetic energy
3. Figure shows three situations in which a positive (d) none of these
particle of velocity v moves through a uniform 6. A positively charged particle projected towards
east is deflected towards north by a magnetic
magnetic field ii and experiences a magnetic field. The field may be
(a) downward (b) upward
force F's. In which situation(s) the orientations
(c) towards west (d) towards south
of the vectors are physically reasonable
7. A charged particle moves along.a circle under (a) the electron will tum to its right
the action of constant electric field and magnetic (b) the electron will tum to its left
field. Which of the following are possible (c) the electron velocity will increase in
00 E=~B-0 00 E=~B=O magnitude
(d) the electron velocity will decrease in
(c) E-0,B=O (d) E-0,B-0. magnitude
8. Given, B = magnetic induction and R = radius of 11. Two particles%and Yhaving equal charges, after
the path, the energy of a charged particle coming being accelerated through the same potential
out of a cyclotron is given by difference, enter a region of uniform magnetic ,
field and describes circular path ofradiusR 1 and
q2BR2 R2 respectively. The ratio of mass ofX to that of
2m (b) 2m
Yis

(a) ( ;JI/2 (b)

9. A magnetic field jj = Bo J exists in the region


a<x<2a and B=-Bo] in the region
2a < x < 3a, where B 0 is a positive constant. A
12.
(c) (;: r (d)

The radius ofcurvature ofthe path of the charged


positive point charge moving with a velocity
particle in a uniform magnetic field is directly
V = v0 i where v0 is a positive constant, enters proportional to
(a) the charge on the particle
the magnetic field atx =a: The trajectory of the
(b) the momentum of the particle
charge in this region can be li_ke :
(c) the energy of the particle '
(d) the intensity of the field

(•) l__JJ_,a 2a 3a
. BBR:h····
0

Vo a
-Bo ............
X
3a
13 . If a particle of charge 10-12 coulomb moving
· along the x-
direction with a velocity 10 5
mis experiences a· force qf 10-10 newton in
y- direction due to magnetic field, _then the
minimum magnetic field is

., ·1 "\j, (a)
(b)
6.25xJ03 teslain z-direction
10-15 teslain z-direction
3
(c) 6.25xI0- tesla in z-direction
(d) 10-3 tesla in z-:-direction
(c) zf i
l-L0,....x 14. An electron and a proton enter region ofuniform
a 2a 3a magnetic field in a direction at right angles to the
field with the same kinetic energy. They describe
circular paths of radius re and rp respectively,
Then
(d) z~x
(a) r,=rp
(b) r, <rp
10. A uniform electric field and a uniform.magnetic (c) r,>rp
field are produced, pointed in the same direction. (d) r, may be less than or greater than rp
An electron is projected with its velocity pointing depending on !lie direction of the magnetic
in the same direction / · ' field.
· (a)the path of proton shall.be more curved than
15. A proton of mass i'.67 x 1·0-27 kg and
that of electron
charge 1.6 x 10-t 9 C is projected with a speed (b) the path of proton shall be less curved than
that of electron
of 2 x 106 m / s at an angle of 60° to the x-axis. If (c) both are equally curved
a uniform magnetic field of0. l 04 iesla is applied (d) path of both will be straight line
along y-axis, the path of proton is 20. A homogeneous electric field E and a uniform
(a) a circle of radius = 0.2 m and time period magnetic field B are pointing in the same
itXlO-? s direction. A proton is projected with its velocity

(b) a circle of radius= 0.1 m and time period parallel to E . It will


(a) go on moving in the same direction with
2itxl0-7 s increasing velocity
(c) a helix of_radius = 0.1 m and time period (b) go on moving in the same direction with
constant velocity.
21tx10-7 s
(c) tum to its right
(d) a helix of radius= 0.2 m and time period (d) tum to its left
4nx10-1 s 21. Two particles A and B of masses mA and m8
respectively and having the same charge are
16. A proton ( or charged particle) moving with
moving ina plane. A uniform magnetic field exists
velocity v is acted upon by electric field .E and
perpendicular to this plane. The speeds of the
magnetic field B. The proton will move
particles are vA and v 8 respectively, and the
undeflected if trajectories are as shown in the figure. Then
(a) Eis perpendicular to B
(b) Eis parallel to v and perpendicular to B
(c) E, B and v are mutually perpendicular and
--~_
• • • • • •• • •

..
.. r-~~
. .,
....
• ·Jt ••••••

...
~
E •••••••••
v=-
B (a) mAvA<m8v8
(d) E and B both are parallel to v (b) mAvA>mBvB
17. A proton, a deuteron and an ex-particle having (c) mA<m8 andvA<v8
the same kinetic energy are moving in circular (d) mA=m8 andvA=v8
trajectories in a constant magnetic field. If/r , rd 22. A charged particle is released from rest in a region
and ra denote respectively the radii o the of steady uniform electric and magnetic fields
trajectories of these particles, then which are parallel to each other the particle will
move in a
(a) ra=rp<rd (b) ra>rd>rp
(a) Straight line (b) Circle
(c) r«=rd>rp (d) rp=rd=ra (c) Helix (d) Cycloid
18. A charge moves in a circle perpendiciirar io a 23. A particle of mass Mand charge Q moving with
magnetic field. The time period of revolution is
velocity v describes a circular path of radius R
independent of
when subjected to a uniform transverse magnetic
(a) magnetic field
field of induction B. The work done by the field
(b) charge when the particle completes one full circle is
(c) mass of the particle
(d) velocity of the particle
19; An electron and a proton with equal momentum (&) BQv21CR (b) ( M;z )21CR
enter perpendicularly into a uniform magnetic
field,-then (c) zero (d) BQ}:1tR
24. , A particle ofcharge-J6x 10~1~ coulomb moving 28.' A

conducting
I
circular ,-loop of radius
,
r carries a
with velocity IO ms~ 1 along the x,axis enters a constant current i. It is placed in a uniform
region where a magnetic field of induction B is
iniig'netic·field jj, such that B is perpendicular
along the y-axis, and an electric field ofmagnitude
to the plane of the loop. The magnetic force
!04 Vim is along the negativez-axis. lfthe charged
acti~g on the l6op is ·
particle 9onti_nues.,moving along the x-~is, the ; .
magnitude of B is . •" (;) ,; f;.jj
,' ' . ). ; .21tr/B
: -(b .
(a) 10-3 Wblm2 (b) 1.oJ Wb!m2 (c), Zero·. ' '.(cl) 1tri/J, .
{~) i'9.~ WI,/,,? •. (d) · ' 10
t
16 Wb!m2
' , .J ' 29. In a moving coil galvanometer, the detlection of
25. A charged particle of mass m and charge qtravels e
the coil is related to.the electrical current i by
on a circular path of ~dius r that' is perpendicular '-the reiatioiJ. 1 •
11
, '

·a
to magnetic field B. 'The time taken' by the
(a) i~tane (b) ;~e
particle to complde 'one revolution is
(c) i ~ 92 (d)' i "oc·./a
2rr.qB , 21tm
(a) ,(b) 30, Four wires each of length 2:0 metres are bent
m qB· 1. .intq,four_loop_s P, Q,R and Sand t,hen suspended
0 <
into uniform magnetic field. Same current is
· 21tmq 21tt/ B • passed in each loop. Which statement is correct
·m .. ,
. (c) ,,(d)

', "'ct],'
B
,. .. 'r. , . . ,., , . . ,,, , , ,,
2,6 •. A very. l9ng str~(ght wire c\lIT/e~ ,a cµrrent I. At . ''
. ,,
'l
tlie instant when a .charge +Q at p,<lint p has ' ' - .. '
- • • • . 'I
p
velocio/ v. ,
as shown, the force. on the charge is Q
' '
l=:::::::::::!J
, .. yL··
I········:--'~ { ~ ..
. . 0 X

(a) opposite to OX (b) alorig OX· ·


(c) opposite to OY· (d) along OY ··
.27_. A rectangular loop carrying a ~urre~t i ii situated _;; J

· near'a long striiight wire s~ch 'that the wire is


, (~). ¢ouple o~ loop P wi.11 b~ tpe highest
. ; , :· '.. parajiel ,io one of t~e. sid~s· ~fihe loo~ an'd is in II ' . , ' ' • .
(b) Couple on lqop Q ,yill be. the highest
' . ' tlie plane of the loop. ff a steady current I is
(c) Co~ple on loop R will b~ the hlgbest
establis~ed in wire as sho"'.n in figure, the loop
,, will . , '1 .:•· _.L·,.• ,,, (d). Couple on' loop Swill be the highlist
31. If ii cu~nt is p;ssed in a spring, it

I'-::.~' •
'·:",'.'
'ir:::Ji.,
, ' , I •'

~
• •


'
' .,. '


.."
'

'.. ' ' 'I


,•. ' (a)" "gets compressed
'(b) ·geis'expanded·
0

(c) oscillates
.. '
,,,

{d) remains unchanged


• !, ,, ., t, • , , , I ,
32. A stream of electrons is p_rojected horizontally
(a) rotate about an axis parallel to the wire to the right. A stiaigli't conductor carrying a
(b) move away from the wire or towards right current is supported parallel-io ·electron stream
(c) : move ;owa~ds the wir~ .. ' · >'' ' -.l "•''• and' above' it. If the current ih the conducto'r is
· c ··from left to right _then what will l:ie the effect on
·cd) remain stationary
\, -_- . - :,r . , electron stream?.
MAGNtri~1f'oR~~?:>JYr MgyJ~~.,~HARGES·•~t¢g'.,i~~crr;;k;.: · :IId
1 1

(a) The electron stream will be'pulled upward 37 .. For a positively charged P~r\i~Ie moving in ax-y
(b) The electron stream will be pulled plane initially along the x-axis, there is a sudden
downwards change in its path due to the presen·ce of electric
(c) The electron stream will be retarted and/or magnetic fields beyond P. The' curved
(d) The electron beam will be speeded up path is shown in thex-y plane and is found to be
towards the right non-circular. Which one of the following
33. Two long conductors, separated by a distance d
combinations is possible.
carry current I 1and I2 in the same direction, They _
exert a force F on each other. Now the current in y
one of them is increased to two times and its
directions is reversed. The distance is also
increased to 3d, The new value of the force
between them is
(a) -2F (b) F 13
(c) 2F /3 (d) -F/3
34. Two thin, long, parallel wires, separated by a
distance d carry a current i A in the same (a) E=O·, B=bi+ck
direction. They will
(a) attract each other with a force of (b) E= ai; B= ck+ai
2
µ0;2_1( 2itd )
(c) E=O;B=c}+bk
(b) repel each other with a force of
(d) E = ai; B = ck+b}
µoi2 I ( 27td2)
38. A particle of charge q and mass m moves in a
(c) attract each other with a force
circular orbit of radius r with angular speed OJ.
ofµ 0;2 !(2nd) The ratio of the magnitude of its magnetic
(d) repel each other with a force of moment to that of its angular momentum depends
on
µ0;2 !(2nd)
(a) OJ and q (b) ro,qandm
35. An ionized gas contains both positive and
(c) q and m (d) Oland m
negative ions. If it is subjected simultaneously
to an electric field along the + x direction and a 39. A and B are two conductors carrying a current i
magnetic field along the+ z direction, then in the.sa'me direction. x and y are two electron
(a) positive ions deflect towards+ y direction beiims moving in the same direction
and negative ions towards - y direction
(b) all ions deflect towards+ y direction ===::!=E""A
(c) all ions deflect towards -y direction ===""""'""B
(d) positive ions deflect towards -y direction ------x
and negative ions towards + y direction ------_v
36. A particle of mass m and charge q moves with a
(a) There will be repulsion between A and B
constant velocity v along the positivex direction.
attraction between x and y
It enters a region containing a uniform magnetic
field B directed along the negative z direction, (b) There will be attraction between A and B,
extending fromx = a to x = b. The minimum value repulsion between x and y
of v required so that the particle can just enter (c) There will be repulsion between A and B
the region x > b is and also x and y ·
(a) qb BI m (b) q(b-a)Blm (d) There will be attraction between A and B
(c) qaB/m (d) q(b+a)B/2m and also x and y
40. A conducting loop carrying a current I is placed 43. Two insulated rings, one of slightly smaller
in a unifonn magnetic field pointing into the plaoe diameter than the other are suspended along their
of the paper as shown .. The loop will have a common diameter as shown. Initially the planes
tendency to of the rings are mutually perpendicular. When a
steady current is set up in each of them

:w~
B tY

/ :
:
(a) the two rings rotate into a common plane
(a) contract
(b) the inner ring oscillates about its initial
(b) expand
position
(c) move towards+ vex-axis
(c) the inner ring stays stationary while the
(d) move towards-vex-axis
outer one moves into the plane of the inner
41. A current carrying loop is placed in a tinifonn ring
magnetic field in four different orientations, I, II, (d) the outer ring stays stationary while the
III aod IV arrange them in the decreasing order inner one moves into the plane of the outer
of potential energy. ring
44. Two very long, straight and parallel wires carry

'0-·"9-'
steady currents I aod I respectively. The distance
between the wires is d. At a certain instant of
time, a point charge q is at a point equidistant
from the two wires in the plane of the wires. Its
instantaneous velocity v is perpendicular to this

ill. 0.B N. r9B


(a) I> III> 11> IV (b) I> II> III> N
plane. The magnitude of the force due to the
magnetic field acting on the charge at this instant
is

(a)
µ 0Iqv
(b)
µ 0Iqv
(c) l>N>II>III (c) III>IV>I>II 2'1fd . 'lfa
42. A metallic block carrying current I is subjected 2µ 0Iqv
(c) (d) 0
to a uniform magnetic induction ii as shown in 'lfa
the figure. The moving charges experience a 45. _An electron moving with a speed u along the
positive x-axis aty = 0 enters a region ofunifonn
force F given by ...... which results in the
lowering of the potential of the face .......... Assume magnetic field jj = -Bok which exists to the
.1 the speed of the carriers to be v right ofy-axis. The electron exists from the region
after some time with the speed vat co-ordinate y,
y then
B y
X X X X X

X X X X X

X X X X X

X X X X X
z e-o--.!!- X X X X x, X

(a) eVBk,ABCD (b) eVBk,EFGH (a) v>u,y<O (b) v=u,y>O


(c) v>u,y>O (d) v=u,y<O
(c) -eVBk, A]tCD (d) -eVBk, EFGH
~ONl)U€10Il ..

46. A proton beam moves along the positive z-axis


through a region of space where there exists a z
uniform magnetic field of magnitude 4.0 T. The
protons have a velocity of 4 x 105 mis in thex-z ,
plane at an angle of 30° to the positive z-axis. (b)
The force on the proton is : y
0
,,
y
, ,,
X

z (c)
0
(a) 25.6x J0· 14 jN (b) -25.6x J0· 14 jN
(c) -12.8xI0· jN (d) 12.8xI0· 14 jN
14
X
47. A wire ABCD is bent in the form shown here in
the figure. Segments AB and CD are oflength 1
m each while the semicircular loop is of radius 1 z
m. A current of 5 A flows from A towards the end
D and the whole wire is placed in a magnetic
field of0.5 T directed out of the page. The force
(d)
acting on the wire is :

0 0 0 0 0 0 0 0 0
B
0 0 X

49. An electron mov.ing with a velocity v1 = I i mis


at a point in a magnetic field experiences a force
0 0 0 0 0 0 0 0 0
F 1 =-ej where e is the charge of the electron. If
0 0 0 0 0 0 0 0 0
the electron is moving with a velocity v2 = (Ii+
(a) 40N (b) 5N If) mis at the same point. It experiences a force
(c) ION (d) 20N F 2 = e (i - j). The force the electron would
48. A charged particle moves along the x-axis in experience if it were moving with a velocity v3 =
crossed magnetic and electric fields given as B v 1 x v2. At the same point is:
= Bz and E = Ey. The motion of the particle as (a) -e(i+j) (b) e(i+j)
observed in the laboratory frame will look like : (c) zero (d) ei
50. A mass spectrograph is a device for separating
z
charged particles having different masses.
Consider two particles of the same charge q but
different masses m 1 and m2 injected into the
(a)
region of a uniform magnetic field B with a known
velocity v normal to the magnetic field as shown
in the figure. The particles are separated by a
X distance_ d, given by :
0 0 0 0 0 0 Eo 2£0
(a) (b)
Bo Bo
0 0 0 0 0 0
Eo
(c) (d) none of these
2B0
0 0
52. A positive electric charge Q is distributed over a
circular ring of radius a. It is placed i~ a horizontal
r,,· ·o····o----~ ·;· ~o plane and is rotated about its axis at a unifonn
angular speed co. A horizontal magnetic field B
exists in the space. The torque acting on the ring
due to the magnetic force is :
(a)
d= l(m2 -m,)vl I
lqBI (a) q coa2 B (b) 2. qcoa2B
I 2(m2 -m,)vl 1 I .
(b) d
lqBI 00 -qcoa2B
n
W -qcoa2B
2n
l(m2 -m,)vl 53. · An jnsulating rod of length / carries a charge q
(c) d
l2qBI distributed unifonnly on it. The rod is pivoted at
an end and is rotated at a frequency v about a
I (m2 -m1)vl ,· fixed perpendicular axis. The magnetic moment
(d) d
l4qBI of the system is :
51. The electric field and the magnetic field in a region (a) . zero (b) ,r qv/2
are given by E = iE0 and B = jB0 . Consider a
frame of reference moving.with a velocity v0 k .
The electric field ih this frame will.be zero ifv 0 is
(c)
I
- nqv/2
2
(d) + nqv/2.

equal to: ·
r---.-l_____c_M_A_G_N_ET{_C_F_o_R_C_E_ON-M~-Vf-NG C~AR_G_ES_l:._J_·C_J_J
.. N-._!J-U-~TOR fijJ
------
~I~ \
~~gnetism
'-_,,.1 ~----- _:MC_Q Type 2 &~S.2
Directions (Qs. 1-7) : Read the following questions (c) path length of the particle in region II is
and select the right choices. More than one options qCB
may be correct. maximum when velocity v < - -
m
I. If a charged particle kept at rest experiences an (d) time spent in region II is same for any
electromagnetic force, velocity v as long as the particle returns to
(a) the electric field must be there region I.
5. A proton moving with a constant velocity passes
(b) the magnetic field must be there
through a region of space without any change
(c) the magnetic field may or may not be there
in its velocity. If i: and jj represent the electric
(d) the electric field may or may not be there.
and magnetic fields respectively, then this region
2. A charged particle moves in a gravity free space of space may have
without change in velocity. Which of the
(a) E=0,B=0 (b) E=0,Bc;,0
following is/are possible?
(c) Ect,0,B=0 (d) Ect,0,Bc;,0
(a) Ec;,0,B=0 (b) E=0,B=0
6. The current sensitivity of a moving coil
(c) E=0,Bc;,0 (d) Ec;,0,Bc;,0. galvanometer can be increased by
(a) Increasing the magnetic field of the
3. If a charged particle goes without any
permanent magnet
acceleration in a region containing electric and (b) Increasing the area of the deflecting coil
magnetic fields, (c) Increasing the number of turns in the coil
(a) i; must be perpendicular to jj (d) Increasing the restoring couple of the coil
7. A particle of charge +q and mass m moving under
(b) v must be perpendicular to i; the influence ofa uniform electric field Ei and a
(c) v must be perpendicular to jj uniform magnetic field Bk follows trajectory
(d) E must be equal to vB. from P to Q as shown in figure. The velocities at
4. A particle of mass m and chargeq, moving with P and Qare vi and -2v) respectively. Which of
velocity v enters region II normal to the boundary the following statement(s) is/are correct
as shown in the figure. Region II has a uniform y
magnetic field B perpendicular to the plane of
the paper. The length of the region II is C. choose
the correct choice(s)
v
-
E-
a

l
Region I Region II Region III
X X X
~:::::f-X
o..-i-----2a
V X X X
E=}_mv2 2v
X X X (a)
C 4 qa
(a) the particle enters region III only if its (b) Rate of work done by electric field at P is

qCB 3 mv2
velocity v > - -
m 4 a
(c) Rate of work done by electric field at Pis.
(b) the particle enters the region III only if its
zero
qCB (d) Rate ofw"ork done by both the fields at Q is
velocity v < -.- zero
m
Directions (Qs. 8-16) : Read th~fo/lowing passages 13. The thermal energy produced in the cup in
and answer the questions that follows. 1.00 his:
(a) 4.07MJ (b) 5.30MJ
passage for Q. 8 to Q.10. I
In a television tube, each of the electrons in the beam (c) 8.17MJ (d) 10.00MJ
has a kinetic energy of 12.0 keV. The tube is oriented
so that the electrons move horizontally froni
geomagnetic south to geomagnetic north. The vertical rassage for Q. 14 to Q.16. '
I
I
component of earth's magnetic field points down and A particle of mass m and charge q is moving in a region•
I
has a magnitude of55.0 µT.. __ .. .l where uniform, constant electric and magnetic.fields.I(
and B are present. E and Bare parallel to each other. Ai
I
8. The direction in which beam,deflects: time t= O,the velocity v0 of the particle is perpendicular
(a) east (b) west
to i:. Assume that it~ speed is always < c, (the spee1
(c) north-east (d) south-west
9. The acceleration of any ;iectron due to the oflight in vacuum). Express all the required answers i~
i
magnetic ·field is : " ierms of t, q, m, the vectors i: and B and their
(a) 3.14xI0 14 m/s 2 (b). 4.28xI0 14 m/s2 magnitudes v0, E and B._,,, ___ ----~
(c) 5.56xI0 12 m/s 2 (d) 6.28xI0 14 m/s 2
10. The transverse deflection of<
the beam after 14. Suppose electric field is acting along positive
travelling 20.0 cm through the television tube : direction of x and vx is the x-component of the
(a) 1.96mm (bh 2.98mm velocity of the particle at any time I, then vx is
(c) 4.24 mm (d) none of these
qE E
I (a) -I (b) -
Passage for Q. 11 to Q.13. m B
~ certain commercial mass spectrometer is used to
I

. .
separate uranium ions of mass 3.92 x 10-25 kg and
'
(c) 2qE I
m
(d) none of these

charge 3.20x!0-19c from related species. The ion~


15. If vy is they-component of the velocity of the
are accelerated through a potential difference of I 00
particle, then
kv and then pass into a uniform magnetic field, where
they are bent in a path of radius 1.0 m. After-traveling
\hrough I 80° and passing through a slit of width 1.00 (a)
!Jl!!l ~_nd_h_e_igl:,!_l ,Q9_~!11, they are collecte\l i!l ~ cup, :

\ ' 11. The magnitude of th~11 p7rpendicular magnetic (b) vy


. (2Bt)
= v0 sm m
field in the separation;, ,..
(a) 495mT _(b) 585mT
(c) 625 mT I \i)gj~mT
,:.,,, ' .
12. If the machine is used ~q-~epar~te out 100 mg of
material per hour, then the current of the desired
ions in the machine is :
(a) 22.7mA (b) 33.51 mA (d)
· (Bqt)
Vy= Vo COS -;;;
(c) 45.2mA (d) (;6,7 mA
' 'J/:f'r;.'j --/ . . . " ' -- .·,.; •",,•:;, •.,,;_ .· ·,-.' _'.'.-_J'·•;".'"_-:?-.: ... C >.,

· MAG~£1l<;:FoiicE ON ['!q~iN.<fC~~is(llr;;(~tiNvi1.qro_f!
16. The resultant velocity, v of the particle at any [Hint: z
time I is given by '

(a) v=v 0 cos(:)

(c) v=( ::}i+v cos( B1 )J


0
1

(d) none of these. X

0=wt= Bqt
m

Eq
v·=at=-1
X X m

Vy== Vo COS0 == Vo COS ( -;;;


Bqt)

and Vz
. e =v0 sm
==v0 sm , (Bqt)
-;;; .

Also v =vx i +vy }+vzk ,]

Multiple matching
17. A square loop of side a and carrying current i as shown in the figure is placed in gravity free space
having magnetic field B = Bof,. Now match following,
Y-~
i
,

Column-I . Column-II
(A) Torque on loop (p) is zero
(B) Net force on loop (q) is in direction (-k)
(C) Potential energy of loop (r) haS minimum magnitudes
(D) Magnetic moment of loop (s) has.maximum magnitudes
,1: r
~--ELECTRWIFYJ& MAGNETISM. ,~-;:- _-_-_-_-_-_-- -- -----_--_ _- ~---~ --~

~ ~--~----. - -
~gne~i~~ ____
,_~ }\1~Q Type 3 &Ulld4e S;3

' Read the two statements carefully to mark the correct option out of the options given below. Select the right
I h .
JC OICe.
(a) If both the statements are true and the Statement- 2 is the correct explanation of Statement- I.
(b) If both the statements are true but Statement - 2 is not the correct explanation of the Statement - I.
I (c) If Stateme,;t- I true but Statement- 2 is false.
(d) If Statement - I is false but Statement - 2 is true.

I. Statement I 6. Statement - 1
Magnetic field can not accelerate the charged Cyclotron is a device which is used to accelerate
particle. the positive charged particles.
Statement - 2 Statement - 2
Magnetic field can not change the speed of the Cyclotron frequency depends upon the velocity
charged particle. of the charged particle.
2. Statement - I 7. Statement - 1
The free electrons_ in a condu?ting wire are in A current carrying loop placed in a magnetic field
continuous thermal motion. If such a wire, must experience a torque.
without current is placed in a magnetic field, the Statement- 2
force on the wire is zero. Torque on the loop is given by -r=MBsin0.
Statement - 2 8. Statement- I
The each free electron in the wire experiences The net charge in a current carrying wire is,zero
magnetic force. and .so magnetic force on the wire in magnetic
3. Statement - I field is zero.
The net magnetic force on a current carrying loop Statement - 2
in magnetic field is always zero. The force on a current carrying wire is given by
Statement - 2 F-Bifsin0.
The resultant force on the loop is the vector sum 9. Stateme11t - I
of the forces on its elements. A charged particle in a region passes undeviated.
4. Statement - I The region must not have electric field
The kinetic energy of a charged particle in a Stateme11t - 2
perpendicular magnetic field B is K. When the The region may have electric field only or both
magnetic field becomes 3B, the kinetic energy of electric and magnetic field.
the particle becomes 3K. 10. Statemelll- I
Statement - 2 A charged particle moves in a unifonn magnetic
The magnetic force always acts perpendicular field. The velocity of the particle at same instant
to the velocity vector of the particle. makes an acute angle with the magnetic field.
The path of the particle is a helix with constant
5. Statemelll - I
pitch.
Cyclotron does not accelerate electron.
Stateme11t - 2 .
Statement - 2
The force on the particle is given by
Mass of the electron is very small.
MAGNETIC FORCE ON MoviNG CHARGES&. CONDUCTOR

11. Statement - 1 Stateme/11- 2


The figure shows the circular paths of two
particles : electron and proton that travel at the The force Fe= (iex ii).

same speed in a unifonn magnetic field ii, which 13. A charged particle moving through crossed
is directed into the page. The electron follows fields with the forces i;, and Fe in opposition.
the path of smaller radius.
E
If v =
8 , then particle moves undeviated.

I'
0 Statement- 2

If v < E the charged particle bends towards


s'
Statement - 2 electric field.
The radius of path in the perpendicular magnetic 14. Stateme/11- 1
A proton of charge +e and of mass m enters a
'. mv
field is given by r =- . unifonn magnetic field ii= Bl with an initial
qB
12. Statement- 1 velocity v=v0 ) +v0y]· The x-component of
The figure shows a current i through a wire in a proton velocity vox remain constant.
uniform magnetic field ii, as well as the magnetic Stateme/11 - 2
force Fe acting on the wire. The direction of The x and y components of proton velocity : vox
and v0 ,, change but speed of the proton remain
field must be along negative y-axis.
constant.
y
- - "- - -
;'I 1 ,~[-Jct-_i-,rI-12-.-.·I-(b-l_-~I_2_3_1 (cl n 34 ,~I-(c_l_l 45 I (ctl
I 2 I
(b) -:I 13 I (d) I 24 I (b) I 35 I (c) I 46 I (c)
.I 3 I (ctl :I 14 I (bl 25 I (bl I 36 ,1 (bl I 41 (c)
::1 _ 4 ,I ___ (cl_J _1_~.J .(~L-- 26 I_Jctl _,1 --~! _ii (bl I 48 (b)
:1 5 J (~) ,I 16 -1 (c)_ - _2? _ l __ (~t :I 38 __ I (c) I 49 (c)
i:l_s __ _l__(a) _I 1~__ I.(a)_ 2~ [_(c) !I 39 I (b) .1 50 (b)
;' I_ 1 _ 'I (al - :1 18 _ I (ct2 _29 _ I Jbl__ :1 4o I (bl_ I 51 (a)
;q _-~ J 1d) J _19 I_ (c) 3_0 __ I_ (d) ij 41_ _I (c)- ii 52
-
(b)
;i I 9 j (a) j 20 I (a) I 31 j (a) I 42 I (a) 'I 53 I (ctl
:· I 10 I (ctl ,I 21 J _ (bl ·1 32 I (bl I 43 I (al 1 I
11
1_ 11 ___ L_ (c) __d 22 __ 11_ (a) J 3~ _[j_c) _ ,j__ 44 _d _(d)___ I - I

__ :_ ___ -_--.-::"'--=-.:.=-=-=----·--==--=--:.=-----=-- ..::-=- -----"-~- -'----""--- ---- --.--.:.:.=-::=---,- ---- - -·-

!! I 1 / I (a) ( c) !I _6 q (a) ( b) (c) ii 11 I (a~ _ iI 16 :[ ( b) 1

!i I 2 _!I (b) (c) (d) 11_ !____ jl (a)(b) ~d) JI J


_1~ _ ~\_ ~--- _:j _____ !
j)L3 _JI_ (a), (b) ti 8- !I (a) JI __1_3 __il ___~)___ J -- -·'.-----'--'-------
1:1 4 _ :I (a) (c) JI 9 _JI (d) \I _14_ :I (a) ~
iI 5 II (a) (bl (ct) 11_1~ JI_ -~i___lL--1_:_J _~--J.---L_-_-___-__ I_ -- - - - ___:
" 17. (A)-p, s; (B)-p; (C)-s; (D)-q

,,__
e --~---- ~ - - ~ - - ~ - - ~ - - ~ - - ~ - -
/! I 1 ii ii 4 11 ULtdl jl_ _ii _Jb_J__ I__ .I __ (bl___:
'

(ctJ (ctl JI 10 13 __
111 2 :I (a) IL_5:_JI_ '(a) JI 8 ![ (d) JI_ 11_ _Jl_ __(a) __ I 14 j_Jc)_ !
1! I ___! ___J_ (~) - II _ _!_ _JI_ _(c) ii 9 ii (d) :I 12 ·1 - (a) I '
&xetd4e s. 4
1. A charge of l .O µC moves with a speed of 2.0x l 06 m/s along the positive x-axis. A magnetic field B of
strength ( 0.20} + 0.40k) T exists in space. Find the magnetic force acting on the charge.

Ans. (-0.8} + 0.4k) N
2. A proton, a deutron and an alpha particle moving with equal kin~tic energies enter perpendicularly into
r r
a magnetic field. If rP, rd and r a are the respective radii of the circular paths, find the ratio ...!!... and ...!!... ·
rd ra

I
( Ans. .Ji.' 1
3. '
An alpha particle is accelerated by a potential difference of I 04 V. Find°the change in its direction of
motion, ifit enters normally in a region of thickness 0.1 m having transv<irse magnetic induction O.l T.

(Given mass of a- particle 6.4 x I o-27 kg ). Ans. 30°.


4. A particle having a charge of20 µC and mass 20 µg moves along a circle of radius 5.0 cm under the action
ofa magnetic fieldB = l.O T. When the particle is at a pointP, a uniform electric field is switched on and
it is found that the particle continues on the tangent through P with a uniform velocity. Find the electric
field. Ans. 0.5 Vim, the direction of field is radially outward atP.

S. A narrow beam of singly-charged carbon ions, moving at a constant velocity of 6.0 x l o4 m/ s, is sent
perpendicularly in a rectangular region having uniform magnetic fieldB = 0.5 T(see figure). It is found
that two beams emerge from the field in the backward direction, the separations from the incident beam
being 3.0 cm and 3.5 cm. Identify the isotopes present in the ion beam. Take the mass of an
27
ion = A (l.6 X 10- ) kg, where A is the mass number.

X X X

:C:
X X X

12 C and 14 C.
Ans.
6. A particle of mass m= l.6x !0-27 kg and chargeq = l.6x 10-19C moves ata speed of l.OxJ07 mis. It enters
a region ofuniform magnetic field at a point£, as shown in figure. The field has a strength of l.O T. (a)
The magnetic field directed into the plane of the paper. The particle leave the region of the field at the
pointF. Find the distanceEFand the angle 9. (b) lfthe field is coming out of the paper, find the time spent
by the particle in the region of the magnetic field after entering it at£. v
__ J

X X

4t X X
'F
~· '} X X

• f X X

O<,. t X X

·.. : •' X X
E
X X
'1~
X X
C

Ans. (a) 14 cm (b) 4.?x 10-8 s.


7. Figure shows a convex lens of focal length 12 cm lying in a unifonn magnetic field B of magnitude 1.2 T
parallel to its principal axis. A particle having a charge 2.0x I o-3c and mass 2.0x 10-5 kg is projected
perpendicular to the plane of the diagram with a speed of 4.8 mis. The particle moves along a circle with
its centre on the principal axis at a distance of 18 cm from the lens. Show that the image of the particle
goes along a circle and find the radius of that circle.

Ans. 8cm.
8. At the moment t = 0 an electron leaves one plate ofa parallel plate capacitor with a negligible velocity. An
accelerating voltage, varying as V = at, where a= I 00 V/s, is applied between the plates. The separation
between the plates is £ = 5.0 cm. What is the velocity of the electron at the moment it reaches the
opposite plate ? Ans. 16 km/s
9. An electron accelerated by a potential difference V= 1.0 kV moves in a uniform magnetic field atan angle
ex= 30° to the vector jj whose modulus is B = 29 mT. Find the pitch of the helical trajectory of the
2
electron. Ans. p=2rc~2mV/eB coscx=2.0cm.
10. An electron is released from the origin at a place where a uniform electric field E and a uniform magnetic
field B exist along the negative y-axis and the negative z-axis respectively. Find the displacement of the
electron along the y-axis when its velocity becomes perpendicular to the electric field for the first time.
y

El :'...
.
-··--r y ®B
~--~-------x
2Em
Ans. y = - 2- ·
eB
'-i_ _ _-'--_ _ _
1
M_A_GN_m:_···-'ro_.··_F_o_;c_-;_._o~_~jv.r~~-~~~GES & CONDUCT~~-:~~
11. A current loop of arbitrary shape lies in a uniform magnetic field B. Show that the net magnetic force
acting on the loop is zero.
12. A wire of length £ carries a current i along the x-axis. A magnetic field exists which is given as
B= Bo (7 + j + i) T. Find the magnitude of the magnetic force acting o~ the wire. Ans. -./2.Boif
13. A rectangular wire-loop of width a is suspended from the insulated pan of a spring balance as shown in
figure. A current i exists in the anticlockwise direction in the loop. A magnetic field B exists in the lower
region. Find the change in the tension of the spring if the current in the loop is reversed.

X X X X

X X X Bx
X X X X

X X a X X

Ans. 2Bia
14. (a) A wire loop carrying a current I is placed in xy-plane as shown in figure. If a particle of charge+ Q
and mass m is placed at the centre P and given a velocity v along NP. Find its instantaneous
acceleration.
(b) !fan external uniform magnetic field ii= Bi is applied, find the force and the torque acting on the
loop due to this field. ·

.I -······--
· -. . YL
#+~- '\
:
X

,v/!a_____________./

0.109µ 0 JQV ,
Ans. (a) a= , at 30° with negative x-axis.
ma
(b) 1 = 0.6136 a2IB alongy-axis.
15. A coil in the shape of an equilateral triangle of side 0.02 m is suspended from a vertex such that it is
hanging in a vertical plane between the pole pieces of a permanent magnet producing a horizontal
magnetic field ofSx I o-2 T. Find the couple acting on the coil when a current of0.I A is passed through
it and the magnetic field is parallel to its plane. Ans. 5-Jj x I o-7 N - m.
16. A circular loop carrying a current i has wire of total IengthL. A uniform magnetic field B exists parallel to
the plane of the loop. (a)Find the torque on the loop. (b) If the same length of the wire is used to form a

Bif! BiI:
square loop, what would be the torque? Which is larger? Ans. (a) - - (b)--
47t 16
IEl!E0TRIBI;r,Y, Wi.! IMAGNF!IISM
.· ;;,;,.
17. Consider a solid sphere,ofmass m which_has a charge q distributed uniformly over its volume. The
sphere is rotated about a diameter with an angular speed co. Show that the magnetic mome~t Mand the

angular momentum L of the sphere are related as M =_!J_ i.


2m
18. A proton moves at a constant velocity of +50 mis along an x-axis through crossed electric and magnetic
. -V
fields. The magnetic field is B=(2.o}) mT. What is the electric field? Ans. -'{).IO k - .
m

19. An electron moves through a uniform magnetic field given by B= B) +(3Bx)J. Ata particular instant,
the electron has the velocity v=(2.0i+4.o})m!s and the magnetic force acting on it is

· -19 k') N.
( 6.4xl0 Find Bx. Ans.-2.0T.
20. Two concentric, circular wire loops, of radii 20.0 and 30.0 cm, are located in the.ry-plane; each caries a
clockwise current of7 .00 A (see figure).

--t-t--1<:--t----t-----X
¥-+-30cm

(a) Find the net magnetic dipole moment of this system.


(b) Repeat for reversed current in the inner loop. Ans. (a)2.86A-m2,(b) LIOA-m2•
21. A beam of electrons whose kinetic energy is K emerges from a thin-foil "window" at the end of an
accelerator tube. There is a metal plate a distanced from this window and perpendicular to the direction
of the emerging beam (fignre). Show that we can prevent the beam from hitting the plate ifwe apply a

uniform magnetic field B such that B <! ~ 2~~ , in which m and e are the electron mass and charge.
ed

How should B be oriented ?

Electron
beam
Plate

i---d--1
Tube
22. A wire of 62.0 cm length and 13.0 g mass is suspended by a pair of.flexible leads in a uniform magnetic
field of magnitude 0.44 T (figure). What are the magnitude and direction of the current required to
remove the tension in the supporting leads ?

~
~ X X X
I• •I
62cm

Ans. 467 m A towards right.


23. Figure shows a wire ring of radius a that is perpendicular to the general direction ofa radially symmetric,
diverging magnetic field. The magnetic field at the ring is everywhere of the same magnitudeB, and its
direction at the ring everywhere makes an angle 0 with a normal to the plane of the ring. Find the
magnitude and direction of the force the field exerts on the ring if the ring carries a current i.

· Ans. 21ta i Bsin0 (up).


24. Figure shows a rectangular 20-tum coil of wire, of dimensions 10 cm by 5.0 cm. It carries a current of0.10
A and is hinged along one long side. It is mounted in the zy-plane, at 30° to the direction of a uniform
magnetic field of magnitude 0.50 T. Find the magnitude and direction of the torque acting on the coil
about the hinge line.

.-
.....
Hinge .-
line .-..··
.-.-
......
)s:--r=~-'--~-x
30°
B ../
.-
z

Ans. 4.3xJ0-3 N-m; negativey.


25. Two protons move parallel to each other with an equal velocity v = 300 km/s. Find the ratio of forces of
magnetic and electrical interaction of the protons.

2
Ans. v =lx!0-6
2
C
------- ---- -- ------------,
M ·. 'El1EC·TR1€JTY'.&MAGNETISM-_-_\_;_ _ ____' ~ ---- ---------'
26. In figure, the long straight wire carries a current of3O A and the rectangular loop carries a current of2O
A. Calculate the resultant force acting on the loop. Assume that a= 1.0 cm,.b =s 8,O.cm, and L = 30 cm.

30A

20A
• a{ • .:

( ·-
,,
'V 1, •

, , . Ans.3.2mN,towardsthewire.
27. Figure sh~ws a wood cylinderofmassm= 0.250 kg and le~gtbl ~0.!00 m, withN= IO.O,tur~s of wire
wrapped around it longitudinally, so that the plane of the wire coil conta;'ns-tlie axis of the cylinder. what
a
is the'le~st current i tlirough the coil that ~iii preven°( the ~ylinder from ;dlling down pl~ne inclined at
an angle 0 to the horizontal, in the presence ofa vertical, uniform magnetic field of magnitude 0.500 T, if
the plane of the coil is parallel to the inclined plane?

·-

. ,,

'· .,.
,,
A11s.2.45A.
28. Figure shows a current loop ABCDEFA carrying a current 2 A. The sides of the loop are parallel to the
coordinate axes, with AB= I 0.0 cm, BC= 20.0 cm, and FA= 5.0 cm. Calculate the magnitude and direction
of the magnetic dipole moment of this loop. '

C D

1 "'2,

\ \ . ~ - - - ·_.-+--F-j~c_i_n_y
B A
~IO.Dem
X
0
CHAPTER
0 J; I•
Batteries,
buzzer or

0 tiJAeperand
electronic
circu!ls in
haodlG _.-

0 Wires in shatt Coif of Wre works


as electr¢rrognet,

,,

6.1 MAGNETIC FIELD'OF MOVING CHARGE


,,
We know that a.point charge q, at rest in the observer's inertial frame, produces an electric field·along the radius
vector and is giyen by,

E _I )..'L;:_
. ,· ,, , ( 4rtEo ,-3

If the charge' is moving relative to the observers inertial frame, it produces a magnetic field in addition to electric
field. The magnitude of which 'is proportional to the speed of the charge relative to the observer provided
(v:~. c). The magn~ti~ fi~ld ~ector B at the poi~t P, a distant pfro~ the charge qmoving with a. v~lo~lty ~ ,,is
;
found to be , , , --
., .'
s (µ 0
~(vxr).
47t ,.J
... (I)
,.
q
V
s
.'
' . . •l~·:
Fig. 6.1 ,
The direction ofB is thus perpendicular to th~ plane of v and ;: , It is in the direction of advance of a· right
handed screw rotated from v to ;: , Its magnitude is given by

B ... (2)
The following points should be remembered regarding with the magnetic field.
J:''· The magnetic field B is zero at all points on a line on which charge moves. That is when8= 0 or8= 180°,
B =O.
2 It is maximum in the plane perpendicular to the v and through the charge, as sin 8 =I, at all points in
this plane:

., =
8 180°
8=0°
(a) (b)
Fig. 6.2
3. B remains unaltered in magnitude at all points on the circumference of circle passing through P and
lying in a plane perpendicular to v with its centre on the velocity direction.
4. The direction of B is given by either:
(a) Maxwell's right hand screw rule: Ifthe direction of current through the conductor or the direction
of velocity of positive charge is represented by the linear motion of the screw motion, then the
direction of the magnetic field can be represented by the direction of rotation of the screw.
• (b) Right hand claps rule : If a current carrying conductor is elapsed in the right hand so that the
thumb indicates the current direction, then the direction of the magnetic field is represented by the
finger tips round the wire.
Electromagnetic field
So far, we have considered electric and magnetic fields separately, without establishing any clear relation
between them. This could be done only because the two fields were static. In other cases, however, it is
impossible. It will be shown that electric and magnetic fields must always be considered together as a single
total electromagnetic field. In other words, it turns out that electric and magnetic fields are in certain sense the
components ofa single physical object which we call the electromagnetic field. The division ofthe electromagnetic
field into electric and magnetic fields is to relative nature since it depends to a very large extent to the reference
system in which the phenomenon are considered.
Force between moving charges
The force acting on a charge q2, moving with velocity v2 in a magnetic field produced by charge q I moving with
a velocity v1 is
The magnitude of force which they exert on each other
µ0 q,q2V1V2
Fm 47t r2 r
For v, v2 =v Fig. 6.3

µo q1q2 2
Fm = - --v
41t. r2
.....(i)

In addition to the magnitude force, there is an electric force between them, whose magnitude is given by
I qlq2
Fe = 41tEo 7· ......(ii)

This force is of repulsive nature. On dividing equation (i) by (ii), we have

As c = ·~µolEo·

[F,:- = v2-] .....(3)


' F.
I...'..!__ CZ

Since v < c, and so Fm< Fe. As F,n <Fe, so the net force between the charges is of repulsive nature.

6.2 THE BIOT SAVARTS LAW


Biot-Savarts law is the fundamental law of the magnetics. It gives the magnetic field by a small current element
and is based on the experimental facts. Let us consider a small element d f. of a wire of cross-sectional area A.
carrying current i. If n is the numberof charge carries per unit volume, each of charge q and moving with a drift
velocity iid , then
neAv;i
The quantity of charge flowing in time dt, dq idt = neAv jdt)
Since vd(dt) =di, dq neAdf.

The magnetic field due to the current element d f at any point P at a distance r
µ 0 dq(vd x,)
dB = 41t r3
µ 0 (neAdR)(vdxr)
47t r3

µ ( A ) (df.xr)
- 0 ne vd Fig. 6.4
41t r3
-- ----····---,-,
; - µo idfxr '
... (I)
or 'dB = 41t ~ !
This is called Biot-Savarts law.
IEPI-· ELECTRICl'IY,&,MilGNETJSl\1~'

The field due to the whole current carrying conductor is given by .

B JdB
fQ_ji(dfxr) fQ_ ifXr
or a 4lt ,.3 4it ,-3
... (2)

Magnetic field due to a straight current carrying conductor


Consider, a straight wire carrying current i. Its ends subtend angles 8 1 and 8 2 at
ti
point Pat which field is to be determined.
Take a small element df ofthe wire at a distance e from 0. The magnetic field due
to element is
I
f
µo id£sin(90°+8) .
dB = 2 . ". (1)
41t X l.11----,.___,,..__=,.p
The direction of field atP is perpendicular to the plane of the diagram and going
into it. The direction of the field is the same for all elements of the wire and hence
net field due to the wire is obtained by integrating equation (i).

e
From the diagram, ,. tan e
or e rtan8 Fig. 6.5

and df rsec 2 8d8.


Putting in equation (i), .,
µ 0 i(r sec 2 8d8)
dB 2
cos 8
41t X

X
Also -= sec8 or X = rsec8
r

µo i
dB --cos8d8
41t r

. 0,
Total field B
.µ0
-- I J cos8d8
41t r
-02

or B µoi(.8
--- '8)
sm 1 +sm 2
4rt r
/.
__,
We can draw magnetic field lines in the same way that of electric field lines.
A tangent to a magnetic field line gives the direction of the magnetic field
existing at that point. For a long straight wire, the field lines are circles with
their centres on the wire.
Fig. 6.6
Special cases :
r 81 =O
e2 = 12 P

r
rc/2 -
B
a

rc/2
r

- r
(a) (b) Variation of B withr. (c)
Fig. 6.7
(a) Field due to a long straight wire (b) Field along the end of the wire
1[ It
8 1 =- and 8 2 =-
2 . 2
µo ;
B=-,-
21t r
Force between two parallel current carrying wires
Consider two long straight wires kept parallel to each other at a
distance rand carrying currents ; 1 and ;2 respectively in the same
direction.
Magnetic force on the small element d £ of the wire 2 due to the
magnetic field of I is
di' i2 (d£xB1)
The magnitude of the force
dF i2 ( df)/Ji sin 90°

where ·B
. 1 = ~i
21t r
µo i1i2 (df.)
dF Fig. 6.8
2it r
dF ~i1i2
or df 2rc r

Thus force per unit length of wire 2 due to I is ~ iiiz . The·same amount of force I exerts on 2.
· 2it r
The direction of force on wires are towarciiieach other that is attraction (By FLHR).
If direction of currents in wires are opposite, the force between them will be repulsive.

(a) Attraction. (b) Repulsion.


Fig. 6.9
,,

,----- - -·--- - -- -- - - -·~---·-- ·--·--····-··- - -- -,,.} --· - ~-·-- - - • -


rnr•
- - - - · - · · - - · .. - - · - ~----- :;---·-'>;,';•j·-~

There is no electrical inferaction between the wires, because they have no net charges,

Field due to a circular current carrying coil


Consider a circular loop ofradius a carrying curteni i, We ha~e to find the magnetic field at a point Pon the axis
of the loop at a distan~ex from the centre of the loop. Consider a current element i df. of the wire, The magnetic
field at P due to this element is

µo , df.xr
dB = -.1--
41t r2
I,_ ' '
As d f. is perpendicular to the plane of the paper,'a9d ,so· d £ x·, must lie in the plane of paper, as shown in
figure.
The magnitude of the field

dB

-, •;,

Fig. 6.10
Another element on the diametrically opposite'ioi~t produces_the same amount of field on the point P. The
resultant field due to these elements= 2dB sin cf:.·
r1···
The effective field of one element at P, r•c·_ ··,
= dBsina
Field due to complete loop

B JdBsina
. ,

f
2lta 'd'
-µ0 , -smcx
I < .
.,4!t , o r 2 .

µo ix2rra a
~-~-x-
41t r2 ·- r ·,
'-

µ 0 2irra 2
or B
41t r3
l .,· ••
' ' '
Since r .= ~a2 +x2
_, ---- -- - - - - . ··--1
µoid2 ;
B -
- Z(a2,,,,;2)3/2
... (I)

As i(mz2) iA =,M, where A is the area of the loop


= magnetic moment of the loop,
-- -~o ··-·-;M·--1
, B = 41t · (a2 +x2i312: ... (2)
Field at a point far away from the centre x >>a, ,,
µ 0 2M
... (3)
B = 41t XT '·"
From equation(!), we can get field at the centre ofloop,x-= O

B = µoi
2a
If there are N turns in the loop

B = µoNi
"' (4)
2a

l. Equations (3) and (4) are derived for circular loop; but they can be used forother shane of the loon also
which has symmetry about the axis of the loop. 1
2. In equation (4), N may be a whole number or fraction or decimal number. ~
3. For the. figure shown •. • _.-

N ((l)
-
27t
··., .• -"""- ..
--~-- p
. ;;
0
' B
B = . Fig. 6.JJ
ia
· 4. The magnetic field at a point not on the axis of the loop'ls mathematically difficult to calculate. The field
lines for the circular loop are not circles, but they are closed curves that link the conductor.

Magnetic field due to a circular loop -------!c-------x


0
Fig. 6.12 Fig. 6.13
Field between two similar coaxial circular loops
Let us consider two loops, each having N turns and carrying current i are placed at a distance 2d apart.
. I
i /<-' -
'
i a

Fig. 6.14
Assuming the current is flowing in the sam.e direction in each coil, the magnetic field at a short distance x from
midway point O '.J

2
•· 110Nia [ I I ]
~~. 2 (a2 +x/)3/2 + (a2 +x/)3/2

\I .,.2[·· ,.,· + .··1 . . , ]


·r-- - . Jlon•a . . .
2 [a2 +(d+x)~JJ/2
.
[a2 +(d~x)2J312
I
. : dB
The field will be uniform between the loops, if dx = 0 i.e.,
\· ':,',"\! l\_ i,

11oNia2 [(-3)
2
. (tl+x)_ ~3
. [a 2 +(d+:,:)2-)5 12
(d-x)
[a 2 +(d-x)2J512
]=o -
', - 1 um, .
or (d+.t)[a2+(d-x)2]5i:'; '"' ·(d-x)[a2+(d+x)2]512 ......(i)
. t,•1 ~~ d1
Now [a2:(d+x/J5/2 ,;, [a2'+d2+x2+2xd]5/2
Since X is small, so neglecting ,,2' _.;e have ..

, ... , 2 2•,2[I+-=--=-··
i !::e (a +d )°
., . ,. !·
sxd ]
a2+·d2
• - • !_ ~ . . .
v--.
2xd

. Sxd ]
Similarly [a 2 +(d-x) 2l5/ 2·j ~ ·.c</2 +d2>s,2[1 a2 +d2 ..
. r ' -
::: ~ :.._
Substituting these values in equation (i), we get ., i..i,': ,.H

(d+x) [ l - 25xd 2 ] = (d-x)


. [ I+ 25xd 2 ]
, a +d . a,+d

5d 2 /
or ~~"'7---1, :. d=a/2, or a=2d
(a2 +d2)

and B

or B.= -.. (2)

··..-.............
--............. .
r 0 r

, C I
Fig. 6.15 Variation of magnetic field between the loops.

E:z. 1 A wire carrying current i has the configuration shown injig. 6.16. Two semi-infinite straight
. '
sections, both tangent to the same circle, are connected by a circular arc, of central angle 8, along
the circumference of the circle, with all sections lying in the same plane. What must 8 be in order
for B to be zero at the centre of the circle?
Sol. The field due to the straight parts of the wire
,~/)e
(
2
B1 = [~~ ~]. \ .....•.... :·~-.
out of the plane Fig. 6.16
and field due to curved part of the wire

.
, into the plane of the wire.
2R
The resultant field at the centre to be zero ·'
B1 = B2

or 2[~_!_] I
'
=>8=2rad Ans.
4it R
2R
.,, , ''
6.3 SOLENOID

A solenoid is a wire wound in a cl6sely ;paced spiral over a hollow cylindrical non-conducting core. The wire
is coated with an insulating material so that the adjacent turns physically touch each other, but they are
electrically insulated. ·
dNm ,ub:

s
Solenoid
N
l[]t'.:J6'
Cul section of; solenoid
i - ___J Ii,..---x
' .
i

1
dx
(a) ;-r; · (b) (c)
,.,. '. . ;7
, .,Fig. 6.17
dN. =···ndx
if n is the number of turns perunit length, each c.arrying a current i, uniformly wound round a cylinderofradius
a, then the number oftums in length dx is ndx::rhus the magnetic field at the axial point P due to the element
. 1~ i..\ • -
Tr·. µ0 (ndx)i
''dB, = 2(a2+x2)3/2
The direction of magnetic field is along the axis .o(the solenoid and in the sense of advance of a right handed
screw. From geometry, we have /
' ,\ \
x "=f. acot{l80°-8)=-acot8
\ • !: ' 1'. ,..,.,,~1. . .' • ,/ ~
and dx = acosec28d8 and hence
· µ 0 ni sin 8d8 ~-- -
' ,, dB
' 2. ' . .r,-

f . . i .. ·. ·,
•1 ••
l!,:' • • I ' , I .. •, .82 ,, \!
') '.JI ,., µoni
Total field ··•:s <= -2- sm8d8
•:: Y,i .. 01.

µoni a
,11,= - -l-cosel/

or Ir-B
'

=
'

-
2
µoni
2
-[cos81-cos82_J~
'
l ... (1)
Special cases :
Case 1: Solenoid is of infinite length and·the.point chosen is at the middle
J, •
81 =0,82=1t

c:··---.. '.. . '. ·. . . · ..9 ......... ····o


0..... .....)
Fig. 6.18
~-.:' .' ' . f1.A<3NETlC EFFEC'.f' CJE CURRENT . ffl
Case 2: Solenoid is of infinite length and the point is at the end of the solenoid

01 =it/2' 02 =It
I 1!7-' ,
µon!,:r L
B =
2

c:·····
0 - - .
Fig. 6.19
1··· 0 =1tf2

I. If the length of the solenoid is large compared with its radius, the internal field near its centre is very
nearly uniform and parallelto axis, and the external field near the centre is very small. In practice we take :
the magnetic field inside very tightly wound long solenoid is uniform everywhere and zero outside it.

Fig. 6.10
2. If some material medium is present inside solenoid, the magnetic field inside it is B= µ,µoni ;-whereµ,
is the relative permeability of the medium.

6,4 TOROID OR ANCHOR RING

It is a solenoid of small radius bent round to form a toroid. In an ideal


toroid, the field is confined entirely within the core and is uniform. The
value of magnetic field at any point on the mean circumferential line is
given by
B = µ0 ni

N
if N is the total turns in the toroid, then n
2pR'

µ 0(2:} Fig. 6.10

B = µo ( 2
.NI·
itR/
c---- --- ----- ------,
i µoNi
or lB"'21tR· I
- - · - - - - · ··- -·· __J
where R is the mearirad,us of the ring.
Ex.2 A steady current is s·et up in a cubical network of
I . I
.. ....
resistive wires, connected as injig. 6.22. Find magnetic
field at the centre of the cube. .
.

Sol.. . .The currents in the wires distribute symmetrically about /-···


the dotted diagonal. And so currents of resistances of
one face cancel the magnetic field ofits front face. Thus
i
Bnet = 0. Ans.
I
,l
Ftg. 6.22
Ex.3 A current I flows along a thin wire shaped as a regular polygon with n sides which can be inscribed
into a circle of radius R. Find the magnetic induction at the centre of the polygon.Analyse the
obtained expression atn~ =. ·
. 21t
Sol. In a regular polygon ofn-sides, each ofits side subtends an angle - at the centre of the polygon.
n
It
The angle 8 = - . If B I is the field produced by each side of the polygon, then total magnetic
n
induction

. It . It]
[sm-+sm-
= nµo 1 n n
41t r

where r = Rcos-
It Fig. 6.23
n
µo 2sin it/ n
Thus B n- 1 x - - - -
41t Rcos1t!n

µon/ ~n~.
Ans.
27tR n
For n ~ oo, we can write

B· . [tan1t/n]
= -µo/ 1,m
" 2R n->~ it/ n

µo/
=.
Ans.
2R
Polygon ofinfinite sides is a circle, and so its magnetic induction at the centre is equal to that due
to a circular loop.

Ex. 4 A current/= S.0 A flows along a thin wire shaped as shown injig. 6.24. The radius ofa curved part
of the wire Is equal toR = 120 mm, the angle 24' = 90°. Find the magnetic induc~on of the field at
the point 0.
•• r ·,,
3
Sol. The curved part of the wire is equal to
4 turn. If B I and B2 are the magnetic inductions due to the
curved and straight' parts of the wire, then

and
~ (sin45°+sin45°)
1 ..........4.$'·····. ~..
41t · r
where r = Rcos45° Fig. 6.24
µ 0 / 2sin45°
41t Rsin45°

Thus total magnetic induction,


B

On substituting the values, we get


B = 28µT. Ans.

·Ex, S Find the magnetic induction of the field at the point O in a current carrying wire has the shape
shown in jig. 6.25 a, b, c. The radius of the curved part of the wire Is R, the linear parts are
assumed to be very long.

Jl\_ 0 .

I I
(a) (b) (c)
Fig. 6.25
Sol. (a) Straight part of the wire produces no magnetic induction at 0. So magnetic induction is only
due to the curved part.

' B=
· µo(-!-)1
2 =µo J Ans.
ZR; 4R
(b) The magnetic induction due to upper straight wire at O will be zero and due to lower straight

. µof_., .The .magnellc. .mduct1on


part ts
. due to curved part 1s. .
. Thus total mducllon
.
·
4
. "" : 2R

Ans.
(c) In this case all parts of the wire produc~ non zero magnetic induction at 0.
\J.,.,,,:·,.}' I,."!',•' ",lhr'.1, • li,•J,· l•,•'·l·,•••,, 1 1,,
1
, ·~1
The total induction· = 2B;,raight +B curved
! , ; •I ' • . ! ,, r
\ .
' , _." . µo - I
~
. ~ 2 µof+ 2
471R 2R

"'
.- '
- , .:.1 ,•,. l Ans.

r ; -'· .,
Ex. 6 Find the magnetic induction_ of the field atthe point O if the wire carrying a currentl has the
. I.
shape shown in jig. 6.26 a, b, c. The ra~i!JS ofthe curved part of wire is R, the linear parts of the
wire are.very long. ' ··
.1! • ) ·_, J,,,, _- ' '1, 1 ' · · · ' '

,,
z z
z
, ''

.• '•t., ·~ . ·:
.- -·
/
·~
Fig. 6.26. . -·

Sol. (a) ·The magnetic ind~ction due to-s~ight p: isBsiraight ~ 2[ :~]<-k), and due to curved
part is
,1 ·.~. 1.... [, ' .;. r:I 'fl,.: : ..... ,. j' l .">
ill.•,{."".

8-curved ,•=
',i:
[µ (½ }](-it·
0

2R
/I ·' ' .,

Thus total magnetic lriduction ·


·-::,, :', ••• ' J i . •."l \·.I :1. t.'1 . • . T , _:_. ~n
B = Bsin:ight + Bcurved

·, ,,,
2[·µof_·]{-f>+[µol]<-~{'.' ··
471R · 4R

. :
• ."J Ans.
(b) i jj =
-
Bstraight
-
+ Bcurved
nt 1·
,, ' .

= [ µof (-k)+~(-i)J+[µo( ½}](-i)


. 47tR 47tR 2R

µof • .
= ~47tR[(it+l)i+k]. Ans.

(c) In this case current'in the curved conductor is divided into two parts. The length of upper
I 31
curved part is thrice that of hidden lower part and so currents in them are and
4 4
respectively. Thus

jj = -
Bstraight
-
+ Bcurved

=. ., - -
µof('
1+
47tR,
-k·) ,· ..
'
: Ans.

Ex. 7 A non-conducting sphere ofradius R charged uniformly with surface density a rotates with an
angular velocity roabout the axis passing through its centre. Find the magnetic induction at the
centre of the sphere.
Sol. Take an element in the form of ring of angular width dfJ at a position fJ. The charge on ring

dq = a(dA) = a(21tR sin fJ)RdfJ


Due to i~s rotation, the equivalent current

dq _ a(21tR sin fJ)RdfJ


dt - (21tl ro) L~
,,,_ ---.-·Jt.~···--·· ........
Rsine

roaR 2 sin fJdfJ


Th~ magnetic induction at the centre of the sphere
·'
0
dB

µo 2idA
41t
0

7
2 Fig. 6.27
_ µ 0 2(roaR sin fJdfJ) {( R sin e)2}
41t R3
=
444 !: ., , , .,. ~.'' .. . .
.
~
r.h=J"l. . ' •, . .. "
~~
t:=:::I" """'"".. jt; ~-~.,.

n/2 . ~..,
B µo rooR J sin3 ode
2 0
.\
,;

;,;· µ;
... ,

.
.
rooR
n/2.
JO-cos2 B)sin BdB
0
.

, ' n/2

, '
= Y;, ~~ J.(l-_9qs B)dcos0 2
' .
!. 0 ' .
'.':l~r· ·--·~rc/2'·
= µo
·2 rooRI cos. 0 --3-o
cos3 01 ·.

Ez.8 : A stniight segment_ OC (oflength L meter) of a circuit carrying a.current# A Is placed along the
x-axis (see jig. 6.28). Two Infinity ·1ong straight wire A and B, each extending from
: =_: oo to+ oo , are fU<ed aty =-a·metre itndy =+:a metre respectively as shown injig. 6.28. If the
wire A and Beach carry a current; A into the plane of the paper, obtain the expression-for the
force acting on the segment OC. What will be the force on OC If the current in the wire B is
• I' ~
- reve~~. -.1 - . . ,.,--,;i ~ .. _- :_, 1••• ~ -. ·, :·::- · ; - : ; ;~--:_-~:-. •1 - -, • _--~-.i,: _;:·~ , :.,
, ... ' ' : •ii. " , '. I,.,•:(!),',•
y~ _t ' ,,
,
'\', .
. ' -.

i
. -~- '·
C
'Jl===-"""'==S.----+.:x
-a
z .A
,.
Fig. 6.28 . . . Fig. 6.29
.Sol. The magnetic field due to the wire at a distanc~;,: from_ 0 be obtained as follows: can
If B I and B2 are the magnetic fields of wi_re A. and B ·
respectively then resultant field at P
B 2B1 cosa As(B 1 =B2 )

= 2,µoi cos a
2:rrr

= -~ flo !.x.:. = µ(i ix


21ti- r ·1t·r2

. ' µo . 'ix
lt_ (x2 +a2) a!ongy-axis.
The force on the small length dx of the conductor OC
' '· dF = , i(dtxii)
/di x'B(·,))
" ' I, ' .J • • , ,

',,
. '
The force on the whole conductor
2
- µ 0i (-k >JL xdx
F = 7t ·x2+a2
. 0

·-2 ' '


µo• (-k)[in(L2 +a 2)-lna2 ]
· -2lt .

- µoi2 (-k)ln(L2 +~2}


21t,, 02
Ans.
. ' '

Ex. 9 . Inside a homogeneous.long straight wire of circular cross-section, there Is a circular cylindrical
cavity whose. axis Is parallel to the conductor.axis and displaced relative to it by a distance £ .A
direct current of density J flows along the wire. Find magnetic induction jj inside the cavity,
Sol. By the principle of superposition, th~ required quaniity can be given b•'..

. ii ;= Bo-ii', . .
where B0 is the magneli,c field of the conductor without
cavity, while jj, is the !Dagneti~ field of.the conductor,
which has removed, · · ·
· iio ir. · µo j(lti.Z)r
Here '. Bo = 21t ~ = 21t i,2
This expression can b~ i-epresented in vector form Fig. 6.31

Bo = µo2-(}xi').
..

· Similarly jj• = µo (jxi')


2

..Thus jj B0 -ii•=
.
~[}x(i' -i")]
' ... 2 .· ,,_. ·,·
1
From the ~gure ;: £+r'? r-r'=i,
.. jj = ~(]><£).'
2
Ans.
6.5 AMPERE'S LAW
We know that the lines of magnetic field are continuous and do not arise from any source in the way as lines of
electric field originate from charges. These thus form loops without any beginning or end. This property may be
used to go through the nature of magnetic field and for calculating the field in certain situation of high symmetry.

The field of a long round current carrying wire B = µo. !_. The field lines are concentric around the wire in the
21t r
plane of the paper if the wire is perpendicular to this plane. The line integral around the path of radius r, starting
at any point and returning to the same point is ; B

gis.de = sgid£coso 0

s"'de
'f
= µo ,!...x21tr
21t r
µ0i. Fig. 6.32
Although the above result is derived for the special case of the field of a long straight conductor, the law is true
for conductors and path of any shape. Thus the line integral of the magnetic field B around any closed path is
equal to µ0 times the net current across the area bounded by the path. Hence

[gis.dt = µoi:.~-j
It is called Ampere's law. It plays the same role in magnetics as Gauss's law plays in electrostatics.

The mag~etic field B' on ibe l~ft hand side in Ampere's law is the resultaht field due to all the currents :
existing anywhere while on the right hand side the current i;n is due to the conductors inside the close ·
loop.
2 Let us consider a closed plane curve as shown in figure. If direction of integration is taken along the
path as shown then iI and i3 will be positive and i2 will be negative.
Thus the total current crossing the loop is (i 1 - i 2 + i 3). Any current outside the area is not included in
writing the right hand side of the equation. Thus ;1

gis.d"e = µ0U1 -i2 +i3)


I'
Direction of ·
integration

Fig. 6.33

3. Consider two current carrying conductors, carrying currents


i 1 and i2 in the same direction and a close path as shown in
fig. 6.34. For the close path

Fig. _6.34
t
k
0
-JJ;,.. ~l_"i;~--~;_;
\
C 'EFFEC'li'OE
w;;;,,r;-:r•., ·{n
€URBENTf!.t
1:,; ~~.-:"'-::0_"

Application· oCAmper:e's
• ·
law·
, .I ' , ~ -4
. .
! . ·
. · · :·::,'-,, ·: · ·\
. ·,' . · : ' • .• .
In some cases of practical importance, symmetry considerations make it possible to cse Ampere':
compute the n;,agnetic field caused by, a certain current carrying conductor. Few guiding principles, ati1 _ ~-~•
to those stated for Gauss's law are following.· •
I. If B is tangent to the integration path everywhere and has
the same magnitude on each point of the path, then B
B£ = µoi.
Here f is the length of the close path. Fig. 6.35
2. If jj is everywhere perpendicular to the path or some portion of the path, then that portion of the path
makes no contribution to the line integral.
3. In the integral pB.df: jj is'always the res~ltant magnetic field at each point of the path. In general this
field is caused partly by currents linked by the path and partly by currents outside. Even when no
current is linked by~path the field at eoints on the_P._ath need _not b~ zero. !':' ~~s~! ho~ev::~~ i.d1'
is always zero. -
4. Some Judgement is required in choosing an integration path. Two useful guiding principles are that the
point or points at which the field is to. be determined must lie on the path, and that the path must have
enough symmetry so that the integral can be evaluated. i
Magnetic field. due.
". to long cylindrical
' . -wire
For r?. R:
The inagneti~ field at each point of ~ircular path surrounded the
conductor is tangential. Therefore for circular path I
ps.dt µoi

or p Bdfcos0° µoi

or Bpde = µoi
or Bx2rrr µoi Fig. 6.36

or B = ~_i_
21t r
For r<R:
If the current is distributed uniformly through the cross-section of
B
the wire, then the current inside the path is
· ir 2
-'-xw-2
7!R.2 ="ii
Now pB.dt
o,'---R,,------<:r
2
ir
or µox- Fig. 6.37
R2
µ 0 ir
or B = 21tR2
- •I

For r=R
Ex. 10 Each of the eight conductors in.fig. 6:38 carries 2.0 Aof currentinto or outof the page. Two paths
- . ' l, " • ' - .

,- · are Indicated for the line integral f a.dl. What is the value of the integral for the path (a) atthe
left and (b) atthe right? '' " '·

0 ®

(a) (b)
Fig. 6.38

Sol, (a) - ByAmpereiaw fa.de =µ iin;i~close.l~opi;• .:.(+2-2+2)A=+2A


0
' ' ' '·: .

fa.de =µo(+2)=+2µo, Ans.

(b) l~thisloopi;.=(2-2+2-2)=0, :. fide ~o- -- Ans.


,_

Ex.11 If current density in the conducting wire is proportional to the distance r from the ails ofthe

~~~
' . . .
.
conductor, then find magnetic field atthe position r< R, where Ris the radius of cross section of
' L ,· , ' •, . ,\ ,
'
,, , ' .
'
~ - .
Sol.' Let current density j = kr, where r is the distance faim the axis of the wire. If current in the wire is
i, the_n

R '
= "J jX21trdr
0 '

,, R ..
' Jkrx_21trdr .
0 '
Fig. 6.39

21rkR3' ·>

3 • - ,,

Anl current insi~e the close path of radius r ''


r . r .
Jjx21trdr =fkrx21trdr
0 .0

27tla- 3 ir3 .
= -3-=,Jif

(R3' 3)- ' -


,µo ----
,r . . .
or Bx21tr
,,

I
.. '

' - .• 2
B .:_. µ01 ~
which gives Ans.

I
'.' . - \ :_ . :.;
: ,21t RJ

·----------------·-,,
: • 'I
It is more:usefu!Jo Write Ampere's;Ja\\'.,as follows : I

~--'~ :; < cj;'ll.rll = µoJJM j I


Thin walled hollow current carrying tnhe

For x';?.R, ~--


. 21t r · 1=·
I i
For x<R,- • 1
B 0. Fig. 6.40 l r
B ·<.::::
'l
!
.(: ;R
o'---R-,......,.~---r
I
Fig.. 6.41... . .. .. . -- - .. --·
.'. '~ J.

I
0
1
Ex. 12 Fij. 6'.46 show a' cross-s ectioli ofa large nietai"slie~t carrying an electric current al~ngits iixis.
·L:,(J-..,.::·,~~- ...• --· -- )
The current in a strip of width di is kdl wherekis constant. Find the magnetic field ata point ,

......_,,n· ..:-·- . , . .
X

• -

..
,
- ' •
. (b). ,
.
• • • • • f

. .
· . Fig. 6.4J,
- . -
• j -
. - .. -
.-,

., .
,•. (a)
/
L
Sol. Consider'two ;tripsR and S of the sheet situat;;lsy~etrically on the two side of P. The magnetic
field at P above sheet and below sheet is parallel to sheet as shown in figtlre. There is no field
perpendicular to the sheet. ~

.. R
M '
ELECTRICI'IY & MAGNETISM
'
Now applying Ampere's law to the close path 1-2-3-4-1 as shown infig. 6.43 ;

fa.de = µoiin

2 3 4 I
\ Ja.de+ f ii.di+ fB.df +fB.df =µ (kf) 0
I 2 3 4

or Bf+O+Bf+O

or B Ans.

&MW
' 1. Magne~ic field in this case in independent of x.
2 Large ll)etal C\Irrent carrying sheets: The magnetic field at different position is shown in figure.
I
I
I
\ • I
\ IMMMMMMMMII, I IMNMMM M MMM JI
I
I
IMMl•_!MMMMMM U. I c&OOOOOOOOOOOOOOOOOO@).

• t t I
I
= B1 +B2 =µok I
\
'·, I
Ex. - '-.,,1~. 6.44 shows a cross-section ofa long thin ribbon of width b that is carrying a uniformly
a total current i into t~e page. Calculate the magnitude and direction of the magnetic
'"• plane of the ribbon at a distance 'a' from its edge.
-~ ~dxJ-
1X;i?~-·~81 8-'tXo 81,B?: $k:.ir;ocU2 0'11J 1~b1.-.:i. ....
-1~!HI~wotd bnc l'!.l~rl?. ::rtodc •\ lG bl::iil
"ig. 6.44 .J»rl> orll ol wluoibnoq10q
'Tom P, the curi:ent in the element
ow

,. •o
\
distribute
lie/d jj
.-
.
· atapointp·
In la,,~
-
. (a+b)dx
The total field .B µ0 I J
2n·b · -;-
a

µo _}_It 1<a+b)
21t b nx a

Ans.

---- -----
'
i

I. Unlike the gravitational and :lectric field, thereis no scalar potential associated with the magnetic field. !
· 2. µ 0 a=41t x 10-7 Tm/A is an exact number and not an empirical constant.
i 3. Ampere's law is not independent of the Biot-Savart law. It can be derived from the Biot-Savart law. .Its '
1 relationship to the Biot-Savart law is similar to the relationship between Gauss's law and Coulomb's l
I
'----- __law.______________ __ _ __________ _ ________ _ _____________________ _ •

Long solenoid
For the field at point inside the solenoid, let us consider a rectangular path ABCDA as the path of integration.
This path is particularly simple as : ·
(a) Outside the solenoid, along CD, we fan assume field to be zero as it is many times weaker than that
inside the solenoid ·
D
f B•dt 0
C

(b) Along BC and DA, B is either zero (foroutside parts ofBC and DA) or perpendicular to B (for the parts
inside the solenoid) D c;

=
A
fB•df=O
D
..... • , • • • • , 1, ,,uu .....
A R Jj
(c) Inside the solenoid jj is constant and is along AB. lo--p'--->I

B
f B•df = Bf - Fig. 6.45
A
Now apply Ampere's Law for closed loopABCDA, we get
f fl. d£ µoiio

B C D A
or JB·df+ f B·dR+ JB·df+ Jfl-de
A B C D

or Bf
As ij 0 =(nf)i
,--------,.,.,
~B-~~o~
-
Ex. 14 Consider the current carrying loop shown infig. 6.46 formed ofradial line and segments of
circles whose centres are atpointP. Fi'!!!_the magnitude and direction'of B at pointP.
Sol. Magnetic field by straight parts of the loop is zero because point P lies on their axis. The field
produced by curved parts is

µon1i - µon2i .
......
2a 2b
60° I
As - -n
360° -6- 2
+-~---!--.... ----:~;(·~·

B -
I I) out of the page
µoi (
-;-b b
Ans: Fig. 6.46
12

Ex. 15 A loop, carrying a currenti, lying in thep~ne ofth_e paper, is in the field of a long straight wire
with constant current i0 (inward) as sho)Yn in.fig. 6.47. Find the torque acting on the loop.
. i

/
I

Sol. The field due to current carrying wire is tangential to every point on the circular portion of the loop
and hence the force~ acting on these segments are zero.
Now consider two small elements oflength drat a distance r from the axis symmetrically as shown
infig. 6.48. \
y
A
(dF) (-k)
I
r

2tsin0
.,

A
(dF) (k)

,,-
X

\ ' Fig. 6.48


The magnitude of the force experienced by each element is
dF = Bidr

. (µo . io )idr
21t r
On element I it is into the page and on 2 it is out of the page
d~ = dFx2rsin0
..
(µ2 :i dr )x2rsin 0

Now total torque,

f
µ 0 i0 i sin 0 dr
lt
a

µoioi . o(b -a.)


--s100 Ans.
lt

Ex. 16 A circular loop of radius R is bent along a diameter and given a shape as shown in the figure. One
of the semicircles (KNM) lies in thexz-plane and other one (KLM) in theyz- plane with their
centres at the origin current iis flowing through each of the semicircles as shown infig. 6.49.
(a) A particle of charge q is released atthe origin with a velocity v=-v0i .Find the instantaneous
forcefon the particle. Assume that space is gravity free.
(b) If an external uniform magnetic field B} is applied, determine the forces F 1 and F2 ?n the
semicircles KLM and KNM due to this field and the net force Fon the loop.
Sol. (a) The magnetic field due to the loop at the origin
y

= µoi (-i +k)


4R
Force on charge q at origin K
ft = q(vxB)
z

Fig. 6.49

µoqivo [-ix(-i +k)]


4R , ·

Ans.
(b) In a uniform field the force on the curved loop is fr= i(fxB), where g -4 vector from one
end ofloop to the other end.
i(KMXB) = i[ZR(-k)xB}] = ZBiRi

and i[ZR(-k)xB}] = ZBiRi


F FKLM+FKNM
4BiR}. Ans.

Ex. 17 A circular loop ofradius ,carrying a current iis held at the centre of another circnlar loop of
radius R ( >> r) carrying a current/, The plane of the smaller loop makes an angle 30° with that
of the larger loop. If.the smaller loop is held fixed in this position by applying a single force at a
point on its periphery. What would be the minimum magnitude of this
force?

Sol. The magnetic field a\ the centre of the smaller loop, B = µof .
. ZR

Fig. 6.50
According to right hand screw rule, it is along as shown injig. 6.50. Since smallerloop is placed at
its centre and assuming that field is uniform all over it, the torque exerted by magnitude field.
't = MBsin0

(iitr 2 )x µof sin30°


ZR

µ 0 rcifr 2
4R
if Fis the force at the periphery then, to keep the loop at its position

µ 0 rcifr2
Fxr
4R

or
F = µ0rcifr. Ans.
- - 4R
[(;\+ i-)x,-,
)\f,

Ex. 18 Along straight wire carries a current i. A pa~\j~\f0"aving a positive charge q and mass,;,, kept at
a distancex0 from the wire is projected towar<!fjlwitlta speed v. Find the minimum separation
between the wire and the particle.
' ' j,

Sol. Let the particle be initially atP. The magnetic field jj at any point to the right of the wire is along
the negative z-axis. The magnetic force on the particle is, therefore, in thexy- plane. As there is no
initial velocity along the z-axis, the motion will be in the-')!- plane. Also its speed remain unchanged.
As the field is not uniform, so the path of the particle not remain as circle.
y
We have, v = vxi+vy}

and jj (~~ -~ )(-k)


The force on the particle
·, .r:, i
F qvxB

q(v)+vyj)x(- ~: tr
• 1,.......

Xo~JJ..
•'Flg,i-rM1
r. gni2t!nq
QI .::ta

- (-
qvyµoi}
- - 1+ ( -
2m
qvxµoi
-- J
2m
y iu ~bi2 J~J
0 riJi"N "08 J
.Ioa
1~:S: 51£ 2'.)bi2
,, bnn ::,gt;q
Fx =- qvyµoi
Thus ax ,foil '(d U~J1:>X5
m 2rr.mx

.... (i)

to 22nrn 10.J
where k

dvx dvx dx dvx


Also -=---=v-- ...(ii)
dt dx dt xdx

As V 2 +vy 2 v2 (constant)
X ,muhdiliupo 10'1
On differentiating
2vxdvx + 2vydvy O
10
which gives vxdvx -vydvy ...(iii)
From equations (i), (ii) and (iii), we have

vydvy . 10,c rnon AO!lo lnn1m A 0~ .:K:!I


dx tiuoib orlT .U..l> -~l\ 11[
ui ,,n rl:rn3 .rbn9 fn.~r .o =I"
dx dvy b19it :lii9n~mn ~rU bnF[ (n)
or 1
X k .girnt2 gno! 'thtinitni nA (d)
Initially at x =; x 0 , vy =0 -c:1ih9'f tiu:nb !l 1rndn 9rlJ
At minimum separation from wire, vx= 0, vy =-v .: ~:r1015IU zi 1r.llV/ .tiu::rrb
no gnibn ~31CiJ 91lt 2i tcrl1ff

Thus
X

I-x
Jx
f dvy ',, t ~fl}O~:) ~HJ:t
xo 0 k
or lenxrx. Iv:[
or
Rn.!_
Xo
=
-vk
V
-
or X xoe k

21tmv

x 0e µoqi Ans.

Ex. 19 A copper wire with cross-sectional areas, bentto make three sides ofa square can torn about a
horizontal axis 00'. The wire is located in uniform vertical field. Find"the magnetic induction-if on-
passing a currenfi ihrough the wire the latter deflects by a~ anele e·. . .
' - '
Sol. Let side of square is a, The magnetic field B makes
180° with OP and 0° with QO' therefore forces on these
sides are zero, The force on side PQ = Bia, acts out of
page and A to the plane of the figure, The torque
exerted by field
't = (Bia)xacos8

Let mass of each side is m, then restoring torque

mgx~sin8+mgxasin8+mgx~sin8
, 2 2
2mgasin0
2 (Sap) a sin 0
For equilibrium,
Bia 2 cos0 2Spga 2 sin8

2 Spg tan 8.
or B Ans.
i

Ex. 20 A current of lOA flows around a closed path in a circuit which is in the horizontal plane as shown
in jig. 6.53. The circuit consists 'o.ight alternating arcs of r_adir r 1 ,= 0.08 m "iuid ··.
r2 =0.12 m each. Each arc subtends the same angle atthe centre. ·
{a) Find the magnetic field produced by this circuit at the centre.
(b) An infinitely long straight'wire carrying a current of lOA is passed through the centre of
the above circuit vertically with the direction of the current being into the plane of the
circuit. What is the force acting on the wire atthe centre due to the current in the circuit?
What is the force acting on the arc A CD and the straight segment CD_ due to the current at
the centre?
Sol. (a) The magnetic field due to straight parts of the
r - ~
loop is zero, because centre O lies on their axis.
The total field at the centre .Y

the direction of the field is out of the page, i.e. alongz-axis Fig. 6.53

(
4itxlo-7 )x10(-'
4 0.08 0.12
+-')
= 65.45x10-6 T. Ans.
(b) As the field of the circuit at centre and conductor are anti parallel, therefore force on it is zero.
Further, field due to current carrying conductor at centre will be tangentially to each point of
AC. Therefore force of part AC is zero. While field is perpendicular to part CD, but varies from
CtoD.
Let at any distance x from the centre.

µo_
B
2Jt X
The force on length dx of CD
dF Bi (dx)

_!l:Q_ .!.. )idx Fig. 6.54


( 2Jt X

Total force F

or ( 2x!0-7 )x (10) xi
2
n[ 00.08·12 ]
8.lxJ0-6 N. Ans.
Ex. 21 Two particles each having a mass ,n are placed at a separation din a uniform magnetic field Bas
shown in fig. 6.55. They have opposite charges of equal magnitude q. At time t= O, the particles are
projected towards each other, each with a speed ofv.Supposethe Coulomb force between charges
is switched off.
(a) Find the maximum value of vmofthe projection speed so thatthe two particles do not collide.
V
(b) What would be the minimum and maximum separation between the particles if v = ; ?

(c) At what instant will a collision occur between the particles, if v = 2 vm ?


(d) Suppose v= 2vm and collision between the particles is completely inelastic. Describe the
motion after the collision.
Sol. (a) X X X X X X X X X X X X X X X X X X X X

X X X X

X X /1°\ X X

X
:'t_// (~1: X

X X
, (a) (b)
Fig. 6.55
d
They projected with such a speed vmso that each moves in a path of radius r =
2 .If they will
not collide, then
d mvm
r = -2 = -
qB
-

or
= qBd
Ans.
vm 2m

vm
(b) For V =
2

= qBd
4m'

I mv=~=~
-(~1
r
qB qB 4

Both the particles move on the path of radius : .

d
From the figure Xmin
2
d 3d
and xmax d+-=- Ans.
2 2
(c) When particle velocity CI .,
.v = 2vm = qBd r
m

m(qBd)
r = _m_v =_..cm~_ d
qB qB
V V

. d
2.
They will collide at a horizontal distance Fig. 6.56

sin0

or 0 =
1t radian
6
Therefore each particle travel for time I before collision

1 = I.._a=(Z1tm)x(¾)=
21t qB 21t 6qB 1tm Ans.

(d) When collision is perfectly inelastic, the particles stick together.


The resultant momentum of the combined mass in horizontal direction becomes zero.
Therefore the combined mass moves along a straight line drawn upward in the plane offigure
through the point of collision. Because magnetic field does not exert force on it, qn,t = 0.
Momentum before collision momentum after collision
mv sin0 + mv sin0 (2m)v
,I v sin0

1 V
vx-=-
2 2

Ans.

Ex. 22 Given.fig. 6.57 shows a coil bent with all edges oflength lm and carrying a current of IA. There
exists in space a uniform magnetic field of2 Tin the positive y-direction. Find the torque on the
loop.
Sol. The forces on the edges FG and BC are zero. The forces on the other edges are : I
C

FAB su(i),
FcD su(-1),
FHG su(-1),
FEF su(i).
X Fig. 6.57
, ' I

The torque of the forces FAB and F HG is zero and that of F CD and F EF is zero.
The forces on edges AH and DE are equal and opposite and so constitutes a net torque. Thus

FAll su(-k) and FnE =BiC(k)

Torque,

Ex.-23:.Calculate in brief:
(a) Magnetic field atthecentre of the loop.,
(b) Magnetic field at the centre of the loop.
(c) Current in the loop, so that it tilts about the edge,
(d) Magnetic moment due to the circulating electron.
(e) Condition of balancing a conductor of weight mg.
(I) Condition of balancing a conductor on an smooth inclined plane.
Sol. (a) Magnetic field at the centre of the loop:
e, = 3£2
R1 = 3R2
and so a

B = µon1i1 _ µon2i1 90~\


2a 2a i2. ~

Fig. 6.58

(b) Magnetic field at the centre of the loop:


........

,,..
-,(¾.··
.•
,_:.:--··
Since

(c) Circular current carrying loop placed in B, tilt about point: Fig. 6.59
A
iweight = imagnetic field n

mgxr iAB sin 90°

mg
s
Fig. 6.60
(d) Circulating electron :

e e

ev
Fig. 6.61
21tr
M iA
ev 2 evr
-X1tr = -
27tr 2
e
(e) Balancing of conductor :

mg
µo i1i2
( 21t h
)e ----+-----''-
~ixedl h

i1
Fig. 6.62
(f) Balancing of conductor :
Fmcos 8 mg sine

Bi£cos8 mg sin8

Fig. 6.63

Ex. 24. Eight wires cutthe page perpendicularly at the points shown in jig. 6.64. A wirefabeled with the
integer k (k =1, 2, .... 8) carries the currentk i0• For those with oddk, the current is out of the
page; for those with even k, it is into the page.

Evaluate f B•df. along the closed path in the direction shown.


······
. <-~::::::..·····--········ ··························
:' ... ... -····
!f

(.
\\_ 01

····<::::,.-.-_-_·_._·_·····. ........ . ---···::········......... )


·················•·
Fig. 6.64
Sol. In accordance with Ampere's law

µo (iin)
-'-5µoio, Ans.

Ex. 25 A uniform; constant magnetic field Bis directed at an angle of 45° to thex-axis in .zy-plane: PQRS
is a rigid square wire frame carrying a steady currenti0, with its centre at the origin 0.At time
t=O, the frame is at rest in the position shown in the figure, with its side parallel toxandyaxis.
Each side of the frame is of mass Mand length L.

y
s

···---->:i·l"---+--....,;'"~·

Fig. 6.65
(a) What is the torque 'f about O acting on the frame due to magnetic field?
(b) Find the angle by which the frame rotates under the action of this torque in a short interval
oftimeAt, and the axis about which this rotation occurs (Alis so short that any variation in
the torque d'!ring this interval may be neglected).
Sol. (a) Given, magnetic field

B s(cqs45°i+sin45°])= :h,(t+ ])
L2 k
MxB·

i(Axii)=io[L2 kx :h,(t+J)]
2
ioL -
= - B (-1+1.
o ~)
Ans.
.fi

(b) It is clear from above result that axis of rotation is= (-;j }.e., SQ.

Moment of inertia of the frame about axis of rotation SQ:


Moment of inertia of the frame about an axis passing through O and perpendicular to the
plane ofthe frame.
·l\f~~NETic EFFEcro~;c"RRENf M
4[ ~~ +M(½J]
2
2
=~ML

By perpendicular axis theorem


2/SQ

/SQ

We know that ,: = Ia

Thus angular rotation

0 _l_a( At ) 2
2

_I_(]_ ioB ) 81 2
2 2 M

Ans.

Ex. 26 A rectangular loop PQRSmade from a uniform wire has length a, width band mass m. It is free to
rotate about the arm PQ, which remains hinged along a horizontal line taken as the y-axis (see fig.
6. 66). Take vertically upward direction as the z-axis. A uniform magnetic field B=(3/ + 4]) B 0
exists in the region. The loop is held in the-'J'·plane and a current/is passed through it. The loop
is now released and is found to stay in the horizontal position in equilibrium.
z

{)\t. .
- - nm2n:>1 ... _
~

Fig. 6.66
(a) What is the direction of the current in PQ?
(b) Find the magnetic force on th~ aim)lS•. ___~
.(c) Find the expression ofl i'IJ~r1,0s of B 0 , ~' b and.m.

'"·" .11,H
Sol. (a) For the equilibrium of the loop in horizontal plane, the net torque on it must]le zero. Thus if
'fmg is the torque exerted by the weight and 'tmag is the torque due to magnetic force, then

ifmg +ifmag 0

tmag -tmg ...(i)

Here tmg (mgf j)


and tmag = MxB
I AxB

= 1[aix(-b])]x(3i+4k)B 0

(Assuming direction of current in the loop clockwise.)

1[ab(-k)]x (3i + 4k) B0


= -3abl] . .
Substituting these values in equation (i), we get

-3ablj = -( mgf}
mg
I = 6bBo Ans.
Positive sign with I indicates that direction of current in PQ is correctly assumed.
(b) Force on the arm RS is given by
fr (1exs)
i(-b])x(3t+4,qB0
1Bo(-4t+3k)N Ans.

,, (c) Ithasbeenobtained_inpart(a): __ _

Ex. 27 Ring of radius R having uniformly distributed charge Q is mounted on a rod suspended by two-'
identical strings as shown in jig. 6.67. The tension in stringi"in equilibriunils T0• Now a ~ertical
magnetic field is switched on and the ring is rotated at constant angular velocity ro. Find the
. . ·- -- - . . . - .. 3T,
maximum ro,vith which the ring can be rotated if the strings can lvithstand a maximum tension T
~----D-----~ '
Sol. Initially mg ... (i)
If robe the frequency corresponding to the breaking of the string, then current in the ring

Q Q Qro
i = r=Zlt= 21t
0)

Magnetic moment of the loop


M iA
The torque experiences now
= MBsin90°
t
= iAB. V; IB
If T1 and T2 are the tensions in the strings now , then Fig. 6.68

(Ji -Tz) ~ = iAB

2iAB
or ...(ii)
D
Also T1 +T2 = mg ... (iii)
Solving equations (ii) and (iii), we get

and T2 ~g-C:)
3T0
Tl (Given). Hence
2
3T0 2To + roQ xitR2 xB
2 2 27tD

roQBR 2
or To
D

DTo
~r 0) Ans.
QBR 2 .

/
/~-~ ; '-;L'.i:
agnet1sm £,;·i~!:,,,;; ·McQtypel
~---- ···--·- &~ 6.1
''Z_Y .
II
1. The figure shows three long, straight, parallel, (a) I III
equally spaced wires with identical currents
(b) II
either into or out of the page. If FA, F8 and Fe
are the 1 magnitudes of forces on them (c) III
respectively, then
(d) IV
A B C
0 9 181 5. In a coaxial, straight cable, the central conductor
(a) FA >FB >Fe (b) FA ,. =FB
' >Fe and the outer conductor carry equal currents in
opposite directions. The magnetic field is zero
(c) FB >Fe >FA (d) Fe >FA >FB
(a) inside the inner conductor
2. The figure shows three equal currents i (two
(b) inside the outer conductor
parallel and one antiparallel) and four closed
(c), in between the two conductor
loops. Which loop has greatest value off B•df? (d) outside the cable.
6. A current-carrying straight wire is kept along the
:;-Loop2 axis of a circular loop carrying a current. The
straight wire
(a) will exert an inward force on the circular loop
(b) will exert an, outward force on the circular
loop
(c) will not exert any force on the circular loop
(d) will exert a force on the circular loop parallel
to itself.
I,
7. The magnetic field at the origin due to a current
(a) 1 (b) 2
(c) 3 (d)° 4 element idf placed at a position i' is
3. The figure shows four arrangements of circular
loops of radius r or 2r, centered on vertical axes _µ 0 idfxi' µ 0 idfxi'
(perpendicular to the loops) and carrying (a) (b) 41t . r3
41t
identical currents in the direction indicated, in
which arrangement, the magnitude of the net _):'.Q_ i'xidf.
magnetic field at the dot is the greatest : (c) 41t r3 (d) none of these

8. Two infinitely long conducting wires carry


currents, as shown in figure. If the horizontal
wire carrying current produces a magnetic field
B at the point P(-2d,-d,0), then the resultant
magnetic field at this point is

(a) -2Bk
I II Ill IV
7
(a) I (b) II (b) -,/2. Bk
(c) III (d) IV
2A i
4. Figure shows a uniform magnetic field B and (c) ,/2. Bk di
four straight-line paths of equal lei:igths. Which
(d) zero ):.... "iif" .. 4A
path has the greatest value of f B•dR
9. A length£ of wire carries a steady current/. It is 13. A vertical wire kept inzx-plane carries a current
bent first to form a circular plane coil of one tum. from Q to P (see figure). The magnetic field due
The same length is now bent inore sharply to to current will have the direction at the origin 0
z
give a double loop of smaller radius. The magnetic along p
field at the centre caused by the same current is
(a) OX
(a) a quarter of its first value
(b) unaltered (b) OX x'---0 I X
(c) four times of its first value
(c) OY
(d) a halfofits first value
10. A vertical straight conductor carries a current (d) OY I
z' Q
vertically upwards. A pointP lies to the east ofit
14. PQRS is a square loop made of uniform
at a small distance and another point Q lies to
conducting wire the current enters the loop at P
the west at the same distance. The magnetic field
and leaves at S. Then the magnetic field will be
atP is
(a) greater than at Q
Q R
(b) same as at Q
(c) less than at Q
(d) greater or less than at Q depending upon
the strength of the current
11. An infinitely long straight conductor is bent into
the shape as shown in the figure. It carries a
current of i ampere and the radius of the circular
loop is r metre. Then the magnetic induction at
its centre will be (a) maximum at the centre of the loop
(b) zero at the centre ofloop
(c) zero at all points inside the loop
(d) zero at all points outside of the loop
15. An electron moving in a circularorbit ofradius r
makes n rotation per second. The magnetic field
produced at the centre has a magnitude of
() µ 0 2;(1t+!) (b) µo 2i (1t-l)
a 1t r 4,c r µ0 ne µonze
(c) zero (d) infinite (a) (b)
2r 2r
12. The magnetic induction at the centre O in the
figure shown is
(c)
µ0 ne· \ (d) zero
21tr
16. Two long straight wires are set parallel to each
other. Each carries a current ; in the same
direction and the separation between them is 2r.
The intensity of the magnetic field midway
between them is
(a) µ 0i/r ·
µ 0i ( _!_ __
I ) µoi ( _I_+_! ) p
(a) (b)
4 R1 R2 4. R1 Rz (b) 4µ 0i/ r

(c) zero
(c) -µoi ( R1-R2 ) (d) -µoi (R1+R2 )
4 4 (d) µ 0i/ 4r
I • i')

17. A part ofa long wire carrying a current i is bent


into a circle of radius r as shown in figure. The
net magnetic field at the centre O of the circular
loop is
,p
J.L p
(2)
(I)

(a)
,l

µoi
4r
-
(b)
-
µoi
2r
B

~ (3)

(c) µoi (it+!) (d) µoi(1t-!)


21tr
(a) (-%) =(% ) : (3:-½)
21tr
18. The earth's magnetic field at a given point is
(b) (-%+1}(%+1}(3;.+½)
0.5 x I o-5 Wb-m-2. This field is to be annulled
by magnetic induction at the center of a circular
It It It
conducting loop of radius 5.0 cm. The current (c) --·-·3-
2. 2. 4
relJuired to be flown in the loop is nearly
(aj 0.2A (b) 0.4A
(c) 4A (d) 40A (d) (-%-1}(%-¾}(3:+½)
19. The magnetic field due to a current carrying
circular loop ofradius 3 cm at a point on the axis 22. An infinitely long conductor PQR is bent to form
at a distance of 4 cm from the centre is 54 µT. a right angle as shown. A current/ flows through
What will be its value at the centre of the loop PQR. The magnetic field due to this current at
(a) 250 µT (b) 150 µT the point Mis H1• Now another infinitely long
straight conductor QS is connected at Q so that
(c) 125 µT (d) 75 µT
the current is //2 in QR as well as in QS, The
20. Two concentric coils each of radius equal to
current in PQ remaining unchanged. The
21t cm are placed at right angles to each other. 3
magnetic field at Mis now H2. The ratio H 1I H 2
ampere and 4 ampere are the currents flowing in
H
is given by
each coil respectively. The magnetic induction
M
in weber / m2 at the centre of the coils will be
(a) -I
2
(µ 0 =41tx10-7 Wb/A.m) 90°
(b) -00-
I
p Q S +oo
(a) 5xI0-5 (b) ?xI0-5 90°
10-5 2
(c) ]2x1Q-5 (d) (c) R
3
21. The magnetic field at the centre ofa circular coil
of radius r is 1t times that due to a long straight (d) · 2
wire at a distance r from it, for equal currents. 23. A nori-planar loop of conducting wire carrying a
Figure here shows three cases : in all cases the current I is placed as shown in the figure. Each
circular part ·has radius r aild straight ones are of the straight sections of the loop is of length
infinitely long. For same current the B field at the 2a. The magnetic field due to this loop at the
centre P in cases I, 2, 3 hav~ the ratio pointP (a, 0, a) points in the direction
, ''
·, 26. A steady current i flows'in a small square loop of
·;,, . .wire ofside'l' in a horizontal plane. The loop is
y now folded about its middle such that half of it
··lies in a vertical plane. Let µ 1 and µ 2
I I~

respectively denote the magnetic moments due


to the current loop before and afte; folding. Then

X
(a) µ 2 =0
(b) µ 1 and µ 2 are in the same direction
2a

(a) .fi.
l ( -J+k
' ') (b) - I ( -J+k+i
, , ')
(d) ~=(Jz)
J3. . . ; . 27. Which of the following graphs shows the

(c)
Ili+j+k
·,13· , ~) (d) I C
.fi. i+k')
variation of magnetic induction B with distance
r from a long wire carrying current

24. A long straight wire along the z-axis carries a


current/ in the negative z direction. The magnetic
(•) B ~ 0) B ~
field vector B at a point having coordinates
(x,y) in thez= 0 plane is , r r

(•) 'L
µ0I(y1-x}) µ0I(xi-y])
(a) ·21t(x2+y2) (b) 2x(x2 + i)
« B~
,. ,.
28. .Two long parallel wires are at a distance 2dapart.
.They carry steady equal currents flowing out of
25. Wires I and 2 carrying currents i I and i 2 the plane of the paper, as shown. The variation
respectively are inclined at an angle 9 to each · of the magnetic field B along the 'iinexx' is given
other. What is the force on a small element di of 'by . ·,
wire 2 at a distance ofr from wire 1 (as shown in

_lltll,
(btux
figure) due to the magnetic field of wire 1 B
t ·IX
(a)
~1.-d"""',
d~ir

1w··- . f
(a) µo i;i2 di tan 9 (b)
27tr

µo ''di
- 1··di
-µo 112
21tr
·
sm

µo ··di Sin
·
e
!
e
(c)
x.

.
71
I d .d
•x'
(
I:·,
(d) \lU·Ir
X•

d d
•x'

(c) -1112 cos 9 (d) --Ztlz


27tr 47tr
29. Two parallel beams of protons and electrons, given to the slab after complete discharge is
carrying equal currents are fixed at a separation (given that the capacitance of the capacitor is C
d. The protons and electrons move in opposite and the plates are separated by a distance d).
directions. P is a point '?n a line joining the
beams, at a distance x from any one beam. The Q, C
magnetic field atP is B. If Bis plotted againstx, ,_-,-i
which of the following best represents the ~,. l d
resulting curve
1
----'---'~ll
,.,M ;,D,
.___
-Q,

(a) CQrfJ (b) dBQ0


(c) zero (d) infinity
33. A wire oflength L metre carries a current I ampere,

~~~ ~
n tC
(c) (d)
is bent in the form of a circle. The magnitude of
its magnetic moment in MKS units is :

(a) 41t!L2 (b)


21i!-
7t
30. An infinitely long wire has a.charge 'A. per unit
length. The wire moves along its axis with a Ji!- IL2
velocity v (v<< c, the velocity oflight). The ratio (c) (d) 47t
1t
of the magnetic field to the electric field at a point
r from the wire is : 34, A charge q is moving with a velocity v1 = l i mis
at a point in a magnetic field and experiences a
(a) Proportional tor
forceF 1 =q (-lj+ 1 k) N. If the charge is moving
(b) inversely proportional to r with a velocity v2 = 1 j mis at the same point, it
(c) directly proportional to 'A. experiences a force F 2 = q (I i - 1 k) N. The
(d) independent of r magnetic induction B at that point is :
31. A thin uniform ring ofradiusR carrying uniform (a) (li+lj+lk)Wb/m2
charge Q and mass M rotates about its axis with (b) (li-lj+lk)Wb/m2
angular velocity ro. The ratio of its magnetic (c) (-li+ lj-lk)Wb/m2
moment and angular momentum is :
(d) (Ii+ lj-lk) Wb/m2
M
(a) _g_ (b) Q.
M

Q M
(c) (d) 2Q
2M
32. A slab of resistance R is inserted between two
parallel plates of a capacitor charged to Q0 . The
capacitor is discharged through the solid. A
magnetic field is present throughout and is out
of the plane of the paper. The total momentum
/~\ ""\-.---,-,-=--------
a'gnetism
'\_J
MCQ Type 2 . &UledJe 6,2
') 11

I. Two coaxial solenoids I and 2 of the same length Passage for Q. 5 to Q. 7


are set so that one is inside the other. The number Take a tightly-wound solenoid of radius a and length
of turns perunit length are n 1 and n2 . The current. R, the numberof tum's perunitfongth in it is n. It carries
i I and i2 are flowing in opposhe directions. The current i. Consider a small element oflength dx of the
magnetic field inside the inner coil is zero. This solenoid at a distance x from one end. This contains
is possible when hdx turns and can be assumed as a current carrying
· [more than one option may be correct] loop. The magnetic field due to whole solenoid will be
(a) i1 ,;, i and n1 _= n2 the sum of magnetic field due to such elements.
2

(b) i1 =i2andn1 ,;,n2 5. The magnetic field due to this solenoid at the
(c) i1 =i2 andn1 =n2 centre of its axis is :
µo; . µ0 n i
(d) i1n1 = i2n2 (a) (b)
2a 2a
Passage for Q. 2 to Q.4 i'
µ 0 ni µ0n i
A coaxial line carries the same current i up the inside
conductor
,
of radius a as down the outer conductor ofI
inner radius b and outer radius C.
(c)
~ . 2
(d)
pj; 2

6. The magnetic field due to the solenoid at the


centre of its axis' in the situation a >>f. is :
µ0 n i
(a) (b) µ0 ni
2
. . µ0 nif µ0 ni£
2. The magnetic field at a distance r from the axis, (c) (d)
for r<a is: 2a a
7. The magnetic field due to the solenoid at the
µo .!..'.:. centre of its axis in the situation a<< eis:
(a) zero (b) 2it a 2
µ 0 11 i
µo !.!!_ ):Q_ i (a) zero (b) -2-
(c) 4it r 2 (d) Zit r
(c) µ 0ni (d) infinite
3. The magnetic field at a distance r from the axis,
fora<r<bis: 8. Six point charges, each of the same magnitude q,
are arranged in different manners as shown in
(a) zero (b) µo i Column-II. In each case, a point Mand line PQ
4it r
passing through M are shown. Let E be the

(c)
1:Q_i (d)
µo .!..'.:. electric field and Vbe the electric potential at M
Zit r Zit a 2 (potential at infinity is zero) due to the given
4. The magnetic field at a distance r from the axis, charge distribution when it is at rest. Now, the
forb< r<cis:
whole system is set into rotation with a constant
):Q_ i µo r2 -b2 angular velocity about the line PQ. Let B be the
(a) (b)
Zit r Zit c2-b2 magnetic field at M and µ be the magnetic

µo i [·
(c) - - I
2 b2] \
moment of th'e system in this condition. AsSlline
each rotating charge to the .equivalent.to a steady
Zit· r . :2 =bl _ (d) .none of these
·current.
Column-I Column-II
(A) E=O Charges are at the corners of a regular hexagon. M is at the
(p) centre of the hexagon. PQ is perpendicular to the plane of the
(B) V;<O hexagon.

(C) B=O
Charges are on a line perpendicular to PQ at equal intervals. M
(D) µ ;< 0 (q) is the mid-point between the two innermost charges.

Charges are placed on two coplanar insulating rings at equal


intervals. M is the common centre of the rings. PQ is
perpendicular to the plane of the rings.
(r)

Charges are placed at the corners ofa rectangle of sides a and


2a and at the mid points of the longer sides.Mis at the centre
of the rectangle. PQ is parallel to the longer sides.

e-
(s)

I
p

9.
(t) + e +
~----1---l.--1------I'-
,. M-
Q
Charges are placed on two coplanar, identical insulating rings
at equal intervals.Mis the mid-point between the centres of the
- rings. PQ is perpendicular to the line joining the centres and
coplanar to the rings.
Two wires each carrying a steady current I are shown in four configurations in Column I. Some of the
resulting effects are described in Column II. Match the statements in Column I with the statements in
column II and indicate your answer by darkening appropriate bubbles in the 4 x 4 matrix given in the ORS.
Column/ Column/I
(A) Point Pis situated midway between (p) The magnetic fields (B) at P due to the currents
the wires. in the wires are in the same direction.

(B) Point Pis situated at the mid-point of the (q) The magnetic fields (B) at P due to the currents
line joining the centers of the circular in the wires are in opposite directions.
wires, which have same radii.

{):{)
(C) Point Pis situated at the mid-point of the (r) There is no magnetic field at P.
line joining the centers of the circular
wires, which have same radii.

(D) Point P is situated at the common center (s) Thewires repel each other.
of the wires.

@
['
,,;
......

Read the iw<Ystatements carefully tl'> mark the correct option out of the options given below. Select the right
·choice.··
(a) Jfbol:Kihe statements are true,and the Statement - 2 is the correct explanation, of State111ent • 1.
(b) Ifb~thithestatements are true butState,nent - 2 is northe·correct explanatfon of the Statement- I.
(c) IfState,nent-1 true·butStale111e/tt-2 is false.
(d) IfState1neni-1 is false butStatement-2 is true.

1. State,nent. 1 5. State111ent- 1
Alternating current shows no magnetic effect. The magnetic field due to a very large current
State,nent - 2 carrying loop is zero at its centre.
The magnetic field produced by alternating .Statement- 2 ·
. current i~ complete cycle is z;ero_.
2. State,nent - 1 Magnetic field at the centre ofloop is, B = µoi .
2R
A direct current flows through a thin conductor
_ produces magnetic field only outside the 6. Statement - 1
conductor. Figure shows cross-sections of two long straight
State111ent - 2 wires; the left-hand wire carries current in directly
There is no flow of charge carriers inside the out ofthe page. Jfthe magnetic field due to the
conductor. two currents is to be zero at P, the current i2 must
3. Statement - 1 be into the page. ' ·
A current i flows along the long straight and thin p
walled pipe. Then magnetic field at any point
inside the pipe is zero. Statement - 2
State111ent - 2 i2 > i1,
7. Statement - 1
fs.dl= µoi. Figure shows a current carrying circular loop.
4. State111ent - 1 '' The,magnetic field at the centre ofloop is zero.
Ampere's·law used for the closed loop shown in
figure i; written as f B•d£= µ 0( ii - i;). Right
i ' i
side ofit does not include i3, because it produces
no magnetic field at the loop.

State111ent - 2
Magnetic field at the centre ofloop is,
Statement - 2
The line integral of magnetic field prnduced by B= µoni
i3 over the close loop is zero. 2R
8. Statement - 1
Figure shows two identical currents i and a
Forthepath2, pB•df=-µo.
close path encircling them. For the close path 11. Statement- I
Figure shows a current carrying conductor and
pii-de= 2;.
a close path. For the close path pB•df =0.

Statement - 2
Statement- 2
For the close path pB•df = 0. For the close path, the magnetic field at each
9. Statement- 1 point on the path is zero.
The force between two parallel current carrying I 2. Statement - I
conductors carrying currents in same direction The magnetic field of a long solenoid is ~ni at
is attractive because there is no electrical its centre.
interaction between them. Statement - 2
Statement - 2 The magnetic field at any point outside the
The force between two electrons streams moving solenoid on its bisector is zero.
in the same direction repulsive because there is 13. Statement - I
no magnetic interaction between them. Two long current carrying wires are placed
10. Statement - 1 perpendicular to each other. The riet force exerted
Two square conducting loops carry currents 3A by one wire on the other is zero.
and I A as shown in figure.

For the path I, pB•df=-2µo.

~. 3
A
/!A Path!
/4.

Q0
Pa tl1 2 --•:

Statement - 2
..... ..
... .
." • • ·:
~
Statement - 2
The net torque exerted by one wire on the other
is zero .

!.
---
----
~ &:telld4e 6, 1
A-··"·-
---- .. ~------ . . . - --
I 1 I. (c) I ! I _(a) I _13 J (d) :I_ 19 . I (a) l 25 I (c) I 31 ~
;I 2 I (cl I 8 ! (a) [ 14 J (b) J 20 _I (a) i 26 _ J (c) j 32 ~
I 3 I . (dl I 9 ! (c) j 15 j (a) [ 21 I (a) I 27 '[ (c} J 33 ~
I 4 \ (a) I 10 I (b) I 16 I (c) I 22 I (c) I 28 I (b} :1 34 [7a)
I 5 I (dl I 11 I (b) i 17 I (c)
:I
I 23 I (d) ! 29 ·1 (c) r-:
I 6 [ (c) J 12 j (a) [ 18 I (b) 24 [ (a} [ 30 [ (d} ~

=--c--=:-,.,..--- ---· -- . .
~~Type2
,~,.,
A ~ &:telld4e 6,2
··- -- ·- . .

·:J __ ! _jl _(c)(~! ,I_ - 3 - J (c) r 5 'I (d) 'I_ 1 __ f (c'.


.I 2
:1 (b}
I 4 r--Tcl I;.=...c:---:._:
6 I (C)

8. (A)~ (p, r, s); (B) ~ (r, s); (C)~ (p, q, t); (D) ~ (r, s)
9. (A) --->q, r; (B)---> p; (C) ---> q, r; (D)---> q, s

•esm L/
,J'ype 3 .,
; --
i~ -- -
&:telld4e 6. s

ii~~~J~ ;j·-·;I ;· :[.


1 -_(d) _ I_ 7 ~, (a)_ ij 10 (b) I_ 13 J__(c)
iI ~ _j (c) __ :I 5 :J
_(a) I 8 iJ (d} :JT1 ~ I~ J
i[\_ 3 _J ___(~L _lj_ -~- jJ ___ Ja). J_ 9 __ I _jc)_ .~112 (bl " l __ L _ __
&¾elld4e 6. 4
1. Consider a long, straight wire of cross-sectional area A carrying current i. Let there be n free electrons per
unit volume. An observer places himself on a trolley moving in the direction to the current with a speed
i ,
v = -A and separated from the wire by a distance r. Find the magnetic field as seen by the observer.
ne ·
µoi
Ans: B=--.
• 2rrr
2. In Bohr model of hydrogen atom, the electron circulates around nucleus on a path of radius 0.51A at a
frequency of6.8xJ0 15 rev/s. Calculate the magnetic field induction setup at the centre of the orbit What

is the effective dipole moment? Ans: 14 ~ , 9.0xlo-24 A-m 2


m
3. At a certain position, earth's magnetic field of 39 µTis horizontal and directed due north. Suppose the
net field is zero exactly 8.0 cm above a long, straight, horizontal wire that carries a constant current.
What are (a) the magnitude and (b) the direction of the current? Ans: (a) 16 A(b) west to east.
4. A long, straight wire carries a current i. Let B I be the magnetic field at a point Pat a distanced from the
wire. Consider a section oflength Cof this wire such that the pointP lies on a perpendicular bisector of
the section. LetB2 be the magnetic field at this point due to this section only, Find the value of d/C so that
B2 differs fromB 1 by I%. Ans: 0.07.
5. A square loop ofwire of edge length a carries current i. Show that, the magnetic field at the centre of loop
. B 2,fi, µoi
IS - ·
!ta
6. A square loop of wire of edge length a carries current i. Show that the magnitude of the magnetic field
4 0
produced at a point on the axis of the loop and a distancex from its centre is B µ ia' •
2 2 2 2 112
n{ 4x +a )(4x + 2a )

7. ln figure, a straight wire of length L carries current i. Show that the magnitude of magnetic field atP, a

perpendicular distance R from one end of the wire is B = µoi L l/ •


2
47tR ( L2 + R2)

lp
R

-; L
l
8.

the current at point Pis B = - -µoi


-.
Brra
I
In figure, a straight wire oflength a carries a currenti. Show that the magnitude of the magnetic field by

Ji a-p

1r
9. (i) -A charge of I coulomb is placed at one end ofa non-conducting rod·of length 0.6 m. The rod is
rotated in a vertical plane·about a horizontal axis passing through the ·other end of the rod with
angular frequency 104 1t rad/s. Find the magnetic.field at a point on the axis of rotation at a distance
of0.8 m from the centre of the path.' · • · • ,
(ii) Now half the charge is removed from one end and placed on the other end. The rod is rotated in a
vertical plane about horizontal axis passing through the mid point of the rod with the same angular
frequency. Calculate the magnetic field at a poini on the axis at a distance of0.4 m from the centre
ofrod. Ans: (i) 11.3 gauss (b) 22.6 gauss.
10. A pair of stationary and infinitely long bent wires are placed in thexy-plane as shown in figure. The wires
carry currents of i = IOA each as shown.
The segments L_ and Mare along the x-axis. The segments P and Q are parallel to the y-axis such that
OS= OR= 0.02 m. Find the magnitude and direction of the magnetic induction at the origin 0.
C •'

Y.1._·""
: ,.·o
R I :- i
oo--··········" _ _ _ _ __._.....______________ x
""

L 0! S M

p i

. ., · _.. "" . _ . Ans:lo-4Wb!m2._


11. In the circuit offigure, the curved segments are arcs of circles of radii a and b with common centreP. The
' . .
straight segments are along radii. Find the magnetic field B at the point P, assuming a current i in the
circuit.

' µoi8 ( I I )
Ans: 41t, -;;-,; , outofpage,
. p ' .
12. Figure shows tlie cross-section of a long conducting cylinder with inner radius a = 2.0 cm and outer
radius b =_ 4.0 _cm. The cylinder carries a current out of the page, a1.1d the current density in the
cross-section is given by J = cr2, with c = 3.0x I 0 6 Aim4 and r in-metre. What is the magnetic field B at
apoinuhat is3.0 cm from the central axis of the cylinder?

Ans:B=2.ox10-5 T.

13. A long wire carrying a current i is bent to form a plane angle 8. Find the magnetic field Bat a point on the
bisector of this angle situated at a distance x from the vertex. ,
-· µ 0r e
Ans: --cot-
2itx. 4
l•L Four long copper wires are parallel to each other, their cross-sections forming the corners of a square
with sides ~ = 20 cm. A 20 A current exists in each wire in the direction shown in figure . .What are the
magnitude and direction of B at the centre of the square?

Ans: 80 µT up the page.


15. Find the magnitude and direction of magnetic field induction at point O due to current carrying wire
shaped in figure. The current I= 5.0A, R = 12.0 cm and angle 0 = 90°.
(j

I 0

..·••··
. . . a··
·"· . f?.
···....
E F Ans::,!.8xJ0-5 T.
16. Two long parallel wires carrying current 2.5 and I ampere in the same direction (directed into plane of the
paper) are held at P and Q respectively such that they are perpendicular to the plane of the paper. The
points P and Qare located at a distance of 5 metre and 2 metre respectively from a collinear pointR(fig.).
p Q R V
®·············®·· ..··········~-__,.,__ _ x
11 = 2.SA / 2 = IA

B
f,- 3m --+----2m --I
(i) An electron moving with a velocity of4x 10+5 mis along the positivex-direction experiences a force
of magnitude 3.2x 10-20 Nat the pointR. Find the value ofI.
(ii) Fmd all the positions at which a third long parallel wire carrying a current of magnitude 2.5 A may be
placed so that the magnetic induction atR is zero. Ans:(i)4A(ii)± Im
17. The current density inside a long, solid cylindrical wire of radius a is in the direction of the central axis

and varies linearly with radial distance r from the axis according to J = J 0 !:... Find the magnetic field
a 2
inside the wire. Ans: µoJor
3a
18. Figure shows a cross-section of a long cylindrical conductor
of radius a containing a long cylindrical hole of radius b.
The axes of the cylinder and hole are parallel and are a distance
d apart; a current i is uniformly distributed over the shaded area.
(a) Use superposition to show that the magnetic field at the centre of the hole is
µ 0 id
~,!~:
B :

I
(b) Discuss the two special cases b = 0 and d= 0.
(c) Use Ampere's law to show that the magnetic field in the hole i~ uniform.
0

19. Figure shows a square loop ofedge a made ofa uniform wire. A currenti enters the loop at the point A and
leaves it at the point C. Find the magnetic field at the point P which is on the perpendicular bisector of
·a
AB at a distance
4 from it.
C
D----.:::.-,..._

p
............ ·1~14
A B
2µoi ( I 1 )
Ans : ~ ,JS - 3.JG , out of page.
20. Four long, straight wires, each carrying a current of 5.0 A, are placed in a plane as shown in figure. The
points of intersection form a square of side 5.0 cm.
(a) Find the magnetic field at the centre P of the square.
(b) Qp Q , Q and Q are points situated on the diagonals of the square and at a distance from P that
2 3 4
is equal to the length of the diagonal of the square. Find the magnetic fields at these points.

QI Q,.
·····...___ ...•···•·•
•·• ..··
. .:::1. . .
··········.. Q,

Ans:(a)Zero(b)Q 1 : l.lx!0-4T, 8,Q2 :zero;Q3 : l.lxl0-4T,©; andQ4 :zero.


21. A long straight wire carries a current of IO A directed along the negative y-axis, as shown in figure. A
uniform magnetic field B of magnitude 10-6 Tis directed parallel to the x-axis. What is the resultant
0
magnetic field at the following points?
(a) x=O,z=2m;
(b) x=2m,z=O; z
(c) x = O,z=--0.5 m.

Ans: (a) zero (b) 1.41 x 10-6T,45° inxy-plane (c) 5:oox10--0T, +x-direction.
22. A 200-tum solenoid having a length of25 cm and a diameter of 10 cm carries a current of0.30 A. Calculate
the.magnitude of the magnetic field B inside the solenoid. Ans: 0.30 mT.
23. A toroid having a square cross section, 5.00 cm on a side, and an inner radius of 15.0 cm has 500 turns
and carries a current of0.80 A. What is the magnetic field inside the toroid at
(a) the inner radius and (b) the outer radius of the toroid? Ans: (a) 533 µT (b) 400 µT.
24. A long solenoid with I 0.0 turns/cm and a radius of7.00 cm carries a current of20.0 mA. A current of 6.00
A exists in a straight conductor located along the central axis of the solenoid. (a) At what radial distance
from the axis-will the direction of the resulting magnetic field be at 45.0° to the axial direction? (b) What
is the magnitude of the magnetic field there? Ans: (a)4.77 cm; (b) 35.5 µT.
25. A capacitor ofcapacitance 100 µFis connected to a battery of20 volt for a long time and then disconnected
from it. It is now connected across a long solenoid having 4000 turns per metre. It is found that the
potential difference across the capacitor drops to 90% of its maximum value in 2.0 second. Estimate the
average magnetic field produced at the centre of the solenoid during the period Ans: !61tx 10-8 T.
26. A current/ flows along a lengthy thin-walled tube of radius R with longitudinal slit of width h. Find the
induction of the magnetic field inside the tube under the condition h << R.

Ans : B = µo hl , where r is the distance from the cut. ·


41t2Rr
27. A current/flows in a long straight wire with cross-section having the form ofa thin half-ring ofradiusR
(figure). Find the induction of!he magnetic field at the point 0 ..

Ans: B= µof.
it2R
28. Find the current density as a function of distance r from the axis ofa radially symmetrical parallel stream
of electrons if the magnetic induction inside the stream varies at B = br", where b and a are positive

constant. Ans: J(r)=(: }I+a)r"-


0
1

29. Find the magnetic moment ofa thin round loop with current if the radius of the loop is equal to Rand the

21tR 3B
· magnetic induction at its centre is equal to B. Ans: M = - - . - .
µo
30. A very long straight solenoid has a cross-section radius R and n turns per unit length. A direct current
/ flows through the solenoid. Suppose that x is the distance from the end of the solenoid, measured
along its axis. Find :
(a) the magnetic induction B on the axis as a function of x; draw an approximate plot of B vs the ratio
x/R;
(b) the distance x 0 to the point on the axis at which the value of B differs by TJ =I% from that in the
middle section of the solenoid. ·

Ans: (a) B = µon/ [1 x ], wherex > 0 outside the solenoid andx < 0 inside the solenoid.
2
~x 2 + R 2 (BIB0)

R(l-2TJ)
(b) Xo SR.
2)TJ(I-TJ)

----~0 ~-=-xUi
0 CHAPTER cla..,oer

·o
electromagnet

l~
switch
CQntact

__. , I"I
0 i
adju_stina-:!l:.l
~ .... ,...-:. ~ ?Crew ~
-~:::__~...JO
0 '

IT
iron
armature

1P·ermanent spnng --1---.cJ


(
I' :Mag"net and
"
:r
Magn~e~ic Properties
tl~[ 11f2 l,bstances
-
,

7 .1 PERMANENT MAGNET : AN INTRODUCTION

=!:g_2M
In the previous chapter, we have seen that a current carrying loop, produces magnetic field jj
47t r3

at an axial point. HereM= iA is the magnetic moment of the loop. Vector A represents the area vector of the
currentloop. Current loop in magnetic field experiences a torque t = Mx jj . We also know that the electric field

2P
. 1 . E- = -l - - . f the ct·1poean
o f the d1poe1s - ont heax1s0 1 d 1texpenencesatorque't=PXE.
. . - - -
47tEo r 3
The above similarity of current loop with the electric dipole makes the following hypothetical model :
(1) There are two types of charges; positive magnetic charge (+m) and negatfve magnetic charge (-m). A
magnetic charge m placed in a magnetic field jj experiences a force

F = mB ...(1)
The force on a positive magnetic charge is along the field ii , and the force on negative charge is
opposite to the field.
(2) A magnetic charge m produces a magnetic field

at a distance r from it.


B = µo
41t?
m
Ill
.~r---+1
-
Fig. 7. I
B ... (2)

(3) A magnetic dipole is formed when two equal and opposite charges are placed at some separation.
........................
Magnetic dipole moment, M = mf. -If!..._._ f ____:{II
Fig. 7.2
(4) A current loop of area A carrying current i may be replaced by a magnetic dipole of moment
M = ml=iA.

-Ill

s

Fig. 7.3
Bar magnet : Bar magnet is the permanent magnetic dipole. ~agnctic Ieng~
(i) Magnetic length =0.84 x geometric length
s(• ~N
(ii) Magnetic charge or pole strength = m
SI unit ofm is A-m or NIT.
I. •I
Geometric length
(iii) Magnetic moment M =Pole strength x magnetic length Fig. 7.4
m ,~---~~
l M _ = _m..:_J
SI unit of M = A-m 2 or J/T.

Fig. 7.5 Magnetic fie/ti of bar 111ag11et


Field of bar magnet is strong at its ends and negligibly small at its middle.
Ifwe break a bar magnet, it seems, to isolate in a single pole or monopole. However, we cannot get an isolated
pole even if we break the magnet down to its individual atoms and then to its electrons and nuclei. Each
fragment has a north pole and south pole. Thus the simplest magnetic structure that can exist is a magnetic
dipole. Magnetic monopoles do not exist.

7.2 COULOMB'S LAW IN MAGNETISM


According to Coulomb, the magnetic force between two magnetic charges m 1 and m2 placed at a separation r
is given by
!----
·p----'
-(µo·) m1m2-7

L - 4x __ _!_~_-_1
Magnetic field due to a magnetic pole: ifm0 is the magnetic charge and Fis the force exerted
by another magnetic charge m, then magnetic field at a distance r from m can be defined as
ft
B •······················-
,.__r----, B
Ill

Fig. 7.6

=
µo
=-4 2·
m
1t r
PERMANENT MAGNET AND MAGNE11CPROPERTIES OIJ;~IJ~TANCES . IJ:'4
Magnetic potential
For a pole strength m, the field at a distance r is

µo m
B = 4,c '~2 '

and radially away from the pole. The potential at a distance r is given by,

- ·r
v -fBar

Also B = _dV l
dr

Magnetic potential due to a dipole


Consider a magnetic dipole of moment M. If m is the pole strength and f is the distance between the poles, then
M = mf. If r 1 and r 2 are the distances of point P from the poles, then p
e
r--cos0
2
r,
and r+Icos0
2
Magnetic potential at P,
V VN+Vs f/2 t/2

~~[;~-~] Fig. 7.7

= µom(
4 ,c 2
fc:se
f 2
J
r --cos 0
4
Putting mC = M, and neglecting c2 in comparison tor, we get
,--
µ 0 Mease
V
4,c J2-.. --
Magnetic field of a dipole
The magnetic field at any point P at an angular position e at a distance r from the centre of the dipole can be
obtained by resolving it in to two components. One component Br which is along the radius vector and the
other Ba which is perpendicular to radius vector.\! ""·
B
./ ······ ... B,
rdO 'dr
l ~-
Be' (t

'
dO
II

,, ,. \
\ (0+a)
_ _-4---E..,._ .... t;\ ...
0 M.
'-- I ,I~
(a) (b)
Fig. 7.8
The radial component of magnetic field
l
1.. ',, .-
dV
Br - --d-r ,

= "c-.!!.._[µ 0 Mease] ...(1)


· dr 41t ,2

1 µ 0 2Mcose
n-~~1,.,:~1t r3

dV dV
and Ba =,..:-· = - - -
· , dr rde

'-~[:]
~.!..!!.._[µ 0 Mease]
r de 41t ,.2
~ -::'· IJ
,r, -"·, µ 0 Msine
,~/1'., 41t r3 ...(2)
The resultant magnetic field
j/•~~'JB/+B02
On substituting the values of B,. and Ba, we get
r~--,µ-o--:-M.-../:;:~::::~o::::'s=·z_=e:::+::::1-,
B , ;;,, -~ ... (3)
~7t , r
If it makes angle (t with r, then
'
•.. -Ba
tan a. t, :::::i i-
Br

or ... (4)
Special cases :
''/f)jj j, ,,,
On the axis of dipole; 9 =0,
I,
Case I:
0

--
,• rrtv·, __.
µo M 7J
V 47t·;,

µo 2M
B -.-3-, Fig. 7.9
• 41t r
and a = 0
Case 2 : On the equator of dipole, 9 = 90°,
V 0 i
B tr
µo M t
B
4!t ,3 M
and a 180°. Fig. 7.10
- -;,\t
Field of a bar magnet
Case 1 : On the axis of the magnet or end on position : For a magnet oflength f., magnetic field on the axis
of the magnet at a distance r from its c~~tre.
µ0 2Mr
B = , 4It [ f2 ] 2 , for f. << r,
. ,2 _ _
4

Case 2 : On the equator of the magnet or broad side on position:

µo ,M µo M
8 for f<<r,B=--
41t ,3
- , 41t [ f.2 ].3.
2 '
;_ r2+-
,. 4

. Torque on a magnet in magnetic field


When a magnet is placed in an uniform magnetic field, its poles
experience equal and opposite forces and so net force Fnet on the
magnet is zero. But the magnet experiences a torque given by,
~ = mBxf.sin9
MBsin9
In vector fonn, we can write '

, :1 Fig. 7.11
Work done in rotating the magnet : The torque exerted by the agent for any angle 9 is MB sin9. Thus work done
in rotating the magnet from 9 1 to 02,
82 82

w J~d0= JM Bsin0d9 -.
01 01

or w MB[~os 91 - cos 92 J-1


Potential energy in magnetic field
lfwe take potential energy at e, ;,, 90° to be zero, then potential energy at 82 = 0,

1- u_::_ ~r_:?~~~ -cos0]


\ or !_U = _-MBcos0_)
Magnetic force between two short magnetic dipoles
(i) Consider two short dipoles M I and M 2 are placed on the same axis at a separation r. The force of
interaction between them.

1+--r-+1
Allmction
-
(ii)
- -lvfi
I+-- r-+1
Repulsion
M2
Fig. 7.12

When dipoles are placed on mutually perpendicular axes, the force of interaction between them

M1
--,,---+-
t.·112
F
1+--r-+1
Fig. 7.13
Ex. 1 (a) Indicate the positions and the nature of equilibrium of a
number of magnetic needles arranged in a straight line at
equal distances from one another.

(b) A strong horseshoe magnet is closed by an iron plate A fig.


(a). The weight of the plate corresponds to the lifting force
of the magnet, and the magnet can easily hold the plate. If
the poles of the magnet are now touched on the sides with a
plate B made of soft iron, the plate A will drop at once. If the B
plate B is removed the magnet will again be capable of
holding the plate A. Explain this phenomenon. A

(c) A long rod made of softiron is secured in a vertical position.


If a strong magnet A is brought to the top of the rod as shown (a)
inh7.~ ~7.14
(i) The rod will be magnetized so intensely as to retain at its other end several small pieces
ofiron. If the same magnet A is applied to the side of the rod nea_r the bottom end fig. 7.14c (ii)
the magnetization will be weak and the pieces of iron will fall. Explain why the magnet A acts
differently in these two cases.
(d) A strong magnet of magneto alloy can hold a chain consisting of several cylinders made of
soft iron fig. 7.J4(d). What will happen to the cylinders if a similar magnet is brought up
from below to this chain? The magnets are arranged with their like poles facing. Whatwill
happen to the cylinders if the magnets have their opposite poles facing?
PERMANENT MAGNET AND MAGNETIC PROPERTIES OF SUBSTANCES

. s
'

N
A

!:I"'~!
,,q,. ~I
(i) (ii) (d) (e)
(c)
Fig. 7.J.I
(e) Two identical horseshoe magnets are linked by~heir opposite poles as shown injig. 7.J4(e) ,
One of the magnets has round it a coil A whose ends are connected to a galvanometer G. If the
magnets are detached, the pointer o(th-e galvanometer will be deflected at this instant
through a certain angle. If the magnets are connected again the pointer of the galvanometer
will also be deflected, but this time in the opposite direction.
Indicate the causes for the deflection of the pointer of the galvanometer.
(I) Permalloy can be magnetized appreciably in the terrestrial magnetic field and does not
possess residual magnetism, i.e., it is the softest material as far as magnetism is concerned.·

How will a magnetic needle on a vertical axis near a long bar made of this alloy behave if:
(i) the bar is vertical jig. 7.15 (I);
(ii) "'
the bar is horizontally placed along the magnetic meridian;
.

· . •,;! '

., '' '

(I) (g)
/
Fig. 7.15 <):1· ,:
"

(iii) the bar is in a horizontal plane perpendic~(ar to the magnetic meridian.


Willthe behaviour of the needle change in these three cases when the bar is turned? ·
(g) A small thin iron nail is suspended from a light fire-proof thread. A strong electromagnet is
placed near-the nail.fig. 7._1 S(g). The flame from a powerful gas burner is directed precisely
between the nail and the magnet and licks the nail when it is deflected by the magnet. If the
windings of the electromagnet are energized the nail will be at once deflected into the flame
and will then be ejected from itto assume its original position. After a lapse of time the naU
,, will-again be drawn to the magnet..
Explain what causes these periodic oscillations of the nail.
Sol. (a) The arrangement shown in Fig.(i) corresponds to the position of unstable equilibrium and
that in Fig.(ii) corresponds to stable equilibrium.

t -I.
I
t +I
t
t
t
(i)
(a)
f
(ii)

Fig, 7.16
(b) Upon contact with plate B, part of the magnetic lines of force are short-circuited through this
plate (Fig. (b)). The number of the lines of force penetrating through plate A sharply decreases.
As a result the interaction between the magnet and plate A also decreases and the latter
drops.·
(c) In the second case most of the magnetic lines of force are short-circuited inside the part of the
rod adjoining the magnet (Fig. 7.17. (c)) and the rod cannot, therefore, be magnetized as
intensely as in the first case.
(d) In the first case,-as the lower magnet is brought nearer, the cylinders will be detached one
after another from the chain and attracted to the lower magnet.

(b)
Fig. 7.17
In the second case, when the pole of the opposite sign is drawn closer, the "strength" of the
chain will increase as the lower magnet is brought near.As"soon as the second magnet crimes
in contact with the lower cylinder, the magnet will be attracted to the chain and will remain
hanging on it.
,.

.•.'
_(e) . The magnets are detached owing to a sharp decrease in the number ofthe lines of force
passing inside each ~agnet. At the moment of detachment an e.m.f. is induced in the coil due
to the reduction in the number of the lines of force.
(f) (i) The bar will be magnetized due to the action of the vertical component of the terrestrial
magnetic field and the magnetic needle will be attracted as it is brought near the ends of
the bar. '
·(ii) The bar wiil be magnetized by the action of the horizontal component of the terrestrial
magnetic field. The magnetic'needle will always tum towards the nearest end of the bar.
be
(iit) The bar will not magnetized by the magnetic field of the Earth and the needle will not
change its position when the bar is brought close.
It is assumed in all the three cases that when the needle is brought close to the bar the
distance between them is still large enough for the additional magnetization of the bar .
due to the external magnetic field of the needle to be neglected. -
If the bar is turned, the behaviour of the needle will not change in any of the three cases .
.(g) Iron loses its magnetic properties at quite a high temperature and behaves as any other non-
magnetic substance (copper, glass, etc.). When the nail is heated in the flame of a burner to
this temperature the interaction of the magnet and the nail.abruptly decreases, the nail returns
to the initial position, leaves the flame and gets cool. The magnetic properties of the nail thus
cooled are regained, the forces of interaction between the nail and the magnet increase and
the nail is again drawn to the magnet. ·

7.3 EARTH'S MAGNETISM


It is a well known fact that freely suspended magnet or current carrying solenoid rests in specific direction,
called magnetic meridian. It shows that earth has its own magnetic field. The modem theory about earth
magnetic field is that, the earth rotates about an axis and has the surrounding ionised region due to interaction
of cosmic rays, Uue to rotation of earth the surrounding ionised region gives rise to strong electric current
which causes magnetic field. Its value on earth surface is 1 gauss.

Fig. 7.18

Elements of earth's magnetic field :


To ·know ab.out earth's magnetic field, we need three Geographical
,a, informations. They are : · meridian
(1) Magnetic declination (,jl): Angle between
geographical and magnetic meridian is knOWJl as angle: Magnetic
of declination. It has an average value 17 .5°. meridi!Ul

Fig. 7.19
(2) Angle of inclination or dip (0) : It is the angle between the magnetic field of earth and the horizontal at
that place. It is zero at magnetic equator and 90° at poles.
In the magnetic northern hemisphere, the vertical component of earth's field points downward.
(3) Horizontal component of earth's magnetic field (Bnl : At any place other than magnetic poles, there is
horizontal component of field.
Bcos0 ... (i)

and vertical component Bv Bsin0 ... (ii)


By squaring and adding equations (i) and (ii), we get
1B = )s1 +Bp , .... (1)
------~-J
Dividing equation (ii) by (i), we get

....(2)

True dip and apparent dip


The angle of dip in magnetic meridian plane is called true dip (0), and angle of dip in different plane from
magnetic meridian plane is called apparent dip.
We know that true dip
Magnetic meridian
... (i) M.M.
Any plane
On any other plane at an angle a from the meridian plane, the
horizontal component of earth magnetic field will be B Hcosa while
vertical component remain as such. Thus apparent dip,

Bv a
tan0' = BH cos a ... (ii)
B,.
From equations (i) and (ii), we get Fig. 7.20

tan0
tan0' .... (iii)
COSCl

Compass needle and dip needle


Compass needle gives the direction of BH and the dip needle gives direction of B .

Compass
needle
B

Fig. 7.21
::;;;:,_- ,; ;; ·: -. 0;....-"' ---,'"< '·:·-·. . . .f /;;;: ;; •• ,; •

PERMANENT MAGNET AND MAGNETIC PROPERTIES OF SUBSTANCES

Magnetic map: It is found that many places have the same value of magnetic elements. The lines drawn by
joining all places on the earth having same value of magnetic element form magnetic map.
lsogonic line: This is the line joining the places of equal angles of declination.
Agonic line : This is the line which passes through places having zero declination. Magnetic meridian
itself is a agonic line.
lsoclinic line : This is the line joining the points of equal dip.
Aclinic line : This is the line joining the places of zerodip. Magnetic equator is an aclinic line.
Isodynamic line: This is the line joining the places of equal value of horizontal components of earth magnetic
field.

Ex. 2 If0 1 and 02 are the angles of dip observed in two vertical planes right angles to each other and 0
be the true angle of dip, show that cot2 0 = cot20 1 + cot20 2 •

Sol. We have, tan8 = Bv ... (i) M.M


BJi
The value of vertical component remain same but horizontal
component in two perpendicular planes a~e .. BH cos Ct and

BH sin a as shown infig. 7.22.

Bv tan8
BH cos a cos a

lane
or cosa. - - ... (ii) Fig. 7.22
lane,

Bv lane
and tan8 2
BH sin a
= sin ex

lane
or sina. ... (iii)
tan 8 2
Squaring equations (ii) and (iii), and adding, we have

cos 2 cx+sin 2 ex
tan 2 8 ( ---+I 2I
tan 2 e, tan 82
J
1
or
- - - + - 2- -
tan 2 8 tan2 e, tan ·82

or cot28 cot28 I + cot28 2. Proved

Ex. 3 A bar magnet of length 8 cm and having a pole strength of 1.0 A-m is placed vertically on a
horizontal table with the south pole on the table.A neutral point is found on the table at a distance
6.0 cm north of the magnet. Calculate the earth's horizontal magnetic field.
Sol. "' •The magnetic field at P due to south pole

Bs
and due to north pole

B,v
Its component along north is B,v cos0, On substituting the value of 0, it is
µo m x r
41t (c2+r2) ~f2+r2

µ0 mr
41t I
(c2+r2')z
For neutral point at P, we have
By = Bs-B,vcose

µo [m
41t r2

10-7 xi[. I
0.06 2

22xI0-6 T Ans.

Ex.4 Fig. 7.24 shows some of the equipotential surfaces of the magnetic scalar potential. Find,the
magnetic fieldB at a pointin the region.
y
Sol.
B

L:;;,:';~~~=-::,,:f!::::.....,~::::__-~... x(c1n)
40
Fig. 7.24

We know that B
_AV= (V2-f'i)
Ar Ar
f'i-Vz
Ar
(0.2-0.l)x!0-4
O.lsin30°
2.0x!0-4T. Ans.
PE#MANENT MAGN.er%vn MAGNETIC PROPERTIES OF SUBSTANCES
. ,, ., . ,;, ' ' \; ',- ' . ' . "

Ex. 5 A magnetic dipole of magnetic moment 0.72-./2. A-m2 is placed horizontally with the north pole
pointing towards east. Find the position of the neutral point if the horizontal component of the
earth's magnetic field is 18 µT. ·
Sol. Given M = 0.72-Ji. A-m 2

B CJ. •••
•P
BH1
Fig. 7.25
From the figure a+e 90°
a (90-0)
We have,
./3 -Ji.
lane
tana
2 fl

tan0
or tan(90° -0) -- Fig. 7.26
2

tane
or cote --
2
or 2

e tan- 1 ( -./2.) Ans.

Let neutral point is at a distance r, then


BH = B

µ 0 M ~3cos 2 0+ 1
= 47t r3

or 18x!0-6
7
10- x0.72-./2. ~
r3

On solving r = 0.20m. Ans.

Ex. 6 In order to keep the needle (seejig. 7.28) in a horizontal position a load Pis suspended from its top
end. Find the magnitudes of the horizontal and vertical components of the intensity of the terrestrial
magnetic field. Calculate the total intensity of the terrestrial magnetic field. The dip angle is 0.
The pole strength is m.
' ..
Sol. The magnetic needle will be acted by twoJ'orces tending to tum it, -their moments are: the moment
of the force due to the terrestrial magneti. field acting on the poles of the needle
~ = (mB)f
and the moment produced by the load 111B"
f
suspended'from the needle and is equal to P .
2
Thus for the equilibrium of the needle ,

mB/ = p!:__ mB.,. p


2 i.----- e----
p
Hence, BV = Fig. 7.28 Ans.
2111
The horizontal component of the terrestrial magnetic field
BH Bv tan0,
and total field Ans.

Ex.7 Two long equally magnetized needles are freely suspended by their like poles from a hook as
shown in jig. 7.29. The length of each needle is f cm and the weight is W. In equilibrium the
needles make an angle a with each other. Determine the magnetic pole strength of the needles.
The magnetic pole strength is conceii_trated atthe ends of the needles.

F
. J4-07 W \A

· Fil!_·. 7.29\
Sol. Let us consider one of the needles. It will be acted by the weight W, which acts through the centre
. - ..- . .
. I 2
of gravity Band the repulsive force between the poles F = : acted through the point A. For the
2

needle to be in equilibrium the sum of the moments of the forces acting on the needle should be
equal to zero, i.e.,
- -f • a a
· P-sm- Ffcos-
2 2 2

Here \ \
m2 ,. -
F. = \ -
· . ex
2 , where a=2fsm- ..
\a 2
\ \
: .PERMANENT MAGNET AND MAGNE'tl¢ PROPERTIES OF SiiBSTANCES

2 a ,i
m cos-,
P. a 2
Thus -sm-
2 2 4£ 2 sin 2 ~
2

m Csin a J2Ptan a. Ans.


2 2

Ex. 8 A magnetic needle with length Cand poles ofpole strength± m is attached to a wooden bar oflength
L (see fig. 7.30) and placed in a uniform magnetic field of intensity B. The bar and needle can
revolve around point 0. Find the magnitude of the moment of force that will cause the rotation of
the bar around point Oifthe bar makes an angle a with the direction of the lines of force of the
magnetic field.
Sol. The moment of forces about 0, acting upon each pole of the needle will be equal to mB,
(C + L) sin aand-mBL sin a.

Fig. 7.30
Hence, the net moment of the force acting on the bar is
~ = mB(C+L)sina-mBLsina
mBC sin a= (mC) B sin a
MBsina. Ans.
.. l.

!BJ@ ,'he earth's magnetic field is not due to a large bar magnet inside it because the earth's core is hot:
and molten. It may be due to the convection currents around the equator. As to what dynamo effect,
sustains: this current, and why ·the earth's field reverses polarity every million years or so, We do not;
. '
·--·know. ·---· ...... __ . --···· -··--------- __ ·"·--· _ _________ -----·

7.4 MOVING COIL GALVANOMETER

It is used to detect the direction of direct current. It is based on the interaction between current carrying loop
with the magnetic field. In a:moving coil galvanometer, the coil is suspended between the pole pieces of a strong
magnet. The poles are made cylindrical and a soft iron cylindrical core is placed within the coil. This makes the
field radial. In such a field the plane of the loop always remains parallel to the field, and 0 = 90°. The deflecting
torque,
'tdef NiBA sin90°
NiBA ... (i)
This torque deflects the coil. As soon as coil starts rotating a restoring torque is set up in the suspension wire.
If a is the angle of twist in the suspension wire and C is the torsional rigidity, then restoring torque
~,est Ca ... (ii)
From equations (i) and (ii), we have
NiBA

(N~A)a

or
r--·
t
.-·-1
·1 =· ka,, :...(!)
~------·---1
where k =-C- is known as galvanometer constant.
NBA

(a) Side view (b) Top view

Fig. 7.31
Current sensitivity (s): Current sensitivity is defined as

da a
S = di or simply i

Therefore S = NBA1·.. .....(2)


.- C ·,

7.5 TANGENT LAW

When a compass needle is placed in two perpendicular fields, B and By, the needle rests at an angle 0

with By. In equilibrium

B
tan0
By

=> ["'11'~ .B y1?0 l


This is known as the tangent law of perpendicular fields: Fig. 7.32
7.6 TANGENT GALVANOMETER
Tangent galvanometer is an instrument to measure direct curr~nt. It is based ~n tangent law. ·

·circular
cojl
7.1
Bi
~ ................. MM
BH

- - - Leveling screw
Fig. 7.33
The current to be measured is passed through coil set in M.M. produces a magnetic field at the centre that is
. N'1
perpendicular to the plane of the coil, B = µiR • In addition to this field, there is earth magnetic field with

horizontal component BH• so the compass needle placed at the centre of the galvanometer, defects in, the
direction ofresultant field. Thus

.µ 0 Ni
or
ZR

2RBn)
- - tan 9
( µoN
ktan9
. 2RB
where k = µo:: is called reduction factor.
· Sen.sitivity : Good sensittvity means that change in deflection is large for a given fractional change in
current. We have
i k tan9
or di k sec29 d9
di ksec 2 9 d0
or
ktan0
d0
sin a cos0

2d0
sin20
d9 sin20
or
di 2i
For maximum sensitivity sin 20 = I, or 0 = 45°
So the tangent galvanometer is most sensitive when the deflection is around 45°.
•l
',,

7.7 VIBRATION MAGNETOMETER -·--,. . -c,;_.,,,.-.:. 1-., ...·.:,' _;·_ •1


Vibration magnetometer is used for comparison' ·of•m'agnetic f:elds·. at different places and also to comparb
magnetic moments of the magn~t;.-First ofall'sJhhe magnetometer with its ma~et along the meridian. ·

Magnetic
meridian--

.,.
l,, ,, • ' ',•
"
Fig. 7.34
Then the experimental bar magnet is deflecteq ~rm;n i~ m_ean posit_ion by external magnet and left free to oscillate
in Bn . The restoring torque _acts on .the magnet for small·angle.8
.,_
. '

rg 't -~d !/dBH Sip.9

or 'a.'!1::::: MBif(~a) [for small-8, sinff= 8]


·'' I .
Compare with a ~e. we get
ol ,, ~ '"JMBH
'dffi:•.-;-- !
.. .. - / -

and !. T_ ~ • ~1C~~~1 ....(!)


' J .
1 , , • ~ , .I ·, 1
1 ' ,,,,. >·: ::;.< .· '.----'-------. ·,
where I is the moment of inertia of the magnet, which is given by .

, / : W(a2+b2) ~
: i 12 . a b
W ~ mass of the magnet. ' I Fig. 7.35

1. Comparison of horizontal componerits of magnetic field at two places


If T1 and T2 are the time periods of moti~ri of_experimental magnet at two different places, then

T1 ""..~2!"
--cc ..-_ .... 1_ ____·- and T2 =2icJ M
M
Bn1- Bn2

' ' .• ,,h •

. , [!r;. ~ tr., ,,, . -. 1..,


!PERMANENT MAGNET AND MAGNEri"ePlloPERTIES ofSuBSiAivcEs ••• ·I

2. Comparison of magnetic moments of the magnets


Suppose M1 and M2 are the.magnetic moments oft!)~ magnets. First place.the magnets on th~ pan with
like poles together, let time period is T1 and then ~p!ik~:IJoles together, let time period js T2, then

(11 + 12)
T1 - 21t
(M,+M2 )BH
IElID
~
(I, +!2)
and

From above equations, we get


T2 21t
(M,-M2)BH
B
Fig. 7.36

i.

-
M,--- -~ . T/2 +·1;22·-1 ....(3)
lM2 T2 -Tj
1--------------"

7 .8 ~EFLECTION MAGNETOMETER

It is used to determine M for a permanent bar magnet. lts'working i~ based on tangent law. It con;ists ~fa
BH
M
small compass needle, which is pivoted at the centre of the magnetometer. The quantity - can be measured
BH .
in two standard positions of the magnetometer. One is called Tan-A position and the other is called Tan-B
position.
1. Tan-A position :
In this position the magnetometer is set perpendicular to magnetic meridian, so that, magnetic field due
to magnet is in axial position and perpendicular to earth's field. Hence by tangent law
BHtan 0 B
µo 2Mr
or
4
1t ( fi~oo ~ r
_. .....(])

°ԲFig. 7.37
2. Tan B position:
In this position the magnetometer is set in south north direction, so that the magnetic field due to magnet
is at its equator. Hence·by tangent law
,,I
BJPn0 B \.
'/ I
dI '
or
µo __M_~
Bn tan0 '.i

471 [
,2+-
£2 ]l 2

4.
,.

-
Fig. 7.38

7,9 GAUSS'S LAW IN MAGNETISM

In electrostatics, we have studied the Gauss's law for electric field, i.e.,

, ' '

where <f, E·dA is the electric flux and qinside is the. net charge enclosed by the closed surface. On the similar
way Gauss's law for magnetic field can be written'as ;

PB• dA = µO"'inside

-m +m ~:;;,···~;-;,-..,
Fl ==;.;;=,J::::,i.
. \ LJ j
••·· ................

(a) (b)
-
\\'' '
_\ Fig. 7.39
I i
where rf, B·dA is the magnetic flux and minsi~ is the net 'magnetic charge' inside the closed surface. In any
closed surface minside =m - m = 0, as isolated P·ole does not exist and so Gauss' law for magnetism, therefore
states that · · ·

Ex. 9 Two identical thin bar magnets, each oflength l and pole strength m are placed at right angles to
each other with the north pole of onJ'touching the south pole of the other. What is the resultant
magnetic moment of the system? '
I' ..

Sol. If Mis the magnetic moment of each bar magnet, ,then M =ml. tz,
s M

j
The resultant magnetic moment

M = .fiM=.J2 me.
M
N
s :r::. N
~- Fig. 7.40

Ex. 10 A magnetic needle is free t~ rotate in a vertical plane which makes an angle 60° with the magnetic

meridian. If the needle stays in a direction m~ki~g an angle of tan" 1 (Tl) with the horizonta~ what
. I
would be the dip at that place ?
M.M.
Sol. ·Given, apparent dip

f1 tan- (
1
1) By

and a 60°
':n a
tan0
We have · tan0' = Bv
,, cosa
or tan0 tan0' cos a Fig. 7.41
2., -11. . I
-x-=-
..fj_ .,2,,.. ..fj
or 0 = 30°. Ans.

. Ex. 11 The frequency of oscillation of the magnet in an oscillation magnetometer in the earth mag!)etic
field is 40 oscillations per minute. A short barinagnet is placed to the north of the magnetometer,
at a separation of20 cm from the oscillating magnet, with its north pole pointing towards north.
The frequency of oscillation is found to increase 60·~sclllations per minute. Calculate the magnetic
moment of the short bar magnet. ., I ·
Horizontal component of the earth's magnetic ·iieid
is 24 µT.

. .J ~MBH
Sol. Previously, f= -
2it
--
I
--1-----B
c::Sc...,~--'N"--"'~ ,c.rs=_=N:::;iO N .
and then,
' Ji,,
:.,.__r_..:
1'2 B8 +B
Fig. 7.42
/2 BH

r:~r
: · _, rb,
BH +B
,_ ,-•nn
or
BH
,,,

I .:~

Of = 1.25

or B l.25BH·
= l.25x24 µT=30xl0-6T
__ The oscillating magnet is in end- on posi\ion of the short magnet. Thus,
., ·1,-_
µ 2M'
B=30xI0- 6 = _Q_ _ _
,., i' . ,._ ·.
~
- :__ - - 4it r3
- :- ;Jt· :~,·
" ~·
. ,'

or ·· · .·'·
30x10-6 x(o.20)3
10-1 x2
1.2 A-m2. Ans.

Ex.12 A long vertical wire carries a steady cur~fnt of 10 A flowing upwards through it at a place where
the horizontal component of earth magneti~ field is 0.3 gauss. What is the resultant horizontal
magnetic field at a point 5 cm from the wire due magnetic north of the wire?
Sol. The magnetic field due to current carrying conductor
i'1 J I• • , B11
µ 0 .!_ 2x10-7 xl0 p
B B
2it r
I
·n-: wire
.;,, __ 0.4x!0-4T= 0.4 gauss
w E
'··and BJi = 0.3 gauss

:. Net field at.P )s~+s'k


,, S·
Jo.42 +0.32 Fig. 7.43
0.5 gauss · Ans.

Ex.13 A magnet is suspended in the magnetic meridian with an untwisted wire. The upper end of the wire
is rotated through 180° to deflect,the magnet by 30° from the magnetic meridian. Now this
ll' . '
magnet is replaced by another magnet and Ihe upper end ofthe wire has to be rotated through 270°
to deflect the magnet by 30° from magu·~'ti~ meridian. Compare magnetic moments of the two
' J ~ J, • •
magnets.
Sol. We know that, ,,..,
_, '

-r = Ca=MBHsin8
', • , _r
where a is the angle of twist and (J.j~,the deflection of the magnet from the magnetic meridian.
For first magnet

..... (i)
I ,, ,

"

For the second magnet

c( 210° -30°) .... (ii)


L
Dividing equation (i) by (ii), we get

M1 150 5
--=..:...." Ans.
Mz 240 8

Ex. 14 A thin magnet is cut into the equal parts by cutting it perpendicular to its length. What is the new
magnetic moment of each part? What is the time period of each part as compared to that of
original magnet if vibrated in the same magnetic field.
Sol, e
If is the initial length of the magnet, then magnetic moment M = mf and its moment of inertia

massxe 2
I - - - - . When the magnet is cut into, two equal parts, the magnetic moment
12
f M
M m-=- and
2 2

I
r = 8
'I ·,,1

If Tand 7' are the initial and final time periods, then

On comparing above equations, we get


T
T =
2

7 .10 MAGNETISM AND ELECTRON ' ·:•;

Any material is magnetic because of the motions of electrons Wiinih it. There are two motions of electrons: spin
and ~rbital, each involving a magnetic dipole moment that produce; a magnetic field in the surrounding space.
Explanation of these requires quantum physics which is beyond the scope of the book, so here we shall discuss
only that part which can be understood by the previous knowledge of the subject. ·
Magnetic moment due to orbital mot.ion of the electron
Suppose an electron moving at constant speed v in a circular.path of radius r, counter clockwise as shown in jig.
7.44. The conventional current i will be clockwise. The magiiitu'de of the orbital magnetic dipole moment of
such a current loop
Morbital = iA,.
where A is the area enclosed by the loop, which is m-2, and
Charge e Lomita!
=
time
(2:r)
r
i
ev
21tr
Fig. 7.44 M orbital
ev 2 evr
--X1tr = -
Thus Morbital 21tr
=
2 ... (i)
The direction of this dipole moment is downward, by right-hand rule.
The angular momentum
L = m(rxv)
As ;: a~d V are perpendicular, so
£orbital = mvr sin90° = mvr ...(ii)
Lo,bital is directed upward.
From equations (i) and (ii), we get

. ~- , Mo,bital = . -,~Lo~bital . . ...(!)


This result obtained by classical analysis is in agreement with that given by quantum physics.
· Magnetic moment due to spin of the electron
An electron has an angular momentum due to its spin motion, is called spin angular momentum S. The
magnetic dipole moment due to the spin moti~n of the eJe.ctron is given by
- e -
M,pin =· - - S. ...(2)
. m

19 31
Here e is the elementary charge (l.60x!0- C) and mis the mass ofan electron (9.llx10'" kg). The minus

sign means that Mspin and s are oppositely directed.


s

Mspin
Fig, 7.45. Magnetic dipole .moment due to the spin motion of the electron
The resultant magnetic dipole moment is the vector sum of these dipole moments. Thus
M = Mor~i~al + Mspin.

e - e -
- ·Zm Lorbital - m S
or M = . .:. e. + zs)
(L~,bi1a1 ....(3)
2mw,1: ' '
If the resultant magnetic dipole moment proLd,-u_c_e_s_a_m_a-gnetic field, then the material is magnetic. The types of
magnetic material will be discuss little later in this chapter.

7 .11 MAGNETIC PROPERTIES OF THE MATERIALS

Intensity of magnetisation
Each substance contains a large number of atoms. In general, the magnetic moments of these atoms are
randomly oriented and there is no net magnetic moment in any volume of the material that contains thousands
ofatoms. However, when the material is kept in an external magnetic field, atomic dipoles try to align parallel to
the field. The degree of alignment increases if the intensity of the applied field increases and also if the
temperature is decreased. With sufficient strong field, the alignment is nearly perfect. We then say that the
material is magnetically saturated.
When the atomic dipoles are aligned, partially or wholly, there is a net magnetic moment in the direction of the
field in any small volume· of the material. Thus magnetisation is defined as,
Magnetic dipole moment
l volume

or ......(!)

ra
B=O M=O
'
Fig. 7.46
0- 0- 0-.
0- 0-. <Y
C)-+ <Y 0-.
8¢0,M ¢0

Unit of magnetisation is Nm.


Consider a bar magnet of pole strength m, length i and area ofcross ·section A. The intensity of magnetisation
M mi m
1 = v= Ai =A.
Thus for a bar magnet, the intensity of magnetisation is defined as the pole strength per unit face area.
· Intensity ~f magnetising field
When any material is placed in magnetic field, it gets magnetised. The actual magnetic field inside the material
is the sum of the applied magnetic field and the magnetic field due to magnetisation of the material. Thus
· magnetic field. inside the material
ii = Bo+Bm,
where Eo is the magnetic field in the vacuum produced by magnetis.ing field, and
equal to µii .
Here fl is called intensity of magnetising field. Thus fl can be defined as: Fig. 7.47

0 B0 µ ni .
H = -=--=m.
µo µo
Bm is the magnetic field due to magnetisation of material, which is, Bm =µ 0 l.
. j·, ir:~~~ffl/€m;'~~ ,OO!GlVE:lJ§..M
Magnetic susceptibility
Magnetic susceptibility is defined as the intensity of magnetisation per unit magnetising field. Thus

where X is a dimensionless quantity and ma)' be positive or


I.· :_ ·
.. X " -
' - - - - ~ , -.,_
negative.
• -..=:---
1.• .•·•]·)
H1'" .:
µr < I ;. x_ <o /p'2 <JJio.'M"r?/2'-
>I • x_ --., P.:; n1 f-i/. lnl ······< )
I, •
A,I a I ~ I

~- ~

Magnetic permeability
Magnetic permeability of the material is the measure of degree to which the material can be permeated by
magnetic fie.Id and is defined as the ratio of magnetic field in the material to the magnetising field. Thus
B
µ =
H

Also \ µo =
Hereµ, is ca~ed relative permeabili;r.
Relationship between m, and x
We have,
jj Bo+Bm
or B µ0 H+~l

or B µo1f(1+~)
B I
or I+-
µoH H
As BIH µ,

.l!:... I+-
I
. µo H
or [&_ = l+X.·'J ...... (3)
r-· --~-:
k!J@ Th~ following points in re&arding with B ,md k should be noted. _
i • - .. - ' - - - .
.
- ..
.
-
I
I
I. Hi~ the cau~e and Bis the etfoct: Both measure the i_nteµsity of the magnetic field and_are vectors. B
1
includ~st~e pres~nce of medium ' · 1
B = µH-anii.Q 0 =.µolf.
2 Band# refe~ to external fields,and,not'to magnet's own field.
; ~ -. . -. ,: .. ' _; - ' ._"'-t- . . . •

3. . The above magnetic phenomenori.is:analogous to e!ectrq~tatic .phenoriien~m;just·as "f,,[J and• jj; in.,
electr,o,s.t.a.fa.· cs. ar.e related. to ·b·.OU. .µ?.·· ~h~rges.tfre.e charges aru..! all char.ges re~p~.ctivefy. Simtlar!yJm; :'-Ji
and B are related to··bound ·c!'rrents, .free purrents a!ld, all currents. 1,,, 1s analogous to d1electnc,
'--'-'-----~-
. polarisa\ionP.
Classification of magnetic materials
------'---
·. · ·'-
' --- ·
--~---
' ' ·, · · · · :-
-----· ~,.-~~---------
1. Diamagnetic material : Suppose a material in which individual atom does not have a net magnetic
moment: When such a material is placed in a magnetic.field, dipole moments are induced in the atoms by ·
the applied field. The magnetic 'field due to induced magnetic moment opposes the original field. Thus
the resultant field inside the material is smaller than the applied field. This· type of material is called
diamagneticmaterial. ·

+-0 +-0 +-0


+---B"' B = B0 - Bm
+-0 +-0 +-0

Fig. 7.48
Magnetic moments are induced in all materials whenever a magnetic field is applied. Thus all materials
have the property of diamagnetism. However, if there is a permanent atomic magnetic moment, then
paramagnetism or ferromagnetism is much stronger than diamagnetism and the material does not show
diamagnetic property.
2. Paramagnetic material : Now consider a material whose individual atoms have a net magnetic moment.
When such a substance is placed in magnetic field, an extra magnetic field produces in the material in the
direction of the field. The resultant magnetic field in the material is then greater than the applied field.
The tendency to increase the magnetic field due to magnetisation of material is called paramagnetism,
and material is called paramagnetic material.

0-0-0-
Bnl B=Bo+Bm
0-0-0-

Fig. 7.49
3. Ferromagnetic material. : In some materials, the permanent atomic magnetic moments have strong
tendency to align themselves, even without any external field. These materials are called ferromagnetic
materials. In every unmagnetised ferromagnetic material, the atoms form domains inside the material.
Different domains, however, have different directions of magnetic moment and hence the materials
remain unmagnetised. On applying an external field, these domains rotate and align in the direction of
magnetic field.
B B

~
Unmagnetised
[~[
Domain-growing
(b)
[~[
Domain-aligmnent
(c)
(a)
Fig. 7.50
'
Because of the domain character offerromagnetic materials, even if a small magnetic field is applied,
gives rise to large magnetisation. The resultant field is much larger than the applied field in such a
material.
Curie's law: With the increase of temperature, therandOmisation of atomic magnetic-moments increases
thereby decreasing the magnetisation I. The resultant magnetic field B decreases, which means X
decreases with temperature. The susceptibility of paramagnetic material,,
C
X (C:urie's law)
r'
where C is Curie constant.
Curie temperature '
The temperature at which ferromagnetic substance becomes paramagnetic is called Curie temperatur~.
At temperature above curie temperature the magnetic susceptibility offerromagnetic materials is given by

C'
X = T-Tc

Here Tc --) Curie temperature, for iron it is I 0_43K.

Hyste_resis
Hysteresis is shown only by ferromagnetic substances. In ferromagnetic
materials, by removing external field, the magnetic moment _of some domains
remain aligned in the direction of previous applied magnetising field which H
results a residual magnetism. The !aging ofintensity of magnetisation (I) to.
magnetising field is (H) is known as hysteresis.
I.
OA _--) Retentivity or residual magnetism
OB --) Coercivity Fig. 7.51

Steel . Soft iron ..

I I
7
,-1.---7
' H
H

.t_-1,,
'
, .
'
..
The area of hysteresis loop is large and thus The area of hysteresis loop is less and• thus low
high enernv loss. enerm, loss.
Hi2h retentivitv lare:e coercivitv. Low retel1tivitv. less coercivitv
Less magnetic permeability, I and ·1. Hieb maenetic oermeabilitv, I and Y
Magnetisation and demagrietisation are _Ma~netisa~ion and demagnetisation are easy.
difficult.
Used for making- permanent magnets, and Used for making electromagnet, and thereby used in
thereby used in loudspeaker, microphone etc. dyna~o, -transformer etc.

I;:C pi~~~gn~tism js ~he'µnivers?l property, itir ~rese?t in all ?1ateri~Is: B~~ it is. u:eak ~nd hard to j
.. detbc{ if the Sl.JbStah~e is pa~a or.ferrom~~n~tic. .· ' . . ...• '. , •. :, ·. l
The phenomenon ofm~gnJtic ~y~teresisJs ~imilar to "the elastichYsteresis : Strain m;y ~ot
' - / '' • ', - -_ .. ,-· -· ., < • ' ';':'~

.. be·proporti~nal to'stress; here ii·and fiire·not linearlyrelatedi'.'>


• ". • .• --. ",.,. ', ' " ' • <
.
,pl~trostatic shliMfiig can b~ possibl~ 6y,anyJi\etal. But magnetic shielding ca~ only possibld
by irOn ·.•..,./ ' ·· ,,, ,/;l i'' •
Comparative study of magnetic properties of materials
•' rt .
Diamaenetic substances Paramaenetic substances Ferromaenetic substances
Found in solids, liquids and Found in solids, liquids and Found normally in solids.
0 ases. !lases.
Examples : Ag, Au, Br, Cu, Examples : Al, CuCb, Na, 0 2 Examples: Co, Fe, Ni, Cd, Fe, 04
H,O, NaCl, Sb etc. Mn.-Pt etc. etc.
Universal nrooertv Not universal Not universal
Can be explained on the basis Can be explained on the basis On the basis of domain theory
of orbital motion of electrons of spin and orbital motion of
electrons
When placed in strong When placed in strong When placed in weak magnetic
magnetic field, magnetised magnetic field, magnetised field,magnetised strongly in the
weP_kly in the direction well,_kly in the direction of field direction of field. B = B0+ B m
onnosite of field B = Bo-B m B=Bo+Bm

~tJ @tJ
:

-t. .. ... .J
h;.......
,._._ :.··
~-
·.t. ,_._._,_ _,_._. ,..J ootJ \~.,.,, :.~://:..[=::
Susceptibility X low and Susceptibility :;,low but positiv.e Susceptibility X high and
negative I Jt - I x-1 nositive" IO 2

Relative permeabilityµ,< I µ,> I µ,»I~- H>')

~
E-::::
----.::::::::::
~ ¾I ~
Jependence of X on temperatun
Diamagnetic Paramagnetic Ferromagnetic
X X

i\_
X

T
\__ T Tc
T

Ex. 15 Eachatomofaniron bar (s cmxl cm_xt cm) has a magnetic moment 1.Sxto-23 A- m 2 • What
will be the magnetic moment of bar in the state of magnetic saturation ?
Deiisityofiron 7.78xto3 kg/m 3
Atomic weight ofiron A 58
Avogadronumber,N = 6.02xto 23 tg-mol.
Sol. The nul1_lqer o.f a\oms per unit volume in a specimen

N N pN
n = -=-=-
V' A A
p

7.8xl0 3 x6.02xl023
56

8.38xl0 28 / m3
Total number of atoms in the bar

nV =8.38xl0 28 x(5xlxl)x10-6

4 .19xl023
'
The saturated magnetic moment
Magnetic moment of each atom x total number of
atoms in the bar
l.8xl0-23 x4. 19xJ023
7.54A-m2. Ans.

Ex. 16 A toroid having500 turns of wire and a mean circumferential length.of SO cm carries a cµrrent
of0.3A. The relative permeability of the core is 600.
(a) What is the magneticfield in the core?
(b) What is the magnetic intensity?
(c) What part of the magnetic field is dueto surface currents ?
Sol. (a) The magnetic field in the core of toroid

µNi]-- µo Ni
B = [ 21tR µ, 2itR

60ox( 41tx10-7 )x500x0.3


0.50
0.2262T Ans.
B Ni
(b) H -=--
µ 2itR

500x0.3
0.50 Ans.
(c) ~
41tx10-7 x-300
0,0003768T
We know that,
Bo+Bm
B-B0
0.2262-0.0003768
0.2258T Ans.
Ex. 1 7 The current in the windings on a toroid is 2.0 A. There are 400 turns and the mean circumferential
length is 40 cm. With the aid of a search coil and.charge-measuring instrument, the magnetic
field is found to be 1.0 T. Calculate
(a) the magnetic intensity
(b) the magnetisation
(c) the magnetic susceptibility
(d) the equivalent surface current, and
(e) the relative permeability
µNi
Sol. We have, B
2n:R

µx400x2.0
9r 1.0
0.40
which gives µ 0.0005
J:.. 0.0005
µ,. µo 4n:x10-7
398 Ans(e)

B
Magnetic intensity, H -=---
µ 0.0005
2000Am-1 Ans(a)
We know that

7.94x10 5 A-m Ans(b)


Magnetic susceptibility

I 7.94xl05
X 397 Ans(c)
H 2000

Ex. 18 A bar magnet has a coercivity of_ 4x10 3 Aim. It is desired to demagnetize by inserting it inside
a solenoid 12 cm long and having 60 turns. What current should be carried by the solenoid?
Seil. We have
H ni

H 4x10-3
i
n
( 0~~2)
8A Ans.
Ex. 19 Find (a) the magnetisation/, (b) the magnetic intensity Hand (c) magnetic field Bat the centre of
a bar magnet having pole strength 3.6 A-m, magnetic length 12·cm and cross-sectional area
0.90 cm2•
m 3.6
Sol. (a) Magnetisation, I s(
A 0.90xl0-4

4xl04 A/m
= Ans. ., _Fig. 7.52
(b) Magnetic intensity, due to north pole

l. 3.6
79.6Nm
4
1t (6x10-2 J2
Similarly If, ~6.6Nm
Resultant magnet intensity Fig. 7.53
. H = Hn+H,
159.2 Nm towards the south pole Ans.
(c) The magnetic field ii at the centre is

ii µ 0 (l+E)

( 4itxl0~7 )( 4xl04 -159.2) Fig._ 7.54.


'
5.0xl0~2 T Ans.
The field is towards north pole

Ex. 20 A moving coil galvanometer has a coil of area A and number of turns N.A magnetic field Bis
applied on it. The torque acting on it is given by,:= k i where i is current through the coil. If
moment of inertia of the coil is/ abontthe axis ofrotation · ·
(a) Find the value of kin terms of galvanometer parameters (N, B, A).

7t
(b) Find the value of torsional constant if current i0 produce angular deflection of radian,
2
(c) If a charge Q is passed almost instantaneously through coil, find the maximum angular
deflection in it. ·
Sol. · (a) The torque exerted.by magnetic field on the coil of galvanometer , ·
., .. ,: NiAB ..
Given ,: ki ...
ki Ni AB
or k-NBA Ans. '
(b) ·If C is the 'torsional constant of the head, then

2NAB io
or C Ans.
7t
\'

(c) If 00 is the maximum deflection, then by conservation of energy

_!_c05 = _!_ I ci
2 2

or 0o = Jf oo ... (i)

We know that 't = Ni AB ... (ii)


If L is the angular momentum of the coii, then L =I oo, and equation (ii) can be written as
dl NdQ AB
dt dt
or dl NABdQ
On integrating, we get
L NABQ
or loo NABQ
NABQ
or 0)
I
Substituting this value in (i), we get
&Wtd4e 7,1
1. The figure shows graphs of the electric field 4. The magnetic dipoles in a diamagnetic material
magnitude E versus time t for four uniform electric are represented, for three situations. The three
fields, all contained within identical circulator situations differ in the magnitude of a magnetic
regions. Which of them is according to the field applied to the material. In which situation
magnitudes of the magnetic field the magnetisation of the material is the
greatest : 1,
greatest :
E
----
----
- - ...... A

----
_,......,_ B
............
/
----
_........
---- C
(a) A (b) B
(a) A (b) B
(c) C (d) Equal in A, Band C.
(c) C (d) D
5. The magnetic susceptibility is negative for
2. An electron in an external magnetic field Bext (a) diamagnetic substance
(b) paramagnetic material
has its angular momentum i antip~;allel to Bext .
(c) ferromagnetic material
If the electron undergoes a spin flip so.that i is (d) all the above.
then parallel with Bext , then 6. Electromagnets are made of soft iron because
soft iron has
(a) energy is supplied to the electron
(a) low retentively and low coercivity
(b) energy is lost by the electron
(c) energy neither supplied nor lost · (b) low retentively and high coercivity
(d) none of them. (c) high retentively and low coerrity
3. Figure (a) shows a pair of opposite spin (d) .high retentively and high coerrivity.
orientations for an electron in an external A compass which is allowed to move in a
horizontal plane is taken to a geomagnetic pole.
magnetic field Bext . Figure (b) gives three
It
choices for the graph of the potential energies
(a) will stay in north-south direction only
associated with those orientation as a function
(b) will stay in east-west direction only
of magnitude of Bext . Choices B and C consist (c) will stay in any position
of intersecting lines, choice A of'parallel lines. (d) none of these
Which is the correct choice :·,,· 8. · A tangent galvanometer is connected directly to
u an ideal battery. If the number of turns in the coil
B ''
is doubled, the deflection will
Ai 1
(a) increase (b) decrease
L - i.
Box,
-.-Jext ,41,,
(c) remain same (d) any of these
J._ 9. Two short magnets of equal dipole moment M
ri J are fastened perpendicularly at their centres. The
(a) (b) magnitude of the magnetic field at a distance d
(a) A,A (b) B,B ·r from the centre of the bisector of the right angle
(c) C,C (d) A,A;B,B. is
µo M N 15. A magnet of magnetic moment 50i A-m2 is
(a)
41t d 3 " placed along the _x-axis in a magnetic field
'·-..d 1i =(o.5? +3.o])T.
µo ,/2 M ......,. The torque acting on the
(b)
41t d3 N1 ,s magnet is

µ 0 2M .
···- ..._.., (a) 175kN-m (b) 150kN-m
(c)
41t d3 0

L (c), 1if N-m (d) 25,ffif N-m


µo 2{2M
s 16. The magnetic field lines due to a bar magnet are
(d) correctly shown in
41t d3
10. A very long bar magnet is placed with its north-
pole coinciding with the centre ofa circular loop
carrying an electric current i. The magnetic field
due to the magnet at a point on the periphery of
the loop is B. The radius of the loop is a. The (a) (b)
force on the loop is
(a) nearly 21taiB perpendicular to the plane of
the loop
(b) 21taiB in the plane of the loop
(c) 1taiB along the magnet
(d) zero.
11. An iron rod oflength Land magnetic moment M
is bent in the form of a semicircle. Now its
(c) (d)
magnetic moment will be
2M
(a) M (b)
1t
17. A very small magnet is placed in the magnetic
M meridian with its south pole pointing north. The
(c) (d) M1t
1t null point is obtained 20 cm away from the centre
12. A magnetic needle lying parallel to a magnetic of the magnet. If the earth's magnetic field
field requires Wunits of work to turn it through (horizontal component) at this point be 0.3 gauss,
60°. The torque required to maintain the needle the magnetic moment of the magnet is
in this position will be
(a) s·.oxl0 2 e.m.u. (b) l.2xl0 3 e.m.u.
(a) ,.fiw (b) w ' ·-
(c) ,2.4xl0 3 e.m.u. (d) 3.6xl0 3 e.m.u.
,./3 w
(c) 2 . (d) 2W 18. At a place, if the earth's horizontal and vertical
components of magnetic fields are equal, then
13. A magnet of magnetic moment M is rotated .'
the angle of dip will be
through 360° in a magnetic field H, the work done
(a) 30° (b) 90°
will be
(c) 45° (d) 0°
(a) MH (b) 2MH
(c) ·21tMH (d) Zero . · 19. A short magnet of moment 6.75 Am2 produces a
1 neutral point on its axis. Ifhorizontal component
14. Force between two identical bar magnets whose
centres are r metre apart is 4.8 N, when their axes of earth's magnetic field is 5x!O-s Wb/m 2 ,
are in the same line. If separation is increased to
then the distance of the neutral point should be
2r, the force between them is
(a) · 2.4N (b) 1.2N (a) 10cm (b) 20cm
(c) 0.6N (d) 0.3 N (c) 30cm (d) 40cm
\ 'c\
20. Time period of a freely suspended magnet does
0.34x!0-4T is along the plane of the coil. The
not depend upon
magnetic intensity is
(a) Length of the magnet
(b) Pole strength of the magnet "' (a) i.96xl0-4 T
(c) Horizontal component of earth's magnetic
field (c) l.96xl0 4 T
(d) Length of the suspension thr~'ad 27. The length of a magnet is large compared to its
21. Two magnets of same size and ITia·ss make width and breadth. The time period of its
0

respectively IO and 15 oscillations per minute at oscillation in a vibration magnetometer is 2 s.


certain place. The ratio oftheir magnetic moments The magnet is cut along its length into three
iS 'JI<'.. equal parts and three parts are then placed on
(a) 4:9 (b) 9:4 i,; ·' each other with their like poles together. The time
(c) 2: 3 (d) 3: 2 ·" period of this combination will be
22. The period of oscillation of a magnet in vibration (a) 2 s (b) 2 / 3 s
magnetometer is 2 sec. The period of oscillation (c) 2../3 s (d) 2/ .,J3s
of a magnet whose magnetic ITloment is four times
28. Relative perme,ability of iron is 5500, then its
that of the first magnet is
mangetic susceptibility will be
(a) I sec (b) 4 sec
(a) 5501 (b) 5499
(c) 8 sec (d) 0.5 sec
(c) 5500 x !07 (d) none of these
23. A magnetic needle suspended by a silk thread is
(d) Paramagnetic and ferromagnetic materials
vibrating in the earth's magnetic field. If the
29._ Curie temperature is the temperature above which
temperature of the needle is increase~ by 500°C,
(a) a paramagnetic material becomes
then '
ferromagnetic · ·· ·
(a) the time period decreases
(b) a ferromagnetic' material becomes
(b) the time period remains unchanged
paramagnetic
(c) the time period increases
(c) a, paramagnetic material becomes
(d) the needle stops vibrating
diamagnetic
24. The time period of oscillation of a bar magnet
(d) a ferromagnetic material becomes
suspended horizontally along the magnetic
diamagnetic
meridian is T0. If this magnet is replaced by
30. Liquid oxygen remains suspended between two
another magnet of the same size and pole
poles faces of a.magnet because it.is
strength but with double the mass, the new time
(a) diamagnetic' (b) paramagnetic
period will be
(c) ferromagnetic (d) antiferromagnetic
To 31. A bar magnet is placed north south with its north
(a) To (b)
2 ..fi_ pole due north. The points of zero magnetic field
will be in which direction from the centre of the
(c) ..fi.T0 (d) 2T0 magnet
25. A magnet makes 40 oscillations per minute at a (a) north and south
place having magnetic field intensity of (b) east and west
(c) north-east and south-west
O. !x!0-5T. At another place, it takes 2.5 sec to
(d) north-west and south-east.
complete one vibration. The value of earth's 32. Two identical magnetic dipoles of magnetic
horizontal field at that place is moments 1.0 A-m2 each, placed at a separation
(a) 0.25x!0-6T (b) 0.36x!0-6T of2m with their axis perpendicular to each other.
The resultant magnetic field at a point midway
(c) 0.66x!0-8T (d) l.2x!0-6 T between the dipoles is
26. The magnetic needle ofa tangent galvanometer
is deflected at an angle 3 0° due to a magnet. The (a) 5x!0-7 T (b) --/sxJ0-7 T
horizontal component of earth's magnetic field (c) 10-7 T (d) None of the_se
33. Two identical short bar magnets, each having 36. A small coil CwithN=200 turns is mounted on
magnetic moment M, are placed a distance of2d one end of a balance beam and introduced
apart with axes perpendicular to each other in a between the poles ofan electromagnet as shown
horizontal plane. The magnetic induction at a in figure. The cross sectional area of coil is
0
- -iroint midway between them is A = I.O cm2, length of arm OA of the balance
j,eam is f = 30 cm. When there is no current in
(a) (b) the coil the balance is in equilibrium. On passing
f
a current = 22 mA through the coil the equilibrium
is restored by putting the additional counter
(d) weight of mass !J.m = 60 mg on the balance pan.
Find the magnetic induction at the spot where
34. The true value of angle of dip at a place is 60°, coil is located.
the apparent dip in a plane inclined at an angle
of30° with magnetic meridian is
-I 1
(a) tan - (b) M
2
0
l·"c
(c) (d) None of these ' 11! """\re::
B
71
-S--,V
35. A bar magnet of magnetic moment 3 .0 A-m2 is (a) :·o.4 T (b) 0.3 T
placed in a uniform magnetic field of2 x I o-5 T. (c) 0.2 T (d) 0.1 T
If each pole of the magnet experiences a force of
6 x 1o-4 N, the length of the magnet is
(a) 0.5 m (b) 0.3 m
(c) 0.2m (d) 0.1 m

,1


Directions (Qs. 1-5) : Read the following questions 5. Which of the following statements are true about
and select the right choices. More than 'one options the magnetic susceptibility Xm of paramagnetic
may be correct. y_
substance
1. The figur, shows two diamagn.~tic spheres
located near the south pole of a bar magnet (a) value of Xm is inversely proportional to
)R
~--~
s );IoI o2 the absolute temperature of the sample
(b) Xm is positive at all temperature
(a) The force on sphere I is di.rected towards
the magnet tdl b (c) Xm is negative at all temperature
(b) The force on sphere 2 is directecl away from
the magnet -'.'",h''} (d) Xm does not depend on the temperature of
(c) The magnetic dipole mome~i' 6f sphere is the sample
directed towards the magnet
,, ,--·------ ·-- --
Passage for Q. 6 to Q. 8
-·----------- -1
(d) The magnetic dipole moment of sphere 2 is
directed away from the magnet. \\n electron with kinetic energy Ke travels in a circul~
2. Figure shows a loop model (loop L) for a ~ath that is perpendicular to a uniform magnetic field)
diamagnetic material ,: - · -1 (he electron's motion subject only to the force duet~
the field.
L ..__ -
·. .
- - ·-··-·------~- - --~- ---------·,,.,_
j
6. The magnetic dipole moment of the electron due
sG to its orbital motion has magnitude :

KeB
(a) The net dipole moment of the loop.directed (a) K,fl (b)
2
towards the magnet ,,
(b) The net dipole moment of the.loop directed (c) Ke/ B (d) Ke/2B
away from the loop 7. The magnetic dipole moment of a positive ion
(c) The loop gets attracted towards the magnet with kinetic energy K under the same
(d) The loop gets repelled by the magnet. circumstances :
3. A ferromagnetic material is placed in aii'extemal (a) K;B (b) K;f B
magnetic field. The magnetic dom~ln (c) K;f2B (d) Zero
(a) may increase in size
8. An ionized gas consists of 5.3 x 1021 electrons/
(b) may decrease in size .n.
m 3 and the same number density of ions. Take
(c) retain their size
,,r' the average electron kinetic energy to be
(d) nothing can be said.
n-,:i.
4. Mark out the correct options. r: 6.2 x I 0-20 J and average ion kinetic energy to
,
(a) Diamagnetism occurs in all,)Daterials
be 7.6xI0-21 J. The magnetisation of the gas
(b) Diamagnetism results from· the partial
alignment of permanent magnetic moment. when it is in a magnetic field of 1.2 T is
(c) The magnetizing field intensity His always (a) 150A/m (b) 215A/m
zero in free space. i~ J'· · (c) 295A/m (d) 3!0A/m
[1(.
(d) The magnetic field of induced magnetic
moment is _opposite to the applied field.
• • • 0 • • • • • -~ • • ' ' ·_

Read the twostatemimts carefully to mark the correct option out of the options given below. Select thenght
choice. ·· · , · · · · · '
(a) If both· the statements are true. and the Statement: 2 Is the correct expianation of Statement - 1'. · ·
(b) Ifbolh the statements are true, but Stateme.nt - 2 is n<>I the correct explanation'of tlie Statement - 1. ·
(c) If Statement- I true butStatement-2 is false. . . . ·
(d) If Staiement - 1 is false but Stalement
.. - 2 is true. .

1. Statemeni-· 1 7, Statement - 1
·The poles of magnet can not be ·separated .by . Reduction factor (k) of a tangent galvanometer
breaking into two pieces. . helps i.iu~ducing deflection to current.
Statement - 2 Statement - 2
The magnetic moment will be reduced to half Reduction factor increases with increase of
' . ln!
current.
. when a magnet is broken into two equal pieces.
8. Statemeni- 1
2. Statement - 1
Paramagnetic substances possess the property
When radius of a circular current carrying loop of diamagnetism.
is doubled,. its magnetic moment becomes four Statement - 2
times. Diamagnetism is the universal property, it is
Statement - 2 present in all substances.
Magnetic moment is given by M=. iA. 9. Statement- 1
3. Statement- 1 The ferromagnetic substance do not obey Curie's
Gauss's law is not applicable in magnetism. law.
Statement - 2 Statement -2
Isolated pole does not exist. ,. ~t Curie point a ferromagnetic substance start
· behaving as a paramagnetic substance.
4, Statement - 1
10. Statement- I
We cannot think of magnetic field configuration ,· For making permanent magnets, steel is preferred
with three poles. over soft iron.
Statement - 2 .Statement-2
-.
0

A bar magnet does not exert a torque on itself Steel possesses high retentivity.
due to its own field. 11. Statement- 1
5. . Statement -1 Soft ircill is used as transformer core.
The earth's magnetic field is due to iron present Statement -2
in its core. Soft iron has low hysteresis loss.
12. Statement- 1
Statement - 2
To protect any instrument from external magnetic
At a high temperature magnet losses its
field, it is put inside an iron body.
magnetism.
Statement - 2
6. Statement - 1 Iron has high permeability.
The tangent galvanometer can be made more 13. Statemeiit-1
sensitive by increasing the number ohums of ,The sensitivity ofa moving coil galvanometer is
its coil. increased by placing a suitable magnetic material
Statement - 2 as a coreinside the coil.
Current through galvanometer is proportional to St~t~m~nt - 2
the number oftums of the coil. Soft iron has high magnetic permeability and
c.annot"l,~ easily magnetized or demagnetized.
·1 , ,,/~-ir:c
"

t4~
a5fnet1sm
~
-
,-~ - -- ~ , _ _ ,•,_,,,•_ -.O"e _ _ _ ;.;.!,.Z:..,.; ·=~-' ••' C:;'--;;"'::,:..:;,.~··-=::;.&.'::,;-:::::;;::_~,....=.:;.;:_;;._:;..•..;:.,;;:;'C:-','",,;.":.f ""'?; ~-.::=.~=.,-,<=:..,-.._:_"~-""O;~C"<

!!I 1 l (b) _ _,I 4_ l Ji .. 7 !i (c) Ji


(c) 10 JI __ (~) Ji_ _1~_ __ JI ___(ct_J ·
:'I 2 [_ ~ J- 5 :i._(d).Ji .8 ii (a) II 11 I _(Cl_)_ JI...... - Ii . J
:1 3 .1 (a) 'I 6 :I _(bLJI . 9_ JI. .J~) JI 1 _ I _(a) IL_
2 ii ____ - _I
~'-·-~·-·"-"·-·~--~--·~------ &Ulrd4e 7.4
neti?rtt~ < 'l"~i!.ii1: •· ·suBJEc.T"1·V·E
~9·.'
•. \~
-
1. At a certain place the horizontal component ofearth's mag~etic field is Ji times the vertical component
What is the angle of dip at that place ? Ans: 30°.
2. A bar magnet of magnetic moment 2.0 A-m2 is free to rotate about a vertical axis passing through its.
centre. The magnet is released from rest from easi-west position. Find the kinetic energy of the_ magnet
as it takes north-south position. [Horizontal component of earth's field is 25 µT]. Ans: 50 µJ.
· 3. A magnet is suspended in such a way that it oscillates in the horizontal plane. If it makes 20 oscillations
per minute at a place where dip angle is 30° and 15 oscillations per minute at a place where dip angle is

16
60°. Find the ratio of total earth magnetic field at the two places. Ans:
9
Ji
4. Find the magnetic field due to a dipole of magnetic moment 1.2 A-m2 at a point I m away from it in a
direction making an angle of 60° with the dipole axis.

Ans: l.6xl0-1 T at an angle a with the radial line where a= tan-I ( ~).

5. A magnet is IO cm long and its pole strength is 12 A-m. Find the magnitude of the magnetic field B at a
point on its axis at a distance 20 cm from it. Ans: 3.4x10-5 T.
6. A dip circle shows an apparent dip of 60° at a place where the true dip is 45°. If the dip circle is rotated
through 90°, what apparent dip will it show? Ans: coi- 1(0.816).
7. The needle of a deflection galvanometer shows a deflection of60° due to a short bar magnet at a certain

distance in tan A position. If the distance is doubled, what will be the deflection ? Ans: tan-I ( ~)

8. A moving-coil galvanometer has 100 turns and each tum has an area 2.0 cm2• The magnetic field produced
by the magnet is 0.01 T. The deflection in the coil is 0.05 radian when a currentof!O mA is passed through
it. Find the torsional constant of the suspensicn wire. Ans: 4.0x 10-5 N-m/rad.
9. Two long bar magnets are placed with their axes coinciding in such a way that the north pole of the first
magnet is 2.0 cm from the south pole of the second. If both the magnets have a pole strength of 10 A-m,
find the force exerted by one magnet on the other. Ans : 2.5 x 10-2 N.
10. A bar magnet has a length 8 cm. The magnetic field at a point at a distance 3 cm from the centre in the
broad side-on position is found to be 4x I 0-6 T. Find the pole strength of the magnet.
Ans: 6>:< 10-5 A-m.
11. A magnetic needle is free to rotate in a vertical plane which makes an angle of 60° with the magnetic

meridian. !fthe needle stays in a direction making an angle of tan-I (}) with the horizontal, what

would be the dip at that place ? Ans: 30°.


12. An iron rod of volume 1()-4 m3 and relative penneability 1000 is placed inside a long solenoid wound with
5 turns/cm. If a current of0.5 A is passed through the solenoid, find the magnetic moment of the rod.
Ans: 25A-m2•
13. Assume that each iron atom has a pennanent magnetic moment equal to 2 Bohr magnetons (1 Bohr

magneton = 9.27xl0-24 A-m2). The density of atoms in iron is 8.52x 10 28 atoms/m3 •

(a) Find the maximum magnetisation fin a long cylinder of iron.


(b) Find the maximum magnetic fieldB on the axis inside the cylinder.
Ans: (a) l.58xla6 Alm (b)2.0T.
14. An ideal solenoid having 40 turns/cm has an aluminium core and carries a current of2.0A. Calculate the
magnetisation I developed in the core and the magnetic field B at the centre. The susceptibility of
aluminium=2.3xl0-5• ' Ans:1=0.l8Alm."
15. The susceptibility of annealed iron at saturation is 5500. Find the permeability of annealed iron at
saturation. Ans: 6.9x 10-3•
16. Imagine rolling a sheet of paper into a cylinder and placing a bar magnet near its end as shown in figure.
(a) Sketch the magnetic field lines that pass through the surface.of the cylinder.

(b), What can you say about the sign of B.dA for every area dA on the surface?
(c) Does this contradict Gauss's law for magnetism ?

( ~ · ~ ) - - - ~ ~ Central axis

Ans: (b) sign is minus; (c) no, there is compensating positive flux through open end near magnet.
17. ·A compass needle made of pure iron (with density 7900 kg/m3) has a lengthL of3.0 cm, a width of 1.00
mm, and a thickness of 0.50 mm. The magnitude of the magnetic dipole moment of an iron atom is

M Fe = 2. lxl0-23 J IT. If the magnetisation of the needle is equivalent to the alignment ofl0% of the

atoms in the needle, what is the magnitude of the needle's magnetic dipole moment M?
Ans:2.7xlQ-3 JIT.
18. A short magnet oscillates in an oscillation magnetometer with a tiine period ofO.!Os where the earth's
horizontal magnetic field i_s 24 µT. A downward current of 18 A is established in a vertical wire placed 20
'cm east of the magnet. Find the new time period. Ans: 0.076 s.
19. A deflection magnetometer is placed with its ann in north-south direction. How and where should a short

magnet having M = 40 A-m2 be placed so that the needle can stay in any position ?
BH
Ans: 2.0 cm from the needle, north pole pointing towards south.
20. A tangent galvanometer shows a deflection of 45° when 10 mA of current is passed through it. If the
horizontal component ofthe earth's magnetic field is BH= 3.6x 10-,; Tand radius .of coil is 10 cm, find the
number of turns in the coil. Ans: 570.
0 CHAPTER
0
0
0
0

Eleclr«magnetic
rt)lff-i,on
8, 1 ELECTROMAGNETIC INDUCTION AN INTRODUCTION

In 1931, Michael Faraday suggested that if electricity moving in a wire produces magnetism, then opposite
might be true; a magnet moving near a wire could produce electricity. He moved a magnet in and out of a coil of
wire, and electricity flowed in the coil. This is called electromagnetic induction. The electric current flows only
while the magnetic field moves or varies. Ifthe magnet and coil are still, no current flows in the coil. Electromagnetic
induction is used in hundreds of machines and devices, like electric motors, generators etc. The energy needed
to tum the magnet is provided by energy sources such as steam, moving water or the wind etc.

8.2 MAGNETIC FLUX

In fluid mechanics, we defined a quantity; rate of fl.ow, Q = vAcos0, in electrostatics; an electric flux,
<l>e = EAcose. On the same way here we can define magnetic flux. The magnetic flux through small element of
surface dA is defined as;
B1.dA
(Bcos0)dA
iJ-dA
The total m·agnetic flux through the surface

<1>B = Jil-dA. Fig. 8,1

When iJ is uniform over a plane surface with total area A,


$8 = B1.A=BAcos8.
More about f8 ,;

I. We may take dA positive, pointing out of the surface.

9l! dA
dA positive dA positive
(a) (b)
Fig. 8.2
2. Magnetic flux is a scalar quantity, but it may be negative.
3. Its SI unit is tesla-m2, which is called the weber. Thus
I weber = I Wb = IT-m2.
4. If the elemental area of dA is right angles to the field lines, then

B = d<!>e
dA
Thus, the magnetic field is equal to the flux per unit surface area at right angle to the magnetic field, and
so it is also called magnetic flux density. Its unit is weber/m2 (Wb/m2).
;,
ii

5. A
I=
I l 77 B



0 = O ;•lie= BA cos 0° 0 90 ; q,8 = BAcos 90° 11


=lltl =O ii .
e= 180°, <j,8 = BA cos 180°
=-BA
Fig. 8.3

8.3 FARADAY'S LAW OF EMI


According to Faraday, whenever there is change in magnetic flux linked with the circuit, there induces an emf in
the circuit. The rate of change of magnetic flux is equal to the induced emf. Thus

Le = -~-,]
Here negative sign indicates that induced emf opposes the change in flux.

Total magnetic flux (Magnetic-flux linkage)


!fa closed loop in which an enifis induced contains not one but N turns, then induced emfe will be equal to the
sum of the emfs induced in each tum. And if the magnetic flux associated by each tum is the same and equal to
<j,8 , the total flux <!>Total through the surface extended over such a complex loop can be represented by
<!>Total = N<j> B·
This quantity is called the total magnetic flux, or the magnetic flux-linkage. In this case, the emf induced in the
loop is defined by the formula

e
_ d<i>Total = -N d<l>e _
dt dt
E:LECTROMAGNETl!;]/':JDC,CTION

I. · When the field B varies with time as well as the configuration or arrangement of the loop in the field, i
the induced emf should be calculated by taking in to consideration of two factors. So induced emf can :
be written as :

e = pE·dR =- ~; + p[vxB}dc
Here the first term is due to the time variation of the magnetic field, while the second is due to the ·
motion ofthe loop. '
2 When magnetic field changes with both time and space:

.3. An emf is always induced whenever there is change in magnetic flux in the circuit. But current is :
induced only in closed circuit.

1
8.4 LENZ S LAW

Lenz's law is used to get the direction of induced current. Which means that we can apply it directly only to a
closed conducting loop. According to Lenz's law an induced current in a closecl conducting loop will appear in
such a direction that opposes the change which produces it.
More about Lenz's law
Whether we push the magnet towards the loop or pull it out from the loop, we shall always experience a resisting
force and thus will have to do work. From the principle of conservation of energy, this work must exactly equal
to the thermal energy that appears in the coil because there are only two energy transfer that take place in this
isolated system. The faster we move the magnet, the more rapidly we do work, and thus greater the rate of
production of thermal energy in the coil. If we cut the loop and then do experiment, there will be no induced
current, no thermal energy, no resisting force on the magnet and no work requires to move it.
To understand Lenz's law, let us apply it to a specific case; namely, the first of Faraday's experiments.
Lenz's law 1 :
Let us consider a magnet with its north pole facing a closed conducting loop moves towards it or with the south
face moves away from it.
When magnet moves towards the closed loop, the near face of loop becomes north, so that it repels the
incoming magnet (according to Lenz's law), and therefore the induced current in the loop will induce in counter
clockwise direction.

. _S(___N....,Q-
(a) (b)
Fig. 8.4
lfthe magnet moves away from the closed loop with its N-pole facing it, the facing side of the loop behaves like
·south pole and opposes the receding magnet. The direction of induced current in the loop will be clockwise.
Lenz's law 2 : ; ,_,_.
When the magnet moves with north pole towards the closed conducting loop, the flux through the loop
increases, the induced current, therefore will be in such a direction; its magnetic field produces flux in opposite
to the flux of the magnet.

s N Field of S N.
F::::<--induced
current

(a) (b)

Fig. 8.5
When magnet with its north pole moves away from the loop, the f\ux through the loop decreases. The induced
as
current in the loop, try to compensate this decrease in flux and therefore will be in the direction shown in the
figure.

Inthe experiment, we ha:e moved the ma~et, but the ~ame effect will be observed.if we move the lo~p I•

andkeepthemagnetati::st-''.. . . . . . . .. •J",
.
"2. , The .inauced magnetic field" ,foes not. oppose the magnetic field' of the magnet but it opposes· the·
!I change in this field. ~---·--~-J
Flux change and induced emf
The magnetic flux, <j, = BAcos0 has three variables, and so flux will chan~e if either of them changes. Thus :·

(i) . When o· alone is changing, . e = Acosa( !}


(ii) When A alone is changing, e = Bcosa( :~}

(ili) When 0 alone is changing, e =


BA dcosa.
dt
In case of closed loop, a current will induce. If R is the resistance of the looj>, then induced current

The induced charge in the loop

dq iindt=[1Jdt=~

or ,.&q . ~~.:.:]
Ex. 1 A.closed conducting loop is placed in a uniform magnetic field as X X X X

shown infig. 8.6. Find the direction ofinduced current in the loop
when
(i)
(ii)
magnetic field is increasing with time
magnetic field is decreasing with time.
:Q:.
X X X X
Fig. 8.6
Sol. (i) In case when field increases, the flux of which increases into the page and so the field of the
induced current will be in upward direction. For this the current in the loop will be in
counterclockwise direction.
(ii) The direction of induced current will be in clockwise.

Ex. 2 A closed conducting loop is placed in uniform magnetic field which points into the plane of the
loop. What will be the direction of induced current when loop is expanding.
X X X X

·o
X

X
X

X
X

Fig. 8.7
xB

Sol. When area of the loop increases, the flux will increase into the page, and so the field of induced
current must be out of the page. For this the direction ofinduced current will be counterclockwise.

Ex. 3 Two closed conducting loops are placed parallel to each other. The current in one of the loops
starts increasing. Will the loops attract or repel?

Loop 1 Loop2
Fig. 8.8
Sol. When current in first loop increases, the flux associated in second loop is also increases. !o
compensate this increase in flux the loops must move away from each other; means they will repel
each other.

Ex.4 In which case the flux changes and an emfinduces?


(i) If magnet is rotated about its axis in the loop.
(ii) Loop is placed at rest in uniform field.
(iii) A closed loop moves in uniform field.
(iv) Loop is placed with its axis paralleHo a current carrying conductor.
i l c..._ ....~·

Sol. The flux change in each case is zero, and so induced emf is zero. Fig. 8.9
Ex. 5 Figure 8.JOshows a conducting loop consisting ofa half circle of radius r= 0.20 m and three
straight sections. The half-circles lies in a uniform magnetic field B that is directed out of the
page; the field magnitude is given by B = t2 + 2t+ 5, with Bin tesla and tin second.An ideal battery
with emf~ =2Vis connected to the loop. The resistance of the loop is 20 •

• • • • • • •
• • . .. • • •
• • • • •
• • ·········•····
.......... • •' t •
·····
r/2
'-_-ll-+~----' .i.

Fig. 8.10
(a) What are the magnitude and direction of the emf induced around the loop by field at
t= 10s?
(b) What is the current in the loop at t= 10s?
Sot (a) The flux of the field through the halfloop (upto the field)

According to Faraday's law

2
dcl> 1tr d 2
Iel = dt=-2- dt (t +21+5)

1tr2
-(21+2)
2

2
it(0.2) (2x10+2)
2
1.38 V. Ans.
(b) The induced emf causes the current in clockwise direction around the loop; the battery emfs
tends to drive a current in counter clockwise direction. Because Sbat is greater than Sind• and
so the net emf Snet is counterclockwise. Thus

Snet
i =
R

2-1.38
2
0.31A Ans.
'iLECTROM~GNETle·INDUCTION

Ex. 6 Space is divided by the line AD into two regions. Region I is field free and region II has a uniform
magnetic field B directed into the plane of the paper. ACD is a semicircular conducting loop of
radius r with centre at O, the plane of the loop being in the plane of the paper. The loop is now made
to rotate with a constant angular velocity Ol about an axis passing through O and perpendicular to
plane of paper. The effective resistance of the loop is R.

Region I Region II
A X
X X X

X X X

X
B B
C X X

X X X X X X

DX X X X X X

(a) (b)
Fig. 8.11
(i) Obtain an expression for the magnitude of induced current in the loop.
(ii) Show the direction of current when the loop is entering into region II.
(iii) Plot a graph between the induced emfand the time ofrotation for two periods ofrotation.
Sol. (i) Suppose the loop is rotated by small angle d0 in time dt. As it is rotated with constant angular

velocity, so d8 =rodt. The area ofthe loop into the field, dA = ..'..r (rd0) =Cda. The magnetic
2 2
flux through this area
dq, = BdA
2
B!.._d0
2
According to Faraday's law, induced emf

dq, Br 2 d0
e = --=-----
dt 2 dt

d0
As (J)
dt

Br2
lei --ro. Ans.
2
(ii) When loop enters into the magnetic field, the flux into it increases. By Lenz's law the induced
current in the loop will be anticlockwise so as to compensate the increasing flux. After each
T . .
2 the loop starts coming out from the magnetic field, and so the direction of current reverses,
i.e., it becomes clockwise.
2 •
(iii) The magnitude of the induced emf e = Br OJ is constant. It changes in sign each after
2
21t
T = OJ =~. The gr~~h b.etween induced emf and time is shown in jig. 8. 12.
2 2 OJ .
n· '.
2
Br 0J
2
''
.
.
:
01-~--~--··' ~ ~ - ~ ~ - '- ~ - (
: 1t/oo : 21t/oo : 31t/o> 41r/oo
Br2 ro '
t---
2
Fig. 8.12

Ex. 7 A plane loop shown in jig. 8.13 is shaped as two sqnares with sides a= 0.20 m and h =0.10 m and
is introduced into a uniform magnetic field at right angles to the loop's plane.

d
b
,,,.
II

Fig. 8.13
The magnetic induction varies with time as B = B 0 sinOJt where B0 = lOmT and OJ = 100
rad/s. Find the amplitude of current induced in the loop if the resistance per unit length is equal
top= SO milli ohm/m. The inductance of the loop is negligible.
Sol. The loops are connected in such a way that if the current is clockwise in I loop, then it will be
anticlockwise in the II loop (see figure). The emf in loop I
2
dlj> =d(Ba )
dt dt

a 2 ~ ( B0 sin OJI)
dt
B0a 2 0J cos OJI
Similarly, the emfin the II loop,

b 2 -d ( B0 sinOJI ) ·
. dt

The net emf in the circuit


enet el - e2

s0 (a 2 -b 2 )0JcosOJI.
Given resistance per unit length of the loop wire is p, thus total re_sistance
R = 4(a+b)p.
The induced current

enet
R -

Bo (a 2 -b 2 )co cos cot


4(a+b)p

B0 (a-b)cocoscot
4p
The amplitude of the current

B0 (a-b)co
4p
On substituting the values given, i0 = 0.5 A. :,: Ans.

Ex. 8 Figure a:u illustrated plane figures mad~ of thin conductors are located in a uniform magnetic
field directed away from a reader beyond the plane of the drawing. The magnetic induction starts
diminishing. Find how the currents induced in these loops are directed.

8©© (a) (b)


'
(c)
I
(d)
Fig. 8.14
Sol. As magnetic field is decreasing into the plane of the figure, so induced current will compensate this
decrease by flowing in clockwise direction. Thus
(a) in round conductor the current flows clockwise, there is no current in the connector.
(b) in the outside conductor, clockwise;
(c) in both round conductors, clockwise : no current in the connector.
(d) in the left-hand of the figure eight, clockwise.

Ex. 9 A magnet falls into (a) closed conducting loop; (h) open conducting loop. Discuss the motion of the
magnet.

~ []
c:J;D ~
(a) (h)
Fig. 8.15
Sol. (a) When magnet falls into the loop, the induced current in the loop will be in such a direction
that the facing side of the loop opposes the incoming and outgoing magnet. So when north
pole approaches towards the loop, the facing side of the loop becomes north and the
acceleration of the magnet becomes less than g.
(b) Ifthere is a cut in the loop ·and magnet falls into it, there will be no induced current in the loop
and hence the acceleration of the magnet remains equal tog.

Ex. 10 Magne_t falls into a long conducting tube. Discuss its motion.
Sol. When magnet falls into the tube, the flux into it changes and so current will induce along its
periphery. Because of the induced current the acceleration of falling magnet becomes less than g.
The acceleration continuously decreases and becomes zero. There after magnet falls with constant
velocity. This can be understand as; the tube can be assumed to be madeup of number of rings.
Each ring decreases the acceleration of falling magnet and ultimately acceleration of magnet becomes
zero. It means initially with the increasing of velocity of magnet, flux changes with increasing rate,
and induced current increases in magnitude, which opposes the motion with greater force. After
falling some distance the rate of change of flux becomes constant, which is enough to provide
retarding force equal to weiglit of the magnet.
,,, ' .

"
'
L

(a) (b)
Fig. 8.16

8.5 MECHANISM OF EMI


Consider a conducting rod of length f moving with constant velocity v which is perpendicular to a uniform
magnetic field B directed into the plane of paper. Let the rod is moving toward right as shown in.fig. 8.17. The
free electrons also move to the right as they are trapped within the rod.
y p p

lr
X X ++X X X
-

r·&-,
X X X X
'1'
X

X
X •

X
l:
evB
--x
X

X
vx
X

X
-

0 X Q
Fig. 8.17
~ • nip · - • •

The ma~etic field ~xerts force ~n ;iie fr~e ele~ions: F~ =· ~~( vxii), so they move to;ard the end Q within the
' . -l. ., • ,
rod. The end P of the rod becomes positively charged while end Q becomes negatively charged, hence.an
• ' , \\ ,, , • 'I ,l

electric field E is set up within the rod which exerts force on the .free electrons in opposite to magnetic force.
Atequilibrium , _ , ,....' ,.,,· · • I ~

0
q I'_, __ \
or -eE+(-e)(vxii) = o.
.or
The inducecl emf across the rod·

i,, • '
or ... (l)
. .
...case. ' under
In. the . consideration
. -. .. . . . : . ·-· ,:

... (2)

Ex. 11 Find the ipduced emfacrossthe ends ofthe co!'ducting rod in the following situations. The length
oftheconductoris£. · ·,.
X X• X

'· : Lv
x~ X X

(a) (b) (c)


·I

XJ?i:X
X

Q_ ..•-····
••••
.. p
V

X, X X

(d) (e)
~ l 'i
Fig. 8.18 ' .,. ·-1 j

Sol. (a) In this. case we can make v perpendicular to length-of the rod or 2perpendicular to v. Thus
e = B(vsin9)£ or Bv(£sin9) ,
~ Bvf sin,8 Ans.
- - (b) --If-we take the plane-of-motion of the rod as xy, then
-~ ; . • • l' •' •; ~ ' I i • _; • ' ; ;!: ; ;. • ~ •

>'. 'i ,,,· : ; -,L


:·,,J
..
'· .- ' ; .;L• 1 ; O' l ' e =·--Jt~7,x(~;J)}-(i1) ' , ! 1,

i .. •

= -vBR (k·t) = o·
(c) e = f[vtxBt}(e]) = 0
(d) e = f(v]xBt)•ti
= vBf(-k·t) =0
(e) If PQ line makes 8 with the velocity vector, then
e = B(vsin8)PQ.

Ex. 12 A closed loop of wire consists of a pair of equal semicircles, ofradius 3.7 m, lying in mutually
perpendicular planes. The loop was formed by folding a plane circular loop along a diameter until
the two halves became perpendicular.A uniform magnetic field B of magnitude 76 mT is directed
perpendicular to the fold diameter and makes equal angles (= 45°) with the planes of the semicircles
as shown in jig. 8.19. The magnetic field is reduced to zero at a uniform rate during a time interval
of 4.5 ms. Determine the magnitude of the induced emf and the direction of the induced current in
the loop during this interval.

Fig. 8.19
Sol. The change in flux in the given time interval ti.I

ti. <j) (1tr 2 ) B ·cos 45°


By Faraday's law, the induced-emf
,.
ti.<j) 1tr 2 B cos 45°
,.,e
tJ.t ti.I

2
1t(3.7) x76xl0-3 xTz

4.5xl0-3

5lxl0-3 V~51 mV Ans.


The direction of the current is clockwise (see figure).

Ex. 13 The dotted circle in jig. 8.20 shows the region in which a permanent magnetic field ii is localised
(it is directed perpendicularly to the plane of the figure). This region in encircled by a fixed
··metallic ringR. By moving the sliding contacts to the other side of the ring, we introduce the
magnetkflux <j) into the closed contour containing a galvanometer G (I-initial position, 2-final
position), Will the galvanometer show a current pulse?
2

,,····
••····•·•··••·•••·•·•··...
)
,'
G

.-··
··-·········

Fig. 8.20

Sol. There will be no induced current. There i~ no curr~ntsince in this case both ! and Lorentz force

are equal to zero : the magnetic field B is constant and the closed loop moves in the region where
there is no magnetic field.

Ex. 14 (a) A small rectangular wire frame falls freely in the space between the wide poles of a sufficiently
strong electromagnet fig. (a).

'
··-O··tru·i
s t·····l······tB
,_, __..,.,, ............. ,,.
~
"', . '

Fig. 8.21 (a)


Show the direction of the currents induced in the frame when the middle of the frame passes
through the positions A, Band C. How will the frame move in these sections ?
(b) A small pendulum consisting of a metal thread, a ball and a sharp point immersed in a cup of
mercury fig. (b) makes part of au electric circuit. The pendulum is placed in the space
.between the broad poles of an electromagnet and swings in the plane perpendicular to the
Jines of force of the magnetic field. During the oscillations the sharp point of the pendulum
point of the pendulum remains immersed in mercury.

Fig. 8.21(h)
ELECTRICI1Y,& MAGNETISM

How will the magnetic field affect the motion of the pendulum? What is the direction of the
currents in the circuit of the pendulum?

t
'~f?
._ _ _ ::::1. _

ffi
Fig. 8.21 (c)
.
·-...
.....::~
····cs...
Fig. 8.21 (d)
(c) A copper wire connected to a closed circuit is surrounded by a thick iron shell fig. (c) and
introduced together with the shell into the space between the poles ofan electromagnet The
iron shell acts as a magnetic screen for the wire.
Will an e.m.f. be induced in the wire?
(d) An aircraft flies along the meridian. Will the potentials atthe ends of its wings be the same?
Will the potential difference change if the aircraft flies in any other direction with the same
velocity?
(e) A rectangular wire frame rotates with a constant velocity around one ofits sides pa~allel to
a current carrying rectilinear conductor nearby fig. (e).

Fig. 8.2l(e)
Indicate the positions in which the maximum and the minimum e.m.f.s will be induced in the
frame.
(I) Two circular conductors are perpendicular to each other as shown in fig. (I).
Will a current be induced in the conductor Aifthe current is changed in the circuitB?
Sol. (a) As the frame passes through the position A the current will flow counterclockwise. When
passing through the position B there is no change in flux through the coil and hence no
induced current. In the position C the current will flow clockwise.
(b) As the pendulum oscillates, the periodic changes in the area enclosed by the circuit will
induce currents in the latter. The induced currents will be directed so that their magnetic field
compensates for the change in the flux of the magnetic lines of force penetrating the area of
the circuit.
When the pendulum swings so as to increase the area enclosed by the circuit the current will
flow counterclockwise, and if the motion is such as to decrease the area enclosed by the
circuit, the current will be clockwise. The interaction of the magnetic field ofinduced currents
with the field ofa permanent magnet will further damp the oscillations of the pendulum.
(c) An emf will be induced because the introduction of the wire into the space between the poles
of the magnet will change the flux passing through the area enclosed by the circuit.
(d) There will be p.d:across the ends of the wing. if Bv is the vertical component of earth field and
v the horizontal velocity of the aircraft, then
e = Bv vf.
(e) The emf will be minimum when the frame arranges itself in the plane passing through the
rectilinear conductor.
(fl, There will be no induced current.

Induced emf/ current in moving conductor slides along a stationary U-shaped


conductor
Methodl: When conductor moves perpendicular to magnetic field, the charge carriers inside it experience
magnetic force (qvB). When a chargeq moves from Q toP through a distance£, the work done by
the force Fis W=Ff= qvBf ;.
B xm P xP'x
The emf is the work done per unit charge

w
X

R
X

X
V
x l!ex
=!P- t
£
e -=vBf
q X

X
X

Q
X il
::
Q'
X
!
i-llx-1

Fig. 8.22
Methodll: By Faraday's law
Let conductor PQ slides a distance Ax in time l!.t, the change in magnetic flux in this time
l!.<j, Bl!.A = B(fAx)

and [el l!.<I> = se(Ax)


/!,.t /!,.t

Bfv

e Bvf
I. Induced current -=--
R R
2. Magnetic force on the conductor: Conductor PQ experiences a force opposite to the directio_n
of motion

F rn = Bi- f=B(Bvf)f= B2vf2


'" R R

3. Power dissipated in moving the conductor :

Pagent

4. Electrical power : Electrical power dissipated through the resistance

pthermal
!EIJEGTRl<ErnY. IS',! Nffi.GNE'FISM

More about direction of induced current


Rule 1 : The direction of induced current in a closed circuit with moving conductor can be obtained as
follows : Place a positive charge on moving conductor, and then find direction of force on it by
Fleming left hand 1,1Ie. The charged particle will move in the direction of force, which will be the
direction of induced current.
Rule 2 : Direction of induced current can also be obtained by Fleming right hand rule. According to this, if
middle finger, fore finger and thumb ofright hand are held mutually perpendicular, then fore finger
·represents the direction of magnetic field, thumb represents the direction ofvelocity ofthe conductor;
and middle finger will represent the direction of induced current.

Ex. 15 Let us consider a clo~ed conducting loop is moving in a uniform magnetic field, which is
perpendicular to plane of the loop. Discuss about induced emfio the loop.
Sol.
Methodl: (a) The magnetic flux through the loop is <j>8 =BA.Till the loop X BX X X

remain entirely inside the field, the flux will not change and so
the net induced emfin the loop will be zero. :pGxc:Q:
xf x x V"
X X X X
xSx xRx
Fig. 8.23

Method II: We can split the loop into its four sides as shown in fig. 8.24. X X X X

The induced emf across side PQ and SR is zero, while induced


emf across PS and QR is e =Bvf. Forthe closed loop PQRS,-the
~1e=Ofx
X X X X
net emfbecomes,e-e = 0. e e •
x TX x ~v
X Lx..__._j X

X
S Xe=OX R X

Fig. 8.24
(b) Now let us consider closed conducting loop coming out from
the magnetic field. When one of its sides QR comes out of the
field, the emf acrossPQ, QR and RS are zero. But there is induced
emf across PS, which is e = Bvf. So the net induced emf across
the loop is also e. The direction of induced current in the loop
will be clockwise. ·
BX X p X X

{c) Consider the closed conducting loop coming out of the uniform
magnetic field. But velocity vector is perpendicular to one of its
diagonals." ·

X X RX X

Fig. 8.25
· The induced emf across the loop e =Evy.As the loop is moving to the right, y increases and
therefore induced emf increases and becomes maximum •max= Bvx(PR). There after
induced emf starts decreasing and becomes zero when entire loop comes out of the field.
,. ft·

(d) In the case shown injig. 8.26, the induced emf across the ends of the conductor
e = Bv(R 1 sin8 1 +£ 2 sin8 2 )·
X X px. X X

X X

X X

X X

Fig. 8.26

8.6 ROTATING CONDUCTOR


_ X X X VX X X
I. Rod rotating in uniform magnetic field B
Consider a rod of length e is rotating about an axis passing
through one of its ends with constant angular velocity co in x~x
X -XQX
.
X I X X X X
an uniform magnetic field B as shown in jig. 8.27. t+-X di:
Induced emf across the element X X X X X
(de) Bvx (d.x)
B(IDX)dx 2
Bcot
Emf across the entire rod e=--
0~--;!1- 2
e
e fBco xd.x Fig. 8.27
0

2 1
I-~=
• Vp--'.. Vg o= _Bco e_' .•

t . 2
1 x X
2. Cycle wheel X X X

Flux cutting by each metal spoke is same. Each spoke becomes B


X X
2 •
cell of emf e = Bcof . All such cells are in parallel fasnion, X X
2
therefore en,t = e. Each point on the periphery of wheel has X X X X

same potential.
Fig. 8.28
3. Faraday disc dynamo :.A metal disc can be assumed to be madeup of
number of radial conductors. The emf induced across each
2 B
conductor is e = BcoR . All such conductors behave like a number
2
of cells in parallel. Therefore

BcoR 2
enet e=-2-.
Fig. 8.29
r . ----·~,, . --:""-·-·;;}"';:,:-:v?-~----:--·~-v:; '"! - ~ " ' ' . -4 "'<·-,~ - " ' , _ . - , - - - , :-~r~:-,7
," , The,induced,eril£in.'1frot'ating,conductor does,not depend o'n the 'shape ofibe conouctor;
""---_c..~_,-',·:·,-'-,,_-~,'- i : S/'•":_,· ___ '' ' '·;· ,_,' ·_,.,_-,.- --. r;(.'··,1 ~,. • , '
', "I
'..;__j

Ex. 16 A conducting rodPQ is rotated in a magnetic field about an axis passingthrougb 0. The one end
of the rod is at a distance a and other end is at a distance b from 0. Find induced emf across the
ends of the rod.
Sol.· The induced emf across, the element of length dx is
The emf across the whole rod
de = 'Bvx dx =B(rox)dx. 0 p
I+-x--1 l+-
1«--' a -+1 dx ·
Q
. --'---......
b l+---b----+<
e Brof xdx Fig.
. . 8.30,,, .. __

"
Ans.

,I

Ex. 1 7 lnfig. 8.31 shows a bent rod rotating about its end o in a plane perpem!icular to the magnetic field
B . The part 0A of the rod is non-conducting while the partAB is conductjng. Find the induced ·
einfbetween the endsA andB. · ,1 ••

Sol. Suppose straight length of the rod is f. The distance of the end Bfrom 0
2 2
OB2 = OA 2 +(2R) = £2 +(2R)

A,,---..

~ ........,
0 ................. '.,.,................ B '.· _)

Fig. 8.31
The p.d. between A and B can be calculat~d as :
VA-VB,=. (Vo-Vo)-'(Vo-VA)- ...(i)
2
Here Bro(
- ~ - - Bro [ f 2 +4R 2.]
~OB)
2 ' , · 2· ,J,

' . ~ ,
' ' ", ' ' 2'· , 1 ,._

and· V V Bro (OA)2 = Brof .


0- A = 2

2 ..
,

Substituting these values in equation (i), we get


'.
VA-VB = B;[(t 2
+4R2 )-r].
,

B·ro ·
.... 2
The result obtainedwill remain same if conductor AB is straighi.
x4R.2 =2BroR 2 • Ans.
Ex. 18 A uniform wire of resistance per unit length A. is bentinto a semicircle ofradius a. The wire
rotates with angular velocity min a horizontal plane about a vertical axis passing through C.A
uniform magnetic fieldB exists in space in a direction perpendicular to paper inwards.
(a) Calculate potential difference between points A and D. Which point is at higher
potential?
(b) If points A and Dare connected by a conducting wire of zero resistance, then find the
potential difference between A and C.

(0 6)
C C
i ·=::.:·--. 0B
! d'•. ............
A ···•·· ········.~~f..•:.~.............. D 1 ·~·-.
A .................(-. ..........
··oh·t ....· D
0 0
(a) (b)

Fig. 8.32
Sol. (a) The distance of end A from C,

. 0
AC = 2asm-
2
Th, distance of end D from C,
0
CD - 2acos-
2
The potential differences :

Bro (AC)2
2

and B2ro ( CD)2

Thus (vc-Vv)-(vc-VA)

Bt[(2acos~J2-(2asin~r]

Brox(2a)2 [cos2 ~-sin2 ~]


2 2 2
2Brod2 cos0 Ans.
(b) When A and D are connected together, the current starts flowing fromA towards B. The
resistance of the loop c;
R = (length ofwireAD)X A · ~

Thus current in the closed loop, 7ta A. _-_f e, Ri'r~


I

VA-VD A • D
i
R (c)

2Booa 2 cos8 2Booacos8


Ans. Fig. 8.33
7taA 7tA
The equivalent circuit is shown infig. 8.33 (c).
The resistance R 1 = (a8)A.
The p.d. between A and C now becomes
VA-Ve = e 1+iR 1

2Booa2 sin2 (~)+


2
2Booacos8 (a8A)
1tA .

8. 7 INDUCED ELECTRIC FIELD

Consider a metal ring is placed in a uniform external magnetic field as shown in jig. 8.34 (a). The field extends
upto cylindrical region ofradiusR. Ifwe increase the intensity of the field, the magnetic flux through the ring will
then change and by Faraday's law an induced emf and thus induced current will flow in the ring. If there is a
current in the metal ring, an electric field must be present at various points within the ring, and it must have been
produced by the changing magnetic flux. This induced electric field En is just as real as an electric field by static
charges. Thus we can say that changing magnetic field produces an electric field.
Ifwe replace the metal ring by a hypothetical circular path of radius rand magnetic field is increasing at the
constant rate dB/dt, the electric field induces at various points around the circular path. Due to symmetry it will
be tangent to each point of the path (see jig. 8.34 b).
Another form of Faraday's law
If En is the induced electric field, then work done by the force of this field in moving a test charge q around the
0
path is (qof!) (21tr). ·
Suppose e is the induced emf, then work done_ on the test charge in one revolution is eq0 . Thus we can write
eq 0 (q 0En) (21tr)
or e
More generally En(21tr) can be written as

According to the Faraday's law d~B


e = -~,

= -~~B
. dt
-----
' · Metai'ring ' . Circular
: path
·~ ',.
' '

(a) If the magnetic field increases at (h) Induced electric fields app~r
a steady rate, a constant induced current at various points even the ring
appears in the metal ring. is removed.
Fig. 8.34
Magnitude of En
- dq>iJ
We have; PEn·dl
dt :
(I) Forr<R: X

2
d(Bx1tr )
En X21tr
dt
r dB
or En --- Fig. 8.35
2 dt
(2) Forr<::R:
2
d(Bx1tR )
En X21tr =
dt,
2
or = _ R (dB)
,En 2r dt ·

Difference between .electric potential and induced emf


Electric field can be produced 1iy static charge or by changing magnetic field. And electric field produced by any
means exert forces on charged particles. The electric field produced by static charges never form closed loop,
but induced electric fields form closed loop. The field produced by a static charge is of conservative nature
while induced electric field is ofnon-conservative nature, and so its line integral over a closed path is nqt zero.
Thus we can say that electric potential has meaning only for fields produced by static charges; it ~as no
meaning for induced electric field. ·
- - - , - - - - - - - -··· - ----~----·-------
I . I
• I.
I2
h:nz's faw is consistent with the principle of conservation of energy. ·
Thdnduced emf in Faraday's law does not have a chemical or electrostatic'origin. The work done inj
, 1t

I
L_ carry.in.g a charge ar.ound ~he :lo~ed loo··· pis not zero . If£." is the induced electric field associated.:ith

an induced emf, then pE ·dl,t 0. Thus here induce~ electric field is not a co_nservative field.
Ex. 19 A conducting rod is placed in a changing magnetic field as shown in.fig, 8.36. Find induced emf
across its length.
Sol. Let us consider a rod oflength f is placed in magnetic field, which is perpendicular to the plane of
paper and pointing into it. The field is changing at the constant rate of dB/dt.

Now consider a point A in the rod, it is at a distance r =-.!!__ from the centre of field. The induced
cose

electric field at A, r dB
E,.
2 dt

d dB
2cos0 dt

d
r=--
cose

Fig. 8.36
The component of field along the rod
d dB
E = E cos8=--
11 2 dt
:. Potential difference between ends of the rod
d dB
e = .Ef=-f-
2 dt
Case (1) : If rod is placed along the diameter
d = 0, :. e=O.
Case (2): If the rod is outside the changing field

R 2 dB
E = --cose.
2d dt

Ex. 20 Figure 8.37 shows a conducting loop ab e defa made of six segments ab, be, ed, de, efand fa, each
oflength f. Each segment makes a right angle with the next so that ab c is in the.zy- plane, c de
in .>y· plane and e fa in the yz- plane. A uniform magnetic field B exists along the x-axis. If the
dB
magnetic field changes at a rate -;ii, find the emfinduced in the loop:
Sol. Since magnetic field is along ; axis, there is flux only across the face oafeo, q,8 = Bx e2 • The flux
0

across any other face is zero. The induced emf, z


a b
2
d<j,B d ( B£ )
e ---=
dt dt
l B

-e2 ( !} Ans.
0 C
X

d
Fig. 8.37
Ex. 21 A square loop of edge 'a' havingNturns is rotated with a uniform angularvelocitymaboutoneof
its diagonals which is kept fixed in a horizontal position. A uniform magnetic field B exist, in the
vertical direction. Find the emf induced in the coil.
Sol. At any instant, 0 = OJI and the flux through the loop X X X X Q) X

.O'
q, 8 NBA cos0 X
,------
X ,,.
,
X X
NBa 2 cosCJJ/ .. .· B
..... •
X X X X
-d4>n
Induced emf, e =
dt .x X X X

.,···
X X X X
d [NBa~ cos mt]
Fig. 8.38
dt
NBa 2ro sin(l)f Ans.

If the same loop is rotated about an axis parallel to one of its sides and passing through centre of square, the
induced emf will be same
X X X X X

X X X X X

0)
• .,)G. •••• -x .............. ~
··C e =NBA@ sinml
.)( X X X X

X X X X X

Fig. 8.39

Ex. 22 Figure 8.40 shows a straight, long wire carrying a current i and a rod oflength f coplanar with
the wire and perpendicular to it. The rod moves with a constant velocity v in a direction parallelto
the wire. The distance of the wire from the centre of the rod isx. Find the motional emfigduced in
the rod
Sol. The magnetic field due to wire at a.distance x from the wire,.

µo ._i_
Bx-21tx
The induced emf across the ends of the element dx,
de B,vdx
I+- ( ----i
~ 11=
'
,. (µ i)·
'_Q_,_
21t X
vdx Fig.8.40

:. Emf induced across whole length of the r9d

(x+f)
e
J µo2ic .;ov
- .,dx
\ ' I

.. (,xi-2I) , X

1.1 •.-
(x+½)
? ) "'
-
µo _.
IV
J -
dx

21t (x-f} X

,. '· x+i
µo ivlfnxl
21t x-
1
2

'µo . , ·(x+2f)
-,v,n(
21t f)
x--
2
, L!.:,
µo . , - - ·
-IV,c,0
21t
(2x+f)
2x-f · Ans.

., .. ,I

Ex. 23 A wire of mass m and length£ can freely slide on a pair of parallel, smooth, horizontal rails placed
in a vertical magnetic field B (/ig.,8.,f1). The rails are connected by a capacitor of capacitance C.
The electric resistance of the rails and the wire is zero. If a constant force Facts on the wire as
shown in the figure, find the acc'el~ration of the wire.
X X X X X X X
~

---
X

xc Fx
X

=> C
r-7=-=F
. . . e-Bv(
·T . J
X X

X X

X X, X X X X X

· Fig. 8.41
,'.lt \'
' '
Sol. Let any instant the velocity of wire is v, the induced emf,
e = Bvf
and charge on capacitor q Ce
CBvt:tr'·'"· · ·
,1.

and current in the wire

dv
As - is the acceleration a,
dt
CBaf.
The resisting force on wire exerted by magnetic field
Fre, Bif = B( CBaf)f
= B2 cae2
Now by Newton's second law
F-Frcst ma
or · F- B2Caf 2 ma

a = Ans.

Ex. 24 Figure 8.42 shows a rectangular conducting loop of resistance R, width L, and length b being
pulled at constant speed is through a region ofwidth_din which a uniform magnetic field B is set
up by an electromagnet.
(a) Plot the flux cj> 8 through the loop as a function of the position x of the right side of
the loop. Assume that L = 40 mm, b = 10 cm; d = 15 cm, R = 1.6 Q, B = 2.0 T, and
v=l.Om/s. 1,1,,1•

(b) Plotthe induced emf as a function of the po~itio'n of the loop.


,;,,: ,=', ,,

(c} Plotthe rate of production of thermal energy in the loop as a function of the position ofloop.
.,..,-.1

l+----d---;.,
~······x·······x······":K······~
: :
~b~--,? X X ~
"'e.~" - ~
. ( . ~
X ~:
:xxxxX!
~x-i :
t.)S ••••••• x.......K ...•.• x....... i.

Fig. 8.42
~\
<j,B
~

~ 8 ·;.1r----+,-----:----- :-----:
Sol. (a) g., 4 ----'-----L----
I I I I
---- ,_____ ,
I

.,.. I I 1
I
1
I I
I
I I

0 j __ _.,,c__ - - J _____ .l __ - - - L - - - - _1_ - - - " - " - - - - ' - '

Fig. 8.43
e . '
'
'
80 --- - -:- -- - - -,- -- - - T- - - - _;....._--:,--_.;_ - - - - ,
I I I I

$' 40 -----:-----~-----~----- -----:----- . --. -~


~~ I I" ,I I.
(b) ";'.; O'--...... ----;--- _-' -----:----- .:
I I I I I

--40-----r------ :- .___ -----: ·----:----- :-----:


-80 -~ .
,, --- -·-~---· _:_____ ~-----:
Fig. 8.44

(c)
'.
4 I , I

~ - - J - ____ J_-_. _:.:~~ " _____ :_ ____ _,__ _.


O coil coil ent~ri~1g .: coil ·in:· coil l~a_\ing ; coil
'---'o'-'u"t-"'------~··~·~--'-------+'-'o"u"-t~---- x(cm)
0 5 /.· 10 15 20 25
, • Fig. 8.45 ·
(a) The magnet flux is zem when the loop is not in the field; it is <j,8 =BL_ x, when loop is entering
into the field. It increase~ linearly and becomes maximum when x = b ·

i.e., q,8 =BL b =2x(4ox10-3 )x(10x 10-2 ) =8 mWb.


When the loop is leaving,the field, it is ~8 = BL[b-(i--:_d)]
l' -. : ·, . .
:x · ·····x····~""x ······x······x:
ix x, x·· · ~[b-(x-d))
/x X.
:Ix· x x x- '-~~
x -11i:'{i~, . .~.t.-- ·
• X '
bs••••••• X'-~-·-·~~······4-·---~-~
Pig.. 8,46 .

~!: ',
. } l' :: '

(b) The induced e~f is equal t\::_ which can be written ~~ .


d<j, 8 d<j,8 dx · dq, 8
e -. di - - dx dt - - dx · v
dq, 8 -' .
where - - is the slope of curve (a).
dx
From Oto !Ocm

d<j,9 - _8_=80
dx 0.10
\
e -80xl=-80mV. ·
From 10cm to 15 cm

d/i>n
0
dx
e 0.
From 15 cm to25 cm -....
'

d/i>n
-80
dx
e = -(-80xl)=+80mV

(c) We have P
e2
R

3
(soxio- /
pmax =4mW
1.6
and pmin 0. Ans.

Ex, 25 Figure 8.47 shows a rectangular loop of wire immersed in a non-uniform and varying magnetic
field s· that is perpendicular to and directed into the page. The field's magnitude is given hy B =
4t 2x 2 , with B in tesla, I in second, and x in metre. The loop has width
W =3.0 m and height H = 2.0 m. What are the magnitude and direction of the induced emf~ around
theloopatt=0.!Os? ., , ..
y

TH
,.1...-dA

0B
,t:
·t i,ru1uced

1 1--- X ---ldJ'-
1/"'·-_ ___.. ' '' \
\
Fig. 8.47
Sol. Take an element of thickness dx, its area dA = Hdx. The magnetic flux through this area
d/iJ 8-dA. = BdAcos0° = BdA
B(Hdx) ,, '
4t2x2 Hdx
The total flux through the entire loop
3.0
q, = Jd$=4t 2
H Jx 2dx
.' ' , 0

3.0
3
4t 2 H ~ =721 2 (H=2.0m)
3
0
Now by Faraday's law, the magnitude of induced emf
!I!, J_ •

'dt dt
{_!:; I ,:J , ·144,
At , t L' :::;:\ O.1Os,
;e
J > '
=
'
144xQ_IQ
l4_4V Ans_
Direction of induced emf: The Jiu>< of' ii through the loop is into the page and is increasing in
magnitude with time_ Accordinffo Lenzfs law, the field of the induced current must oppose this
increase and so is directed out of the 'j;age_ Therefore the direction of induced current or emf is
counterclockwise in the loop (se_e figur~)--__ _
'' ... ' ~"

Ex. 26 A square loop of side 12 cm with'its'sideparallel tox andy axis is moved with a velocity of8 cm/
sin the positivex-direction in a magnetic field pointing towards positive z-direction. The field has
a gradient of 10-3 T/cm_ Find the magnitude of the induced emf if field changes at the rate of
0.1 T/s. 1F(', ·z:'.'

S.ol. We know that, the induced emf in this case is given by


;-irl_;; ~ - -- -
.,, . ,. ... ,A[aB dB]
lei'=' ai+v ax

. 144xl0-4[o-I+8xl0~ :J
3

- 155 mV. Ans-

Ex. 27 A wirePQ oflength f, mass m and resisfanceR slides on a smooth, thick pair of metallic rails
joined at the bottom as shown in jig. 8.~8. The plane of the rails makes an angle 8 with the
horizontal. A vertical magnetic field B exists in the region. If the wire slides on the rails at a
constant speed v. Show that

mgRsin8 -
B = v£2 cos1 o .
B
p,~-------,f.~ (90 + 0)(h'-__,.F
-
V
mg
~---------
e.......... _..
(a) (b)
'- --F_ig- 8.48
Sol. The induced emf across the ~ire PQ

e -~·. -Bvf sin(9O' + 0)


'Bvf•cos0
The current in the wire,
e Bvfcos0
l d_ =
R R
The magnetic force on the wire ,:.l,ni.
F = Bil, its direction is as shown in.fig. 8.48
The wire moves with constant velocity,
mg sine F case
or mg sine Bil case

or mgsin0 B( Bvf ~ose }case

mgRsine
B = . ·~c 2 c~s 2 0 . Proved.
,,

Ex. 28 Figure 8.49 shows a metallic squa~e frame o!edge a in a vertical plane. A uniform magnetic field
B exists in the space in a direction perpendi~ular to the plane of the figure. Two boys pull the
opposite corners of the square to deform itinto a r,hombus. They start pulling the corners at t=O
and displace the corners at a uniform speed u. , ,
(a) '
Find the induced emf in the frame at the instant when the angles at these corners reduce to
60°.
(b) Find the induced current in the frame atthis instant if the total resistance of the frame isR.
(c) Find the total charge which flows through a side of the frame by the time the square is
deformed into a straight line. ·'

Sol. (a) The situation is shown in.fig. 8.50.

r
e e

Fig. 8.50
1
The loop can be split into two ideniical parts. The emf across each part will be
e = . B(a sin30° x 2)u
I
Bau
~net' = 2e=2Bau Ans.
...,,....·~~~·:,--~..._~___,_·_:~~:-i_j
~-.--E-£E_·_c_m_·_1_c_~-.-.&·~~?i~~ ~--'---·_·~_ '1•_.

(b) The induced current in the frame


enet 2Bau
- ·--=--
R R
Ans.

(c) Total charge flows,

q -M- =<1>1-•l>z
---
R R
where, <1>1 BA=.B(a2) ·
<1>2 = 0
I.
Ba 2 ----
q -·-· Ans.
R

Ex. 29 The rectangular wire frame, shown inpg. 8.51 has a width f, mass m, resistance Rand a large
length.A uniform magnetic field B exists to the left of the frame. A constant force Fstarts pushing
the frame into the magnetic field att=O. · ·
(a). Find the acceleration of the frame when its speed has increased to v.
(b) Show that after some time the frame will move with a constant velocity till the whole frame
enters into the magnetic field. Find the velocity
(c) Show thatthe velocity attime I is given by.,·

= v0 (l -e-Ft/mt,•(})
4
V

X X
B '
X X
[ ...... ~ T
X X F f
X X ,..... l
X X X

Fig. 8.51
Sol. (a) The induced emf in the loop, when its left side enters into the field e = Bvf, and the induced
e .
current i = R, the restoring force exerted by magnetic field

F' = Bif=B(;}

s(s;c)e= s: 2

Now by Newton's 11 law


F-F' ma

B 2vf 2
or F--- ma
R

a Ans.
L
(b) Whh the increase in speed, the acceleration of frame decreases. At a particular value ofr0 the
acceleration of the frame becomes zero. There after frame will move with constant velocity

FR-,vf 2B2
0 ·=
mR

FR
which gives V vo= £2 8 2
(c) We have,

dv
-
FR-vi 2B 2
dt mR

J dv 2 2
V

o( FR:: B ) Jdt
0

in( FR-:: 2 2
B ) v

or -t
, -( f2B2)
mR ·
0

or fn[FR:;2
82
]-en[~~] l2B2
.---t
mR

2 -f2B2
or ln[FR:;2B ] ---t
mR

fn[FR(l:f~ )] -£2 8 2
or ---1
mR

or in(!-~) Ft
mv0 .

or 1-..2'... ~' -c-Ftlmv0


e
Vo
'.
or · v· = Vo [i-e(-Fllmvo)] Proved
Ex, 30 A wire bent as a parabolay= cx2 is located in!' uniform magnetic field ofinductionB, the vector
B being perpendicular' to the plane -xy. At the· moment t ,= 0 a conitector starts sliding
translationwise from the parabola apex with a constant acceleration a. Find the emf of
electromagnetic induction in the loop thus formed as a function ofy.
I
y

Fig. 8.52
Sol. Let us consider an element of dy at a distance y from 1the apex. The magnetic flux associated
dip 8 B(2x x'ay) ' ...(i)
We have y c x2

which gives X

.,
In timet 1 y = '-a/2
2

or ...(ii)

From equation (i), we get

d$s B(2xxdy)
dt iii
or Bx2xxv
where V = at
; I d$s
Bx2xxat
iit

Bx2fc:ax)¥

or lei By~ Ans.

Ex. 31 The magnetic field in the cylindrical region is shown injig. 8.53 increases ata constant rate of
20.0 mT/s. Each side of the square loop abed and defa has a length ofl.00 cm and a resistance of
4.00 0. Find the current (magnitude and sense) in the wire adif.
(a) the switchS1 is closed butS2 is open
(b) S 1 is open butS2 is closed,
(c) bothS1 andS2 are open
(d) bothS1 andS2 are closed;
,t: t•· .,,

Fig. 8.53
Sol. (a) Switch S 1 is closed and S2 is open : The flux is confined in loop adef
The induced emf in the loop

dtf>a d(BA)
I el --=--
dt dt

-A(!)
(1x1x10-4 )x( 2ox10-3 )

2x10-6v

e
The current in wire ad,
R
where R is the resistance of the loop= 4x4 =16 Q

e 2xl0-6
R 16
1.25x:10-7A Ans.

The direction of current in ad is to oppose the increasing flux. Thus it is from a to d.


(b) S 1 is open but S2 is closed.
Now flux is associated with closed loop abed therefore induced emf will occur in it.
i =. 1.25 x 10-7 A from dto a Ans.
(c) When both the switches are open : No flux is associated with
any l o o p . er-----,d--,_~c
e = 0 andi=O.
,(d) When both the switches are closed: The flux is associated with
both the loops, and there is induced emf in both the loops. The
equivalent circuit is as shown. Since e 1 = e2 = e.
The p.d. across a and d = 0 f a b
:. Current in wire ad, i'= 0 Fig. 8.54

Ex. 32 Jhemagneticfieldina~egion_i~givenby B=k ~ y whereLisafixedl~ngth.Aconductingrod

oflength L lies along y-axis between the origin and the point (0, L, 0). If the rod moves with a .
velocity v=vof;lind the emf induced between the ends of the rod.
Sol. Consider a small element of width dy at a distance y from the origin. The induced emf across it
de = Bv(dy) F

Boy) vody
(l

The induced emf across whole length of the rod·


-
Vo
LB .
}-.L~-------+X
e = J_Q_v0 ydy
oL

B0 v0 L z
Ans. Fig. 8.55
2

Ex. 33 A rectangular metaUic loop oflength f. and width bis placed coplanerly with a long wire carrying
a current i. The loop is moved perpendicular to the wire with a speed v in the plane containing the
wire and the loop. Calculate the emfinduced in the loop when the rear end of the loop is at a
distance a from the wire.

Fig. 8.56
Sol. Method I: Consider an element of width dx in the loop. The magnetic 1lux across the element
d~B = Bx(bdx)

µo .!..bdx
21t X

The total 1lux across the loop

(x+e) dx
~B
µo ib J-
21t X
X

Now, e = - d~s =-µoib .'!_[en(x+f)-f.nx]


dt 21t dt

_µoib !!_[in(x+f.)-f..nx]x dx
21t dx dt

e = _µoib
. 2it x+f.
.
[-1__ .!.]xv
X
' ;-~it,.:./4-,_·_·
l-~.-- ;;~.-:: - -
whenx=a,

µ0 ;bev
21ta(a+P.) An.<.
Method II : Whenx= a, the position of the loop is shown infig 8.57. The magnetic field at left arm
of the loop
µo i B1 B2
81 = 2Jt-;;
Similarly, magnetic field at right arm of the loop ~v
µo i a---1
82 · (a + f )---,-,-,1
= 21t (a+ e)
Now induced emf across left and right arms are Fig. 8.57
e1 8 1vbande2 =82vb
enet el- e2
vb [8 1-82] le 1 e,_
TI -
~----~1
b[µo i µo i ] Fig. 8.58
V 21t -;;-2Jt a+e

µoi(bev)
21ta( a+ e) Am.

Ex. 34 (a) Figure 8.59shows a conducting circular loop ofradius a placed in a uniform, perpendicular
magnetic field B. A thick metal rod OA is pivoted atthe centre 0. The other end of the rod
touches the loop at A. The centre O and a fixed point Con the loop are connected by a wire OC
of resistance R.A force is applied atthe middle point of the rod OA perpendicularly, so that
the rod rotates clockwise at a uniform angular velocity co. Find force.
(b) Suppose the wire connecting O and C has zero resistance but the circular loop has a
resistance R uniformly distributed along its length. The rod OA is made to rotate with
uniform angular speed m as shown in the figure. Find the current in the rod w,hen
LAOC=90°.
X X X
~··j X

X x.· .. X

X X
( [y<
'c X

0 A
X X X X

X X F X

X X X X

Fig. 8.59
Sol. (a) . When rod rotates with constant angular velocity, the·induced emf across its ends
2
8ma
e = V0 -VA=--
2

e 8ma2
The current in the wire OC, i= - = - -
R 2R
The same current will flow across OA from A to.O.
Therefore the force exerted by magnetic field on the rod
Fm = Bia
2 2 3
=" s(Broa )a= B roa
2R, 2R

The rod is to be rotated with constant angular velocity so the torque of net force about centre
· ofloop is equal to zero. i.e., ·
(F-F.,)xOD .0
or F Fm

B2roa3
--- Ans.
"2R

X X X )(•'= X ;,
X x.-···
.·· t,_
C

SJ
X
iz
-x
X

X
: o&-lx
X F
AX
X

X
JR
4,·0
e
90°
i
-R4 =>
R
4
A
X X X X A

. (a) The equivalent circuit. (b) The equivalent circuit.

Fig. 8,6.0
' .
(b);, The effective resistance between A & C .

,. -
. 3R
R' 16

Now current in the rod OA

·' Broa 2
.!_==_2_

R' (~:)
/

8 Broa 2
' .
Ans.
3 R

Ex. 35 A coil of N turns with the cross-sectional area A is placed inside a long solenoid. The coil is rotated
at a constant angular velocity roaroundthe axis coinciding with its diameter and perpendicular to
the axis of the solenoid. The magnetic field in the solenoid varies according to the law B=B0sin
OJI'. Find the emfinduced in the coil; ifatthe instant I= 0 the coil axis coincided with the axis of the
solenoid.
Sol. At any instant, the total magnetic flux through the coil is ·
<I> NBA cos ml
N(B0 sin ml) A cos ml

.!.. NBo A sin 2cot


-2

1
d [- NB0 A sin 2cor]
_d<I> = 2
By Faraday's law e
di dt

I
--NB0 A (2co) cos2wt.
2
-NBoAco cos2m1: Ans.

Ex. 36 A wire frame of area 3.92xt~ m2 and resistance 20 Q is suspended freely from a 0.392 m long
thread. There is a uniform magnetic field ofO.784 tesla and the plane of wire-frame is perpendicular
to the magnetic field. The frame is made to oscillate under gravity by displacing itthrough 2x 1o-
2m from its initial position along the direction of magnetic field. The plane of the frame is always
along the direction of thread and does not rotate about it. What is the induced emf in wire frame as
a function of time? Also find the maximum current in the frame.
Sol. At the intant, when thread makes an angle 0 with the vertical,
. the magnetic flux through the frame
0
qi "' BA cos0
The induced emf
d<j, . d0
e --=BAsm0-
dt dt ...·(1')
For small e, sine =e. mg
The restoring torque on the frame
~ = --mg(t sin0) Fig. 8.61

d 20 mgt (-e)
or
dt 2 I

mgt (-e)
,,,g2

(!) (-e)
Putting (!) of, we get
2
de 2
--+co 8 0 ... (ii)
dt 2
This represents SHM, ,and so
0 = 00 sinrot
Substituting value of0 in equation (i), we have

e = BA(00 sin rot)!!_ (0 0 sin rot)


dt
BA 0 0 sin rot (0 0 ro cos rot)

= .!.2·BAro0a2 sin 2001


2
/g =~ 9.8 =Ss-1 0 = x 0 = 2x10-
Here 0)
vc o.392 . • 0 c o.392

22
· and e ½x(0.784)x(3.92x 10
4 )xsx( ;,~~~ }in IOI

2;io-6sinl01. . . , , , .•
Maximum induced emf
emax 2 x 10-6 V

and
em••= 2xl0-6 10
_7 A Ans.
R 20

Ex. 3 7 An infinitesimally small bar magnet of dipole moment Mis pointing and moving with the speed v
in !hex-direction. A small closed circular conducting loop of radius a and negligible self-inductance
lies in theyz-plane with its centre at x =O, and its ,axis coinciding with the x-axis. Find the force
oppositing the motion of the magnet, if the resistance·of the loop is R. Assume thatthe distancex
of the magnet from the centre of the loop is much greater than a.
Sol. Suppose magn'et is at a distance x from the centre of the loop.
The magnetic field due to the magnet at the centre of the loop
µo 2M
B - ---
--: 41t x3
The magnetic flux due to the magnet a M
X

BA= µo 2M (1ta2) V

41t x3
Induced emf in the loop
y l---x---1

e = _ d$
dt
= _.!!_[µo 2M
dt 41t x3
(1ta2)] Fig. 8.62

2
µo Ma (]__ dx )
, 2 x4 dt

'dx
As v,
dt
[' BLECTROMAGNETiq,INDUCTION . ffl'
e

e 3µ 0 Ma 2 v
Induced current,
R 2xfR
If M100P is the magnetic moment of the loop, then
Mloop = iA

The opposing force F µO 6 M magnet M1oop


47t x4

Ans.

Ex. 38 Two straight conducting rails from a right angle where their ends are joined. A conducting bar in
contact with the rails starts at the vertex at time I= 0 and moves with a constant velocity of
5.20 mis along them, as shown in jig. 8.63. A magnetic field with B =0.35 Tis directed out of the
page. Calculate

• • • • • • •
V
• • • •
• •
• • • • •
iJ
• • • • • •
• • • • • • •
Fig. 8.63
(a) The flux through the triangle formed by the rails and bar at t= 3.0 sand
(b) the emf around the triangle at that time.
(c) lfwe write the emf e =at", where a and n are constants,·
what is the value of n?
Sol. Suppose joint of the rails is the origin of the coordinate y
axis. The distance moved by the bar h\Jime I,
.-
V
The corresponding x value is
111, ;X,i = y tan45° = y.
The distance ,RQ. 7' 2x=2y
'. '~ · ~\ 2vt.
' I .
-1
(a) The area of the triangle OPQ ~"' ( 2x) y = xy
\\7
Thus the flux through the,triangle
,I
<j)B
\
BA=B(xy) 1I
or <j) 8 B(vt) xvr
,,, I Bv212 ..
-I
2
Att=3.0 s, <j)~ ~-· 0.35x(5.20) x3 2.0=85.17Wb
(b) The induced emf around the triangle
_u~~ 1

. e' = d<j)B
dt
',·
\\} \:'
J[sv21 2]
dt
Bv2 x 21
~- = 2B v2r
2
·= 2x0.35x(5.20) t
( 18.93 I
·,id
1
At t=3s, • e_-;:-)I8.93xf=56.79V
(c) Induced emf e = 18.93 I.
On comparing withe= at", we get n = I
r;) t)':p .J
Ans.
Ex. 39 A shaped as a semi-circle ofradius!a rotates about an axis 00' with an angular velocity roin a
uniform magnetic field ofinduction B (seefig, 8.65). The rotation axis is perpendicular to the field
direction. The total resistance of the circuit is equal to R. Neglecti~g the magnetic field of induced
current, find the mean amounJ.ofthermal power being generated in theloop during a rotation
period.

0·0'

lJ i1 : ·~ ,1v
Fig. 8.65
I
,, J{ ! 1 ' "·
1,., 2 "
Ill 11 J()

Sol. The area of the loop, A '1t<! '.' ,' ·, ..


2
lf0 is the angle between magnetic field and.noimal to the loop at any time t, then
. 0 = rot, . ' , . .
and.flux qi = BA cos oY

B( ~i'') cos COi

According to Faraday's law e "'dq> ' '


'd1

.B ('~.., 2)'co' sin col


'

Induced current e
R -
-(Borrt<?)
R
.
smrol
2
The intantaneous power g~nerated
1 2
p ei ='('Bro" rra 2 ) .sm
· R
2 COi
2
T

Thus mean power pmean


I Pdl
_o__
T
2)2 T
(
Brorra
2
-RI Jsm. 2 COi di
0
'F
2 2
Brorra ) _I . Ans.
( 2 2R

8.8 SELF INDUCTANCE


We 1can define an inductor as an arrangement that can be used to produce a known magnetic field.
We can express the connection between capacitors and inductors as follows :
Capacitor is to electric field Inductor is to magnetic field

t.''.~ V ~
~:LL~ .....~ J
........... '
:::::::::::: n
• x• :,;•x• x• }t • x• x• >" x• x• x

c=!L L=Nq>
V i
SI unit ofC is C/V The product N(jl is called flux linkage.
I CIV= I farad SI unit of inductance is T. m2!A.
We call this as the henry (H)
Sumbol: --j ~ Symbol:~
8. 9 SELF INDUCTION

Whenever there is change in magnetic flux,electromagnetic inductioi:i will take plac~. - .L


Ifthere is a time varying current in a coil the magnetic field of this
current will also change. And this leads to the -~ariation of the
magnetic flux through the coil, and hence to the appearance of an ~----<1----Ww-
induced emf. <; R
Fig. 8.66
Thus, the variation of current in a circuit will cause ~n induced emf
in this circuit. This phenomenon is called self induction.
In the absence of any ferromagnetics in the space ,surrounding the coil, the total flux through the coil are
proportional to the current i, and we can write
C'B_P_n_=_L_;_! ...(I)
where Lis called the coefficient of self induction ~fthe circuit. The inductance L depends on the shape and size
of the loop as well as on the magnetic properties'ofthe surrounding medium. The unit of inductance is henry.

Emf of self-induction
According to Faraday's law, a variation in the cu~ent of the c;rcuit causes an induced emf. Thus on differentiating
equation (I), we get
j,I

L di
dt' dt
.,, "
1-,,.,,,-:-- t dr -,
or 1 1e =;= """"-- ...(2)
dt
Thus when current in the coil increases, the induce_d emf in the coil opposes the applied emf, and so net emf of
the circuit
:,1
ldi . '
1';-e=l';-- d, 0
Snet ->
dt dt
'----~.11.---,..J ~- ·.~:
s
• . • Ldi Fig. 8.67
~nJt ~. ~- dt

If current in the coil decreases, then the indube~ ~?1f in the coil favours the applied emf, and so net emf ofthe

Ldi -,
circuit l;net 1 ~1 l';+e=l';+- di O
-<
dt di
• I;
Steps for finding L Fig. 8.68
1. Assume current i in the coil.
2. Determine the magnetic field due to this current in the coil.
3. Obtain flux linkageNcj)8 .
. 4. Compare with Ncj) = Li to getL
Calculation of self inductance
l. Circular coil :
Let us consider a circular coil of radius rand cont~ini.I)g N-tums. Suppose it-carries a current i. The
magnetic field due to this current '' ·
_. ; .. '! r1

µ Ni
= ' -0 - '
2r, 1,j

And total flux NBA

N(µ0Ni)1tr2
'2r
,, ,,r,~

Inductance may be viewed as electrical inertia: It is analogous to inertia in mechanics. It does not
oppose the current, but it opposes the change in current.

Solenoid : Consider a long solenoid of cross-sectional area A. Take a length f near the centre of this
solenoid. The number of flux linkages for this section of the solenoid is
N$ B = (nl) (BA)
or = nf (µ0ni) A.
Now compare with, N$8 = Li, we get
-~---·-·--:- l
[_ L_ = .J¾i n2 Af . . _1
Inductance per unit length for a long solenoid near its centre

Fig. 8.69

3. Toroid of circular cross-~ection ·:


Consider an air-core toroid of cross-sectional area A and_ mean radius r is closely wound with N turns
of wire. We neglect the variation of B across the cross-section, assuming its average value to the very
nearly equal to be value at the centre of the cross-section. Then the flux linkage
N$ 8 N(BA) A

= N(·µoNi)A .
21tr

Fig. 8.70
4. Inductance of toroid ofrectangular cross-section: The magnetic field (assuming unifonn) at a distance ..
r from the centre of toroid,
i;'1l- . ll
, , , µ 0Ni
B \= ·--
21tr
where i is the current in toroid. The flux, cj>8 over the entire cross-section must be found out by
integration, ·
If h(dr) is the area of the elementary strip shown, then we have,

<l>B = JiidA
b .
1

"
"~
[I
Jn(hdr)
a

..i.
i h
: ar f~BT
1---h
µ 0 Nih Jdr
21t r Fig. 8.71
'' , .. a

,I. µoNi h b
•,r]-;=,~i---£~-
2lt ·a
J ,_i, ;.1> : ,

Total flux linkage

b
µ 0 N 2hfn-
Therefore a
21t

or

"
.T
Ex. 40 A 12 Hinductor carries a steady current of2.0 A. How can a 60 V self-Induced emf be made to
appear in the inductor ?

Sol. We know that lei

di 1.:1 = 60 = 5 Ns.
dt L 12
Thus current in the inductor should change at the rate of 5 Ns.
.-----.--~---------------~,,.-------------......-..
'------"-···-'1"'""'"'->_.. _____;,;_~----~·-_E_t_'~_c_m_o_MA_G_N_ET_1c_·_JN_D_u_c_n_o_N_M
Energy in an inductor
A changing current in an inductor causes an emf, the source supplying the current must maintain a potential
difference between its terminals and hence must supply energy to the inductor. When an inductor carries an
di . "
instantaneous current i which is changing at the rate dt ' the induced emf is equal to L di and the power P
dt'
supplied to the inductor is
.di
P=ei= L1 -
dt ·' :
The energy dU supplied in time dt is Pdt
or dU = Pdt=Lidi
and the total energy supplied while the current increases from zero to a final value I is
I
V = Lf idi
0

I 2
or U = -LI
2

t,Jt# Aft . fima I stead y va Iue, di O, the power mput


er the current has reac hed its
. dt
. . zero. Th e energy
1s

that has·been supplied to the inductor is used to establish the magnetic field in and around the inductor,
where it is stored as a form of potential energy as long as the current is maintained. When current is
reduced to zero, this energy is returned to the circuit which supplied it. If the current is suddenly
interrupted by opening a switch, the energy may be dissipated in an arc across the switch contacts
The energy can be considered as associated with the magnetic field itself, and a relationship can be
developed which is analogous to that obtained for electric field energy.
The self inductance of solenoid,
L = '11on2 Al

and u .!._u2
2

½(~ 0n
2
Ae)12

Thus the magnetic energy per unit volume

=
u
. '
lii:I ri;..;;.;T c·· ''rb3!~1.·a1··J\l~·E,,n'"s1"'0M·0~~~*'-:i~~~-"~~li!~
et· ' ~._.,..,.HJ l'lYc,;:~:~ . :~>-~,,::~~~~;i~~~i~~Wi;,l'::l~
Electrical and magnetic quantities
·Electrical Magnetic
·N~
L Definition C=!L L=--B
V i
2. Dimensions C =Eo x a length L=µ 0 x a length
3. Constants e 0 =8.85pF/m µ 0 = L26µH Im·
2
4. Energy stored u =}_cv 2 =L Ua=}_Li2
e 2 2C 2
I 2
B2.
5. Energy density 11, =-e0 E "B = - -
2 2µo
L
6. Time constant ~=RC ~=-
R

Ex. 41 A resistor of 10 and an inductor of 1 H are connected in series across a source of 1OV. Suppose
the current in the circuit is changing by the added device. Find the net emf of the circuit when
di di
(a) - =+5Ais, (b) - =-5A/s.
~ ~
IO 10V lH
-----vM---11 odOO
Fig. 8.7i
Sol. (a) The induced emf in the inductor is given by

e = -L di
dt

For -di +5A/s,.


dt
e = -I x5=-5V.
The net emfofthe circuit
Snet I0-5=5V Ans.

(b) For -di -5 A/s,


dt
e -Ix (-5)=5V
Thus the net emf of the circuit
Snet 10+5=15V Ans.
8.10 RL - DC CIRCUIT
Consider a circuit having inductor and resistor in series with a direct current source of emf S·
As soon as the switch Sis closed, the current in the resistor starts increasing. If the inductor were not present,

the current would rise rapidly to a steady value ; . Because of the inductor, however, self induced emf

e = L di appears in the circuit; from Lenz's law, this emf opposes the rise of ~e ~urrent, which means it
dt
opposes the battery emf. Therefore

s-L dtdi
I'

,_
'
\., I

~R , ,1 ,r ,"' I R
'
. J.• •
s, ~'· J1 -didi s ,_
' Y" · -1::J ~ 'llJ,.':-:,·
'. fTHJ •i-- :1, •., . ,.:JJ, ,'!
,·r
• _,J[J I

(a) ~
7.'
s .. : . :._,
--. .., ;-:
Fig. 8_73 _, _.. _ (b) EITective circuit,
Now by loop rule
, I di

s-L dt iR ...(i)

L di
or s-iR
dt
di dt
or
(s-,iR) L
Integrating both sides of above equation, we get
i di -_,
.! (s-iR)

or
L ''
•-/
or £n(s-iR)-£ns
Jt)
or

or .....(ii)

.
Substituting siR" = i0, maximum current and
LIR ,;, time constant ip equation (ii), we haye
i
t
't ,,
/ 0.64i0 ....... ,
i'
or - - ... (I) i
!'
-·. ' ' l -· ' , ' o~---'~--~--,-+t·
,_!.
Fig. 8.74
,---''t, we have
, ,. . . ~---
; = 'o·(1 -e-1)
\ = 0.638 ;0 = 0.64 i0

\
..,
u

. ' .: ' ......,..,..,-27 .


G,u. = ;,u 'C~e·• };:1 ... c2i
' '· l ' _,_ '

or 0
:....1....-:·---- .,• ,·"f~-:-.-,~·-J .

Decay of current di
e=L-
After estabJishing current in the circuit (i0 = ~ I R), the source is
disconnected from the circuit. And the circuit is short circuited. The
current in the circuit starts decaying. The energy stored in inductor
will change into thermal energy. The equation for decaying current R
can be obtained by putting~= 0 in previous equation (i), we have ·
. di s
-L- = -iR
. ;di • I

di-.·
or · Fig. 8.75
·;

Integrating above equation, we get J~i


. l
=
'O

or !tnil; 0
=
'/
't
I
or. ... fni~fnio·= Fig. 8,76.
T

i
or fn- = -I
io -R
r·· .•
or [..:. ' - ...(!)
Fort= T,- i = ioe- 1 =0.37.io:- Thus the inductive time constant can also be defined as the time in which the
·decaying current be,comes 0.37 times the maximum current; - · ·
Energy at any time : .i ''1 ~

_, ]2
u I I -
~Li 2'=-L i0e<
2 2
[
'.
(2IL''o2)e -~t
.

I
,u = .Uoe-? "'" 1 ' =-
t
... (2)
2

More about inductor

I. All= 0, i = i 0 ( 1-e0) = 0 and at I==, 1. = 'o. (1 - e -i = 'o. = Ji:~ It means m' ductor offers mfimte
. .

resistance initially and zero resistance after very long time: It becomes ineffective when current becomes
steady in the circuit.
2. Inductor does not oppose the current, but it opposes the change in current. It causes delay in growing
or decaying of chrrent. Therefore it also called electrical inertia.

Ex. 42 A coil has an inductance of53 mH and a resistance of0.35 0.


(a) If a 12 V emf is applied across the coil, how much energy is stored in the magnetic field after
the current has built up to its maximum value?
(b) After how many time constants will half the maximum energy be stored in the magnetic field ?
Sol. (a) The maximum current in the coil

5_ =_E__ =34.3A
.R 0.35
The maximum energy stored

½u/ =½(s3x10-3 )(34.3) 2


31J Ans.
(b) We know that, energy at any time in the coil is given by

u ( -,)2
U 0 1-e'
For U = U0 ! 2, we have

Uo
-I )2
U0 ( 1-e <
2
-I
or e<

I
or 123
~

I 1.231 Ans.
.L'HE LC-OSCILLATIONS
~onsider a LC- circuit shown in fig. 8. 77, ;a resistimceless inductor is connected between the tenninals of a
charged capacitor. At the instant when connections are made,, the capacitor starts to discharge through the
inductor. At a later instant, the capacitor has c-ompletely discharged and the potential difference between its
tenninals has decreased to zero. The current in the inductor has meanwhile establishes a magnetic field in the
space around it. This magnetic field now decreases, inducing an emf in the inductor in the same direction as the
current. The current therefore persists, although _,;ith decreasing magnitude, until the magnetic field has
disappeared ·and the capacitor has been charged in the opposite sense to its initial polarity. The process now
repeats itself in the reverse direction, and in the absence of any energy losses, the charges on the capacitor
surge back and forth indefinitely. This process is called electrical oscillations. From the energy state point, the
oscillations·of an electrical circuit consist of a transfer of energy back and fmth from electric field of capacitor
to the magnetic field of the inductor, the total energy associated with the circuit remaining constant. This is
analogous to the transfer of energy in an oscillating mechanical system from kinetic to potential, and vice versa.
Let a capacitor Cis given an initial charge Q and, at/= 0 is connected to the inductorof selfinductanceL. The

, ( 1 ) 0
, .
magnetic energy o{the inductor L ;2 at any time co rresponds to the kinetic en,ergy of the oscillating mass-
, '
2 ' ' ' ' ' " - .. . '· . ! ' '

spring system. The potential energy of the capacitor(


' 2C~. I''
q2,)
corresponds to the potential energy of-the spring
,,1 I,,, ,

of energy·' . ,. ,,, ,·,:


' C·
. 1 L .2 .
- IT-
g?; ,,;. (?2
2 2C 2C .•.'(;,_-:-

Differentiating above equation w.r.t. time, we get L


I di I dq
-Lx2ix-+-x2qx- 0 Fig. 8,77
2 dt 2C dt

or L di +..'L 0
dt ·c
'dq
As
dt'
·,
d2q q.. . ·. ,
~+- = o -I ...(!)
di- LC____ J,.J ,
Compare above equation Ylith differential equation of oscillations of_mass-spring system, i.e.,

d 2x
--+c.lx
2 0, we get
dt

This is called natural frequency of the LC circuit.


ro = &• ... (2)

Time period T 21r,../Lc


Also, q Q cos(C!ll + <!>)
Comparison behveen electrical oscillations with the oscillations of mass spring system . T , ••
t= 0 t= T/4... /.=.T/2 1=37,4 t= T

~
rl
i
. LJ
r- .cf]
i
rl
i
- L~
~
C

i
dJ
'
UE=f/2C UE=O '
UE =q-/2C ._ UIJ =O '
UTJ = c{/2C
U8 =0 U8 = l/2Lr' U8 =0 l!B = 1/2 Lr' U8 =0
P.E. = 1/2kx-'
2
P.E. = 1/2/...,-' P.E.=O P.E. = 1!2kx P.E.=O
K.E. =O 2 K.E. = 0 2 K.E. =O
K.E.= 112ml' K.E.= l/2mv

~ ~
-" ~:
'
, ~
- V

~
Mfun Mean Mean. Mean Mean
position position position position position

U constant
uIJ

UB
}{--"...L-'-'>..L.--'"-"--"""-~ 1
T/2 T
Fig. 8.78

q2 Q2
-=-cos 2 (cot+tj,)
2C 2C

I Qi
and -Li2 =-sin 2 (cot+tJ>)
2 2C

8.12 DAMPED OSCILLATIONS IN AN RLC CIRCUIT

In the previous case we have assumed that the LC- circuit contains no resistance. This _is an idealisation, of the
circuit for every real inductor there is resistance associated with the windings, and there may be resistance in
the connecting wires as well. The effect is to dissipate the electromagnetic.energy and convert it to heat; thus,
resistance in an electric circuit plays a role analogous to that of friction in the mechanical system.
Suppose an inductor of self-inductance L and a resistor of resistance R are connected in series with the
capacitor. If the capacitor is initially charged, it starts discharging at the instant the connections are made, but
because of i2R losses in the resistor, the energy of the inductor when capacitor is completely discharged is less
than the original energy of the capacitor. In the same way, the energy of the capacitor where the magnetic field
has collapsed is still smaller, and so on. ·If the resistance R is relatively small, the circuit oscillates, but with
damped harmonic motion. As R is increased, the oscillations die out more rapidly. At a sufficiently large value
of R, the circuit no longer oscillates an!" is said to be critically damped. Fo.r ~till larger resistance it is overdamped.
'. , . .' .. ' . .
+O -0
~-_,- c L

Fig. 8.79
Using loop rule to the circuit yields the equation :
Rate of dissipation of electromagnetic energy= power generated as heat

or -PR

or. ?:i. dq +!:.x2ix di= -PR


2Cdt 2 dt
2
or -,
c
,-.-2
q.+L·dq
dt
= -PR

--·;i7.q 11 dq q ,-,-----~!'
or . [:_"":il+Ldt+LC = 0 I ...(!)
The abov.e equation is analogous to damped harmonic oscillator of mechanical system. i.e.,

d2x +b dx+kx = 0
dt 2 dt
The solution of equation (I) can be
Rt
q Qe 2L cos(rod1+4>) ... (2)

2
Here, l - ( Zl
LC R ) , is called damed frequency

CJ

.•... -····
__

Fig. 8.80 Variation of charge of capacitor with time.


[
Critical damping

As R increases ro becomes smaller and smaller; when R2 = 4L the quantity under the underroot becomes zero,
. C
and the case is called critical damping.

8.13 MUTUAL INDUCTION


Consider two coils placed close together, a steady current i in one coil will set up a magnetic flux <)>8 linking with
the other coil. If we change i with time, an emf will appear in the second coil. This is called mntual induction.
'-

+ -
Coil 1 Coil 2
Fig. 8.81

For ease ofrepresentation, the two coils shown infig. 8.81 are not actually drawn as close-packet: In close- '
packet coils the turns have the same flux through them. ·

Consider two circular closed packed coils are placed near each other and sharing a common central axis. There
is steady current i I in coil 1, set up by the battery in the external circuit. This current produces a magnetic field
ii1,
which produces magnetic flux N2,t>z 1 in coil 2, linked by its N2 turns.
We can define the mutual inductance M21 of coil 2 with respect to 1 as
.. - . -· -····. 7

-~21 :_~~;21 j
or it can be written as

If i I changes with time, then


di- d<)>
M21 _, = N2 _2_1
dt. dt
According to Faraday's law, the right side of the equation is equal to -e2 • Thus
di-
e2 -M21-'
dt
By doing similar treatment for first coil, we get
By experiments, M 12 = M 21 = M. 'fhus
,, _, ...(I} -

and ... (2)


', :i;:_•.-r, ,r_·
The SI unit of Mis henry.
The coefficient of mutual induction depends on the shape; size, and mutual arrangement ofthe coils, as_ well as
the magiletib"perrneability of the medium surrounding the'coils.
Reciprocity the·orem
Calculations show (and experiments confirm) that in the absence of material medium between the coils, the
coefficients_-M12 ~nd _M21 are ~qua! : ·
1::-!1 z' ~a;' ~28
This property of mutual inductance is called the reciprocity theorem. Because ofthi; reason, we do.not have to
distinguish between M 12 and M 21 and can simply speak of the mutual inductance of two circuits.

I
Ex. 43 Two circular loops 1 and 2 whose centres coincide lie in a plane (s1ejig. 8.82). The radii of the
lo_ops are a 1 and a 2• Current i flows in loop 1. Find the "}agn~tic flux c>z as~ociated by loop 2, if a 1<
. <a2.

c=:>1i . '.,
2
I
Fig. 8.82
Sol. 'The direct calculation of the flux <l>2'is clearly~ rather complicatea problem since the configuraiion
of the fie.Id iiselfis complicated. Howeve~, the applicaiion of the reciprocity1hoorem greatly simplifies
the solution of the problem. Let us pass the'same current i through loop 2. Then the magnetic flux
.i, 1 created by this current through loop 1 can .be easily found, provided that aj < < ai : it is

sufficient to multiply the magnetic fieldB at the centre of the loop ( B = ::: ) by the ar~a 1rai2 of
the circle.
Thus

Ans.

Ex. 44 A loop with current i has the shape of a rectangle. Find the magnetic flux <I> through the hatched
half-plane (seejig. 8.83) whose boundary is at a given distance from the contour.Assume that this
half-plane and the loop are in the same plane.
c ____ _ __E·~~---~--1R==o_MA_-_G_~=ET.==lC==JN=D=_U-C=1:=l=O=N__
0 ~
dr i

,ofJ:7 1-a--j
cp=?

1----b--_=-----
Fig. 8.83
Sol. In this case, the magnetic field of current i has a complex configuration,and hence it is very difficult
to calculate directly the flux <I> in which we are interested. However, the solution can be considerably
simplified by using the reciprocity theorem. Suppose that current i flows not around the rectangular
contour but along the boundary of the half-plane, enveloping it at infinity. The magnetic field
created by this current in the region of the rectangular loop has a simple configuration as this is the
field of a straight current. Hence we can easily find the magnetic flux cp' through the rectangular
contour
cp = <I>'

(a+b) .
f
a
_1:1:Q_ .'..
21t X
fdx

µoif
21t
fn(a+b).
a
Ans.

Steps for finding M


I. Imagine current i I in the first coil.
2. Determine magnetic field due to i I in the second coil.
3. Obtain flux linkage,N2 cp2 =N2 (B 1A2).
4. Compare with, N/~2 =Mi I and find M.
Mutual induction between circular coils
Consider two circular close-packed coils, the smaller (radius R2, with turns N2) being coaxial with the larger
(radius R 1, with N 1 turns) and in the same plane. Assuming R 1 >> R2. Imagine a currenti 1 in the larger coil, the
µ0N1i1
value of B I at the centre of coil, B 1 = 2R
I

Fig. 8.84
- --- ·-- .....,..., .:-~-
M EtECTRICI'IY &
. _ . _ _ . , __ _ _ _ _ _~ , , 4 . . '
MAGNETISM
_ _ _ _ _ _ ,_ --· ··*·------ ---· ·- ~- __ ,
Because we have assumed that R 1> > R2, we iriay take B I to the magnetic field at all points within the boundary
of the smaller coil. Therefore the number offlux linkage for the smaller coil is
N2<1'21 N2( 8 1l (1tR/)

N (µ0N1i1
2 2R1
)!tR~-

Now compare with

µ01tNt Nz R/
M, =
2R1

Mutual induction between solenoid


I .
and coil
A long solenoid of length eand cross-sectional area A is closely wound with N1 turns of wire. A small coil ofN2
turns surrounds at its centre. A current ; 1 in the solenoid set up a magnetic field B at its centre of magnitude

11~(~·\}
t - ¥JLll+ - i
I
,...._---(-----i

F,ig. 8.85
The flux through the central section is equal to BA, and since all of this flux links with the small coil, then
N2<1'21 = N2( 8 1A2)
:'' ·1.- 1,

Now compare with M i 1, we get

N1N,A
M µo /
Coefficient of coupling
Let us consider two coils placed close to each other. For them

M12 = N1<1'12 and


'2

Similarly N1<1'1 and l2 = N2<1'2


ti 12

If coils are closely packed, that is, flux of coil I is completely associated with 2 and vice versa, then
<1'12 <1'21
Also M 12 = M21 =M
N1,N2-ct>1 ct>2
J1 i::!J!

or M
If there is some leakage of flux between the coils, then

M < ~L1 c2'


In general, we can write

.. M. = k~L1 Lt
• _,7 f: lr;r
where k is called coefficient of coupling; 0 s; k s; I.
For tight coupling k = I. for loose coupling k < I
Magnetic flux linked in secondary
k can be defined as, k
Magnetic flux linked in primary

s k=I k<l
Fig. 8:86
k = ()

8.14 - COMBINATIONS OF INDUCTORS I - 1,

Series : Let two inductors L I and L2 are connected in series.


Considering only self inductance of inductors, then total induced i----:- e1 - - ' - - - - e , - - - - i
emf
e =
L
or -L di
di clildl
L 1---~--- e ------ol
Parallel: At the junction Fig. 8.87
ii+ i2 ... (i)
As the inductors are in parallel, so .,
el e 2 =e c,
L di1 dii' di
or - Id/ -Li-=-L-
di dt
K ... (ii)

di1 K : cli2 -K
or --.-=- i . I.
di -Li di. ½ ~
, ] 1,1: l+------e

-di
di --
K
-L
(.)

'.!
Fig. 8.88

--
.lffeJ=:.__E_LE_c_r._:u_1t_1_1Y_&_;,_,M_;A_G_,N_ETJ_,_·s_M_·------"------'"'-'-'----''
:11' ~
From equation (i), we have

-di di1
-+-
di2
dt dt dt

K K K
- ---
or
-l -Li ½
-I - -- i' - I
or
L
-+-
l1 ½ -,

Ex. 45 In the circuit shown injig. 8.89, the'emfl; of the source, its internal resistance, and the inductances
L 1 andL2 of superconducting coils ~re known. Find the currents established in the coils after key
K has been closed.

; ,. K
Fig. 8.89
Sol. Suppose i 1 and i2 are the currents in the respective coils and i is the current drawn from the battery,
then

r
ii+ i2 ...(i)
Inductors L 1 and L 2 are in parallel, and so e 1 = e2
''
or '½ di2
dt
On integrating both sides, we have
L1i1 ... (ii)
Solving equations (i) and (ii), we get

and Ans.

·~x. 46 A small cylindrical magnet Mis placed at-the centre ofa thin coil ofradius a, containingN turns
(see figure). The coil is connected to a galvanometer. The resistance of the circuit is R.After the
magnet had been rapidly removed from the coil, a charge q passed through the galvanometer. Find
the magnetic moment of the magnet.
ELECTROMAGNETIC INDUCTION
L

Sol. We know that, the charge flow

q
IA<i>I
R'
LJM
where l<1>1-<1>;1=1°-<1>I Fig. 8.90

= . <I>, ...(i)

q = R

<j> is the magnetic flux through the coil at the beginning of the process.

The quantity <j> can not be determined directly. This difficulty, however, can be overcome by using
the reciprocity theorem. We mentally replace the" magnet by a small current loop creating in the
surrounding space the same magnetic field as that of the magnet. If the area of the loop is A and
current in it is i, then its magnetic moment M= iA. According to reciprocity theorem M 12 = M21 and
the problem is reduced to determine the magnetic flux through the area A of the loop, which creates
the same current i, but flowing in the coil. Assuming the field to be uniform within the loop, we have

... (ii)

Substituting the value of <j> in equation {i), we have

q = µoN (iA)
2aR

iA Ans.

Ex. 4 7 Two coils, A of 12500 turns and B of 16000 turns lie in parallel planes so that 60% of flux
produced in A links with B. It is found that a current of SA ioA produces a flux of0.6 mWbwhile
the same current in B produces 0.8 mWb. Determine (i) mutual induction, (ii) coefficient of
coupling.
Sol. For coil A,

12500 x( 0.6x I o-3 )


NA<i>A
LA
5
0.15 H
ForcoilB,

16000x( 0.8 X 10-3)


Ns<i>B
LB
5
0.26H, ·.
•·-·•·- .. ,- ..-- · ··.c· ••.• , :·· · ·c'"fr .• ,.,., . . :·:·:·.• · ·'
ELECTRICIIY & MAGNETISM \: · .
___, .-- -- ----- .. ,__ .,. -~--·~-.. -- . ' · .' ·
.-- --- -,_~_ .. ..,,,..-_.._ ........ - ~ - --~ -

For the mutual induction. we can Write

, · Na~B
M -

( l 6000x0.6x0.6x 1 o-J)
5
1.15H
Thus coetlicient of coupling

M 1.15
~Li L2 .J0.15x0.26
0.586 An.,.
Total inductance
When coils are connected together in a circuit, they offer both selfindu,ction.and mutual induction. Their total
induction will be the algebraic sum ofselfinducti/m and mutual induction.
(i) Suppose two coils of self inductances Li and L2 are connected in series in such a way that their fluxes
are additive.

(/., + .\[) (/.,+.I[)

Coil I Coil 2

.!'• '.
" . Fig. 8.91
& ! !J·
If dt is the rate of increase of current h1 t~.~:~oils, then self induced emf in coil I,
di
e; - -Li dt
Mutual induced emf in coil I, due to change in current in coil 2
di
-M-
dt
Self induced emf in coil 2,
di
-Lz-
dt
Mutual induced emf in coil 2, due to change in current in coil I,
di
e/· = -M-
dt
Total induced emf in the combination

... (i)
_-:·.:,- _E.,L;~~;;~~~~!1.C INDUCTIO;---~
If LTotal in the total or equivalent induced emf in that single coil, then
If I 1,

e = l di ..,(ii)
- Total tjt
On equating equations (I) and (2). we get
LTotal - = L1 + lz + 2M ...(I)
(ii) When the coils are so connected that their fluxes are in opposite directions.
di
If J is the rate of increase or current in the coils. then
"' di A X
(l0 -MJ
--
(!
I
= -li-
d/.
I 11 Ill "- "-
".
di.
+,\1 -
dt

Fig. 8.92

'[ii'
and +M-
dt

The total induced em!'

Thus total inductance .... (2)

Ex. 48 Two coils with terminals T1, T 2 and T3, T 4 respectively are placed side by side. When measured
separately, the inductance of the first coil is 1200 mH and that of second is 800 mH. With T2
joined to T_,, the inductance between T1 and T4 is 2500 mH. What is the mutual inductance
between the two coils? Also, determine the inductance between T1and T3 when T2 is joined to T4•

(b)

Fig. 8.93
Sol. Given. L 1= 1200 mH, L2= 800 mH.
In.fig. 8.93 (a), LTotal 2500 mH. Thus
2500 L 1+L 2 +2M
or 2500 l200+800+2M
M 250mH.
In.fig. 8.93 (b)
LTotal l1+ lz- 2M
l200+800-2 X 250
1500mH' Ans.
~
•-·---'·-·_E_,_,L_E_C_T._R_IC_I_TY_.,_&"-)~MA_·_G_N_'ETl_··-S~M_··_ _ _ _ _ _ _,_"~,''_,._,_ _ _ __.~:_]

In accordance with the total inductance, the total energy of the two coils is the sum of energies due to their
self induction and mutual induction between them : Thus

U = 2lL.2 1,_.2 .,..


111 +2.LJ'l '2 +mt1l2,

when both the coils carrying currents in same direction, and


1 .2 I . .2 ..
U = -L1 11 +-Lz 12 -M1112,
2 2
when current are in opposite directions.

Ex. 49 A rectangular conducting loop in the verticalxz-plane has length L, width W, massM and resistance
R. It is dropped lengthwise from rest. At t= 0 the bottom of the loop is at a height h above the
horizontalx-axis. There is a uniform magnetic field B perpendicular toxz-plane, below the x-axis.
The bottom and top of the loop cross this axis at I= t I and/= t2 respectively. Obtain the expression
for the velocity of the loop for the time 11 t. t 2 •.

z
1V -,.
T
l
F

()
®13
mg
t X

Fig. 8.94
Sol. For O<; t <; 11; there is no induced current and hence loop falls freely. Thus its velocity is given by

v, = gt, = ~2gh
For 11 ,; I ,; 12 ; the emf induces across the bottom side of the loop, which is BvW, and induced

BvW
current i = R in counter clockwise direction. Due to this, it experiences an upward force, F=

BvW) s 2vw 2
BiW=B ( R w= R
Now using Newton's second law for the motion of the loop, we have

B 2vW 2 dv
mg--- m-
R dt

dv
or dt
[g 2 2
B vW ]
mR
I~ ELECJHOMAGNETIC INDUCTION

On integrating above expression, we have

I
Jdt
,,

2
vw
s mR
2
} -en { g
82 82w2]
v,mRw2 }] = [-(1-1,)--
or
[ en { g mR

or
e
[ a'w']
- (1-ti)-.-
mR

or V = Ans.

Ex. 50 A thermocole vessel contains 0.5 kg of distilled water at 30°C. A metal coil of area
5 x 10-3 m2, number of turns 100, mass 0.06 kg and resistance 1.6 0 is lying horizontally
at the bottom of the vessel. A uniform, time varying magnetic field is setup to pass vertically
through the coil at time I= 0. The field is first increased from zero to 0.8 T at a constant rate
between Oand 0.2 sand then decreased to zero at the same rate behveen 0.2 and 0.4 s. This cycle
is repeated 12000 times. Make sketches of the current through the coil and the power dissipated
in the coil as functions of time for the first two cycles. Clearly indicate the magnitudes of the
quantities on the axes. Assume that no heat is lost to the vessel or the surroundings. Determine
the final temperature of the water under thermal equilibrium. Specific heat of the metal= 500 Ji
kg-Kand specific heat of water= 4200 J/kg-K. Neglect the inductance of the coil.
Sol. The induced emf in the coil

e = -Nd<j,=-NA(dB)·
di dt
Induced current

!._=_NA(dB)
R R dt .

Given 0.8-0 =4T/s.


0.2-0
,·:'·. . . ~ ~:;\"'\'½!~--·-~.
ELECTRICl1Y &
-
MAGirnsM
"> . ..:,. ·- ' - ·~--- -····
:;-··. :

Thus for the time interval Oto 0.2 s.

100(5x10-3 )
----'-----'-- X 4
1.6
-1.25 A.
In the interval 0.2 to 0.4s. dB!dt is negative and so induced current will be+ 1.25 A.
Power dissipated P = ;2R = 1.25 2 x 1.6
= 2.5W
The total energy dissipated in 12000 cycles
E Pt
2.5 X ( 12(}(){) X Q_4)
12,101 1

1(.1)

+
II f-------+---...--~-• /(.,)
: 11,2 : 0.-l : 0.6 11.8
I---
(a)
/'(II")

2.5f-------------
+
f-------------~--- /(S)
(b)

Fig.H.95 (al Variation or ctirrcnt with time. (b) Variation or power ,nth time.

lfll.Tis the change in temperature' of water~ then by conservation of energy


12x l<P (mtc 1+m 2c2)t.T

12xI03

12x!03
( 0.5 X 4200 + 0.06 X 500)

5.6°C
Final temperature T;+f>T
JO+ 5.6 = 35.6°C Ans.

Ex. 51 A metal bar AB can slide on two parallel thick metallic rails separated by a distance C.A resistance
Rand inductance Lare connected to the rails as shown in jig. 8.55. A long straight wire carrying
a constant current i0 is passed in the plane of the rails and perpendicular to them as shown. The
bar AB is held at rest.at a distancex0 from the long wire. At I= 0, it is made to slide on the rails
away from the wire. Answer the following questions :
ELECTROMAGNETIC INDUCTION

.,

II
r················X,,··············-1

Fig. 8.55

di dlj)
(a) Find a relation among, i, di and di, where i is the current in the circuit and lj) is the flux
of the magnetic field due to the long wire through the circuit.
(b) It is observed that attime I= T, the metal bar AB is at a distance of 2~0 from the long wire and
the resistance R carries a current i1• Obtain an expression for the net charge that has flown
through resistance R from t= 0 to/= T.

(c) The bar is suddenly stopped at time T. The current through resistance R is found to be ~

at time 2T. Find the value of RL in terms of the other given quantities.
di
Sol. (a) If e is the induced emf and - is the rate of increase of current in the circuit. then by
dt
Kirchhofl's II law, we have
. di
e-,R-L- 0
dt
. di
or e = 1R+L-
di

or
l~;I .R l di
I+--
dt
... (i)

Fig.8.97 TI1e equivalent circuit.

(b) Equation (i) can be written as


dlj) R(idt) + ldi
or dlj) R(dq)+ldi
hir:I 0
E:11 ___________________________
ELECTRiCl'IY·& MAGNETISM ___,
On integrating, we get

$J q i;
J dq, RJ dq+LJ di
$; 0 0

<<1>1- <I>;) = .Rq + Li1


:. Charge flown during the time t = 0 to I= Twill be

q = .,.(ii)

The change in flux can be obtained as


2xo 2xo .
J BdA = J µoio (ldx)
2Jtt

µoiof en 2.
21t·
On substituting this value in equation (ii), we get
µoioe - L i1
q = - - e n 2- -
21tR R
(c) When the bar is stopped, the induced errifbecoines zero, arid so from ~quation (i)
di R
--dt
L
On integrating, we have

!L
1.
~i
I
R2T
--LTf dt
IJ

!L R
or [en{: --(2T-T)
L

RT
or en(¾) L
RT
or fn4
L
L T
Ans.
R fn4

Ex. 52 Two infinitely long parallel wires carrying current i= ; 0 sinoY in opposite directions are placed
at a distance 3a apart.Asquare loop of side a ofnegligible resistance with a capacitor of capacitance
Cis placed in the plane of wires as shown. Find the maximum current in the square loop.Also
sketch the graph showing the variation of charge on the upper plate of the capacitor as a function
of time for one complete cycle taking anticlockwise direction for the current in the loop as
positive. ·
r---- - - - -
! _ _ _ _ _ _ _ _,_ELI:~OMAGNETIC INDUCTION, ~

(a)
ill dx
x---11--
(b)

Fig. 8.98
Sol. Take an element of thickness dx at a distance ofx from left wire. The magnetic field at the position
of element
µ 0i µo i
B - +
21tx 21t (3a-x) upwar
d

µoi
21t
[.!__+-'
x
-]
3a-x
The magnetic fiux through the area of the element
dq, B(dA) = B (adr)

= µoi [.!__+-
21t x 3a-x
1
- ] (adr)

The total magnetic flux through the loop

<I> = µoia
21t
J[.!__+-'
a
x 3a-x
-]dr
ia [ Cnx-Cn (3a-x) ]2a
µ 0-
-
21t a

µoia [(en2a-Cna)-(ena-Cn2a)]
2Jt

- en 2a
µoia
-
1t
Given i = i0 sincot,

µo (i0 sinwt)aCn2a
1t
dq,
Induced emf lei dt

The maximum value of induced emf


~- . ELECTRICI1Y ~-MAGNETISM -~- ----- ... - ~- . ..,.. --- _,....' i._., .... '
Impedance of the capacitor

z
roe
Maximum value of the current

An.1·.

The charge on the capacitor


q Ce
µf/0 aroC In
~ ~ - - - - COSOO/
2a]
[ 1[

Thus maximum value of charge

'lo
= [µ 0 i0 aroC
1t·
(n2a]
According to the equation i = i 0 sinOY, current and hence flux in the loop increases initially in
upward direction, and so direction of induced current makes upper plate of the capacitor positive.
q

<Ju ·••••·••••••·•·•••·•·•••···•·•••

ll 1--~~-:7-,-2+---T_ ____.

-qn ···············

Fig.8.99 Variation of chmgc q with time.

Ex. 53 A thin wire ring of radius a and resistance ris located inside a long solenoid so that their axes
coincide. The length of the solenoid is equal to f, its cross-sectional radius to b. At a certain
momentthe solenoid was connected to a source ofa constant voltage V. The total resistance of the
circuit is equal to R. Assuming the inductance of the ring to be negligible, find the maximum value
of the radial force acting per unit length of the ring.
Sol. The self inductance of the solenoid
l = µ0 n 21tb 1 r.
where n is the number of turns per unit length of the solenoid. When solenoid is connected to a
source of a constant voltage V. a current is set-up. The current is given by

; = -V (1-e--
tR)
L
R
The flux through the coil
<l>coil

lei
The current in the coil

e
i'
r r
-- -- ....-.. - .--~· . -

: ~----~~ __ ;_,.,.,, ELECTROMAGNETIC INDUCTION


••• ----·· .:., ___ - __ - ~ ~. !.,__ -

, The force on unit length ,on the ring


· · · F = ·ifr\ 1=ui011hi'
2
µ" [.!'R.'..(1-e-'
11
.
i.}'xµo"""
r
~Ve-'Ri. ..x~]L_

~ ' - - - - J·e_,R
µ/1w2v2,,2 I. ( I - e -'~)
I.
[ rRL

dF
For maximum value ofF. -,- = O;
<,/

I
,By doing this the maximum value of the term is found lo be
4 . Thus
., ., ., , , , ., .,
µ0-na-v-,,- µ 0-m,-v-n-
4rLR 4r(µ0n2nh21')R

µ0 a-v-
' '
Ans.
4rRb2 t

8.15 EDDYS CURRENT ,, '


So far we have considered only instances in which currents resulting from induced em!' s were confined to well-
defined paths provided by the wires like coils or rings. However, one finds masses of metal located in changing
magnetic field or moving in a magnetic field, with the result induced currents circulate throughout the volume
ol'the metal. Because of their general circulatory nature. these are referred to as eddy currents.

Eddys

Fig. 8.100
Consider a metal sheet entering into a magnetic field perpendicular to the plane of the sheet. The magnetic flux
through the sheet increases. The current induced in the body of sheet in the form of eddys as shown in ,fig.
8./00.
The direction of eddy currents are such that, they compensate the increasing flux (by Lenz's law). Accordingly.
when metal sheet enters into the field, the direction of eddys are counterclockwise. When it is leaving the field
the direction of eddys will be clockwise.

8.16 AC GENERATOR OR DYNAMO

It is used to convert mechanical energy into electrical energy.


Construciion : The main components of ac generator are :
(i) Armature coil : It consist of large number of turns of insulated copper wire wound over iron core.
(ii) Magnet: Strong permanent magnet (for small generator) or an electromagnet (for large generator) with
I cylindrical poles in shape.
(iii) Slip rings: The two ends of the armature coil are connected to two brass rings R 1 and R2 • These rings
rotate along with the armature coil.
(iv) Brushes : Two carbon brushes (B I and B2), are pressed against the slip rings. These brushes are
connected to the load through which the output is obtained.
Principle: It works on the principle of electromagnet~c induction. According to it when a coil·is rotated in
magnetic field, an emfis induced in the coil. The coil may be rotated by water energy, steam energy oroil energy.
Let at any instant magnetic flux through armature coil,
Ncj> 8 NBA cos0 = NBA cosOY
dcj> •
The induced emf e = - -8-=NBAw sinro,,
dt
or e = e0 sinOY, where e0 = NBAro.

and induced current

(:;"'

B C

N s
i
A D

3f4
-io .................... :

Output

Fig. 8.101 AC Generator


s; 17 DC GENERATOR
It produces .direct current. It is possible by providing split rings ·or commutator in place of slip rings. DC
generator consists of:· 1 ,

(i) armature coil,


· . (ii) magnet,
(iii) · split rings,
(iv) · brushes.
For DC generator
output current

i r -_-_ , N

'oLYI\ • I
T/2 :r

Output
Fig. 8.IOZ DC Generator

8.18 DC MOTOR
It is an electrical machine which converts electrical energy into mechanical energy.
Principle : It i_s based on the fact that a current carrying coil placed in magnetic field experiences a torque.
Because of this torque the coil starts rotating.
Construction : It consists.of
(i) strong magnet.. B ·C
(ii) armature,
(iii) split rings, N S
(iv) brushes.

'---->A D ..__ __,

Fig. 8.103
Back emf:
As soon as motor starts rotating, the flux changes in the coil, because of this change in flux, an emf isinduced
in the coil which opposes.the emfofthe battery. This induced emf is called back emf. The net emfofthe circuit
e net, ·= ~• - e• where e = NBAro sinrot = kro
Annature current
enel = s-e
·R R .

s-kro
R
Armature current i will be maximum, when motor is just sta,rted i.e., co= 0, ;max= 'f)R. And minimum when motor
picks full speed

Efficiency of motor :
Ifs is the potential applied and i is the current in armature, then
si i2 R + mechanical power
Mechanical power s i-i2R = i (~ - iR)
ie (e is the back emf)
Available mechanical power
Efficiency input electrical power

ei e
"f;=t
11 is maximum, when (e 1) is maximum

Let X = ei=e(~~e)
' ' ' '
or ( es~e2)
'.. :":.• '-:.
!dx .,,
x to be _maximuffi, - =
'de . ' . ' 0 ,,
or ~.:.z.,' ·= o
JI I ,

which gives e 5. :.11=50%.


2

8,19 TRANSFORMER
Transformer is used to change the voltage in ac ci!"uits.
(I) It is based on principle of mutual induction.
(2) It works with ac only.
(3) It can not change the voltage without change in current. There will inverse change in current. For ideal
transfonner
pout Pin
or Vs is Vp ip

. , Vs _ ip
=> Vp is
... (Q
(4) Same flux links in both.primary. and secondary, so the induced emf per turn is.the same in·each.
.,
d4>s dq,p
i.e.,
dt dt
1§14cmoMAGNETICIND.UCTION . ; fiD
'• :.;· '· ., . ,,', ... d~p,,.,,,
For primary coil Va =,,.,,ep=Np--,
.-1" 1 ! ''."""~ ,t.:: ' dt, •, , ·:1 '~ ;·

,• •.••. " ·.: ";' a.•.'.··,, "d~s· .,


and for secondary coil 'vs = e =Ns--
s C dt . ->:' ,. '1 .:

es i dNsn
-=-- ...(ii)
.ep Np
From equations (i) and (ii), we get
' , If
;Vs ·es ip ... Ns·--
-=-=-=K ...(1) •
Vp ep, .. is Np
where K is called transformer ratio. K> ( for step-up ir~risformer and K < 1 for step down transformer.
- - - Laminated iron core
"

Output
Vs

Fig. 8.104 ;
(5) Symbol of transformer is
Losses in transformer
The output power of a transformer is necessarily less than the input power because of unavoidable losses.
These losses consist of PR losses in the primary and secondary coils, and hysteresis and eddy losses in the
core. Hysteresis losses are minimised by the use of iron core, and eddy losses are minimised by laminating the
core. The electrical resistance between the surfaces of the la'jninations (by insulating varnish) effectively
confines the eddy currents to individual laminae. The resulting path is greatly increased, with consequent
increase.in resistance. Hence, although the induced emf is not'altered, but the currents and their heating effects
are minimised.
,;I,, •. 1 ) - l

• I ~ ; •,

,01JJ!!
Cancellation
ctfoct
rf,1j1.J '

(a) Solid core with large (b) Laminated core (c) Effective eddys current
eddys in whole - . with small
. eddys.
. ,,
volume of core.
Fig. 8.1os.·' ·
·1
Efficiency of a transformer ( 11 ) 7.
.--: 1-"---·-
7
I
7 ·------
1 • p . V. .
·' 111(%) = ...J!!l1.x'1oo=~XIOO .-:.(2)
i- - Rin .___· ,,.._Vp __ip
_______
,, 1,'1

In practice 11 can be achieved from 90% to 99%.


Ex.54 A transformer has 500 primary turns and.10 secondary turns.
(a) If Vpis 120 V (rms), what is Vs with an·open circuit?
(h) If the secondary now has a resi~tive load of 15 il, what are the currents in the primary and
secondary? ~·- 1111,1
. ; ..1~ fl•
Sol. Given,Np=500, N,.= 10, Vp= 120V.
(a) For a transformer

.!:£ N_,
Vp Np

N,
V ·= Vp-·
s Np

120~=2.4 V Ans.
500
(b) The current in secondary coil
v,. 2.4
i_., =-
Rs 15
0.16A
We know that

ip ,NS
is I 'N p

ip ,.s Np
-
NS

10
- ,0.16x 500

=• 3.2mA Ans.

Ex. 55 A toroidalsolenoid has a mean radius of0'.12 m and a cross-sectional area 20 x 10'4 m2• It is found
that when the current is 20A, the energy stored is 0.1 J. How many turns the winding have?
. ' .
Sol. For toroidal solenoid energy stored ..

'U ·= -,
I L'2
2

. -- ·:. L [2x10-7 xN 2 x2ox10-4] ·( · ).2


or o·J '= - . X 20
· . 2 · 0.12
. , .
'N = 387 Ans.
,-----..,.------

Ex.56 An inductor ofinductance3H and resistance 6 Q is connected to the terminals of a battery of emf
12 V and of negligible internal resistance. Find
(a) the initial rate of increase of curre11! i,n .the fircuit, .
(b) the rate of increase of current at the instant when the current is IA,
(c) the current 0.2 s after the circuit is closed,
(d) the final steady state current.
Sol. Given;H=3H, R=6Q ,s= 12 V,

(a) We know that .... (i)

where

/ .5_
di io--
-e t
s-
R-
= - e t =--::et
I I

dt "'" l
,- l
R
di
Att= 0, le = s·=g=4Als
0
Ans. .
dt l l. 3
l 3
(b) The time of i= IA: -=-=-
R 6 2

From equatiori'(i) i,-e-{)


I
or e • 2

Now
di S --I =12
-et -X-
I
dt L 3 2
2A/s Ans.

(c) ioC,,..e,,:)

0.659A Ans.
•,\,

(d) Steady current s_=E_~2A , ,\\ Ans.


R 6, ., . I\," ·1
I / ; : I ' \ ' \\

Ex.57 Consider the circuit shown infig. 8.106. With switi,'1,s'1 clo~ed ~tl-~tlie,other two switches open,
the circuit has a time constant 'tc- With switch S 2-~Iosed and the'~ther two switches open, the
circuit has a time constant'tL. With switchS3 closed ~nd' rt~er ,lwo switches open, the circuit
oscillates with a period T. Find T. ' ·
'·Eiicmic11Y &MAGNETISM .7
---------'--·----------------j

s,
l _.,; C R
)

Fig. 8.106
Sol. When only S 1 is closed, the circui/ is active through C and R,
~c = CR
Similarly ~L = LIR
~C~L =-Lc-
Now when S3 is closed, the circuit elements are land C
T 21t,JLC
' Ans.
21tJ~c ~L

Ex. 58 In the given circuit containing an inductor and resistors. Find the time constant of the circuit.

Sol. For the given circuit, the effective resistance across the
inductor

•c·•1,1L·. 2 1
~ = -=-=-s Ans. Fig. 8.107
'- ' R '8 4

Ex.59 Two parallel wires whose centre! a!:._e at a distanced


apart carry equal currents in opposite directions.
Neglecting the flux within the wires themselves, find
the inductance of a length of such pair of wires, given dx
that the radius of the wires isa.
Sol. a
The magnetic field at distancex from left wire due to
the current in the two wires' is
xl]J+--(d- x)

B ~. '. ;(.!.+-•-)
21t d-xX

Fig. 8.108
Now consider an element of width dx and length f. The flux enclosed between wires
<!> 8 = Js(ldx)=- J - + -
µoiC d-a[ I I ]
dx
2it X, d-X
I ~-('' Q '

· · -µo
= iC [ fnx-fn (d-x
, )]d-a
21t a
. L;·· " fr,
."

_µ 0 iC(cn(d-a)-cn-a
2it ; a (d-a)
)
2

2it
cn(d-~a)
µ 0 iC
a
= µ 0 iC cn(d-a)
1t a
':',

The inductance l Ans.

Ex. 60 For the given circuit here, find the


(a) energy stored in C
(b) energy stored in L
(c) current in each circuit element and
(d) voltage across each circuit element.

8Q 2Q

,,
200 µF 12Q -=-1ov JOQ JmH

Fig. 8.109
Sol. After reaching steady state, capacitor stops the current. While inductor offers zero resistance. The
current in the right side of the battery will pass through inductor.
The above circuit can be reduced as follows :

B C D

,oo,,[ 20Q · JmH

i2
. ' <•
F E
Fig. 8.110
In close loop ABCFA,
20 i1 10

_l_A = 0.5 A
2
and in close loop FCDEF, 10
~ i2 5A
(a) The p.d. across the capacitor p.d. across 12 Q resistor

I
Energy stored in capacitor

.!..cv 2
·2
I . . .
= -x200x10--{j x62
2
3.6inJ Ans.
(b) The energy stored in inductor .... ·

=--!..li2
.2
I. - .
= x(3x10-3 )x(s)2
2
37.5ml .
.=
(c) The current in each circuit elements are sh.own_ as follows.,

'
•,

Zero _sn
200 µF 12n 10n 3mH

Zero current
. Fig>8.Jll
{d) The p.d. across each circuit element are 'shown in.fig. 8.1 / 2
,, ' .... :.: .
·10v
·+

+ +·
6V 6V ov ov

"Fig. 8,112

.·Ex. 61 ·, A voltage waveshape of the form V(/) =Kt3 is applied to acircuit efoment att_ime t= O. Find current _
through a resistor R, an inductor Land a capacitor C.

V Kt 3
Sol. Curreni through resistor i. ;;= -=-- Ans.
. R R
Ldi
To an inductor . Kt 3
dt
i KI
or · Jdr. = ~ft>dt ·
o L6 .
4
or Kt -• Ans.
4L
---·-_,.,.-~:-,-;-'"!-·- ..,.....- . ' . ' - ~ _ , . -_ _ _ _ _ ,,,,,, . ,. -~
· ./ -~ '"• • 0 • : ' . • \; • ~ •. ' ,: '•' :i ' • ~1;EuCTiloMAGNEilcflNriuCTioN , · ~}
~~ ,h I"/ • ".• _.::,_.,.~...;i.,_._...,..__~._-,: ' ',, . ' , . • . ' • . ,-, :t..::.::..~,b,..v~ -~· ,, , • _;___ .

Forcapacitor, q = CV
= CKt3
3
.dq d(CK1 )
Current
'dt = dt.
3CKt2 Ans.
' '

Ex. 62 In jig. 8.'JJ3,E;= 100 V,Rt= 10.on ,R2 =20.0'1 ,R3 =30.0'1, andL=2.00 H. Find the values
ofi 1 andi2 • · ·
(a) immediately after switch Sis closed;
(.b) a long time later;
(c) immediately after switchS is opened a_gain
(d) a long time later•..

At
- +
r
~
-
Iii i I B
ii

R,.
II,
C

f,

F E
I)

Fig; 8.Jl3
Sol. (a) · immediately after switch Sis closed, inductor offers infinite resistance, therefore current in its
arm will be zero. Fdrd_osed lo?pABEFA . . .

(b) After a long time, inductor becomes ineffective. The effective circuit becomes

A
s . .R_,... i,. B JI
•••3•. i.,
C
;,

Sa :,-+ ~ R,
F D
E
Fig.. 8.114
. E;
iI RR
RI +· 2 3
R2 +R3

100
IO+ 20x30
20+30
4.55A
. . 2.
We have i 1 = ;2 + ;3 and 20 ; 2 = 30 i 3 ~ 13 =:i"'2
. 2i2
4.55 '2+-
3
or i2 2.73A,i3 =1.82A
(c) Immed(ately aft~r swit~h Sis open:d, ; = 0 _and i~ =_i3 = 1.82A
.- l.! ''i { ' ' !\. · ...
,, ,·t ., . 1
.' ,.., ..
,.
I',. ,- .!

.. '\,'

,,.
'· , r.

Fig. 8.115 ·
(d) After a long time, the magnetic energy stored in inductor will change into heal and therefore
;, = i2 = 0.

Ex.63 In the circuit diagram shown in jig. 8.116, R =!Oil, L =SH,!;= 20 V, i= 2A. This current is
decreasing at a rate of 1.0 A/s. Find Vab at this instant.
R ; L i;
a ~1----o b
Fig. 8:116_
·sot. The induced emf across inductor
.R . ; e = 5V i;
-Ldi
e = a-t----111--- b
dt i-------Voo------i
-5x,(-!.O) Fig. 8.117
sv
As the current is decreasing the inductor can be replaced by a source of emf 5V in such a manner
so that it compensate the decreasing current. It to be like as shown in jig.8.117.
Now by loop rule · -----
-Ri+e-~-Vab = 0
or V0 b = Ri+~-e
'!0 X 2+20-5
35V Ans.

Ex. 64 Find the time dependence of the currentflowing through the inductance L of the circuit shown in
jig. 8.118 after the switch Sis shorted atthe moment/= 0.

Sol. Let at any instant the current in the inductor is ; 1 and increasing at the rate of d; . The induced emf
di-

across L(e = L dii ) opposes the increasing current. So it to be as shown in the circuit.
dt . -. -
In close loop AB CDEFA,
-e-{i 1+i2)R+~ = 0 ...(i)
. ,''"!,

In close loopB CDE B,


-e+i2R = 0 ... (ii)

~s
· From equation (ii), we have i 2 = e IR ;,
B C
substituting this value in equation (i), we get
-e-{i 1+e/R)R+~ 0 ;?
or -2e-i 1R+~ 0
s ..
2l di1
,:R
> l !,
or
dt (i,+i,) R
F E D
or Fig. 8.118

A s !3 ;, C
in(~-i1R) ;, t
i2
(-R) o 2l d(
sa-s ~II a:-e=l_!!.
de
Rt
or
2l
F
(i,+i,)
-~- E D,
Rt
or Fig. 8.119
2l

or

\
&U'td4e F.I
. 1. The graph gives the magnitude B(t) ofa uniform 4. Figure shows three circuits \vith identical
magnetic field that exists throughout .a· batteries, inductors, and resistors. In_ which
conducting loop, perpendicular to the pl~ne of circuits the time taken to reach 50% of the
the loop. In which region, the magnitude of maximum current is greate~t.:
induced emf will be greatest:

lJ

(a) I
000 (I) (2)

(b) 2
(3)

IV (c) 3 (d) 2,3


5. Figure shows a· long conductor carrying
increasing current in it. Three rectangular wire
(a) I ·. (b) IJ: loops with edge length l, 2l and 3l are placed
(c) nr (d) IV without touching the conductor. In which loop
2. The figure. shows three ·situatioil_s in which the induced current is maX.imum: ·
.identical circular conducting loops are in uniform
magnetic fields that are either increasing or
· . decreasing: in magnitude at identical rates. In
each, the dashes line coincides with a diameter. ·
In Which' situation the magnitude of the induced·
current is maxi~um : .

(I) (2) . (3)

(a) l (b) 2
.. '" •. (c) 3 (d) 1,3
I 11 . lll 6. If the circular conductor undergoes thermal
(a) I (b) I, 11 · expansion while it is in a uniform magnetic field,
(c) ll;lll (d) · 1, lll a current will be induced _in clockwise around it.
3. The figure· shows. three circuits with ideniical If Bis the magnitude of the magnetic field; then
batteries, i_nductor, ijO.d registers. lri.which circuit,- its direction is :
the current through the battery· is greatest just ,
after closing of.tHe switch

(a)
oog
. ~ . ~). d)
I (b) 2
(a) Bi (b) Bj

(c) 3 (d) 1,3 (c) -Bj (d) Bk


7. A circular loop of resistance Rand enclosing an
area A is subjected to a magnetic field a0e-b1
perpendicular to its plane, 8 0 and b being
constants, and I is the time. What is the induced o',,
current at I =O ?
Loop
(a) AB0b (b) B0bR
R A (a) OV. (b) lOV
(c) IOOV (d) none
ABoR BohR II. Figure shows· a circular region in which an
(c) b (d) 2A increasing uniform magnetic field is direcied out
8. The magneti~ field through a loop of area. I m2 oftlie page, as well as a concentric circular path
and of resistance IO O , changes with time as
shown in figure. The induced current in the loop
along which f, E•dc· is to be evaluated. The
att= Is table gives the initial magnitude, and the time
interval for the increase, in three situations. The
B('l).
situation according to the greatest. magnitude of
the electric field induced along·th.e path :

Situation Initial field · Increase Time

. 1(s)
A 81 M1 Lil1
0 2 4 6 B 28, lilJi '12 111,
(a) 0.1 A (b) 0.2A 111,.
(c) 0.4A (d) 1A C 8 1/4 1181
9. A square loop with 2.0 m sides is perpendicular
to a uniform magnetic field,.with half the area of . .
the loop in the field is shown in figure. The loop
contains a 20.0 V battery with negligible interval
resistance. If the magnitude of the field varies
with time according to B = 0.042-0.87 I, with Bin
tesla and I in second. The net emfofthe circuit is (a)
(c) C
6""~
A (b) · B
(d) A andB
12, The switch in the circuit offigure has been closed
. for a very long time when it is then thrown io b.
The resulting current through the inductor is
indicated in the figure for four sets of values for
the resistance Rand inductance L : (I) R0 and L0
. (2).2 R0 and L0, (3) R0 and 2L0. (4) 2R0, 2L 0.
(a) 20.0V (b) 18.26V .Select the correct matching : _
(c) 21.74 V (d) none

~~'
10. In figure, a circular loop of wire IO cm in diameter
is placed with its normal N at an angle 0 = 30°
with the direction ofa uniform magnetic field ii .
of magnetic 0.50 T. The loop isthen.rotated such
that N rotates in a cone about th~ field direction "(a)·.
A-1,B-i,C-3,D-5. .
at the constant rate of I 00 rev/min; the angle 0 (b) A-4,8-3, C-2,D-I
remains. unchanged during the process. The _(c) A-2,B-4,C-1,D-3
inauced emf in the lo.op : .. . (d) none
13, Transforll)ers are used 18. · ·A series woulo de motor has a total resistance of
(a) in AC circuits only 1.5 ohm. When connected across a 115 volt and
(b) in DC circuit only running at a certain.speed· it draws a current of
(c) in both AC and De circuits ·IO A. The back em fin the motor is
(d) neither in AC nor in DC circuits. (a) IOOV (b) 115V
14. A generator supplies to the primary-coil of a (c) 15V (d) I.SY
19. Which ofthe following can produce the maximum
transformerof50 turns. If the secondary coil has
induced emf?
500 turns, then the secondary voltage is
(a) 50A,DC (b) 50A,50HzAC
(a) IOOV · (b) 500V · (c) 50A, 50Q Hz AC (d) IOOA, DC.
(c)· ssov (d) I_ooov· 20. Two circular locips of equal radii are placed
15: Consider the situation shown ·in figure. If the coaxially at some separation. The first is cut and
•switch-is closed and after some time it is opened a battery is inserted in between to drive a current"

,.,· . o·o
again, the closed loop.will show

, ... ___ I--"


(a) a clockwise.current
(b) an anticlockwise current
.'
21.
in it. The current changes slightly because of
the variation -in resistance with temperature.
During this period, the two loops
(a) attract each other(b) repel each other
(c) do not interact (d) none of these
A metal rod is moved with a constant velocity v
in a magnetic field. A potential difference appears
(c) ,an anticlockwise current and then clockwise_ across the two ends ,
(d) a clockwise current and·then an anticlock (a) ifvff (bJ ifv[lii
wise current.
I 6. In the circuit ~hown, @ indicated a µniform (cJ ift[lii
r
(d) none of these
magnetic field, which is directed into the page 22. e
A rod of length rotates with a uniform angular
and decreasing in magnitude at the rate of velocity co about its perpendicular bisector. A
I 50T/s. The reading ofammeter is: uniform magnetic field B exists parallel to the
axis of rotation. The potential difference between
10n two ends of the rod is

(a) J.soo2 e (b) Boo2f.


2
Size of the loop
' 2 (c) 2Boo 2{ (d) zero
= 10•10 cm 23. A magnet NS is suspended from a spring and
• _5V while it oscillates, the magnet moves in and out
of the coil C. The.coil is connected to a
(a)' 0.15A '' ·(b) 0.35A · galvanometer G. Then as the magnet oscillates,
'(c) · 0.50A (d)0.65A
17. A· circular wire loop of radius r is placed in a
region of magnetic field B such that the plane of
the loop makes an angle 8 with the direction of
B . Consider the following in this regard :
I. Change in B with time
2. Change in r with time
3. B being non-uniform in space
4. Change in 8 with time (a) G shows deflection to the left and right with
The conditions for an induced emf in the loop constant amplitude
(b) G shows deflection on one side
would include
·•• (C) 'G shows no deflection
(a)· land4 (b) l,2and4· (d) · G shows deflection to the left ana right but
(c) I and3 (d) 2,3and4, the amplitude steadily' decreases.
24.· A coil having n turns and resistanc~ R Q is' Biv "· 'lilv
connected with ·a galva~om~ter of r~sistance
(a) R clockwise (b) R anticlockwise

4R Q. This combination is moved in time I 2B/v


(c) R anticlockwise (d) zero
second from a magnetic field W1 weber io W2
weber. The induced current in the circuit is 28. One conducting U tube can slide inside another
as shown in figure, maintaining electrical
W2 -fl~ contacts between the tubes. The magnetic field
(a) (b) Bis perpendicular to the plane of the figure. If
5 Rnt
each tube moves towards the other at a constant
speed v then the emf induced in the circuit in
terms of B, eand v where l is the width of each
(c) (d)
tube, will be
25. An infinitely long cylinder is kept parallel to an
B,~-=:;-\c'iJ
uniform magnetic field B directed along positive
'
z axis. The direction of induced current as seen ,
from the z axis will be
(a) clockwise of the +vez axis
J-V
.
\'~

(b) anticlockwise of the +vez axis


A D
(c) zero,
(d) along the magnetic field (a) zero (b) 2 Blv
(c) Blv (d) -B
26. An electric potential difference will be induced
29. The inductance of a closed-packed coil of 400
between the ends of the conductor as shown in
turns is 8 mH.Acurrent of5 mAis passed through
the diagram, 1Vhen the conductor moves in the it. The magnetic flux through each turn of the
direction coil is
I
(a) -µ 0 Wb (b)
41t
I
(a) p (b) Q (c) -µ 0Wb (d) 0.4~ Wb
31t
(c) L (d) M 30. The adjoining figure shows two bulbs Bj and
27. A conducting square loop of side L and B2 resistor Rand an inductor L. When the switch
resistance R moves in its plane with a uniform Sis turned off
veloc,ity v perpendicular to one of its sides. A ~-w.--1 B.\---"-s, __
magnetic induction B constant in time and space, R
pointing perpendicular and into the plane of the
loop exists everywhere. The current induced in
the loop is L

'B
, ., _ C ,,,__
, B 11 I - - - - - - - - - '

':·o·- ,
X
• )<

X
' "

>
,, ,<;

X
V
(a) Both B I and B2 die out promptly
(b) Both B I and B2 die out with some delay
(c) B I dies out promptly but B2 with some
I. '; ,. ,, delay
(d) B2 dies out promptly but B I with some
,
delay ·
31. An electron moves along the line AB, which lies · 35. A small square loop of wire of side .l is placed
in the same plane as a circular loop of conducting inside a large square loop of wire of'side ·
L (L > C): The loop are coplanar and their centre
wires as. shown in the diagram: What will be the
coincide. The mutual.inductance of the system
directiqn of current induced if any, in the loop is proportional to
·,
(al e
IL · (b) 2/ L • e

0
(c) L le . (d) L2 le .
36. A wire oflength I m is moving at a speed of2ms- 1
perpendicular to its length and a homogeneous
magnetic field of0.5 T. The ends of the wfre are
A-~------8
joined to a,circuit ofresistance,.9 ..Q. The rate at
(a)
no current will be induced which work is being done to keep.the wire moving
· '(b)
the current will be clockwise .. ·at constant speed is .
(c)
the current wilt be anticlockwise
(d)
the current will cha~g~ .direction as the (a) _!_w (bl .!.w
12. 6,
·electron passes by
· 32. A thin semicircular conducting ring of radius R I
(c) -W (d) IW
Js falling with its plane vertical in a horizontal 3
. magnetic induction B. At the position MNQ, the 37. A uniform but time-varying magnetic field ·a(t) ·
speed of the ring is Vand the potential difference exists in a circular region of .radius a· and. is
·developed across the ring is · directed into the ,plane of the paper, as. shown.
The magnitude of the induced electric fieid at ,
X y x· X X point P at a distance r from the centre of the ·
B circular region - ·
){ X ·xN X· X

X X
X X V X X · 'B(I)' ·· P·
X X .X .

X X X X .x ' ' X
X

X X X X X
!yf Q .X X
X !<
X X X X
(a) Zero a
X
(b) Bv1CR2 I 2 and Mis at higher potential X
X

(c) ltRBVand Q is at higher poiential


(d) · 2RBVand (2 is at higher potential . I
i
33. A circular loop ofradi~s R carrying current lies . (a) is zero (b) decreas~s ·as -
r
in xy; plane with its centre at origin. The total , .,
. . ·I
m·agnetic flux through xy:plane is · ·
(c) increases as r ' · (d) decreases as 2 ·
(a) directlyproportional to/. r
(b) _directly proportional to R 38. A coil of wire having finite inductance and
. (c) directly proportional to R2 resistance has acopducting ri_ng placed coaxially
(d) zero within it. The coil·is connected to a battery at
34. Two identical circular loops of metal wire are lying time t = 0, so that a tiine-dependenfcurrent1 1(t)
on a table without touching each ·other.' Loop-A starts tlowi~g ihrough the coil. If' / 2(,) is the
c~rries ·a current which increases with time. In . current induced iii the ring and B(t) is the
response, the Ioop-B magnetic field at the axis of the coil due to/1(1),
(a),' remains ~iationary . then as a functioii'of.time (I >.O), the product.
I2(t) B(t)
(b)' is. attracted by the .loop-A
(a)· increases with time
(c) is repelled by the loop-A
(b)' decreases with time
(d) rotates about its CM, with CM fixed ( CMi~
(c) does not vary ;,,1th time
·.the centre.of mass).
(d) passes through a maximum
~l;_~:~;_··-"-'----"'-~-~---'--,'--'----E_u_cm_o_MA_G_N_-ET._1clNDU0~.;--~
39. Two circular coils can be arranged in any of the 43. A conducting wire frame is placed in a magnetic
three situations shown in the figure. Their mutual field which is directed into the paper. The
inductance will be magnetic field is increasing at a constant rate.

~;.<r:
The directions of induced current in wires AB

.c= 0 and CD ar:


~~~o
1

(AJ (BJ
(aJ maximum in situation (AJ
(CJ
X~~ X

X X X X DX
(bJ maximum in situation (BJ (a) B to A and D to C
(cJ maximum in situation (CJ (b) AtoBandCtoD
(dJ the same in all situations (c) A to Band D to C
40. As shown in the figure, P and Qare two coaxial (d) B to A and CtoD .
44. Shown in the figure is a circular loop of radius r
conducting loops separated by some distance.
and resistance R. A variable magnetic field of
When the switch Sis closed, a clockwise current
Ip flows in P (as seen by EJ and an induced induction B = B0e-t is established inside the
coil. If the key (K) is closed, the electrical power
current IQ,, flows in Q. The switch remains closed developed right after closing the switch is equal
to
for a long time. When Sis opened, a current IQ,
X Bx li
X X
flows in Q. Then the directions of IQ,, and X
X X
X X
X X
X
IQ, ( as seen by E) are X X

BJrrr 2 Bo ,o,.J
(aJ (b)
R R
BJrr 2r 4R BJrr2,-4
(c) (dJ
5 R
45. A highly conducting ring of radius R is
perpendicular to and concentric with the axis of
(a) respectively clockwise and anticlockwise
a long solenoid as shown in fig. The ring has a
(bJ both clockwise narrow gap of width din its circumference. The
(cJ both anticlockwise solenoid has cross sectional area A and a uniform
(dJ. respectively anticlockwise and clockwise internal field of magnitude B 0• Now beginning at
41. A short-circuited coil is placed in a time-varying I i= 0, the solenoid current is steadily increased
magnetic field. Electrical power is dissipated due to so that the field magnitude at any time t is
given by B(tJ = B 0 + at where ex> 0. Assuming
to the current induced in the coil. lfthe number
that no charge can flow across the gap, the end
of turns were to be quadrupled and the wire ofring which has excess of positive charge and
radius halved, the electrical power dissipated th~ magnitude of induced e.m.f. in the ring are
would be respectively
(aJ halved (b) the same
(c) doubled (d) quadrupled
42. A coil ofinductance 8.4 mH and resistance 6 n
is connected to a 12 V battery. The current in the
coil is 1.0 A at approximately the time
(aJ 500 sec (bJ 20 sec (aJ X,Aa (bJ
(c) 35 milli sec (dJ I milli sec (cJ Y,rrA 2cx (dJ
3 E 1::
46. The current in a LR circuit builds up to th of
4
its steady state value in 4s. The time constant of
(a)_ (b)
this circuit is

I 2
(a) --s (b) --s
fn2 fn2 E

3 4
(c) --s (d) --s
fn2 fn2 (c) (d)
47. A conducting ring ofnidius I_ meter is placed in
an uniform magnetic field B of 0.0 I telsa
50. An alternating current of frequency 200 rad/sec
oscillating with frequency I 00 Hz with its plane and peak value IA as shown in the figure, is
at right angles to B. What will be the induced applied to the primary of a transformer. .If the
electric field coefficient of mutual induction between the
(a) 1t volt/ m (b) 2volt/m primary and the secondary is 1.5 H, the voltage
induced in the secondary will be
(c) I0volt/m (d) 62 volt/ m (a) 300V i ..
The graph shows the variation in magn~ti~ flux

:; :~ :fY'v,
48.
$(I) with time through a coil. Which of the
statements given below is not correct? ,
(d) 471 V
51. A horizontal loop abed is moved across the pole
B pieces of a magnet as shown in fig. with a
constant speed v. When the edge ab of the loop
enters the pole pieces at time / = 0 sec, which
one of the following graphs represents correctly
the induced emf in the coil
A

(a) There is a change in the direction as well as


I
e 1·,
lb -,·
N
0
=-<~~?l
magnitude of the induced emf between B d~----'.
andD
(b) The magnitude of the induced emf is
l_,_s V

49.
maximum between B and C
(c) There is a change in the direction as well as
magnitude of induced emf between A and C
(d) The induced emf is zero atB
The variation of induced emf(£) with time (1) in
(•) "L.,
OL_t O
'L I

,., 1u1,,,, :prr.


a coil if a short bar magnet is moved along its
axis with a constant velocity is best represen_ted
as

~IS~NI~
52. A rectangular loop is being pulled'at a·constant 54; The current i in an induction coil varies with time
speed v; through a region of certain thickness d, i according fo the graph·shbWn in figure:
·,, I Cl/
··. in which a· uniform magnetic field.B.is set up. '. ' r: '
,'.fhe graph between· position x of the right hand 1ti llflll?J
.edge of the loop and the induced emf E will be
,----d----
'\ '' ':"x 'x " X x .X x;

X X X x Bx x· Which of the following graphs shows the


......
X
X X X X X induced emf(£) in the coil with time?

Grx E

EL .
X X X X

X X X X X X
(b)
. <•>
•X X X X X. X
... , ... ,
,.,.1 j •'J
ODL__t
E
0''---~-

.. ,~·it>"1.1i

(c) (d)

53.
, :~,,, :fv-,
A flexible wire bent in the form ofa circle is placed
55. A square loop of side 5 cm enters a magnetic
,,L -field with I cms- 1. The front edge enters the
,.,, :magnetic field at I = 0 then which graph best
depicts emf
,.._,
5cmxxxxxx
,O,.x x x x x xB:::0.6T
in a uniJ'orm magnetic field perpendicular to the'
plane of the coil. The radius of the coil changes X X X X X X

as shown in figure. The graph of induced emf in


the coil is represented by
r

(a)
1,..kl_
0
X

"'
l---20cm~

0 5 15 20 I(.<)
F
'J'...• II J
10
e
,;:,@
20 l(s)

(b) .. :., lb,;,,:,· F


(a)
,., 1
' i' n,._ l"'- ' C
I X
l,, 2 5
."'0
·, '.(cj --S---L..-L20-1(,)
15
e

·liL
:.; I ~IT
,·11 ,
'" 1,:
(c) ·~(d)
\,tr·. "10
-"' X
15 20, .t(s)
0 ' I 1 (d) 0 5
I 2 O l 2
56. A magnet is made to oscillate with a particular 58. A sliding wire of length 0.25 m and having a
frequency, passing through a coil as sh'own in resistance or' 0.5 ·Q moves along conducting
figure. The time variation of the magnitude of guiding rails AB and CD with a uniform sp~ed of
e.m.f. generated across the coil during ori~ cycle 4 mis. A magnetic ·field of0.5 T exists ,normal to
is the plane of ABCD directed into-the page. The
guides are short -circuited with resistances of 4
and 2 Q as shown. The current through the
sliding wire is:

X X X X X
A M B

4Q
X i
0.25m
0.5Q
X

2n
X

4m/s
X X X X

C D
X

X
X

X
tC
X
0.5 T
X

X
X

(a) 0.27 A (b) 0.37 A


(c) LOA (d) 0.72A
59. The magnetic fi~ld in a region is given by

B = Bo (I+~),;. A square loop of edge-length


57. A line charge (cr perunit length) is in the form of
a circular wheel of radius a and is free to rotate in dis placed with its edges along the x and y-axes.
a horizontal plane. There is a magnetic field inside The loop is moved with a constant velocity
B = B0k extending upto a radius b (b < a). lfthe v = v0 i. The emf induced in the loop is :
magnetic field is switched off, the frequency of (a) zero (b) v0 B0d
the wheel is given by(/ is the moment ofinertia) 2
v 0 B 0 d3 v 0 B0 d
(c) a2 (d) a
60. A circuit contains an inductance L, a resistance
Rand a battery of emf~. The circuit is switched
on at I= 0. The charge flown through the battery
in one time constant (t) is:
.,
2~~
(a)
R.e
(bl l.~
R e
1ta 2baB .
(a) - - - clockwise as seen from above
I
(c)
..2...
2Re (d) zero
2
(b) 1tab aB clockwise as seen from above 61. Two coils, one primary of 500 turns and one
I secondary of25 turns, are wound on an iron ring
2 ' ' of mean diameter 20 cm and cross.:sectional area
(c) 1tabaB
- - - antic wise as seen r,'
·1ock. rom abave 12 cm2• Ifthe permeability of iron is 800, the mutual
I inductance is :
2 . (a) 0.48H (b) 2.4H
(d) xabaB
- · iockwise
- - antic · as seen f,rom abave
I . (c) 0.12H (d) 024H
S,
62. In the given circuit containing an inductor and 64:r, A voltage wave shape of the form V(t) = Kt3 is
resistors, the time constant of the circuit is : · '.',"'applied to a circuit element at time I= 0. The
~ 'c'µrrent through a resistor (R), an inductor (L)
. '.' and a capacitor ( C) respectively are :

K 3 K t3 3CKt 4
(a) -I - -and--
R ' L 3 4

K 4 K t 4
3KC t2
2
(b) -I --and---
4R 'L 4 2
I I
(a) -s (b) -s 4
4 2
()
c
K
R
t3 'LK !_and
4
3CKr 2
I I
(c) -s (d) -s
3 5 3
K 3 K t 3
63. The current through an inductor of I H is given (d) - t - - and CKt
.R 'L 3
by i= 31 sin I. The voltage across the inductor of
65: A short magnet is allowed to fall along the axis
I His:
of a horizontal metallic ring. Starting from rest,
(a) 3 sin t + 3 cos I (b) 3 cos I+ 1 sin t
the distance fallen by the magnet in one second
(c) 3 sin t+3tcos t (d) 3tcos t+ sin t
maybe:
,, (a) 4.0m (b) 5.0m
(c) 6.0m (d) 7.0m

r ,Ji, 1
Directions (Qs. 1-5) : Read the following questions (a) The current in R I is zero just after closing
and select the right choices. More than one options the switch S.
may be correct. (b) The current in R 1 is maximum just after
I. The figure shows four wire loops, with edge closing the switch S.
lengths of either for U. All four loops will move (c) The current in R2 is zero just after closing
of the switch S.
through a region of uniform magnetic field B at
(d) The currents in the resistors are maximum
the same constant velocity. In which loop the of their values a long time after closing the

oero o-~:
emfinduced is maximum: switch S.
3. L, C and R represent inductance, capacitance
·s and resistance respectively. Which of the
following have dimensions of frequency ?
I II III IV
L I
(a) I (b) II
(c) III (d) N
·(a)
C (b) .fie
2. Figure shows a circuit with two resistors and an R I
ideal inductor. (d) L (d) RC
4. A conducting loop is placed in a uniform magnetic
, field with its plane perpendicular to the field. An
R emf is induced in the loop if
(a) it is translated
(b) it is rotated about its axis
(c) it is rotated about its axis.
(d) it is expended
5. Two different coils have self-ind'\<;tanc,e 8. The induced emf in co.ii P when current in the
L 1 = 8 mH,L2 = 2mH. The current in one coil is coil Q decreases from SA to 3A in 0.04 s:
increased at a constant rate. The current in the (a) I0.28xl0-3 V (b) 15.75xl0-3 V
second coil is also increased at the same ftlte.At
a certain instant oftime, the power given to the (c) 19.nxio..:3 v (d) 21.nx10-3 V
two coils is the same. At that time the current, 9. Charge passing through coil P i_f its resistance is
the induced voltage and the energy stoled in 8ohm:
the first coil are i 1, V1 and W1 respectively.
Corresponding values for the second coil at the (a) 9.86x10-5 c (b) I 2.48x 10-5 c
same instant are ;2, V2 and W2 respectiveiy. Then
(c) 14.29xl0-5 C (d) 18.86 X I0-5 C
; I [,
(a) -1= - (b) .i=48 . ~
i2 4 i2 Passage for Q. 10 to·Q. 12
A thin non-conducting ring of mass m radius a carrying
a charge q rotates freely about its own axis which is
(c)
ff/.
-2=4 (d)
V2
-=- '. vertical. At the initial moment, the ring was at rest and
Wi Vj 4
po magnetic field was present. At instant t=0, a µnifo;,,,
6. Two metallic rings A and B, identical in shape magnetic field is switched on which is vertically
and size but having different resistivities-pA and downwards and increases with time according to the
p8 , are kept on top of two identical solenoids as law B =B0t. Neglecti~g magnetism emf induced due to
shown in the figure. When current I is switched rotational motion of the ring.
on in both the solenoids in identical manner, the
rings A and B jump to heights hA and h 8 ,
respectively, with hA > h 8 . The possible
relation(s) between their resistivities and their
masses mA imd m8 is(are)

A B
10. The magnitude of electric field at any point of
the ring is:

(a)
E= a Bo .
(b) E=aB0
2

(c) E= ltBo (d) zero


rr,·) 2
(a) pA>p8 andmA=m8 11. The angular acceleration of the ring and its
.(b) pA<p8 andmA=m8 direction of rotation as seen from above
(c) pA>p8 andmA>m8
Boq Boq
(d) pA<p8 andmA<m8 (a) (b)
m 2m
2B0 q 1t Bo q
(c) (d)
Passage for Q. 7 to Q. 9 m m
A circular coil P of IO00 turns and radius 2 cm is placed 12. Power developed by the forces acting on the
coaxially at the centre ofanother circular coil Q of I 00 ring as a function of time is :
turns and radius 20 cm.
q2 Bla 2t q2Bla2 t
7. The mutual induction between the coils is : (a) (b)
2m 4m
(a) 8.94xl0-4 H (b) 6.08xl0-4 H
1tq2Bla 2t
(c) 4.49xl0-4 H (d) 3.94xl0-4 H (c) (d) none of these
m
.o l"OJ''
Passage for Q. 13 to Q. 15 . 14. 'The total Lorentz force acting on the loop in
• ru,:., ·,
.. :rm 1, 2
lj .JJ(,> F= B~;v(-J)N
':, i, d [l

+ z direction, B 0and a,are positive constants. A square 2


,·· I .•!!.,~
loop EFGH of side a and mass m and resistance R; in . ,,; • -(b)
- =
F B0-a-- + j N v( ")
R
zy-plane, start falling under the influence of gravity. •-. v,k>- IHJ!
Note the directions ofx and y axes in figure. Ir: 11, 2 2
.•
/

>iT(~) F _B_,,o_;_v (-J) N


0-~~~~~~-x
X X X X X

:Eo~F:r
( d) none of these
15. The terminal speed of the loop is
mgR mgR
X X X X •. (a) Ro2 a 2 (b) B0a
X
HX X X
GX
y
_.,,1;;rlp fll 2mgR mgR
13. ir ,; is the sp;ed of the loop at any ti;,,e, the /f) B/a2
(d)
2B/a2
induced current in the loop is : . ;_, ,.biun
r •• rl:_i;, .¥:
B av "
(a) R0 anticlockwise ·'J nm,
h,-,r
\
2B av .' : ·. ',d ; ~ (JI
(b) _o_
R .
ant1clockw1se
'' w Lr,
0 2B av
' (c) -ii.- clockwise

B0 av' · -
"
(d) -R- anticlockwise

Multiple matching
16. Column/. Column//
(A) Dielectric ring uniformly charged (p) Constant electrostatic field out of system
(B) Dielectric ring uniformly charged (q) Magnetic field strength
rotating with angular ve_locity co
(C) Constant current in ring i (r) Electric field (induced)
(D)" i = i 0cosiix {s) Magnetic dipole moment
• ,1 \

·i· •

:r. lqA
,· [ I r}
' .
·'
- '
J '. ! .

• J • •
I " il
,,
- - .- .- -- . --.. .,·-·------------~--.:;-,--;-·-;---;,'-...-,:·~-;-7
Read the two statements carefully-to mark th.e correct option-outo.fthe options ghien below. Selecnh,e_right 'i
, · ; , :, · ' , r, : ,~ , · - ."' ··' · . l' ' ·~
ch01ce. . . .· ··• ···: ·.1
(a) If both the statements are true and the Statement~ 2 is the correct explJnation of Statemenl'-1 . .· ·. I
(b) If both the statements are true but Statement -2 is not the c~rrect explanatio~ of the S~te;nent'- 1.- · !
(c) If Statement- I true butStatement-2 is false. i
(d) If Statement- 1 is false but Statement- 2 is true. __ , J

1. Statement - 1 6. Statement - 1
Induced emf will always occur whenever there is Faraday's laws are consequence of conservation
change in magnetic flux. of energy.
Statement - 2 Statement - 2
Current'always induces whenever there is change In a purely resistive ac circuit, the current legs
in magnetic flux. · behind the emf in phase.
2. Statement- 1 7. Statement - 1
An emf can be induced by moving a conductor Only a change in magnetic flux will maintain an
in a magnetic field. induced current in the coil.
Statemelll - 2 Statement - 2
An emf can be induced by changing the magnetic The presence of large magnetic flux through a
field. coil maintain a current in the coil of the circuit is
3. Statement- 1 continuous.
Figure shows a closed conducting loop and two 8. Statement - 1
· axes of rotation. The magnetic fields is acted An artificial satellite with .a metal surface is
towards right. The emf in the loop will induce moving above the earth in a circular orbit. A
when the loop is rotated about axis-I current will be induced in satellite if the plane of
f Axis-2
the orbit is inclined to the plane of the equator.
Statement - 2
The induced emf across the ends of the moving
conductor is given by e = Bvf. sine:
;..-+--B 9. Statement- 1
---1--~-•············E) Axis-! Figure shows an emf e induced in a coil. It
happens due to rightward decreasing current.
e-
~
Statement - 2 Statement - 2
The emf in the loop will induce when the loop is . di
rotated about the axis-2. In the coil.selfinduced emf
. e = -L-d
t.
4. Statement - 1
Lenz's law violates the principle of conservation 10. Statement - ·1
Figure shows a metallic conductor moving in
of energy.
magnetic field. The induced emf across its ends
Statement - 2
Induced current always opposes the change in is zero.
magnetic flux responsible for its production. ------B
5. Statemelll - 1
An aeroplane flies along the meridian, the
potential at the ends ofits wings will be non zero. · Statement - 2
Statement - 2 The induced emf across the ends ofa conductor
Whenever there is change in magnetic flux emf is given bye= Bvf.sin0.
induces.
11. Statement - 1 17, Statement - 1
In the phenomenon of mutual induction, self An inductor is connected to a battery through a
induction of each of the coil persists. · switch. The emf induced in the inductor is much
Statement - 2 larger when the switch is opened as compared to
Self induction arises due to change in current in the emf induced when the switch is closed.
the coil itself. In mutual induction current Statement- 2
changes in both the individual coil.
12. Statement - 1 L di
A transformer cannot work in de supply.
Induced emf in an inductor, e = -di .
Statement - 2 18. Statement - 1
There is no change in flux due to de. Figure shows a horizontal solenoid connected
13. Statement - 1 to a battery and a switch. A copper ring is placed
Eddy currents are produced in any metallic on a smooth surface, the axis of the ring being
conductor when magnetic flux is changed around horizontal. As the switch is closed, the ring will
it move away from the solenoid.
Statement - 2
Electric potential determines the flow of charge.
14. Statement - 1
The quantity LIR possesses dimensions of time.
,/
Statement - 2
To reduce the rate ofincreases of current through
a solenoid should increase the time constant Statement - 2

(~)- Induced emf in the ring, e = - ~; .


15. Statement- 1 19. Statement- I
Acceleration of a magnet falling through a long Consider the situation shown in figure. When
solenoid decreases. switch is closed, a clockwise current will flow in
Statement - 2 the loop.
The induced current produced in a circuit always
flow in such direction that it opposes the change
to the cause that produced it.
16. Statement - 1
Figure shows the electric field E in a charged
Statement - 2
parallel plate capacitor. The electric field E can Direction of the current in the loop is according
not drop abruptly to zero (as it suggested at point to the Fleming's left hand rule.
P), as one moves perpendicular to the field, say, 20. Statement - 1
along the horizontal arrow in the figure. Fringing A flat, railroad car with a wooden base moves to
of the field lines always occurs in actual the right with a constant velocity.A voltmeter V1
capacitors, which means that E approaches zero is connected across the rails of the car and
in a continuous and gradual way another voltmeter V2 is connected across the axle
+Q of the car and in the car. The reading of V1 is V
volt.

• ....... ·
-Q Statement - 2
Statement O 2
The reading of V2 will be 2Vvolt.
J - ._ dq,8
e= rE•dl= - - - .
. dt
~m,
~-
m ~~,·11•~~;·~~~~~
BfE€1™,'f~~(;NETI~ ~ , ._,- {~ - ~~~.~:f:~f:'Jir~~""
.
,
af':\sffl'
--~,,::,-s
lfr('g~rpii'p·',~'t';<,- .:.d ·' ·. t ' ! ~ f 1
,.~:ik.:~:~~~~~L.~~::-..:,~~~---~ :.·-: ;:r~· v: -, ~
- • ' : ,,,

,•• J
,· • •' I •

I 1 I (bl I 12 I..(cJ I ·23 :!" (dJ r--34"-TJ"-r-45· I ,l (aJ I 56 1 (aJ


1 2 fTci> I :13. -.I ,(aL rJ.f; I (bl f-35 ..i (bl I 46 . I (bl I 51 I (cJ
j 3 \ (c) I 14 I (d). \ _25 \ (c) iJS- - \ (Ii) \ 47 1· (b) \ 58 l(a)
.J 4.. I (cl I 15 (dl I 26 I
1 (dJ i : (bl · 48 · (dJ
31 I I I .59 ildJ
I 5 '(" (b) I 16 r (d) • ( · 27· J (d) [ 38. : (d) ( 49 . j (a) I _60 - -l(b)
-I s. J (dl I 17 .. I .(bl. ·1 2~ 1. (bl i 39 l i~l l 5o ., I (bl -1 61. · rm
i 7 ! (a) '! 18 ·I' (a) I -29 · j (a) I 40 [ (d) j 51 [ (d) ·I 62 l(a)
I 8 I (a) I. 19 I (c) I 30 I (c) I 41 i' (b) 152--:-, 1 (b) 'I 63. 1--:(c)
1 9 ( (c) ! 20 /7aT 'I 31 i (d) ,--42 I (d). J 53_ / (b) i 64 iTc) -
! 10 \ (a) j 21 j (d) ) 32 j (d) J 43 i (a) , i, 54 j (c) J 65 [{a)
l 11 j (c) j 22 j (d) I 33 I (d) I 44 I ·(d) !, 55
., .. ' I (c) ·. l · : ··1

,.I.
~ ,, ' ,>¥ • •
. •
' 'I ): I
·, ,.-.
I 1
I (a, b)
I 6 1 · ·- ,(b,, d~: I _· 11 I ' (b) -
'' ' !

I 2 'I (b, C, d) I 7
I (d)
I · 12 , . L . (b r
I 3 I (b,. c,.d)
I ·8
I. Jc> "· I -13 . I . (a)
"'

.. 1 .. _4, · , . .,I . (c, __d) 1 ·: '"9 I (a> · .I . ~1 .J (c)


I 5 , J (a, c, d) · / 10
I . (a> .. - I 15 ·· _ I ~a) ,,
,,,

t- •,, '(' 1' l ' f ', . ' •• - I


16. · '(A),p;(B)-q,s;(C)-q,-s;(D)~q,r,s ,, , .. •· - - __ -,. ,.. . ,
.. , ,., ~-,,_l -, •, . . . . ,.. • ' • •: • • •

. , • • :W::,~""'1r<=....,,=r, ' ,,. I·,,

-a~--11\-~4~12~.:~::-;.,~-&~
---- .
-g,3
~- . . . -~'"------
.
-
' '

I 1 I I (c) I 11 . I
(c} I 6
'
(a>- •I 16 I (a)
I 2 I (b) I _( C )· I ·12 I I 1· (a) I 17 I (a)
I 3 I ( d >_ ·8 - I (a) I . 13- I 1- (b) I --18 I (a)
:1 4 j_(d).,j 9: ·.J.,'(a)·· ~ 4 I (b) I 19 , I · (.c >
I 5· I (a> I 10· I (a) 1. _15 I (a) I 20 -I ( c)
- . , .. ·-·.'
. & ~ F.4
I. A small loop ofareaA is inside of, and has its axis in the.same direction as, a long solenoid ofn turns per
unit length and currenti. If i = i0 sinOJI, find the emf induced in the loop. Ans: e = µ 0 ni0 Aro cosrol.
2. A uniform magnetic field B exists in a direction perpendicular to the plane of a square frame made of
copper wire. The wire has a diameter of2 mm and a total length of 40 cm. The magnetic field changes
dB T ·
with time at a steady rate -d = 0.02 - . Find the current induced in the frame. Resistivity of copper
I s ·
= I.7x Io-" W-m. Ans:9.3xl0-2 A.
3. A conducting circular loop having a radius of 5.0 cm, is placed perpendicular to a magnetic field of0.50
T. It.is removed from the field in 0.50 s. Find the average emf produced in the loop during this time.
· Ans:7.8xl0-3 Y.
4. The magnetic flux through the loop shown in figure increases according to the relation

= 1.0 I + 6.0 12 , where q,8 is in mill iweber and t in second.


q, 8
(a)· What is the magnitude of emf induced in the loop when t =·2.0 s ?
(b) What is the direction of the current through R?
. '
B
• • • • • •
- . • • • • •
,. ')

• ' • • • • •
, ·1 ' · •• '.1 .• .
· • •
• • • •
..
• • • • • •
/
/

• ... • R
•. • •
- 1 Ans: (a) 3 I mV (b) rightto left.
5. (a) The magnetic field in a region varies asshown in figure. Calculate the average induced emf in a
conducting loop ofarea 2.0x 10-3 m2 placed perpendicular to the field in each of the IO ms intervals
shown. · ·
(b) In which intervals is the emfnot constant? Neglect the behaviour near the ends of IO ms intervals ..

B(T)

0.03 --------··· i-----------;-----------, -......... i

0.02 · --------:------- ·{ -------; -····----·:


0.0 I_ ·········- ' ··········1·--··.. ··· . ··········~

. ·'.. ",' .,
IL~.-'-10-.--";'-0--3.l..~-~;o- t(ms~ ., ,. .._.,
I "! l. J

Ans:(a)-2.0 mV,-4.0 mV,4.0mV,2.0mY;(b) 10msto20msand20mstci30ms.


6. In figure let the flux through the loop be •"8(0) at time t=O. Then let the magnetic field B vary in
continuous but"unspecified way, in both magnitude and direction, so that at time I the flux is represented
by <!>it).
IJ
• • • • • •
• • • • • •
• • • • • •
• • • •
• • • •
• • • • • •
• • • R• • •

(a} Show that the net charge q (t) that has passed through resistor R in time I is
I , - . .
q(t) =R [<l>e (0)-<l>e (1)] and is independent of the way B has changed,
(b) If <!>s(t) = <j> 8 (0)
in a particular case, we have q(t) = 0. Is the induced current necessarily zero
throughout the interval from O to t? Ans: (b) no.
7. A conducting loop of area 5.0 cm2 is placed in a magnetic field which varies sinusoidally with time as
B= B0 sinOJtwhereB0 = 0.20 T and OF300 s- 1• The normal to the coil makes an angle of60° with the field.

Find (a) the maximum emf induced in the coil, (b) the emfinduced at I= (~)s
900
and (c) the emfinduced

at/=( )s.
6~0 Ans: (a) 0.015 V (b) 7.SxJ0--3 V (c)zero.

8. Figure shows a horizontal magnetic field which is uniform above the dotted line and· is zero below it. A
long, rectangular, conducting loop of width L,-and mass m and resistance R is placed partly above and
partly below the dotted line with·the lower edge parallel to it. With what velocity should it be pushed
\fownwards so that it may continue to fall without any acceleration ?
X X 'X X X

X X X X X

X X X X X

-~ ... ... -~- .. ~--···


~

L
. r
I' . .Ans: V= mgR
' ~~ ;
9. 'in.air plane, with a 20 m wing spread is plying at 250 mis straight south parallel to e;rth's surface. The
earth's magnetic field.has a horizontal component of2x10-5 Wb/m2 and angle of dip is 60°. .· .: ·. .-
Calculate the induced emf between the plane tips. . · Ans: 0.173 V.
-..
10. Two concentric coplanar circular loops made of wire, with resistance per unit length JO Q/m have
diameters 0.2 m and 2 m. A time varying potential difference (4 + 2.51) volt is· applied to the larger loop.
Calculate the current in the smaller loop. Ans: 1.25 A
I I. a
Figure shows square loop of side 5 cm being moved towards right at a constant speed of I cm/s. The
front edge enters the 20 cm wide magnetic field alt= 0. Find the emf induced in the loop at (a) t= 2s, (b)
t= 10 ,, (c) t= 22 sand (d)t= 30 s.
X X X

B=0.6T
X X X

5cm~
X X X

< X X
i--20 cm-i
Ans:(a) 3xJo-4V, (b) zero, (c) 3xJo-4Vand (d) zero.
12. Figure shows a circular wheel of radius I 0.0 cm whose upper half, shown dark in the figure, is made of
iron and the lower halfof wood. The two junctions are joined by an iron rod. A uniform magnetic field B
of magnitude 2.00x I 0-4 T exists in the space above the central line as suggested by the figure. The
wheel is set into pure rolling on the horizontal surface. Ifit takes 2.00 second for the iron part to come
down and the wooden part to go up, find the average emf induced during this period.

,e
X X X X X

X X

X X

Wood
. Ans: J.57xJ0-6V.
13. A pair of parallel horizontal conducting rails of negligible resistance shorted at one end is fixed on a
table. The distance between the rails is L. A conducti~g massless rod of resistance R can slide on the
rails frictionlessly. The rod is tied to a massless string-passes over a pulley fixed to the edge of the table.
A mass m tied to the other end of the string hangs vertically. A constant magnetic field B exists
perpendicularto the table. If the system is released from rest, calculate
(a) the terminal velocity achieved by the rod and
(b) the acceleration of the mass at the instant when the velocity of the rod is half the terminal velocity.
mgR
Ans: (a) Vr = - - (b) g/2.
2
B L2

14. A current ; = 3.36(1 + 21) x 10-2 A increases at a steady rate in a long straight wire. A small circular.loop
of radius I o-3 m has its plane parallel to the wire and is placed at a distance of I m from the wire. The
resistance of the loop is 8.4 x J o-4 Q. Find the magnitude and the direction of the induced current in
the loop. Ans: 5.0x I 0- 11 , anticlockwise.
15. The rectangular loop is figure, of area A and resistance R, rotates at uniform?ngularvelocity roabout the
y-axis. The loop lies in a uniform magnetic field B in the direction of the x-axis. Sketch the following
graph:
(a) •the flux <I> through the loop as function of time (let t = 0 in the position shown);
(b) the induced emf in the loop.
z

e
<I>
+BA '!Uc,>

f-~-+-~--r+I

-BAw

Ans: (a) <I> =BA cosrot (b) e = BAcooinrot


16. A flexible circular loop 10 cm in diameter lies in a magnetic field 1.2 T, directed into the plane of the
diagram in figure. The loop is pulled at the points indicated by the arrows, forming a loop to zero area in
0.2s.
(a) Find the average induced emf in the circuit.
(b) What is the direction of the current in R?
r·:
: -!
x X X X

a
X X
Jxi X

i : R
X X
!\
! :
X
b Ans :(a) 0.0471 V (b) Fromatob.
X X
: ·: X .,
;
• • ...l
17. The cube is figure, I m on a side, is in uniform magnetic ffeld of0.2 T, directed along the positive y-axis.
Wires A, C, and D move in the direction indicated, each with a speed of0.5 mis.
(a) What is the motional emfin each wire?
(b) What is the potential difference between the terminals of each?
z

Ans:(a)0,0.0707,0.141 V(b) 0,0.0707,0.141 V


y

X
18. The magnetic field within a long, straight sqlenoid of circular cross s_ectioh'and.radius R is increasing at
. . . '.1 :1•
dB
a rate dt.
(a) What is the rate of change offlilx through a bircle of radius r 1 inside the solenoid, normal to the axis
of the solenoid, and with centre of the solenoid,? ' ., · _ · · .'
(b) Find the induced electric field En inside_ the_soJenoid, at a distance r 1 from its axis. Show that the
direction of this field in a diagram.
(c) What is the induced electric field outside the solenoid at a distance r 2 fro~1 th~ axis?
R
(d) What is.the induced emf in a circular tum of radius - ?·
-2
·(e) of radius R?
(I) ofradius 2 R ?

(dB) (1.) (dB)


·dt _ R2(dB) - -1tR2(dB) 2(dB) z(dB)
Ans:(a)lt'J
2
dt (b) 2
(c) - -
_2r2 -dt _4 -dt
(d) (e) 1tR -
dt (I) 1tR -dt
19. The magnetic field 11 at all points within the shaded circle of figure is 0c5 T. It is clirected into the plane
of the diagram and is decreasing at the rate ofO. l T/s.
X X

X X

(a) What is the shape of the field lines of the induced En-field within the shad_ed circle?
(h) What are the magnitude and direction of this field at any point of the circular conducting ring of
radius 10 cm, and what is the emf in the ring?
(c) What is the current in the ring, if its resistance is 2 Q ?
(d) What is the potential difference between points a and b of the ring?
Ans :(a) Circles, clockwise (b) 0.005 V/m,3.14 mV (c) 1.57 mA (d) zero
20. A square conducting loop, 20.cm on a side, is placed in the magnetic field 0.5 T, directed into the plane
of the diagram and decreasing at the rate of0. l T/s.

X X X

X_XUQ
X X XX
X t _ X X
20C X
X I X X
-20cm- ·
xxxxxx
. X · X X

(a) Prove that the component of En. along the loop has the same value afevery point ofth_e loop and_
is equal to that at the ring.of previous problem.
(b) What is.the~~rrent in the loop, if its resistance is 20?
(c) What is the p~tential difference between points a and b?
Ans: (b) 2.00mA (c) zero
21. .A square conducting loop, 20 cm on a side, is placed in the magnetic field 0.5 T, directed into the.plane
of the diagnim with the side ac along the diameter and bat the centre of the field and decreasing at the
rate ofO. I T/s.
(a) What is the induced emf in side ac?
(b) What is the induced emf in the loop?
(c) What is the current in the loop, if its resistance is 2 Q ?
(d) What is the potential difference between points a and c? Which is the higher potential·?
(e) Find the induced field En in each side of the square loop.
(1) Find the induced emf in each side
(g) Find the electrostatic field Ee in each side.
(h) Find the potential differences Vue' Vee' Veg' and Vga· What should be the sum of these potential
differences? ~,--,.-,,- .
X !

:,f~ ~ ~
X X l-:20 ---t<·
j cm
X i X
Ans: (a) zero (b)4 mV (c)2 mA (d)zero (e) 5.00 mV/m (1) l.OOmV (g)zero(h) zero
22. A wire is bent into three circular segments, each of radius r = IO cm, as shown in figure. Each segment
is a quadrant ofa circle, ab lying in thexy-plane, be lying in theyz-plane, and ca lying in thezx-plane. (a)
If a uniform magnetic field ii points in the positive x-direction, what is the magnitude of the emf
developed in the wire whenB increases at the rate of5.0 mT/s? (b) What is the direction of the current
in segment be ?
z

X
Ans: (a) 39.25 µV, (b) fromcto b.
23. Considerthe situation shown in figure. The wires P 1Q1 and P2 Q2 are made to slide on the rails with the
same speed 5 cm/s. Find the electric current in the 19 n resistor if:
(a) both the wires move towards right and
(b) if P 1Q 1 moves towards left but P2Q2 moves towards right
P1 P,

T
4cm
• X

2Q
X

-20
X X X
.
190

l
X X X X X
Ans: (a) 0.1 mA, (b) zero.

Q, Q2 B= LOT
24. In figure a 120-turn coil of radius 1.8 cm and resistance 5.3 Q is placed outside a solenoid. The current
in the solenoid is 1.5 A and it reduces to zero at a steady rate in 25 ms. What current appears in the
coil? The number of turns per unit length of the solenoid is 220 turns/cm and its diameter [):a 3.2 cm.
Coil

Solenoid
~--;- __ ..
Ans:30mA
25. A rectangular coil of N turns and oflength a and width b is rotated at frequency fin a uniform magnetic
field B, as indicated in figure. The coil is connected to co-rotating cylinders, against which metal
brushes slide to make contact. ·
Sliding contacts
B X X X X X X

T
..!&-b
X X X X X

R
lx X X X X X

. xi X X
a
X
X I X

(a) Show that the emf induced in the coil is given (as a function of time I) by e = ea sin {2rcji),
where ea = 2rc/ NabB. This is the principle of the commercial alternating-current generator.
(b) Design a loop that will produce an emf with ea= 150 V when rotated at 60.0 rev/s in a uniform

magnetic field of0.500 T. Ans: (b) Nab = (:re) 2


m
26. A wire loop enclosing a semi-circle of radius a is located on the boundary ofa uniform magnetic field of
induction B (figure).At the moment!= 0 the loop is set into rotation with a constant angular acceleration
a about an axis O coinciding with a line of vector iJ on the boundary. Find the emf induced in the loop
as a function of time t. Draw the approximate plot of this function. The arrow in the figure shows the emf
direction taken to be positive.
\

0
B
X X

X- X

I . 2
Ans: e; = -(-1)" Ba w, wheren =I, 2, ....... is the numberofthe half-revolution thatthe loop performs
' 2

,
at the given moment I. The plot e;(t) is shown in figure where In =
~2rcn
a .
-···

·····

27. Consider the situation shown in 'ii11ur~. The wire PQ has a negligible resistance and is made to slide on-
the three rails with a constant speed of 5 cm/s. Find the current in the IO n resistor when the switch S
is thrown to (a) the middle rail (b)the bottom rail.

T
2 cm
X

X ,.
X

X
X

X
px
~

X
X

X
..
10n

+
2cm
j_
X X X X X
s
V B= I.OT
' , X X Qx X

Ans: (a)O. I mA, (b) 0.2 mA.


28. In figure, the square loop of wire has sides oflength 2.0 cm. A magnetic field is directed out of the page;
its magnitude is given by B = 4.0 t 2y,·where Bis in tesla, tin second, and yin metre. Determine the emf
around the square at I= 2.5 s and give its direction.
y

., ~--


• • •







• • • •
B
• • • •
---"-----+X
• • • • •
Ans.: 80 µV, clockwise.
29. The current in a coil of self-induction 2.0 henry is increasing according to i = 2 sin fl ampere. Find the
amount of energy spent during.the period when the current changes from Oto 2 ampere. Ans :4 J
30. A very small circular loop of area 5x I0-4 m2 , resistance 2 ohm and negligible inductance is initially
coplanar and concentric with ,a much larger fixed circular loop of radius 0.1 m. A constant current of I
ampere is passed in the bigger loop and the smaller loop is rotated with angular velocity OJ rad/s about
a diameter. Calculate (i) the flux linked with the smaller loop, (ii) induced emf and (iii) induced current in
the small loop, as a function of time.
Ans: (i)_nx Io-9 cos rot (ii) nx 10-9 rosin rot V (iii) l.57x I a-" rosinox ampere.
3 I. Two long parallel wires of zero resistance are connected to each other by a battery of 1.0 V. The
separation between the wires is 0.5 m. A metallic bar, which is perpendicularto the wires.and of resistance
IO Q , moves on these wires when a magnetic field of 0.02 T is acting perpendicular to the plane
containing the bar and the wires. Find the steady state velocity of the bar. If the mass of bar is o'.002 kg,
find its velocity as a function of time Ans: V = 100(1-e-o.oos,)
'•
32, ... At a giy~n [nstant the current and self-i_nduced en:ifin an in,ductor are,di~ecteq,as indic?ted in figure ..
(a) Is , t~e current
,, '
increasing
,
or decreasing ? ,
• • ~ •
. .• .I • , . • . .
,,.
(b) The induced emf is 25 Vand the rate of change of the current is 25 kA/s; find ~he indµctance.
: .-·- ·, l;L . . , C
,.... ,f ___.. .i
0000 ., •
Ans: (a)
The current in the inductor decreasing.
, '. · •· (b) I mH.
33. The current in an RL circuits drops from 1.0 A to IO mA in the first second following removal of the
battery from the circuit. If Lis IO H, find the resistance R in the circuit. A11s: 46 Q
34. A coil of inductance I Hand resistance 10 Q is connected to a resistanceless battery of emf50 Vat time
,,1 '
I= 0. Calculate the ratio of the rate at which magnetic energy is stored in the coil to the rate at which
1
energy is supplied by the battery at t = 0.1 s. ' •• Ans: 0.36
35. The current (in ampere) in an inductor is given by/= 5 + I 61, where tis in second. The self-induced emf
initis IOmV.Find
(a) the self-inductance and· .
(b) the energy st~red in the, inductor and the power supplied to it at I = Is.
Ans:(a)6.25x Jo-4H(b)210mW.
36. A solenoid having an inductance of 6.30 µH is connected in_ series with a 1.20 k Q resistor.
. (a) If a 14.0 V battery is switched across the pair, how long will it take for the current through the
··<
resistorto reach 80.0 % of its.final value ? .
. (b) What is the current through the resistor at time.I;,, 1.0 <?
A11s: (a) 8.45 ns (b) 7.37 mA.
37. In figure!;= 100 V,R 1= 10.0 Q, R2 = 20.0 Q, R3 = 30.0 Q andl=2.00 H. Find the values ofi 1 and i2
1.

·'
/,

(a) immediately after the closing of switch S,


(b) a long time later, (c). immediately after reo!)ening of switch S, and (d) a long time afterthe reopening.
Ans: (a) i1 = ;2 = 3.33 A; (b) i1 = 4.55 A, i2 = 2.73
.
A; (c)-i1 = O, i2 = 1.82 (reversed); (d) i1 = i2 = 0.
'
38. In the circuit shown in figure, switch Scis closed at time t='O. Thereafter, the constant current source, by
varying its emf, maintains a constant current i out of its upper terminal. (a) Derive an expression for the
·current through the inductor as a function of time (b) Show 'thai the current through the resistor equals
,,
;he current through ;he inductor at time I=_( ~f, n2. "'·

s
Constant
current·
source
R L ·(i
Ans:(a)• -e-Rt/L)
.
.' ·~·
t); ."/~
\.
I (,,;.
g:' /l:'it¥<::TRICI:iY1&i~GNffi.~M{Jt~ 1::'l~'."\•·- 1

39. An inductorofinductanceL=400 mH and resistors ofresistances R1 = 2 Q and ·R2 = 2 Q areconnectel


to a battery of emf~= 12 Vas shown in the figure. The internal resistance of the ~attery is negligible. The
switch Sis closed at I= 0. ·
What is the potential drop across L as a function of time ? After the steady state is reached, the switch
is opened. What is the direction.and the magnitude of current through R1 as a function of time?

L
-
Ans: Yi=~ .-R2t/L, i= 6 .-10,

40. In a circuit shown A and Bare two cells of same emf~ but different internal resistances r 1 and r2 (r 1> r2)
respectively, Find the value of R such that the potential difference across·the terminals of cell A is zero
a long time after the key K is closed.
R
R

R.
I; I;
- j-------1
-' r1 ,.,
R . 4 .
R C
Ans: R=-(1j-r2)
. 3
R

s
41. A circuit containing a two position' switch Sis shown in figure.
(a) The switch Sis in position-,b,Find the potential difference VA- VB ·and the rate of production in
jouleheatinR 1• • • · -
(b) If now the switch Sis put in position 2 at/= 0, find
(i) Steady current in R4 and _ _
(ii) the time when the current R4 is half the steady value, Also calculate the energy stored in the
inductor L at that time.
C:=2µF
R.=2Q
, ·

R1 =2Q

R,=30
L= IOmH
I
Ans: (a) VA- VB = -SV,
(b). (i) i0 e=0.6 A,
· (ii) J.386X IQ-JS,
4.SxI0-4 J,
.ELECTROMAGNETIC .INDUCTION

42. An inductor of inductance 2.0 mH is connected across a charged capacitor of capacitance 5.0 µF and the
resulting LC circuit is set oscillating at its natural frequency. LerQ denote the instantaneous charge on
the capacitor and I the current in the circuit. It is found that the ·maximum value of charge Q is 200 µC.

(a) WhenQ= IOOµC,whatisthevalueof/:1?

(b) When Q = 200 µC, what is the value of I?


(c) Find the maximum value of/.
(d) When/ is equal to one-half its maximum value, what is the value of IQ I?
Ans :(a) 10 4 Ns
(b) /=O
(c)/max=2A
(d) Q= 173.2 µC.
43. A metal rod OA of mass m and length r is kept rotating with a constant angular speed co in a vertical plane
about a horizontal axis at the end 0. The free and A is arranged to slide without friction along a fixed
conducting circular ring in the same plane as that of rotation. A unifonn and constant magnetic induction
Bis applied perpendicular and into plane of rotation as shown in figure. An inductor Land an external
resistance R are connected through a switch S between the point O and the point Con the ring so as to
form an electrical circuit. Neglect the resistance of the ring and the rod. Initially the switch is open.
(a) What is the induced emf across the terminals of the switch ?
(b) The switch Sis closed at time I= 0.
(i) Obtain an expression for the current as a function of time.
(ii) In the steady state obtain the time-dependence of the tor,que required to maintain the constant
angular speed, given that the rod OA was initially along the positive x-axis at time I =0.

Ans :(a)
Bcor2 x ;"1 x x B
2

(b) (i) i= s;; (1-e-R,tL)


2

R r_;
B2cor 4 mgr X X X
(ii) ~=---+--cosOll.
4R ·2
44. Figure shows, in cross-section, two coaxial solenoids. Show that the mutual inductance M for a length
f of this solenoid-solenoid combination is given by M =1tR?tµ 0n1n2 in which n 1 and n2 are the
respective numbers of turns per unit length and R I is the radius of the inner solenoid. Why does M
depend on R I and not on R2 ?

Solenoid 2
----~--~--,.,,·-:-,.,.""·--,,--,,-~~-,
ELECTRICI'IY &)'fAGNETI_S_M_._:_'._.-~-_,__-.'"~,,--'-,__':~;_ ":,,·; .:/··... j -
45. A rectangular loop of N close-packed turns is positioned near a long straight wire as shown in figure.
What is the mutual inductance M for the loop-wire combination ?

- i

µo
Ans:--· Ne ( b)
en -l+-
2,c a

46. Figure shows a coil of N2 turns wound as shown around part ofa toroid of N 1 turns. The toroid's inner
radius isa, its outer radius is b, and its height ish. Show that the mutual inductance Mforthe toroid-coil
. . . µoN 1N?h b
com bmat1on 1s M =----- en-.
2,c a

47. A transformeron a utility pole operate at Vp= 8.5 kV on the primary side and supplies electrical energy
to a numberofnearby houres at Vs= 120 V, both quantities being rms values. Assume an ideal step-down
transformer, a purely resistive load, and a power factor of unity.
(a} What is the turns ratio N,JNs of the transformer?
(b) The average rate of consumption in the houres served by the transformer is 78 kW. What are the
rms currents in the primary and secondary of the transformer? ·
(c) What is the resistive load Rs in the secondary circuit? What is. the corresponding relative load Rp
in the primary?
Ans:(a)= 7l(b)9.2A,650A(c)Rp=926Q R,=0.18 Q.
)
CHAPTER
0
0
o.
0 /""''
I_
'o

:;_" ;.J·...
_::_·_:_~
,, ·-

9.1 ALTERNATING CURRENT (AC)


Definition : The current which we use commonly in our houses is alternating current. Alternating current
or potential is one whose direction changes periodically, often in a sinusoidal manner. Figure shows two
alternating currents; one sinusoidal and other square/rectangular.

Jof---~
+
T
1---.,,T-rl--~::--+ I
112
-i ____ . __ .,_____,
IT
'CJ "'
.'
Sinusoidal Rectangular
(a) (b)
Fig. 9.1

Skin effect : A direct c·urrent flows uniformly thro~ghout the cross-section of the conductor, while an
alternating"current, flows mainly along the surface of the conductor. This is know as skin effect.

Flow of DC through Flow of AC through


the conductor the conductor
(a) (b)
Fig. 9.2
By increasing surface area, we can decrease the resistance for ac. It can be possible by maki~g a cable which
made of number of thin wires having same amount of metal as in the case of a single wire cable.

A cable made of thin wires


(c)

Fig. 9.2
Why alternating current used in practice?
I. AC potential can very easily be changed by using transformer according to the requirement.
2. We can minimise transmission losses (i2R) by providing cable made ofa number of thin wires.
3. AC is more readily adaptable in rotating machinery such as generators and motors.
Alternating current and potential : Figure 9.3 (a)_ and (b) represent alternating current and·· .
potential respectively. They can be represently by the equations i = i 0 sin rot and V =, V, sin rot.

,~
eor V

0 r--,---=-=:-:--=--1
T/4 T/2 3T/4 T
-io ------------- .:...jo --------

(a) i = i,,sin o,/ (b) V= V, sin (I)/ (c) i =i, cos (I)/,

where i 0 is peak value of current where V0 is the peak of ppteritial.


OJ is angular frequency in rad/s. Fig. 9.3
The circuit symbol for an ac source is 0-
Phasor diagram
Analysis of alternating current circuit is facilitated by 'the use of vector diagrams
similar to those used in the study of SHM. In such diagrams, the instantaneous
value ofa quantity that varies sinusoidally with time is represented by the projection
onto a horizontal orvertical axis of a vector oflength corresponding to the amplitude
of the quantity and rotating anticlockwise with angular velocity w. In the cont.ext
of ac-circuit analysis, these rotating vectors are often c:::alled phasors, and diagram
containing them are called phasor diagrams.
From the diagram, the projection on x-axis diameter gives
V V0 cos rot,
and projection on y-axis gives V = V0 sin rot. Fig. 9.4
9.2 AVERAGE AND ROOT MEAN SQUARE (RMS) VALUES -
Average value :
The average value xav of any quantity x(t) that varies with time over a time interval 11 to 12, is defined as
- 12

Jx(t)dt
,, I ,, .
,, I (12- 11) Jx(t)dt
Jdt
. ,,
,,
Let us now calculate average value of sinusoidally varying quantity, i = i 0 sinOJI.
(i) Average over half cycle :
T/2
i0 J sin OJI dt
0

(f)
T

Fig. 9.5

::rr]2
2; {2;) 0

io [-cos1t-(-coso)]
It
2i0
It
· (ii) Average over complete cycle :
d
. TJsmrot
.
t
'o T -
0

2
- i0 Jsin( 1t1)d1
T0 T .
T
27ll .
-cos-
io · T
T
(~) 0

,!Q__ [-cos21t-(-cosO)]
21t
0
(iii) Average ofhalfrectitied wave.:, ...
',• . . _,.
. -~-· -
T
F, '._,,_
' '
I 2
- Jsinrotdt
To o,---~-~-~-~-
n2 T 3'/'l2
Fig. 9.6
(iv) Average offull rectified wave: .'
I T ,
iav =--· T J~m (I)/ dt
0
T/2 T
_2;0
Fig. 9.7
It
RMS value:
I. Mathematically nns ofnumerous values i 1, i2 , .... , in is defined as
.2
11
+1.22 +.......... +,,,.2
n T
-io -------------
Fig. 9.8
For a quantity changes continuousl)', we can write

J ;2 dt fi2dt
T~
1
_o__ = _o__
• rms T T
Jd,
0
2. RMS value of i = i0 sinmi':·we can write
T
J;2 dt
.2 ._o__
1rms
T '"
.• .2 T
_ ·!Q_ Jsin 2 lit dt
- T
0

.it H1-c;2rol ]dr

;J
- 2x2m]dI
Jr[! -cos---
·2T T
0

T
. 47tl
.2 srn-
.!Q_I ___ T_
= 2T 41t
T o
-2 . • . .
..'Q:..[(r-o)-o]
, :-: l ' I '
2T f' ·~ •t'I!, i~ T:• ,:ir'• ,
-2
=: _!Q_
. • }, ',l, .r- : t~" ,!
f• t; .,· ''
= .'-.2'·
~ ~0.707 ;0
- '
....(1)

- - - ·, -v. -- . - - .
Similarly V =. _Q_ = 0.707 Vo .,.. (2) ,-
. !"::.. ,·•: l. :: .,,. -. 7·. ··_Fi· ,, · · ·=: .,A.'

Remember
I T I T· . ·-a-- .
I. -JsinOJldl=O -' J cos OJI di =0
To To
T
I T 2 · I
2 - IJ.2
SID OJI dl=-
I - J cos OJI di=-
·T
O
2. r
O... 2

Where average and RMS values are used ?


The usual moving-coil galvanometer, however, has very large moment of inertia to follow the instantaneous
values of current. It average out the fluctuating tor_que on its coil, and its deflection is proportional to the
~ver;:tge current.·
RMS values of voltage and curre.nt are useful 'in finding power or energy loss in accircuits.
Most ac meters are calibrated to read not the maximum value of the current or voltage, but the rms value of
current or voltage.
rms value
Form factor ( R1 )
average value ·

Ex. 1 An alternating current is given by the equation i=it sin rot+i2 cos rot. Find its rmsvalue.

Sol i2 ·(;1 sinOJl+i2 cosOJ1) 2 •

· i( sin 2 OJi+.;f cos2 OJl+2i1 i2 sinOJtcosl:ol


or ;2 2
;12 sin OJI + ;2 cos2 OJI + i1;2 sin 20JI
2
-•·--·
_.·,t -, : •• ,• ~ .-
--- ... -- -- --- ---- - ---.--,--
-:i :··-~ :.: ·.·ij •• -
and

.Ans.
In our country the power supply for domestic purpose is 220 V and 50 Hz. That is,
Vrms = 220 V,or V0=.J2 x220 = 31 IV
and/= 50 Hz, or ro= 2it x 50 = I 00 lt rad
V V0 sinOY=311 sin IOOltl.

I
Ex. 2· The current in a discharging LR circuit is given by ; = ;0e t where'tis the time constant of the
circuit. Calculate therms current for the period t= 0 tot= 't.
I
Sol. Given,·

.2 t 2,
I -
and JLJe<dt
't 0

t
2,
; 2 e t

~(-¾t
.2
to (-'t ) [ e-2 -eOJ
= ~x 2

= - ;a2 [,--']
2 ·e2

Ans.

Ex. 3 Find the average and effective values of the saw-tooth·wave form as shown injig. 9.9.
V

50 ............................ ··············

Fig. 9.9
Sol. Since voltage increases linearly, so

= 0+50=25V
vav 2
Let Vis the instantaneous value of voltage between O to 2s, then
50
V = - I =251
2
The nns or effective value of the voltage is

-625 f 2d
2
I I
2 0

32
625
!_ = 833.3 V
2 3
0
V,,ns = 28.9V Ans.

Ex.4 Determine therms value of a semicircular wave which has a maximum value / 0•
Sol. The equation of a semicircular wave is
x2+y2
or y2 y

-2
'rms

/o L.---~----~-+ X

2 lo f(
-lo
2
10 -x2)J}o Fig. 9.10 +io

3 lo
I 2 X
- 10 x--
210 3
-lo

- I [( 103 --1-
1{\ ( -103 + - Il )~
210 3 j 3

21/
3

or :..)¾·
- -lo
3
Ans.
~~fk~~;;j~~~;~;~).:~~(·g,'::~i:·:·,.~~0:~~YJ ·
Circuit elements :
In circuits, we have three circuit-elements: resistor, capacitor and inductor. All these three offer resistance for
ac. The resistance of resistor is real while the resistance offers by capacitor and inductor is virtual.
Resistance :
The opposition offered by the resistor is called_ resistance. It is independent of frequency of ac.
Capacitive reactance (Xcl :
The opposition offered by the capacitor in an ac circuit is called capacitive reactance. It is a virtual resistance.
It is given by
r· f
Its unit is ohm. Xe ·= · .we = 21tfC ·
: - .· ~ -· -·-
Fordc,oo=O, Xe= co.

0)
(a) (b) (c)

Fig,·9.11
Inductive- reactance (XJ : .. ..
• • ' •• • ' I • • - '
It is the resistance offered by the inductor in aii ai: circuit. It is also a virtual resistance. Its unit is ohm. It is given
by ,·.~·
xt_~}ci>L- _
Fordc,w=O,Xl =O
Impedance (Z) : ·
The total resistance of ac circuit is c~lled impedance.
Admittance (A) :
, • , - , . I
Reciprocal of impedance is known as admittance; i.e·.,. A=-.
z It's unit is mho.

9.3 THREE SIMPLE CIRCUITS


I. A resistive circuit: Suppose that an alternating potential difference V = V0 sinOll is applied to resistance
R, then current in resistor

V. .
-V = _Q_SID 0)/
R R
or i Et._= ;0 _sin?1f,. ...(I)

where ;0 = Vo , is the amplitude of the current.


.' R
Current and voltage both .vary with sinoY, hence they are in same phase.
•1 ;\) •

R Virtual a,ds

L
,__ _ ,.:;, - - - + I
v,' f'' \::

2,r Real iLxis


'
F' = J,~1 sinro{
Phasor diagram
,., I V1 R. = V0 sinwl
iu = i 0 sinml

Fig. 9.li ·
. ' .
2. A capacitive circuit : If a capacitor of capaciiance C is connected across alternating source, the charge
· ·.,at·any'instant,-., :1 ' \. • · , .,,,, ··: - ,;; •

dq · .
.,Current - = CV.00JcosOJI
di

_Vo()sin(OJ1+~)=!:'!Lsin(.OJ1+~)
I .2 Xe · 2
OJC ,. : , -.

or' :..(2)

I
' '
where OlC .= Xe, is called capacitive reactance,

icL
· V = V0 sin~o/_
• ·;,_ !! : !_i_ t :· ·- •
.o~F'.·;{·
. ' .•
90° .
V

_- _;!_ . ?.: .,-,t


., .

Ji'
'
v v·
._ C ~-~ O sin-().)t Ve -. '
ic = i0 sin(mr ~ n/2) virtual axis
., .. \ t ._,#, I .·1' '
' \ 1
. •
, ·' -J:ig.: 9.i3
I , , '
. • , t , ~"i 1
3. An_ i~~u~tive circuit ; Suppose that_ an altem~iing potential V= Vo sjn~ is _applied ~cross· _a pure
ind~ctor of self inductance L. From the definition of inductance, we can write I
L dii_
dt

ldii
or
dt

. Vo
or -sin rot dt
'L

or

V,
= .,_Q__COSOOl+c
ool ;
The value of the constant over complete cycle will be zero, Xi.= ool, is called inductive reactance.

ii =e
,'v0
·xi sin
( 1t)
oot-2

or ...(3)

14----V--.i
/,

V= J/0 sin@r
/
J--'L = V0 sin CJ1t
i1_ = i0 sit1(ro1- 1112)

Fig, 9,/4

Phase a~d amplitude relations for alternating currents and voltage


Circuit element Symbol Impedance Phase of the current Phase angle~ Amplitude relation
Resistor R R In phase with VR
It It
Capacitor C Lead Ve by +-
2 2
It
Inductor .l
2 ' ..
9.4 RC-CIRCUIT

Consider a circuit with resistor R and capacitor C, connected in series with an alternating source of potential V.
If VR and Ve are the p.d. across resistor and capacitor respectively, then

R C

V V
C ---------------

Fig. 9.15

2
(i) Supply voltage V Jvi+Ve 2 =J(iR) +(iXeJ2

(ii) Impedance z ~ =JR2+Xe2 =JR2+(rolC J


(iii) Current i0sin(W + cj>)

Xe
-=--
where tam!>
R roCR

(iv) Peak current

9.5 LR-CIRCUIT

R L
V ,. ----------------
''
VL
VR V '' '
'''
'
':
'-----"v}----'
i
,,,'
V = V0 sinrol R

Fig. 9.16
Consider a circuit with a resistor Rand an inductor L connected in series with an alternating source of potential
V. If VR and VL are the p.d. across resistor and inductor respectively, then

(i) Supply voltage v = Jvi +v/ = J(iR)2 +(;xi) 2


(ii) Impedance z ~ =JR2 +XL2 =)R2 +(rol)2
I
/

(iii) Current ;
0
sin(OY---<j,)
, ' .' -. ~

..
.,
XL rol
;.
'
where tan<!> -=-
R R'

(iv) Peak current


,'vo Vo
io, -=
z 2
~R +X/
9.6 LC-CIRCUIT ,,, I'
. '
Consider a circuit with inductor Land capacitor C connected in series with an alternating potential V. If VL and
Vcare the p.d. across inductor and capacitor respectively, then '
I•
V
-L L
L C

VL Ve

\ tJ,, ·f----•

V= V0 sinwr JlC '


Fig. 9.17
(i) Supply voltage V =' VL-Vc=i(XL-Xc)

,V I
(ii) lmpedanc~ z a::. -:-=XL -Xe= rol--_.
I roC

(iii) Current _i = i0 sin( rot±~)

(iv) Peak current


I
rol--
roC
9.7 SERIES RLC-CIRCUIT .. . ,.. ,,'
Consider aRLC- series circuit shown in.fig. 9.18. If Vn, VL and Vcare the p.d. across the resistor; iiiductor anil
capacitor respectively, then
.,,

R-v_.
R L C L
V,.
.,
·,
-4,.._,0'-_ _ __J '''
L__ _ _
VR
~--~·---0) '
(I),.__ J'• .j• ot,:
V= V0 shiwl
VC

- , . Fig. 9.18 _,. I'


• I ---·--:.:-.
IL_ _ __
\:! ;s. ·,;,,
--· _ ..•,:,,
---··-::~,.
_____
•·! •>,•:,,· .
-":;<·t:'.-t-;;,:
________
+''l'j·--··
'" _' A'c· AND,~.:E··.ia_.:w:A·VES
. , .... ,

(i) Supply voltage V Jv/ +(VL -Ve )2=J(iR) 2+(iXL -iXc )2

(ii) Impedance• z 'V =JR2+(XL-xcJ2


I ,
=JR2+(roL--l-)2
roe

(iii) Current i0 sin(rot±q,)

I
rol-~
where tancjl roe
R

9.8 RLC, RESONANCE CIRCUIT

Resonance will occur when oscillating quantities, here i and V are in same phase, i.e.
q, 0
0 VL

I-
·Of roL
roe

or

where OliJ is called resonance frequency:· - · VC


At resonance, Zmin R. Fig. 9.19

i0 = 1'1R -----------

Fig. 9.20. Variation ofcurrenti with ro.


V
Current: = -
z
V

at ro 0, i=O

at ro

at ro 00, i'= 0
Remember:
... , H, ,_
l I,','-, Jli ' } 1
ro=co0 ,roL=- co<co0 ,roL<- ro>OJo,roL>-
roC , ,. roe roC
cj) = 0, current in phase ·'current leads potential current lags potential
with potential 'I,~ '\ '

I f. ', ' .
rol rol rol

J_
0)
I I
(0. roe roe
.'
Fig. 9.21
'.

Ex. 5 A 60 Hz voltage of230 V effective value is impressed on an inductance of0,265 H.


(i) Write the time equation for the voltage and the resulting current. Let the zero axis ofthe
voltage wave be at t=0.
(ii) Find the maximum energy stored !n the inductance.

Sol. Vo = ..fi: V.-ms = ,/2. x 230 V;


1 !) f I:• '
f = 60Hz, :.ro=21tf=21tx60=1201t
( ~-
xl -:= OJL = 1201tx0.265 = 100 Q
i '
(i) The time equation for voltage

V = 230,/2. sin l 20!tl.

'·)

i = , 2.3,/2 sin(12oru-f)
,.
-2.3,/2 cosl20ltl. Ans.

(ii) Energy stored, -2l L''o 2

.!..xo.265x( 2.3,/2)2
2 .
1.41 Ans.
,,

Ex. 6 The voltage applied to a purely inductive coil of selfinductance 15.9 mH is given by the equation
. V = 100 sin 314t+ 75 sin 942 t+ 50 sin 1570 t.
Find the equation of the resulting cur~e'nt,v:iv~:
Sol. The standard equation of the voltage can be, 'Y,'i\ten as :
V = V01 sinoo;t+V02 sinoo2t+V03 sin0l:JI
On comparing this with the given equation, we have
rad -3
314-,XLI =oo1L=314xl5.9x!0 =5!2
s

rad , -3
942-,XLz =rozL=942xl5.9xI0 =!SQ
s '

rad
1570 , XLJ = Ol:JL = I570x!5.9x!0-3 = 25 n
s . .

Hence:

0 I J ';:'.!,\''
Vo2 = 75 =5 A
XL2 IS·'··· ' ·

and

Thus i01 sin OJil + i 02 sin fl)zl + i03 sin Ol:JI


'. \

= 20sin( 3141-% )+(;sin( 9421-% )+2sin(1s101-%) Ans.


.,.. ,1,,

9.9 POWER IN AC CIRCUITS V


' in which there is a phase difference <j> between i and V. Current i
Consider a circ~it
can be resolved into two mutually perpendicular components; icos<j> along the
direction of applied voltage V, and i sin<j> which is perpenclicular to V.
The power consumed by the circuit is given by the product of Vand that component ' .
,,,' I
of the current i which is in phase with V. ,
i sin<!>
So Vxicos<j> Fig.9.22

. . V i0 · V0 i
0 0
· and Vnns X Inns X cos <j> = r;:; r;:; COS <j> = - - COS <j>
v2 v2 2
where cos<j> is called power factor.
This power consumed is called true power or active powei

~
.R
cos<j>
z
where cos<j>!> I. Fig. 9.23
R
1
. EL" E.C""'R.ICl"n>",:&·::;MA:"',. G.N··=1s··M ·:>t/t -" _,. r· -·,· ··. ·. ·. .'·_.,i.·.:~
' •. ;s •
- .c)d;::•i!;i"
. .';. · :::"'·.·.> ..,.·.z.i.•_:,·.·..•·.'1i..,·.•,;r.·.·.'.,,.t.·,·,,.. ,'.· :."..•,-.·.
· '! ., .H• l., ,_. "-~· .• ·,:•.•.·....

- .
m.• • ·

-~-... .,.
!.¼¥!# .' ...
-
\;'_t. ;_,_.;

In an ac circuit the produc.t of rms voltage and rms current gives 'apparent power

[. pav l pp -- .,~v n_ns


X '
rnns ··. .. ~.----.
(i) When circuit has resistor only,
O; cos<j> = I ,.
and p av vrms' inns
(ii) When circuit has inductor orcapacitor·only,
7t
=
<I>
2 ,cos<j>=OandP .=0 0

Current in such a circuit is called wattless current.

Constant power

Electric ~Thermal power


generator
>---------- R
Fig. 9;24 ·
(wattfull
component V
Active and reactive components of circuit current
Active component is that which is in phase with the applied voltage \ tl
o"~..
V, i.e., i cos<j>. It is also known as wattfull component. Reactive ' ',
component is that component which is perpendicular lo V, i.e., i /
sin<j>. It is also known as wattless or idle component. ,/ . :,,,'
.,. i·sinq, (wattless·componcnl)
Instantaneous power arid ·average pow~.r .' Fig: 9.25 _
Consider the most general case, in .which current and.~o)tage differ. in phase by an.angle <j>. The instantaneo.us
power
Pinst Vi

[V sin Oll] [io sin (rot-<!>)]


V0 i0 sin Oll [sin rotcosq,-cosrotsinq,]

V0i~ [sin 2 rotcosq,-sinrotcosrotsinq,]


,
PinSt~·=
/

V0 i0 [ sin 2 ro~ cos cp sin 2~ sin cp] llT.2


-
T O
sm rot dt·= -,
2
I]
}1] I
[].. lT sin 2rot dt = 0
T o . . .
or p
av
~. • V0i0
. ,
[(.!..)cosc)>-o]
2 ..
= Voio cos(j>
2
. ~ - ~-- . . - '

or : Pav = Vnns inn,, cos<!>; .....(!)


Half power points and band width

We know that Pav = --cos(j>


Voio
2
"

Vo Vo 'R
-x-x-
. 2 Z Z

V/R "
= iz 2 · >/"' -

For RLC series circuit, z


~R
2
+f ~-~Cr'
v:2R
0
-

J]
~av

2[ R2:( ml- ro'c

At ffi=ffio
· =.· ~
LC' Pav = ·Po

V. 2_
_o_
,.,.
..2R -c -.-,· 2' --- - - -- -
, Pav
, P0R .....(2)
. lR2 +( rol--~C rJ.
·At ro O,P~v=O
At ·ro = co, Pa~ == 0
P,r-·

p
0

PrJ

IL._ _;__ _;__ _;__~=-+ro


00 1 6)0 (i)2

Fig.9.26 variation of average power with ro.


R
For, ....Q., we have
2
Po
2

or

I
or roL--= ±R
coC
The above equation will have two values of co, difference of which gives band width.
I , I
roL-- =+R roL-- =-R
coC roe
2 R l 2 • R l
or ro -ro---=0 ro +ro--- =0
L LC L LC

- R±
L VIL
j(!!..)2 +4(~)
LC·
ro= - - ~ - ~ - ~ -
2
co always be positive, therefore

_f+ITT
' '°2 - 2
_-r+ITT
ffi1 -
2
R
E and width .1.ro = '°2 - ro 1= L
The current corresponds to half power, irrn, = Ji= 0.707 io

Quality factor or Q-factor


The characteristics of a series resonant circuit is determined by the Q-factor of th~ circuit. It ~an be defined as
Z1t maximum energy stored :
Q-factor = energy dissipated per cycle

Resonance frequency COo


-~---~~=--
band width .1.ro

JL~
(~)
·=~ ~-'~,:~ :1 .
r:Q-factor-
L____-_v_c:__ .: ;
or ....(3)
Low R, large Q-factor

High R. low Q-factor

Fig. 9.27
Acceptor circuit
If the frequency of ac supply is varied, the series RLC circuit passes a maximum current and power output for
ro= '%· This is the procedure by which a radio or television receiving set may be tuned to receive the signal from
a desired station which is sending signals at a particular frequency. The circuit is thus known as an acceptor
circuit.

Ex. 7 (a) In a circuit containing a capacitor and an AC source, the current is zero atthe instantthe
source.voltage is maximum. It is consistent with ohm's law ?
(b) Can you have an AC series circuit in which there is a phase difference of 180° between
potential and current.
(c) Is rms value ofaltemating current can equal to peak value of current?
Sol. (a) No.
(b) No.
(c) Yes, in a square wave ..
Choke coil
Choke coil is a device having high inductance and negligible resistance. It is used to control ac in the circuit as
rheostat is used for de. Ideal choke coil has no power loss., But practical choke coi_l has some power loss
because of resistance of inducting coil. Practical choke coil is in LR circuit as discussed earlier.

Iron core

L__...c_...(,,-..,)---__J

Chok~ coil.

Fig. '/.28
Ex. 8 A series circuit contains a resistance of 40, an inductance of0.5 Hand a variable capacitor
across a 100 V, 50 Hz supply. Find
(a) the capacitance for getting resonance,
(b) p.d. across inductance and capacitance
(c) the Q-factor of the series circuit.
Sol. (a) For resonance XL Xe ,' ,,
I.
or rol
roe.
I
e oil= (21t/} 2 l

= 20.3xlo-6 F Ans.
(2x~xso)2 xo.s

(b) Current at resonance= 100 =25A


4
P.d. across inductor oicapacitor ,
. ' ' , , ' I'
vL Vc=iXl'=25(21tx50x0.5) ·
,J, •
3925V · · . ' Ans.
OOol
(c) Q-factor·
R. - - . - .
'• r, . -'-: ,!
., (21tx50)x0.5,
4 ' '
39.25' · :' Ans.
Parallel LCR circuit
Consider parallel LCR parallel circuit as sh'own infig: 9.29.
·/
R

''

Phasor diagram '--''--,t"v}--~


1-· = v(I sinmt
Fig. 9.29
Vo
We have, iR
R
Vo
il
XL

Vo
ic ,,
•X C
'---------------- ----
From phasor diagram ' .., , ..

and tanq, cci:il)


Admittance (A):

Vo
z (iJ+(t-tJ
or
z J(~ J+( J roe-~

Current ;=v (~J+(roc-~LJ


At, ro = 0, j=OO

V
ro O\i, ;min =R JI
R
,,,
ro ~, i = ··oo , u>o
Fig. 9.30

Power p ~ =Vi[ (~ J+( roe- ~L J]


At, ro O,P= 00 I'

vi
ro O\i, Pmin =11
ro oo,P=oo
vi
Resonance: q, 0 -R
ic il gives imin = iR
------------
Cr.\)
(i)
,

or roe Fig. 9.31


rol

=> ro ~ llC =roo


or I 1
2it LC
Jfc
Use of operator j in AC circuits
In complicated ac circuits, the use of the operatorj is of very great importance. In Cartesian form, a phasor A can
be written as,
A = a+jb
where a is the x-component and b is they-component of phasor A.
The value of j = ,/-1,:./=-l,/=-j,and/=t

y
j

A
b
_ _ _ _....J,:'...J.!~L--..1---x
a

Fig. 9.32
From the geometry A Jaz +b2
b
and tancj)
a
By usingj operator we can write XL and Xe in the following manner.
XL jrol
j

and Xe -j( ~c)


-j/j ( ro
1
c) i----•R

Xe=_I_
iroC
or Xe = jroC
Parallel circuit (Rejector. circuit)
Let us consider an ac source is connected across an inductance Lin parallel with C._The resistance in series with
the inductance ·is R. Let the instantaneous value of potential be V and the· corresponding current be i, the
current through the inductance be iL and through the capacitance be ic- These currents il and ie will be almost
in opposite phase if R is very small. The total current, ic C
. + ic
'L "
,11----
V V V R
or ----+-- ..
z (R+ jrol) _I_
jroC

.I +jroC
-
or (R+ jrol) Fig. 9.33
z
_.. , .,.· . -.,_, I. .- ,(R_-,jroI;),.· · . ·'.
,,f?f . ',, l , - ,.,.,.J A. z ·· :.- ')(..
( R+jrol. ·)+JWC
R-Jrol · ·
•~ I_; • 't'.· 1,,. • ·{, '

.•JI

R+ j( roCR 2 +ro3L2C-rol)
R2+ro2l2
:. The magnitude of the admittance
·c~-...--.;..- -~=~- ,. . ..-- '=::s7

!~,. ~.;· . JR
I .A=- = · ·· ·
2 +(wcR 2 ·+w3i c..:~)2~ j
2
· 2 · ·2 -•· · ···· ·_· l
fr/'' . · Z. · ·. ' · . · R '+ro~L · ' ;, , ',·1
LA-~~ -. , . ,_j
For resonance, i will be in phase with V, ifj component ofA becomes zero. i.e.,
roCR2 + o}L2 C--Oll = 0

or

At resonance, Z will be maximum. It will be·

R2 +ro2L2'
z ·= - - - ~ , is called dynamic resistance .
. ,, ' R · · ·1 •

L
CR
At resonance the peak current from supply is, known as make up current, which is

i =
.: ..
_

f.;-· =·,'·CRV~
__.__o
or
I . ··· L ·

For the valhes of L, ~- R a~d


·
~ satisfyin~ the equation ro = J[ LC1 - R:L }the reactive,. component of A is real.
. .
At such ajnaximµm impedance, the current in the circuit is. ~inimum:Thus the parallei ~ircuit does not all6;_; this
freq~ency fromth·e source'to pass through the ci;_,uit. Due to this reason_the circuit with_ such a frequency is
known as a rejector circuit.
' . \ .-
. ' .. ' ' '. . ...
Ex. 9 An ac source lifangular frequency mis fled acro~s a resistor Rand a capacitor Cis series. The
current registered is i. If now the frequency cifthe source is changed to co'3 (but maintaining the
same voltage), the current in the circuit is found to be halved. Calculate the ratio ofreactance to
resistance at the original freque~cy.
Sol. At angular frequency ro,-the curr~ni in CR-circuit is given by

irms ... (i)


, \ • !·'

When frequency becomes co'3,

inns
...(ii)
2

From equations (i) and (ii), we get '' '


,, • ~ J•

5
3R2 -= 2
' ( roC)

1/roC
The ratio
R J¾ ._ Ans.

Ex. 10 In inductor-coil, a capacito~ an AC source ofrms voltage 24V are connected in series. When the
frequency of the source is varied, a maximum rms current of 6.0 A is observed. If the inductor coil
is connected to a battery of emf 12 V and internal resistance 4.0 0 , what will be the current?
Sol. The maximum current Will occurJ1t resonance in series circuit, which is
l
V
i ~ ·z .. ,, ,·
I

or z V=24= 4 Q '-----<'°'J>----'
' "i 6
V=24V
Fig. 9.34
The impedance of the circuit= 4 Q. No": if this coil is connected to a battery of 12 V and internal
resistance 4 Q, the impedance of circuit become, Z' = 4+4=8 Q

., !:'._'.=~=!.SA
ii '= Ans.
Z' :8,,
'· ' ' . ,

' .
Ex. l l A circuit draws a pciw~rof 550 watt from'a source of220 volt, 50 hertz. The power factor of the
'·circuit is 0.8 and the cilrrerit lags in phase behind the po'tential difference. To ~ake thep.ower
factor to the circuit as 1.0, what capacitance will have to be connected wliii'id ·· · ' '
·'- ACANDEM-;JVAVES -M
Sol. The impedance of the circuit

v 2 220 2
,Z -=--=88
p 550

or R2 +(roL)2 882
.
2
"

or R2 +(I001tl) 882 ... (I)

and cos,P -R
z
R
or 0.8
88
R . 70.4Q
From (I), we get
rol = ·42.2n
For power ractor to be I, the resistor remain effective
cos,P I

which gives rol


roe
I
C ro( rol)

(21tx50)x4,2.2
7.5 x 10-5 F
75 µF Ans.
So ca_pacitor of capacitance 75 µ F sh?uld be connected in series in the circuit.

Ex. 12 A box contains L, C and R. When 250 de is applied to the terminals of the box, a current of 1.0 A
flows in the circuit. When an ac source of250 V rms at 2250 rad/sis connected, a current ofl.25
Arms flows. It is observed that the current rises with frequency and becomes maximum at 4500
rad/s. Find the values of L, C and R. Draw the circuit diagram.
Sol. When de is used in the circuit, only resistor remain effective
V 250
R = =--=250Q ...(i)
1 1
When de is used, there is current in the circuit it means capacitor must not be in series. It may be in
parallel to resistor together with inductor.
L ..c
"
-
R
...

Fig. 9.35
When ac source is used in ihe circuit, all the circuit element remain ~ffective ..
z = ·,v = 250 =200Q
i 1.25

or.- ...(ii)
;2 + ( /JlL--
. 1 I _)2
' · roe
With incr~ase of frequency, current increases and becomes maximum at .ro= 4500 rad/s. It means Z
must be minimum. It will when
. I .
rol-- = O
roe

L
or ro2 =
.LC
,1 I
or LC= 2 ...(iii)
ro (4500}2
Aller solving equations (i), (ii) and (iii), we get
4
L -~H
81
. and .C IµF Ans.

Ex. 13 In llLC series circuit, assume R = 5.00 0, L= 60.0 mH, f = 60.0Hz, and V0 = 30.0 V. For
what values of the capacitance would the average rate at which energy is dissipated in the resistor
· he(a) a maximum and (h) a minimum? (c)whatare these maximum and minimum energy dissipation
· rates? What are (d) the corresponding phase angles and (e) the corresponding power factor?
Sol. (a) In series RLC circuit maximum power is dissipated at resonance.

,! At reso~~n~,e ·H;;
-roo· =. LC

' I. .I
or C
.ro2L (21tx60}2 x( 60x10-3)

117µF
2 2 ·
and Po Vo =~=90W Ans.
2R 2x5

Po R2
(b) We have, Pav
R2 + ( rol-roC

Pav will be minimum, when Cis zero that isP8 v=O


I r Ans.
------,....,----- -,---. _-· ' -----, .. ---_ ~- .. ·-- - ~ - . . ·c:-----,. ---:-~
' ' . . ' ', ' AC AND EM-WAVES' .
----------·~ - ---- --~- --· -----~---- ------ --- -- __,un.•,·1·>~'
'·J',.
·,.' · "" ' " ' ~ · · -·- · ··~ '
(c) P0 =90 W, and P,,,=O Ans,
(d) Phase angles : · · '.·:;r t1;, I
• ~·1 x,.
At resonance, q, 0
At min"' P 3 ;,,, q, 900 (:Hi,·
(e) Power factor :
90°
for q, 0, cos<), =;I
for q, 90°, cos<), = 0 Fig. 9.36 R

Ex. 14 A circuit with R = 70 O inserieswithaparallelcombinationofL= 1.5 Hand C=30µFisdriven


by a 230 V supply of angular frequency 300 rad/s.' 1 •
(i) Find the imped~nce of the circuit.
(ii) What is therms value of total current?
(iii) What are the current amplitudes in theL and Carms of the circuit?
(iv) Find the total power input to the circuit an~ ppwer factor.
)
.I
(v) How will the circuit behave if ro = ,.--;:; ?
,;LC I
',\' '
f_ = L5H
Sol. R=70O
.,.

'-------I,-._,)--------'
230V. 300 rad/s
Fig. 9.37_. , ,
The resultant impedance of L and C in parallel ,,.. , :
I : Jlc
-+-
Xl"'Xc
Jfd ,1 J

I I I . C
+1ro
jrol
jrol': ( j~C )

I ~ro2 Le"
'_jrol
- ~J I
jrol •
Z1 1_..0llC
- The total impedance of the circuit
· jrol
z R+Z1 =J/+( )
_ __ · l-ro2 LC ,

2
2 rol·
IZI R +( 1-ro2LC )
(i) Given,R= 70 Q ,L= 1.5 H,C=30 x 10-6 Fand ro=300 rad/s.
1:, _;/ . ,,.

ri !I

\' -
z
·1rl r

,\II I
70
2

+ ( 300x. 1.5
l-300 2 xI.5x30xl0-6
)'
-

, '" ,ti }63.3Q An.<.

(ii) . j vrm,=230=I.-4i°A
Ans. --
'rms 'Z 163.3"
(iii) If il and ic are the current~ in indu~,t~r and_ capacitor resp~ctively, then·

\
i .
and ,
il +ic = ;rms
XL.iL,
b
·= Xcic ,
On solving above equations, wechave
...(i)
...(ii)
\ J I, '

\ ! :~1 .,:
Xe
XL +Xe 'm1s
.

and

Again
= ( j~C ) . _ inn,
il l 'rms - 2
·roL+-- 1-ro LC
,~., J jroC

1.41
0.46A
2
1-300 x l.5x30xI0-6
,.,.,·

. ,,
'" jroL
2
( ro LC)inns
I =
,,,_ c, .r...."'"'( . I ) ro2 LC-I
. jOlL+-.-
'J l'J -. i 1roC

-\\(-1\, (3o02 xI.5x30xl0-6)x1.41

300 2 xl.5x30x 10-6 - I


\\·,. 1.87 A
,-,, R 70
(iv) Power factor cos<!> = -=--=0.43 Ans.
Z 163.3
i'· I. . 2 -
(v) When Ol =-,.;:;-,orro LC-I
'vLC .

jroL an d so/=
. 0
Now z --=oo
1-1 ' ' Ans.
r ~ - - - - - - - - - - - - - - ~ ~ AND l;M-WAVES . M
Ex. 15 In the circuit shown in jig. 9.38, the voltage applied is I 00 V. Find the current in each branch,
volt~ge drop across each branch, the power factor and the power supplied by the line to the c!rcuit.

A ..
J{l .\"L =7rJ.
C •••• ~

Ra=-1£1 XL =3D
' 13 ----

IOOV. 60 Hz.

Fig. 9.38
Sol. If ZA and Z8 are the impedances of the branches A and B respectively, then their combined impedance

ZAZB
ZA+Za
Here (6-8}) andZ8 =4+3j

(6-8J)(4+3J) 48-14}
(6-8J)+(4+3J) 10-5}

(48-14}) (10+5}) 550+ IOOJ


(10-5}) (10+5}) 125
4.4+0.8}
The impedance of the branch C,
Zc 3+7}
Thus total impedance of the circuit
z 2Aa+Zc
(4.4+ 0.8})+(3 + 7JJ
7.4+7.8}

Vo
Total current
z
100 100(7.4-7.8})
(7.4+ 7.8}) (7.4 + 7.8))(7.4- 7.8})

740-780} 740-780}
54.76+60.84 115.6
6.4-6.74}
~·· Current lags behind the applied voltage by

tan<j> = 6.74 "'-45'


--or't'~
6.4
The actual value of the current,.,·,
2 2
';~'";,. ~(6.4) +(6.74)
a,:., :,•er. 9.3 A
Voltage drop across branch C - 11 , ''l'

;,··· Vc;,;fcZc
c',,.(6.4-6.74i) X (3 + 7j)
""' .\ e;;t 66.38+24.58j
IV& J' '"~" ~66.382 + 24.58 2 = 70.8 V
1
or
y ._ ··1
Voltage drop across AB,
VAB ·:a,\' 100-Vc
100-70.8=29.2 V. ic::::. 9.3A
From the figure 6.74A --------------- ,
6 .4 = 0.69
:,
cos<j> ,,,
9.3 ,,,
Thus power V0 ;0 cosq>
100 X 9.3 X 0.69 Fig. 9.39 6·4 A
641.7 W Ans.

I. When a value is given fcir alternating current or voltage, it is ordinarily the rrns value.
2. The power rating of an element used in ac circuits refers to its average value not rrns.
3. Though in a phasor diagram, voltage and current are represented by vectors, but these quantities
are not really vectors themselves~ They are scalar quanties. '·
4. Only resistor is the dissipative element and so no power losses associated with pure inductances
or capacitances.
'I
5. A transformer (step-up) changes low-voltage into a high voltage. The current is reduced by the
,,
same proportion. This does not violate the law of conservation of energy.
}

Ex. 16 In LR series circuit, a sinusoid,l,V:,,.V0 sinmtis applied. It is given thatL=35mH, R = 11 0,


? •Kl~
Vnns = 220 V, - = SO Hz and P = - . Find the amplitude of current is steady state and obtain
2p ' 7
,,( '\
the phase difference between the~current and the voltage. Also plot the variation of current for one
cycle on the given graph. ,.I "· · . .

V
. 1': - 'i.\

Fig. 9.40
Sol. The impedance of the circuit is given by

Z ~R
2
c+)x.2
where roL = 2rrx50x35xl o-3
I IQ '"

Z ~I 12 + I 12I = I l..,J2 Q
Voltage amplitude Vo =·.:..fi.vm"
- ..,J2 x220 V.
V0 ..fi. x220
Current amplitude
Z -I {\ J
I IJ'i.
-:.

20A
If <I> is the phase between V and i, then

Jt
- rad.
4 '
Thus we can write
V 220 ..fi. sin rot

and 20 sin (rot-%)

--·--~;.
·.
- ;) .
Qf---a~-4--\---->c~~-,
T •
-··
1 ' =,

Fig.9. ./1 Variation of Vand iwith time

I
9.10 ELECTROMAGNETIC WAVES ' !

The idea of electromagnetic waves was proposed by Maxwell in 1864. The wave equation for light propagating
in x-direction in vacuum may be written as .~ J
E = £ 0 sin(kx - OY)
where Eis the sinusoidally varying electric field. _
The electric field Eis in theyz- plane, that is perpendicul~r-to direction of propagation of wave.
There is also a sinusoidally varying magnetic field associated with the electric field when light propagates. This
magnetic field is perpendicular to both electric field and direction of propagation of wave. The magnetic field
"B = B0 sin(kx - OY)
~ - - - . - 1-~-----"*;;J:.,0)"'"'~"d&.--,;·'~i-~ • . ' _:-:--· ">"'·". -,.-- •. ~-:,:"~--,~-!'.,"
·~ ; ref~f~i!itiRiqm"& ~<,.r-iETisM ":;!;,.:..........---·""· ----_. -""'----·· .~.;.!;-~~~
Such a combimtion of mutually perpendicular electric and magneti~ fields can represetlt an electromag;etic
wave·in vacuum.

E 13

z R E

Fig. 9.42
More about displacement. -current
According to Kirchhoff's junction rule, for a conducting circuit, in the steady state, the total current into any
given portion must be equal to the current out of that portion. But this rule is not obeyed for a capacitor that is
being charged. In fig. 9.43 there is a conduction current into the left plate but there is no conduction current out
of this plate; similarly, there is conduction current out of the right plate, but not into it.

cr -0

~i~c--ee~IH~~~:~~~i~:hj~~--
+ E
+
+ l
, ...- ... : :.•.::~. i

+
Fig. 9.43
According to Maxwell, as the capacitor charges, the conduction current increases the charge.on each plate, and
this in turn increases the electric field between the plate. Because of the changing electric field there is a current
between the plates, is called displacement current (id): A, current withoui movement ofelectrons. For any charge
density cr, electric field between the plates of capacitor

E = ~=-q-
_,. e 0 Ae 0

or q Eo EA 't.
Eo <I>£
dq d(eo <l>E)
And displacement current
dt dt

or

-·-·· .. ··---------.· .
'""" ---------- . .
The current due to the flow of charges is often called conduction current.
.
' - ---~-
. . , -- --~·---.----
_,. :
-----
'
: ... :;."

. ·,. -··---- ·----' _,_,..._. _,

Generalised Ampere's law : · 1,, ·,.,


It was James Clerk Maxwell who generalised Ampere's law. According to him, if there exists an electric current
as well as a changing electric field, thenAmpere's law can be written as:
. '\
pii•dl µo(ic+id)

or

In vacuum, i,. = 0 therefore ,,


.µ ~ ·.,['d~E]
0 0 dt
Maxwell's equations : . ,
Maxwell discovered that all the basic principles of electro/nagnetism can be formulated in terms of four
fundamental equations called Maxwell's equations. They are
q;,, ,·
(I) Gauss's law for electricity pE-dA ....(I)
Eo

(2) Gauss's law for magnetism pii dA0


,, = 0 ... (2)

- d/j,s '
(3) Faraday's law of induction p E-dr. ... (3)
di

(4) Ampere's law pii-di µo (·'c+Eo--;j,"


d/jJE) ... (4)

- d/jJB
Faraday's law pE·dl =
di
Consider a rectangular path abed in the.,:,,- plane as shown in fig. 9.44. The electric field is parallel to y-axis
and magnetic field is parallel to z-axis. The circulation of E over the close path abed,
y
(E+dE)h-Eh
hdE ...(i)
And the magnetic flux through rectangle, /jJ 8 B(hdx)
d/jJB hdxdB
Differentiating, we get ...(ii)
di di

Now from equations (I), (i) and (ii), we have

hdE -hdidB
' \ <Jt -
dE dB
or dt- B
· dx ~'!..

Fig. 9.44
z
' ..
d[Eo sin(h-rot)J-. : . -
· d . (k'·x-ror_·)] . ,_ , '
or - ~-----,---s·~-~ - [B sm ,,.
dr ' ·· · dt 0
. ...
• /l
,

.
• ... J -
E0 kcos(kx--rot) =, _B0 rocos(kr-rot)

or i~--=--'. (: )so
or "
£0 ~ ··Js0 ... (5)
--1
(l) ; __ '1-:r
where k = c, speed of the light wave.
,. 'I

~ B·dl
- :_ ~ .I . d$E
Ampere's law = Eo µo-- ... (2)
1, dt
Consider the rectangular path ~fghin the xz- plane as shown in jig. 9.45. The circulation of B is
. . ~-B•dl ~-, 1Bh7(B+dB) . ··-·
!.
--hdB ...(i) y
The magnetic flux through the rectangle is
$£ E (h dx)
On differentation gives

d$E "!.. ,, ,. dE
dt - hdxdt ' ..:(ii) ' I',,

Now from equations{2), (i) and (ii), we have \ , ; , . " E


I ' dE I .• , I
·l
,-hdB Eo µ 0 hdx- }-~--'l~'-----+.X
dt

or
dB
dx Eo µ0-
dE
/ .ax
h ¥::/i+dB
--!f-,Ji
•• • .1. ......... , __ qr_ ,, z
;\
or -![s0 sin(kx.-ro1)] ·'·-d[
= · ~o µ 0 dt E0 sin (kx-rot )] Fig. 9.45
,,
or -kB0 cos(kx--rot) = Eo µ0£ 0 (-ro)co;(kx-rot)
I ',"IJ , .,,

or
L
• ,,I

Eo ro .'} f;, ' r J


or -x-
Bo k

or

or ... (6)
9.11 POYNTING VECTOR
~ ' • - • • l ..! ', '
The rate of energy transport per unit area in such a wave is described by a vector S, called the Poynting vector

after John Henry Poynting, who first discussed its properties. S is defined as
" .
-
S = - I ·[- -] =ExH
ExB· - - ..... (7)
µo
Its SI unit is W/m 2 .

Ex. 1 7 A plane electromagnetic wave of frequency 25 MHz travels in free space along !hex-direction. At_

a particular point in the space and time, E = 6.3j Vim. What is ii atthis point?

Sol. The magnitµde of the magnetic field is given by


. . '

B = E =~=2.1x10-8 T.
C 3Xi0 8
J

The vector Ex ii should be along x-direction, and so

]x(A) - /, indicates that A= k.


8 •
Thus 2.lxlo- k T. Ans.

Energy transport
Figure shows rt travelling w·ave, along with a thin box of y
thickness dx and area A. At any instant the energy stored in
the box is E
_ d!) = __du£+_ du 8
,. (uE,+u 8)(A'dt)·.

dU
or z
dV

I 2 B2 Fig. 9.46
-e 0 E +--
2,, 2µ 0

. 2
l·i•2 Bo 2
-E 0 £ 0 sin (kx-rot)+-sin (kx-rot)
2 2µo
.,

, . . . I
The average ofsin 2 (kx---OYJ_o~er o~e complete cycle is 2.
"
dU '•.,I ' 2 Bo 2
Therefore -=-.eo'Eo +-- ....(8)
dV· 4., 4µo

,! .
· uav
. I 2
=. -eo Eo' =-.-..
Bl .. ..(9)
2 211o
Intensity of wave
The energy crossing per unit area per unit time perpendicuiar tci the direction of propagation is called ·the
intensity of wave. From above figure we have
dr cdt
Energy contained u uav (dV) = uav (A dr)
uav Acdl

u
:. The intensity I = .--=UavC
Adt
--· . I-· 2
or . I: = , 3··
. _,_ -,e 0 £ 0 , c. , .... (10)

Momentum
The electromagnetic wave also carries momentum with it The momentum carried by the portion of wave having
' .
energy U is gives by p =
.u
C

Ex. 19 A light beam travelling in !hex-direction is described bytheelectricfieldEY =(300V/m) sin? 1 - ~


C
An electron is constrained to inove along they-direction with a speed of 2.0 x 107 mis. Find the
maxim~m electric force and the maximum magneticforce on the electron.
Sol. The maximum value of electric field is 300 V/m, Therefore the maximum electric force
Fe = qE0 = 1.6 x 10-19 x 300
= 4.8x 10- 17 N
The maximum magnetic field

The maximum magnetic force on electron


Fm = qvB0
19
( I .6xl0~ )x( 2.0xl07 )x10-s

3.2x10- 18 N.
Ex. 20 Calculate the electric and magnetic fields produced by the radiation coming from a 100 watt bulb
at a distance of3 m.Assume that the efficiency of the bulb is 2.5% and it is a point source.

Sol. Bulb radiates energy uniformly in all direction. At a distance of3 m from the bulb, the surface area
of the envelop

The intensity of radiation at this distance is

Power 2.5 I 00
I = --=-X-
area 100 113

= 0.022 W/m2.
As halfofthe intensity is provided by th.e electric field and half by the magnetic field, so

I
2
= 0.01 I.

I 2
Thus 2Eo Erms c 0.011

0.01 IX2 O.l lx2


or Erms = =
Eo C 8.85x10- 12 x3xl08

2.9V/m
The peak value of electric field

Eo ..J2 Enns =../2 x2.9


4.1 Yim Ans.

Enns 2.9
Now -c-= 3xl08

9.6xl0-9 T,

and Ans.

9.12 ELECTROMAGNETIC SPECTRUM


Maxwell predicted the existence of electromagnetic waves. The only familiar electromagnetic waves were visible
light waves. By the end of nineteenth century, X-rays and gamma rays had also been discovered. We know now
were electromagnetic waves include infrared waves, visible light waves, X-rays, gamma-rays, radio waves,
microwaves, and ultraviolet waves. Their classification according to frequency is called the electromagnetic
spectrum.
. ,,_ __ --- - t - - --
.' ,:•;' ;~; :\ ' I.' ., i ". > ,· t~ :·,- ·-
.. _,_,
·;"' . • ~j ·'.
,, _, ,-,
Frequency Wavelength
llz ,,
!
to~.2
10:: 1 ctmnma rays
10~0
1
10 ''
IA
10'8 X-ravs
1nm
1017
101(,
Ultraviolet
·10'' ,.
!014 I Jtm
!Oil I Visible light
Infrared
10 12
10 11
1010. Microwaves 1cm

j,
I
9
10
TV.FM
Im
10'
JU' Radio waves
6
Standard broadcast.

10
1km
10'
I
Long wave
10'
10' I

f:;g.9.47 The electromagnetic spCctnnn. Note the overlap between one type
of wave and che next. There is.no sharp division between the
different regions of cm spectrum.

I, r.
.·,'

''
,' !_ .,·,,,.
.,, .•I'
' .. ~. ,.
-"
<---.-- - . _, -·--» ·-r-=· .
I.
-- TO-••-••.. ----~-- -·--·-·'••" ~

I. A capacitor in an LC oscillator has a maximum


potential difference or 17 V and a maximum
energy or 160 µJ. When the capacitor has a
potential ditlerenceof SY and an energy of I OµJ.
what is the energy stored in the magnetic field '!
(a) 10 µJ (b) 150 µJ (l) (11) (111)

(c) 160µJ (d) 170µJ (a) 11 > 12 > 13 (b) 11 < 12 < 13
2. lfwe increase the driving frequency in a circuit (c) 12 < 11 < 13 (d) 1_, = [,;i;
with a purely resistive load, then amplitude VR
6. In an oscillating LC circuit with L = 50 mH and
(a) remain the same (b) increase C = 4.0 µF, the current is initially a maximum ..
(c) decrease (d) none How long will it take before the capacitor is fully
3. The figure shows a sine curve S(I) = sinCJJI and discharged for the first time :
(a) 7xJ0--4s (b) 14xJ0--4s
three other sinusoidal curves A(/), 8(1) and C(I).
(c) 28 x I0--4s (d) none
each or the form sin (CJJl--41)- Which curve is
7. Charges on the capacitors in three oscillating
according to the most negative value of I\> LC circuits vary as follows : (I) q = 2 cos 4t, (2)
q =4cos t, (3) q=3 cos 41, (4)q =4 cos 21, with q
in coulomb and I in second. In which circuit(s)
·current amplitude is greatest:
(a) .(I) (b) (2)
(c) (3) (d) (4)
8. A charged capacitor and an inductor are
connected in series at time / = 0. Read the
statements :
(a) A (b) B (I) the charge on the capacitor is zero after a
(c) C (d) Band C - T
4. A parallel-plate capacitor with rectangular plates time 2.
is being discharged. A rectangular loop, centered (2) The charge on the capacitor is zero after
on the plates and between them, measures L by
2L : the plates measure 2L by 4L. The fraction of -
t,me
T
the displacement current is encircled by the loop 4
if that current is uniform ' (3) the energy stored in the capacitor maximum
T
at -
(a) (b) 2 2
(4) the energy stored in the inductor is maximum
I T
(c) (d) 8 at -
4 4
5. Figure shows three oscillating LC circuit with Out of these statements; c~rrect statement(s) is/
identical inductors and capacitors. If I 1• 12 , I3 are are
the time taken by the circuits I, 11, 111 for fully (a) 2 (b) 3
discharge, then (c) 1,2 (d) 2.3,4
9. lfwe increase the driving frequency in a circuit
,l'
with a purely capacitive load.·Read the following
statements : ,
(I) amplitude Vc increases I,:
(2) amplitude Ve decrease
(3) amplitude ic increase
(4) amplitude ic decreases
Out of these, the correct statement(s) is/are
·(a) -vex-direction (b) .:.vex-direction
' .
(a) I (b) 1,2
(c) +vez-direction (d) -Vey-direction
(c) 3 (d) 2,3,4
14. Which of the following plots may represent the
IO. lfwe increase the deriving frequency in a circuit reactance of a series LC combination ?
with a purely ind~ctive load. R~ad t~e following I.
(a) .
statements :
(Ii' amplitude VL rem~in_constarit (b) II
(2) amplitude VL increases
(c) Ill
(3) amplitude iL increases
(4) amplitude il decreases · (d) IV IV
Out of these; the correct statement (s) is/ are ·ts. An AC source is rated 2 0 \t, 50 Hz. The average
(a) I . (b) 2 , . 'voltage is calculated in a time interval of0:0 Is. It
·(ri) may be zero· (b) must be zero
. (c) 1,2,3 (d) \,4
l 1. Here are the capacitive reactance and inductive . 220v
reactance, respectively, for three. sinusoidally
(c) is never zero (d) IS .J2
driven_ RLC
.),'
circuits .. : :(I) ·50Q,I00Q 16. An electromagnetic wave going through vacuum
'. ' is described by E = E0 sin(kx--Oll); B = B0 sin
(2) 100 Q,50 Q (3) 100 n, I00Q. Which is in (kx--Oll). Then
·resonance '? (a)' E 0 k= B0oo (b) E 0 B 0 = ook
(a) I (b) 2 (c) E0oo= B0k (d) none.of these,
(c) 3 (d) 1,2 17. Alternating current can not be measured by de
ammeter because
12. ~n alternating current emf device has a smaller
(a) ac cannot pass through de ammeter
resistance than that of the resistive load, to
(b), . averag~ \al~e ?f complete cycle is zero·
increase the transfer of energy from the device
(c) ac is virtua_l , ,
to the load, a transfo~mer will be connected
between two. Then
,. @, ac, changes its direction ,
(a) · Ns should be greater than Np,
18. lfacurrent/give~by / 0 sinf 001-~) flows in
(b) Ns should be less than NI p
(c) N8 =Np an ac circuit a~ross which an ac potential of
(d) none V = Vo sin 001 has been applied, ,then the power
13. The figure here gives the electric field of an consumption Pin the circuit will be
electr6magnetic wave at a ce~ain point and a
certain instant. The wave is transporting energy
P= Volo
\a) ..fi. (b) P=F2V0 J0
in the negative z-directio~. The di~ection of the
magnetic field of_ the wave at that point and ~=Voio
instant is : (c) (d) P=0
2
24. In a LCR circuit capacitance is changed from C
19. An alternating voltage V = 200"2 sin ( I001) is
to 2C. For the resonant frequency to remain
connected to a I microfarad capacitor through unchanged, the inductance should be change
an ac ammeter. The reading oft~e ammeter shall from l to
be (a) 4l' (b) 2l
(a) IOmA (b) 20rnA (c) l/ 2 (d) LI 4
(c) 40mA (d) 80mA 25. In an LCR series ac circuit, the voltage across
20. A 220 V, 50 Hz ac source is connected to an each of the components, l, C and R is 50 V. The
inductance of0.2 Hand a resistance of20 ohm in voltage across the LC combination will be
series. What is the current in the circuit (a) SOV (bl soJzv
(a) JOA (b) SA (c) IOOV (d) 0 V (zero)
(c) 33.3A (d) 3.33A 26. A bulb and a capacitor are connected in series to
21. In the circuit shown below, the ac source has a source of alternating current. If its frequency
voltage V= 20 cos(OY) volt with ro= 2000 radls. is increased, while keeping the voltage of the
the amplitude of the current will be nearest to source constant, then
(a) bulb will give more intense light
-·l----"IIIM-~ (b) bulb will give less intense light
60 (c) bulb will give light of same intensity as
before
5 mH. -H2 50 µF
(d) bulb will stop radiating light
27. An alternating e.m.f. of angular frequency ro is
applied across an inductance. The instantaneous
(a) 2A (b) .3.3A power developed in the circuit has an angular
frequ~ncy
(c) 2 / ../sA (d) ../sA
(0 ro
(a) (b)
22. The power factor of an ac circuit having 4 2-
resistance (R) and inductance (l) connected in (c) ro (d) 2ro
series and an angular velocity ro is
28. The diagram shows a capacitor C and a resistor
R connected in series to an ac source. V1 and V2
(a) RI rol (b) RI ( Rz +ro-L-
, , )"2
are voltmeters and A is an ammeter

I/'
(c) rol IR (d) RI( R2 -ro2L2 ) -

23. An inductor of inductance l and resistor of


R
resistance R arc joined in series a.nd connected
by a source of frequency ro. Power dissipated in
the circuit is
Consider now the following statements
(R2+ro2l2) V 2R I. Readings in A and V2 are always in phase
(a) (b) (R2+ro2l2) II. Reading in V1 is ahead in phase with
V reading in V2•
Ill. Readings in A and V1 are always in phase.
V Which of these statements are/is correct
JRz +ro2l2
(c) ( R2 +ro2l2) (d) (a) I only (b) II only
vi (c) I and II only (d) II and Ill only
29. The voltage of an ac supply varies with ,time (/) 33. When an ac source of e.m.f. e = V0 sin( I00 I) is
. , '( - . ' ,,, . ,··
as V = 120 sin '100 7t / cos I 00 7tl. The maximum connected across a cirCuit, the Phase difference
voltage and frequency respectively·are' between the e:m.f. 'e and the currenfrin the circuit
,,
120 '· is observed to be ic / 4', as shown in the diagram.
(a) 120 volts, J'00 Hz (b) Ji.- volts'. ,,1·00 Hz If the circuit consists possibly only of RC or LC
in series, find the"relationship between the two
(c) 60 volts, 200 Hz (d) 60 volts, I00 Hz ;· - elements· ,I''
30. Match.the 'following· - ,, ,,,
:, ;
' Currents "r.ITl.s. values· -fore·, ···' _,,· ~" ,~,:~'
,, 1• l l I! fl
(I) x0 sin rot (i) XO

., ... Xo _,
(2) x0 sin rot cos rot ,__, ' (ii)
Ji. ,.1' '.• , , , )i ;r.,

..
C
;: . ,( It_.. r,
' .
•' ~

(3) 'Xo~i~~l+XoCOSOll' :·(iii); (2!i.) . (a) R = lkQ, C= IOµF


-
(a) • l.(i),2.(ii),3.(iii) (b) R = lkQ, ,C = lµF
(b) I. (ii), 2. (iii), 3. (i) (c) R = lkQ, l = IOH
(c) I. (i), 2. (iii), 3:'(ii) -, '
(d) None of these (d) R = lkQ, l=IH
31. An. ac source of angJlar frequency w· is fed 34. In an oscillatory circuit, L = 0.4 H, C = 0,0024 µF.
1
across'~ resistor r and a c'8.Paci!or ·c in series. The maximum value of the resistance that should
The current registered is/. If now the' frequency be included is : I ,'
of source is changed to co'3 (but maintaining the (a) 2.58x 104 Q (b) 2.58x J03 Q
sarhe voltage), the current-in the circuit is found
(c) 5.82x I04 Q ·(d) 5.82x I03 Q
to be halved. Calculate the ratio ofreactance to
35.. At I< ci, the c'apaCi°tcir is' charged and the switch
~esistance, at the~o;iginal. frequenc~ co .. '
is open.eel. At I = 0 the 's'witch is closed. The

(a)
'JI (b) , ' ·i ~
5
shortest time T at which the charge on the
1

caPacit~r wi II b; ~era is given ,by :

32.
(c) Jf (d)
i
The output current versus time curve ofa rectifier
I'-'•
., ,, ,L·:J,,
Q,, C

'1'.u '1, , • ,
is shown in_ the figure. The average value of
output current in this case is
. s ,. ,,
(a) · O• · ,.
I I'., . 3
I (a) (b) . -7t..JLC,
(b) _Q_ 1t-/I;c
2 I,. J . 2 :
'-

. 2/o ~../Le.
(c)., ""';;- (c). (d) .21t..JLC,
2'
'
(d) .,o· . '·' ,., "\ ;

I ' ,,.
--------A-C~AR-D_E_M___
W_A_VE_S_~
--- -------------- ~

&wzdae 9.2

Directions (Qs. 1-6): Read the.following questions 5. The magnetic field can be produced by
and select the right choices. More than one options (a) a moving charge
may be correct. (b) a changing electric field
1. Figure shows a parallel plate capacitor and the (c) a non uniform field
current in the connecting wires that is
(d) none of them
discharging the capacitor.
6. An alternating e.m.f. of frequency
I'

.,

..r( 7t5-w)
2
is applied to a series LCR

circuit. For this frequency of the applied e.m.f.


(a) The circuit ·is at resonance and its
impedance is made up only of a reactive
part
(a) The displacement current is leftward. (b) The current in the circuit is in phase with
(b) The displacement current is rightward the applied e.m.f. and the voltage across R
equals this applied emf
(c) The electric field £ is rightward
(c) The sum of the p.d.'s across the inductance
(d) The magnetic field at point P is into the
. and capacitance equals the applied e.m.f.
page.
which is 180° ahead of phase of the current
2. If the inductance L in an oscillating LC circuit
in the circuit
having a given maximum charge Q is increased,
then
A
(d) The quality factor of the circuit is OJl I or
I / OX:R and this is a measure of the voltage
(a) the current magnitude increases
magnification (produced by the circuit at
(b) the maximum magnetic energy increases
resonance) as well as the sharpness of
(c) the maximum magnetic energy decreases
resonance of the circuit
(d) curr~nt magnitude and maximum magnetic
energy remain con_stant.
Directions (Qs. 7-15) : Read the .following passages
3. The reactance of a circuit is zero. It is possible and a~swer the questions thpt .follows.
that the circuit contains
(a) an in\luctor only 'I
Passage for Q. 7to Q.9
(b) a capacitor only
Two coils are cJnnected in series. With 2A de through
(c) an ind,uctor and a capacitor
the cir9uit, the p.ds. across the coils are 20 V and 30 V,
(d) a resis_tor only. respeclive!y. With 2 A ac at 40 Hz, the p.ds. across the'.
4. In anACseries circuit, the instantaneous current coils a;e 140 V and I 00 V respectively. The two coils · ·
is zero when the instantaneous voltage is are connected in series with a 230 V, 50 Hz supply,
maximum. Connected to the source may be I
I
(a) pure inductor /
7. Tihe current in the coils is :
(b) pure capacitor '
(k) IA (b) 1.45A
(c) pure resistor
(d) combination of an inductor and a capacitor (c) 1.55A (d) 2.15A
8. The power factor of the circuit is:
Passage for Q.13 to Q. 15
(a) 0.17 (b) 0.5
A parallel-plate capacitor has square plate 1.0 m on a
(c) I (d) none side as shown in figure. A current of2.0 A charges the
9. The power dissipated in the circuit:
capacitor, producing a uniform electric field E between
(a) 40W (b) 60W
the plates, with £ perpendicular to the plates.
(c) SOW (d) IOOW
•. o.so Ill •
Passage for Q.10 to Q.12
A voltage V = I 00 sin 314

series.

10.
I is applied to a circuit
~onsisting of a 25 n resistor and 80µF capacitor in

The instantaneous current is given by : 13.


__1__
T
Edgc,·icw D Top View

The displacement current through the region


L_lll
.,.

(a) i= 100sin(314t) between the plates


(b) i=2.13sin(314t) (a) Zero (b) I.0A
(c) i=2.13 sin(314t+q,) (c) 2.0A (d) 2.5A
(d) zero 14. The value of dEldt in the region
11. The power consumed
(a) 1.1x10 11 Vim (b) 2.lx!0 11 Vim
(a) 35.5 W (b) 50 W
(c) 56.7 W (d) 90 W (c) 2.3x!o 11 Vim (d) 3.4x!0 11 Vim
12. The p.d. across the capacitor at the instant when
the current is one-half of its maximum value: 15. The value of ef, B.dl around the square dashed
(a) 60.7V (b) 73.SV path
(c) 95.0V (d) IOOV (a) Zero (b) 0.63 µT-m
(c) 0.75 µT-m (d) none of these
w-- ·----- I

JVICQ Type 3

Read the two statements carefully to mark the correct option out of the options given below. Se_lect the right
choice.
(a) If both the statements are true and the Statement - 2 is the correct explanation of Statement - I.
(b) If both the statements are true but Statement - 2 is not the correct explanation of the Statement- I.
(c) If Statement- I true but Statement- 2 is false.
(d) If Stateme11t - I is false but Statement- 2 is true>.. .': ,

I. Statement - I 5. Stateme11t - I
An alternating current is given by i=3 sinOJI + '' The alternating current legs behind the emf by a
7 1t
4cosOJI. The. rms current of it is .fi. A. phase angle of , when AC flows through an
2
inductor.
Statement - 2
Statement - 2
5 The inductive reactan~e ~ncreases as the
The rms current is .fi. A.
frequency of AC source im:rease~.
2. Statement - I 6. Stateme11t- I
A capacitor is connected to a direct current 1
The voltage and current in a series AC circuit~.re
source. Its reactance is infinite. given by V = V0 sinrot and i = i1fOSOJI. The power
Statement- 2 dissipated in the c.i_rcuit is zero.
Stateme11I - 2
Reactance of a capacitor is given by x =_I_.
C COC
Power in AC cirnuit is given .by p = _Voio cos,j>.
3. Statement - I 2

In a circuit containing a capacitor and an AC 7. Statement - I


source the current is zero at the instant the source ' The power is produce,d ~hen a transforr,ner s1fps
1

voltage is maximum. It is not consistent with up the v.pltage. '


Ohm's law. State111e11_t - 2
Statement- 2 In an idea:1 ·transformer VI= constant.
According to Ohm's law, V= iR. 8. Statement- I
4. Statement - I
Hot-wire ammeter
'•
can be used to "measure
; \ cl~- ~
DC.
When the frequency of the AC source in an LCR
Statement- 2
circuit equals the resonant frequency, the
reactance of the circuit is zero, and so there is no DC produces heating e(f!,ct.
current through the inductor or the capacitor. 9. Stateme11t- I
Statemmt-2 Choke coil is preferred over a resistor to control
The net current in the inductoF and capacitor is the current in an AC circuit.
zero. Statement - 2
Power factor of an ideal inductor is zero.
10. Statement- 1 12. ' Statement - 1
A bulb connected in series with a solenoid is The displacement current in a parallel-plate
connected to AC source. If a soft iron core is capacitor of capacitance C can be written as
introduced in the solenoid, the bulb will glow
brighter. . =C (dV)
'd di , where V is the potential
· Statement - 2
On introducing soft iron in the solenoid,, the 1
difference between the plates.
I
electrical inertia increases. Statement - 2
11.. Statement - 1 The displacement current in free space is given
The figure is a view of one plate of a para\lel-
plate c3pacitor from within the capacitor. The
dashed line show two integration paths (path-I
follows the edge of the plate). The value.of

~ B•dC for the path- I is smaller.

.. PaU1-l
,o=·:::::::.::::::.::
i[ ) i-----Path-2
\'-::.......:.......
Statement - 2
/
The value of magnetic field for the path-2 is
smaller. (

...
~-.-,

,·,..)
\ iJ
I

-
I 1 I (b) 1- 8 I (d) I 15 I (a) I 22 I (b) I 29 I (d)
I 2 I (a) I
I
9 I (c) I 16 (a) I 23 I (b) I 30 I (b)
I 3 I (a) I
I 10 I (d) I 17 (b) I 24 I (c) I 31 I (a)
I 4 I (c) I 11 I (c) I 18 (d) I 25 I (d) I 32 I (c)
I 5 I (c) I 12 I (a) I 19 '(b) I 26 I (a) I 33 I (a)
I 6 -I (a) I 13 I (a) I 20 '•(d) I 27 I (d) I 34 I (a)
[ 7 I (c) I 14 I (d) I 21 (a) I 28 I (d) I 35 I (c)

1
I (a, d) I
,-3 - - - ,
6 ( b) ( d) 11 (c)

I 2
I (a, b) I 7 ( C) 12 ( b)
-·----
( C, d) [ 8 (a) 13 (c)
-

'

I 4
I (a, b, d)
I 9 (b) 14 ( C)

I 5
I (a, b) I 10 ( C) 15 ( b)

,~
- -
agnetism
~

I 1 I (d) I 6 I (a) I 11 I ( d)
[_ 2 I (a) I 7 II (d) I 12
- I (a)
-I --3 - --- I (a) I ·8 I ( ~) -I - I
I 4 I ( d) I 9 I I (a) .I -- I
-
I _5 ___ - .I ( b) I 10 I ( d) I - ---- - -I I
u
~---
~: ..
ELECTRICI'JY
--- -- -
& MAGNETISM o

. ,-- . . - - ·- .i· .,

D ___ agneti~m
' ,
'- ;

...,. --
SUBJECTIVE
... , l!, '
&U!Zd4e ti. 4
1. The electric current in a circuit is given b)' i_'== '10 (!...·) for some time. Calculatt; therms current for the
. , t

io
period I= 0 to I = t. 'I Ans: .Ji
2, A current is made up of two components: a SA DC component and a 60 cycle AC sinusoidal component
of peak value 4 A. Write an expression fon1resultant current and calculate the. average current over a
complete cycle and the effective value of currenJ. Ans: i= 5+4 sin 3771: SA; 5.75 A.
3. A 25.0 µF, a 0.10 H inductor and 25.0 Q resistor are connected in series with an ac source whose emf is
.
given bye= 310 sm . 314 t. '.<l ' I

(i) What is the frequency of emf?


(ii) What is rrns value of emf?
(iii) What is reactance of circuit?
(iv) What is impedance of the ~-ircuit?
(v) What is the current in the circuit?
(vi) What are the effective values of voltage across capacitor, inductor and resistor
Ans: (i)50 Hz(ii)219V (iii) 96 Q (iv)"99.2 Q (v) 2.20A(vi) VR=55 V, Vc=280 V, V,_ =69V.
4. An inductor of inductance I00 mH is connected in series with a resistance, a variable capacitance and an
AC source of frequency 2.0 kHz. What should be the, value of the capacitance so that maximum current
may be drawn into the circuit 9 -, Ans: 63 nF.
5. A current of4 A flows in a coil when connected t6 a 12 V DC source. If the same coil is connected to a 12
V, 50 rad/s AC source. a current of2.4 A flows ih the circuit. Determine the inductance of the coil. Also
find the power developed in the circuit if a 2500 µF c,apacitor is connected in series with the coil.
Ans: 0.08 H, 17.28 W.
6. A 60 Hz AC voltage of 160 V impressed across an LR-circuit results in a current of 2 A. If the power
dissipation is 200 W. calculate the maximum value of the back emf arising in the inductance.
Ans: 125V.
7. A LC circuit ( inductance 0.01 H and capaciiance I µF) is connected to an AC source of variable
frequency. If the frequency is varied from I kHz to 2 kHz, then show the consequent variation of current
by a rough sketch.

C
Ans :_t; = 1592 Hz

8. A capacitor of capacitance 12.0 µFis joined to an AC source of frequency 200 Hz. The rrns current in the
circuit is 2.00 A. (a) Find therms voltage across the capacitor (b) Find the average energy stored in the
electric field between the plates of the capacitor. Ans: (a) 133 V, (b) 0.106 J.
.......... ' -·-···· ··--. -----~
ACANDEM-WAVES . ' '
C. •••• - -·· - • -- ------····---~ - --

9. A coil has a resistance or IO Q and an inductance of0.4 henry. It is connected to an AC source of6.5 V,

30 · 5
-Hz. Find the average power consumed in the circuit. A11s: -W.
ll 8
I~. A I 00 V AC source of frequency 500 Hz is connected lo LCR circuit with L = 8.1 mH, C = 12.5 µF and
R = IO Q. all connected in series. Find the potential across the resistance. A11s: IOOV
11. The energy in an oscillating LC circuit containing a 1.25 H inductor is 5.70 µJ. The maximum charge on
the capacitor is 175 µC. Find (a) the mass. (b) the spring constant, (c) the maximum displacement, and (d)
the maximum speed for a mechanical system with the same period.
Ans: (a) 1.25 kg (bl 372 Nim (c) l.75x ]0-4 m (d) 3.02 mm/s
12. A LCR circuit has L = IO mH. R = 3 Q. and C = I, µF. connected in series to a source of 15 cosOli' volt.
Calculate the current amplitude and the average power dissipated per cycle at a frequency that is 10%
lower than the resonant frequency. , , •J Ans: 0.704 A, 0.744 W.
13. An AC source is connected to two circuits as shown. Obtain the current through the resistance R at
resonance in both the circuits.

I.
( . I/ I/

'-----f~}c1~-- - - - '
(a) (b)

> Ans:(a)V/R(b)O.
14. A box contains L. C and R. When 250 DC is applied to the terminals of the box, a current of 1.0 A flows
in the circuit. When an AC source of250 V rms at 2250 rad/sis connected, a current of 1.25 Arms flows.
It is observed that the current rises with frequency and pecomes maximum at 4500 rad/s. Find the values
of L. Cand R. Draw the circuit diagram. 1

4
An.,: L = . H. C= lµF,R =250 Q,
81 L---f~~----'
V
I 5. In the circuit shown in figure the switch is kept in position for a long time. It is then thrown to position
h. (a) calculate the frequ_ency o!'the resulting oscillating current. (b) What is the amplitude of the current
oscillations ?
5HlmH

,_ ____ h
Ans: (a) 275 Hz (b) 364 mA.
1~.on
16. ln figure,, R = 15.0 Q, C=4.70 µF, andl = 25.0 mH. The generator provides a sinusoidal voltage of75.0
V (rms) a~d frequency/= 550 Hz. (a) Calculate therms current(b) Find therms voltages V06 , V6,, V,0 , Vhd'
,,
I V,,J· (c) At what average rate is energy dissipated by each of the three circuit elements ?

a b C

R C
L

Ans: (a) 2.59 A; (b) 38.8 V, 159 V,224 V, 64.2 V, 75.0 V; (c) 100 W for R, 0 for Land C.
17. An LCR series circuit with 100 Q resistance is connected to _an AC source of 200 V and angular
frequency 300 rad/s. When only the capacitance is removed, the current legs behind the voltage by 60°.
When only the inductance is removed, the current leads the voltage by 60°. Calculate the current and the
power dissipated in the LCR circuit. Ans: 2A, 400 W.
18. , In an oscillating series RLC circuit, show that the fraction of the energy lost per cycle of oscillation
AU 2rrR rol
u' is given to-a close approximation by rol . The quantity R is often called the Q of the circuit.

A high-Q circuit has low resistance and a low fractional energy loss ( = ~) per cycle.

19. A light beam travelling in .th; x-direction is described by the electric field Ey = 300 sin
\ .
ro(t _!_ )· An
C

electron is constrained to move along they-direction with a speed of2.0xJ07 mis. Find the maximum
,· electric force and the maximum magnetic force on the electron.
f. Ans: F, =4.8xl0- 17 N,F6 =3.2x J0- 18 N.

20. A.laser beam has intensity 2.5xl0 14 · ~ . Find the amplitude ofelectric and magnetic fields in the beam.
m

s V
Ans: 4.3xl0 - , 1.44 T.
m
0 CHAPTER ,_·~·~·
=
0 .

'
11,...iia'
0 =,as-_:d,
mt,,w~-,.

0
0

.,,.' .'~

I
CHAPTER 1 : ELECTROSTATICS
I. Conductors and insulators :
Conductors are the materials which have large number of free electrons. While insulators have no or
very few free electrons.
2. By conduction method one can charge metals. The charge on the body will be proportional to its size.
3. By method of induction we can charge metals and non-metals both. The magnitude of induced charge q'
is given by

where k is called dielectric constant. For metals k ==, and so q' =q.
4. The elementary charge : Electric charge is quantized : Any charge can be written as ,ne, where
19
11 = ± 1,± 2, .... , and e = l.60x I 0- C. The electric charge is conserved and relativistically not variant.
5. Charge given to metals will always spread on their free surfaces.
6. Coulomb's law: The force between two point charges q 1 and q2 at rest and placed at a separation r in
vacuum or free space is given by

F
'ELECTRICfJY & MA<3NE'i1SM ~

7. If the space of thickness I between the charges is tilled with a dielectric of dielectric constant k. then

F
4
1tEo [(r-1)+1Jk]'
· 8. Coulomb's law in vector form : If charges q 1 and q1 are placed at the position vectors ~ and G
respectively. then force between them

l"tl 1,d (r~ -~ l


I12- -/j-13
9. Electric field: The electric field £ at any point is defined as the force experiences by an unit test charge
placed at that point. Thus if f be the force on charge q0 • then

f
£
'lo

Field due to a point charge £ set up by a point charge at a distance r from the charge is

--,.
41teo ,.-
q

10. Electric potential energy : The change in electric potential energy 1'1U of the system of charges is
defined as
1'1U -Wdectnc..lon.:e ~wagcn1
any posi1ion
Also - [ wclcctrk forcc JOO

As· l../00 0,

any position
U = - [ Wdcctrk force J 00

For two charges q 1 and q 2 placed at a separation r.

U ~ _I_ lft'l2
41teo ,.

I I. Electric potential difference: We can define the potential differencel'J.Vbetween two points/and i in an
electric field as :

1'1V
- [ welectric force l
f

/
/,

Similarly
REVISION (FORMULAE & IMPOHTNAT POINTS)
Potential at any point is defined as

-[ welectric force J:
Potential due to a point charge q at a distance r is given by

I q
V = ---
471£0 ,.

12. Relationship between E and V: We can get

,I B
-f E·dr = f E·dr .I B L
B A

-dV
Also £ Fig. IO.I
dr

For constant field

13. Workdonefrom AtoB:

14. Electric flux: The electric flux offield £ can be defined as:

For constant field£ over the area A. <I>= £Acos8. Fig. 10.2
15. Electric potential. potential energy and flux are scalar quantities but they can be negative. In using
formula of these quantities. place the value or charges with the sign. While in using formula or electric
field do not place the sign of charge.
16. Electric flux due to a charge in the closed surface

<I>= -
Eo
'I
G Fig. /0.3

17. Flux due to the charge out of the closed surface


<I> = 0 0 ·'I Fig. /0.4

18. Gauss's law: According to it. the total electric flux through a Gaussian surface is equal to ( ~ ) times
0
the charge inside the close surface. Thus
. ; '

19. Application of Gauss's law: Using Gauss's law we can derive the following results.
(i) An excess charge on a conductor is located entirely on outer surface of the conductor.
(ii) The electric field due to an infinite nonconducting sheet with uniform surface charge density cr is
perpendicular to the plane of the sheet and has magnitude

a
E = -
2Eo

(iii) The external electric field near the surface of a charged conductor is perpendicular to the surface
and has magnitude '

a
E
Eo

Field E = 0 inside the conductor.


(iv) The electric field at any point due to an infinite line of charge with uniform linear charge density A
is perpendicular to the line of charge and has magnitude
t:
I
E = + + + +r+ + + +i.
. 2JtEQ r
(v) The electric field due to a spherical shell of radms R with charge q Fig. 10.5

E 41tEo :2, for r2' R

0 , forr<R.

20. Conducting sphere ofradius R with charge q (solid or hollow) :

E=--_<J_ (r>R)
41tEo ,-2

' E - - .!f_
41tEo R2
(r=R)

E 0 (r<R)

__ !L
V (r>R)
4ltEo I"

__ !!..
V (r=R)
41tEo R

V=--!L (r < R)
41tEo R
Potential energy of the spherical conductor of radius R with charge q

q2
u = ---
81tEo R
Lo_ _ _ _ - - - - REV,SlON (FORMULAE & lMPORTNJ\T f'blNTS) _M
21. Non conducting sphere of radius Rwith charge q:

E=--!L (r>R)
41tEo ,-2

E (r=R)

E (r<R)

E 0 (r=O)

_ _ g___
V (r>R)
4ite 0 r

V - - !L (r=R)
41tEo R

q(3R2 _,.2)
V (r < R)
41tEo 2R 3

~
1
V = ( 4it Eo ; ) (r = 0)

Potential energy of charged sphere of radius R with charge q

u = _3_,r_
201tEo R

22. Line charge with linear charge density A

E
21tEo r
).
++++++++
at the middle of the line charge
E along.the end of the line charge Fig. 10.6

E_1,

2.fi.1tE 0 r
- ~~·· .''
23. Ring of radius R with a uniformly distributed charge q: +++
qr + ,: ____.
E =
41CEo ( R2 +r 2(
2 + ;,
+
E = 0, (r=0) +
Fig. 10.7
q
6f5 lCEo R 2
I q
V (r>0)
41'Eo ~ R2 +,-2

V - - !!_ (r=0)
4,cE 0 R
24. Energy density in electric field :
I 2
ll = -Eo E (in free space)
2
Electric Dipole : Dipole moment P= q€
Electric field at any position (r, 0):
t· ...

i
P~3cos 2 0+ I .
, ... ,.
E
4,c Eo ,-3 .
Pcos0 )
V ----- -q I +q
4,c Eo ,.2

tan a
tan 0
2
n, .1:7.
L ,._j~_,;
(i) Along the axis of the dipole, 0 = 0 Fig. 10.8
2P
E
4,c Eo ,.3

p
V = 2 •
4,c Eo r
(ii) Along the equator of the dipole, 0 = 90°
p ·£7" .
E
41tEo ?' ___L._
p
V 0. Fig. 10.9
(iii) Force between two short dipoles µlaced on the same axis
p }'., '
F = _I_ 61]P2_
41CEo ,-4
t,.-=r
Force between two short dipoles placed parallel on different axes . Fig. JO.JO
F = _I_ 31]P2
4,c Eo r4
Fig.10.11
REvJSION (FORMULAE & IMPORTNAT POINTS) - - ~
CHAPTER 2 : CAPACITANCE AND CAPACITOR·

1. Capacitor and capacitance : A capacitor is a device which is used to store electrical energy. A capacitor
consists of two isolated conductors (plates) with equal and opposite charges +q and-q. Its capacitance
C is defined as :

C = "
V

where Vis the potential difference between the plates. The SI unit of capacitance is the farad.
2. Capacitance with a dielectric: If the space between the plates of a capacitor is completely filled with a
dielectric material. the capacitance C is increased by a factor k. Thus
cmi.:t.1 = k cair·
In a region that is completely tilled by a dielectric. all electrostatic equations containing Eo must be

modified by replacing E 0 with k Eo.


3. Electric potential energy of the capacitor: The electric potential energy U of a charged capacitor is
given by

'
u = L
2C
= _!_cv 2 .
2

This energy is associated with the capacitor's electric field £ between the plates. The energy density
within an electric field of magnitude is given by

ll
I
= -Eo £-' (In vacuum)
2
4. Redistribution of charges : When two charged conductors are connected by a conducting wire, charge
flows from the conductor of higher potential towards the conductor at lower potential, until their potentials
become equal. If q 1 and q 2 are the charges on the conductors of capacitances C 1 and C 2, then common
potential

V =

If q 1' and q/ are the final charges on them, then


q 1' C 1Vandq/=C2 V
Loss of energy = U;- Ur

I Cif'j 2 + I C2V2-' ) - I( C1 +Cz ) v-.


,
( 2 2 2
5. Combinations of capacitors:

I I I
(i) In series:-=-+-+ ....
C C1 Cz
(ii) In parallel: C= C 1+ C2 + ...
6. Capacitance of spherical conductor ofradiusR

C = "41teo R
7. Parallel plate capacitor:

C (Vacuum between the plates)

C (When dielectric k is placed between

the plates)
If t is the thickness of the dielectric placed between the plates, then

2
. q A
Force between the plates of capacitor F = - - ·
2 Eo

8. Spherical capacitor:

· ab
C 47tEo - - , when outer shell is earthed
b-a

b2
C 41tEo - - , when 'inner shell is earthed.··
b-:~
9. Cylindrical capacitor:

'2JtEo f.
I
'
C
fnb/ a

10. Dielectrics :

(i) Induced charge q'


1
fI
(ii) Polarisation p Induced charge ='L
area A

(iii) Electric displacement D

(iv), Electric susceptibility a


1
:
l I
I
\ l
I .
I____ - - - - · - · · - - - - - - -

11. Kirchhoff's laws for capacitors:


(i) Junction rule : The algebraic sum of charges at any junction is equal to zero i.e., I q =0.
(ii) Loop rule: The algebraic sum of the p.d. across all circuit elements is equal to zero i.e.,

r•+r..'L
" C
= o.

(iii) Sign conventions : -----1 ,__

Fig. 10.12
12. In unbalanced wheat stone bridge, the equivalent capacitance between A and B

2C1C2 +C2C3 +C3C1]


[ C1 +Cz +2C3

Fig. 10.13
13. Charging of a capacitor through resistor:

Charge q = qo (1 -e-1/<)

Current ,. e-flt ,
0

where qo C~and t= CR.

Potential V ~(1-e-111 )
Energy u u.(1-e-11 ·)2
14. Discharging of a capacitor:

Charge q q0e-1/1 ,

. -tit
Current -10 e

Potential V
-2tlt
Energy u U oe
- ; : - - - - - - ......,~."'.'!'-;"-·~ - - - - · - - - - - · ""-'·~_,
;\ '

CHAPTER 3 : DIRECT CURRENT AND ELECTRIC CIRCUITS

I. Electric current : An electric current i in a conductor is defined by

dq
dt
By convention, the direction of electric current is·taken as the direction in which positive charge carriers
would move.
2. Current density : Current density is rel,ated to the current as

'. = fJ-dA
I ,
where dA is a vector perpendicula, to a surface element of area dA, and the integral is taken over any
surface cutting across the conductor. J has the same direction as the velocity of the moving positive
charges.
3. Drift speed : When an electric field E is established in a conductor, the charge carriers (assumed
positive) acquire a drift speed vdin the direction of £; the velocity is related to current density j as
'
J = (ne)vd
4. Resistance of a conductor: If Vis the p.d. applied across the conductor and i is the corresponding
current, then its resistance is defined as :
V

Similarly we can define resistivity p and conductivity cr of a material :


·'•J,,
E
p
cr
=J
Also E Jp
Ifn is the number of free electrons per uni( volume and 'tis the relaxation time, then
2m
.p = -2-
ne 't
The resistance R of a conducting wire of length L and uniform cross-section A is
R = pl_
A
5. Change in p or R with temperature: lfR0 is the resistance ofa wire at temperature 0°C, then resistance
at any temperature t is

R1 ; Ro(i+at)
'
where a is the temperature coefficient ofresistatice. It can be defined as

_I_ dR1
Ro dt
!- ---- - . ·- - - -----;- ----REVl~f~-; (F;;~~~~& IMPO;;,;;T-P;;~f- -~
-----. --- --····------- - --- - - ..,..J· .---- ---- ----- - - - - - - - -
6. Ohm's law: Under given physical conditions the cur;.(nt i produced in the conductor is proportional to
the applied potential difference across the conductor.
7. Combination of resistances: I I

(i) In series : R R1 +-R2+ ...


I ·J
(ii) In parallel: -+-+._,
R . R1 R2
8. Changing the size of the resistor:
If the conductor is stretched from length etoe', then its resistance changes from R to R' as

In terms of radius of cross-section of the conductor: 1

9. EMF of the cell: It is the work done in circulating unit charge in the entire circuit Thus

dW
dq
Potential difference across the terminals of the cell
V = S-ir
where r is the internal resistance of the cell.
In case when cell is charging, the p.d. across the terminals of the cell

V = S+ir-
10. Combination of cells :
(i) In series: If 11 identical cells each of emfs and internal resistance rare connected in series, then

current in external resistor R

[m~!R]
In case when nr < <R,

115-
i =
R
(ii) In parallel : = s
[~+R]
n , d

lncaseR<<nr,

= 115-.
r
------------- -· .
·. ELECTRIC11Y &.MAGNETISM ' .• .·.. ·_ ,
----·~;_ . . . ~ - " ·----.--·~------ - ·---- ----- --·· - - __ - -· ,

(iii) Series-parallel: lfn cells are connected in series and m cel[s_are in parallel, then

r;, = [ ~ ]
nr
-+R
m
For maximum current

nr
~ = R.
m

11. In a series of five identical cells each of emf Sand internal resistance r, if one of the cells is wrongly
connected, then net emf

Snct = 3s and ,.net = Sr


Similarly in a series ofn cells ifm cells are wrongly connected, then

snei = (n-2m)s andrn,,=nr

12. If two cells of emfs St and s2 and internal resistances r 1


and r 2 are connected in· parallel, then p.d. across the
combination

13. Circuit analysis by Kirchhoff's laws: Fig. 10./4


(i) Junction rule: At any junction, the algebraic sum of currents is equal-to zero. i.e.,

Li = O
(ii) Loop rule: In a close loop, the algebraic sum of p.d. across all circuit elements is equal to zero, i.e.,
LV = 0.
For a circuit having resistors and cells
LS+LiR 0

~ -iR
Sign conventions : ----i- ~
~"
Fig. 10.15
14. Wheatstone bridge: It is the combination of four resistances
in the form of bridge. For the balanced bridge with the resistors
P, Q,R andS
p R
Q r- S or

The equivalent resistance across the t~,rminals of the cell · Fig. 10.16

I I
--+--
R,q P+Q R+S
15. Delta-star transformation : A combination of three resistors in the form of delta can be effectively
converted into star. A delta of three resistors R 1, R2 and R3 is equivalent to a star with three resistors
Rr2, R 13 and R23 , where ,, 111

[ R1R2 ]
R12 R1+R2+R3

[ R1R3 ]
Rll R1 +R2+R3

[ R2R3 ]
R23 R1 +R2 +R3
/j
B
11, R,i

A
.·! II_, =>
R, R,;
R,3
C
C ,11.

(a) Delta of three resistors ' (b) Star of three resistors


Fig. 10.17
For any two junctions RAB in delta is equal to RAB in star, similarly RAC and RBc-
16. Metre bridge: It is used to find unknown resistance. If ebe the balanced length and R is the known
resistance, then unknown resistance

S R(~)- = £ ..,•,
17. Potentiometer: It is an ideal device of finding emfof.the cells, internal resistance of the cell etc.
'J

c-

e
Fig. 10.18
If R0 is the resistance of the potentiometer wire, then emf of the cell
-'i
e = ~To"' ,;
e
where is the balancing length and £0 is the length of the potentiometer wire.
Internal resistance

,. = R( C1;/2).
where eI and £2 are the balancing lengths without R and with R.
~-~---------------~----------------"""-
~ ELEf:'IRIC11Y & MA<;NETISM
-------,rI
18. Ammeter: Galvanometer of resistance G and full scale deflection current ig can be converted into an
ammeter of range i by connecting a shunt of resistance S, such that

;--
s
S+G

SG
Resistance of ammeter
S+G
19. Voltmeter: A galvanometer of resistance G and full scale deflection current ig can be converted into a
voltmeter of range V by connecting a large resistance R0 in series. such that

CHAPTER 4 : THERMAL AND CHEMICAL EFFECTS OF CURRENT

I. The rate of energy transfer is called power in an electrical device with a potential difference Vis
P = Vi.
If the device is a resistor, then we can write

v2
p PR=-
R
2. Thermal energy produced in time I
Q = Pi= Vit.
'
In a resistor, electrical potential energy is converted to internal thermal energy via collisions between
charge carriers and atoms.
3. kW-h: It is the commercial unit ofelectrical energy.
lkW-h = 3.6x J06J.
I kW-his known as I unit of electrical energy.

Power(W) x time(h)]
Numberofunits = [ I000

Cost of electricity= number of units x cost of one unit


4. Maximum power theorem :
Power generated in the resistor R

:
P= _I;_ R
~ ', ( R+r ) .

For maximum power

' 1;2
r R and p max = 4R Fig. 10.19
I

---- ------ ··-------,-------, , ... -·


I J
REVISION (FORMULAE & IMPORTNATPOINTS)

5. Electrical appliances : JI d1 1, "

The resistance of any electrical appliance of power ?design and Vdesign can be obtained by

2
vdesign
R
Pdesign

Pdesign
The allowable current
VdeSigh

6. In houses the electrical appliances are connected in parallel. If appliances of powers P 1, P , ... are
2
connected in parallel across the design voltage V, then total power consumed
p P1+P2+ ...
In series :
· t_r ,
I I
-+~+ ...
p 11 P2
7. Fuse wire : In a fuse wire, the change in its temperature A0 for the constant current i is given by

A0

For the given material of fuse wire ; 2 O<: r3.


8. Chemical effect of direct current:
0

(i) Faraday's I law: The a~ount of substance deposited Jr liberated on any electrode is proportional
to the charge flows in the electrolyte solution ..Thus ·
m = ·zq=zit,
where z is called electrochemical equivalent: " ·I
(ii) Faraday's II law : If same amount of charge flows in two different electrolyte solutions., then the
ratio of amounts of substances deposited is proportional to their chemical equivalent. Thus

~ .'.!:!._
m2 W2

9. Faraday constant:

w
F (Faraday constant)
z
IF 965OOC/eq.
10. Seebeck effect: The conversion of thermal energy into ele2trical energy is known as Seebeck effect.
The emf across the junctions of two different metals is given by

bt 2
at+-
2 ,

where a and bare Seebeck's constants.


11. Neutral temperature: It is constant for any thermocouple. Neutral temperature

le +t;
t,,
2

a
Also t,,
b

The maximum value of~ will occuratt,,, which is

~max = b
12. Law of intermediate metal: For thermocouples made of A, B; B, C andA,C

~AB +~BC = ~AC

13. Law ofintermediate temperature: For any thermocouple

14. Thermoelectric power or Seebeck coefficient

S = d~ =a+bt.
dt
15. Peltier coefficient:

1t = !>H
!>Q
=T(d~)-
dt

16. Thomson coefficient :

a !>H
!>Q!>t
(dS)
=- -T dt

CHAPTER 5 : MAGNETIC FORCE ON MOVING CHARGES AND CONDUCTOR

I. Force on a moving charge : The force on a charge q moving with velocity v in a magnetic field is given
by
F = qvBsin0,
and the direction offorcb is given by Fleming left hand rule.
In vector notation, we can write
2. Lorentz force : When a charged particle moves through a region of space having both £ and B field.
the net force on the particle called Lorentz force and is given by

F = q(l+vxa).
3. In magnetic field speed and hence kinetic energy of the charged particle remain constant.
4. When charged particle is projected perpendicular to the magnetic field, its path will be circular.
The radius of path

mv
r
qB

The time to complete the circle

21tm
T =
qB

5. When charged particle is projected at an angle e with the magnetic field, its path will be helical. The
radius of path
v
mvsin8
r
qB
q
/ I
1/
21tmvcose
and pitch , p .= Fig. 10.20
qB

6. A charged particle q enters normally in a uniform magnetic field ft. The magnetic field extends to a
distance x, which is less than or equal to the radius of the path, then deviation angle e is given by

X
sine
r
7. Hall effect: Hall potential is given by

iB
nee.
8. Mass spectrograph: For two isotopes of mass numbers m 1 and m2,

~ .1..
m2 r2

9. Cylotron : If Vis the potential and f is the frequency of the AC source used in cylotron, then K.E. ofthe
particle q will be
K = 2/qV.
I 0. Magnetic force on current carrying conductor:
;

F = Bi£ sine,

and the direction of force can be obtained by Flemings left hand rule. In vector notation F = if x B.
I 1. Force on curved conductor:

Q
F Jidtx s d!/
p/ H
p

12. Torque on a current loop : Fig. /0.21

'f MxB.
where M = Ni A, is called magnetic moment of the loop.
13. Magnetic moment ofan electron moving in a circle of radius r with speed v

evr
M =
2

CHAPTER 6 : MAGNETIC EFFECT OF CURRENT

I. Magnetic field of moving charge:

µo qvsin0
B
4lt ,. 2

and its direction can be obtained by right hand screw rule.


2. The magnetic force between two moving charges q 1 and q 2 is given by

µo c1iq2·v1v2

3. Biot-Savart law:
Fm 47t ,.2
",L,._th
Fig. 10.22
The magnetic field due to the current element d f. at any point Pat a distance i' is given by

dB = µ 0 idf.xr
41t ,-3 .
4. Magnetic field due to a current carrying wire:

µo -; (s1µ0 . e )
.. n +,sm
B = - 1 2 .
4lt r
Magnetic field due to a long straight wire

B µoi , at the middle region of the wire.


21tr

µoi
and B = - at the end of the wire.
41tr
5. The force between two paraUel current carrying wires :

dF µo i1i2
df 21t r
6. Magnetic field at the centre of the circular current carrying coil

µ Ni
B = -0-
2a
The magnetic field on the axis of the circular current carrying coil

B
2 2)3/2
2 ( a +x

7. Magnetic field due to a solenoid

B µ0-
= - ni [ cos0 -cos8 ]
1 2
2
Magnetic field due to a long solenoid at its centre
B = µ0ni.
Magnetic field ofa toroid ofradius R

µoNi
B
21tR
8. Ampere's law: It states that

js-de = µo ;in·

The line integral in this equation is evaluated around a closed loop called Amperian loop. The current i;n
is the net current encircled by the loop.
9. The magnetic field inside a current carrying tube is zero.
10. The magnetic field due to a magnetic dipole on its axis

CHAPTER 7 : PERMANENT MAGNET AND MAGNETIC PROPERTIES OF SUBSTANCES.

1. Bar magnet: Ifm is the magnetic charge and f is the length of the magnet, then magnetic moment of the
magnet

M me
2. Coulomb's law in magnetism:
The magnetic force between two magnetic charges m I and m 2 placed at a separation r is given by

F = µo m1m2
4rr r2
------- ------ ------ - ... - --
/.• . ELECTRICITY &,MAGNETISM
",,;. •...... _. __________ ::i::.._..:, ----~- ·-· • d

3. Magnetic field due to a magnetic charge III at a distance r

4. Magnetic field due to a short magnetic dipole Mat a distance r

and tanu
B !::!!. ~ J3cos 2 0+ I
41t r·1

tan0
2'"
Fig. 10.23
.I/

2
(i) On the axis of the dipole, 0 = 0

!::Q_ 2M
B 41t ,-3

(ii) On the equator. 0 = 90°

µo M
B
. 41t ,.3

5. Torque on a magnet in magnetic field

i MxB
6. Work done in increasing angle from 9t to 92
W = MB(cos0 1--cos02).
7. The potential energy of magnet in magnetic field
V = -MBcose.
8. Elements of earth magnetic field:
(i) Angle of declination"(<!>): Angle between geographical meridian and magnetic meridian is known as
angle of declination.
(ii) Angle of dip (0) : The angle made by dip needle with the horizontal is known as angle of dip. It is
zero at the equator and 90° at the poles.
(iii) Horizontal component of earth magnetic field : If B is the resultant magnetic field at any place, then
horizontal component /
BH = Bcos0.
9. True dip and apparent dip: lf0 is the true dip and 0' is the apparent dip in a plane making angle u with the
meridian plane, then

tan0
tan0' =
cos a
10. Moving coil galvanometer: lfi is the current in galvanometer, then

C
--((
NBA
REVlSlON (FORMULAE & JMPORTNAT POINTS)
11. Tangent galvanometer: lf0 is the reading of the galvanometer, then

2RBH) e
(-µoN- tan,

2RBH)
where ( µoN is called reduction factor.

12. Vibration magnetometer : The time period of magnet of magnetometer

For magnets of magnetic moments M 1 and M2

r/ + 7i2
' '
T2--1i-

13. Gauss's law in magnetism:

14. Magnetic properties of materials:


(i) Intensity of magnetisation

M
I V
(ii) Intensity of magnetising tield

Bo
H -.AlsoH=ni.
µo

(iii) Magnetic permeability

B Bo
µ - and µo = -
H H
Also relative permeability

(iv) If X is the susceptibility, then

µ, = J+x.
15. Diamagnetic: It is found in solids, liquids and gases. Ex. Ag, Au, Bi, Cu, H2O, NaCl, Sb etc.
16. Paramagnetic: It is found in solids, liquids and gases. Ex. Al, CuCl 2, Na, 0 2, Mn, Pt etc.
17. Ferromagnetic: It is normally found in solids. Ex. Co, Fe, Ni, Cd, Fe3O4 etc.
Eml---EtitTR~CI;,
~ - -
&MA. . ·G.·...NETISM
<
:
;;,
----·-.- --;1 :*'-•-1

'"CHAPTER 8 : ELECTROMAGNETIC INDUCTION

I. Magnetic flux: Magnetic flux of the magnetic field B through the normal area A is
<!> 8 = 81.A=BAcose

2. Faraday's law: Whenever there is change in magnetic flux linked with circuit, there induces an emf in the
circuit The rate of change of magnetic flux is equal to the induced emf. Thus

here negative sign indicates that induced emf opposes the change in flux.

Induced charge in time /11, /1Q = ~<I>

3. Motional emf: When a metallic conductor moves in a magnetic field B with velocity v, an emfinduces
across its ends. The induced emf
e = Bvf sin0.
In general, it can be written as:

e = f(vxiJ),ic
4. When a metallic conductor is rotated in normal magnetic field about its one ends, the induced emf
across the ends

2
Brof
e =
2

5. Induced electric field_: If£,, is the induced electric field, then by Faraday's law

e = ,(, £,de = - d<l>e


'f dr

6. Induced emf in a coil rotating in uniform magnetic field


e = NBArosinrot
7. Self induction: The induced emf in the coil itself due to change in current in it is called selfinduction. For
the coil of N turns
N<j> 3 = Li,
where Lis called self inductance.
8. Self induction of circular coil of N turns and radius r

L
- - ·· ·· · · -:·-RE-;;~10N(Fo;;.,~~-&iMP~;{~~TP~~~,--- --~
···--·------ - -· .. ··--·-· ------··-·--····----- ,· {~7- ··-·-·-·---· ...
9. Energy stored in an inductor: Energy in the inductor stores due_tb magnetic field•in it. For any current
'I in the inductor, the energy stored

If Bis the magnetic field in the coil, then

B2
u = --xVol.
2µo
10. RL~DC circuit: For a circuit with time constant't', the growing current iin circuit at any time t

,. = . (1 -e _, '·)
10
R !.

The decay current in the circuit, with initial current ;0


. _, /t
i = ,0 e Fig. /0.24

I I. LC-os_cillations : When a capacitor C with a charge q is connected to an inductor L, the energy of the
circuit oscillates between C and L. If co is the angular frequency of oscillations, then

d2
__ q +co 2q o
dr 2 '

where co

Also T = 2n.fTc.
Electrical energy of capacitor will store in inductor aft.er time T/4 and vice-versa.
12. Mutual inductance: The induced emf in the second coil due to change in current in first coil, is called
mutual induction. If Mis the mutual inductance between the coils, then
N2<i>21 Mi1,
or
13. Mutual-induction between two circular coils ofradii R I and R2 with turns N 1and N 2 is

14. If L 1andL 2 are the self inductances.of two coils, then mutualinduction between them
15. Combination ofinductors :
(i) In series : L

(ii) In parallel:
L

16. AC generator: lfi0 be the maximum current (current amplitude), then


i = i0sincot,

NBAro
where io ---
R
17. Transformer: For the transformer with the turns NP and N, in the primary and secondary coils, the ratio
of output and input potentials

For the ideal transformer

CHAPTER 9: AC AND EM-WAVES


I. Alternating current (AC) : Alternating current is one whose direction changes periodically; i = i0 sinOJl
is the sinusoidal alternating current.
2. RMS value of AC: For sinusoidal AC, i= i0 sinW, the RMS value

io
,fj_ = 0.707 i0.

Vo
Similarly ,fj_ = 0. 707 V0.

For square wave AC, irms = i0 .


3. Three simple circuits :
(i) Circuit having resistor only : The alternating potential difference across a resistor has amplitude
VR = iR; the current is in phase with the potential difference.

I
(ii) Circuit having capacitor only: Ve =iXc in which Xe ;:; roC is the capacitive reactance: the current

here leads the potential by 90°.


(iii) Circuit having inductor only: VL = iX0 in which XL= rol is the inductive reactance; the current here
lags the potential by 90°.
4. Impedance and admittance: The total resistance ofAC circuit is called impedance Z, and reciprocal of
. I
impedance is called admittanceA. ThusA = z.
5. · In India domestic ·supply is 220 V, 50 Hz. Thus'·
.·, '. \• ti _.f: I j ,: ..., [ ' ,•
V0 i20 ,/2 V, a~d

V 220 ',fj_ sin I ooiu.


6. Series RL C circuit: For a series RLC circuit,

and tan<!> =
(rol-fc)
R

current

7. . Resonance : The current amplitude ;0 in a series LRC circuit driven by a sinusoidal external emf is a
t ,•, _ , ~ •, t• ,;-! .1. • , I ' , • .• •

maximum ( i = ~) ~hen the drivi.ng angular frequency ro equals the natural angular frequency ffio

(resonance frequency) of the circuit. Then Xe =XL, <I>= 0, and the current is in phase with ttie potential.

'(17
mo = vu·
8. Power in AC circuits : In a series RLC circuit, the average power Pav equal to the production rate of
- thermal energy in the resistor :
,,

R
Here cos<!> is called power factor and is equal to z. Its value cos<!>~ I. For purely inductive or capacitive

circuit cos0 = 0 and so Pav= 0.


9. ·• EM-waveii : An· EM-wave con~ists of oscillating electric and magnetic fields. An EM-wave travelling
along an x-axis has an electric field E and a magnetic field jj with magnitudes which depend on x and
I:

E E0 sin(kx-rot)

and B
Electric field induces the magnetic field and vice-versa. The speed of EM-waves is c, which can be
written as

E I
C =
B .= )µo Eo
10. Maxwell's equations : Maxwell discovered that all the basic principles,of electromagnetism can be
formulated in terms of four fundamental equations, called Maxwell's equations. These are:

q
(i) Gauss's law for electricity .-pE•dA ;;:=,,-
Eo
:n1• •~, f'~•J/' •

(ii) Gauss's law for magnetis_m pB•dA 0

(iii) Faraday's law of induction pE•df -d~o


dt

(iv) Ampere's law

11. Energy flow: The rate perunit area at which energy is transported via an electromagnetic wave is given
by the Pointing vector ,, .

S'

, • ,, • • • , r ,, · , ·~· i'~ , i,· · /l r•r.. ·.H~· 1·, .• r


12. Intensity of EM-wave: The time averaged rate per unit area at which energy is transported, is called the
intensity of wave : i
_. •' ) )''.! I' ,:'• ' I
t r
I 2
' ,, . ' I = -Eo-Eo,C.
. '2 ,.
The intensity of the waves at distance r from a point source of power Pis

• • _,ri,.
13. Radiation pressure : When a surface intercepts electrqmagnetic radiation, a force is exerted on the
surface. If the radiation is totally absorbed by the surface, the force is
, iA
F = -
C

where A is the area of the surface perpendicular to \he p~th of the radiatio,n.
reflected back along its original path, the force 'is , ,, . ' ' ' ' ,
,r the, radiation_
, ' . ,, .
is totally,
'

2/A
F
C

• I<., ,,
0
0
0
0
0


,11
jl,
'•. .

0 Chapter
0
0 IIJ E'l'etcbro.s,tot.ics
I. Let charge q be placed at a distance x from smaller charge. Then
4e q e
For the equilibrium of this charge we have
-r-x--x-
(4e)q qe
--,
(1-xJ- x'
I
X Ans.
3

2. For the equilibrium of charge q, we have x =~ , and for the equilibrium of charge e, we have

(4e)e qe 4e q e
- + - -2 0
a2 (a/3) -a-x--x-
4e
9 . Ans.

eq (2e)iJ
3. For the equilibrium of the charge q
a' (a+ 1) 2

a --i- =
(v2- I)
l(.fi + I) Ans.
4. For small 8, we have
'·.tan9 sine
!
a
F
'e
2l
For the equilibrium of either ball, we·c·an ~;iti:,
I n,g
e' a
02
mgtan9""'mg l ... (i)°
2
On contact the charge e still remaining on one of the balls will be equally distributed between both balls and condition for
the new equilibrium of the balls will be · ·
b
mgtan0'=mg l ... (ii)
2

Hence from (i) and (ii), we get b Am;.


5. The time period of oscillation in the absence of the field,

. r, 2x II
~i ... (i) .
The period of oscillation after the ball ll~d the capacitor have been charged (upper plaie and 1he ·ball had charges of the
same sign),

T2 2nff) •.• (ii)

2
Squaring (i) and (ii), we get F ·[1i Tz2,
-T/]mi.=- 3mg
If sign of the charge of the ball is reversed,

T Ans.

6. The potential
' '
.'·l, i

,,. II
5 9 ]
[ 3x10
2x9XfoYx _( )
. 0.1
300 V Ans.

V'
.!'..= 300 150 V.
k 2
7. Ir cr is the charge density on each of_ the1-:sphere, then
Q q,+q2 = (4m-2 +4nR2 )a
The work needed to transfer one positive charge to the common centre of the system is equal to the potential Vat the
centre of the system.

V
_l_[~+q2]
41te 0 r R
After solving above equations, we get
,,.,
r 2V R 2V
R+r'
q2=--
R+r

and Q [
r2+
r+R
R']v. Ans.

I ,j.
8, The ntlmber Of molecules in IOO g of water
Each molecule of water has JO electrons. so total charge
0
q 6.02xJ023 x\ °~J0xl.6X]0-l 9
8
5.35xl06C
q'.
The force. F
41te 0 ·;r
( ' •)'
9 xJO'x 5.35x;O
0.1
2.56xJ0 25 N Ans,

___ t_ ' rl'

9. (a) The force between the balls 4JtEo ,-2


,\

9x Io' x bx I o-1J'
0.052
0.144 N
2.5 I mg
(b) sine 5D = 20 =0.05

and cos8 ~l-sin28 =0.99'


All the string the net force on lhe ball is zero. Perpendicular to the string
F Fecos9-mgsin~,
0. J44x0.99-0. l x9.8x 0.05
::: 0.095 N Ans.
(c) The tension in the string, T mgcos8+F;_.sin8
\ 0. Jx9.8x0.99+0.144x0.05
0.98N
(d) The acceleration of each ball
F ·o.095
a - = - - = 0.95mis2
m 0.1

JO. sine
20
-scm-
The force between the balls F

4xio-' N mg 3cm
For the equilibrium of the ball, Tsin8 F

F · 4xl0-3 .
T sin
8
= -.-1- = ~.08 N Ans.
20 ..
and Tcos8 mg
0.08cos8
m 9.8 = 8.2 g. Ans.
11. As the charge has symmetric distribution about the centre. of the ·hexagon, so net electric field at that point must be zero.
12. The charge on th~ element

dq
2,ta
. ,, .. ---.::
dL

The electric field due to the charge of the element at the 'cehtre, a Q

_l_dq I [QdL]
41teo a2 = 41teo· 2rta3

The electric field at the centre of the circular loop, E = 0. If £ 2 be the electric field Of
the remaining wire loop. then
l; l 1 +E2 =o
,
or

Ans.
·'"' \ . r , ,
B

13.(a) We can write -f E·di'


A

(4, 2)
.- f 2oi-(dxi+~v])
(0,0)

-201,1:
~ 80.V ~, Ans,
Do the other parts similarly.
14. The elec~ric flux $ is given by
$ 1 =. E-A
(¾Eoi +~E0] }(o.2i)
u
3
-x0.2x£0
5

~xD.2x2xl03 ~ 240N-~ 2/C.


5
15. Using Gauss's law, we have
........... / ../'x ......
,I /x

2EA

.,. ·E
16. (a) The force between the charges,
I q1q2
F
4ne 0 [(r-t)+,/ki]'
___ qlq2
(b) The force in air ... (i)
41teo r2
and force in the presence of dielectric

... (ii)

... i
9
Given.
4
Ans.
. of angular width d$ at $. The
17. (a) Take an element k charge-
1

dq 1.(Rd$)=1.0 cos$Rd$ - + +
The electric field due to this element at the centre of the ring· +
+
_ _ dq +
4tte 0 -R_ ·_
+
The x componenls of this field get added while y components ca~~~8.Cii Other.-- =. ___ _ ± __
Thus the magnitude of the nel electric field dE
+
n/2
E 4 f
0
dEcos$
- - +++
+
+

(_1 )1·(1.ocos$Rd$)cos$
tt/2
4
4tteo R2
0 • •

tt/2

~
4ne R
Jcos'$d$
0
0

Ans.
X
(b) The electric field due lo charge dq. al P
I I.Rd$
dE
4•eo (R 2 +r2 )
The resultant of two diametrically opposite poinl is 2dE cos$sin 9
So resullant field

f
tt/2

2x2 dEcos$sin9
0

1.oR'
After simplifying, we get E, Ans.
4e 0 (R 2 +r2 )31'
18. The position vector of P y
p
~ (Ji+ j+.i}m (3, I, I)
and ~ J3 2 +1 2 +1 2 Jilm .r,
The position vector Of· i} relative to A
Q
i', (Ji+ j+f)-2i = (i+ }+k)m Q
X
0 A(2, 0, 0)
and 'i J1 2 +1 2 +1 2 = ,jj m
The net electric field at P
E = E1+E2
_·1
47tEo
-[Q, (Ji+ j+k) + Q(i + j+i)]
llJ/2 33/2
Ex to be zero, we have
I [Qx(J) Q, xi]
4neo ~ + 3lt~ 0
-[, ;i,~'~'. ]c 0
-0

19. The electric field due to the thread at a distance r from the end
E -E) +E,}
l.
where E=-- ' ). '
.,. 4ne 0 R ++++++++
,,
The flux of this field through the area. dA = 2wdr is

. (,-E..J+E_,.:i),dA

I
,'""J(-E,J+E,})-(2,u-drH-n
0
' get
After substituting the values and simplifying,' we

Ans.

20, The charge density of each half, A = .JL =2Q


n/2 rt + +
The charge on the element +
dq+
2Qd9
dq + ''
X
Its electric field at P +
+ dE
dE, I )dq
( '41tEo ·R2
The resultant field of the two elements is
' 2d£sin9
The resultant field of the semicircle
,12
E J 2d£sin8
0

[2Qd8)
J-
, 12 '/
1
2 - 7t2 sine
41teo R
0

Q
-,--,.
7t Eo R-
, Ans. .

2 ~a) For the equilibrium of the ball, we _have


Tcos60" mg 60
10cm
mg O.OIXIO
T
cos60° =--,-
1-~----0.--Fc=Eq
./Icm
2 mg
Also Tsin60°
,
,, or 0,2x ,/j Ex4XI0-=-6
2
'.
E 4.33x l0 4, Vim.
2Jt..
(b) The time period of the oscillatiolls is given by

n~
T 0 net
, '

where

4
102 +((4.33x 10 )x4x10-• )'
0.01
20 m/s 2

T {cu =0.45 s Ans.


2n
v20
22. The x and y components of electron's velocity are

Vx = v0 cos 45° }ix106 =3..fix106 mis

and ''r = v0 sin 45° 3..fi X 106 mis


-Ee 2xl03 x l.6x10- 19
Acceleration, ax=O, and ar=--
. m 9.lxJ0-31

-0.35x10 15 m/s 2
Suppose 1 is the time to hit the plate, then

y=d

or 0.02 3..fix1061+l(--0.Jsx10"), 2 ... (i)


2
If x is the distance, then
x=v.~ ,, = 3Jzx106 I ... (ii)
After solving above equations, we get x = 2.72 cm. _
23. By Gauss's law, we have ' ·

ft-dA q;.
•o
For the closed surface s,,qin q
" S2,qin -q

S3,qin q

S5,q;, = q

24.
-After-subsiitllliiig the values, we can get the required answers.
.
The· electric field. bet~een the cYlinders, E ~
;.
2n:eo -,. • which is only due to inner cylinder. For the motion of th~
Positron. wid1ave.
mv2 ·
Eq
r
or mv1 Eqr
Eqr Aqr _ Aq
K=!mv 2 2 = 41te 0 r - 4n;e 0
2
· AftCr subs~ituting th~ values, w.e can get
K ·210 eV. Ans.
25. (a) By using Gauss's law:, we have

ft-dA %,
Eo
q,.J
For ,. < a qin
7 I

qr3
or 'Ex4w 2
7
.• I qr
£
4JtEo UJ
(b) Fora<r<h:

'!in q. ..,
£ ·- ~ 41te
0
q
;r
(c) For b<r<c: . '

q
q[1n(,-'-b3)]
'!in
[1n(c3~h')]
After substituting the value of qin in Gauss's law, we get

£ An.r.
26. (a) The potential at the centre of th'e circle

' ..
R

(b) The potential at point P, a distance, r=JR2+z2

(-5Q)·
V Ans.
4ne 0 JR2+z2 ·
2 7. For the equilibrium of q 1,
0

or kq,q2 (i'2-ii) + kq,q, (ii-i\)


0
I- -e
r2 -'ll 1-'i -r3-1'
For the equilibrium of q3, we have

~I +~2 0

kq,q, (ii -i\) + kq,q3 (,, -i\)


or - 0
- -1'
I~-1) 1-,;-r,-1'
After solving above equations, we can get

~ns.
(..[,i; + [<i;)'
28. The given system of charges is equivalent to, two electric dipoles, each of the moment, P,= qx2f., placed perpendicular
to each other. The field of the dipole at a distance .t from it's centre is given by
p
E
4ne 0 (e' +x2)312
,fj_ p
Thus resultant field, E, 312
41teo ( ;2 + ...-2 )

hx2ql qi
Ans.
2 2)312
41teo ( t +.t
29. The electric field due to the charged thread has two
equal components :
l. ..
E =E.=--.
x .1 4ne R +
0 '
The force on the ring due to Er component becomes +
zero. So the force due to E., cOmp?nent is
+
+
+ Ans.
+ + +
30. (a) The electric field due to the straight parts of the thread becomes zero.
Because each part produces electric field; Ex & E.1,, which will be
cancelled out by the field of the other part. So electric field at O is only
due to the curved part. It is given .by

E
_I _<J__[Sin8] E !Er
········i\i!:-~·-E
4neo R 2 8 .
.
.

l.
(b) The 'electric field due to straight parts of the thread is 2E where Ev=--.
.,.. · .. 4ne
0
The electric field due to curved part,\.

E _I .!L(sin8)
R2
41teo 8

. n
,._D
___
A,_~ __
sm-2
4ne 0 R2 7t
2
n
27tEo R
Cle·arly the resultant field at O due to whole thread becomes ·zero. q
32. The initial potential energy of the system

U;
3xl
4ne 0
[q'] 3q2
7 =4ne Im Im
0
Final -ener~y of the system

_ I
47tEo
[q)+
2
q2 q2]
+ 0.5 0.5
q----~-----49
r= Im

sq'
47tEo
The work done in the process. w U.r-U;

5q2
41te 0
If I is the· required time, then Pt w

w_~
P- 103

2
2x9xl09 x(o.l)
1o'
l.8xl05 s. Ans.
33. For minimum potential energy; the charges of larger magnitudes should be placed farther. Let charge q is placed at a
diStance x from 2q. Thus potential ene'i-gy of the system.

U _1_[(2q)q + q(Sq) + (2q) (Sq)]


4ne 0 x (9-x) 9

dU 2q q
U to be minimum, ~=O
:· - x - - - : - - ( 9 - . r ) -
X 3 cm. Ans.
Electric field at the position of q
l: l:, +l:,
I [
4neo (Jx I 0-2 )'
2q
8q 2
(6xio-2 ) -
J-o
34. The FBD of the device shown in figure.: At lo,west position of the ball.

mvA' ''• B
(T,+F,)-mg ... (i)
I
mg
,;
At highest posi_tion of the ball
r,
0
mv;,
(Ta +'mg)+F, ,;
I ,r,,
To just complete the circle. T8 =0 •.,
A mg
mv~
mg-Fe ... (ii)
I

q'
where F,
41teo f
I 2 I'
Also zmvA
2mv~+mg(U) ...(iii)

· After solving above equations and su~stituting the values, '!'e get

. 35. ·. The FBD is shown· in figure .


VA - . 7.62 m'ts· Ans.

The normal -re'action


N
N, mgcos8+£qsin8
The acceleration
·pnet
a
11!
.· [ mgsin8-(~~qEcos8)] mg
...(iii)
Atler solving equations (i) and (ii)."and substituting the values, we get
a 2.13 m/s 2
. I
l Distance moved, s --=2m
sin.30°
.• I , .
By si;:cond equation of motion ,0+-ur
. 2

F§=l'.345s. Aiu.

36. Let electron is at a distance .r from the centre of the ring. The force on the eleclron,
F -E<•

I q.r ]
- [ _4ne_o (R~ + x2 )J/2 e
'
Forx<<R. ' +
+
F ·. ~[ 4•~o ~; J, • +
+

f_;,_·_1_· ~ ( - r )
Acceleration,
" m 41te 0 mR~
,
Coropare with ~tandard equation of SHM. (I= -oo-x. we get
qe_.
(J)
41te 0 mR3 ·
Ans.

37. The electric field due to any half semicircular rod is given _by

£
_I
41te 0
_g_
R2
[sin8]
8

.1
41teo
q
R~. [ "l
sm2,
~

1
2~ Eo ( 1t;2 )
The resultant elect-~ic fieid at lhe centre of the· circle

£, 2£,;,_I
1tEo
(_L)
7tR2
Ans.

38. The time perioa of O$cillatio~ of simple pendulum i~ given by

T 2•~ 0 f
ne,.
(a) In this case, "net (g-;),
T
·2·HJ
(b) In this case. "net ( g+ ~:}

T
2~
(g+~~)'
• 1t
Ans.
39. The given charge can be completely cove·red by six identiCal plates, so total· tlux·

~ !L
eo
_q_ Ans,
The tlux through .each plate
6e 0
40. (a) Forr> R:

Using Gauss's law, we have ft·dA. qin


•o
1.£
or Ex2w£
•o
l.
=> E
2ne 0 r
(b) For, r < R, qin 0
E 0. Ans.
41. Using Gauss's law, we have

ft.dA -. qi~
•o.
,.
J p 2nrldr
•o
0

2
. 21tt'r
· or E'X2rrrf. -
2eo, .

E
....ec..
·2Eo
42. (a) By Gauss's Jaw
qin
_eo

·Ex21VL - - (q:?)
_::<!_
E
2ne 0,Lr ' l , '

qin
(b)
•o

or Ex2rrrl !L
eo
__
q_
E Ans.
2~e 0 Lr
43. The electric field due to the·"given charge sheet,
E £conducting surface - Ehole

-~[i-J+
2€ 0
X
~R2 +z2
]

cr
Ans.
·'
44. (a) The t_otal charge on the sphere Q
R

J(P, i }m"dr
0

1tp,,i\ An.,·.

(h) By Gauss's law, we have

or Ex4nr1

,
p_,,,.-
or E
4Eo R =
Qrl
41te 0 R4 •

45. By Gauss's law, we have

or £x4ru·2

(q-2JtAa 2 ) 2JtAr 2
~--~+--
Eo Eo •

q_-2JtAa ] ~
2
or ,, E [ . , +
4ro·- 2e 0
For £ to be uniform, q-2rcAa1
q
or A --,, Ans.
21tt,-
46. The potential at any point P is given by
__ g
V
4tte 0 R
The result does not depend on the ·angle subtended. Electric field- at any point P is given by
£ _I JL [sin B]
41te R2 e
0
Clearly the E is maximum when 8 is small. so £ is minimum in c; it is zero.
47. If q be the required charge for the potential then
_ _ '!_
V
41te 0 R
or q 41te 0 VR
The charge emitted per second through the source
n (sx10 10 )ex0.40

If t is the required time, then


!1. = 41teo VR
n (sx1010 )ex0.40
After substituting the values, we get 700 µs. . Ans, •
--·

4_8. The magnitude of the forceS between two charges,


, I ., 2
I . - - q 2 ;x=3cm ... (i)
47tEo X .
The resultant electric force on any charge_particle
F · :2/coS30° ,[J f. ... (ii)
X
F
From the geometry. of the figure 1. : cos30°
y
mg
y

Also l'. sine


l
X

or Ti sine
.-I''
or sin9 J,/J = ,[J =tan 8
100 100 ... (iii)
For 'tti.e equilibrium of the charge, we have
Tsin9 .F
and Tcos8

or ... (iv)
' '
tan8
mg
.After solving above equations and substituting the .._values. we get
. '~ ·. ·1.0lxl0~7 C Ans.
50 .. If q be the charge on each ball, then force betwe~n
' them '
I q' .
F, ---;x=40cm
~rceo x2
For. the equilibrium of either ball, we have
' Tsin8 F,.
and Tcos8 "" mg
F,
tan8
mg.
20
mg
' '
. From ~he geometry, lane
After subst_ituting the values, we get
.. q · I.Olx!0-7 C Ans.
5 I. The rest_Oring torque on the di))ole so formed,
-P£sin8 ~ ,
' < PE(-8)
, •• I

a -=---
,/ I
Comparing with standard equation of SHM. we get
' .
(Pi_
0)
vT·
and T 2n-2nfT
w VP£

2n ~
(ql)E
~· 2nJML
2qE'

The required time, .. ,


~=~J:- Ans.
52. Mechanical energy at A = mechanical energy at B

I 'lr'f,
o, mgx0.30+-- ---
41te0 0.20 • q,
f
After substituting and simplifying. we get 0.20 m

t
,, 3.65 m/s
A
53. Given. R1=0.05m.R2 =0.09m.d=0.!2m.q=I0-6 C.
o.:HJm
Potential of the centre of first loop
l
V, I
41tEo
[ 'I
Fi;"+ Ji2 +R/·.
q l B

and v, I [ q q ]
41tEo R2 + Ji1 + R/·

I[["+
47tEo Hi Jil+R/- )["+
- R2 q q
JJ2+R,2 )I~
After substituting the values and simplit)'ing. we get
i-j-V2 7.08xl04 V. Ans.
54. The electric force on the dust particle
.F,, qE.
and viscous force, F,. 67tr111•.
'
For constant velocity.

or

'I

61txl.6x10-5 xSxJ0- 17 xo.02


6.28xl05

0.48x10- 17 c
48xl0- 19 C
If n be the number of electrons· on the dust particle, then

!1 = 48x10-19 30
11 Ans.
e J.6xl0- 19
55. (i) The restoring force
)'

F -[-1_.....'!'f!L_ __l_.....'!'f!L_]
41teo (a-x)2 41teo (a+x)2

_ qq0[-1___1_]
41teo (a-xf (a+x) 2
q

-(a+x)
X

(a-....,x)-+1
"
aqq0 (-x)
1te 0 ( a 2 -x~, )'

. . for x <<a, F ....!!'!Q_ (-x)


1teo a3

F qq,(-x)
- ....-The acceleratio~ of the charge %· ex
m m-TCeo a3
Compare with standard equation of SHM, ex= -cilx, we have

3
27t = l1t m1te 0 a
Time period,
ro qqo

T Ans.

(ii) In this case net force is not along the mean position, so the motion is not oscillatory.
5 6. The electric field at the centre of the ring is zero, so the force on the charged particle at the centre is zero. Hence if the
particle reaches at the centre of the ring, thell it will not return to P. For minimum value of velocity v, the speed at the
centre must be zero. ..
By conservation of mechanical, we have
(K +U)o

I 2 I (2rtR1.)q 0+-1__ (2nR1.)q


or -mv +
4ne 0 R
2 4ne 0 ~(,/JR)'+ ~2

or Am·.
~2~m·
5 7. By Gauss's law, we have
£
' '
+ +.+ + +"'+ +

or Ex2rcrl "+ + -1- + + + + +


)
). . I
E Ans.
2JtEo r

58. The initial potential energy· of the system Ui


_I_ [q1q2 + q2q3
47tEo I/ I)
+
q1q3
lj
l
99xI09 J

Final potential energy of the system


·_,_ [qlq2 + q2q3 + qlq]]
41teo r1 r1 ".r

'
9xl09 [2x3
0.5
+~+~]
0.5 0.5
198xl09 J.
Work done, w U1-U1

I,, .- 99xI09 J. Ans.

59. The charge on the shells are : cr;41ta 2, qs= -crx4n:a 2


1~.
and qc 2
crx4nC .

(i) The potential, v, _I_· [qA + qB + qc]


4ne 0 a b c
2 2 2
_I_ [41tcra _ 4mJb + 41tcrc ]
41te 0 a b c

\
',',;;': d{
;·;,;'

~(a-b+c)
Eo

v, _l_[qA+qB+qc]
41tEo b b C

2 2 2
_1_ [41tcra _ 41tcrb + 41tcrc ]
41tEo b b C

[a'
-b
-CJ ---+c
Eo
]
b
2

,,,,

Ve
_,l_[qA +!lJL+ qc]
4lieo ,. c · c c

2 2 2
_1_ [41tcr~ _ 41t~b + 41tcrc ]
41tEo C, C C

Ans.

(ii) For VA
C Ans.
60. If 10 be the distance of closest approach, then by conservation of momentum, we have
' \•
mv+O mv'+mv'

~~
V
,,,
2 _,.o_
By conservation of mechanical energy, we have
\
o+.!.mv2
2
I
ee
---'-+2 -mv'
41te 0 'b , 2
(I ')
After substituting the values and simplifying, we get
9.94x10- 13 m. Ans.
61. The electric field at a distance r from the centre of ring is

62. The potential, V -Jt-ar


-J a(yi +x})-(dxi +dy})
J t
-a (ydx, xdy)

-a Jd(xy)
-axy + cOnstant.

63. The potential, V -Jt-ar .


-J[2axyi +a(.?- y2 )j ].(dti +dy})

2
ay[~ -x2] + constant.
64. The induced charges on the hollow sphere are shown ·in figure.

Vi, 41teo
I [" q R2
-;-Hi+ q . l Am.

%1
65. (a) Using Gauss's law pE-dA Eo
,.
where 'l/in Jpx4m·2dr
(I E
,.
2
Jp0(1-j )x4:tr dr
(I

41tpo - - -
[,.' ,., }
l ,. ,., l
3 4R

41tpo - - -
3 4R
Thus Ex4ro· 2
Eo

£ OC(1-~}
3e 0 4R
R

For r~R-. C/in f


(I
Po ( I -j- }4m'dr
7tPoRJ
3
3
7tPoR
Ex41tr 2 3e 0 ,

3
PoR
or £ \2rJ e 0

' dl:.'
(b) For maximum value of £, - · = O.
dr

or ~[OC(1-~)]
dr Je 0 4R 0

,. ·~ -2R3
PoR
and Emax Am·.
9e 0

66. Using Gauss's law, we have !!In.


pE.dA Eo
qin
or Ex4ro·2 Eo
,. ,.
where %n I Poe-ar3 X41tr2dr
E

0
4xpo (I_ ,-m-' )
3n
47tpo (1-e-ar')
£x4nr::! 3ae 0

or
_P_11_( 1_e-m))
E
3ru· 2 e0 ·

p
For ar 3 >> L E ---,
3ae 0 ,.-

For ar3 << L E

Ans.

67. By using Gauss's law. we have ·

px.im.3
or
__
3_
£x4ro·::!
Eo

In position vector. it can be written as

i: ~
3Eo
The electric field due to charge of cavity at a distance' r.

i:·
The net electric field at any point inside the cavity
£net £-£•
~_pr'
3e 0 3Eo

_P_(r-r')
3e 0
pO.
or i: Je 0
[A,;;+,·=,] Ans,
68. If P 1 is the initial excess pressure inside the b_ubble, then
. 4T
P+_-.
,.
and volume of the bubble. v1=1m.3. When bubble is charged. its electric field becomes, E=i and outwrard pressure due

I '
to this is -e 0 £-. Thus if P~ is the final pressure. then
2

According to Boyle's law. we have


After solving above equations. we gel
e0 v 2R _
2 .
------- ,:
I. The equivalent capacitance, C 5+6=11µF.
The charge supplied by the battery, q CV
'.1i°xlO = II0µC. Ans.
2. (a) Let, VA-Ve Vj and Vc-V8 =V2
Vj +V2 50 ... (i)

and 4Vi
or V, ... (ii)

From equations (i) and (ii), we have V, ~V and V2


. 3
°
=_53 V.

50 V.
As
3

Similarly 50 V.
3

(b) The potential difference, != 0 '


3. For the equilibrium of the particle, we have
1
mg .c= Eq
E mg
q

'V
The potential difference across each capacitor is 2'
V
E
2d
V mg
So.
2d q

e0 A d=e 0 A
Given, C
d C

V mg
Now
2(e~A) q

2_e 0 A mg
or V
Cq
After substituting the values, we get V 43 mV. \ Ans.
4. The effective circuit is shown in figure ,JµF_· JµF JµF
a-j Tf----f-7 a-j ']_
2µFT
b---l~J -
21 T1µF
- b---l
3µFJ

•3µF • 3µF d 3µF


(i) (ii)
c-·
IL- _____.:...;,··--
".. ~ - - - - -
(a) Clearly the equivalent capacitance C . lµF.

(b) The p.d. across each of the capacitors nearest to a and h wiil be Joo V. So charge on each one, q=3x300=900µC.

Ans.

5. The given circuit is:

<XJ-
(a) The equivalent capacitance, C is
• I I I
---+---
C C1 +C2 C1 +C2
C· 5µF.
(b) Do this part-similarly.
c, c,
2C1C2 + C1C3 + C2C3]
6. (a) The equivalent capacitance is given by C [ C1 +C;'~ +2C3
2µF 2µF

Here, lµF, C2 = JµF and C3 = 4µF.


I 1 4µ/[µF
After substituting these values in above equation, we get lµF
I,
C !.!.µF
6 . f4µF
I I
(b) The effective circuit is : C !.!.µF
4 .
JµF lµF

7. (a) The given circuit can be redrawn as :

µY~F~->.--
4µF
I (r~ 8µF
ineffective capacitor

4µF
The equivalent capacitance of upper part of the circuit is
4 8
c, 3"i3= 4µ/;·
2µf' 4µF
The total capacitance, C 4+4=8µF.
(b) The effective circuit is :
4µF SµF
4 8 8 4
C -+-+-+-
3 3 3 3
4µF 8µF
SµF. f--l ---12-J
8. (a) The given circuit can be drawn as :
If Vi and V2 be the p.d. across 2µF and 4µF respectively then
Vi+V2 12
I'
and 2V,
V, SV, V2 =4V.

As Vi Vb-Va :. V0 -Vb=-8V.
I
(b) As both parts of the circuit are
symmetrical, so the charges on then may
be:
As the charge on the capacitor connected
between a and b is zero, so Va - Vb =D.
9. In close loop, abcda, 12+:G._2.!__6 0
2 4 6V
d -qll lfi
or <J1 -2q'!. 24 ... (i) I C

Now in close loop. ab/ea,


12V·
I
-(J,
-1 If,- b
12+ q, + (q, +q,) _24
0 "
2 I
or 2q1 +3q2 24 ... (ii) <'
~]v (qJq~Jl-lq•;. '11)
-24
From equations (i) and (ii). we get l/1 -µC.
7

If Vuh is the p.d. then vah -12+ C/'2. =-10.JV. Ans.


2
I 0. The equivalent circuit is shown in figure :
:1 I JOV
Let V, I~~ -Vu

and v, Vs-Ve
V1+V2 JO V ... (i)

and (2C) V, (2C)V2 x; 8 2C


.•. (ii) .·/ ,I I C
From equations (i) and (ii). we have J
Vj 5Vand V2 =5V.

As D. :. VA =5Vand Ve =-5V.

I I
I I. If C is the equivalent capacitance. then -+--
C' c C+C'·
C 0.618C.

12. The equivalent system of capacitors is shown in figure. 11C


If C is the equivalent capacitance, then "

,,C/'2
, "

Cl- = r I
1--
2
1
] 2C =2µF
Am;.
"
''C/4

"
"C/8
I 3. The successive solution is shown in figure. l

After simp!ifying, we get C'=0.38µF.


14. If the equivalent capacitance between A and B is independent of the number of sections, then

J_
ATl_
li__j- T:µF Jc
·n
or C .+ [ - -
0 4C]
4+C
C 4µF. A11s.
_ _ !!__
15. (a) Given. V
47tEo R
The electric field due to sphere. at a distance r from the centre of the sphere. (r > R)
I q R
E -4- 1 = V-:,,
7CEo ,.- ,.-

The electrical energy stored is given by u

or u 'sR -e
1 [VR]' x4m·-dr
0
. , ---:;-_
2 r- .
R
. ,
•1te 0 V-R. Am;.
(b) Energy stored in electric field of sphere beyond 2R.

U' -J2e1 [VR]' 0 ,. 2


2
x4m· dr
2R

Thus u
Eo A
16. (a) Initial capacitance. C d' and final capacitance

Eo A
C
d'
The charge flown. &, q'-q=C'V-CV

e0 Av[:.~~]
(8.86xl0- 12 )x20x\0-4x12x[-1- _ _l _ ]
2x10-3 lxl03

-\.06x\O-IOC.
(b) Energy absorbed by the battery W-W'
cv'-cv'
2
Eo AV [~- ;,]

8.86x10- 12 x2ox10-4x12 2 [ -1- ___l _]


lx!O-3 2x10-3

12.7xl0-IO J
( c) The energy stored

12.7XIO-IU J

and U' .!.cv 2


2

6.35x10- 10 J Aiu..
(d) If x is the separation between the plates. then force between them
e0 v2A
F
----;T
d' d' ·
2
The work done. w JFdt=eo~V Jx- dr2

d ,I

2
Eo AV [_!_ ___!__]
2 d d'
6.35x10-IO J.
17. The force on a dielectric for constant potential is given by,
2
F e 0 bV (k-l).
2il
For equilibrium of the slab,
F, F,
2
e 0 bV/(k 1-I) e 0 bV2 (k2 -J)
or
2d 2d

.!i ~T_
v, VT,°= I Ans,

18. The initial charge, q/ CV.

and final charge, -CV.

The charge flows, 2CV.

The heat developed in connecting wire V(6q)=2CV 2

2x(5x10-6 )x12'
1.44 mJ An.v.
_q_'.=50+100=3.
19. For the constant potential, K q .50
Ans.

20. If q be the charne.of the metal sphere corresponding to the potential V, then

V - - 'f_
41tEo R
or q (4ne 0 R)V

( -1- ) x IO-J Xl=5.5~IO-IO C


9xl09 2

q
o~sox 6.25 x Io10 x 1.6x t 0- 19
6.95 µs Ans.
21. With the dial at 180°, the capacitance C=950PF
The corresponding charge, q CV
950x 10- 12 x400
0.38xl0-6C

(a) For the dial at 0°, capacitance C' sox10- 12 F

The corresponding potential V' .!L 0.38x10-6


C' 50xl0-12
7600 V Ans.
~, ,, '.J,'

(bl Work needed, w .!.c·v·2 -.!.cv 1


2 2

.!.xsox 10-12 x 76002 _.!..x950x I0- 12 x400 2


2 2
l.368x\O-J J Ans.

22. Initial ener~y stored in the capacitors

1 , 1
-x0.lxJO- +-xC') x0=5 J
2 2 -
After connection with ·identical capacitor, the common potential,

C11'J +O =!:'.i._=5V.
V'
C+C 2

The final energy stored, .!.cc+clV'


2

cv 2 =0.1 2 xs 2
2.5 J.
2.5
Ans.
5 2
23. Suppose at some instant, y is the depth of plate inside oil, then
C Cair+Coil

e 0 1(1-y) Ke 0 1y
~~~~+---
d d

e~ I [l+(k-l)y]

'CV=~[£+(k-l)yj
ev
Charge on the capacitor q
d

The current, dq =Eo IV [o+(k-1) dy]


dt d dt
12
8.86x\0- x lx500 [(l 1_ l)x0.00l]
or
0.01
4.425x\0-9 A. Ans.

Eo A1 - v( A1_')
24. (i) The capacitance of capacitor A, --+:'."...'.:.ITl
d d

•o A, (l+K)
2d
12
8.85x\0- x0.04 (I + )
9
2x8.85x10-4

2x\0-9 F

Energy stored in A, .!..cAv2


2

.!.x(2x10-9 ) (I 10) 2
2

l.2lxl0-5 J

(ii) The charge stored in capacitor A. (/,1 CAV =(zx10-'')x110 ,,


2.2x10- 7 C.
When battery is disconnected. the charge on the capacitor remains same. When didectric is removed. the new
capacitance of A.

.,
C'
Eu A =..s:_
d K
0.4xl0_,., F.
,
CJ,t
Final energy stored. U'
A 2(.1'
2
(2.2x10-7)
2x0.4x10-9
6.05 XI o-'J J
Work done- by external agency w U,,i'-Uri

4.84x10-'1

e 0 KA2 9x8.85x 10- 12 x0.02


(iii)The capacitance of B.
d 8.85xl0-1

'l.-1 +O
Common potential. V
C,i'+C8
100 V

Net energy stored, u .!..(c,+c8 )v'


2

1.1x10-5 J An.<i.
25. (b) When inner sphere is earthed, its potential beconies zet'o. To make the potential of inner ~phere to be zero. s·ome
negative charge must be appear on the inner sphere to cancel the _positive potential of outer sphere. If q~ he the charge
on the inner sphere, then -

· VA =0

'La =.!!._x30=24nC(-)
h 10
(a) Capacitance, C CAB +CB

41te 0 (_"}>_)+41te 0 h
b-a

4ne 0 (_i__).
b-a
Am.
26. The force between the plates of capacitor

I V2
-Eo A -
1 d2 .
For the balance,
mg F

I V2
-EuA-
2 d2

Aus.
"'
.. ·.--·.··. · · - ........ ffl
H11vts,mo,S0Lui10Ns · -·
.
.~-- ~.;;.,.. , ~-~ '

27. The capacitances of capacitors are : c,


If 1'1 and 1·2 be the potential drop across C1 and C2• then
V ... (i)

and ... (ii)

V,

The ele1:tric lield between the plates of capacitor I.

£ t=
V [ 1·
d,+d,.
]

If II is the speed or projection. then and


"
--a
I r-'
2

.!.(
2 m
£e } '

From above equations. we get

'("")(']'
2 Ill II

" [ ']'"
Eef-
md1

[ ' ]'" V ef-


Ans.

28. Hint is given in the exercise.


29. Hint is given in the exercise.
30, Hint is given in the exercise ..
31. If V1 and V2 be the p.d. across cap8citors C1 and C2 . then

v, +v, ~

and

V,

Similarly if V:, and V4 be the p.d. across the capacitors C3 and C4 respectively, then

v,
The potential difference Vi -V3
--sc,
- - + -sc,
--
C1+C2 C3+C-1

s(c,c,-c,c,)
(c, +c2 )(c3 +c4 )
32. In close loop,
3
I-A-8-2-/, +.11..- (q,-q,) -I;, 0 ... (i)
Ci C3
and in close loop, A-3-4-B-A

0 .•. (ii) 4

From above equations, we get

The potential difference, Ans.


33. In close loop ABDEA-

0 ... (i)
E I
In close loop CDEFC I;,
2
qi +qi +1;1 -1;2 +!!1_ 0 ... (ii) F q, -q, C
C1 C2 I;,
Fi-om equations (i) and (ii), we get 3
A''--•1------ B
1;,c, -1;,c, -1;,c, +1;,c, '11 -q,

I+ C3 + C2 )
( c, c,
Given, Vc=O

-( q, ~,q') 1; (c, +c )-1; c -1; c


1 3 2 2 3 3
C1+C2+C3
Similarly v2 and V3 can be obtained.

34. When switch S 11• is opened, the capacitors are in series. Thus, the charge on each of the capacitor is

q ... (i)

After closing of the switch Sw, the charges on the capacitors are shown in figure.

In close loop ABEFA, = 0


S,.. B
q,l~,Jc
and in close loop BCDEB, 0
1
1;1
ql
Initially charge of capacitors joining at B is
After shortening of switch the charge on then are
q-q =0.

q 1+q2 -0+Cl;2
F
-:JT~
E
IJD
Cl;,.
Therefore flow of charge through section I.
(q, +q,)-(q-q)
c1;,. Ans.
The tlOw of charge through section 2, -q,-q

-,
35. If C1 and C2 _he the capacities of S 1 and S2, then

..s. ,.
C2 R

(a) If q1 is the charge acquired by S 2 in first contact. then

_q_,_ c, R
Q-q, --"-=-
c, ,.
or q, Q(R:,.)
Before second contact the charge of S 1 becomes Q and so in second contact, the total charge on S 1 and S2 together

Q+q, =Q(l+-R-)
R+r
The charge again distributed in the ratio of their capacitances, and so

'le Q(I+ R:r )(R:r)


Q[R:r +( R:,.f]
q,, Q[-R
R+r
+(__!!__)'
R+r
+....+(-R
R+r
)"]
Q-;:R [ I- ( R+r
R )']

The electrostatic energy stored in S 2•


2 2 2
q,, q/1 q,,
u,, 2c = 2( 4ne 0 R) 8ne 0 R
2

(b) ,.
u_ Ans.

36. The given circuit may be redrawn as :


Eo A1 =-l-xl08 F
c,

L:J
d, 288

. Eo A2 =--1--F
and c, d2 300xl08
(i) The equiv0.Jent capacitance 120V

(ii) The energy stored in capacitors

u 1-cv' =_!_x(l7xl0- 12 )x120 2


2 2

l.224xl0-7 J. Ans.
'
I. We know that. charge ,q Jidt
u

f
Ill

<20+41)d1
0

I 'I'"
201+~

400 C
2 "
Ans.
2. If V0 ·is the zero error in the· voltmeter, then we can-write
V "i, +ir
or 14.4 Vj1 +1.75xr
and 22.4 Jlj1+2.75xr
After simplit)'ing. we get
V., 0.4 V Ans.
3. At the left junction.
3+5+8+i1 0
-6A.
Similarly 9A.
4. We know that
V
or 1.52
I; 1.52 V.
In second case
V l;-i>·
or 1.45 1.52-\xr
r 0.07 !.l Ans,,
5. For maximum current.

R 20xl0 +0=20 !.l


20+10 3

V 5.5
The current, R=20=0.83A.
3
For minimum current

R 20xl0 +JO=!.!Q!.l
20+10 3

!:'._=22_=0 15 A Ans.
R !IQ .
3

·,
6. Upper 4 n resistor is shorted, so current in it is zero. Then other two resistors are in series, so

4 2
( - )=0.2A Ans.
4+6
7. The effective circuit is shown in figure.

JO il IO il JO il

~ -
JO il JO il JO il

I,

lv
The equivalent resistance. R 15 n

:. Current, .!'._=~=0.4A Ans.


R 15
8. At steady state, the p.d. across the capacitor, V0 = 6 V. When switch with S is opened the current starts flowing from
capacitor towards resistor. The initial current

Vo=~=D.6A
R JO

The time constant, T CR=25xI0".6 xJO

25xJO-\
'
We ~now that,

_ e-[ro-3 12sx10-5]
06
o.6e-4
I JxI0- 3 A Ans.

9. The current in 20 n resistor, ;, 40 =2 A.


20
Thus current in R will be, ;, 8+2=10A
In close loop. we have
-i2R-i1xl0+100-40 0
or -IOR-2x!O+ 100-40 0
R 4il
I 0. The currents in different resistors are as shown in figure :
;,
In close loops. we have
I il
'2 n
-2i2-l(i2-ii)+li1 0
.
and -li1+2i2+l(i2-i1) 0 I il (i!-il)
2n I il
After simplifying, we get i1= 6A, i2 =4A. Ans.
;, ;,

I I. The current in the circuit 24=2A


12 ,,I,
J4V
The p.d. across 4 n resistor. 4X2
As, 0, V, =8V
Similarly p.d. across 2 n resistor 2X2
-4V Ans.
:_ .• ,_~l
12. The effective circuit is shown in figure.

/Hl \~Hl

Hi
j
···............ ///
The equivalent resistance. 'R 2!l

Current _13_ =6A


2

12x8 = 96 !l
13. The equivalent resistance R
12+8 20

The current. __2±_=5A An.,·.


96/20

14. i1 3_=0.4A
5
2 ll ,,
,.,
I'., I ll B

i!
4V

4 \
and =0.5A 2V J ll \.
8 \ 3 ll 5Q
4V
In close loop x-A-B-y-x. we have
1,
1, '••••••...
3i1 + 4-3i2 +1-".t;i' 0 A
.,. i1
3.7 V An.\',

15. (a) The equivalent resistance of the circuit


R 1+5=6ll

3x6 =lµF
and C
3+6
The time constant, r CR=2x6=12µs.
(b) P.d. across both the capacitors together
V v0 (1-,-' 1')
1
d1-,-' ')=12(1-±)
HY
If V1 and V2 one the p.d. across JµF and 6µF capacitors, then
V1+ V2 7.6
and 3V1 6V2
V2 2.53 V A11s.
16. The successive reduction of the circuit is as : 3 ll

Clearly equivalent resistance, R 2!l

6 ll

I '
,-
!
17. The effective circuit may be drawn as :

4n 3n 4fl ;,

Jn

20. 5.Q

A
B

6n 15 n
9n
9fl

12n 9fl 12 Q i-4



Clearly the effective resistance R 6.75 n Ans.

18. One unit cell consists of

c~ 2R

So. AD'"'
B

B

(2R+R.,)R
or (2R+R.,.)+R R.,·

After simplit)'ing,-we get Rx=(JJ-l)R. Ans.


19. If R is the resistance between A and B, then R, A
RA'B' A'~R
R2R R
or R R2 +R + 1
B'______lj B
After substituting the values and simplifying, we get R =6 .Q. Ans.
20. The effective circuit is shown in figure

2n
A'o-"ffl\----,...
25 .Q
B
8fl sn
50 n 50.Q
50 n

Thus equivalerit resistance,


+ 50x50/3 +·
R 2 8
50+50/3
22.5 n. Ans.
21. The currents in two successive restores are as :
R! R,
;, ;,
T
;,
R, R,
i~
B ____..L__ _
T

Clearly ;, !!.
2
!i_
or ;, 2

!!L Ans.
R2 2
22. (a) Immediately after the. key is closed, the capacitor offers zero
resistance. The circuit will be completed through least resistance ~~ K
branch, which is shown in figure.

The current,
(b) After a long time capacitors will offer infinite resistance and so
L:J~ K
effective circuit is as :

The current
23. The p.d. across the capacitor R,

v, .fl.= 20 =4V
C · 5

The initial current, _l'._=~=0.8A:


R 5 ·
< CR= 20 x 5 = 100 µs.
The current in the resistor at any time t
• .:.11r.
•oe
,, .

The heat dissipated, H f,, PRdt

,,
f,, [i e-'0
1' ] 2 Rd!

After substituting and simplifying, we get H Ans.


24. (a) The current in the circuit juSt after connection, ·

Vo =~=0.25A.
R 24
(b) Current in the circuit, one time constant after connection are mate,
,,e
. -1/T.

0.25 e- 1
0.09 A. Am.
25. Power delivered by the battery p l;i
or p ~· -1/-r.
~t0 e

or p

Po
For, p 2' we have

Po
2

or 2
0.693 "· Ans.
26. The effective circuit is shown in figure.
Resistance of wires, -R12 =R13 =R34 =R24 =r
,, )1 l'
i• o)! ~

The heat liberated per unit time,


v'
r
From Ohm's law, we can get current through the conductor 3 -' 4· :"
V
i34 r(,[2"+3)"

Thus Q.i4
-2
134 ,._
v'
2.
,-('4+3)
Therefore required ratio is :

fu 2 '
(,[2"+3) =11+6h. Ans.
Q34

27. (a, b) The power of the circuit p i2 R+l;i

or 50 t2 x2+1;xt A~iL-s
R=2n
I; 48 V.
Thus p.d. between A and B,
VAB 48+tx2=50V. Ami'.
28. At steady state, there will be no current in the capacitor. 6n
F G 6V

Em
The current distribution is shown in diagram. In loop (2), H
Q)
;, i=t.5A
4 2n 4n
In loop (3). E 2V ;, D ;, C

+2-2i2 +4i1 -3fi -3 0 0)


i2
i2 IA.
A 3V Jn B
The p.d. across .the capacitor,
VEG 2-2(! =:i-2xl=0

u Ans.

29, In clos_e loop (I), we have 4-10i-2i1 =0, and in close \09p (2), we have

-I0(i-i1)+2+2i1 =0 10n
After solving above equations, we have

_!_A ton
Ans.
7
30. Using Kirchoff's loop rule in loop (1), (2) and (3), we have
I00-1 o;1- IO(i1-i3)- IO(i1-i2 ) ,0 · ... (i)

10(;, -i,)-50+ 10(;, -i,) 0 ... (ii) Ui-i:.> Uri2)


(i 1-i~)
and -l0i3 -I0(i, -i2)+ IO(i, -i,) 0 ... (iii) Q) wn@ sov
After solving above equations, we get
,·i1,,=3.15A, iz=0,i3=1.25A

31. The current distribution in different branches of the circuit is


shown in the diagram.

By using Kirchoffs loop rape_ in four loops. we can get


i2 1.85 A.
\

32. At steady state, there is no current in the capacitor, ·so the effective circuit is as follows :
If V1 and V2 are the p.d. across 4 n and 8 .Q respectively, then
A
Vi+ Vi 24

and
_Ii -
v,
4 8

V, 8 Vand V2 = 16 V

s B
Similarly if V3 and V4 are the p.d. acros_s 4 ~ and 2 .Q· respectively, then
24V
V3+V4' 24

and
v, v,
4 2
v, 16 Vand V4 = BV.
Thus p.d. across capacitor
VA-VB - V3-Vi=8V. A11s.
The discharged circuit is shown in figure.

The effective resistance, R

Current

Ans.

33. During the time interval from Oto 10 , the voltage across the capacitor is zero, the charge on it is also zero, there is no
,'
current through it and hence VcD is zero. From 10 to 2t0, the voltage .across the .capacitor, and hence the charge on
- I
its plates increases linearly and hence a current passes through it. This means that voltage Vcv becomes constant.

During time interval 210 to Jto the voltage a~r~ss the capacitor does not change. Hence current does not flOw, and VcD
is zero. Finally from 3to to Sto, the capacitor is discharged the current through the resistor is negative an~ constant, and r
C

its magnitude is half the value of the current during t 0 to 2to.


- ,,,____---.------- 9 1'~~~m~~,f~~H,NTS"ANe1.$Q~tJT,oNS :l1t~IPJ
D
34. For balanced bridge.

0.6p =_13_
B
0.4p X

X sn. Ans. X

35. Suppose the linear charge density of the cylinder is A.. Take an element of length dt on the surface of the cylinder, the
charge on it is. Jq ='Adr. The electric field at a point on the surface of the cylinder is
,,
<
E 21te 0 a

2rce 01a£

and dq

Thus current. dq
-=21te
di 0 aE - (d')
di
or 21te 0 ,aEv., Ans.
A
36. The leakage current

We know that.

d
A
C

1,1,. i
II d

V VC
Now Ans.

37. The current in the circuit,

The p.d. across any source is, V E1-ir.

As r =R2 is greater. so p.d. across this cell becomes zero. Thus

0
21',,

·•-2•
·" " xR2
D R

(R1+.R2 +R)

or R
38. Using Kirchoft's loop rule in loops (I) and (2), we have
U1+i2l
;, i1
l',-R(;1+i2 )-i1R1 0

and 0 I',, R <D R,@ R,


After solving above equations, and substituting the known values) we get
i1 1.2 A a~l i2 =0.8 A.
'
I',, +I',, ]=,2.::2_-_I__A
39. The current in the circuit. [ R1+R2 ·10+20-10 1

Thus -(I',, +iil,)

-(2+ \'~~~)=--4V.
10, lf'
Ans.
EIJECTRl€f.'IM ~ ffiGNETISM
_, ~·

40. In closed loop 123451, 0 i:, ; R,


5 4
or
Now in close loop 12361, we have 6 i:, I I 3
A B
S2 -iR2 +VAo-C,1 0

VAB (I;, -/;2)+'iR2 2


i:, R,
(l;,-/;2)R1
-0.5V.
R1+R2

41. (a) The current, nc,=5.


nR R
uR
-=a. Ans.

"EH 3-cells
·R
(b) The circuit can be redrawn as :
For upper close loop, we have
nl;-i(nR) A ,1 B
or n(u.R)-o:(nR)
r II I
Ans. ' ·cN-11)-cells
VAB
42. The resistance of x length of the potentiometer wire is

R, Rolx; and R Ro(l-x)


l-:r i

(_Rox_).
IT.[_ + /lo {l - X)
The total resistance of the circuit ~ox+R i
I
e-x · xR ]
[-·-e-+ iR+xR0 Ro.
The circuit current,

The voltage, V

After substituting the values, we get . V [ Rf+Ro(l'-x)-H Ans.

43. (a) We have, 6 V.


As 0, :. VA ;,6V
V,-4V=6-4=2V.

(b) The current in the potentiometer wire,


[R:.l
If x is the required length, then VAD iRAD

_6_xRAsX
or 4
RAB 100
X 66.7 cm. Ans.
( c) As Vo =Ve, so VD - Vc =0, therefore _no current through if.
(d) Do as above.
44. By Kirchoffs I law i1 +i2, R, I;,
Using Kirchoffs loop rule in loops (I) and (2), we have ;, CD R,
l;2 -i1R2 +l;1 +i2R1 0
i2
I;,
and G)
·After substituting the values, and solving, we get, R
O.Q2A. Ans.

45, The circuit current


l;t R1
The p.d.
'
'-[.h=_k]R
':ol R1 +R2 I
A

'
B

1
l;2,k2
Ans.

8 7 ..
..... 6 i~ 5
46. ;, ;,

"a;,- R, R, "~I;,

;, ;, i3
R
2 ;, 3 ,, 4

Applying Kirchoffs junction rule, we have


i1 +i2 ... (i)

and i3 +i4 ... (ii)


Now in the loop 12781,
0 ... (iii)
In loop 1236781,
0 ... (iv)
and. in loop 34563,
0 ... (v)

After solving above equations, we get


[l;(R2 +R3)+1;0R3
R(R2 +R3)+R2R3 .
l Ans.

47. The current in different branches is shown in diagram.


In close loops (I) and (2), we have,
-i1R1 +l;1 +izR2-~2 0

and l;2-i2R2 +l;1 +(i1 +i2)R3 0


After solving above equations, and substituting the values, we get
i1 = 0.06 A.

48. Suppose q is the charge on the capacitor at any instint, and iz is the current, then B
dq 6 S,.. 5 ;, 4
;,
dt
;,
By Kirchoff's first law,
. q
. R : ~q

R
2 3
In close loop 1-2-3-4-5-6~1~

ir+!L-s 0
C

or (
i1 + dq )R+!!_-s 0 ... (i)
di C
,In close loop 2-3-4-5-2,

o. . .• (ii)

From equations (i) and (ii), we get (s-~n


</
dq '
or
JrS- ~)
II
J1 II

or q -
,, l
Cs (1-e--
RC
,..2
V !!_=_f,(J-e-2,IRC) Ans.
C 2 ,
49. The current distribution is shoWn·in figure.

(,~--,1---~------'5
S1 i3
In close loop 1-2-8-7-1,
+i1R3-S2 +i2R2 0. . .. (i)
and in close loop 4-5-6-7-8-4,
-{I -i2R2 + i3Ri 0 ... (ii)
At junct.ion 7, we have
... (iii)
After sol'Ving above equati~ns, we get
s, (R,,- R,) +s,R,]
[R1R2 +R2R3 +R3R1
Potential difference, S2-i3R1

[s,R, (R, +'R,)-'s,R, (R, +R3)]· A11s.


R1R2 + Ri.RJ +R3R1
50. The current distribution is shown in figure. ;, 2 V1-i3)
In closed loop A-6-5-4-8-C-D-A, R,
13
-i2R3-(i2 +i3)R4 +V 0 ... (i)
A- ,:) ,__ B
In closed loop 1-2-3-4-5-6-1,
-i1Ri -(i1-i3)R2 +(i2 +i3) R4 +i2R3 0 ... (ii)
i!
and closed loop 1-2-5-6-1, 6 4
R1
5 R, U2+i3)
.-i1R1 +i2R3 O· .... (iii)
After substituting the values and simplifying, we ~et D
. C
I A. Ans. V

. .
0
0
0

I. Mass of the water, m pV=I000x0.01


10 kg.
The amount of heat required to raise the temperature of water,
H mCl!.T
I 0x4200x(40- I 5)

1.05xl06 J.
If r be the required time, then 0.60 Pt H

H I.0Sx\06
or 0.60P 0.6xlO00

= 29 min. Aus.

R 2 IOx 5 =6.6Hl.
2. The equivalent resistance of the circuit
10+5

The heat evolved in IO minute, H ; 2 Rt= I o2 x6.67xO0x60)

4x 10 5 J.
95238 Cal.
H 95238
The amount of ice required m
L 80
1190 g
Hi 6Q
Am;:.·-..v,,11v--~>N-1..-----,

p
v'
3. If V is the p.d. across 5 Q resistor, then
R
V Jpji_ = .J1 0x5 = Jsii 5Q

The current in 4 n resister,


__I_= Jsii
4+6 10

Thus heat generated in 4 Q resistor,


0 .
H ., (Fo
, x4=IO )' x4=2cal. Am.·.

4. The total resistance of the circuit R 2x6 +i=.?.n


2+6 2

The current drawn from the battery _I:'.=~= 2.4A.


R 5/2
6
The current in 2 n resistor, i' 2.4x-(-)=I.SA.
6+2
The heat produced through 2 n resistor in 15 minute is,
H PRt=l.82 x2x(ISx60)
= 5832 J
(a) If 6.T be the size in temperature of water, then

H = 5832 = 2 _9.C. Alls.


l!.T C 2000
(b) If 6.Q resistor gets burnt, then current in 2Q resistor,
6
- -=2A
2+1
The lieat produced in it in 15 minute ·
H 22 x2x(l5x60)
·- 7200 J.

The rise in temperature 7200 =3.6"C. Ans.


/ff 2000
5. The given circuit is balanced wheat stone bridge type, so its resistance
3Rx6R =2R.
R,
3R+6R
For the maximum power, R, = ,.
2R 4
R 2n.
The maximum power, p s' s
4R =4x4
2

s' Ans.
16
6. (i) In series, time taken to boil water l1 +!2 =15+30=45 minute

(ii) In parallel, time taken to boil water [~ ] = l 5x~O = 10 minute.


'1 +t2 15+30 ·
2 2
7. The resistance Of heater, R V = 100 =I0Q
P 1000

If i is the current in heater, then

JW=2.5A

2.5xl0 = 25
The current iii .resistor R.
R R.
The total current, i1+ii

(2.s+~)

Thus iXRTotal 100

25
or ( 2.5 + R )[~+Io] 100
I0+R
After solving, we get R sn. - Ans.
8. By Faraday law, we have m zq
m 4.62x I o--6
q 14C
z 3.3x10-1

q' 142
The total heat generated H 1960)
2C 2x5x10-2
If H 1 and H2 are the heat generated in 20 .Q and 30 Q resistors, then
1960 ... (i)

i2x20 2
Also ... (ii)
; 2 x30 =3
After solving above equations, we get H, 784 J. Ans.
9. Given, iJ.OOl-0=0.00l°C ,,

The Seebeck emf, e

4
(-46x10-,;)x(o.ool)+(--0. s;1o... }0.001 2 )

-4.6x]0-8 V. Am.
I 0. Charge required to liberate 22.4 litre of hydrogen
2x96500 C
Charge required to liberate I litre of hydrogen
2x96500 C.
q
22.4
q 2x96500/22.4
If, is the required time, then
i 5
29 min. Ans.
5
I I. Mass of gold required, m -x20=lg.
100

We know that

W,fo XZH
ZA11 W11

197 1
· x0.1044x10-4
1.008
20.4lxl0-4
We know that, m Z,1i, it

m Jx!0-3
ZA11i
4 min, A11s.
12. The charge needed to deposit 107.9 g (I mol) of silver
96500 C
96500x2.68
Charge needed to deposit 2.68 g q
107.9
2397 C.

The current in the resistor


'!..= 2397 = 4 A
I IOx60

The heat produced, H PR,


42 x2ox(10x60)
192 kJ. A11s.
13. Mass of the wire, m pAI
9x103 x0.5xl0-6xO.I
0.45xlo-3 kg
pl
Resistance of the wire, R
A
(1.6x ID_, )x(O. l)
O.Sxl0-6

0.32x10-2 Q
The amount of heat needed to melt the wire
H mC!J,.T

(0.45x I o-3 )x( 9x w-2 x4200)x(I 075-25)


178.6 J
(i) If t is the required time, then i.2Rt H
or 10~ X0.32 X\0-2 Xt 178.6
558 s.
(ii) If mass of the·wire is doubled, the heat required becomes zH and resistance of wire becomes zR, so the time required will
be same.
14. If i be the current in the circuit, then V ~-ir
or I0 12-ix2
IA.
The charge flows in half hour.
q ;, = Ix 1800 = 1800 C.
The charge required to liberate 107.9 g of silver
96500 C
107.9xl800
:. I 800 C will liberate.
96500
2 g. Ans.
15. The mass of copper needed for electroplating.
m 2(pAr)

2x9000x I Ox I 0-4 x I Ox I 0-6

l8x10-5 kg

Charge required to deposit copper q


!!!..= 18xl0-5 =600C
z 3xl0-7
The energy spent by the cell
H Vq=l2x600
7.2 kJ Ans.
If /J,.T be the rise in temperature, then
H mC!J.T
H
or !J.T
mC

7.2xl0 3
(1 oox I 0-3 x4200)
17 K. Ans.
16. The charge flows in one hour
q it= 0.5x3600
1800 C.
96500 C of charge liberates the silver
107.9 g
107.9xl800
1800 C of charge liberate
96500
2.01 g Ans.

...
I. F q =(Vxii)
IX 10-('[2 X 106 j X (0,20) + 0.40k)]

-0.8) + OAi N
2. We know that. p JzmK

mv P "12mK
We have. ,. -=-=--
qB qB qB

[2n:i(lqB I
j2(2m)K lqB Ji
J2mK lqB
and Ans.
J2(4m)K /(2q)B
3. The situation is shown in figure.
I
-1111·-
, ~--x··--x
' - X ,.
Given. qV
2 ?' X X

,. { X X X
' ____ x ____
-•_..·"'IL--':.-::: X
19
-0.lm-
2x(2xl .6xl0- )xlO.J
·6.4x10-27
]Ob mis

Radius of path. ,. 1111'

qB

6.4 X 10-l? X \0 6
2x(l.6x 10-19 )x0.l
0.2 m
x 0.1 I
sin8 -=-=-
,. 0.2 2
or 8 30°. Am.
4. For the given condition electric force on the particle is equal to magnetic force. Thus
qvB qE
£ vB

.rqB]
8
[
"'
0.05x20xl0--6xl 2
20x10-9
0.5 V/m. Ans.
,,

mv
5. We have r
qB
rqB
m
V

0.015xl.6xl0- 19 x0.5
or m
6.0xl04
2x 10-26 kg
m
A 12.5
l.6xl0-27
lt w_ill be 12. So the isotopes are of mass number 12 and 14. These are 12c and 14C.
6. (a) The radius of the path
mv (1.6 X I0-27 ) X(I X I07 )
r -0.1 m
qB (l.6x I0- 19 )x I

EF 2rsin45°=2x0.lx }i
0.14 m Ans.
27tm 21t X1.6 X 10-27
(b) Time period, T 6.28xl0- 18 s
qB l.6xl0- 19 xl
When magnetic field is reverse in direction, the particle covers angle 2700 inside magnetic field.
3T
4 = 4.7 X (0--8 S Ans:

mv
7. Radius of the circle, ,. qB

(2xl0-5 )x(4.8)
2x10-3 xI.2
4xJ0-2 m
I
By lens formula,
V -]8 +12
or V 36 cm
I V
Now
0 u

I Ans.

V 1001
8. Electric field, E ----
e e
Ee (1001/e)e
The acceleration of electron, a
m m
dv IOOte
or
dt me

or

50e 12
V ... (i)
ml
dx 50e 2
Again 1
dt me
.<

J d,
I' soeml
- I 2dI
0 0

50et 3
or ., 3ml
,,,(ii)

For X e. we have
50et3
3ml:

[ ']'"
3m£-
50e
Substituting this value in equation (i), we get
213
SOe 3mf 2 _ 16
I' - [
- -] - km/s Ans.
mf SOe
9. If v is the velocity of the electron, then
I '
-mv- eV
2

2nm
Pitch vcosaxr =vcosax--
eB

2nm
- - --cosa
eB
J2•V
m

Ans.

10. Given, i=-EJ and B=-Bk, .y


V
The net force of the electron
F qE+q(vxii) v.<

(-e)(-£j)+(-e)[(v) +v,.j)x(-Bk)]

,eEj ..- ev,~!Jj +ev...Bi ., '---------x


ev.rBi + e(E-v.rB)j

F ev,.B ~ e '~
The acceleration a -=-·-1,--(E-v.rB)J
m m m

dv_r = ev.l'B
... (i)
di m
··dV1, e
and a_,. -·-=-(E-vxB) ... (ii)
di m
Differentiating equation (ii) w.r.t , we get
2
d v...
2
. .. (iii)
d1
Now from equatiqns (i) and (iii) , ·we have
d 2v_,.
... (iv)
2
d1
d 2x
This equation is similar to '-oJ2:r
dt 2

Ol
(:: )
and v_,. A sin(WI+ 8)
Att = 0, i·,.. 0
0 ,, ;Asin(0+8)
or 8 0
dvy
Also ~cos(rot+O) ... (v)
"' dt
In eq (ii) at t = 0, Vx = 0, , .. !1
"' '1/,1 J'' •; ,, "
dv_ .. eE
dt m
From equation (v), At t = 0,
dv_,.
'' dt
ioA ... (vi)

l roA
eE
m

or A

',. ".£
Thus, v_., = , -sinrot ... (vii)
B
For Vy =0,
,.
E
or 0 -sinO)f
B
Ol/ =O • or O)f ::l 0, n: 21t }I;"

lt \' I
For first tue, t=-
Ol
',.
dy E
Now from equation (vii), we have -sinro,
dt B .'
' .,.
or I~-
0 '
BEI
0
sinrotdt

or y
!:._
B

E
I
-cosrot
o, r'·
0

.·aw [-cosrr+cosO]
2£ 2£ 2E_m t) ,. ·, ..
11.
' . '
eB2 • Bro Ans.
B(eB)/m
The force on each element of loop in uniform magnetic field is zero. So there net is found equal to zero.
12. fr
A" -•AA ~·,:,'':\,

1lfixB0{i + J-i:k)]=Bif(k- j)
or F _ =.
11
.J2B0if. Ans.
13. The force on vertical sides of the loop is cancelled out. The force on lower horizontal direction change by
=-Bil-Bil=-2Bil.
Ans.
14. (a) The magnetic field due to wire loop

B
µ,(l!J)l k- µ, . - -1-[sin60°x2]k
2a 4n a cos 60°

µof[I- -..J]],:
-
µ
- "=--(0.222)k 0/ -

2a3 7t 2a

-~(O.llk)
a
GiVCn, v vcos60°i + 1 sin60°j1

(f+vfj)
Force F Q(PXB)

Q[(fi + ''f j }(-:of (0.1 lk) l]


Qv;ol [({+ { j }(-0.1 lk)]

Q vµ,J [+o.J J j- ,/3 xO.I Ji]


a 2 2
0.109µ 11 /Qv
or F
a

[ 0.109µ 0 /Qv]
a Am·.
111£1

(b)_ iAXB

i(Area of loop k x B)
0.6 I 36 a2!B along y-axis Ans.

15. The area of the coil. A fxaxasin60°

I ,/3
-x0.02x0.02x-
2 2
1.7,/JxJ0-4 m2
Torque, iABsin6
0.1 x.Ji x I o-i x5x I0-2 sin 90"
s.Jjx10-1 N-m A11s.
16. (a) Given. 2,t/1 L
l
R
2n

L'
The area of the loop. A nR' =•[2~r 4n
,.
Torque. . iAB

L2 BiL2
ix-B=-- N-m A11s.
4tt 4tt
(b) For square of side a, a L/4
1
•'Area, 1 l L_i
A a =-X---
4 4 16
Torque t lAB

i}
i-xB
16
II/
e;i1·
N-m Ans.
16

17. 1 for a unifonnly charged disc is found equal to


2
: . From the sphere, take a disc of volume dV, !lO we-can write

M
q ]dv
[ (4/3)RR 3
L
m ]dv 2m"
2[ (4/3)•R'
Clearly for the sphere 'of total charge q and mass m
- ..!Li.
M 2m '
18. For constant speed, net force on the><pJrticle is zero. So

F = o = q(E+vxii>
or 0 , = +er£+soix2x10-3j1
or O' [£+0.lOk]

'I
E ---0. IOi VI m Ans.
19. Magnetic force, t q(iixii)

or 6.4xl0- 19 k -,[,2.oi + 4.oj}x(B,j +JB,h]


, ~r -2.0 T Ans.
20. (a)
"' M , i(,11 + A2 )
2.86 A m1
(b) M i(A1 -A1)
I.IO A- m2
1 2
21. If v is the speed of the electron, then 2mv K

V
= ·w
-1--;;;
mv
We have r d=-
eB

,.,
8
-11, ,) '

Thus to prevent the beam from hitting, we have


• I '·

: ..;. .- ie ~
{2mK
'/N·
,l11i,

22. The tension'in lhe leads will be;zero. if


'weight of wire magne'tic force in, upwards direction
or mg Bit
mg
or ·i se=467mA towards right.
23. The magnetic field can be resolved into two com~onents; B~ and B>. These are
B.r bsin0 and B.1.=Bcos8
The force due to Bx is

'"'' B.,idl
f
F "
0

H_,.x2nai
2naiBsin8
The force due lo By is zero.

24. The area of loop. A=O.lx0.05=5xl0-.1m:! .Joi ' '

The torque. '[ NUBsin8

2oxo.1 0x(Sx Io-~),x0.5sin 60°


4.3 x Io-3 N-m Ans.
25. We know that

,-'
(JOOx Io')'
lx!O-o Ans.
(3xl08)2
26. The force between wire and loop
F
,,
l'!!.[(30x20)xl (30X20)XL]
2• a (a+b)
3.2 mN A11s.
27. For the equilibrium of the cylinder. we have

NiABsin8 mgsin8xR
mgR
NAB
0.25x9.8xR
JOx(Lx2R)x0.5
2.45 A Ans.
28. The given loop can be spitited into two loops of area,
A1 0,20 x 0.1 = 0.02 m2

and 0.20 x 0.05 = 0.0 I m2


The net magnetic moment
M ,,A,(-k)+ A,jJ
. ' .... 1!Jl
2(0.02(-k)+O.Olj] -

[o.02j-0.04k] A-m 2 Ans.


:~.
0
0 --~'
,
. ~~

. '
.

.0 :;~0::/~ii f~~-/·'
,p
,- ,..
~

-,~':,. '"·""

f,1
I. Previously the magnetic field is due to the. current c<l~ses due to movement of the free electrons. To an observer in a
·" .J- J

trolley moving in the direction of current the current is due to electrons becomes zero but it is now due to movement of
positive ions. So magnetic field ·
Po _!_
B
2n r

2. The current, 'f_ =-"--=ef.


I T
'1
Poi_ ;,o(ef)
The magnetic field, B., =.. , '2r ------;g- ,,
,. :=, 14 Wb/m 2

Magnetic moment. M' iA =e/(xr 2 )


9xl0-24 A-m2 Ans.
3. The magnetic field of earth at that point becomes zero, when wire produces equal and opposite field. So

Po j_ _
21t • /' C

(39xlO..,,)x0.08
2x10-7
:;::; 16 A from west to east Ans.
-iio i
4. a, 2n ''";f

and .a, Po .i.[sin8+sin8J


4n d

. Po .l.x2sin8 ,,
4n d

1/2
Here sine
2 2
~(l/2) +d

Given [8 8 1
;
1
2
] 0,01

After substituting and simplifying, we get


di£ O.Q7 Ans.
5. The magnetic field at the ce~tre of the loop.·, '

B 4[.EQ..~{sin 45°+ sin 45°)] '

Here, r=a/2,
47t ,. . \/
-"---t~!s·
After simplifying, we get B
i'Tzµ0 i,
J[G
6. The magnetic tield due to one side of th4c square can be calculate~ as··:

Jlo __I._sin 8d.x


dB
41t r 2

u/2 , .•
µo1,8dx
B J
-<1/2
-.--,sm
41t ,.-

µ 0i a
2nR · ~az + 4 Rz n~4x 2 +a2 ~4x 2 +2a 2 · -p '
Now magnetic ticld due to the all the four sides of the S!-]uare can be calculalcd as : i~B
, '
4Bcos8 '
4Ba 4Ba
2R . ~ 4x2 +al

Ans.

7. The magnetic field due to a straight wire is given by

Poi(.O
- ·e)
. - sm 1 +sm ,
B
41t r -

where, r= R. sin8 1 ~, sin02 =0

...l!!L; l . ..-· Ans.


B
41tR ~lz+Rz
8. Same as solution of questions 7.
rp
:'
9. (i) ---------- 0-----o.6;n__ _
---- ---------~---------
!L=_q_= qro
The equivalent current
T 2n/ro 2n
The rotating rod with charge at its end behaves like a circular loop. So

µo iR2
B 2 (R2 +xl)J/2

Here. R =0.6 m and x = 0.8 m~


After substituting the values and simplifying we get,
B 11.3 x 10--< T
tii) lnthiscase i=qro/21t, R=0.3m. x=0.4m
\l B ' 22.6 X J0---4 T Ans.
10. The magnetic field "
B BL +Bp+Bo+B,.,

0+ Po ..!...+ Jlo •.i...+o


47t ,. 41t r

µo ..!...=2x10-1 x....!E.....
,. 21t r 0.02
J()--1 Wb/m 2
I I. Magnetic field due to segment of a circular loop is given by
µ 0 8i
B
~""
9i( -I - -I ) out of page
µ 0-
-
41ta a h
12. Using Ampere's law, we have

Here

J
0

cr'1 x 2rrrdr

'

BX2Wj

or . µ0c (r~ _'i4)


B 41j -

(4nxio-' )(Jx 106 )[0.0J' -0.024 ]


4x0.03
2.0 x 10-s T Ans.
13. Magnetic field is given by

B 2[!!!L.!.(sin81+sin8 2 )]
4n r

2 Po x--;--[sin(90°-8/2)+sin90°]
47t xsin8/2
µ 0i [l+cos8/2] Jloi
-cot8/4 Ans.
2ru- sin 8/2 2m
14. The magnetic field due to all the wire is shown in figure. So a
B101a1 2 ./2B
2fi(Po .!...)
2n r
a
,fiPo __;_
2
2n alfi r . '
4l!!L.!... '
2n a

4x(2x10-7 )x~=80µT up the page. An£


_ 0.20
15. B Bcur.·cd + Bstraight

..J~)i
.:1lL
2R
+ ~.!.(sin 45° +sin 45°)
41t r
Here . r Rcos45°
After substituting the values and solvin·g, we get
B 2.8xl0-s T An"-

..,_
(a)

)JUI •
= 1.1 x \0 4 T, out
. . . . beh,
Similarly at·Q3=I.IXIO~T into the ,page. II will ·J'·
21. The magnetic field at a distance of 2m from the W\ 1 '

B = \ ~\= 'l.'1<.\\\-i ~\\ -~\\\~i


2n r , :'.! · ('!!'
(a) The net magn~tic field at x = 0, z = 2m
• '.. •. - \ i 1. I l- - \
B =. ,,, ~1;-B1i r
J'YJ • ; .,., 1

1~1
10--6f-10-6j =0
--6" --6"• ,,_
(b) ii 'IOi-lOk·\
fi xlO-"T Ans.
(c) Do this part similarly. ,, , : , .'.. ,;
22. The magnetic field at the centre of the solenoid is giv::~y ~
10
B = . --{cos8 1-cosB.,) \ cm
,.. 2 - -
. !l .'
Here cos81 =0.93. cos82 = cos(l80°-81)=-cos81
25 cm

0.JX(2X0.9J)
B
2·r•trt· ·1· ,. 1...;i' 1. ·.•P

,,·'., .' ==, 0.3 mA, , -1:. ·.!r,;i , ,,, ·. Ans.• 11"1
23. (a) The inner radius is I'; ,~- 0.15m, so

-µ,Ni (4nxl0-7)(500)(0.80)
.• • B •,2rrr, 2JtX0.25•' l, ~< 1 "JI

=· µ,Ni _ (4nxio-' )(500)(0.80)


(b)
B • 2"'1, 2Jt(0. I5+ 0.05) \
24.
'4.oo
J0-4 T X Ans.
The field of the solenoid at the point is parallel to ,the solenoid axis and the field of the wire is perpendicular to the
I
solenoid axis. The total field makes and angle of 45° ~ith .the axis if these two fields have equal magnitudes. Therefore
Po ;wire
Ponisorenoid
·, 2n·~·,. .,
.,. _.,. • iwr'.e =4.77XIO.:.:m
. 21tnlso1 .
' ,.

The rCsultant field,

= ' 'Jiµh;sol;n~id
3.55 x 10-5 T
,,
25. We know that, V Voe-tit .. -.
,• '.•
or 0.90V0 Voe:-:.2~\,.
t 19.23 s
As t CR ,, i ,, ic, ' I
• ,, (

'
/
/

,u
'-

'-

'-
,u'

'. Anf.

18. (a)

The magneti

,Po [ia2/(a2-h2)]d
2
27t a

'd
(b) Forthecaseb=O; 8:::::Po' ,andford=O,B=O
2M 2
19. The magnelic field of side AD and BC are equ_al and opposite so they cancel each other.
Bnet = BAa~Boc D'l-------------,lC'.-
'' ''
2
Po -[~(sina+sina)- i/ .(sinP+sinP)] ' ''
4n o/4 ,Jo/4 '' ' ''
Ja/4
. o/2 . o/2 ' '\ p 'I p ,' '
Here, sina---,,===="'==, sinP 2 2
",~,-
2
J(a/2) 2 +(a/4) J(a/2) +(Joi 4) ' ' '' ' '
Aft.er substituting the values, and simplifying, we get ,,_
"'
.... .... ~ ..........
· ,
a/4
B
2JJol( .,/s-
= -;;;- I J;/13
I ) , out of page ' -- B
if2

.i~ - -· - ;
t 19 23
· -19.23xl04!l
··.. R ·c 1oox10"""
'- . ·- .
2: 20
io l.04XI0-4 A
-~.. 19.23xl04
Magnetic field B Poni = Pon(ie-,li)

JBdl
- --=l61txl0-8 T
0
Ans.
2
th
26. The current in slit of width h (for h << R),
2nR
.!'!!_ I Po (/h/2nR)
Magnetic field at a distance r from the slit. B
27t · r 41t r
Pohl
Ans.
4n:2 Rr .
27. Take two elements, as shown in figure. The current in each element.

di .!...da
n
•12

The resultant field B


J 2dBsin8
0

x/2

2 J(;:-f}in8d8
0

Poi
Ans.
tt2R

28. By Ampere's law Poiin =Po J' j(2rr.rdr)


0

or Bx2rrr J
Po j(2rtrdr)

or J
Po j(2rtrdr)

or br(a+I) J
µ 0 jrdr

On differentiation, we get (ex+ l)bra JJoir


h(cx.+l)ra-l
j Ans.
Po
Po 2M
29. We know that, B 41t
0

7
2nR 3B
M Ans.
Po
30. We know that, magnetic field due to a solenoid
1

(a) \ B ·--(cos8
Po"i 1 -cos8 2 )
2

Here, cos91 =r ~ , cos82 =180°


\\ 8 p 02ni [ - ~ x cos180°]
Jx2+k2
, B,.
I. Angle of dip. tan8 '·811 •J

I
>;=
Jj ··,
e 30°. Ans.
2. w N
0 B11
or JtJO K_,-K; 90'

"""
or
" s
J-MBsin8d8
90

or Kf ~ -MB(-cos0°+cos90°)
, MB= 2 x 25 = 50 µ J Ans.

3. At place I, 7j
"60
2
./I ... (i)
20 = 1tv MBH
,, 1

At place II, r, 60
-=21t
. .15
--
MBH,
fuf, ... (ii)

From equations (i) and (ii), we have

BH,
(11 l •BHl

BH, 16
or
BHi 9

As B BH /cos8.

!!!_ BH, cos60° 16 1/2 16


--X---=-X--=--
82 , • 8 11 , cos30' 9 Jj 12 9.jj Ans.

Po MJ3cos 2 8+1 B
4. B
4x· r3
.,,
12
1
io- x ~ JJcos 2 60°+1
I ·=Im

l.6xl0-7T 0'
tane tan60' 'JJ,·
tan a -2-=-2-=2 Ans.
' ,..,. Po iMr -
8 ,, ~· ~-[,-,. ' ' ] '
r-_
'l · .4

_10 _, x.3.:<02xo.11xo:3£

6.
Using lhe relation
:, . {<0.2)'_¥]'
· 3.4,x 10-5 ;
cor 2 8

7. For deflection galvanometer, With short magnet, We can Mite


0,816

Ans,

or

8.
./J
---,.
8
For moving coil galvanometer, we/av~
'.C]
·. [Nnia: S N · S
Ans,
N

C
-
iNBA
·., . -

.!!_Ox I0-.1 )x (I 00Jx(o.o I)X(2 X i'oj


....
.1--
G: :,J

0.05
9.
For long bar magnet, we can Write,
'4.0x,o-s N°tn1rad
F
.IO.
• " . .',h-:: ,
~ ~
1 l ..
•• 2.sx10-- N
' • - ,.,.
Magneric.field due to a ba~ magnet in tire broad,side on position isgiven
. r .
by
··Ans.,
8 ·µo M.
•) .,

. Alier'substituting the values an~ sirnp/ifyi?g We·/iet·


"" ' _.4_,r[-2
' .r + 4
!:..)3~ ., .
.: . : ..·
11.
Giv'en apparent" dip, B = 6x10-5 A·-m
8'
ran-'(21./JJ,
We know rha1,_ ap~Jlrent ~ip -
tan9' lane
.':;"_:
,. _ .cosa
tan8 ~-. -~lane~cosa
,1·,., .•
2 . .· I
'Jj'Xcos60°='Ji
or

At1s.
I
H ni = (500)x0.5
12. We know that, 250 Nm
. µ,H = 1ooox250

=, 2.Sxlo' Nm
M
We have .
7v
M JV
(2.5 x H>') x ( 10"') Ans
25 A-m 2
,.The total magnetic moment per unit volume. i.e., magnetisation
(a) 24
13. (8.52X l0 28 );(2x9.27X 10" )
I
6
"·, 1.5sx10 Nm.
(b) The magnetic field· due to magnetisat.ion.
Bm /Jo/
6
(4ltX l0"7 )x(l.58X 10 )
Ans,
2.0 T
H ni = (4000))<' (2)
14. "we know that. s'x H>3Nm:•
µ,fl= (2.3 x 10-5) x (8 x HP) Ans.
·0.18
Pr I +x
15. We know that = , 11+ 5500 = 5501
7
µ = µ,µ.=(550l)X(4nxl0" )
Ans
= 6.9 X 10-J /
The field is entering into the surface so flux. is negative.
(bl Explanation is given in the answer. 3
(cl V (3xl0"2 )x(lxl0-3 )x(0.5xl0" )
The~ volume of the ne·edle, =' , 1.5 X 10-3 m3

,.l'.v
The mass of the needle
·7900x1.5xlO_.
_1.183 X 10-1 kg

f 1.185xl0"'1x6.02xlO"
The number: of atoms in the needle l 56xl0"3
-=···{27'x toi 1 - • -

The needle's dipole moment 3


~(I .27x 1021 )X(2. IXIOi )
M 10· • ·
2.7 x 10-3 J/T
" I' '

r 1- ·/
r, 2"~ r
MB111
... (i)
18. We know that

Where
/Jo ;
., .;1 .
The magnetic field produced by,~ wire B = 2n°i.
I
"(2x/0-7)X~
- 0.20 ,
I.Sx/O~ T
r:n2 . .:. .'
- :
21t~
--
MBH j ••• : (ii)
2
Using_ equations (I) and (ii) • and substituting the values, we get
T2. · ,;, 0.076 s A11s.
19. For the rie~d1e to stay in any position

Pu 2M·
.,
or
4n"?
\

,- ·'[.!!!l.._
·,I j-
,..__-, 411. 2M
8 ]'"
11
After substituting the values and simplifying we get,
r ,:d;;• b2.0 Cm
20. We know that . ,,
., = f:2RBH }1an8
I
1
\l~N
-After substituting. the values' and simplifying, we get
N = -570. Ans.

~. (, ~,4
~. µ0 niA
p0n(i0 sinmt)A
_rtns,

\I ,, '
\
'
\
¢is ~ . BA ~ Bi<1tf"
~~!!d
\•\ ; />I />I
.o.soi<~
~ ~50
..,. 1.ss-,c.\0-2 -V · r '

~ 1 .llt + 6.llr2
lie
~.,1+\'lt
~ dt
4.
G\"en. \el ~ 1+\'l.,,_2.,,\111"-
\ndu~ed etnf,
·la)
R\gbt to \et\. ~ -~.,-i
cit
~\.
\c1il
e.
lb)
1ne \nduced etnf.
\a)
5-

oo tbe otne"r part s\rni\ar~Y to ms, th< ,ate of change of mall"etic lield is not constant and So emf
30
20 '.! .
f,om \ o ms to 20 ms an~ ms .
is not constant in these ,nt«va\s
,ne f-1' ··. ·,-.'. " ,,
At .•- , M _ 1\li -1\l
1
(b) chatg< ~ow• in anY int«'"'
·.Aq . ~- ..,.-R- ,;---R ',.
•._ .. . ._. .' '
la)
a
\ '. \ ' ll\le(ll)-1\lsV) "
; '•
'.'
~
. - }t

·-'
t·,'
. . ''

If!/ el!)"' 1/e(O), tnen AlJ "'0 ;


I'

lb)
lie
\a)

r
. 1t
ib< emf; at I "' goo ·
\bl
Ans.
'o-s ' .
.- X300cos300 Tr
1t . x-
At I"' 6()0, 600
(c)
...i
r
I
j-;

17. If first case. 1an60°


x,_ =."'!:.
R R
300L
or ./3 100
l 0.58 ll

In second case, ~. Xe =-1- /


''l
1u060°
R roCR
I
or ./3 JOOCxlOO !
C 19.2 µF
i
The imp'-•1.fancc of the cin:ui1 is z I:
I_. I

1 \.
100' +(Jooxo.ss-
JOOxl9.2XIO
.. J'
100n ·

Cutrcnl, !:_ = 200=2A


Z 100
Power dissipated Ynm;mt'I rose:,
c:: 200x2x I =400W
I K. If U0 be 1he initial energy stored in the capacitor, then energy at any time ·1
U(I) Uoe-HII I.

and uc,+ n
The fra~tiona~ loss in energy is
LiU U(l)-U(l+T) e-R,U. ~e-R11<t-TIIL - I' ;
u U(I) (!-HI/I.
r
;= (1-e-HTII.)
i
I

Asstlme In.at
L RT
l ·1s sma II compan.'U
·' lo I • (as rc,s1stnncc
· · sma II ),
1s Also· ,,-RTIL = 1- RT '1·
I.
1
IJ.U
u l-(i-R: )= R: '
2ffil
l
rol
19. l, 1 =300V/m. I
Eo =~=lx10-<• N/A:m
Jxi0 8
The maximum electric force,
C
£0q =JOOxl .6xlO7 19
1 I.-
= 4.8 x 10·7 N.
The maximum magnetic force,
= · q1•8 = (l.6xl0- 19 )x(2x l07)x(lx10-<>)
:;,, 3.2 X 10- 18 N. Am.
211. The inte~sity is given by 1-'
i
I.
I 2
I -e 0 £ C .I
2 t , I,.
or 2.sx1014 ~·- .!.xcs.s6x10- 12 JxliJ x'(Jxrn'i
2

Eo 4.3xl08 V/m

and Eo =l.44T
C I
l-,i
u: , , I
Also. ~ •• =" Ju'

or 4500 ... {iii)

After substi1u1ing ro=2250 rad/s and simplil)'ing equations (ii) and {iii). we gel

l
I 5. (al The frequency of oscillations

I /1'
l 2rc \/LC

'
2rc (S4xl0-3 )x(6.20xl0--{,)
1
, .f.." 275 Hz
(b) The ~axittlum "charg~ stored on the ~apacilor
Q CY = 6.20 x 10-6 x 34
i10.s, 1w• c
If i11 be 1he amplitude of current. •'
then
'
.. ,
, :•I l"'.:!
,/,
1 -2 ~ 'o .' "
'· 'g''t2C,

or g (210.SxlO--<>).:!
54xlo-3 x6.20xl0-6
. ,,
0.364A Ans.
16. (al lmpedanc~ of the ~rcuit

z
0

1;1 +[i'2rcx,sS~) ~2S~_10- 3 . · ·· 1 · · -],


2nx5SOx4.70xl0-6
·, = , 28.5 ll
v1>s,, ___ _
Vnn, =~ = 2.6A
.. Z 28.5
(bl V.,;, inn,R =2.6~15 =38.SV

,,. - •i X,-=2.6x[ I ]
'Jo..·., 1 - 1 • nin 2rcx550x4.70xl0~'
= '159 V
= 1, irmsXL =2.6x[2rcx550x25,:,:-I0-3 J
224 V
v""
Vw
·~-~
v.,,-v,..=224-159=65 v
'""" ',_ V.;;.+YbJ'=!15V
J, , Am.

(C) Power dissipaled by ,resistor,· .P

.=.•. 75x2.6x(~)
28.5 .
= JOO W
The power:dissip:ited by induCtor·and
. capacitor
. will be zero.. .
., ,-,~;t~\Y, -,
~.:f{~;:
~ ·JEDE€ril~~1&t.~Ni;f1~izr:\t:~~~i~( --_;., :.,., i:~:-;.
- ··.·- . ---.,.~-~~- ,.., - __ "
-' '-~---'.:,t-
I

12. The resonance frequency, OJ,.


&= !Ox I o-3X Ix I o--6
ID' rmi/s
Thus OJ• .0,90 OJ, = 9xI03 ,·ad Is

XL wl = 9xio'x(IOxl0-3)
90Q
·1
and Xe we 9x103x10-<>
I II.II n
The impedance, z JR'+(XL -Xe)'

J3 2 +( 111. I 1-90) 2
21.32 n
The current' amplitude,

;,
v,
z
= 15 ·
--=0.704A
21.32

VJR 15 2 x3
Average power, Pm.
2z 2 2x2l.32 2
0.74 W A11s,
13. In series circuit at resonance

V
impedance z =,. R, i =--
R
In parallel circuit at resonance

14.
impedance,
When DC is applied, the resistor is effective
z = :. i=D,

"
,·· R
.!'..
= 250 =250Q
;, ·1 ,
The impedance of LC-branch.
. I
.,
z X, -Xe =(wl--)
OJC
V 250
Current in LC- branch, i2 ... (i)
z
[wl- .:c) ;,

The net current,

i2
M,
'Jil-;;1-
[2J ''
' ;l
'
'

From equation (i), we have


;f1.25' - 12 = 0.75 A

250
0.75·
·lwl--dij Lt·.""! 0 ' ••L
' '
IOOO '
/wl-~.
. wC 3
... (ii)
7, .Ri:sunun':e th:qucncy. J;.
,g
2R lC
-~ ,/1

2n O.Ol><IXI0--4,
"1592 llz
k. ('1) The capaci1ive rcaclance

Xc roe (2nx200)xl2x10-<•
= 66.35 (I
v,. imtoX<' =2 X 66,JS
133 V

(h) Energy slol'l:d. u .!.cv/


2
= .!.><.12x10-<'x(IJJ):?
2
0.11J6 J Ah!f•
~- . Avcmge power consum1..-d is given by ·

'z
260

·.,, ~.s'~IO =0.62SW


26~
Ans.

I 0, X 1. =oo/. =(21txSOO)X8. I= iS.40


1 1
andXc=--= =25.40
roe (2RxSOO)X(12.Sx10... ) "

As X 1_=Xe·· st; n:sonam:e will occur mid l'u=IOOV.

Q'
- 11. We know lhal, u 2C

ti 75 X 10-6 t2
or S.70xio-<1
2C

C 2.69x10-' F
(a) The mass m is co~spondiflg to 1he induc,:tnc~ so.
m 1.25kg

(b) Force constanl -=---~


I
C 2.69x1o-J
372 Nim.
(c) The maximum displaflicmenl is corresponding to 1he ma!ll.hnum charb'C, .and so
.l'm:1~ 175 X t0· 6 ffl,

(d) The ma"imunl spC'ed vn!.I~ com:sponds to lhe maximum cui'rcnt. Thus

Q 175x10_,,
.Ju ~1.25x2.69xlo-3
3.02 x llr' mis An.v.
lmrcdance. z
~ - 219 _..,., A
(\') Current. Z - 99.2 --·- .
,,
(vi)
• iR=2.2x2S:::S,;p
l'c iXc = 2.2 x 127.38 = 2MO O
VI. ;x/. =' 2.2 X 31_.40 = (ti) 0
4. for the maximum cUrrenl. x,.
or
roe

(2ttx2xl0-1 )! xlOOxlO-·'

6Jxl0-')F
V 12
5. The n:sislan.ce of the coil. R i = 4 ;:: 30

Impedance, z E.=m
2.4
As z ~-.
x, 40

and I.
X,_ =.i_=O.OSH
m 50

The rcactance, I -Sil


-;;;c= 50x250Qxro-<>
The lotal impcndance. z JR'+(X 1.-Xcl' =JJ'+(4-SJ'
5P
V R 1-,:? l '
Power -!!!!!.._-~-17..,RW
z:?-S?- ...

6. The impedance. z

v.2
--1!!JL_
R
We know that, Power. p
z2
1602 xR
or 200
so'
R 50 (l
We know tha1, z ~
or 80 J5o'+x,'
. x, 62.5 (l

The back emf, Vl iXL =2x62.S


125 v.
0 Chapf~r(· ';::,, ··;.-.. ,., ' , ' ' - . ·: · ---_i~- ,,

L
.:.
0
0 •r•Y)~ ··~·•1,...ve,$.·:~ -----.-----
.....---- -=--=--=-::-=- - --
'~·

- - - - --->-----.-- . - - · ~ •',_

• I
I. Given, \ .. to-;
- ,
.,-·' ,-
.,..,r
•--:;"

,ti l' I
-,
;- .
f' Pdt
.
-"--=J!.....J1?dt
i ? t

'. T .,,
0

., It '1· =3lo.,
lo
_,,,-3
_- 0

and Ans.

2. Given. 5f4sin0ll
5+4sin(21tx60)1
,5+4sin377t
T

The average value of ~urrent


J (S+~sin377t)d/
- 0
·T T
S_A.
For nns value, (5 +4sin ror) 2 = 25+ 16sin 2 rot +40sin 2 rot
•;,:.}

and 25+~+0=33
.,, 2
I.,
;rm., .ff' =.J33=S.1SA
3. Given, C=25xW--6F. l=O.IOH. R=250.·and e=310sin3l4t
• <, ·
On comparing with, e=e0 sinror, we have
co .3,14,

(i) I ·· ~ =~=·;OHz
.,2n 2n

(ii)

(iii) X, =rol=314x0.10=3l.40 Q and Xe _ __:___ _ 127.38!1


, .. coC 3l4x(2Sxl0-6)

The reactance, X=Xl -Xc=96,.Q .,


The tlux through lhe area (f.dr).

d~ NB(ld<J

4'1:)b1
f
·,;
NB/dr

h
Po -dr
Nf-
21t X
Ji ''
"
p 0 NU e .,,..,,,
Jt I nx.,11
2

µ; Nfi f ( u+h)
- 2Jt- " --
1
,,

On comparing with. 4'1L11al Mi. we get : ,I

M
µ0-Ne
- h·)
. n I+-
71t
c( a
An.I'..

46, The magnetic field of the toroid a distance r from the centre.

µ,N,i
8
2Jtr ·
The total flux through the area (hdr)

h
N,Jµ,N,i (hdr)
• 2Jtr
"
µ 0N 1N,h
, 2n
In(!!..}
a

47, (a)

8500 = 71
120

(b) ip ·.!._ = 78000 = 9.2 A


Vp 8500

and .!_= 78000 =650 A


~-- -.~.. 120

(c) Rp Vp ~ 8500 =926 Q


Ip 9.2 .

and R., -- Ans.


q
42, (~) If ~; is the rate of change of current. then by Kirchofi's second la'

i
fl.+L di 0
C 1/1 L
. 'I
di Q 100x10-<>
or 10" A/'i
di LC 2xlo--'xsx10"""'
(b) ·w~en Q=2QOµC. 1here is no energ)'. ston.-ct in in~uctor.
so i=O.

I l.2 Q~r.J:.O
(c) 2 Ima., 2C

. Qn,:i,
==2A.
·'· •jl\~
.fie
(d) GiVfR I . = ;ma, =IA
2
From 1.~nservation of energy. we can -write .
,.,, ' '1 I, • 'I
\ 2 ' ·: nl
I
-Lil + fl_ "" \!ma,_
2 2C 2C'
After s~bstilutin~ f~e value. and simplifyirg, we get
Q 173.2 µC An<.
43, (a) The indu~~d emf acro~s 1he who!c rod
Bwr 2 ~ ~
" 2 ~---~
(b) The etl\!ctive circuit is shown in figure~
s
If / is the currer!I ~rn.l J,di is the ipcrease in currcnl in t~e circuit then
R

.,-,
, .R
-di- Ltli'

A!\er simplifying. we gel j' =_ ,-_i(l-e-R,1/.)


/I

(i) or ·i An.1:.

(ii) Torque. t force x moment arm

[ Frnaiin,,.'lk + /\wigh1 ]r I;?


[Bir+"!gcosrot]r/ 2

B2ror~ mgrcosw
--+~--- All.\;,
,•..:: 4R 2
44. The f!ux across tirs, ~l~noid

,} M µ 0 rtlRfn1112
'
45, Magn~I!,;; field due to current ca")'ing wire at 'a distance, x from wire
X

Po i
.,. 8 2~·;
When switch S is· o~n~d. the inducloi opposes the decay of c:urr'ent, SO' dirCCtion· Of Currerit' in· R2 'remaiqs sanic whii~~
the direclion ot~ c~rrenl in R1 is reversed,. If i3 be the current then

cli, +,
IR ,+11,
"R
,.
l -· ~ 0
cit . • •
· Aller simp~i~ing, we _gCt ·

1,.·
oc-1IR1+-'!)IL
' .~ fl/
6e-112+?)f0,4

40.
6e-lRI

After a long time st,:ady state will reach and inductor offers zero_ resistance and capacitor offers infinite resistance. The·
' .
effective ci~cuil is shown in figure. . ,1 ,

The currenl in the circuil


21;
(1:+~+1;) ,'

For potc~tial difference across lhe terminals of ~ell A is zero.


' ~.r,

·c
~._r, ' JR/2
or ~-iiJ · =. 0 ' ~ . JR/2
or • I
. .

.j ' :1' 1' s,


R -(r.,-r,)
3· • R_,=20
a b
41. (a) Ar Steady si31e. 1he capacilor offers
infinite resistance. and 1he effective cin:uit
is shown in ligure. 1<')
By loop rule, we can wrile R2 =m ,IJ-_.,,_c_ _ _ _ __
u,+2(1,+i,)-12 C 0 " i1 'l12V
and u 2 +2v1+1,J-J 0
Aller simplifying above equati(!n, we have
-~-....• 'f--'-'-,.,,,,--''-"~----------1.·'
11=3.5 A. 12 =-I A "·'' 'I 1 R2 =2ll ·2 o'f
JV
£ 2 -i2R2 =3-(-;-l)x~=S V
(b) When the switch is pul in position 2•. lhe equivalent circuil is:·
T~e s~ea~y current · '.
1;,
3 211
io s=0.6A
B
Th~ curren1 at nnr time is givCn by· .. JV
,,_ _,, , ,,
io(l-e-R111.) J{l
io
_,, IDmH
or 0,5· (1-elDxlO-))

·· l,J86xl0·3 s
Th'e current al l~e. ins1a111 C

/" ~=O.JA
;•
2 . I. 1')!', .• ,

I,
-, :. Ene~gy slored. u
,,,
.!_l/2 =4.SxlO::' J
2 .
' ·, '·
--,

3,7,;,11 ·_,c~!. .-i,i:_,i)n~l_l)e~\a!~IY.l:.~~~f. ~!,(?~i!l.Gif!f..~wJ.t~_h?1,i~~~ft~~1.0~~~~~ 1jp~Q}t.~..r.e~i_s_t?,1,1ce,,.~o_; .·, ·;., . , •.• -...r .,r J.' ,

;,,11 = 11i2=~=3.33A,,:.~ ••:·.1-.;, I_


, c10+20'
(b) After long time. in_ductor offers zero resistan_ce so
,.
V
i,
I0+ 20x30
4.55A ..,,, -~ I) ;/ / I • ,l,_ ·~

20+30.
'\.
and i2 , .: A and 1.1 =1.82A
.= ,, 2.73
(C) At\er opening of the switch. · '\,

i,
(d)
JR! • (a) ,11:·The dircction·•or culTI:nts•may~be ·as Shown 'in fig'.Ure:: 1 • ::,. ~-,i · .,. ·.,1

By junction rule "! "'' !!


,,.
;.:__ i1+i2 ... (i)
By loop rule. we have ) \\i !
! ,, • . I I
i1R-l(di2 /di) = 0 ... (ii)
i ,,, ,,
., Since -=0,
;
di
ell
• :. -
.dt
(di,)
= (di,)
-
dt
,;
,1·,,'

constant
s i, ;,

current
R IOQ
Thus from equation (ii), we have
"i
i;R+l( ~:) O I
\\
.:: (jii)

At\er simplifying equations (i) and (iii), w,e ~et


--!·. .,,~~ ;, •\C
,e-Rf/l
'
;-;, =;[Y:.:e.:.1ull]', .'
(b) '
..
and
When
ii
i, i,
;·,·1.''. ,,• . j " I,, • ~.

or ie-Rtll 111-e-Rt/l]
,.
.£1n2.

39. .BY !(!Op rule, we..._have


R '
.. ,..... ,, . ' ,, :.,,,,., '!. • ! :

'\ 0 ... (i) A.~·~--~i~'-·-,B,--i'~-,M


i,
', di-.' . : \ '! \ , I,
and ~-L-· -,2R2 . =.... 0.
dt• ...• " ....... ,' ' .;, '. ,.,,(ii)... ,,, I;
.· -• I ,I_ ,, .. iRj
,,l
• I•,•. •
Aller simplifyilig equations_ (i) and (ii), we get
I s
u l !! ,, l_(l-e-R11,!I·) D'-----C'c---~N
i, - R2 ,
•1:1 l .,
The potential difference across in4uctor
I a•
v, '.:..L!:!!L

di

(,. ;, ~· 12. . t
At steady state, i, -=-,::;,6A,
R1 2· ·

and
._ ....,11 ,::'-
,, = . 3_=_13_=6A'
R2 2
,-.,:• • ' ~:• I •
.~'
.•
/;;: . L:»1?~
-"----='----''·-._,_-_I
31. Suppose field acts into the page. The force 0J 1the·,,'1b~r dUb ·1J current i. F =Bif = s(-R' )1'
As soon as bar starts moving it experiences a·\e~a~ding iOrce,
n~o.02 T
F'
,,
By Newton's second law, we have
ma
,,.,,., ...
or ·rd··)
m-
d1
After simplify and substituting the values, we get JIIJ',,l r1
' '
1' !00 (I -- e-;(l·oos,) Ans.
, t1
~~di
32. (b) We have e
di

_i_e_=~= lxl0-3 A
L (di/di) 25x10~

33. We have, ·~ . -tRIL


to~

or 10-l lxe-1n110
~ R 46 Q

£stored .,;2 /R
34. The required ratio,
Esuppli~d V.02 1 R
1 1"-
" ··,:-Vl
I
Vl(l-:-e-,11)
vl va2
r \~, (l-e-,/t)2

[:_,{''~"]]'
0.36 Ans.
I .,
l di
35. (a) We have, [el
di

(e) IOxl0-3
L ,:• I•
(dUd1) ~(5+16/)
• ~ I) di
6.25 x: 10--i H
(b) The power supplied .P., t'.i 1
d
..,
• 1o·x10-3 x(5+161)
-ii
.10 X JO·l X (5 + 16)
=:= 1~_10 mW. A11s.
'J -J - 1-
36. (a) We have _.;;. J Vo(l-e-1/t)
1; -·
\ ,. '
L
or 9-!Uoi,, = - 'iQ(l-e-111 ) 't'=-
R .. l·
. r., ' I
After substituting the values and siinplify:ing, we get
,, ,
1
_, ,.~ .~J _8.45 ns.
(b) Ans.

',
27. (a) ThC i(!duci:d emf. n,·e
,J ,\X 0.05 x,0,02
I x 10~> V
e ix10-1
Thus curr!!11l! -=---
R 10
0,1 mA
(b). e IM
I x o.os x 0,04·
2rx l,0-3·v

/ e ·~2xi(fb
. T~us 'cur/enl. -=---
R 10
•=· o.2 mA
The magnetic n_ux throllgh the strip of area (CiQ'), •re.

The 101al Ou,c through l~e square


d. (4t 2y)ltzy

J' 2
4, t_n(1'.
0

· The il]duccd emt: . lei d• =·4,e'


di
All= 2.5 s. lei 4 x 2.S x (0.02)'
8 x 10-6 V . ·Ans. ; . :
I

29, For 0 isint 2 ;:::::)/=0

and 2 2sin~l;:::::)t 2 =!:


' (1 2-·
· :. I=.Ji
.
..,...
2·.

The em;rgy spent. f

·1 ., .:,
-L(2sinrr
1 2 t;

2lsin 2 t
' . ., :·
2x2sin· R/2·= 4J.
JO. (i) . The nu~ t~rough the :smaller _loop

• .(2RPoi )A, cosro/ ·":·-~,.;~ . .


2
. ·2.
(4RX I0-7)x lx(SxlO"')
OsW.:-.
2xo.1
- JtXI0-9 COSltll

d~ .
. (ii) The induced einf, Jel i=nxl0-9msinCJ>I
di

(iii). The induced_ current, !...= I.S7xlO~~sin~.


R
,.
'... ,~:
. nr,. , !
22.-· ,The it)agneiic flux ~hfou_gh l~e siructure BA= 8 -
4

d$
dt
=.,.,(dB). 4 dt -
,'-; I J• . .,
'n(0.1)- x(5xl0-')
4
/'= 39.25 µV An...
23. ThC emf induced across each of the win:
•:'
e s,,e = 1 x o.os x o.04
-.. 21 mv
, j~

>x2
f?no:t 2mVand ,·=---=1(1
2+2
.\.q
The Current.
··e~., ·- 2 -·o I A
-- ----. m
R 19+1
. )~ thiS, caS~. · o. :.
24: · The induced emJ, lei A(:)
~I' Po"(~') (1tR)
2

, n(I.Sxl o-2 ) 2 x(4nx 10-7)X220X I o2

1.s' )
X - - - 1 =.6M.6inV '
( 25x10--

Current, i '. ""·' .!.=JOmA


R
.25. :r-~-e ·n·ox 1hro~gh :1.h~ ~~i-1 ·31 ·any instant
\"W-'
~\.
NBAcosCIJl

e - d~ =NBAsin(l)I , .
"'
NB(ab)x2nf
' sin 21!/i
-. .

26. .. ; fltix'·ihro'Ugh
.·The . . ·- ·- the. . small iUta.
.
'

~
' ,f

.,
BA=B( a9tt)
.,.
Ba 1
-0
2

·1~d~uul~-~(;'--
,
.· - - · . .
·. ,. -···
e-''· .=; ;yl~ =-·Bu' (
,di . 2
d~)
dt
,
., -,. -,.,
Bo 2
---Ol
2
·_.111 tilTICt,'.· I ro ,.- at
.. ·'
.J .... :
·e
-i;l.
-----
~ 2
....... : -,_,..
·r;me tak:el to -~~~Plete .h3_1f rotation ·n=.!..w 2
2

r~ ··genera1
,.
17. (a, b) ForwireA: e=l'=8vtsin8=0 '"··
For wire C: e= Bvfsin45°=D.0707V
For wire D : e = Bv./2{ sin 90°,= 0.14 IV A11s.
18. (a) The ma-gnetic flux through t-h~:~i~cli"'or~dius r 1, ·i

41 BA cos 0° = Brr.ri2
'
Thus
.• I
"' def>'
dt ,,, ru;'(di)·
_d~· ,,
(b) We have , pl,,:?i . dt
1
or ~j''' -1Tlj
. ,.
'(dB)
-
dt

/ .. ' '
E;,
I''
-%(1)
(c) For 12>R £/lx2roz "-nR'( d,)
R 2(d8)
E,, Ans.
2r dt
2
Do other parts similarly ,; l ·,
'
'E,,i = -'-(dB)=-.Q:.l.x0.1=--0.005 Vim
19 (b)
·,·J~.2dt_\\ 2

The induced-emf, .' e


' nr·' (dBJ.
di =n(O.I) 2 xO.I
= 3:14 x 10-3 V A11s.
3 14
(c) The induced current. =' '-~= · =l.57xIO-J A
'R 2
(d) Zero. '
X
20. (a) Induced emf -,.

or
X

-f-.
D
-a-
,\- X
-,.
X

0.20·,-.• : ;;.:t
---xO. I= --0.005 Vim
~t 4 Q

\• d$ ., dB
(b) Induced ~mf,
1
1,Y- =·1 -=a-x-
-:~ dt di
(0.20) 2xO.I =4XIO-J V
,.
q'.i"
Induced current, ..:._ =~ =2 X l0-J
R . 2:t1•
A•
21. (a) The induced emf in any side of the l,09~ .is given by -.:,

e .. ~.:,~2c:sar:)
where d is the- distance of the side from ;the centre of the field. Here d = 0, :., .• e =0
(b) The induced emf, [el =·: •'( d,)
J
= . (0.20) 2x0.1=4xI0-3 V
Do the rother parts accordingly.
..~-
I
When iron part comes down. the final flux
<l>t IQ

2
-~qi= 1tBr =l.57x10-"v Ans, 1 •
,,.
lei · - llJ 2!!,,J
'

13, The induced emf across the rod '.!. "\


e ,:Bvl
,' I
e B,•l
The induced current. li -=--
R R
The force on the rod, F Bil

(a) Thus for terminal velocity,


mg)r:= F
I \H
,, )
,=
·-.h \ R
----..,i, , J-' - mgR
Am.
'
. I,..' ' ,
tflL'
(b) If a be the acceleration, then
mg
mg-- = ma
2, \\~_, '
g/2 Am.
14. The magnetic flux through the loop,
BA

µoi
( 21tr
)A
I, .
1.,...:. x..,.__,._
11,'! i- ·- ~ • •
.
1· ,,
d$
le(
r, dt
l ( µ,;
or lel
d -
- A)
dt 21tr
'
sx I o- 11 A, anticlockwise
15. (a) The magnetic flux through the lo~p.·~'
cf> BAcos8=8AcosOJ/
d$ -d(BAcosrot)
(b) The induced emf, ,' e dt di ,,..

16. (a) The initial flux through the loop.


BAcosin OJ/ ·, .., ,·, ,!!
~ns.
' ' 'I,
$; = - BA
and 0
, • i" ,l'l . ' . j• 1 , Ii I'.,

The average _i~~uc~d.;mf.~ J,el ,; = qif~qii


1
"j ~ · . - - ~
, .
2
BA l.2xn(0.05)
I ---'---'-'
-l At 0.2
0.0471 V.
10
From a to b..

-.-...
8. If V is 1he vi:loci1y of 1he loop. lhen induced emf
e VBL.
VBL
and induced currcnl, -=--
e
R R·

The force on 1he loop Bil = B[l'BLJL .


R
VB 2i 1 tR
For no accelera1ion mg VB 2i2JR
V mgR!B 1L1 Ans.

9. We have. tanO .!!r...


s,,
B,. B11 tm9.
=- 2 x 10-il x tan 60° = 2Jj xto-5 T
I .

The induced emt', B,.,·t

(2"3 XIO~')x250X20
0.173 V An.f.

10. The resislance of 1he loops, 2,rr, xl0=2nxO.lxlO


6.281).
and 2nr, =2nxlxl0
62.Sll.
FluX in 1he smaller loop, • B1A1
µ-'.,:. "I
-""--mr
2r1

l'o[f, ],rr,'
2r2

2r2

The induced currenl, -~ =[d~/ Jt]


R1 R1·
After subs1i1uting the value and simplifying we get
I 1.25 A. Ans.
11. l•) At , o;i 2~; the length bf the loop inside· magnetic field, .
.\" I x 2 = 2 cm. s0 elTlf will induce due to ri~ht sidt of the loop,
e Bvl = 0.6xO.Olx0.20
\ JxlO-'V
(b) At,~ 10 s, .T _Ix 1.0 = IO cm, The entire loop is inside magnetic field and so
e = 0
(c) Alt=22s. x- lx22=22Cm. Th~ ~·~; is induced due to left side of the loop, ~ __
e . • 3 x lo-:'. V.

12,
(d) At t = 20 s, the loop will out or the field so; e = O'
Thf niagnetic tlux through lh~ iron pan 1
,.,
,,J
....
,' /
Bx!!....
2
•.
·,.,,
· Books specially designed to improve your
eo rehe i fj
Analytieal Skills
Chapterwise collection of challenging problems on the new pattern of HT-JEE ~

~ IIT-JEE -~ IIT-JEE ~ IIT-JEE


PhVSiCS Chemistrv Mathematics
eu~ ~ fjkdteM,µ;r,

The Best
Problem Books
on the new pattern
of HT-JEE.
l\ecommended
au over

You might also like